Sunteți pe pagina 1din 485

1

CASES (PROPERTY)
POPERTY
DAVAO SAWMILL v. CASTILLO (G.R. No. L-40411, Aug 7, 1935)………………………….. page 2
BERKENKOTTER v. CU UNJIENG (G.R. No. L-41643, July 31, 1935) ……………………. page 4
LOPEZ v. OROSA (G.R. Nos. L-10817-18, February 28, 1958) ……………………………………….. page 7
TUMALAD v. VICENCIO (G.R. No. L-30173, September 30, 1971)…………………………… page 10
ASSOCIATED INSURANCE v. IYA (G.R. Nos. L-10837-38, May 30, 1958) ……………… page 17
MAKATI LEASING v. WEAREVER (G.R. No. L-58469, May 16, 1983) ………………….. page 20
BD. OF ASSESSMENT APPEALS v. MERALCO (G.R. No. L-15334, January 31, 1964) ..page 23
MERALCO v. BD. OF ASSESSMENT APPEALS (G.R. No. L-46245, May 31, 1982) …… page 27
MERALCO v. BD. OF ASSESSMENT APPEALS (G.R. No. L-47943, May 31, 1982) ……. page 31
CALTEX v. BD. OF ASSESSMENT APPEALS (G.R. No. L-50466, May 31, 1982) ………. page 33
BENGUET CORP. v. BD. OF ASSESSMENT APPEALS (G.R. No. 106041, January 29, 1993) . p. 36

OWNERSHIP

LA BUGAL B’LAAN TRIBAL ASSN. v RAMOS (G.R. No. 127882, January 27, 2004) … page 42
ON RECONSIDERATION (G.R. No. 127882, December 1, 2004) ……………………………. page 98
USERO v CA (G.R. No. 152115, January 26, 2005) ……………………………………………….. page 305
GOVERNMENT v CABANGIS (G.R. No. L-28379, March 27, 1929) ……..………………… page 309
CEBU ACETYLENE v BERCILLES (G.R. No. L40474, August 29, 1975) …………………. page 313
Vda De TANTOCO v MUNICIPAL COUNCIL (G.R. No. L-24950, March 25, 1926) .… page 315
SALAS v JARENCIO (G.R. No. L-29788, August 30, 1972) ………………………………….. page 32o
MUNICIPALITY OF SAN MIGUEL v FERNANDEZ (G.R. No. L-61744, June 25, 1984) … p 330
ZAMBOANGA DEL NORTE v CITY OF ZAMBOANGA (G.R. No. L-24440, March 28, 1968) p.333
CHAVEZ v PEA AMARI (G.R. No. 133250, July 9, 2002) …………………………………… page 340
ON MOTION FOR RECONSIDERATION (G.R. No. 133250, May 6, 2003) …………… page 383

(OTHER SPECIFIC RIGHTS FOUND IN THE CIVIL CODE)

HILARIO v SALVADOR (G.R. No. 160384, April 29, 2005) ………………………….……. page 426
SAMPAYANG v CA (G.R. No. 156360, January 14, 2005)……………………………………. Page 433
SANTOS v AYON (G.R. No. 137013, May 6, 2005) ……………………………………………. Page 438
GANILA v CA (G.R. No. 150755, June 28, 2005 …………………………………………………… page 443
ROSS RICA SALES CENTER v SPS. ONG (G.R. No. 132197, August 16, 2005) ….. page 449
SERINA v CABALLERO (G.R. No. 127382, August 17, 2004) …………………………. Page 463
PEREZ v MENDOZA (G.R. No. L-22006, July 28, 1975) ………………………………... page 470
DIZON v CA (G.R. No. 101929, January 6, 1993) …………………………………………... page 477
LUNOD v MENESES ……………………………………………………………………………. Page 482
2

DAVAO SAWMILL v. CASTILLO (G.R. No. L-40411, Aug 7, 1935)

Republic of the Philippines


SUPREME COURT
Manila

EN BANC

G.R. No. L-40411 August 7, 1935

DAVAO SAW MILL CO., INC., plaintiff-appellant,


vs.
APRONIANO G. CASTILLO and DAVAO LIGHT & POWER CO., INC., defendants-appellees.

Arsenio Suazo and Jose L. Palma Gil and Pablo Lorenzo and Delfin Joven for appellant.
J.W. Ferrier for appellees.

MALCOLM, J.:

The issue in this case, as announced in the opening sentence of the decision in the trial court and as
set forth by counsel for the parties on appeal, involves the determination of the nature of the
properties described in the complaint. The trial judge found that those properties were personal in
nature, and as a consequence absolved the defendants from the complaint, with costs against the
plaintiff.

The Davao Saw Mill Co., Inc., is the holder of a lumber concession from the Government of the
Philippine Islands. It has operated a sawmill in the sitio of Maa, barrio of Tigatu, municipality of
Davao, Province of Davao. However, the land upon which the business was conducted belonged to
another person. On the land the sawmill company erected a building which housed the machinery
used by it. Some of the implements thus used were clearly personal property, the conflict concerning
machines which were placed and mounted on foundations of cement. In the contract of lease
between the sawmill company and the owner of the land there appeared the following provision:

That on the expiration of the period agreed upon, all the improvements and buildings
introduced and erected by the party of the second part shall pass to the exclusive ownership
of the party of the first part without any obligation on its part to pay any amount for said
improvements and buildings; also, in the event the party of the second part should leave or
abandon the land leased before the time herein stipulated, the improvements and buildings
shall likewise pass to the ownership of the party of the first part as though the time agreed
upon had expired: Provided, however, That the machineries and accessories are not included
in the improvements which will pass to the party of the first part on the expiration or
abandonment of the land leased.

In another action, wherein the Davao Light & Power Co., Inc., was the plaintiff and the Davao, Saw,
Mill Co., Inc., was the defendant, a judgment was rendered in favor of the plaintiff in that action
against the defendant in that action; a writ of execution issued thereon, and the properties now in
question were levied upon as personalty by the sheriff. No third party claim was filed for such
properties at the time of the sales thereof as is borne out by the record made by the plaintiff herein.
Indeed the bidder, which was the plaintiff in that action, and the defendant herein having
consummated the sale, proceeded to take possession of the machinery and other properties
described in the corresponding certificates of sale executed in its favor by the sheriff of Davao.

As connecting up with the facts, it should further be explained that the Davao Saw Mill Co., Inc., has
on a number of occasions treated the machinery as personal property by executing chattel mortgages
in favor of third persons. One of such persons is the appellee by assignment from the original
mortgages.
3

Article 334, paragraphs 1 and 5, of the Civil Code, is in point. According to the Code, real property
consists of —

1. Land, buildings, roads and constructions of all kinds adhering to the soil;

xxx xxx xxx

5. Machinery, liquid containers, instruments or implements intended by the owner of any


building or land for use in connection with any industry or trade being carried on therein and
which are expressly adapted to meet the requirements of such trade of industry.

Appellant emphasizes the first paragraph, and appellees the last mentioned paragraph. We entertain
no doubt that the trial judge and appellees are right in their appreciation of the legal doctrines flowing
from the facts.

In the first place, it must again be pointed out that the appellant should have registered its protest
before or at the time of the sale of this property. It must further be pointed out that while not
conclusive, the characterization of the property as chattels by the appellant is indicative of intention
and impresses upon the property the character determined by the parties. In this connection the
decision of this court in the case of Standard Oil Co. of New York vs. Jaramillo ( [1923], 44 Phil., 630),
whether obiter dicta or not, furnishes the key to such a situation.

It is, however not necessary to spend overly must time in the resolution of this appeal on side issues.
It is machinery which is involved; moreover, machinery not intended by the owner of any building or
land for use in connection therewith, but intended by a lessee for use in a building erected on the land
by the latter to be returned to the lessee on the expiration or abandonment of the lease.

A similar question arose in Puerto Rico, and on appeal being taken to the United States Supreme
Court, it was held that machinery which is movable in its nature only becomes immobilized when
placed in a plant by the owner of the property or plant, but not when so placed by a tenant, a
usufructuary, or any person having only a temporary right, unless such person acted as the agent of
the owner. In the opinion written by Chief Justice White, whose knowledge of the Civil Law is well
known, it was in part said:

To determine this question involves fixing the nature and character of the property from the
point of view of the rights of Valdes and its nature and character from the point of view of
Nevers & Callaghan as a judgment creditor of the Altagracia Company and the rights derived
by them from the execution levied on the machinery placed by the corporation in the plant.
Following the Code Napoleon, the Porto Rican Code treats as immovable (real) property, not
only land and buildings, but also attributes immovability in some cases to property of a
movable nature, that is, personal property, because of the destination to which it is applied.
"Things," says section 334 of the Porto Rican Code, "may be immovable either by their own
nature or by their destination or the object to which they are applicable." Numerous
illustrations are given in the fifth subdivision of section 335, which is as follows: "Machinery,
vessels, instruments or implements intended by the owner of the tenements for the industrial
or works that they may carry on in any building or upon any land and which tend directly to
meet the needs of the said industry or works." (See also Code Nap., articles 516, 518 et seq.
to and inclusive of article 534, recapitulating the things which, though in themselves movable,
may be immobilized.) So far as the subject-matter with which we are dealing — machinery
placed in the plant — it is plain, both under the provisions of the Porto Rican Law and of the
Code Napoleon, that machinery which is movable in its nature only becomes immobilized
when placed in a plant by the owner of the property or plant. Such result would not be
accomplished, therefore, by the placing of machinery in a plant by a tenant or a usufructuary
or any person having only a temporary right. (Demolombe, Tit. 9, No. 203; Aubry et Rau, Tit.
2, p. 12, Section 164; Laurent, Tit. 5, No. 447; and decisions quoted in Fuzier-Herman ed.
Code Napoleon under articles 522 et seq.) The distinction rests, as pointed out by
Demolombe, upon the fact that one only having a temporary right to the possession or
enjoyment of property is not presumed by the law to have applied movable property
4

belonging to him so as to deprive him of it by causing it by an act of immobilization to become


the property of another. It follows that abstractly speaking the machinery put by the Altagracia
Company in the plant belonging to Sanchez did not lose its character of movable property
and become immovable by destination. But in the concrete immobilization took place because
of the express provisions of the lease under which the Altagracia held, since the lease in
substance required the putting in of improved machinery, deprived the tenant of any right to
charge against the lessor the cost such machinery, and it was expressly stipulated that the
machinery so put in should become a part of the plant belonging to the owner without
compensation to the lessee. Under such conditions the tenant in putting in the machinery was
acting but as the agent of the owner in compliance with the obligations resting upon him, and
the immobilization of the machinery which resulted arose in legal effect from the act of the
owner in giving by contract a permanent destination to the machinery.

xxx xxx xxx

The machinery levied upon by Nevers & Callaghan, that is, that which was placed in the plant
by the Altagracia Company, being, as regards Nevers & Callaghan, movable property, it
follows that they had the right to levy on it under the execution upon the judgment in their
favor, and the exercise of that right did not in a legal sense conflict with the claim of Valdes,
since as to him the property was a part of the realty which, as the result of his obligations
under the lease, he could not, for the purpose of collecting his debt, proceed separately
against. (Valdes vs. Central Altagracia [192], 225 U.S., 58.)

Finding no reversible error in the record, the judgment appealed from will be affirmed, the costs of this
instance to be paid by the appellant.

Villa-Real, Imperial, Butte, and Goddard, JJ., concur.

BERKENKOTTER v. CU UNJIENG (G.R. No. L-41643, July 31, 1935)

Republic of the Philippines


SUPREME COURT
Manila

EN BANC

G.R. No. L-41643 July 31, 1935

B.H. BERKENKOTTER, plaintiff-appellant,


vs.
CU UNJIENG E HIJOS, YEK TONG LIN FIRE AND MARINE INSURANCE COMPANY,
MABALACAT SUGAR COMPANY and THE PROVINCE SHERIFF OF PAMPANGA, defendants-
appellees.

Briones and Martinez for appellant.


Araneta, Zaragoza and Araneta for appellees Cu Unjieng e Hijos.
No appearance for the other appellees.

VILLA-REAL, J.:

This is an appeal taken by the plaintiff, B.H. Berkenkotter, from the judgment of the Court of First
Instance of Manila, dismissing said plaintiff's complaint against Cu Unjiengs e Hijos et al., with costs.
5

In support of his appeal, the appellant assigns six alleged errors as committed by the trial court in its
decision in question which will be discussed in the course of this decision.

The first question to be decided in this appeal, which is raised in the first assignment of alleged error,
is whether or not the lower court erred in declaring that the additional machinery and equipment, as
improvement incorporated with the central are subject to the mortgage deed executed in favor of the
defendants Cu Unjieng e Hijos.

It is admitted by the parties that on April 26, 1926, the Mabalacat Sugar Co., Inc., owner of the sugar
central situated in Mabalacat, Pampanga, obtained from the defendants, Cu Unjieng e Hijos, a loan
secured by a first mortgage constituted on two parcels and land "with all its buildings, improvements,
sugar-cane mill, steel railway, telephone line, apparatus, utensils and whatever forms part or is
necessary complement of said sugar-cane mill, steel railway, telephone line, now existing or that may
in the future exist is said lots."

On October 5, 1926, shortly after said mortgage had been constituted, the Mabalacat Sugar Co., Inc.,
decided to increase the capacity of its sugar central by buying additional machinery and equipment,
so that instead of milling 150 tons daily, it could produce 250. The estimated cost of said additional
machinery and equipment was approximately P100,000. In order to carry out this plan, B.A. Green,
president of said corporation, proposed to the plaintiff, B.H. Berkenkotter, to advance the necessary
amount for the purchase of said machinery and equipment, promising to reimburse him as soon as he
could obtain an additional loan from the mortgagees, the herein defendants Cu Unjieng e Hijos.
Having agreed to said proposition made in a letter dated October 5, 1926 (Exhibit E), B.H.
Berkenkotter, on October 9th of the same year, delivered the sum of P1,710 to B.A. Green, president
of the Mabalacat Sugar Co., Inc., the total amount supplied by him to said B.A. Green having been
P25,750. Furthermore, B.H. Berkenkotter had a credit of P22,000 against said corporation for unpaid
salary. With the loan of P25,750 and said credit of P22,000, the Mabalacat Sugar Co., Inc., purchased
the additional machinery and equipment now in litigation.

On June 10, 1927, B.A. Green, president of the Mabalacat Sugar Co., Inc., applied to Cu Unjieng e
Hijos for an additional loan of P75,000 offering as security the additional machinery and equipment
acquired by said B.A. Green and installed in the sugar central after the execution of the original
mortgage deed, on April 27, 1927, together with whatever additional equipment acquired with said
loan. B.A. Green failed to obtain said loan.

Article 1877 of the Civil Code provides as follows.

ART. 1877. A mortgage includes all natural accessions, improvements, growing fruits, and
rents not collected when the obligation falls due, and the amount of any indemnities paid or
due the owner by the insurers of the mortgaged property or by virtue of the exercise of the
power of eminent domain, with the declarations, amplifications, and limitations established by
law, whether the estate continues in the possession of the person who mortgaged it or
whether it passes into the hands of a third person.

In the case of Bischoff vs. Pomar and Compañia General de Tabacos (12 Phil., 690), cited with
approval in the case of Cea vs. Villanueva (18 Phil., 538), this court laid shown the following doctrine:

1. REALTY; MORTGAGE OF REAL ESTATE INCLUDES IMPROVEMENTS AND


FIXTURES. — It is a rule, established by the Civil Code and also by the Mortgage Law, with
which the decisions of the courts of the United States are in accord, that in a mortgage of real
estate, the improvements on the same are included; therefore, all objects permanently
attached to a mortgaged building or land, although they may have been placed there after the
mortgage was constituted, are also included. (Arts. 110 and 111 of the Mortgage Law, and
1877 of the Civil Code; decision of U.S. Supreme Court in the matter of Royal Insurance
Co. vs. R. Miller, liquidator, and Amadeo [26 Sup. Ct. Rep., 46; 199 U.S., 353].)

2. ID.; ID.; INCLUSION OR EXCLUSION OF MACHINERY, ETC. — In order that it may be


understood that the machinery and other objects placed upon and used in connection with a
6

mortgaged estate are excluded from the mortgage, when it was stated in the mortgage that
the improvements, buildings, and machinery that existed thereon were also comprehended, it
is indispensable that the exclusion thereof be stipulated between the contracting parties.

The appellant contends that the installation of the machinery and equipment claimed by him in the
sugar central of the Mabalacat Sugar Company, Inc., was not permanent in character inasmuch as
B.A. Green, in proposing to him to advance the money for the purchase thereof, made it appear in the
letter, Exhibit E, that in case B.A. Green should fail to obtain an additional loan from the defendants
Cu Unjieng e Hijos, said machinery and equipment would become security therefor, said B.A. Green
binding himself not to mortgage nor encumber them to anybody until said plaintiff be fully reimbursed
for the corporation's indebtedness to him.

Upon acquiring the machinery and equipment in question with money obtained as loan from the
plaintiff-appellant by B.A. Green, as president of the Mabalacat Sugar Co., Inc., the latter became
owner of said machinery and equipment, otherwise B.A. Green, as such president, could not have
offered them to the plaintiff as security for the payment of his credit.

Article 334, paragraph 5, of the Civil Code gives the character of real property to "machinery, liquid
containers, instruments or implements intended by the owner of any building or land for use in
connection with any industry or trade being carried on therein and which are expressly adapted to
meet the requirements of such trade or industry.

If the installation of the machinery and equipment in question in the central of the Mabalacat Sugar
Co., Inc., in lieu of the other of less capacity existing therein, for its sugar industry, converted them
into real property by reason of their purpose, it cannot be said that their incorporation therewith was
not permanent in character because, as essential and principal elements of a sugar central, without
them the sugar central would be unable to function or carry on the industrial purpose for which it was
established. Inasmuch as the central is permanent in character, the necessary machinery and
equipment installed for carrying on the sugar industry for which it has been established must
necessarily be permanent.

Furthermore, the fact that B.A. Green bound himself to the plaintiff B.H. Berkenkotter to hold said
machinery and equipment as security for the payment of the latter's credit and to refrain from
mortgaging or otherwise encumbering them until Berkenkotter has been fully reimbursed therefor, is
not incompatible with the permanent character of the incorporation of said machinery and equipment
with the sugar central of the Mabalacat Sugar Co., Inc., as nothing could prevent B.A. Green from
giving them as security at least under a second mortgage.

As to the alleged sale of said machinery and equipment to the plaintiff and appellant after they had
been permanently incorporated with sugar central of the Mabalacat Sugar Co., Inc., and while the
mortgage constituted on said sugar central to Cu Unjieng e Hijos remained in force, only the right of
redemption of the vendor Mabalacat Sugar Co., Inc., in the sugar central with which said machinery
and equipment had been incorporated, was transferred thereby, subject to the right of the defendants
Cu Unjieng e Hijos under the first mortgage.

For the foregoing considerations, we are of the opinion and so hold: (1) That the installation of a
machinery and equipment in a mortgaged sugar central, in lieu of another of less capacity, for the
purpose of carrying out the industrial functions of the latter and increasing production, constitutes a
permanent improvement on said sugar central and subjects said machinery and equipment to the
mortgage constituted thereon (article 1877, Civil Code); (2) that the fact that the purchaser of the new
machinery and equipment has bound himself to the person supplying him the purchase money to hold
them as security for the payment of the latter's credit, and to refrain from mortgaging or otherwise
encumbering them does not alter the permanent character of the incorporation of said machinery and
equipment with the central; and (3) that the sale of the machinery and equipment in question by the
purchaser who was supplied the purchase money, as a loan, to the person who supplied the money,
after the incorporation thereof with the mortgaged sugar central, does not vest the creditor with
ownership of said machinery and equipment but simply with the right of redemption.
7

Wherefore, finding no error in the appealed judgment, it is affirmed in all its parts, with costs to the
appellant. So ordered.

Malcolm, Imperial, Butte, and Goddard, JJ., concur.

LOPEZ v. OROSA (G.R. Nos. L-10817-18, February 28, 1958)

Republic of the Philippines


SUPREME COURT
Manila

EN BANC

G.R. Nos. L-10817-18 February 28, 1958

ENRIQUE LOPEZ, petitioner,


vs.
VICENTE OROSA, JR., and PLAZA THEATRE, INC., respondents.

Nicolas Belmonte and Benjamin T. de Peralta for petitioner.


Tolentino & Garcia and D. R. Cruz for respondent Luzon Surety Co., Inc. Jose B. Macatangay for
respondent Plaza Theatre, Inc.

FELIX, J.:

Enrique Lopez is a resident of Balayan, Batangas, doing business under the trade name of Lopez-
Castelo Sawmill. Sometime in May, 1946, Vicente Orosa, Jr., also a resident of the same province,
dropped at Lopez' house and invited him to make an investment in the theatre business. It was
intimated that Orosa, his family and close friends were organizing a corporation to be known as Plaza
Theatre, Inc., that would engage in such venture. Although Lopez expressed his unwillingness to
invest of the same, he agreed to supply the lumber necessary for the construction of the proposed
theatre, and at Orosa's behest and assurance that the latter would be personally liable for any
account that the said construction might incur, Lopez further agreed that payment therefor would be
on demand and not cash on delivery basis. Pursuant to said verbal agreement, Lopez delivered the
lumber which was used for the construction of the Plaza Theatre on May 17, 1946, up to December 4
of the same year. But of the total cost of the materials amounting to P62,255.85, Lopez was paid only
P20,848.50, thus leaving a balance of P41,771.35.

We may state at this juncture that the Plaza Theatre was erected on a piece of land with an area of
679.17 square meters formerly owned by Vicente Orosa, Jr., and was acquired by the corporation on
September 25, 1946, for P6,000. As Lopez was pressing Orosa for payment of the remaining unpaid
obligation, the latter and Belarmino Rustia, the president of the corporation, promised to obtain a bank
loan by mortgaging the properties of the Plaza Theatre., out of which said amount of P41,771.35
would be satisfied, to which assurance Lopez had to accede. Unknown to him, however, as early as
November, 1946, the corporation already got a loan for P30,000 from the Philippine National Bank
with the Luzon Surety Company as surety, and the corporation in turn executed a mortgage on the
land and building in favor of said company as counter-security. As the land at that time was not yet
brought under the operation of the Torrens System, the mortgage on the same was registered on
November 16, 1946, under Act No. 3344. Subsequently, when the corporation applied for the
registration of the land under Act 496, such mortgage was not revealed and thus Original Certificate
of Title No. O-391 was correspondingly issued on October 25, 1947, without any encumbrance
appearing thereon.
8

Persistent demand from Lopez for the payment of the amount due him caused Vicente Orosa, Jr. to
execute on March 17, 1947, an alleged "deed of assignment" of his 420 shares of stock of the Plaza
Theater, Inc., at P100 per share or with a total value of P42,000 in favor of the creditor, and as the
obligation still remained unsettled, Lopez filed on November 12, 1947, a complaint with the Court of
First Instance of Batangas (Civil Case No. 4501 which later became R-57) against Vicente Orosa, Jr.
and Plaza Theater, Inc., praying that defendants be sentenced to pay him jointly and severally the
sum of P41,771.35, with legal interest from the firing of the action; that in case defendants fail to pay
the same, that the building and the land covered by OCT No. O-391 owned by the corporation be sold
at public auction and the proceeds thereof be applied to said indebtedness; or that the 420 shares of
the capital stock of the Plaza Theatre, Inc., assigned by Vicente Orosa, Jr., to said plaintiff be sold at
public auction for the same purpose; and for such other remedies as may be warranted by the
circumstances. Plaintiff also caused the annotation of a notice of lis pendens on said properties with
the Register of Deeds.

Defendants Vicente Orosa, Jr. and Plaza Theatre, Inc., filed separate answers, the first denying that
the materials were delivered to him as a promoter and later treasurer of the corporation, because he
had purchased and received the same on his personal account; that the land on which the movie
house was constructed was not charged with a lien to secure the payment of the aforementioned
unpaid obligation; and that the 420 shares of stock of the Plaza Theatre, Inc., was not assigned to
plaintiff as collaterals but as direct security for the payment of his indebtedness. As special defense,
this defendant contended that as the 420 shares of stock assigned and conveyed by the assignor and
accepted by Lopez as direct security for the payment of the amount of P41,771.35 were personal
properties, plaintiff was barred from recovering any deficiency if the proceeds of the sale thereof at
public auction would not be sufficient to cover and satisfy the obligation. It was thus prayed that he be
declared exempted from the payment of any deficiency in case the proceeds from the sale of said
personal properties would not be enough to cover the amount sought to be collected.

Defendant Plaza Theatre, Inc., on the other hand, practically set up the same line of defense by
alleging that the building materials delivered to Orosa were on the latter's personal account; and that
there was no understanding that said materials would be paid jointly and severally by Orosa and the
corporation, nor was a lien charged on the properties of the latter to secure payment of the same
obligation. As special defense, defendant corporation averred that while it was true that the materials
purchased by Orosa were sold by the latter to the corporation, such transactions were in good faith
and for valuable consideration thus when plaintiff failed to claim said materials within 30 days from the
time of removal thereof from Orosa, lumber became a different and distinct specie and plaintiff lost
whatever rights he might have in the same and consequently had no recourse against the Plaza
Theatre, Inc., that the claim could not have been refectionary credit, for such kind of obligation
referred to an indebtedness incurred in the repair or reconstruction of something already existing and
this concept did not include an entirely new work; and that the Plaza Theatre, Inc., having been
incorporated on October 14, 1946, it could not have contracted any obligation prior to said date. It
was, therefore, prayed that the complaint be dismissed; that said defendant be awarded the sum P
5,000 for damages, and such other relief as may be just and proper in the premises.

The surety company, in the meantime, upon discovery that the land was already registered under the
Torrens System and that there was a notice of lis pendens thereon, filed on August 17, 1948, or within
the 1-year period after the issuance of the certificate of title, a petition for review of the decree of the
land registration court dated October 18, 1947, which was made the basis of OCT No. O-319, in order
to annotate the rights and interests of the surety company over said properties (Land Registration
Case No. 17 GLRO Rec. No. 296). Opposition thereto was offered by Enrique Lopez, asserting that
the amount demanded by him constituted a preferred lien over the properties of the obligors; that the
surety company was guilty of negligence when it failed to present an opposition to the application for
registration of the property; and that if any violation of the rights and interest of said surety would ever
be made, same must be subject to the lien in his favor.

The two cases were heard jointly and in a decision dated October 30, 1952, the lower Court, after
making an exhaustive and detailed analysis of the respective stands of the parties and the evidence
adduced at the trial, held that defendants Vicente Orosa, Jr., and the Plaza Theatre, Inc.,
were jointly liable for the unpaid balance of the cost of lumber used in the construction of
the building and the plaintiff thus acquired the materialman's lien over the same. In making the
9

pronouncement that the lien was merely confined to the building and did not extend to the land on
which the construction was made, the trial judge took into consideration the fact that when plaintiff
started the delivery of lumber in May, 1946, the land was not yet owned by the corporation; that the
mortgage in favor of Luzon Surety Company was previously registered under Act No. 3344; that the
codal provision (Art. 1923 of the old Spanish Civil Code) specifying that refection credits are preferred
could refer only to buildings which are also classified as real properties, upon which said refection
was made. It was, however, declared that plaintiff's lien on the building was superior to the right of the
surety company. And finding that the Plaza Theatre, Inc., had no objection to the review of the decree
issued in its favor by the land registration court and the inclusion in the title of the encumbrance in
favor of the surety company, the court a quo granted the petition filed by the latter company.
Defendants Orosa and the Plaza Theatre, Inc., were thus required to pay jointly the amount of
P41,771.35 with legal interest and costs within 90 days from notice of said decision; that in case of
default, the 420 shares of stock assigned by Orosa to plaintiff be sold at public auction and the
proceeds thereof be applied to the payment of the amount due the plaintiff, plus interest and costs;
and that the encumbrance in favor of the surety company be endorsed at the back of OCT No. O-391,
with notation I that with respect to the building, said mortgage was subject to the materialman's lien in
favor of Enrique Lopez.

Plaintiff tried to secure a modification of the decision in so far as it declared that the obligation of
therein defendants was joint instead of solidary, and that the lien did not extend to the land, but same
was denied by order the court of December 23, 1952. The matter was thus appealed to the Court of
appeals, which affirmed the lower court's ruling, and then to this Tribunal. In this instance, plaintiff-
appellant raises 2 issues: (1) whether a materialman's lien for the value of the materials used in the
construction of a building attaches to said structure alone and does not extend to the land on which
the building is adhered to; and (2) whether the lower court and the Court of Appeals erred in not
providing that the material mans liens is superior to the mortgage executed in favor surety company
not only on the building but also on the land.

It is to be noted in this appeal that Enrique Lopez has not raised any question against the part of the
decision sentencing defendants Orosa and Plaza Theatre, Inc., to pay jointly the sum of P41,771.35,
so We will not take up or consider anything on that point. Appellant, however, contends that the lien
created in favor of the furnisher of the materials used for the construction, repair or refection of a
building, is also extended to the land which the construction was made, and in support thereof he
relies on Article 1923 of the Spanish Civil Code, pertinent law on the matter, which reads as follows:

ART. 1923. With respect to determinate real property and real rights of the debtor, the
following are preferred:

xxx xxx xxx

5. Credits for refection, not entered or recorded, with respect to the estate upon which the
refection was made, and only with respect to other credits different from those mentioned in
four preceding paragraphs.

It is argued that in view of the employment of the phrase real estate, or immovable property, and
inasmuch as said provision does not contain any specification delimiting the lien to the building, said
article must be construed as to embrace both the land and the building or structure adhering thereto.
We cannot subscribe to this view, for while it is true that generally, real estate connotes the land and
the building constructed thereon, it is obvious that the inclusion of the building, separate and distinct
from the land, in the enumeration of what may constitute real properties 1 could mean only one thing
— that a building is by itself an immovable property, a doctrine already pronounced by this Court in
the case of Leung Yee vs. Strong Machinery Co., 37 Phil., 644. Moreover, and in view of the absence
of any specific provision of law to the contrary, a building is an immovable property, irrespective of
whether or not said structure and the land on which it is adhered to belong to the same owner.

A close examination of the provision of the Civil Code invoked by appellant reveals that the law gives
preference to unregistered refectionary credits only with respect to the real estate upon which the
refection or work was made. This being so, the inevitable conclusion must be that the lien so created
attaches merely to the immovable property for the construction or repair of which the obligation was
10

incurred. Evidently, therefore, the lien in favor of appellant for the unpaid value of the lumber used in
the construction of the building attaches only to said structure and to no other property of the obligors.

Considering the conclusion thus arrived at, i.e., that the materialman's lien could be charged only to
the building for which the credit was made or which received the benefit of refection, the lower court
was right in, holding at the interest of the mortgagee over the land is superior and cannot be made
subject to the said materialman's lien.

Wherefore, and on the strength of the foregoing considerations, the decision appealed from is hereby
affirmed, with costs against appellant. It is so ordered.

Paras, C.J., Bengzon, Padilla, Montemayor, Reyes, A., Bautista Angelo, Labrador, Concepcion,
Reyes, J.B.L. and Endencia, JJ., concur.

Footnotes

1 Article 415 of the new Civil Code (Art. 334 of the old) enumerates what are considered
immovable property, among which are land, buildings, roads and constructions of all kinds
adhered to the soil.

TUMALAD v. VICENCIO (G.R. No. L-30173, September 30, 1971)

Republic of the Philippines


SUPREME COURT
Manila

EN BANC

G.R. No. L-30173 September 30, 1971

GAVINO A. TUMALAD and GENEROSA R. TUMALAD, plaintiffs-appellees,


vs.
ALBERTA VICENCIO and EMILIANO SIMEON, defendants-appellants.

Castillo & Suck for plaintiffs-appellees.

Jose Q. Calingo for defendants-appellants.

REYES, J.B.L., J.:

Case certified to this Court by the Court of Appeals (CA-G.R. No. 27824-R) for the reason that only
questions of law are involved.
11

This case was originally commenced by defendants-appellants in the municipal court of Manila in Civil
Case No. 43073, for ejectment. Having lost therein, defendants-appellants appealed to the court a
quo (Civil Case No. 30993) which also rendered a decision against them, the dispositive portion of
which follows:

WHEREFORE, the court hereby renders judgment in favor of the plaintiffs and
against the defendants, ordering the latter to pay jointly and severally the former a
monthly rent of P200.00 on the house, subject-matter of this action, from March 27,
1956, to January 14, 1967, with interest at the legal rate from April 18, 1956, the filing
of the complaint, until fully paid, plus attorney's fees in the sum of P300.00 and to pay
the costs.

It appears on the records that on 1 September 1955 defendants-appellants executed a chattel


mortgage in favor of plaintiffs-appellees over their house of strong materials located at No. 550 Int. 3,
Quezon Boulevard, Quiapo, Manila, over Lot Nos. 6-B and 7-B, Block No. 2554, which were being
rented from Madrigal & Company, Inc. The mortgage was registered in the Registry of Deeds of
Manila on 2 September 1955. The herein mortgage was executed to guarantee a loan of P4,800.00
received from plaintiffs-appellees, payable within one year at 12% per annum. The mode of payment
was P150.00 monthly, starting September, 1955, up to July 1956, and the lump sum of P3,150 was
payable on or before August, 1956. It was also agreed that default in the payment of any of the
amortizations, would cause the remaining unpaid balance to becomeimmediately due and Payable
and —

the Chattel Mortgage will be enforceable in accordance with the provisions of Special
Act No. 3135, and for this purpose, the Sheriff of the City of Manila or any of his
deputies is hereby empowered and authorized to sell all the Mortgagor's property
after the necessary publication in order to settle the financial debts of P4,800.00, plus
12% yearly interest, and attorney's fees... 2

When defendants-appellants defaulted in paying, the mortgage was extrajudicially foreclosed, and on
27 March 1956, the house was sold at public auction pursuant to the said contract. As highest bidder,
plaintiffs-appellees were issued the corresponding certificate of sale.3 Thereafter, on 18 April 1956,
plaintiffs-appellant commenced Civil Case No. 43073 in the municipal court of Manila, praying, among
other things, that the house be vacated and its possession surrendered to them, and for defendants-
appellants to pay rent of P200.00 monthly from 27 March 1956 up to the time the possession is
surrendered.4 On 21 September 1956, the municipal court rendered its decision —

... ordering the defendants to vacate the premises described in the complaint;
ordering further to pay monthly the amount of P200.00 from March 27, 1956, until
such (time that) the premises is (sic) completely vacated; plus attorney's fees of
P100.00 and the costs of the suit.5

Defendants-appellants, in their answers in both the municipal court and court a quo impugned the
legality of the chattel mortgage, claiming that they are still the owners of the house; but they waived
the right to introduce evidence, oral or documentary. Instead, they relied on their memoranda in
support of their motion to dismiss, predicated mainly on the grounds that: (a) the municipal court did
not have jurisdiction to try and decide the case because (1) the issue involved, is ownership, and (2)
there was no allegation of prior possession; and (b) failure to prove prior demand pursuant to Section
2, Rule 72, of the Rules of Court.6

During the pendency of the appeal to the Court of First Instance, defendants-appellants failed to
deposit the rent for November, 1956 within the first 10 days of December, 1956 as ordered in the
decision of the municipal court. As a result, the court granted plaintiffs-appellees' motion for
execution, and it was actually issued on 24 January 1957. However, the judgment regarding the
surrender of possession to plaintiffs-appellees could not be executed because the subject house had
been already demolished on 14 January 1957 pursuant to the order of the court in a separate civil
case (No. 25816) for ejectment against the present defendants for non-payment of rentals on the land
on which the house was constructed.
12

The motion of plaintiffs for dismissal of the appeal, execution of the supersedeas bond and withdrawal
of deposited rentals was denied for the reason that the liability therefor was disclaimed and was still
being litigated, and under Section 8, Rule 72, rentals deposited had to be held until final disposition of
the appeal.7

On 7 October 1957, the appellate court of First Instance rendered its decision, the dispositive portion
of which is quoted earlier. The said decision was appealed by defendants to the Court of Appeals
which, in turn, certified the appeal to this Court. Plaintiffs-appellees failed to file a brief and this appeal
was submitted for decision without it.

Defendants-appellants submitted numerous assignments of error which can be condensed into two
questions, namely: .

(a) Whether the municipal court from which the case originated had jurisdiction to
adjudicate the same;

(b) Whether the defendants are, under the law, legally bound to pay rentals to the
plaintiffs during the period of one (1) year provided by law for the redemption of the
extrajudicially foreclosed house.

We will consider these questions seriatim.

(a) Defendants-appellants mortgagors question the jurisdiction of the municipal court from which the
case originated, and consequently, the appellate jurisdiction of the Court of First Instance a quo, on
the theory that the chattel mortgage is void ab initio; whence it would follow that the extrajudicial
foreclosure, and necessarily the consequent auction sale, are also void. Thus, the ownership of the
house still remained with defendants-appellants who are entitled to possession and not plaintiffs-
appellees. Therefore, it is argued by defendants-appellants, the issue of ownership will have to be
adjudicated first in order to determine possession. lt is contended further that ownership being in
issue, it is the Court of First Instance which has jurisdiction and not the municipal court.

Defendants-appellants predicate their theory of nullity of the chattel mortgage on two grounds, which
are: (a) that, their signatures on the chattel mortgage were obtained through fraud, deceit, or trickery;
and (b) that the subject matter of the mortgage is a house of strong materials, and, being an
immovable, it can only be the subject of a real estate mortgage and not a chattel mortgage.

On the charge of fraud, deceit or trickery, the Court of First Instance found defendants-appellants'
contentions as not supported by evidence and accordingly dismissed the charge, 8 confirming the
earlier finding of the municipal court that "the defense of ownership as well as the allegations of fraud
and deceit ... are mere allegations."9

It has been held in Supia and Batiaco vs. Quintero and Ayala10 that "the answer is a mere statement
of the facts which the party filing it expects to prove, but it is not evidence;11 and further, that when the
question to be determined is one of title, the Court is given the authority to proceed with the hearing of
the cause until this fact is clearly established. In the case of Sy vs. Dalman,12 wherein the defendant
was also a successful bidder in an auction sale, it was likewise held by this Court that in detainer
cases the aim of ownership "is a matter of defense and raises an issue of fact which should be
determined from the evidence at the trial." What determines jurisdiction are the allegations or
averments in the complaint and the relief asked for. 13

Moreover, even granting that the charge is true, fraud or deceit does not render a contract void ab
initio, and can only be a ground for rendering the contract voidable or annullable pursuant to Article
1390 of the New Civil Code, by a proper action in court. 14 There is nothing on record to show that the
mortgage has been annulled. Neither is it disclosed that steps were taken to nullify the same. Hence,
defendants-appellants' claim of ownership on the basis of a voidable contract which has not been
voided fails.
13

It is claimed in the alternative by defendants-appellants that even if there was no fraud, deceit or
trickery, the chattel mortgage was still null and void ab initio because only personal properties can be
subject of a chattel mortgage. The rule about the status of buildings as immovable property is stated
in Lopez vs. Orosa, Jr. and Plaza Theatre Inc.,15cited in Associated Insurance Surety Co., Inc. vs. Iya,
et al. 16 to the effect that —

... it is obvious that the inclusion of the building, separate and distinct from the land, in
the enumeration of what may constitute real properties (art. 415, New Civil Code)
could only mean one thing — that a building is by itself an immovable
property irrespective of whether or not said structure and the land on which it is
adhered to belong to the same owner.

Certain deviations, however, have been allowed for various reasons. In the case of Manarang and
Manarang vs. Ofilada,17 this Court stated that "it is undeniable that the parties to a contract may by
agreement treat as personal property that which by nature would be real property", citing Standard Oil
Company of New York vs. Jaramillo. 18 In the latter case, the mortgagor conveyed and transferred to
the mortgagee by way of mortgage "the following described personal property." 19 The "personal
property" consisted of leasehold rights and a building. Again, in the case of Luna vs.
Encarnacion,20 the subject of the contract designated as Chattel Mortgage was a house of mixed
materials, and this Court hold therein that it was a valid Chattel mortgage because it was so expressly
designated and specifically that the property given as security "is a house of mixed materials, which
by its very nature is considered personal property." In the later case of Navarro vs. Pineda,21 this
Court stated that —

The view that parties to a deed of chattel mortgage may agree to consider a house as
personal property for the purposes of said contract, "is good only insofar as the
contracting parties are concerned. It is based, partly, upon the principle of estoppel"
(Evangelista vs. Alto Surety, No. L-11139, 23 April 1958). In a case, a mortgaged
house built on a rented land was held to be a personal property, not only because the
deed of mortgage considered it as such, but also because it did not form part of the
land (Evangelists vs. Abad, [CA]; 36 O.G. 2913), for it is now settled that an object
placed on land by one who had only a temporary right to the same, such as the
lessee or usufructuary, does not become immobilized by attachment (Valdez vs.
Central Altagracia, 222 U.S. 58, cited in Davao Sawmill Co., Inc. vs. Castillo, et al.,
61 Phil. 709). Hence, if a house belonging to a person stands on a rented land
belonging to another person, it may be mortgaged as a personal property as so
stipulated in the document of mortgage. (Evangelista vs. Abad, Supra.) It should be
noted, however that the principle is predicated on statements by the owner declaring
his house to be a chattel, a conduct that may conceivably estop him from
subsequently claiming otherwise. (Ladera vs. C.N. Hodges, [CA] 48 O.G. 5374): 22

In the contract now before Us, the house on rented land is not only expressly designated as Chattel
Mortgage; it specifically provides that "the mortgagor ... voluntarily CEDES, SELLS and
TRANSFERS by way of Chattel Mortgage23 the property together with its leasehold rights over the lot
on which it is constructed and participation ..." 24 Although there is no specific statement referring to
the subject house as personal property, yet by ceding, selling or transferring a property by way of
chattel mortgage defendants-appellants could only have meant to convey the house as chattel, or at
least, intended to treat the same as such, so that they should not now be allowed to make an
inconsistent stand by claiming otherwise. Moreover, the subject house stood on a rented lot to which
defendats-appellants merely had a temporary right as lessee, and although this can not in itself alone
determine the status of the property, it does so when combined with other factors to sustain the
interpretation that the parties, particularly the mortgagors, intended to treat the house as personalty.
Finally unlike in the Iya cases, Lopez vs. Orosa, Jr. and Plaza Theatre, Inc. 25 and Leung Yee vs. F. L.
Strong Machinery and Williamson, 26 wherein third persons assailed the validity of the chattel
mortgage,27 it is the defendants-appellants themselves, as debtors-mortgagors, who are attacking the
validity of the chattel mortgage in this case. The doctrine of estoppel therefore applies to the herein
defendants-appellants, having treated the subject house as personalty.
14

(b) Turning to the question of possession and rentals of the premises in question. The Court of First
Instance noted in its decision that nearly a year after the foreclosure sale the mortgaged house had
been demolished on 14 and 15 January 1957 by virtue of a decision obtained by the lessor of the land
on which the house stood. For this reason, the said court limited itself to sentencing the erstwhile
mortgagors to pay plaintiffs a monthly rent of P200.00 from 27 March 1956 (when the chattel
mortgage was foreclosed and the house sold) until 14 January 1957 (when it was torn down by the
Sheriff), plus P300.00 attorney's fees.

Appellants mortgagors question this award, claiming that they were entitled to remain in possession
without any obligation to pay rent during the one year redemption period after the foreclosure sale,
i.e., until 27 March 1957. On this issue, We must rule for the appellants.

Chattel mortgages are covered and regulated by the Chattel Mortgage Law, Act No. 1508. 28 Section
14 of this Act allows the mortgagee to have the property mortgaged sold at public auction through a
public officer in almost the same manner as that allowed by Act No. 3135, as amended by Act No.
4118, provided that the requirements of the law relative to notice and registration are complied
with. 29 In the instant case, the parties specifically stipulated that "the chattel mortgage will
be enforceable in accordance with the provisions of Special Act No. 3135 ... ." 30(Emphasis supplied).

Section 6 of the Act referred to 31 provides that the debtor-mortgagor (defendants-appellants herein)
may, at any time within one year from and after the date of the auction sale, redeem the property sold
at the extra judicial foreclosure sale. Section 7 of the same Act 32 allows the purchaser of the property
to obtain from the court the possession during the period of redemption: but the same provision
expressly requires the filing of a petition with the proper Court of First Instance and the furnishing of a
bond. It is only upon filing of the proper motion and the approval of the corresponding bond that the
order for a writ of possession issues as a matter of course. No discretion is left to the court. 33 In the
absence of such a compliance, as in the instant case, the purchaser can not claim possession during
the period of redemption as a matter of right. In such a case, the governing provision is Section 34,
Rule 39, of the Revised Rules of Court 34 which also applies to properties purchased in extrajudicial
foreclosure proceedings.35 Construing the said section, this Court stated in the aforestated case
of Reyes vs. Hamada.

In other words, before the expiration of the 1-year period within which the judgment-
debtor or mortgagor may redeem the property, the purchaser thereof is not entitled,
as a matter of right, to possession of the same. Thus, while it is true that the Rules of
Court allow the purchaser to receive the rentals if the purchased property is occupied
by tenants, he is, nevertheless, accountable to the judgment-debtor or mortgagor as
the case may be, for the amount so received and the same will be duly credited
against the redemption price when the said debtor or mortgagor effects the
redemption. Differently stated, the rentals receivable from tenants, although they may
be collected by the purchaser during the redemption period, do not belong to the
latter but still pertain to the debtor of mortgagor. The rationale for the Rule, it seems,
is to secure for the benefit of the debtor or mortgagor, the payment of the redemption
amount and the consequent return to him of his properties sold at public auction.
(Emphasis supplied)

The Hamada case reiterates the previous ruling in Chan vs. Espe.36

Since the defendants-appellants were occupying the house at the time of the auction sale, they are
entitled to remain in possession during the period of redemption or within one year from and after 27
March 1956, the date of the auction sale, and to collect the rents or profits during the said period.

It will be noted further that in the case at bar the period of redemption had not yet expired when action
was instituted in the court of origin, and that plaintiffs-appellees did not choose to take possession
under Section 7, Act No. 3135, as amended, which is the law selected by the parties to govern the
extrajudicial foreclosure of the chattel mortgage. Neither was there an allegation to that effect. Since
plaintiffs-appellees' right to possess was not yet born at the filing of the complaint, there could be no
violation or breach thereof. Wherefore, the original complaint stated no cause of action and was
prematurely filed. For this reason, the same should be ordered dismissed, even if there was no
15

assignment of error to that effect. The Supreme Court is clothed with ample authority to review
palpable errors not assigned as such if it finds that their consideration is necessary in arriving at a just
decision of the cases. 37

It follows that the court below erred in requiring the mortgagors to pay rents for the year following the
foreclosure sale, as well as attorney's fees.

FOR THE FOREGOING REASONS, the decision appealed from is reversed and another one
entered, dismissing the complaint. With costs against plaintiffs-appellees.

Concepcion, C.J., Dizon, Makalintal, Zaldivar, Castro, Fernando, Teehankee, Barredo, Villamor and
Makasiar, JJ., concur.

Footnotes

1 Exhibit "A," page 1, Folder of Exhibits.

2 See paragraph "G," Exhibit "A," supra.

3 Exhibit "B," page 4, Folder of Exhibits.

4 Page 2, Defendants' Record on appeal, page 97, Rollo.

5 Page 20, Id., page 115, Rollo.

6 Now Section 2, Rule 70, Revised Rules of Court, which reads that —

"SEC. 2. Landlord, to proceed against tenant only after demand. — No landlord, or


his legal representative or assign, shall bring such action against a tenant for failure
to pay rent due or to comply with the conditions of his lease, unless the tenant shall
have failed to pay such rent or comply with such conditions for a period of ... five (5)
days in the case of building, after demand therefor, made upon him personally, or by
serving written notice of such demand upon the person found on the premises, or by
posting such notice on the premises if no persons be found thereon."

7 See CFI order of 20 February 1957, pages 21-25, Defendants' Record on Appeal.

8 Page 31, Defendants' Record on Appeal, page 213, Rollo.

9 See Municipal court decision, pages 17-18, Defendants' Record on Appeal, pages
199-200, Rollo.

10 59 Phil. 320-321.

11 Emphasis supplied.

12 L-19200, 27 February 1958, 22 SCRA 834; See also Aquino vs. Deala, 63 Phil.
582 and De los Reyes vs. Elepaño, et al., G.R. No. L-3466, 13 October 1950.

13 See Canaynay vs. Sarmiento, L-1246, 27 August 1947, 79 Phil. 36.

14 Last paragraph, Article 1290, N.C.C., supra.


16

15 No. L-10817-18, 28 February 1958, 103 Phil. 98.

16 No. L-10827-38, 30 May 1958, 103 Phil. 972.

17 No. L-8133, 18 May 1956, 99 Phil. 109.

18 No. L-20329, 16 March 1923, 44 Phil. 632.

19 Emphasis supplied.

20 No. L-4637, 30 June 1952, 91 Phil. 531.

21 No. L-18456, 30 November 1963, 9 SCRA 631.

22 Emphasis supplied.

23 Emphasis supplied.

24 See paragraph 2 of Exhibit "A," page 1, Folder of Exhibits.

25 Supra.

26 Supra.

27 See Navarro vs. Pineda, supra.

28 Effective 1 August 1906.

29 See Luna vs. Encarnacion, et al., No. L-4637, 30 June 1952, 91 Phil. 531.

30 See paragraph "G," Exhibit "A," supra.

31 Section 6, Act No. 3135, as amended, provides:

"In all cases in which an extrajudicial sale is made under the special power
hereinbefore referred to, the debtor, his successor in interest or any judicial creditor
or judgment creditor of said debtor, or any person having a lien on the property
subsequent to the mortgage or deed of trust under which the property is sold, may
redeem the same at any time within the term of one year from and after the date of
the sale; and such redemption shall be governed by the provisions of sections four
hundredand sixty-four to four hundred and sixty-six, inclusive, of the Code of Civil
Procedure, in so far as these are not inconsistent with the provisions of this Act."
(Emphasis supplied) .

32 Section 7, Act No. 3135, as amended, states: .

"In any sale made under the provisions of this Act, the purchaser may petition the
Court of First Instance of the province or place where the property or any part thereof
is situated, to give him possession thereof during the redemption period, furnishing
bond in an amount equivalent to the use of the property for a period of twelve
months, to indemnify the debtor in case it be shown that the sale was made without
violating the mortgage or without complying with the requirements of this Act..."
(Emphasis supplied) .

33 See De Gracia vs. San Jose, et al., No. L-6493, 25 March 1954.
17

34 "SEC. 34. Rents and profits pending redemption. Statement thereof and credit
therefor on redemption. — The purchaser, from the time of the sale until a
redemption, and a redemptioner, from the time of his redemption until another
redemption, is entitled to receive the rents of the property sold or the value of the use
and occupation thereof when such property is in possession of a tenant. But when
any such rents and profits have been received by the judgment creditor or purchaser,
or by a redemptioner, or by the assignee or either of them, from property thus sold
preceding such redemption, the amounts of such rents and profits shall be a credit
upon the redemption money to be paid; ..."

35 See Reyes vs. Hamada, No. L-19967, 31 May 1965, 14 SCRA 215; Emphasis
supplied.

36 No. L-16777, 20 April 1961, 1 SCRA 1004.

37 Saura Import & Export Co. vs. Philippine International Surety Co., et al., No. L-
15184, 31 May 1963, 8 SCRA 143, 148; Hernandez vs. Andal, 78 Phil.198, See also
Sec. 7, Rule 51, of the Revised Rules of Court. Cf. Santaells vs.Otto Lange Co., 155
Fed. 719; Mast vs. Superior Drill Co., 154 Fed., 45, Francisco, Rules of Court (1965
Ed), Vol. 3, page 765.

ASSOCIATED INSURANCE v. IYA (G.R. Nos. L-10837-38, May 30, 1958)

Republic of the Philippines


SUPREME COURT
Manila

EN BANC

G.R. Nos. L-10837-38 May 30, 1958

ASSOCIATED INSURANCE and SURETY COMPANY, INC., plaintiff,


vs.
ISABEL IYA, ADRIANO VALINO and LUCIA VALINO, defendants.

ISABEL IYA, plaintiff,


vs.
ADRIANO VALINO, LUCIA VALINO and ASSOCIATED INSURANCE and SURETY COMPANY.
INC., defendants.

Jovita L. de Dios for defendant Isabel Iya.


M. Perez Cardenas and Apolonio Abola for defendant Associated Insurance and Surety Co., Inc.

FELIX, J.:

Adriano Valino and Lucia A. Valino, husband and wife, were the owners and possessors of a house of
strong materials constructed on Lot No. 3, Block No. 80 of the Grace Park Subdivision in Caloocan,
Rizal, which they purchased on installment basis from the Philippine Realty Corporation. On
November 6, 1951, to enable her to purchase on credit rice from the NARIC, Lucia A. Valino filed a
bond in the sum of P11,000.00 (AISCO Bond No. G-971) subscribed by the Associated Insurance and
Surety Co., Inc., and as counter-guaranty therefor, the spouses Valino executed an
alleged chattel mortgage on the aforementioned house in favor of the surety company, which
encumbrance was duly registered with the Chattel Mortgage Register of Rizal on December 6, 1951.
It is admitted that at the time said undertaking took place, the parcel of land on which the house is
18

erected was still registered in the name of the Philippine Realty Corporation. Having completed
payment on the purchase price of the lot, the Valinos were able to secure on October 18, 1958, a
certificate of title in their name (T.C.T. No. 27884). Subsequently, however, or on October 24, 1952,
the Valinos, to secure payment of an indebtedness in the amount of P12,000.00, executed a real
estate mortgage over the lot and the house in favor of Isabel Iya, which was duly registered and
annotated at the back of the certificate of title.

On the other hand, as Lucia A. Valino, failed to satisfy her obligation to the NARIC, the surety
company was compelled to pay the same pursuant to the undertaking of the bond. In turn, the surety
company demanded reimbursement from the spouses Valino, and as the latter likewise failed to do
so, the company foreclosed the chattel mortgage over the house. As a result thereof, a public sale
was conducted by the Provincial Sheriff of Rizal on December 26, 1952, wherein the property was
awarded to the surety company for P8,000.00, the highest bid received therefor. The surety company
then caused the said house to be declared in its name for tax purposes (Tax Declaration No. 25128).

Sometime in July, 1953, the surety company learned of the existence of the real estate mortgage over
the lot covered by T.C.T. No. 26884 together with the improvements thereon; thus, said surety
company instituted Civil Case No. 2162 of the Court of First Instance of Manila naming Adriano and
Lucia Valino and Isabel Iya, the mortgagee, as defendants. The complaint prayed for the exclusion of
the residential house from the real estate mortgage in favor of defendant Iya and the declaration and
recognition of plaintiff's right to ownership over the same in virtue of the award given by the Provincial
Sheriff of Rizal during the public auction held on December 26, 1952. Plaintiff likewise asked the
Court to sentence the spouses Valino to pay said surety moral and exemplary damages, attorney's
fees and costs. Defendant Isabel Iya filed her answer to the complaint alleging among other things,
that in virtue of the real estate mortgage executed by her co-defendants, she acquired a real right
over the lot and the house constructed thereon; that the auction sale allegedly conducted by the
Provincial Sheriff of Rizal as a result of the foreclosure of the chattel mortgage on the house was null
and void for non-compliance with the form required by law. She, therefore, prayed for the dismissal of
the complaint and anullment of the sale made by the Provincial Sheriff. She also demanded the
amount of P5,000.00 from plaintiff as counterclaim, the sum of P5,000.00 from her co-defendants as
crossclaim, for attorney's fees and costs.

Defendants spouses in their answer admitted some of the averments of the complaint and denied the
others. They, however, prayed for the dismissal of the action for lack of cause of action, it being
alleged that plaintiff was already the owner of the house in question, and as said defendants admitted
this fact, the claim of the former was already satisfied.

On October 29, 1953, Isabel Iya filed another civil action against the Valinos and the surety company
(Civil Case No. 2504 of the Court of First Instance of Manila) stating that pursuant to the contract of
mortgage executed by the spouses Valino on October 24, 1952, the latter undertook to pay a loan of
P12,000.00 with interest at 12% per annum or P120.00 a month, which indebtedness was payable in
4 years, extendible for only one year; that to secure payment thereof, said defendants mortgaged the
house and lot covered by T.C.T. No. 27884 located at No. 67 Baltazar St., Grace Park Subdivision,
Caloocan, Rizal; that the Associated Insurance and Surety Co., Inc., was included as a party
defendant because it claimed to have an interest on the residential house also covered by said
mortgage; that it was stipulated in the aforesaid real estate mortgage that default in the payment of
the interest agreed upon would entitle the mortgagee to foreclose the same even before the lapse of
the 4-year period; and as defendant spouses had allegedly failed to pay the interest for more than 6
months, plaintiff prayed the Court to order said defendants to pay the sum of P12,000.00 with interest
thereon at 12% per annum from March 25, 1953, until fully paid; for an additional sum equivalent to
20% of the total obligation as damages, and for costs. As an alternative in case such demand may not
be met and satisfied plaintiff prayed for a decree of foreclosure of the land, building and other
improvements thereon to be sold at public auction and the proceeds thereof applied to satisfy the
demands of plaintiff; that the Valinos, the surety company and any other person claiming interest on
the mortgaged properties be barred and foreclosed of all rights, claims or equity of redemption in said
properties; and for deficiency judgment in case the proceeds of the sale of the mortgaged property
would be insufficient to satisfy the claim of plaintiff.
19

Defendant surety company, in answer to this complaint insisted on its right over the building, arguing
that as the lot on which the house was constructed did not belong to the spouses at the time the
chattel mortgage was executed, the house might be considered only as a personal property and that
the encumbrance thereof and the subsequent foreclosure proceedings made pursuant to the
provisions of the Chattel Mortgage Law were proper and legal. Defendant therefore prayed that said
building be excluded from the real estate mortgage and its right over the same be declared superior to
that of plaintiff, for damages, attorney's fees and costs.

Taking side with the surety company, defendant spouses admitted the due execution of the mortgage
upon the land but assailed the allegation that the building was included thereon, it being contended
that it was already encumbered in favor of the surety company before the real estate mortgage was
executed, a fact made known to plaintiff during the preparation of said contract and to which the latter
offered no objection. As a special defense, it was asserted that the action was premature because the
contract was for a period of 4 years, which had not yet elapsed.

The two cases were jointly heard upon agreement of the parties, who submitted the same on a
stipulation of facts, after which the Court rendered judgment dated March 8, 1956, holding that the
chattel mortgage in favor of the Associated Insurance and Surety Co., Inc., was preferred and
superior over the real estate mortgage subsequently executed in favor of Isabel Iya. It was ruled that
as the Valinos were not yet the registered owner of the land on which the building in question was
constructed at the time the first encumbrance was made, the building then was still a personality and
a chattel mortgage over the same was proper. However, as the mortgagors were already the owner of
the land at the time the contract with Isabel Iya was entered into, the building was transformed into a
real property and the real estate mortgage created thereon was likewise adjudged as proper. It is to
be noted in this connection that there is no evidence on record to sustain the allegation of the
spouses Valino that at the time they mortgaged their house and lot to Isabel Iya, the latter was told or
knew that part of the mortgaged property, i.e., the house, had previously been mortgaged to the
surety company.

The residential building was, therefore, ordered excluded from the foreclosure prayed for by Isabel
Iya, although the latter could exercise the right of a junior encumbrance. So the spouses Valino were
ordered to pay the amount demanded by said mortgagee or in their default to have the parcel of land
subject of the mortgage sold at public auction for the satisfaction of Iya's claim.

There is no question as to appellant's right over the land covered by the real estate mortgage;
however, as the building constructed thereon has been the subject of 2 mortgages; controversy arise
as to which of these encumbrances should receive preference over the other. The decisive factor in
resolving the issue presented by this appeal is the determination of the nature of the structure litigated
upon, for where it be considered a personality, the foreclosure of the chattel mortgage and the
subsequent sale thereof at public auction, made in accordance with the Chattel Mortgage Law would
be valid and the right acquired by the surety company therefrom would certainly deserve prior
recognition; otherwise, appellant's claim for preference must be granted. The lower Court, deciding in
favor of the surety company, based its ruling on the premise that as the mortgagors were not the
owners of the land on which the building is erected at the time the first encumbrance was made, said
structure partook of the nature of a personal property and could properly be the subject of a chattel
mortgage. We find reason to hold otherwise, for as this Court, defining the nature or character of a
building, has said:

. . . while it is true that generally, real estate connotes the land and the building constructed
thereon, it is obvious that the inclusion of the building, separate and distinct from the land, in
the enumeration of what may constitute real properties (Art. 415, new Civil Code) could only
mean one thing — that a building is by itself an immovable property . . . Moreover, and in
view of the absence of any specific provision to the contrary, a building is an immovable
property irrespective of whether or not said structure and the land on which it is adhered to
belong to the same owner. (Lopez vs. Orosa, G.R. Nos. supra, p. 98).

A building certainly cannot be divested of its character of a realty by the fact that the land on which it
is constructed belongs to another. To hold it the other way, the possibility is not remote that it would
result in confusion, for to cloak the building with an uncertain status made dependent on the
20

ownership of the land, would create a situation where a permanent fixture changes its nature or
character as the ownership of the land changes hands. In the case at bar, as personal properties
could only be the subject of a chattel mortgage (Section 1, Act 3952) and as obviously the structure in
question is not one, the execution of the chattel mortgage covering said building is clearly invalid and
a nullity. While it is true that said document was correspondingly registered in the Chattel Mortgage
Register of Rizal, this act produced no effect whatsoever for where the interest conveyed is in the
nature of a real property, the registration of the document in the registry of chattels is merely a futile
act. Thus, the registration of the chattel mortgage of a building of strong materials produce no effect
as far as the building is concerned (Leung Yee vs. Strong Machinery Co., 37 Phil., 644). Nor can we
give any consideration to the contention of the surety that it has acquired ownership over the property
in question by reason of the sale conducted by the Provincial Sheriff of Rizal, for as this Court has
aptly pronounced:

A mortgage creditor who purchases real properties at an extrajudicial foreclosure sale thereof
by virtue of a chattel mortgage constituted in his favor, which mortgage has been declared
null and void with respect to said real properties, acquires no right thereto by virtue of said
sale (De la Riva vs. Ah Keo, 60 Phil., 899).

Wherefore the portion of the decision of the lower Court in these two cases appealed from holding the
rights of the surety company, over the building superior to that of Isabel Iya and excluding the building
from the foreclosure prayed for by the latter is reversed and appellant Isabel Iya's right to foreclose
not only the land but also the building erected thereon is hereby recognized, and the proceeds of the
sale thereof at public auction (if the land has not yet been sold), shall be applied to the unsatisfied
judgment in favor of Isabel Iya. This decision however is without prejudice to any right that the
Associated Insurance and Surety Co., Inc., may have against the spouses Adriano and Lucia Valino
on account of the mortgage of said building they executed in favor of said surety company. Without
pronouncement as to costs. It is so ordered.

Paras, C.J., Bengzon, Montemayor, Reyes, A., Bautista Angelo, Labrador, Concepcion, Reyes,
J.B.L., and Endencia, JJ., concur.

MAKATI LEASING v. WEAREVER (G.R. No. L-58469, May 16, 1983)

Republic of the Philippines


SUPREME COURT
Manila

SECOND DIVISION

G.R. No. L-58469 May 16, 1983

MAKATI LEASING and FINANCE CORPORATION, petitioner,


vs.
WEAREVER TEXTILE MILLS, INC., and HONORABLE COURT OF APPEALS, respondents.

Loreto C. Baduan for petitioner.

Ramon D. Bagatsing & Assoc. (collaborating counsel) for petitioner.

Jose V. Mancella for respondent.


21

DE CASTRO, J.:

Petition for review on certiorari of the decision of the Court of Appeals (now Intermediate Appellate
Court) promulgated on August 27, 1981 in CA-G.R. No. SP-12731, setting aside certain Orders later
specified herein, of Judge Ricardo J. Francisco, as Presiding Judge of the Court of First instance of
Rizal Branch VI, issued in Civil Case No. 36040, as wen as the resolution dated September 22, 1981
of the said appellate court, denying petitioner's motion for reconsideration.

It appears that in order to obtain financial accommodations from herein petitioner Makati Leasing and
Finance Corporation, the private respondent Wearever Textile Mills, Inc., discounted and assigned
several receivables with the former under a Receivable Purchase Agreement. To secure the
collection of the receivables assigned, private respondent executed a Chattel Mortgage over certain
raw materials inventory as well as a machinery described as an Artos Aero Dryer Stentering Range.

Upon private respondent's default, petitioner filed a petition for extrajudicial foreclosure of the
properties mortgage to it. However, the Deputy Sheriff assigned to implement the foreclosure failed to
gain entry into private respondent's premises and was not able to effect the seizure of the
aforedescribed machinery. Petitioner thereafter filed a complaint for judicial foreclosure with the Court
of First Instance of Rizal, Branch VI, docketed as Civil Case No. 36040, the case before the lower
court.

Acting on petitioner's application for replevin, the lower court issued a writ of seizure, the enforcement
of which was however subsequently restrained upon private respondent's filing of a motion for
reconsideration. After several incidents, the lower court finally issued on February 11, 1981, an order
lifting the restraining order for the enforcement of the writ of seizure and an order to break open the
premises of private respondent to enforce said writ. The lower court reaffirmed its stand upon private
respondent's filing of a further motion for reconsideration.

On July 13, 1981, the sheriff enforcing the seizure order, repaired to the premises of private
respondent and removed the main drive motor of the subject machinery.

The Court of Appeals, in certiorari and prohibition proceedings subsequently filed by herein private
respondent, set aside the Orders of the lower court and ordered the return of the drive motor seized
by the sheriff pursuant to said Orders, after ruling that the machinery in suit cannot be the subject of
replevin, much less of a chattel mortgage, because it is a real property pursuant to Article 415 of the
new Civil Code, the same being attached to the ground by means of bolts and the only way to remove
it from respondent's plant would be to drill out or destroy the concrete floor, the reason why all that the
sheriff could do to enfore the writ was to take the main drive motor of said machinery. The appellate
court rejected petitioner's argument that private respondent is estopped from claiming that the
machine is real property by constituting a chattel mortgage thereon.

A motion for reconsideration of this decision of the Court of Appeals having been denied, petitioner
has brought the case to this Court for review by writ of certiorari. It is contended by private
respondent, however, that the instant petition was rendered moot and academic by petitioner's act of
returning the subject motor drive of respondent's machinery after the Court of Appeals' decision was
promulgated.

The contention of private respondent is without merit. When petitioner returned the subject motor
drive, it made itself unequivocably clear that said action was without prejudice to a motion for
reconsideration of the Court of Appeals decision, as shown by the receipt duly signed by respondent's
representative. 1 Considering that petitioner has reserved its right to question the propriety of the
Court of Appeals' decision, the contention of private respondent that this petition has been mooted by
such return may not be sustained.

The next and the more crucial question to be resolved in this Petition is whether the machinery in suit
is real or personal property from the point of view of the parties, with petitioner arguing that it is a
personality, while the respondent claiming the contrary, and was sustained by the appellate court,
22

which accordingly held that the chattel mortgage constituted thereon is null and void, as contended by
said respondent.

A similar, if not Identical issue was raised in Tumalad v. Vicencio, 41 SCRA 143 where this Court,
speaking through Justice J.B.L. Reyes, ruled:

Although there is no specific statement referring to the subject house as personal


property, yet by ceding, selling or transferring a property by way of chattel mortgage
defendants-appellants could only have meant to convey the house as chattel, or at
least, intended to treat the same as such, so that they should not now be allowed to
make an inconsistent stand by claiming otherwise. Moreover, the subject house stood
on a rented lot to which defendants-appellants merely had a temporary right as
lessee, and although this can not in itself alone determine the status of the property, it
does so when combined with other factors to sustain the interpretation that the
parties, particularly the mortgagors, intended to treat the house as personality.
Finally, unlike in the Iya cases, Lopez vs. Orosa, Jr. & Plaza Theatre, Inc. & Leung
Yee vs. F.L. Strong Machinery & Williamson, wherein third persons assailed the
validity of the chattel mortgage, it is the defendants-appellants themselves, as
debtors-mortgagors, who are attacking the validity of the chattel mortgage in this
case. The doctrine of estoppel therefore applies to the herein defendants-appellants,
having treated the subject house as personality.

Examining the records of the instant case, We find no logical justification to exclude the rule out, as
the appellate court did, the present case from the application of the abovequoted pronouncement. If a
house of strong materials, like what was involved in the above Tumalad case, may be considered as
personal property for purposes of executing a chattel mortgage thereon as long as the parties to the
contract so agree and no innocent third party will be prejudiced thereby, there is absolutely no reason
why a machinery, which is movable in its nature and becomes immobilized only by destination or
purpose, may not be likewise treated as such. This is really because one who has so agreed is
estopped from denying the existence of the chattel mortgage.

In rejecting petitioner's assertion on the applicability of the Tumalad doctrine, the Court of Appeals
lays stress on the fact that the house involved therein was built on a land that did not belong to the
owner of such house. But the law makes no distinction with respect to the ownership of the land on
which the house is built and We should not lay down distinctions not contemplated by law.

It must be pointed out that the characterization of the subject machinery as chattel by the private
respondent is indicative of intention and impresses upon the property the character determined by the
parties. As stated in Standard Oil Co. of New York v. Jaramillo, 44 Phil. 630, it is undeniable that the
parties to a contract may by agreement treat as personal property that which by nature would be real
property, as long as no interest of third parties would be prejudiced thereby.

Private respondent contends that estoppel cannot apply against it because it had never represented
nor agreed that the machinery in suit be considered as personal property but was merely required and
dictated on by herein petitioner to sign a printed form of chattel mortgage which was in a blank form at
the time of signing. This contention lacks persuasiveness. As aptly pointed out by petitioner and not
denied by the respondent, the status of the subject machinery as movable or immovable was never
placed in issue before the lower court and the Court of Appeals except in a supplemental
memorandum in support of the petition filed in the appellate court. Moreover, even granting that the
charge is true, such fact alone does not render a contract void ab initio, but can only be a ground for
rendering said contract voidable, or annullable pursuant to Article 1390 of the new Civil Code, by a
proper action in court. There is nothing on record to show that the mortgage has been annulled.
Neither is it disclosed that steps were taken to nullify the same. On the other hand, as pointed out by
petitioner and again not refuted by respondent, the latter has indubitably benefited from said contract.
Equity dictates that one should not benefit at the expense of another. Private respondent could not
now therefore, be allowed to impugn the efficacy of the chattel mortgage after it has benefited
therefrom,
23

From what has been said above, the error of the appellate court in ruling that the questioned
machinery is real, not personal property, becomes very apparent. Moreover, the case of Machinery
and Engineering Supplies, Inc. v. CA, 96 Phil. 70, heavily relied upon by said court is not applicable to
the case at bar, the nature of the machinery and equipment involved therein as real properties never
having been disputed nor in issue, and they were not the subject of a Chattel Mortgage. Undoubtedly,
the Tumalad case bears more nearly perfect parity with the instant case to be the more controlling
jurisprudential authority.

WHEREFORE, the questioned decision and resolution of the Court of Appeals are hereby reversed
and set aside, and the Orders of the lower court are hereby reinstated, with costs against the private
respondent.

SO ORDERED.

Makasiar (Chairman), Aquino, Concepcion Jr., Guerrero and Escolin JJ., concur.

Abad Santos, J., concurs in the result.

Footnotes

1 p. 52, Rollo.

BD. OF ASSESSMENT APPEALS v. MERALCO (G.R. No. L-15334, January 31, 1964)

Republic of the Philippines


SUPREME COURT
Manila

EN BANC

G.R. No. L-15334 January 31, 1964

BOARD OF ASSESSMENT APPEALS, CITY ASSESSOR and CITY TREASURER OF QUEZON


CITY, petitioners,
vs.
MANILA ELECTRIC COMPANY, respondent.

Assistant City Attorney Jaime R. Agloro for petitioners.


Ross, Selph and Carrascoso for respondent.

PAREDES, J.:

From the stipulation of facts and evidence adduced during the hearing, the following appear:

On October 20, 1902, the Philippine Commission enacted Act No. 484 which authorized the Municipal
Board of Manila to grant a franchise to construct, maintain and operate an electric street railway and
electric light, heat and power system in the City of Manila and its suburbs to the person or persons
making the most favorable bid. Charles M. Swift was awarded the said franchise on March 1903, the
24

terms and conditions of which were embodied in Ordinance No. 44 approved on March 24, 1903.
Respondent Manila Electric Co. (Meralco for short), became the transferee and owner of the
franchise.

Meralco's electric power is generated by its hydro-electric plant located at Botocan Falls, Laguna and
is transmitted to the City of Manila by means of electric transmission wires, running from the province
of Laguna to the said City. These electric transmission wires which carry high voltage current, are
fastened to insulators attached on steel towers constructed by respondent at intervals, from its hydro-
electric plant in the province of Laguna to the City of Manila. The respondent Meralco has constructed
40 of these steel towers within Quezon City, on land belonging to it. A photograph of one of these
steel towers is attached to the petition for review, marked Annex A. Three steel towers were inspected
by the lower court and parties and the following were the descriptions given there of by said court:

The first steel tower is located in South Tatalon, España Extension, Quezon City. The findings
were as follows: the ground around one of the four posts was excavated to a depth of about
eight (8) feet, with an opening of about one (1) meter in diameter, decreased to about a
quarter of a meter as it we deeper until it reached the bottom of the post; at the bottom of the
post were two parallel steel bars attached to the leg means of bolts; the tower proper was
attached to the leg three bolts; with two cross metals to prevent mobility; there was no
concrete foundation but there was adobe stone underneath; as the bottom of the excavation
was covered with water about three inches high, it could not be determined with certainty to
whether said adobe stone was placed purposely or not, as the place abounds with this kind of
stone; and the tower carried five high voltage wires without cover or any insulating materials.

The second tower inspected was located in Kamuning Road, K-F, Quezon City, on land
owned by the petitioner approximate more than one kilometer from the first tower. As in the
first tower, the ground around one of the four legs was excavate from seven to eight (8) feet
deep and one and a half (1-½) meters wide. There being very little water at the bottom, it was
seen that there was no concrete foundation, but there soft adobe beneath. The leg was
likewise provided with two parallel steel bars bolted to a square metal frame also bolted to
each corner. Like the first one, the second tower is made up of metal rods joined together by
means of bolts, so that by unscrewing the bolts, the tower could be dismantled and
reassembled.

The third tower examined is located along Kamias Road, Quezon City. As in the first two
towers given above, the ground around the two legs of the third tower was excavated to a
depth about two or three inches beyond the outside level of the steel bar foundation. It was
found that there was no concrete foundation. Like the two previous ones, the bottom
arrangement of the legs thereof were found to be resting on soft adobe, which, probably due
to high humidity, looks like mud or clay. It was also found that the square metal frame
supporting the legs were not attached to any material or foundation.

On November 15, 1955, petitioner City Assessor of Quezon City declared the aforesaid steel towers
for real property tax under Tax declaration Nos. 31992 and 15549. After denying respondent's petition
to cancel these declarations, an appeal was taken by respondent to the Board of Assessment
Appeals of Quezon City, which required respondent to pay the amount of P11,651.86 as real property
tax on the said steel towers for the years 1952 to 1956. Respondent paid the amount under protest,
and filed a petition for review in the Court of Tax Appeals (CTA for short) which rendered a decision
on December 29, 1958, ordering the cancellation of the said tax declarations and the petitioner City
Treasurer of Quezon City to refund to the respondent the sum of P11,651.86. The motion for
reconsideration having been denied, on April 22, 1959, the instant petition for review was filed.

In upholding the cause of respondents, the CTA held that: (1) the steel towers come within the term
"poles" which are declared exempt from taxes under part II paragraph 9 of respondent's franchise; (2)
the steel towers are personal properties and are not subject to real property tax; and (3) the City
Treasurer of Quezon City is held responsible for the refund of the amount paid. These are assigned
as errors by the petitioner in the brief.

The tax exemption privilege of the petitioner is quoted hereunder:


25

PAR 9. The grantee shall be liable to pay the same taxes upon its real estate, buildings, plant
(not including poles, wires, transformers, and insulators), machinery and personal property as
other persons are or may be hereafter required by law to pay ... Said percentage shall be due
and payable at the time stated in paragraph nineteen of Part One hereof, ... and shall be in
lieu of all taxes and assessments of whatsoever nature and by whatsoever authority upon the
privileges, earnings, income, franchise, and poles, wires, transformers, and insulators of the
grantee from which taxes and assessments the grantee is hereby expressly exempted. (Par.
9, Part Two, Act No. 484 Respondent's Franchise; emphasis supplied.)

The word "pole" means "a long, comparatively slender usually cylindrical piece of wood or timber, as
typically the stem of a small tree stripped of its branches; also by extension, a similar typically
cylindrical piece or object of metal or the like". The term also refers to "an upright standard to the top
of which something is affixed or by which something is supported; as a dovecote set on a pole;
telegraph poles; a tent pole; sometimes, specifically a vessel's master (Webster's New International
Dictionary 2nd Ed., p. 1907.) Along the streets, in the City of Manila, may be seen cylindrical metal
poles, cubical concrete poles, and poles of the PLDT Co. which are made of two steel bars joined
together by an interlacing metal rod. They are called "poles" notwithstanding the fact that they are no
made of wood. It must be noted from paragraph 9, above quoted, that the concept of the "poles" for
which exemption is granted, is not determined by their place or location, nor by the character of the
electric current it carries, nor the material or form of which it is made, but the use to which they are
dedicated. In accordance with the definitions, pole is not restricted to a long cylindrical piece of wood
or metal, but includes "upright standards to the top of which something is affixed or by which
something is supported. As heretofore described, respondent's steel supports consists of a framework
of four steel bars or strips which are bound by steel cross-arms atop of which are cross-arms
supporting five high voltage transmission wires (See Annex A) and their sole function is to support or
carry such wires.

The conclusion of the CTA that the steel supports in question are embraced in the term "poles" is not
a novelty. Several courts of last resort in the United States have called these steel supports "steel
towers", and they denominated these supports or towers, as electric poles. In their decisions the
words "towers" and "poles" were used interchangeably, and it is well understood in that jurisdiction
that a transmission tower or pole means the same thing.

In a proceeding to condemn land for the use of electric power wires, in which the law provided that
wires shall be constructed upon suitable poles, this term was construed to mean either wood or metal
poles and in view of the land being subject to overflow, and the necessary carrying of numerous wires
and the distance between poles, the statute was interpreted to include towers or poles. (Stemmons
and Dallas Light Co. (Tex) 212 S.W. 222, 224; 32-A Words and Phrases, p. 365.)

The term "poles" was also used to denominate the steel supports or towers used by an association
used to convey its electric power furnished to subscribers and members, constructed for the purpose
of fastening high voltage and dangerous electric wires alongside public highways. The steel supports
or towers were made of iron or other metals consisting of two pieces running from the ground up
some thirty feet high, being wider at the bottom than at the top, the said two metal pieces being
connected with criss-cross iron running from the bottom to the top, constructed like ladders and
loaded with high voltage electricity. In form and structure, they are like the steel towers in question.
(Salt River Valley Users' Ass'n v. Compton, 8 P. 2nd, 249-250.)

The term "poles" was used to denote the steel towers of an electric company engaged in the
generation of hydro-electric power generated from its plant to the Tower of Oxford and City of
Waterbury. These steel towers are about 15 feet square at the base and extended to a height of
about 35 feet to a point, and are embedded in the cement foundations sunk in the earth, the top of
which extends above the surface of the soil in the tower of Oxford, and to the towers are attached
insulators, arms, and other equipment capable of carrying wires for the transmission of electric power
(Connecticut Light and Power Co. v. Oxford, 101 Conn. 383, 126 Atl. p. 1).

In a case, the defendant admitted that the structure on which a certain person met his death was built
for the purpose of supporting a transmission wire used for carrying high-tension electric power, but
claimed that the steel towers on which it is carried were so large that their wire took their structure out
26

of the definition of a pole line. It was held that in defining the word pole, one should not be governed
by the wire or material of the support used, but was considering the danger from any elevated wire
carrying electric current, and that regardless of the size or material wire of its individual members, any
continuous series of structures intended and used solely or primarily for the purpose of supporting
wires carrying electric currents is a pole line (Inspiration Consolidation Cooper Co. v. Bryan 252 P.
1016).

It is evident, therefore, that the word "poles", as used in Act No. 484 and incorporated in the
petitioner's franchise, should not be given a restrictive and narrow interpretation, as to defeat the very
object for which the franchise was granted. The poles as contemplated thereon, should be understood
and taken as a part of the electric power system of the respondent Meralco, for the conveyance of
electric current from the source thereof to its consumers. If the respondent would be required to
employ "wooden poles", or "rounded poles" as it used to do fifty years back, then one should admit
that the Philippines is one century behind the age of space. It should also be conceded by now that
steel towers, like the ones in question, for obvious reasons, can better effectuate the purpose for
which the respondent's franchise was granted.

Granting for the purpose of argument that the steel supports or towers in question are not embraced
within the termpoles, the logical question posited is whether they constitute real properties, so that
they can be subject to a real property tax. The tax law does not provide for a definition of real
property; but Article 415 of the Civil Code does, by stating the following are immovable property:

(1) Land, buildings, roads, and constructions of all kinds adhered to the soil;

xxx xxx xxx

(3) Everything attached to an immovable in a fixed manner, in such a way that it cannot be
separated therefrom without breaking the material or deterioration of the object;

xxx xxx xxx

(5) Machinery, receptacles, instruments or implements intended by the owner of the tenement
for an industry or works which may be carried in a building or on a piece of land, and which
tends directly to meet the needs of the said industry or works;

xxx xxx xxx

The steel towers or supports in question, do not come within the objects mentioned in paragraph 1,
because they do not constitute buildings or constructions adhered to the soil. They are not
construction analogous to buildings nor adhering to the soil. As per description, given by the lower
court, they are removable and merely attached to a square metal frame by means of bolts, which
when unscrewed could easily be dismantled and moved from place to place. They can not be
included under paragraph 3, as they are not attached to an immovable in a fixed manner, and they
can be separated without breaking the material or causing deterioration upon the object to which they
are attached. Each of these steel towers or supports consists of steel bars or metal strips, joined
together by means of bolts, which can be disassembled by unscrewing the bolts and reassembled by
screwing the same. These steel towers or supports do not also fall under paragraph 5, for they are not
machineries, receptacles, instruments or implements, and even if they were, they are not intended for
industry or works on the land. Petitioner is not engaged in an industry or works in the land in which
the steel supports or towers are constructed.

It is finally contended that the CTA erred in ordering the City Treasurer of Quezon City to refund the
sum of P11,651.86, despite the fact that Quezon City is not a party to the case. It is argued that as the
City Treasurer is not the real party in interest, but Quezon City, which was not a party to the suit,
notwithstanding its capacity to sue and be sued, he should not be ordered to effect the refund. This
question has not been raised in the court below, and, therefore, it cannot be properly raised for the
first time on appeal. The herein petitioner is indulging in legal technicalities and niceties which do not
help him any; for factually, it was he (City Treasurer) whom had insisted that respondent herein pay
27

the real estate taxes, which respondent paid under protest. Having acted in his official capacity as
City Treasurer of Quezon City, he would surely know what to do, under the circumstances.

IN VIEW HEREOF, the decision appealed from is hereby affirmed, with costs against the petitioners.

Bengzon, C.J., Padilla, Bautista Angelo, Labrador, Concepcion, Reyes, J.B.L., Barrera and Regala,
JJ., concur.
Makalintal, J., concurs in the result.
Dizon, J., took no part.

MERALCO v. BD. OF ASSESSMENT APPEALS (G.R. No. L-46245, May 31, 1982)

Republic of the Philippines


SUPREME COURT
Manila

SECOND DIVISION

G.R. No. L-46245 May 31, 1982

MERALCO SECURITIES INDUSTRIAL CORPORATION, petitioner,


vs.
CENTRAL BOARD OF ASSESSMENT APPEALS, BOARD OF ASSESSMENT APPEALS OF
LAGUNA and PROVINCIAL ASSESSOR OF LAGUNA, respondents.

AQUINO, J.:

In this special civil action of certiorari, Meralco Securities Industrial Corporation assails the decision of
the Central Board of Assessment Appeals (composed of the Secretary of Finance as chairman and
the Secretaries of Justice and Local Government and Community Development as members) dated
May 6, 1976, holding that Meralco Securities' oil pipeline is subject to realty tax.

The record reveals that pursuant to a pipeline concession issued under the Petroleum Act of 1949,
Republic Act No. 387, Meralco Securities installed from Batangas to Manila a pipeline system
consisting of cylindrical steel pipes joined together and buried not less than one meter below the
surface along the shoulder of the public highway. The portion passing through Laguna is about thirty
kilometers long.

The pipes for white oil products measure fourteen inches in diameter by thirty-six feet with a
maximum capacity of 75,000 barrels daily. The pipes for fuel and black oil measure sixteen inches by
forty-eight feet with a maximum capacity of 100,000 barrels daily.

The pipes are embedded in the soil and are firmly and solidly welded together so as to preclude
breakage or damage thereto and prevent leakage or seepage of the oil. The valves are welded to the
pipes so as to make the pipeline system one single piece of property from end to end.

In order to repair, replace, remove or transfer segments of the pipeline, the pipes have to be cold-cut
by means of a rotary hard-metal pipe-cutter after digging or excavating them out of the ground where
28

they are buried. In points where the pipeline traversed rivers or creeks, the pipes were laid beneath
the bed thereof. Hence, the pipes are permanently attached to the land.

However, Meralco Securities notes that segments of the pipeline can be moved from one place to
another as shown in the permit issued by the Secretary of Public Works and Communications which
permit provides that the government reserves the right to require the removal or transfer of the pipes
by and at the concessionaire's expense should they be affected by any road repair or improvement.

Pursuant to the Assessment Law, Commonwealth Act No. 470, the provincial assessor of Laguna
treated the pipeline as real property and issued Tax Declarations Nos. 6535-6537, San Pedro; 7473-
7478, Cabuyao; 7967-7971, Sta. Rosa; 9882-9885, Biñan and 15806-15810, Calamba, containing the
assessed values of portions of the pipeline.

Meralco Securities appealed the assessments to the Board of Assessment Appeals of Laguna
composed of the register of deeds as chairman and the provincial auditor as member. That board in
its decision of June 18, 1975 upheld the assessments (pp. 47-49, Rollo).

Meralco Securities brought the case to the Central Board of Assessment Appeals. As already stated,
that Board, composed of Acting Secretary of Finance Pedro M. Almanzor as chairman and Secretary
of Justice Vicente Abad Santos and Secretary of Local Government and Community Development
Jose Roño as members, ruled that the pipeline is subject to realty tax (p. 40, Rollo).

A copy of that decision was served on Meralco Securities' counsel on August 27, 1976. Section 36 of
the Real Property Tax Code, Presidential Decree No. 464, which took effect on June 1, 1974,
provides that the Board's decision becomes final and executory after the lapse of fifteen days from the
date of receipt of a copy of the decision by the appellant.

Under Rule III of the amended rules of procedure of the Central Board of Assessment Appeals (70
O.G. 10085), a party may ask for the reconsideration of the Board's decision within fifteen days after
receipt. On September 7, 1976 (the eleventh day), Meralco Securities filed its motion for
reconsideration.

Secretary of Finance Cesar Virata and Secretary Roño (Secretary Abad Santos abstained) denied the
motion in a resolution dated December 2, 1976, a copy of which was received by appellant's counsel
on May 24, 1977 (p. 4, Rollo). On June 6, 1977, Meralco Securities filed the instant petition for
certiorari.

The Solicitor General contends that certiorari is not proper in this case because the Board acted
within its jurisdiction and did not gravely abuse its discretion and Meralco Securities was not denied
due process of law.

Meralco Securities explains that because the Court of Tax Appeals has no jurisdiction to review the
decision of the Central Board of Assessment Appeals and because no judicial review of the Board's
decision is provided for in the Real Property Tax Code, Meralco Securities' recourse is to file a
petition for certiorari.

We hold that certiorari was properly availed of in this case. It is a writ issued by a superior court to an
inferior court, board or officer exercising judicial or quasi-judicial functions whereby the record of a
particular case is ordered to be elevated for review and correction in matters of law (14 C.J.S. 121-
122; 14 Am Jur. 2nd 777).

The rule is that as to administrative agencies exercising quasi-judicial power there is an underlying
power in the courts to scrutinize the acts of such agencies on questions of law and jurisdiction even
though no right of review is given by the statute (73 C.J.S. 506, note 56).

"The purpose of judicial review is to keep the administrative agency within its jurisdiction and protect
substantial rights of parties affected by its decisions" (73 C.J.S. 507, See. 165). The review is a part of
29

the system of checks and balances which is a limitation on the separation of powers and which
forestalls arbitrary and unjust adjudications.

Judicial review of the decision of an official or administrative agency exercising quasi-judicial functions
is proper in cases of lack of jurisdiction, error of law, grave abuse of discretion, fraud or collusion or in
case the administrative decision is corrupt, arbitrary or capricious (Mafinco Trading Corporation vs.
Ople, L-37790, March 25, 1976, 70 SCRA 139, 158; San Miguel Corporation vs. Secretary of Labor,
L-39195, May 16, 1975, 64 SCRA 56, 60, Mun. Council of Lemery vs. Prov. Board of Batangas, 56
Phil. 260, 268).

The Central Board of Assessment Appeals, in confirming the ruling of the provincial assessor and the
provincial board of assessment appeals that Meralco Securities' pipeline is subject to realty tax,
reasoned out that the pipes are machinery or improvements, as contemplated in the Assessment Law
and the Real Property Tax Code; that they do not fall within the category of property exempt from
realty tax under those laws; that articles 415 and 416 of the Civil Code, defining real and personal
property, have no application to this case; that even under article 415, the steel pipes can be
regarded as realty because they are constructions adhered to the soil and things attached to the land
in a fixed manner and that Meralco Securities is not exempt from realty tax under the Petroleum Law
(pp. 36-40).

Meralco Securities insists that its pipeline is not subject to realty tax because it is not real property
within the meaning of article 415. This contention is not sustainable under the provisions of the
Assessment Law, the Real Property Tax Code and the Civil Code.

Section 2 of the Assessment Law provides that the realty tax is due "on real property, including land,
buildings, machinery, and other improvements" not specifically exempted in section 3 thereof. This
provision is reproduced with some modification in the Real Property Tax Code which provides:

SEC. 38. Incidence of Real Property Tax.— There shall be levied, assessed and
collected in all provinces, cities and municipalities an annual ad valorem tax on real
property, such as land, buildings, machinery and other improvements affixed or
attached to real property not hereinafter specifically exempted. *

It is incontestable that the pipeline of Meralco Securities does not fall within any of the classes of
exempt real property enumerated in section 3 of the Assessment Law and section 40 of the Real
Property Tax Code.

Pipeline means a line of pipe connected to pumps, valves and control devices for conveying liquids,
gases or finely divided solids. It is a line of pipe running upon or in the earth, carrying with it the right
to the use of the soil in which it is placed (Note 21[10],54 C.J.S. 561).

Article 415[l] and [3] provides that real property may consist of constructions of all kinds adhered to
the soil and everything attached to an immovable in a fixed manner, in such a way that it cannot be
separated therefrom without breaking the material or deterioration of the object.

The pipeline system in question is indubitably a construction adhering to the soil (Exh. B, p. 39, Rollo).
It is attached to the land in such a way that it cannot be separated therefrom without dismantling the
steel pipes which were welded to form the pipeline.

Insofar as the pipeline uses valves, pumps and control devices to maintain the flow of oil, it is in a
sense machinery within the meaning of the Real Property Tax Code.

It should be borne in mind that what are being characterized as real property are not the steel pipes
but the pipeline system as a whole. Meralco Securities has apparently two pipeline systems.

A pipeline for conveying petroleum has been regarded as real property for tax purposes (Miller
County Highway, etc., Dist. vs. Standard Pipe Line Co., 19 Fed. 2nd 3; Board of Directors of Red
30

River Levee Dist. No. 1 of Lafayette County, Ark vs. R. F. C., 170 Fed. 2nd 430; 50 C. J. 750, note
86).

The other contention of Meralco Securities is that the Petroleum Law exempts it from the payment of
realty taxes. The alleged exemption is predicated on the following provisions of that law which exempt
Meralco Securities from local taxes and make it liable for taxes of general application:

ART. 102. Work obligations, taxes, royalties not to be changed.— Work obligations,
special taxes and royalties which are fixed by the provisions of this Act or by the
concession for any of the kinds of concessions to which this Act relates, are
considered as inherent on such concessions after they are granted, and shall not be
increased or decreased during the life of the concession to which they apply; nor
shall any other special taxes or levies be applied to such concessions, nor shall
0concessionaires under this Act be subject to any provincial, municipal or other local
taxes or levies; nor shall any sales tax be charged on any petroleum produced from
the concession or portion thereof, manufactured by the concessionaire and used in
the working of his concession. All such concessionaires, however, shall be subject
to such taxes as are of general application in addition to taxes and other levies
specifically provided in this Act.

Meralco Securities argues that the realty tax is a local tax or levy and not a tax of general application.
This argument is untenable because the realty tax has always been imposed by the lawmaking body
and later by the President of the Philippines in the exercise of his lawmaking powers, as shown in
section 342 et seq. of the Revised Administrative Code, Act No. 3995, Commonwealth Act No. 470
and Presidential Decree No. 464.

The realty tax is enforced throughout the Philippines and not merely in a particular municipality or city
but the proceeds of the tax accrue to the province, city, municipality and barrio where the realty taxed
is situated (Sec. 86, P.D. No. 464). In contrast, a local tax is imposed by the municipal or city council
by virtue of the Local Tax Code, Presidential Decree No. 231, which took effect on July 1, 1973 (69
O.G. 6197).

We hold that the Central Board of Assessment Appeals did not act with grave abuse of discretion, did
not commit any error of law and acted within its jurisdiction in sustaining the holding of the provincial
assessor and the local board of assessment appeals that Meralco Securities' pipeline system in
Laguna is subject to realty tax.

WHEREFORE, the questioned decision and resolution are affirmed. The petition is dismissed. No
costs.

SO ORDERED.

Barredo (Chairman), Guerrero, De Castro and Escolin, JJ., concur.

Justice Abad Santos, Concepcion, Jr., JJ., took no part.

Footnotes

* The Real Property Tax Code contains the following definitions in its section 3:

"k) Improvements - is a valuable addition made to property or an amelioration in its


condition, amounting to more than mere repairs or replacement of waste, costing
labor or capital and intended to enhance its value, beauty or utility or to adapt it for
new or further purposes. "
31

"m) Machinery - shall embrace machines, mechanical contrivances, instruments,


appliances and apparatus attached to the real estate. It includes the physical facilities
available for production, as well as the installations and appurtenant service facilities,
together with all other equipment designed for or essential to its manufacturing,
industrial or agricultural purposes." (See sec. 3[f], Assessment Law).

MERALCO v. BD. OF ASSESSMENT APPEALS (G.R. No. L-47943, May 31, 1982)

Republic of the Philippines


SUPREME COURT
Manila

SECOND DIVISION

G.R. No. L-47943 May 31, 1982

MANILA ELECTRIC COMPANY, petitioner,


vs.
CENTRAL BOARD OF ASSESSMENT APPEALS, BOARD OF ASSESSMENT APPEALS OF
BATANGAS and PROVINCIAL ASSESSOR OF BATANGAS, respondents.

AQUINO, J.:

This case is about the imposition of the realty tax on two oil storage tanks installed in 1969 by Manila
Electric Company on a lot in San Pascual, Batangas which it leased in 1968 from Caltex (Phil.), Inc.
The tanks are within the Caltex refinery compound. They have a total capacity of 566,000 barrels.
They are used for storing fuel oil for Meralco's power plants.

According to Meralco, the storage tanks are made of steel plates welded and assembled on the spot.
Their bottoms rest on a foundation consisting of compacted earth as the outermost layer, a sand pad
as the intermediate layer and a two-inch thick bituminous asphalt stratum as the top layer. The bottom
of each tank is in contact with the asphalt layer,

The steel sides of the tank are directly supported underneath by a circular wall made of concrete,
eighteen inches thick, to prevent the tank from sliding. Hence, according to Meralco, the tank is not
attached to its foundation. It is not anchored or welded to the concrete circular wall. Its bottom plate is
not attached to any part of the foundation by bolts, screws or similar devices. The tank merely sits on
its foundation. Each empty tank can be floated by flooding its dike-inclosed location with water four
feet deep. (pp. 29-30, Rollo.)

On the other hand, according to the hearing commissioners of the Central Board of Assessment
Appeals, the area where the two tanks are located is enclosed with earthen dikes with electric steel
poles on top thereof and is divided into two parts as the site of each tank. The foundation of the tanks
is elevated from the remaining area. On both sides of the earthen dikes are two separate concrete
steps leading to the foundation of each tank.

Tank No. 2 is supported by a concrete foundation with an asphalt lining about an inch thick. Pipelines
were installed on the sides of each tank and are connected to the pipelines of the Manila Enterprises
Industrial Corporation whose buildings and pumping station are near Tank No. 2.
32

The Board concludes that while the tanks rest or sit on their foundation, the foundation itself and the
walls, dikes and steps, which are integral parts of the tanks, are affixed to the land while the pipelines
are attached to the tanks. (pp. 60-61, Rollo.) In 1970, the municipal treasurer of Bauan, Batangas, on
the basis of an assessment made by the provincial assessor, required Meralco to pay realty taxes on
the two tanks. For the five-year period from 1970 to 1974, the tax and penalties amounted to
P431,703.96 (p. 27, Rollo). The Board required Meralco to pay the tax and penalties as a condition for
entertaining its appeal from the adverse decision of the Batangas board of assessment appeals.

The Central Board of Assessment Appeals (composed of Acting Secretary of Finance Pedro M.
Almanzor as chairman and Secretary of Justice Vicente Abad Santos and Secretary of Local
Government and Community Development Jose Roño as members) in its decision dated November
5, 1976 ruled that the tanks together with the foundation, walls, dikes, steps, pipelines and other
appurtenances constitute taxable improvements.

Meralco received a copy of that decision on February 28, 1977. On the fifteenth day, it filed a motion
for reconsideration which the Board denied in its resolution of November 25, 1977, a copy of which
was received by Meralco on February 28, 1978.

On March 15, 1978, Meralco filed this special civil action of certiorari to annul the Board's decision
and resolution. It contends that the Board acted without jurisdiction and committed a grave error of
law in holding that its storage tanks are taxable real property.

Meralco contends that the said oil storage tanks do not fall within any of the kinds of real property
enumerated in article 415 of the Civil Code and, therefore, they cannot be categorized as realty by
nature, by incorporation, by destination nor by analogy. Stress is laid on the fact that the tanks are not
attached to the land and that they were placed on leased land, not on the land owned by Meralco.

This is one of those highly controversial, borderline or penumbral cases on the classification of
property where strong divergent opinions are inevitable. The issue raised by Meralco has to be
resolved in the light of the provisions of the Assessment Law, Commonwealth Act No. 470, and the
Real Property Tax Code, Presidential Decree No. 464 which took effect on June 1, 1974.

Section 2 of the Assessment Law provides that the realty tax is due "on real property, including land,
buildings, machinery, and other improvements" not specifically exempted in section 3 thereof. This
provision is reproduced with some modification in the Real Property Tax Code which provides:

Sec. 38. Incidence of Real Property Tax. — They shall be levied, assessed and
collected in all provinces, cities and municipalities an annual ad valorem tax on real
property, such as land, buildings, machinery and other improvements affixed or
attached to real property not hereinafter specifically exempted.

The Code contains the following definition in its section 3:

k) Improvements — is a valuable addition made to property or an amelioration in its


condition, amounting to more than mere repairs or replacement of waste, costing
labor or capital and intended to enhance its value, beauty or utility or to adapt it for
new or further purposes.

We hold that while the two storage tanks are not embedded in the land, they may, nevertheless, be
considered as improvements on the land, enhancing its utility and rendering it useful to the oil
industry. It is undeniable that the two tanks have been installed with some degree of permanence as
receptacles for the considerable quantities of oil needed by Meralco for its operations.

Oil storage tanks were held to be taxable realty in Standard Oil Co. of New Jersey vs. Atlantic City, 15
Atl. 2nd 271.

For purposes of taxation, the term "real property" may include things which should generally be
regarded as personal property(84 C.J.S. 171, Note 8). It is a familiar phenomenon to see things
33

classed as real property for purposes of taxation which on general principle might be considered
personal property (Standard Oil Co. of New York vs. Jaramillo, 44 Phil. 630, 633).

The case of Board of Assessment Appeals vs. Manila Electric Company, 119 Phil. 328, wherein
Meralco's steel towers were held not to be subject to realty tax, is not in point because in that case
the steel towers were regarded as poles and under its franchise Meralco's poles are exempt from
taxation. Moreover, the steel towers were not attached to any land or building. They were removable
from their metal frames.

Nor is there any parallelism between this case and Mindanao Bus Co. vs. City Assessor, 116 Phil.
501, where the tools and equipment in the repair, carpentry and blacksmith shops of a transportation
company were held not subject to realty tax because they were personal property.

WHEREFORE, the petition is dismissed. The Board's questioned decision and resolution are
affirmed. No costs.

SO ORDERED.

Barredo (Chairman), Guerrero, De Castro and Escolin, JJ., concur.

Concepcion, Jr., J., is on leave.

Justice Abad Santos, J., took no part.

CALTEX v. BD. OF ASSESSMENT APPEALS (G.R. No. L-50466, May 31, 1982)

Republic of the Philippines


SUPREME COURT
Manila

SECOND DIVISION

G.R. No. L-50466 May 31, 1982

CALTEX (PHILIPPINES) INC., petitioner,


vs.
CENTRAL BOARD OF ASSESSMENT APPEALS and CITY ASSESSOR OF PASAY, respondents.

AQUINO, J.:

This case is about the realty tax on machinery and equipment installed by Caltex (Philippines) Inc. in
its gas stations located on leased land.

The machines and equipment consists of underground tanks, elevated tank, elevated water tanks,
water tanks, gasoline pumps, computing pumps, water pumps, car washer, car hoists, truck hoists, air
compressors and tireflators. The city assessor described the said equipment and machinery in this
manner:

A gasoline service station is a piece of lot where a building or shed is erected, a


water tank if there is any is placed in one corner of the lot, car hoists are placed in an
34

adjacent shed, an air compressor is attached in the wall of the shed or at the
concrete wall fence.

The controversial underground tank, depository of gasoline or crude oil, is dug deep
about six feet more or less, a few meters away from the shed. This is done to prevent
conflagration because gasoline and other combustible oil are very inflammable.

This underground tank is connected with a steel pipe to the gasoline pump and the
gasoline pump is commonly placed or constructed under the shed. The footing of the
pump is a cement pad and this cement pad is imbedded in the pavement under the
shed, and evidence that the gasoline underground tank is attached and connected to
the shed or building through the pipe to the pump and the pump is attached and
affixed to the cement pad and pavement covered by the roof of the building or shed.

The building or shed, the elevated water tank, the car hoist under a separate shed,
the air compressor, the underground gasoline tank, neon lights signboard, concrete
fence and pavement and the lot where they are all placed or erected, all of them used
in the pursuance of the gasoline service station business formed the entire gasoline
service-station.

As to whether the subject properties are attached and affixed to the tenement, it is
clear they are, for the tenement we consider in this particular case are (is) the
pavement covering the entire lot which was constructed by the owner of the gasoline
station and the improvement which holds all the properties under question, they are
attached and affixed to the pavement and to the improvement.

The pavement covering the entire lot of the gasoline service station, as well as all the
improvements, machines, equipments and apparatus are allowed by Caltex
(Philippines) Inc. ...

The underground gasoline tank is attached to the shed by the steel pipe to the pump,
so with the water tank it is connected also by a steel pipe to the pavement, then to
the electric motor which electric motor is placed under the shed. So to say that the
gasoline pumps, water pumps and underground tanks are outside of the service
station, and to consider only the building as the service station is grossly erroneous.
(pp. 58-60, Rollo).

The said machines and equipment are loaned by Caltex to gas station operators under an appropriate
lease agreement or receipt. It is stipulated in the lease contract that the operators, upon demand,
shall return to Caltex the machines and equipment in good condition as when received, ordinary wear
and tear excepted.

The lessor of the land, where the gas station is located, does not become the owner of the machines
and equipment installed therein. Caltex retains the ownership thereof during the term of the lease.

The city assessor of Pasay City characterized the said items of gas station equipment and machinery
as taxable realty. The realty tax on said equipment amounts to P4,541.10 annually (p. 52, Rollo). The
city board of tax appeals ruled that they are personalty. The assessor appealed to the Central Board
of Assessment Appeals.

The Board, which was composed of Secretary of Finance Cesar Virata as chairman, Acting Secretary
of Justice Catalino Macaraig, Jr. and Secretary of Local Government and Community Development
Jose Roño, held in its decision of June 3, 1977 that the said machines and equipment are real
property within the meaning of sections 3(k) & (m) and 38 of the Real Property Tax Code, Presidential
Decree No. 464, which took effect on June 1, 1974, and that the definitions of real property and
personal property in articles 415 and 416 of the Civil Code are not applicable to this case.
35

The decision was reiterated by the Board (Minister Vicente Abad Santos took Macaraig's place) in its
resolution of January 12, 1978, denying Caltex's motion for reconsideration, a copy of which was
received by its lawyer on April 2, 1979.

On May 2, 1979 Caltex filed this certiorari petition wherein it prayed for the setting aside of the
Board's decision and for a declaration that t he said machines and equipment are personal property
not subject to realty tax (p. 16, Rollo).

The Solicitor General's contention that the Court of Tax Appeals has exclusive appellate jurisdiction
over this case is not correct. When Republic act No. 1125 created the Tax Court in 1954, there was
as yet no Central Board of Assessment Appeals. Section 7(3) of that law in providing that the Tax
Court had jurisdiction to review by appeal decisions of provincial or city boards of assessment
appeals had in mind the local boards of assessment appeals but not the Central Board of Assessment
Appeals which under the Real Property Tax Code has appellate jurisdiction over decisions of the said
local boards of assessment appeals and is, therefore, in the same category as the Tax Court.

Section 36 of the Real Property Tax Code provides that the decision of the Central Board of
Assessment Appeals shall become final and executory after the lapse of fifteen days from the receipt
of its decision by the appellant. Within that fifteen-day period, a petition for reconsideration may be
filed. The Code does not provide for the review of the Board's decision by this Court.

Consequently, the only remedy available for seeking a review by this Court of the decision of the
Central Board of Assessment Appeals is the special civil action of certiorari, the recourse resorted to
herein by Caltex (Philippines), Inc.

The issue is whether the pieces of gas station equipment and machinery already enumerated are
subject to realty tax. This issue has to be resolved primarily under the provisions of the Assessment
Law and the Real Property Tax Code.

Section 2 of the Assessment Law provides that the realty tax is due "on real property, including land,
buildings, machinery, and other improvements" not specifically exempted in section 3 thereof. This
provision is reproduced with some modification in the Real Property Tax Code which provides:

SEC. 38. Incidence of Real Property Tax.— There shall be levied, assessed and
collected in all provinces, cities and municipalities an annual ad valorem tax on real
property, such as land, buildings, machinery and other improvements affixed or
attached to real property not hereinafter specifically exempted.

The Code contains the following definitions in its section 3:

k) Improvements — is a valuable addition made to property or an amelioration in its


condition, amounting to more than mere repairs or replacement of waste, costing
labor or capital and intended to enhance its value, beauty or utility or to adapt it for
new or further purposes.

m) Machinery — shall embrace machines, mechanical contrivances, instruments,


appliances and apparatus attached to the real estate. It includes the physical facilities
available for production, as well as the installations and appurtenant service facilities,
together with all other equipment designed for or essential to its manufacturing,
industrial or agricultural purposes (See sec. 3[f], Assessment Law).

We hold that the said equipment and machinery, as appurtenances to the gas station building or shed
owned by Caltex (as to which it is subject to realty tax) and which fixtures are necessary to the
operation of the gas station, for without them the gas station would be useless, and which have been
attached or affixed permanently to the gas station site or embedded therein, are taxable
improvements and machinery within the meaning of the Assessment Law and the Real Property Tax
Code.
36

Caltex invokes the rule that machinery which is movable in its nature only becomes immobilized when
placed in a plant by the owner of the property or plant but not when so placed by a tenant, a
usufructuary, or any person having only a temporary right, unless such person acted as the agent of
the owner (Davao Saw Mill Co. vs. Castillo, 61 Phil 709).

That ruling is an interpretation of paragraph 5 of article 415 of the Civil Code regarding machinery that
becomes real property by destination. In the Davao Saw Mills case the question was whether the
machinery mounted on foundations of cement and installed by the lessee on leased land should be
regarded as real property for purposes of execution of a judgment against the lessee. The sheriff
treated the machinery as personal property. This Court sustained the sheriff's action. (Compare with
Machinery & Engineering Supplies, Inc. vs. Court of Appeals, 96 Phil. 70, where in a replevin case
machinery was treated as realty).

Here, the question is whether the gas station equipment and machinery permanently affixed by Caltex
to its gas station and pavement (which are indubitably taxable realty) should be subject to the realty
tax. This question is different from the issue raised in the Davao Saw Mill case.

Improvements on land are commonly taxed as realty even though for some purposes they might be
considered personalty (84 C.J.S. 181-2, Notes 40 and 41). "It is a familiar phenomenon to see things
classed as real property for purposes of taxation which on general principle might be considered
personal property" (Standard Oil Co. of New York vs. Jaramillo, 44 Phil. 630, 633).

This case is also easily distinguishable from Board of Assessment Appeals vs. Manila Electric Co.,
119 Phil. 328, where Meralco's steel towers were considered poles within the meaning of paragraph 9
of its franchise which exempts its poles from taxation. The steel towers were considered personalty
because they were attached to square metal frames by means of bolts and could be moved from
place to place when unscrewed and dismantled.

Nor are Caltex's gas station equipment and machinery the same as tools and equipment in the repair
shop of a bus company which were held to be personal property not subject to realty tax (Mindanao
Bus Co. vs. City Assessor, 116 Phil. 501).

The Central Board of Assessment Appeals did not commit a grave abuse of discretion in upholding
the city assessor's is imposition of the realty tax on Caltex's gas station and equipment.

WHEREFORE, the questioned decision and resolution of the Central Board of Assessment Appeals
are affirmed. The petition for certiorari is dismissed for lack of merit. No costs.

SO ORDERED.

Barredo (Chairman), Guerrero, De Castro and Escolin, JJ., concur.

Concepcion, Jr. and Abad Santos, JJ., took no part.

BENGUET CORP. v. BD. OF ASSESSMENT APPEALS (G.R. No. 106041, January 29, 1993)

Republic of the Philippines


SUPREME COURT
Manila

EN BANC
37

G.R. No. 106041 January 29, 1993

BENGUET CORPORATION, petitioner,


vs.
CENTRAL BOARD OF ASSESSMENT APPEALS, BOARD OF ASSESSMENT APPEALS OF
ZAMBALES, PROVINCIAL ASSESSOR OF ZAMBALES, PROVINCE OF ZAMBALES, and
MUNICIPALITY OF SAN MARCELINO, respondents.

Romulo, Mabanta, Buenaventura, Sayoc & De los Angeles for petitioner.

CRUZ, J.:

The realty tax assessment involved in this case amounts to P11,319,304.00. It has been imposed on
the petitioner's tailings dam and the land thereunder over its protest.

The controversy arose in 1985 when the Provincial Assessor of Zambales assessed the said
properties as taxable improvements. The assessment was appealed to the Board of Assessment
Appeals of the Province of Zambales. On August 24, 1988, the appeal was dismissed mainly on the
ground of the petitioner's "failure to pay the realty taxes that fell due during the pendency of the
appeal."

The petitioner seasonably elevated the matter to the Central Board of Assessment Appeals,1 one of
the herein respondents. In its decision dated March 22, 1990, the Board reversed the dismissal of the
appeal but, on the merits, agreed that "the tailings dam and the lands submerged thereunder (were)
subject to realty tax."

For purposes of taxation the dam is considered as real property as it comes within
the object mentioned in paragraphs (a) and (b) of Article 415 of the New Civil Code. It
is a construction adhered to the soil which cannot be separated or detached without
breaking the material or causing destruction on the land upon which it is attached.
The immovable nature of the dam as an improvement determines its character as
real property, hence taxable under Section 38 of the Real Property Tax Code. (P.D.
464).

Although the dam is partly used as an anti-pollution device, this Board cannot accede
to the request for tax exemption in the absence of a law authorizing the same.

xxx xxx xxx

We find the appraisal on the land submerged as a result of the construction of the
tailings dam, covered by Tax Declaration Nos.
002-0260 and 002-0266, to be in accordance with the Schedule of Market Values for
Zambales which was reviewed and allowed for use by the Ministry (Department) of
Finance in the 1981-1982 general revision. No serious attempt was made by
Petitioner-Appellant Benguet Corporation to impugn its reasonableness, i.e., that the
P50.00 per square meter applied by Respondent-Appellee Provincial Assessor is
indeed excessive and unconscionable. Hence, we find no cause to disturb the market
value applied by Respondent Appellee Provincial Assessor of Zambales on the
properties of Petitioner-Appellant Benguet Corporation covered by Tax Declaration
Nos. 002-0260 and 002-0266.

This petition for certiorari now seeks to reverse the above ruling.
38

The principal contention of the petitioner is that the tailings dam is not subject to realty tax because it
is not an "improvement" upon the land within the meaning of the Real Property Tax Code. More
particularly, it is claimed —

(1) as regards the tailings dam as an "improvement":

(a) that the tailings dam has no value separate from and independent
of the mine; hence, by itself it cannot be considered an improvement
separately assessable;

(b) that it is an integral part of the mine;

(c) that at the end of the mining operation of the petitioner


corporation in the area, the tailings dam will benefit the local
community by serving as an irrigation facility;

(d) that the building of the dam has stripped the property of any
commercial value as the property is submerged under water wastes
from the mine;

(e) that the tailings dam is an environmental pollution control device


for which petitioner must be commended rather than penalized with a
realty tax assessment;

(f) that the installation and utilization of the tailings dam as a pollution
control device is a requirement imposed by law;

(2) as regards the valuation of the tailings dam and the submerged lands:

(a) that the subject properties have no market value as they cannot
be sold independently of the mine;

(b) that the valuation of the tailings dam should be based on its
incidental use by petitioner as a water reservoir and not on the
alleged cost of construction of the dam and the annual build-up
expense;

(c) that the "residual value formula" used by the Provincial Assessor
and adopted by respondent CBAA is arbitrary and erroneous; and

(3) as regards the petitioner's liability for penalties for


non-declaration of the tailings dam and the submerged lands for realty tax purposes:

(a) that where a tax is not paid in an honest belief that it is not due,
no penalty shall be collected in addition to the basic tax;

(b) that no other mining companies in the Philippines operating a


tailings dam have been made to declare the dam for realty tax
purposes.

The petitioner does not dispute that the tailings dam may be considered realty within the meaning of
Article 415. It insists, however, that the dam cannot be subjected to realty tax as a separate and
independent property because it does not constitute an "assessable improvement" on the mine
although a considerable sum may have been spent in constructing and maintaining it.

To support its theory, the petitioner cites the following cases:


39

1. Municipality of Cotabato v. Santos (105 Phil. 963), where this Court considered the dikes and gates
constructed by the taxpayer in connection with a fishpond operation as integral parts of the fishpond.

2. Bislig Bay Lumber Co. v. Provincial Government of Surigao (100 Phil. 303), involving a road
constructed by the timber concessionaire in the area, where this Court did not impose a realty tax on
the road primarily for two reasons:

In the first place, it cannot be disputed that the ownership of the road that was
constructed by appellee belongs to the government by right of accession not only
because it is inherently incorporated or attached to the timber land . . . but also
because upon the expiration of the concession said road would ultimately pass to the
national government. . . . In the second place, while the road was constructed by
appellee primarily for its use and benefit, the privilege is not exclusive, for . . .
appellee cannot prevent the use of portions of the concession for homesteading
purposes. It is also duty bound to allow the free use of forest products within the
concession for the personal use of individuals residing in or within the vicinity of the
land. . . . In other words, the government has practically reserved the rights to use the
road to promote its varied activities. Since, as above shown, the road in question
cannot be considered as an improvement which belongs to appellee, although in part
is for its benefit, it is clear that the same cannot be the subject of assessment within
the meaning of Section 2 of C.A.
No. 470.

Apparently, the realty tax was not imposed not because the road was an integral part of the lumber
concession but because the government had the right to use the road to promote its varied activities.

3. Kendrick v. Twin Lakes Reservoir Co. (144 Pacific 884), an American case, where it was declared
that the reservoir dam went with and formed part of the reservoir and that the dam would be
"worthless and useless except in connection with the outlet canal, and the water rights in the reservoir
represent and include whatever utility or value there is in the dam and headgates."

4. Ontario Silver Mining Co. v. Hixon (164 Pacific 498), also from the United States. This case
involved drain tunnels constructed by plaintiff when it expanded its mining operations downward,
resulting in a constantly increasing flow of water in the said mine. It was held that:

Whatever value they have is connected with and in fact is an integral part of the mine
itself. Just as much so as any shaft which descends into the earth or an underground
incline, tunnel, or drift would be which was used in connection with the mine.

On the other hand, the Solicitor General argues that the dam is an assessable improvement because
it enhances the value and utility of the mine. The primary function of the dam is to receive, retain and
hold the water coming from the operations of the mine, and it also enables the petitioner to impound
water, which is then recycled for use in the plant.

There is also ample jurisprudence to support this view, thus:

. . . The said equipment and machinery, as appurtenances to the gas station building
or shed owned by Caltex (as to which it is subject to realty tax) and which fixtures are
necessary to the operation of the gas station, for without them the gas station would
be useless and which have been attached or affixed permanently to the gas station
site or embedded therein, are taxable improvements and machinery within the
meaning of the Assessment Law and the Real Property Tax Code. (Caltex [Phil.] Inc.
v. CBAA, 114 SCRA 296).

We hold that while the two storage tanks are not embedded in the land, they may,
nevertheless, be considered as improvements on the land, enhancing its utility and
rendering it useful to the oil industry. It is undeniable that the two tanks have been
installed with some degree of permanence as receptacles for the considerable
40

quantities of oil needed by MERALCO for its operations. (Manila Electric Co. v.
CBAA, 114 SCRA 273).

The pipeline system in question is indubitably a construction adhering to the soil. It is


attached to the land in such a way that it cannot be separated therefrom without
dismantling the steel pipes which were welded to form the pipeline. (MERALCO
Securities Industrial Corp. v. CBAA, 114 SCRA 261).

The tax upon the dam was properly assessed to the plaintiff as a tax upon real
estate. (Flax-Pond Water Co. v. City of Lynn, 16 N.E. 742).

The oil tanks are structures within the statute, that they are designed and used by the
owner as permanent improvement of the free hold, and that for such reasons they
were properly assessed by the respondent taxing district as improvements. (Standard
Oil Co. of New Jersey v. Atlantic City, 15 A 2d. 271)

The Real Property Tax Code does not carry a definition of "real property" and simply says that the
realty tax is imposed on "real property, such as lands, buildings, machinery and other improvements
affixed or attached to real property." In the absence of such a definition, we apply Article 415 of the
Civil Code, the pertinent portions of which state:

Art. 415. The following are immovable property.

(1) Lands, buildings and constructions of all kinds adhered to the soil;

xxx xxx xxx

(3) Everything attached to an immovable in a fixed manner, in such a way that it


cannot be separated therefrom without breaking the material or deterioration of the
object.

Section 2 of C.A. No. 470, otherwise known as the Assessment Law, provides that the realty tax is
due "on the real property, including land, buildings, machinery and other improvements" not
specifically exempted in Section 3 thereof. A reading of that section shows that the tailings dam of the
petitioner does not fall under any of the classes of exempt real properties therein enumerated.

Is the tailings dam an improvement on the mine? Section 3(k) of the Real Property Tax Code defines
improvement as follows:

(k) Improvements — is a valuable addition made to property or an amelioration in its


condition, amounting to more than mere repairs or replacement of waste, costing
labor or capital and intended to enhance its value, beauty or utility or to adopt it for
new or further purposes.

The term has also been interpreted as "artificial alterations of the physical condition of the ground that
are reasonably permanent in character."2

The Court notes that in the Ontario case the plaintiff admitted that the mine involved therein could not
be operated without the aid of the drain tunnels, which were indispensable to the successful
development and extraction of the minerals therein. This is not true in the present case.

Even without the tailings dam, the petitioner's mining operation can still be carried out because the
primary function of the dam is merely to receive and retain the wastes and water coming from the
mine. There is no allegation that the water coming from the dam is the sole source of water for the
mining operation so as to make the dam an integral part of the mine. In fact, as a result of the
construction of the dam, the petitioner can now impound and recycle water without having to spend
for the building of a water reservoir. And as the petitioner itself points out, even if the petitioner's mine
41

is shut down or ceases operation, the dam may still be used for irrigation of the surrounding areas,
again unlike in the Ontario case.

As correctly observed by the CBAA, the Kendrick case is also not applicable because it involved
water reservoir dams used for different purposes and for the benefit of the surrounding areas. By
contrast, the tailings dam in question is being used exclusively for the benefit of the petitioner.

Curiously, the petitioner, while vigorously arguing that the tailings dam has no separate existence, just
as vigorously contends that at the end of the mining operation the tailings dam will serve the local
community as an irrigation facility, thereby implying that it can exist independently of the mine.

From the definitions and the cases cited above, it would appear that whether a structure constitutes
an improvement so as to partake of the status of realty would depend upon the degree
of permanence intended in its construction and use. The expression "permanent" as applied to an
improvement does not imply that the improvement must be used perpetually but only until the purpose
to which the principal realty is devoted has been accomplished. It is sufficient that the improvement is
intended to remain as long as the land to which it is annexed is still used for the said purpose.

The Court is convinced that the subject dam falls within the definition of an "improvement" because it
is permanent in character and it enhances both the value and utility of petitioner's mine. Moreover, the
immovable nature of the dam defines its character as real property under Article 415 of the Civil Code
and thus makes it taxable under Section 38 of the Real Property Tax Code.

The Court will also reject the contention that the appraisal at P50.00 per square meter made by the
Provincial Assessor is excessive and that his use of the "residual value formula" is arbitrary and
erroneous.

Respondent Provincial Assessor explained the use of the "residual value formula" as follows:

A 50% residual value is applied in the computation because, while it is true that when
slime fills the dike, it will then be covered by another dike or stage, the stage covered
is still there and still exists and since only one face of the dike is filled, 50% or the
other face is unutilized.

In sustaining this formula, the CBAA gave the following justification:

We find the appraisal on the land submerged as a result of the construction of the
tailings dam, covered by Tax Declaration Nos.
002-0260 and 002-0266, to be in accordance with the Schedule of Market Values for
San Marcelino, Zambales, which is fifty (50.00) pesos per square meter for third class
industrial land (TSN, page 17, July 5, 1989) and Schedule of Market Values for
Zambales which was reviewed and allowed for use by the Ministry (Department) of
Finance in the 1981-1982 general revision. No serious attempt was made by
Petitioner-Appellant Benguet Corporation to impugn its reasonableness, i.e, that the
P50.00 per square meter applied by Respondent-Appellee Provincial Assessor is
indeed excessive and unconscionable. Hence, we find no cause to disturb the market
value applied by Respondent-Appellee Provincial Assessor of Zambales on the
properties of Petitioner-Appellant Benguet Corporation covered by Tax Declaration
Nos. 002-0260 and 002-0266.

It has been the long-standing policy of this Court to respect the conclusions of quasi-judicial agencies
like the CBAA, which, because of the nature of its functions and its frequent exercise thereof, has
developed expertise in the resolution of assessment problems. The only exception to this rule is
where it is clearly shown that the administrative body has committed grave abuse of discretion calling
for the intervention of this Court in the exercise of its own powers of review. There is no such showing
in the case at bar.
42

We disagree, however, with the ruling of respondent CBAA that it cannot take cognizance of the issue
of the propriety of the penalties imposed upon it, which was raised by the petitioner for the first time
only on appeal. The CBAA held that this "is an entirely new matter that petitioner can take up with the
Provincial Assessor (and) can be the subject of another protest before the Local Board or a
negotiation with the local sanggunian . . ., and in case of an adverse decision by either the Local
Board or the local sanggunian, (it can) elevate the same to this Board for appropriate action."

There is no need for this time-wasting procedure. The Court may resolve the issue in this petition
instead of referring it back to the local authorities. We have studied the facts and circumstances of
this case as above discussed and find that the petitioner has acted in good faith in questioning the
assessment on the tailings dam and the land submerged thereunder. It is clear that it has not done so
for the purpose of evading or delaying the payment of the questioned tax. Hence, we hold that the
petitioner is not subject to penalty for its
non-declaration of the tailings dam and the submerged lands for realty tax purposes.

WHEREFORE, the petition is DISMISSED for failure to show that the questioned decision of
respondent Central Board of Assessment Appeals is tainted with grave abuse of discretion except as
to the imposition of penalties upon the petitioner which is hereby SET ASIDE. Costs against the
petitioner. It is so ordered.

Narvasa, C.J., Gutierrez, Jr., Padilla, Bidin, Griño-Aquino, Regalado, Davide, Jr., Romero, Nocon,
Bellosillo, Melo and Campos, Jr., JJ., concur.

Feliciano, J., took no part.

# Footnotes

1 Secretary of Finance Jesus Estanislao as chairman with Secretary of Justice


Franklin M. Drilon and Secretary of Local Government Luis T. Santos as members.

2 Francisco, Philippine Mining Law, Vol. 1, 2nd Ed., p. 274.

(OWNERSHIP)

LA BUGAL B’LAAN TRIBAL ASSN. v RAMOS (G.R. No. 127882, January 27, 2004)

EN BANC

G.R. No. 127882 January 27, 2004

LA BUGAL-B'LAAN TRIBAL ASSOCIATION, INC., represented by its Chairman F'LONG


MIGUEL M. LUMAYONG, WIGBERTO E. TAÑADA, PONCIANO BENNAGEN, JAIME TADEO,
RENATO R. CONSTANTINO, JR., F'LONG AGUSTIN M. DABIE, ROBERTO P. AMLOY, RAQIM L.
DABIE, SIMEON H. DOLOJO, IMELDA M. GANDON, LENY B. GUSANAN, MARCELO L.
GUSANAN, QUINTOL A. LABUAYAN, LOMINGGES D. LAWAY, BENITA P. TACUAYAN, minors
JOLY L. BUGOY, represented by his father UNDERO D. BUGOY, ROGER M. DADING,
represented by his father ANTONIO L. DADING, ROMY M. LAGARO, represented by his father
TOTING A. LAGARO, MIKENY JONG B. LUMAYONG, represented by his father MIGUEL M.
LUMAYONG, RENE T. MIGUEL, represented by his mother EDITHA T. MIGUEL, ALDEMAR L.
43

SAL, represented by his father DANNY M. SAL, DAISY RECARSE, represented by her mother
LYDIA S. SANTOS, EDWARD M. EMUY, ALAN P. MAMPARAIR, MARIO L. MANGCAL, ALDEN S.
TUSAN, AMPARO S. YAP, VIRGILIO CULAR, MARVIC M.V.F. LEONEN, JULIA REGINA CULAR,
GIAN CARLO CULAR, VIRGILIO CULAR, JR., represented by their father VIRGILIO CULAR,
PAUL ANTONIO P. VILLAMOR, represented by his parents JOSE VILLAMOR and ELIZABETH
PUA-VILLAMOR, ANA GININA R. TALJA, represented by her father MARIO JOSE B. TALJA,
SHARMAINE R. CUNANAN, represented by her father ALFREDO M. CUNANAN, ANTONIO
JOSE A. VITUG III, represented by his mother ANNALIZA A. VITUG, LEAN D. NARVADEZ,
represented by his father MANUEL E. NARVADEZ, JR., ROSERIO MARALAG LINGATING,
represented by her father RIO OLIMPIO A. LINGATING, MARIO JOSE B. TALJA, DAVID E. DE
VERA, MARIA MILAGROS L. SAN JOSE, SR., SUSAN O. BOLANIO, OND, LOLITA G.
DEMONTEVERDE, BENJIE L. NEQUINTO,1 ROSE LILIA S. ROMANO, ROBERTO S. VERZOLA,
EDUARDO AURELIO C. REYES, LEAN LOUEL A. PERIA, represented by his father ELPIDIO V.
PERIA,2 GREEN FORUM PHILIPPINES, GREEN FORUM WESTERN VISAYAS, (GF-WV),
ENVIRONMETAL LEGAL ASSISTANCE CENTER (ELAC), PHILIPPINE KAISAHAN TUNGO SA
KAUNLARAN NG KANAYUNAN AT REPORMANG PANSAKAHAN (KAISAHAN), 3 KAISAHAN
TUNGO SA KAUNLARAN NG KANAYUNAN AT REPORMANG PANSAKAHAN (KAISAHAN),
PARTNERSHIP FOR AGRARIAN REFORM and RURAL DEVELOPMENT SERVICES, INC.
(PARRDS), PHILIPPINE PART`NERSHIP FOR THE DEVELOPMENT OF HUMAN RESOURCES IN
THE RURAL AREAS, INC. (PHILDHRRA), WOMEN'S LEGAL BUREAU (WLB), CENTER FOR
ALTERNATIVE DEVELOPMENT INITIATIVES, INC. (CADI), UPLAND DEVELOPMENT INSTITUTE
(UDI), KINAIYAHAN FOUNDATION, INC., SENTRO NG ALTERNATIBONG LINGAP PANLIGAL
(SALIGAN), LEGAL RIGHTS AND NATURAL RESOURCES CENTER, INC. (LRC), petitioners,
vs.
VICTOR O. RAMOS, SECRETARY, DEPARTMENT OF ENVIRONMENT AND NATURAL
RESOURCES (DENR), HORACIO RAMOS, DIRECTOR, MINES AND GEOSCIENCES BUREAU
(MGB-DENR), RUBEN TORRES, EXECUTIVE SECRETARY, and WMC (PHILIPPINES),
INC.4 respondents.

DECISION

CARPIO-MORALES, J.:

The present petition for mandamus and prohibition assails the constitutionality of Republic Act No.
7942,5 otherwise known as the PHILIPPINE MINING ACT OF 1995, along with the Implementing
Rules and Regulations issued pursuant thereto, Department of Environment and Natural Resources
(DENR) Administrative Order 96-40, and of the Financial and Technical Assistance Agreement
(FTAA) entered into on March 30, 1995 by the Republic of the Philippines and WMC (Philippines),
Inc. (WMCP), a corporation organized under Philippine laws.

On July 25, 1987, then President Corazon C. Aquino issued Executive Order (E.O.) No.
2796 authorizing the DENR Secretary to accept, consider and evaluate proposals from foreign-owned
corporations or foreign investors for contracts or agreements involving either technical or financial
assistance for large-scale exploration, development, and utilization of minerals, which, upon
appropriate recommendation of the Secretary, the President may execute with the foreign proponent.
In entering into such proposals, the President shall consider the real contributions to the economic
growth and general welfare of the country that will be realized, as well as the development and use of
local scientific and technical resources that will be promoted by the proposed contract or agreement.
Until Congress shall determine otherwise, large-scale mining, for purpose of this Section, shall mean
those proposals for contracts or agreements for mineral resources exploration, development, and
utilization involving a committed capital investment in a single mining unit project of at least Fifty
Million Dollars in United States Currency (US $50,000,000.00).7

On March 3, 1995, then President Fidel V. Ramos approved R.A. No. 7942 to "govern the exploration,
development, utilization and processing of all mineral resources."8 R.A. No. 7942 defines the modes
of mineral agreements for mining operations,9 outlines the procedure for their filing and
approval,10 assignment/transfer11 and withdrawal,12and fixes their terms.13 Similar provisions govern
financial or technical assistance agreements.14
44

The law prescribes the qualifications of contractors 15 and grants them certain rights, including
timber,16 water17 and easement18 rights, and the right to possess explosives.19 Surface owners,
occupants, or concessionaires are forbidden from preventing holders of mining rights from entering
private lands and concession areas.20 A procedure for the settlement of conflicts is likewise provided
for.21

The Act restricts the conditions for exploration,22 quarry23 and other24 permits. It regulates the
transport, sale and processing of minerals,25 and promotes the development of mining communities,
science and mining technology,26and safety and environmental protection.27

The government's share in the agreements is spelled out and allocated,28 taxes and fees are
imposed,29 incentives granted.30 Aside from penalizing certain acts,31 the law likewise specifies
grounds for the cancellation, revocation and termination of agreements and permits. 32

On April 9, 1995, 30 days following its publication on March 10, 1995 in Malaya and Manila Times,
two newspapers of general circulation, R.A. No. 7942 took effect.33 Shortly before the effectivity of
R.A. No. 7942, however, or on March 30, 1995, the President entered into an FTAA with WMCP
covering 99,387 hectares of land in South Cotabato, Sultan Kudarat, Davao del Sur and North
Cotabato.34

On August 15, 1995, then DENR Secretary Victor O. Ramos issued DENR Administrative Order
(DAO) No. 95-23, s. 1995, otherwise known as the Implementing Rules and Regulations of R.A. No.
7942. This was later repealed by DAO No. 96-40, s. 1996 which was adopted on December 20, 1996.

On January 10, 1997, counsels for petitioners sent a letter to the DENR Secretary demanding that the
DENR stop the implementation of R.A. No. 7942 and DAO No. 96-40,35 giving the DENR fifteen days
from receipt36 to act thereon. The DENR, however, has yet to respond or act on petitioners' letter.37

Petitioners thus filed the present petition for prohibition and mandamus, with a prayer for a temporary
restraining order. They allege that at the time of the filing of the petition, 100 FTAA applications had
already been filed, covering an area of 8.4 million hectares,38 64 of which applications are by fully
foreign-owned corporations covering a total of 5.8 million hectares, and at least one by a fully foreign-
owned mining company over offshore areas.39

Petitioners claim that the DENR Secretary acted without or in excess of jurisdiction:

x x x in signing and promulgating DENR Administrative Order No. 96-40 implementing Republic Act
No. 7942, the latter being unconstitutional in that it allows fully foreign owned corporations to explore,
develop, utilize and exploit mineral resources in a manner contrary to Section 2, paragraph 4, Article
XII of the Constitution;

II

x x x in signing and promulgating DENR Administrative Order No. 96-40 implementing Republic Act
No. 7942, the latter being unconstitutional in that it allows the taking of private property without the
determination of public use and for just compensation;

III

x x x in signing and promulgating DENR Administrative Order No. 96-40 implementing Republic Act
No. 7942, the latter being unconstitutional in that it violates Sec. 1, Art. III of the Constitution;

IV
45

x x x in signing and promulgating DENR Administrative Order No. 96-40 implementing Republic Act
No. 7942, the latter being unconstitutional in that it allows enjoyment by foreign citizens as well as
fully foreign owned corporations of the nation's marine wealth contrary to Section 2, paragraph 2 of
Article XII of the Constitution;

x x x in signing and promulgating DENR Administrative Order No. 96-40 implementing Republic Act
No. 7942, the latter being unconstitutional in that it allows priority to foreign and fully foreign owned
corporations in the exploration, development and utilization of mineral resources contrary to Article XII
of the Constitution;

VI

x x x in signing and promulgating DENR Administrative Order No. 96-40 implementing Republic Act
No. 7942, the latter being unconstitutional in that it allows the inequitable sharing of wealth contrary to
Sections [sic] 1, paragraph 1, and Section 2, paragraph 4[,] [Article XII] of the Constitution;

VII

x x x in recommending approval of and implementing the Financial and Technical Assistance


Agreement between the President of the Republic of the Philippines and Western Mining Corporation
Philippines Inc. because the same is illegal and unconstitutional. 40

They pray that the Court issue an order:

(a) Permanently enjoining respondents from acting on any application for Financial or
Technical Assistance Agreements;

(b) Declaring the Philippine Mining Act of 1995 or Republic Act No. 7942 as unconstitutional
and null and void;

(c) Declaring the Implementing Rules and Regulations of the Philippine Mining Act contained
in DENR Administrative Order No. 96-40 and all other similar administrative issuances as
unconstitutional and null and void; and

(d) Cancelling the Financial and Technical Assistance Agreement issued to Western Mining
Philippines, Inc. as unconstitutional, illegal and null and void.41

Impleaded as public respondents are Ruben Torres, the then Executive Secretary, Victor O. Ramos,
the then DENR Secretary, and Horacio Ramos, Director of the Mines and Geosciences Bureau of the
DENR. Also impleaded is private respondent WMCP, which entered into the assailed FTAA with the
Philippine Government. WMCP is owned by WMC Resources International Pty., Ltd. (WMC), "a
wholly owned subsidiary of Western Mining Corporation Holdings Limited, a publicly listed major
Australian mining and exploration company."42 By WMCP's information, "it is a 100% owned
subsidiary of WMC LIMITED."43

Respondents, aside from meeting petitioners' contentions, argue that the requisites for judicial inquiry
have not been met and that the petition does not comply with the criteria for prohibition and
mandamus. Additionally, respondent WMCP argues that there has been a violation of the rule on
hierarchy of courts.

After petitioners filed their reply, this Court granted due course to the petition. The parties have since
filed their respective memoranda.

WMCP subsequently filed a Manifestation dated September 25, 2002 alleging that on January 23,
2001, WMC sold all its shares in WMCP to Sagittarius Mines, Inc. (Sagittarius), a corporation
46

organized under Philippine laws.44WMCP was subsequently renamed "Tampakan Mineral Resources
Corporation."45 WMCP claims that at least 60% of the equity of Sagittarius is owned by Filipinos
and/or Filipino-owned corporations while about 40% is owned by Indophil Resources NL, an
Australian company.46 It further claims that by such sale and transfer of shares, "WMCP has ceased
to be connected in any way with WMC."47

By virtue of such sale and transfer, the DENR Secretary, by Order of December 18, 2001, 48 approved
the transfer and registration of the subject FTAA from WMCP to Sagittarius. Said Order, however,
was appealed by Lepanto Consolidated Mining Co. (Lepanto) to the Office of the President which
upheld it by Decision of July 23, 2002. 49 Its motion for reconsideration having been denied by the
Office of the President by Resolution of November 12, 2002,50 Lepanto filed a petition for
review51 before the Court of Appeals. Incidentally, two other petitions for review related to the
approval of the transfer and registration of the FTAA to Sagittarius were recently resolved by this
Court.52

It bears stressing that this case has not been rendered moot either by the transfer and registration of
the FTAA to a Filipino-owned corporation or by the non-issuance of a temporary restraining order or a
preliminary injunction to stay the above-said July 23, 2002 decision of the Office of the
President.53 The validity of the transfer remains in dispute and awaits final judicial determination. This
assumes, of course, that such transfer cures the FTAA's alleged unconstitutionality, on which
question judgment is reserved.

WMCP also points out that the original claimowners of the major mineralized areas included in the
WMCP FTAA, namely, Sagittarius, Tampakan Mining Corporation, and Southcot Mining Corporation,
are all Filipino-owned corporations,54 each of which was a holder of an approved Mineral Production
Sharing Agreement awarded in 1994, albeit their respective mineral claims were subsumed in the
WMCP FTAA;55 and that these three companies are the same companies that consolidated their
interests in Sagittarius to whom WMC sold its 100% equity in WMCP.56 WMCP concludes that in the
event that the FTAA is invalidated, the MPSAs of the three corporations would be revived and the
mineral claims would revert to their original claimants.57

These circumstances, while informative, are hardly significant in the resolution of this case, it involving
the validity of the FTAA, not the possible consequences of its invalidation.

Of the above-enumerated seven grounds cited by petitioners, as will be shown later, only the first and
the last need be delved into; in the latter, the discussion shall dwell only insofar as it questions the
effectivity of E. O. No. 279 by virtue of which order the questioned FTAA was forged.

Before going into the substantive issues, the procedural questions posed by respondents shall first be
tackled.

REQUISITES FOR JUDICIAL REVIEW

When an issue of constitutionality is raised, this Court can exercise its power of judicial review only if
the following requisites are present:

(1) The existence of an actual and appropriate case;

(2) A personal and substantial interest of the party raising the constitutional question;

(3) The exercise of judicial review is pleaded at the earliest opportunity; and

(4) The constitutional question is the lis mota of the case. 58

Respondents claim that the first three requisites are not present.
47

Section 1, Article VIII of the Constitution states that "(j)udicial power includes the duty of the courts of
justice to settle actual controversies involving rights which are legally demandable and enforceable."
The power of judicial review, therefore, is limited to the determination of actual cases and
controversies.59

An actual case or controversy means an existing case or controversy that is appropriate or ripe for
determination, not conjectural or anticipatory,60 lest the decision of the court would amount to an
advisory opinion.61 The power does not extend to hypothetical questions62 since any attempt at
abstraction could only lead to dialectics and barren legal questions and to sterile conclusions
unrelated to actualities.63

"Legal standing" or locus standi has been defined as a personal and substantial interest in the case
such that the party has sustained or will sustain direct injury as a result of the governmental act that is
being challenged,64alleging more than a generalized grievance.65 The gist of the question of standing
is whether a party alleges "such personal stake in the outcome of the controversy as to assure that
concrete adverseness which sharpens the presentation of issues upon which the court depends for
illumination of difficult constitutional questions."66 Unless a person is injuriously affected in any of his
constitutional rights by the operation of statute or ordinance, he has no standing.67

Petitioners traverse a wide range of sectors. Among them are La Bugal B'laan Tribal Association, Inc.,
a farmers and indigenous people's cooperative organized under Philippine laws representing a
community actually affected by the mining activities of WMCP, members of said cooperative, 68 as well
as other residents of areas also affected by the mining activities of WMCP. 69 These petitioners have
standing to raise the constitutionality of the questioned FTAA as they allege a personal and
substantial injury. They claim that they would suffer "irremediable displacement" 70 as a result of the
implementation of the FTAA allowing WMCP to conduct mining activities in their area of residence.
They thus meet the appropriate case requirement as they assert an interest adverse to that of
respondents who, on the other hand, insist on the FTAA's validity.

In view of the alleged impending injury, petitioners also have standing to assail the validity of E.O. No.
279, by authority of which the FTAA was executed.

Public respondents maintain that petitioners, being strangers to the FTAA, cannot sue either or both
contracting parties to annul it.71 In other words, they contend that petitioners are not real parties in
interest in an action for the annulment of contract.

Public respondents' contention fails. The present action is not merely one for annulment of contract
but for prohibition and mandamus. Petitioners allege that public respondents acted without or in
excess of jurisdiction in implementing the FTAA, which they submit is unconstitutional. As the case
involves constitutional questions, this Court is not concerned with whether petitioners are real parties
in interest, but with whether they have legal standing. As held in Kilosbayan v. Morato:72

x x x. "It is important to note . . . that standing because of its constitutional and public policy
underpinnings, is very different from questions relating to whether a particular plaintiff is the real party
in interest or has capacity to sue. Although all three requirements are directed towards ensuring that
only certain parties can maintain an action, standing restrictions require a partial consideration of the
merits, as well as broader policy concerns relating to the proper role of the judiciary in certain areas.["]
(FRIEDENTHAL, KANE AND MILLER, CIVIL PROCEDURE 328 [1985])

Standing is a special concern in constitutional law because in some cases suits are brought not by
parties who have been personally injured by the operation of a law or by official action taken, but by
concerned citizens, taxpayers or voters who actually sue in the public interest. Hence, the question in
standing is whether such parties have "alleged such a personal stake in the outcome of the
controversy as to assure that concrete adverseness which sharpens the presentation of issues upon
which the court so largely depends for illumination of difficult constitutional questions." (Baker v. Carr,
369 U.S. 186, 7 L.Ed.2d 633 [1962].)

As earlier stated, petitioners meet this requirement.


48

The challenge against the constitutionality of R.A. No. 7942 and DAO No. 96-40 likewise fulfills the
requisites of justiciability. Although these laws were not in force when the subject FTAA was entered
into, the question as to their validity is ripe for adjudication.

The WMCP FTAA provides:

14.3 Future Legislation

Any term and condition more favourable to Financial &Technical Assistance Agreement contractors
resulting from repeal or amendment of any existing law or regulation or from the enactment of a law,
regulation or administrative order shall be considered a part of this Agreement.

It is undisputed that R.A. No. 7942 and DAO No. 96-40 contain provisions that are more favorable to
WMCP, hence, these laws, to the extent that they are favorable to WMCP, govern the FTAA.

In addition, R.A. No. 7942 explicitly makes certain provisions apply to pre-existing agreements.

SEC. 112. Non-impairment of Existing Mining/Quarrying Rights. – x x x That the provisions of Chapter
XIV on government share in mineral production-sharing agreement and of Chapter XVI on incentives
of this Act shall immediately govern and apply to a mining lessee or contractor unless the mining
lessee or contractor indicates his intention to the secretary, in writing, not to avail of said provisions x
x x Provided, finally, That such leases, production-sharing agreements, financial or technical
assistance agreements shall comply with the applicable provisions of this Act and its implementing
rules and regulations.

As there is no suggestion that WMCP has indicated its intention not to avail of the provisions of
Chapter XVI of R.A. No. 7942, it can safely be presumed that they apply to the WMCP FTAA.

Misconstruing the application of the third requisite for judicial review – that the exercise of the review
is pleaded at the earliest opportunity – WMCP points out that the petition was filed only almost two
years after the execution of the FTAA, hence, not raised at the earliest opportunity.

The third requisite should not be taken to mean that the question of constitutionality must be raised
immediately after the execution of the state action complained of. That the question of constitutionality
has not been raised before is not a valid reason for refusing to allow it to be raised later.73 A contrary
rule would mean that a law, otherwise unconstitutional, would lapse into constitutionality by the mere
failure of the proper party to promptly file a case to challenge the same.

PROPRIETY OF PROHIBITION AND MANDAMUS

Before the effectivity in July 1997 of the Revised Rules of Civil Procedure, Section 2 of Rule 65 read:

SEC. 2. Petition for prohibition. – When the proceedings of any tribunal, corporation, board, or person,
whether exercising functions judicial or ministerial, are without or in excess of its or his jurisdiction, or
with grave abuse of discretion, and there is no appeal or any other plain, speedy, and adequate
remedy in the ordinary course of law, a person aggrieved thereby may file a verified petition in the
proper court alleging the facts with certainty and praying that judgment be rendered commanding the
defendant to desist from further proceeding in the action or matter specified therein.

Prohibition is a preventive remedy.74 It seeks a judgment ordering the defendant to desist from
continuing with the commission of an act perceived to be illegal.75

The petition for prohibition at bar is thus an appropriate remedy. While the execution of the contract
itself may be fait accompli, its implementation is not. Public respondents, in behalf of the Government,
have obligations to fulfill under said contract. Petitioners seek to prevent them from fulfilling such
obligations on the theory that the contract is unconstitutional and, therefore, void.
49

The propriety of a petition for prohibition being upheld, discussion of the propriety of the mandamus
aspect of the petition is rendered unnecessary.

HIERARCHY OF COURTS

The contention that the filing of this petition violated the rule on hierarchy of courts does not likewise
lie. The rule has been explained thus:

Between two courts of concurrent original jurisdiction, it is the lower court that should initially pass
upon the issues of a case. That way, as a particular case goes through the hierarchy of courts, it is
shorn of all but the important legal issues or those of first impression, which are the proper subject of
attention of the appellate court. This is a procedural rule borne of experience and adopted to improve
the administration of justice.

This Court has consistently enjoined litigants to respect the hierarchy of courts. Although this Court
has concurrent jurisdiction with the Regional Trial Courts and the Court of Appeals to issue writs of
certiorari, prohibition, mandamus, quo warranto, habeas corpus and injunction, such concurrence
does not give a party unrestricted freedom of choice of court forum. The resort to this Court's primary
jurisdiction to issue said writs shall be allowed only where the redress desired cannot be obtained in
the appropriate courts or where exceptional and compelling circumstances justify such invocation. We
held in People v. Cuaresma that:

A becoming regard for judicial hierarchy most certainly indicates that petitions for the issuance of
extraordinary writs against first level ("inferior") courts should be filed with the Regional Trial Court,
and those against the latter, with the Court of Appeals. A direct invocation of the Supreme Court's
original jurisdiction to issue these writs should be allowed only where there are special and important
reasons therefor, clearly and specifically set out in the petition. This is established policy. It is a policy
necessary to prevent inordinate demands upon the Court's time and attention which are better
devoted to those matters within its exclusive jurisdiction, and to prevent further over-crowding of the
Court's docket x x x.76 [Emphasis supplied.]

The repercussions of the issues in this case on the Philippine mining industry, if not the national
economy, as well as the novelty thereof, constitute exceptional and compelling circumstances to
justify resort to this Court in the first instance.

In all events, this Court has the discretion to take cognizance of a suit which does not satisfy the
requirements of an actual case or legal standing when paramount public interest is involved. 77 When
the issues raised are of paramount importance to the public, this Court may brush aside technicalities
of procedure.78

II

Petitioners contend that E.O. No. 279 did not take effect because its supposed date of effectivity
came after President Aquino had already lost her legislative powers under the Provisional
Constitution.

And they likewise claim that the WMC FTAA, which was entered into pursuant to E.O. No. 279,
violates Section 2, Article XII of the Constitution because, among other reasons:

(1) It allows foreign-owned companies to extend more than mere financial or technical
assistance to the State in the exploitation, development, and utilization of minerals,
petroleum, and other mineral oils, and even permits foreign owned companies to "operate
and manage mining activities."

(2) It allows foreign-owned companies to extend both technical and financial assistance,
instead of "either technical or financial assistance."
50

To appreciate the import of these issues, a visit to the history of the pertinent constitutional provision,
the concepts contained therein, and the laws enacted pursuant thereto, is in order.

Section 2, Article XII reads in full:

Sec. 2. All lands of the public domain, waters, minerals, coal, petroleum, and other mineral oils, all
forces of potential energy, fisheries, forests or timber, wildlife, flora and fauna, and other natural
resources are owned by the State. With the exception of agricultural lands, all other natural resources
shall not be alienated. The exploration, development, and utilization of natural resources shall be
under the full control and supervision of the State. The State may directly undertake such activities or
it may enter into co-production, joint venture, or production-sharing agreements with Filipino citizens,
or corporations or associations at least sixty per centum of whose capital is owned by such citizens.
Such agreements may be for a period not exceeding twenty-five years, renewable for not more than
twenty-five years, and under such terms and conditions as may be provided by law. In cases of water
rights for irrigation, water supply, fisheries, or industrial uses other than the development of water
power, beneficial use may be the measure and limit of the grant.

The State shall protect the nation's marine wealth in its archipelagic waters, territorial sea, and
exclusive economic zone, and reserve its use and enjoyment exclusively to Filipino citizens.

The Congress may, by law, allow small-scale utilization of natural resources by Filipino citizens, as
well as cooperative fish farming, with priority to subsistence fishermen and fish-workers in rivers,
lakes, bays, and lagoons.

The President may enter into agreements with foreign-owned corporations involving either technical
or financial assistance for large-scale exploration, development, and utilization of minerals, petroleum,
and other mineral oils according to the general terms and conditions provided by law, based on real
contributions to the economic growth and general welfare of the country. In such agreements, the
State shall promote the development and use of local scientific and technical resources.

The President shall notify the Congress of every contract entered into in accordance with this
provision, within thirty days from its execution.

THE SPANISH REGIME AND THE REGALIAN DOCTRINE

The first sentence of Section 2 embodies the Regalian doctrine or jura regalia. Introduced by Spain
into these Islands, this feudal concept is based on the State's power of dominium, which is the
capacity of the State to own or acquire property.79

In its broad sense, the term "jura regalia" refers to royal rights, or those rights which the King has by
virtue of his prerogatives. In Spanish law, it refers to a right which the sovereign has over anything in
which a subject has a right of property or propriedad. These were rights enjoyed during feudal times
by the king as the sovereign.

The theory of the feudal system was that title to all lands was originally held by the King, and while the
use of lands was granted out to others who were permitted to hold them under certain conditions, the
King theoretically retained the title. By fiction of law, the King was regarded as the original proprietor
of all lands, and the true and only source of title, and from him all lands were held. The theory of jura
regalia was therefore nothing more than a natural fruit of conquest.80

The Philippines having passed to Spain by virtue of discovery and conquest, 81 earlier Spanish
decrees declared that "all lands were held from the Crown."82

The Regalian doctrine extends not only to land but also to "all natural wealth that may be found in the
bowels of the earth."83 Spain, in particular, recognized the unique value of natural resources, viewing
them, especially minerals, as an abundant source of revenue to finance its wars against other
nations.84 Mining laws during the Spanish regime reflected this perspective.85
51

THE AMERICAN OCCUPATION AND THE CONCESSION REGIME

By the Treaty of Paris of December 10, 1898, Spain ceded "the archipelago known as the Philippine
Islands" to the United States. The Philippines was hence governed by means of organic acts that
were in the nature of charters serving as a Constitution of the occupied territory from 1900 to
1935.86 Among the principal organic acts of the Philippines was the Act of Congress of July 1, 1902,
more commonly known as the Philippine Bill of 1902, through which the United States Congress
assumed the administration of the Philippine Islands.87 Section 20 of said Bill reserved the disposition
of mineral lands of the public domain from sale. Section 21 thereof allowed the free and open
exploration, occupation and purchase of mineral deposits not only to citizens of the Philippine Islands
but to those of the United States as well:

Sec. 21. That all valuable mineral deposits in public lands in the Philippine Islands, both surveyed and
unsurveyed, are hereby declared to be free and open to exploration, occupation and purchase, and
the land in which they are found, to occupation and purchase, by citizens of the United States or of
said Islands: Provided, That when on any lands in said Islands entered and occupied as agricultural
lands under the provisions of this Act, but not patented, mineral deposits have been found, the
working of such mineral deposits is forbidden until the person, association, or corporation who or
which has entered and is occupying such lands shall have paid to the Government of said Islands
such additional sum or sums as will make the total amount paid for the mineral claim or claims in
which said deposits are located equal to the amount charged by the Government for the same as
mineral claims.

Unlike Spain, the United States considered natural resources as a source of wealth for its nationals
and saw fit to allow both Filipino and American citizens to explore and exploit minerals in public lands,
and to grant patents to private mineral lands.88 A person who acquired ownership over a parcel of
private mineral land pursuant to the laws then prevailing could exclude other persons, even the State,
from exploiting minerals within his property.89 Thus, earlier jurisprudence90 held that:

A valid and subsisting location of mineral land, made and kept up in accordance with the provisions of
the statutes of the United States, has the effect of a grant by the United States of the present and
exclusive possession of the lands located, and this exclusive right of possession and enjoyment
continues during the entire life of the location. x x x.

x x x.

The discovery of minerals in the ground by one who has a valid mineral location perfects his claim
and his location not only against third persons, but also against the Government. x x x. [Italics in the
original.]

The Regalian doctrine and the American system, therefore, differ in one essential respect. Under the
Regalian theory, mineral rights are not included in a grant of land by the state; under the American
doctrine, mineral rights are included in a grant of land by the government.91

Section 21 also made possible the concession (frequently styled "permit", license" or
"lease")92 system.93 This was the traditional regime imposed by the colonial administrators for the
exploitation of natural resources in the extractive sector (petroleum, hard minerals, timber, etc.).94

Under the concession system, the concessionaire makes a direct equity investment for the purpose of
exploiting a particular natural resource within a given area.95 Thus, the concession amounts to
complete control by the concessionaire over the country's natural resource, for it is given exclusive
and plenary rights to exploit a particular resource at the point of extraction. 96 In consideration for the
right to exploit a natural resource, the concessionaire either pays rent or royalty, which is a fixed
percentage of the gross proceeds.97

Later statutory enactments by the legislative bodies set up in the Philippines adopted the contractual
framework of the concession.98 For instance, Act No. 2932,99 approved on August 31, 1920, which
provided for the exploration, location, and lease of lands containing petroleum and other mineral oils
52

and gas in the Philippines, and Act No. 2719,100 approved on May 14, 1917, which provided for the
leasing and development of coal lands in the Philippines, both utilized the concession system.101

THE 1935 CONSTITUTION AND THE NATIONALIZATION OF NATURAL RESOURCES

By the Act of United States Congress of March 24, 1934, popularly known as the Tydings-McDuffie
Law, the People of the Philippine Islands were authorized to adopt a constitution. 102 On July 30, 1934,
the Constitutional Convention met for the purpose of drafting a constitution, and the Constitution
subsequently drafted was approved by the Convention on February 8, 1935. 103 The Constitution was
submitted to the President of the United States on March 18, 1935.104 On March 23, 1935, the
President of the United States certified that the Constitution conformed substantially with the
provisions of the Act of Congress approved on March 24, 1934.105 On May 14, 1935, the Constitution
was ratified by the Filipino people.106

The 1935 Constitution adopted the Regalian doctrine, declaring all natural resources of the
Philippines, including mineral lands and minerals, to be property belonging to the State.107 As adopted
in a republican system, the medieval concept of jura regalia is stripped of royal overtones and
ownership of the land is vested in the State. 108

Section 1, Article XIII, on Conservation and Utilization of Natural Resources, of the 1935 Constitution
provided:

SECTION 1. All agricultural, timber, and mineral lands of the public domain, waters, minerals,
coal, petroleum, and other mineral oils, all forces of potential energy, and other natural
resources of the Philippines belong to the State, and their disposition, exploitation,
development, or utilization shall be limited to citizens of the Philippines, or to corporations or
associations at least sixty per centum of the capital of which is owned by such citizens,
subject to any existing right, grant, lease, or concession at the time of the inauguration of the
Government established under this Constitution. Natural resources, with the exception of
public agricultural land, shall not be alienated, and no license, concession, or lease for the
exploitation, development, or utilization of any of the natural resources shall be granted for a
period exceeding twenty-five years, except as to water rights for irrigation, water supply,
fisheries, or industrial uses other than the development of water power, in which cases
beneficial use may be the measure and the limit of the grant.

The nationalization and conservation of the natural resources of the country was one of the fixed and
dominating objectives of the 1935 Constitutional Convention.109 One delegate relates:

There was an overwhelming sentiment in the Convention in favor of the principle of state ownership of
natural resources and the adoption of the Regalian doctrine. State ownership of natural resources
was seen as a necessary starting point to secure recognition of the state's power to control their
disposition, exploitation, development, or utilization. The delegates of the Constitutional Convention
very well knew that the concept of State ownership of land and natural resources was introduced by
the Spaniards, however, they were not certain whether it was continued and applied by the
Americans. To remove all doubts, the Convention approved the provision in the Constitution affirming
the Regalian doctrine.

The adoption of the principle of state ownership of the natural resources and of the Regalian doctrine
was considered to be a necessary starting point for the plan of nationalizing and conserving the
natural resources of the country. For with the establishment of the principle of state ownership of the
natural resources, it would not be hard to secure the recognition of the power of the State to control
their disposition, exploitation, development or utilization.110

The nationalization of the natural resources was intended (1) to insure their conservation for Filipino
posterity; (2) to serve as an instrument of national defense, helping prevent the extension to the
country of foreign control through peaceful economic penetration; and (3) to avoid making the
Philippines a source of international conflicts with the consequent danger to its internal security and
independence.111
53

The same Section 1, Article XIII also adopted the concession system, expressly permitting the State
to grant licenses, concessions, or leases for the exploitation, development, or utilization of any of the
natural resources. Grants, however, were limited to Filipinos or entities at least 60% of the capital of
which is owned by Filipinos.lawph!l.ne+

The swell of nationalism that suffused the 1935 Constitution was radically diluted when on November
1946, the Parity Amendment, which came in the form of an "Ordinance Appended to the Constitution,"
was ratified in a plebiscite.112 The Amendment extended, from July 4, 1946 to July 3, 1974, the right
to utilize and exploit our natural resources to citizens of the United States and business enterprises
owned or controlled, directly or indirectly, by citizens of the United States: 113

Notwithstanding the provision of section one, Article Thirteen, and section eight, Article Fourteen, of
the foregoing Constitution, during the effectivity of the Executive Agreement entered into by the
President of the Philippines with the President of the United States on the fourth of July, nineteen
hundred and forty-six, pursuant to the provisions of Commonwealth Act Numbered Seven hundred
and thirty-three, but in no case to extend beyond the third of July, nineteen hundred and seventy-four,
the disposition, exploitation, development, and utilization of all agricultural, timber, and mineral lands
of the public domain, waters, minerals, coals, petroleum, and other mineral oils, all forces and sources
of potential energy, and other natural resources of the Philippines, and the operation of public utilities,
shall, if open to any person, be open to citizens of the United States and to all forms of business
enterprise owned or controlled, directly or indirectly, by citizens of the United States in the same
manner as to, and under the same conditions imposed upon, citizens of the Philippines or
corporations or associations owned or controlled by citizens of the Philippines.

The Parity Amendment was subsequently modified by the 1954 Revised Trade Agreement, also
known as the Laurel-Langley Agreement, embodied in Republic Act No. 1355.114

THE PETROLEUM ACT OF 1949 AND THE CONCESSION SYSTEM

In the meantime, Republic Act No. 387,115 also known as the Petroleum Act of 1949, was approved
on June 18, 1949.

The Petroleum Act of 1949 employed the concession system for the exploitation of the nation's
petroleum resources. Among the kinds of concessions it sanctioned were exploration and exploitation
concessions, which respectively granted to the concessionaire the exclusive right to explore for 116 or
develop117 petroleum within specified areas.

Concessions may be granted only to duly qualified persons118 who have sufficient finances,
organization, resources, technical competence, and skills necessary to conduct the operations to be
undertaken.119

Nevertheless, the Government reserved the right to undertake such work itself. 120 This proceeded
from the theory that all natural deposits or occurrences of petroleum or natural gas in public and/or
private lands in the Philippines belong to the State.121 Exploration and exploitation concessions did
not confer upon the concessionaire ownership over the petroleum lands and petroleum
deposits.122 However, they did grant concessionaires the right to explore, develop, exploit, and utilize
them for the period and under the conditions determined by the law.123

Concessions were granted at the complete risk of the concessionaire; the Government did not
guarantee the existence of petroleum or undertake, in any case, title warranty. 124

Concessionaires were required to submit information as maybe required by the Secretary of


Agriculture and Natural Resources, including reports of geological and geophysical examinations, as
well as production reports.125Exploration126 and exploitation127 concessionaires were also required to
submit work programs.lavvphi1.net

Exploitation concessionaires, in particular, were obliged to pay an annual exploitation tax,128 the
object of which is to induce the concessionaire to actually produce petroleum, and not simply to sit on
54

the concession without developing or exploiting it.129 These concessionaires were also bound to pay
the Government royalty, which was not less than 12½% of the petroleum produced and saved, less
that consumed in the operations of the concessionaire. 130 Under Article 66, R.A. No. 387, the
exploitation tax may be credited against the royalties so that if the concessionaire shall be actually
producing enough oil, it would not actually be paying the exploitation tax. 131

Failure to pay the annual exploitation tax for two consecutive years, 132 or the royalty due to the
Government within one year from the date it becomes due,133 constituted grounds for the cancellation
of the concession. In case of delay in the payment of the taxes or royalty imposed by the law or by the
concession, a surcharge of 1% per month is exacted until the same are paid. 134

As a rule, title rights to all equipment and structures that the concessionaire placed on the land belong
to the exploration or exploitation concessionaire.135 Upon termination of such concession, the
concessionaire had a right to remove the same.136

The Secretary of Agriculture and Natural Resources was tasked with carrying out the provisions of the
law, through the Director of Mines, who acted under the Secretary's immediate supervision and
control.137 The Act granted the Secretary the authority to inspect any operation of the concessionaire
and to examine all the books and accounts pertaining to operations or conditions related to payment
of taxes and royalties.138

The same law authorized the Secretary to create an Administration Unit and a Technical
Board.139 The Administration Unit was charged, inter alia, with the enforcement of the provisions of
the law.140 The Technical Board had, among other functions, the duty to check on the performance of
concessionaires and to determine whether the obligations imposed by the Act and its implementing
regulations were being complied with.141

Victorio Mario A. Dimagiba, Chief Legal Officer of the Bureau of Energy Development, analyzed the
benefits and drawbacks of the concession system insofar as it applied to the petroleum industry:

Advantages of Concession. Whether it emphasizes income tax or royalty, the most positive aspect of
the concession system is that the State's financial involvement is virtually risk free and administration
is simple and comparatively low in cost. Furthermore, if there is a competitive allocation of the
resource leading to substantial bonuses and/or greater royalty coupled with a relatively high level of
taxation, revenue accruing to the State under the concession system may compare favorably with
other financial arrangements.

Disadvantages of Concession. There are, however, major negative aspects to this system. Because
the Government's role in the traditional concession is passive, it is at a distinct disadvantage in
managing and developing policy for the nation's petroleum resource. This is true for several reasons.
First, even though most concession agreements contain covenants requiring diligence in operations
and production, this establishes only an indirect and passive control of the host country in resource
development. Second, and more importantly, the fact that the host country does not directly
participate in resource management decisions inhibits its ability to train and employ its nationals in
petroleum development. This factor could delay or prevent the country from effectively engaging in
the development of its resources. Lastly, a direct role in management is usually necessary in order to
obtain a knowledge of the international petroleum industry which is important to an appreciation of the
host country's resources in relation to those of other countries. 142

Other liabilities of the system have also been noted:

x x x there are functional implications which give the concessionaire great economic power arising
from its exclusive equity holding. This includes, first, appropriation of the returns of the undertaking,
subject to a modest royalty; second, exclusive management of the project; third, control of production
of the natural resource, such as volume of production, expansion, research and development; and
fourth, exclusive responsibility for downstream operations, like processing, marketing, and
distribution. In short, even if nominally, the state is the sovereign and owner of the natural resource
being exploited, it has been shorn of all elements of control over such natural resource because of the
55

exclusive nature of the contractual regime of the concession. The concession system, investing as it
does ownership of natural resources, constitutes a consistent inconsistency with the principle
embodied in our Constitution that natural resources belong to the state and shall not be alienated, not
to mention the fact that the concession was the bedrock of the colonial system in the exploitation of
natural resources.143

Eventually, the concession system failed for reasons explained by Dimagiba:

Notwithstanding the good intentions of the Petroleum Act of 1949, the concession system could not
have properly spurred sustained oil exploration activities in the country, since it assumed that such a
capital-intensive, high risk venture could be successfully undertaken by a single individual or a small
company. In effect, concessionaires' funds were easily exhausted. Moreover, since the concession
system practically closed its doors to interested foreign investors, local capital was stretched to the
limits. The old system also failed to consider the highly sophisticated technology and expertise
required, which would be available only to multinational companies. 144

A shift to a new regime for the development of natural resources thus seemed imminent.

PRESIDENTIAL DECREE NO. 87, THE 1973 CONSTITUTION AND THE SERVICE CONTRACT
SYSTEM

The promulgation on December 31, 1972 of Presidential Decree No. 87,145 otherwise known as The
Oil Exploration and Development Act of 1972 signaled such a transformation. P.D. No. 87 permitted
the government to explore for and produce indigenous petroleum through "service contracts."146

"Service contracts" is a term that assumes varying meanings to different people, and it has carried
many names in different countries, like "work contracts" in Indonesia, "concession agreements" in
Africa, "production-sharing agreements" in the Middle East, and "participation agreements" in Latin
America.147 A functional definition of "service contracts" in the Philippines is provided as follows:

A service contract is a contractual arrangement for engaging in the exploitation and development of
petroleum, mineral, energy, land and other natural resources by which a government or its agency, or
a private person granted a right or privilege by the government authorizes the other party (service
contractor) to engage or participate in the exercise of such right or the enjoyment of the privilege, in
that the latter provides financial or technical resources, undertakes the exploitation or production of a
given resource, or directly manages the productive enterprise, operations of the exploration and
exploitation of the resources or the disposition of marketing or resources. 148

In a service contract under P.D. No. 87, service and technology are furnished by the service
contractor for which it shall be entitled to the stipulated service fee.149 The contractor must be
technically competent and financially capable to undertake the operations required in the contract. 150

Financing is supposed to be provided by the Government to which all petroleum produced


belongs.151 In case the Government is unable to finance petroleum exploration operations, the
contractor may furnish services, technology and financing, and the proceeds of sale of the petroleum
produced under the contract shall be the source of funds for payment of the service fee and the
operating expenses due the contractor.152 The contractor shall undertake, manage and execute
petroleum operations, subject to the government overseeing the management of the
operations.153 The contractor provides all necessary services and technology and the requisite
financing, performs the exploration work obligations, and assumes all exploration risks such that if no
petroleum is produced, it will not be entitled to reimbursement. 154 Once petroleum in commercial
quantity is discovered, the contractor shall operate the field on behalf of the government.155

P.D. No. 87 prescribed minimum terms and conditions for every service contract. 156 It also granted the
contractor certain privileges, including exemption from taxes and payment of tariff duties, 157 and
permitted the repatriation of capital and retention of profits abroad.158
56

Ostensibly, the service contract system had certain advantages over the concession regime. 159 It has
been opined, though, that, in the Philippines, our concept of a service contract, at least in the
petroleum industry, was basically a concession regime with a production-sharing element.160

On January 17, 1973, then President Ferdinand E. Marcos proclaimed the ratification of a new
Constitution.161Article XIV on the National Economy and Patrimony contained provisions similar to the
1935 Constitution with regard to Filipino participation in the nation's natural resources. Section 8,
Article XIV thereof provides:

Sec. 8. All lands of the public domain, waters, minerals, coal, petroleum and other mineral oils, all
forces of potential energy, fisheries, wildlife, and other natural resources of the Philippines belong to
the State. With the exception of agricultural, industrial or commercial, residential and resettlement
lands of the public domain, natural resources shall not be alienated, and no license, concession, or
lease for the exploration, development, exploitation, or utilization of any of the natural resources shall
be granted for a period exceeding twenty-five years, renewable for not more than twenty-five years,
except as to water rights for irrigation, water supply, fisheries, or industrial uses other than the
development of water power, in which cases beneficial use may be the measure and the limit of the
grant.

While Section 9 of the same Article maintained the Filipino-only policy in the enjoyment of natural
resources, it also allowed Filipinos, upon authority of the Batasang Pambansa, to enter into service
contracts with any person or entity for the exploration or utilization of natural resources.

Sec. 9. The disposition, exploration, development, exploitation, or utilization of any of the natural
resources of the Philippines shall be limited to citizens, or to corporations or associations at least sixty
per centum of which is owned by such citizens. The Batasang Pambansa, in the national interest,
may allow such citizens, corporations or associations to enter into service contracts for financial,
technical, management, or other forms of assistance with any person or entity for the exploration, or
utilization of any of the natural resources. Existing valid and binding service contracts for financial,
technical, management, or other forms of assistance are hereby recognized as such. [Emphasis
supplied.]

The concept of service contracts, according to one delegate, was borrowed from the methods
followed by India, Pakistan and especially Indonesia in the exploration of petroleum and mineral
oils.162 The provision allowing such contracts, according to another, was intended to "enhance the
proper development of our natural resources since Filipino citizens lack the needed capital and
technical know-how which are essential in the proper exploration, development and exploitation of the
natural resources of the country."163

The original idea was to authorize the government, not private entities, to enter into service contracts
with foreign entities.164 As finally approved, however, a citizen or private entity could be allowed by the
National Assembly to enter into such service contract. 165 The prior approval of the National Assembly
was deemed sufficient to protect the national interest. 166 Notably, none of the laws allowing service
contracts were passed by the Batasang Pambansa. Indeed, all of them were enacted by presidential
decree.

On March 13, 1973, shortly after the ratification of the new Constitution, the President promulgated
Presidential Decree No. 151.167 The law allowed Filipino citizens or entities which have acquired lands
of the public domain or which own, hold or control such lands to enter into service contracts for
financial, technical, management or other forms of assistance with any foreign persons or entity for
the exploration, development, exploitation or utilization of said lands. 168

Presidential Decree No. 463,169 also known as The Mineral Resources Development Decree of 1974,
was enacted on May 17, 1974. Section 44 of the decree, as amended, provided that a lessee of a
mining claim may enter into a service contract with a qualified domestic or foreign contractor for the
exploration, development and exploitation of his claims and the processing and marketing of the
product thereof.
57

Presidential Decree No. 704170 (The Fisheries Decree of 1975), approved on May 16, 1975, allowed
Filipinos engaged in commercial fishing to enter into contracts for financial, technical or other forms of
assistance with any foreign person, corporation or entity for the production, storage, marketing and
processing of fish and fishery/aquatic products.171

Presidential Decree No. 705172 (The Revised Forestry Code of the Philippines), approved on May 19,
1975, allowed "forest products licensees, lessees, or permitees to enter into service contracts for
financial, technical, management, or other forms of assistance . . . with any foreign person or entity for
the exploration, development, exploitation or utilization of the forest resources." 173

Yet another law allowing service contracts, this time for geothermal resources, was Presidential
Decree No. 1442,174 which was signed into law on June 11, 1978. Section 1 thereof authorized the
Government to enter into service contracts for the exploration, exploitation and development of
geothermal resources with a foreign contractor who must be technically and financially capable of
undertaking the operations required in the service contract.

Thus, virtually the entire range of the country's natural resources –from petroleum and minerals to
geothermal energy, from public lands and forest resources to fishery products – was well covered by
apparent legal authority to engage in the direct participation or involvement of foreign persons or
corporations (otherwise disqualified) in the exploration and utilization of natural resources through
service contracts.175

THE 1987 CONSTITUTION AND TECHNICAL OR FINANCIAL ASSISTANCE AGREEMENTS

After the February 1986 Edsa Revolution, Corazon C. Aquino took the reins of power under a
revolutionary government. On March 25, 1986, President Aquino issued Proclamation No.
3,176 promulgating the Provisional Constitution, more popularly referred to as the Freedom
Constitution. By authority of the same Proclamation, the President created a Constitutional
Commission (CONCOM) to draft a new constitution, which took effect on the date of its ratification on
February 2, 1987.177

The 1987 Constitution retained the Regalian doctrine. The first sentence of Section 2, Article XII
states: "All lands of the public domain, waters, minerals, coal, petroleum, and other mineral oils, all
forces of potential energy, fisheries, forests or timber, wildlife, flora and fauna, and other natural
resources are owned by the State."

Like the 1935 and 1973 Constitutions before it, the 1987 Constitution, in the second sentence of the
same provision, prohibits the alienation of natural resources, except agricultural lands.

The third sentence of the same paragraph is new: "The exploration, development and utilization of
natural resources shall be under the full control and supervision of the State." The constitutional policy
of the State's "full control and supervision" over natural resources proceeds from the concept of jura
regalia, as well as the recognition of the importance of the country's natural resources, not only for
national economic development, but also for its security and national defense. 178 Under this provision,
the State assumes "a more dynamic role" in the exploration, development and utilization of natural
resources.179

Conspicuously absent in Section 2 is the provision in the 1935 and 1973 Constitutions authorizing the
State to grant licenses, concessions, or leases for the exploration, exploitation, development, or
utilization of natural resources. By such omission, the utilization of inalienable lands of public domain
through "license, concession or lease" is no longer allowed under the 1987 Constitution. 180

Having omitted the provision on the concession system, Section 2 proceeded to introduce "unfamiliar
language":181

The State may directly undertake such activities or it may enter into co-production, joint venture, or
production-sharing agreements with Filipino citizens, or corporations or associations at least sixty per
centum of whose capital is owned by such citizens.
58

Consonant with the State's "full supervision and control" over natural resources, Section 2 offers the
State two "options."182 One, the State may directly undertake these activities itself; or two, it may enter
into co-production, joint venture, or production-sharing agreements with Filipino citizens, or entities at
least 60% of whose capital is owned by such citizens.

A third option is found in the third paragraph of the same section:

The Congress may, by law, allow small-scale utilization of natural resources by Filipino citizens, as
well as cooperative fish farming, with priority to subsistence fishermen and fish-workers in rivers,
lakes, bays, and lagoons.

While the second and third options are limited only to Filipino citizens or, in the case of the former, to
corporations or associations at least 60% of the capital of which is owned by Filipinos, a fourth allows
the participation of foreign-owned corporations. The fourth and fifth paragraphs of Section 2 provide:

The President may enter into agreements with foreign-owned corporations involving either technical
or financial assistance for large-scale exploration, development, and utilization of minerals, petroleum,
and other mineral oils according to the general terms and conditions provided by law, based on real
contributions to the economic growth and general welfare of the country. In such agreements, the
State shall promote the development and use of local scientific and technical resources.

The President shall notify the Congress of every contract entered into in accordance with this
provision, within thirty days from its execution.

Although Section 2 sanctions the participation of foreign-owned corporations in the exploration,


development, and utilization of natural resources, it imposes certain limitations or conditions to
agreements with such corporations.

First, the parties to FTAAs. Only the President, in behalf of the State, may enter into these
agreements, and only with corporations. By contrast, under the 1973 Constitution, a Filipino
citizen, corporation or association may enter into a service contract with a "foreign person or
entity."

Second, the size of the activities: only large-scale exploration, development, and utilization is
allowed. The term "large-scale usually refers to very capital-intensive activities."183

Third, the natural resources subject of the activities is restricted to minerals, petroleum and
other mineral oils, the intent being to limit service contracts to those areas where Filipino
capital may not be sufficient.184

Fourth, consistency with the provisions of statute. The agreements must be in accordance
with the terms and conditions provided by law.

Fifth, Section 2 prescribes certain standards for entering into such agreements. The
agreements must be based on real contributions to economic growth and general welfare of
the country.

Sixth, the agreements must contain rudimentary stipulations for the promotion of the
development and use of local scientific and technical resources.

Seventh, the notification requirement. The President shall notify Congress of every financial
or technical assistance agreement entered into within thirty days from its execution.

Finally, the scope of the agreements. While the 1973 Constitution referred to "service
contracts for financial, technical, management, or other forms of assistance" the 1987
Constitution provides for "agreements. . . involving either financial or technical assistance." It
59

bears noting that the phrases "service contracts" and "management or other forms of
assistance" in the earlier constitution have been omitted.

By virtue of her legislative powers under the Provisional Constitution, 185 President Aquino, on July 10,
1987, signed into law E.O. No. 211 prescribing the interim procedures in the processing and approval
of applications for the exploration, development and utilization of minerals. The omission in the 1987
Constitution of the term "service contracts" notwithstanding, the said E.O. still referred to them in
Section 2 thereof:

Sec. 2. Applications for the exploration, development and utilization of mineral resources, including
renewal applications and applications for approval of operating agreements and mining service
contracts, shall be accepted and processed and may be approved x x x. [Emphasis supplied.]

The same law provided in its Section 3 that the "processing, evaluation and approval of all mining
applications . . . operating agreements and service contracts . . . shall be governed by Presidential
Decree No. 463, as amended, other existing mining laws, and their implementing rules and
regulations. . . ."

As earlier stated, on the 25th also of July 1987, the President issued E.O. No. 279 by authority of
which the subject WMCP FTAA was executed on March 30, 1995.

On March 3, 1995, President Ramos signed into law R.A. No. 7942. Section 15 thereof declares that
the Act "shall govern the exploration, development, utilization, and processing of all mineral
resources." Such declaration notwithstanding, R.A. No. 7942 does not actually cover all the modes
through which the State may undertake the exploration, development, and utilization of natural
resources.

The State, being the owner of the natural resources, is accorded the primary power and responsibility
in the exploration, development and utilization thereof. As such, it may undertake these activities
through four modes:

The State may directly undertake such activities.

(2) The State may enter into co-production, joint venture or production-sharing agreements
with Filipino citizens or qualified corporations.

(3) Congress may, by law, allow small-scale utilization of natural resources by Filipino
citizens.

(4) For the large-scale exploration, development and utilization of minerals, petroleum and
other mineral oils, the President may enter into agreements with foreign-owned corporations
involving technical or financial assistance. 186

Except to charge the Mines and Geosciences Bureau of the DENR with performing researches and
surveys,187 and a passing mention of government-owned or controlled corporations,188 R.A. No. 7942
does not specify how the State should go about the first mode. The third mode, on the other hand, is
governed by Republic Act No. 7076189(the People's Small-Scale Mining Act of 1991) and other
pertinent laws.190 R.A. No. 7942 primarily concerns itself with the second and fourth modes.

Mineral production sharing, co-production and joint venture agreements are collectively classified by
R.A. No. 7942 as "mineral agreements."191 The Government participates the least in a mineral
production sharing agreement (MPSA). In an MPSA, the Government grants the contractor 192 the
exclusive right to conduct mining operations within a contract area 193 and shares in the gross
output.194 The MPSA contractor provides the financing, technology, management and personnel
necessary for the agreement's implementation.195 The total government share in an MPSA is the
excise tax on mineral products under Republic Act No. 7729,196 amending Section 151(a) of the
National Internal Revenue Code, as amended.197
60

In a co-production agreement (CA),198 the Government provides inputs to the mining operations other
than the mineral resource,199 while in a joint venture agreement (JVA), where the Government enjoys
the greatest participation, the Government and the JVA contractor organize a company with both
parties having equity shares.200 Aside from earnings in equity, the Government in a JVA is also
entitled to a share in the gross output.201The Government may enter into a CA202 or JVA203 with one or
more contractors. The Government's share in a CA or JVA is set out in Section 81 of the law:

The share of the Government in co-production and joint venture agreements shall be negotiated by
the Government and the contractor taking into consideration the: (a) capital investment of the project,
(b) the risks involved, (c) contribution of the project to the economy, and (d) other factors that will
provide for a fair and equitable sharing between the Government and the contractor. The Government
shall also be entitled to compensations for its other contributions which shall be agreed upon by the
parties, and shall consist, among other things, the contractor's income tax, excise tax, special
allowance, withholding tax due from the contractor's foreign stockholders arising from dividend or
interest payments to the said foreign stockholders, in case of a foreign national and all such other
taxes, duties and fees as provided for under existing laws.

All mineral agreements grant the respective contractors the exclusive right to conduct mining
operations and to extract all mineral resources found in the contract area. 204 A "qualified person" may
enter into any of the mineral agreements with the Government.205 A "qualified person" is

any citizen of the Philippines with capacity to contract, or a corporation, partnership, association, or
cooperative organized or authorized for the purpose of engaging in mining, with technical and
financial capability to undertake mineral resources development and duly registered in accordance
with law at least sixty per centum (60%) of the capital of which is owned by citizens of the Philippines
x x x.206

The fourth mode involves "financial or technical assistance agreements." An FTAA is defined as "a
contract involving financial or technical assistance for large-scale exploration, development, and
utilization of natural resources."207 Any qualified person with technical and financial capability to
undertake large-scale exploration, development, and utilization of natural resources in the Philippines
may enter into such agreement directly with the Government through the DENR. 208 For the purpose of
granting an FTAA, a legally organized foreign-owned corporation (any corporation, partnership,
association, or cooperative duly registered in accordance with law in which less than 50% of the
capital is owned by Filipino citizens)209 is deemed a "qualified person."210

Other than the difference in contractors' qualifications, the principal distinction between mineral
agreements and FTAAs is the maximum contract area to which a qualified person may hold or be
granted.211 "Large-scale" under R.A. No. 7942 is determined by the size of the contract area, as
opposed to the amount invested (US $50,000,000.00), which was the standard under E.O. 279.

Like a CA or a JVA, an FTAA is subject to negotiation.212 The Government's contributions, in the form
of taxes, in an FTAA is identical to its contributions in the two mineral agreements, save that in an
FTAA:

The collection of Government share in financial or technical assistance agreement shall commence
after the financial or technical assistance agreement contractor has fully recovered its pre-operating
expenses, exploration, and development expenditures, inclusive.213

III

Having examined the history of the constitutional provision and statutes enacted pursuant thereto, a
consideration of the substantive issues presented by the petition is now in order.

THE EFFECTIVITY OF EXECUTIVE ORDER NO. 279

Petitioners argue that E.O. No. 279, the law in force when the WMC FTAA was executed, did not
come into effect.
61

E.O. No. 279 was signed into law by then President Aquino on July 25, 1987, two days before the
opening of Congress on July 27, 1987.214 Section 8 of the E.O. states that the same "shall take effect
immediately." This provision, according to petitioners, runs counter to Section 1 of E.O. No.
200,215 which provides:

SECTION 1. Laws shall take effect after fifteen days following the completion of their publication
either in the Official Gazette or in a newspaper of general circulation in the Philippines, unless it is
otherwise provided.216 [Emphasis supplied.]

On that premise, petitioners contend that E.O. No. 279 could have only taken effect fifteen days after
its publication at which time Congress had already convened and the President's power to legislate
had ceased.

Respondents, on the other hand, counter that the validity of E.O. No. 279 was settled in Miners
Association of the Philippines v. Factoran, supra. This is of course incorrect for the issue in Miners
Association was not the validity of E.O. No. 279 but that of DAO Nos. 57 and 82 which were issued
pursuant thereto.

Nevertheless, petitioners' contentions have no merit.

It bears noting that there is nothing in E.O. No. 200 that prevents a law from taking effect on a date
other than – even before – the 15-day period after its publication. Where a law provides for its own
date of effectivity, such date prevails over that prescribed by E.O. No. 200. Indeed, this is the very
essence of the phrase "unless it is otherwise provided" in Section 1 thereof. Section 1, E.O. No. 200,
therefore, applies only when a statute does not provide for its own date of effectivity.

What is mandatory under E.O. No. 200, and what due process requires, as this Court held in Tañada
v. Tuvera,217is the publication of the law for without such notice and publication, there would be no
basis for the application of the maxim "ignorantia legis n[eminem] excusat." It would be the height of
injustice to punish or otherwise burden a citizen for the transgression of a law of which he had no
notice whatsoever, not even a constructive one.

While the effectivity clause of E.O. No. 279 does not require its publication, it is not a ground for its
invalidation since the Constitution, being "the fundamental, paramount and supreme law of the
nation," is deemed written in the law.218 Hence, the due process clause,219 which, so Tañada held,
mandates the publication of statutes, is read into Section 8 of E.O. No. 279. Additionally, Section 1 of
E.O. No. 200 which provides for publication "either in the Official Gazette or in a newspaper of
general circulation in the Philippines," finds suppletory application. It is significant to note that E.O.
No. 279 was actually published in the Official Gazette220 on August 3, 1987.

From a reading then of Section 8 of E.O. No. 279, Section 1 of E.O. No. 200, and Tañada v. Tuvera,
this Court holds that E.O. No. 279 became effective immediately upon its publication in the Official
Gazette on August 3, 1987.

That such effectivity took place after the convening of the first Congress is irrelevant. At the time
President Aquino issued E.O. No. 279 on July 25, 1987, she was still validly exercising legislative
powers under the Provisional Constitution.221 Article XVIII (Transitory Provisions) of the 1987
Constitution explicitly states:

Sec. 6. The incumbent President shall continue to exercise legislative powers until the first Congress
is convened.

The convening of the first Congress merely precluded the exercise of legislative powers by President
Aquino; it did not prevent the effectivity of laws she had previously enacted.

There can be no question, therefore, that E.O. No. 279 is an effective, and a validly enacted, statute.
62

THE CONSTITUTIONALITY OF THE WMCP FTAA

Petitioners submit that, in accordance with the text of Section 2, Article XII of the Constitution, FTAAs
should be limited to "technical or financial assistance" only. They observe, however, that, contrary to
the language of the Constitution, the WMCP FTAA allows WMCP, a fully foreign-owned mining
corporation, to extend more than mere financial or technical assistance to the State, for it permits
WMCP to manage and operate every aspect of the mining activity. 222

Petitioners' submission is well-taken. It is a cardinal rule in the interpretation of constitutions that the
instrument must be so construed as to give effect to the intention of the people who adopted it. 223 This
intention is to be sought in the constitution itself, and the apparent meaning of the words is to be
taken as expressing it, except in cases where that assumption would lead to absurdity, ambiguity, or
contradiction.224 What the Constitution says according to the text of the provision, therefore, compels
acceptance and negates the power of the courts to alter it, based on the postulate that the framers
and the people mean what they say.225 Accordingly, following the literal text of the Constitution,
assistance accorded by foreign-owned corporations in the large-scale exploration, development, and
utilization of petroleum, minerals and mineral oils should be limited to "technical" or "financial"
assistance only.

WMCP nevertheless submits that the word "technical" in the fourth paragraph of Section 2 of E.O. No.
279 encompasses a "broad number of possible services," perhaps, "scientific and/or technological in
basis."226 It thus posits that it may also well include "the area of management or operations . . . so
long as such assistance requires specialized knowledge or skills, and are related to the exploration,
development and utilization of mineral resources."227

This Court is not persuaded. As priorly pointed out, the phrase "management or other forms of
assistance" in the 1973 Constitution was deleted in the 1987 Constitution, which allows only
"technical or financial assistance." Casus omisus pro omisso habendus est. A person, object or thing
omitted from an enumeration must be held to have been omitted intentionally.228 As will be shown
later, the management or operation of mining activities by foreign contractors, which is the primary
feature of service contracts, was precisely the evil that the drafters of the 1987 Constitution sought to
eradicate.

Respondents insist that "agreements involving technical or financial assistance" is just another term
for service contracts. They contend that the proceedings of the CONCOM indicate "that although the
terminology 'service contract' was avoided [by the Constitution], the concept it represented was not."
They add that "[t]he concept is embodied in the phrase 'agreements involving financial or technical
assistance.'"229 And point out how members of the CONCOM referred to these agreements as
"service contracts." For instance:

SR. TAN. Am I correct in thinking that the only difference between these future service
contracts and the past service contracts under Mr. Marcos is the general law to be enacted by
the legislature and the notification of Congress by the President? That is the only difference,
is it not?

MR. VILLEGAS. That is right.

SR. TAN. So those are the safeguards[?]

MR. VILLEGAS. Yes. There was no law at all governing service contracts before.

SR. TAN. Thank you, Madam President.230 [Emphasis supplied.]

WMCP also cites the following statements of Commissioners Gascon, Garcia, Nolledo and
Tadeo who alluded to service contracts as they explained their respective votes in the
approval of the draft Article:
63

MR. GASCON. Mr. Presiding Officer, I vote no primarily because of two reasons: One, the
provision on service contracts. I felt that if we would constitutionalize any provision on service
contracts, this should always be with the concurrence of Congress and not guided only by a
general law to be promulgated by Congress. x x x.231 [Emphasis supplied.]

x x x.

MR. GARCIA. Thank you.

I vote no. x x x.

Service contracts are given constitutional legitimization in Section 3, even when they have
been proven to be inimical to the interests of the nation, providing as they do the legal
loophole for the exploitation of our natural resources for the benefit of foreign interests. They
constitute a serious negation of Filipino control on the use and disposition of the nation's
natural resources, especially with regard to those which are nonrenewable.232[Emphasis
supplied.]

xxx

MR. NOLLEDO. While there are objectionable provisions in the Article on National Economy
and Patrimony, going over said provisions meticulously, setting aside prejudice and
personalities will reveal that the article contains a balanced set of provisions. I hope the
forthcoming Congress will implement such provisions taking into account that Filipinos should
have real control over our economy and patrimony, and if foreign equity is permitted, the
same must be subordinated to the imperative demands of the national interest.

x x x.

It is also my understanding that service contracts involving foreign corporations or entities are
resorted to only when no Filipino enterprise or Filipino-controlled enterprise could possibly
undertake the exploration or exploitation of our natural resources and that compensation
under such contracts cannot and should not equal what should pertain to ownership of
capital. In other words, the service contract should not be an instrument to circumvent the
basic provision, that the exploration and exploitation of natural resources should be truly for
the benefit of Filipinos.

Thank you, and I vote yes.233 [Emphasis supplied.]

x x x.

MR. TADEO. Nais ko lamang ipaliwanag ang aking boto.

Matapos suriin ang kalagayan ng Pilipinas, ang saligang suliranin, pangunahin ang salitang
"imperyalismo." Ang ibig sabihin nito ay ang sistema ng lipunang pinaghaharian ng iilang
monopolyong kapitalista at ang salitang "imperyalismo" ay buhay na buhay sa National
Economy and Patrimony na nating ginawa. Sa pamamagitan ng salitang "based on,"
naroroon na ang free trade sapagkat tayo ay mananatiling tagapagluwas ng hilaw na
sangkap at tagaangkat ng yaring produkto. Pangalawa, naroroon pa rin ang parity rights, ang
service contract, ang 60-40 equity sa natural resources. Habang naghihirap ang
sambayanang Pilipino, ginagalugad naman ng mga dayuhan ang ating likas na yaman.
Kailan man ang Article on National Economy and Patrimony ay hindi nagpaalis sa pagkaalipin
ng ating ekonomiya sa kamay ng mga dayuhan. Ang solusyon sa suliranin ng bansa ay
dalawa lamang: ang pagpapatupad ng tunay na reporma sa lupa at ang national
industrialization. Ito ang tinatawag naming pagsikat ng araw sa Silangan. Ngunit ang mga
landlords and big businessmen at ang mga komprador ay nagsasabi na ang free trade na ito,
ang kahulugan para sa amin, ay ipinipilit sa ating sambayanan na ang araw ay sisikat sa
64

Kanluran. Kailan man hindi puwedeng sumikat ang araw sa Kanluran. I vote no. 234 [Emphasis
supplied.]

This Court is likewise not persuaded.

As earlier noted, the phrase "service contracts" has been deleted in the 1987 Constitution's Article on
National Economy and Patrimony. If the CONCOM intended to retain the concept of service contracts
under the 1973 Constitution, it could have simply adopted the old terminology ("service contracts")
instead of employing new and unfamiliar terms ("agreements . . . involving either technical or financial
assistance"). Such a difference between the language of a provision in a revised constitution and that
of a similar provision in the preceding constitution is viewed as indicative of a difference in
purpose.235 If, as respondents suggest, the concept of "technical or financial assistance" agreements
is identical to that of "service contracts," the CONCOM would not have bothered to fit the same dog
with a new collar. To uphold respondents' theory would reduce the first to a mere euphemism for the
second and render the change in phraseology meaningless.

An examination of the reason behind the change confirms that technical or financial assistance
agreements are not synonymous to service contracts.

[T]he Court in construing a Constitution should bear in mind the object sought to be accomplished by
its adoption, and the evils, if any, sought to be prevented or remedied. A doubtful provision will be
examined in light of the history of the times, and the condition and circumstances under which the
Constitution was framed. The object is to ascertain the reason which induced the framers of the
Constitution to enact the particular provision and the purpose sought to be accomplished thereby, in
order to construe the whole as to make the words consonant to that reason and calculated to effect
that purpose.236

As the following question of Commissioner Quesada and Commissioner Villegas' answer shows the
drafters intended to do away with service contracts which were used to circumvent the capitalization
(60%-40%) requirement:

MS. QUESADA. The 1973 Constitution used the words "service contracts." In this particular
Section 3, is there a safeguard against the possible control of foreign interests if the Filipinos
go into coproduction with them?

MR. VILLEGAS. Yes. In fact, the deletion of the phrase "service contracts" was our first
attempt to avoid some of the abuses in the past regime in the use of service contracts to go
around the 60-40 arrangement. The safeguard that has been introduced – and this, of course
can be refined – is found in Section 3, lines 25 to 30, where Congress will have to concur with
the President on any agreement entered into between a foreign-owned corporation and the
government involving technical or financial assistance for large-scale exploration,
development and utilization of natural resources.237 [Emphasis supplied.]

In a subsequent discussion, Commissioner Villegas allayed the fears of Commissioner


Quesada regarding the participation of foreign interests in Philippine natural resources, which
was supposed to be restricted to Filipinos.

MS. QUESADA. Another point of clarification is the phrase "and utilization of natural
resources shall be under the full control and supervision of the State." In the 1973
Constitution, this was limited to citizens of the Philippines; but it was removed and substituted
by "shall be under the full control and supervision of the State." Was the concept changed so
that these particular resources would be limited to citizens of the Philippines? Or would these
resources only be under the full control and supervision of the State; meaning, noncitizens
would have access to these natural resources? Is that the understanding?

MR. VILLEGAS. No, Mr. Vice-President, if the Commissioner reads the next sentence, it
states:
65

Such activities may be directly undertaken by the State, or it may enter into co-production, joint
venture, production-sharing agreements with Filipino citizens.

So we are still limiting it only to Filipino citizens.

x x x.

MS. QUESADA. Going back to Section 3, the section suggests that:

The exploration, development, and utilization of natural resources… may be directly undertaken by
the State, or it may enter into co-production, joint venture or production-sharing agreement with . . .
corporations or associations at least sixty per cent of whose voting stock or controlling interest is
owned by such citizens.

Lines 25 to 30, on the other hand, suggest that in the large-scale exploration, development and
utilization of natural resources, the President with the concurrence of Congress may enter into
agreements with foreign-owned corporations even for technical or financial assistance.

I wonder if this part of Section 3 contradicts the second part. I am raising this point for fear that foreign
investors will use their enormous capital resources to facilitate the actual exploitation or exploration,
development and effective disposition of our natural resources to the detriment of Filipino investors. I
am not saying that we should not consider borrowing money from foreign sources. What I refer to is
that foreign interest should be allowed to participate only to the extent that they lend us money and
give us technical assistance with the appropriate government permit. In this way, we can insure the
enjoyment of our natural resources by our own people.

MR. VILLEGAS. Actually, the second provision about the President does not permit foreign investors
to participate. It is only technical or financial assistance – they do not own anything – but on
conditions that have to be determined by law with the concurrence of Congress. So, it is very
restrictive.

If the Commissioner will remember, this removes the possibility for service contracts which we said
yesterday were avenues used in the previous regime to go around the 60-40
requirement.238 [Emphasis supplied.]

The present Chief Justice, then a member of the CONCOM, also referred to this limitation in scope in
proposing an amendment to the 60-40 requirement:

MR. DAVIDE. May I be allowed to explain the proposal?

MR. MAAMBONG. Subject to the three-minute rule, Madam President.

MR. DAVIDE. It will not take three minutes.

The Commission had just approved the Preamble. In the Preamble we clearly stated that the Filipino
people are sovereign and that one of the objectives for the creation or establishment of a government
is to conserve and develop the national patrimony. The implication is that the national patrimony or
our natural resources are exclusively reserved for the Filipino people. No alien must be allowed to
enjoy, exploit and develop our natural resources. As a matter of fact, that principle proceeds from the
fact that our natural resources are gifts from God to the Filipino people and it would be a breach of
that special blessing from God if we will allow aliens to exploit our natural resources.

I voted in favor of the Jamir proposal because it is not really exploitation that we granted to the alien
corporations but only for them to render financial or technical assistance. It is not for them to enjoy our
natural resources. Madam President, our natural resources are depleting; our population is increasing
by leaps and bounds. Fifty years from now, if we will allow these aliens to exploit our natural
resources, there will be no more natural resources for the next generations of Filipinos. It may last
66

long if we will begin now. Since 1935 the aliens have been allowed to enjoy to a certain extent the
exploitation of our natural resources, and we became victims of foreign dominance and control. The
aliens are interested in coming to the Philippines because they would like to enjoy the bounty of
nature exclusively intended for Filipinos by God.

And so I appeal to all, for the sake of the future generations, that if we have to pray in the Preamble
"to preserve and develop the national patrimony for the sovereign Filipino people and for the
generations to come," we must at this time decide once and for all that our natural resources must be
reserved only to Filipino citizens.

Thank you.239 [Emphasis supplied.]

The opinion of another member of the CONCOM is persuasive240 and leaves no doubt as to the
intention of the framers to eliminate service contracts altogether. He writes:

Paragraph 4 of Section 2 specifies large-scale, capital-intensive, highly technological undertakings for


which the President may enter into contracts with foreign-owned corporations, and enunciates strict
conditions that should govern such contracts. x x x.

This provision balances the need for foreign capital and technology with the need to maintain the
national sovereignty. It recognizes the fact that as long as Filipinos can formulate their own terms in
their own territory, there is no danger of relinquishing sovereignty to foreign interests.

Are service contracts allowed under the new Constitution? No. Under the new Constitution, foreign
investors (fully alien-owned) can NOT participate in Filipino enterprises except to provide: (1)
Technical Assistance for highly technical enterprises; and (2) Financial Assistance for large-scale
enterprises.

The intent of this provision, as well as other provisions on foreign investments, is to prevent the
practice (prevalent in the Marcos government) of skirting the 60/40 equation using the cover of
service contracts.241 [Emphasis supplied.]

Furthermore, it appears that Proposed Resolution No. 496,242 which was the draft Article on National
Economy and Patrimony, adopted the concept of "agreements . . . involving either technical or
financial assistance" contained in the "Draft of the 1986 U.P. Law Constitution Project" (U.P. Law
draft) which was taken into consideration during the deliberation of the CONCOM. 243 The former, as
well as Article XII, as adopted, employed the same terminology, as the comparative table below
shows:

DRAFT OF THE UP PROPOSED ARTICLE XII OF THE


LAW CONSTITUTION RESOLUTION NO. 496 1987 CONSTITUTION
PROJECT OF THE
CONSTITUTIONAL
COMMISSION

Sec. 1. All lands of the Sec. 3. All lands of the Sec. 2. All lands of the
public domain, waters, public domain, waters, public domain, waters,
minerals, coal, minerals, coal, minerals, coal,
petroleum and other petroleum and other petroleum, and other
mineral oils, all forces of mineral oils, all forces of mineral oils, all forces of
potential energy, potential energy, potential energy,
fisheries, flora and fauna fisheries, forests, flora fisheries, forests or
67

and other natural and fauna, and other timber, wildlife, flora and
resources of the natural resources are fauna, and other natural
Philippines are owned owned by the State. resources are owned by
by the State. With the With the exception of the State. With the
exception of agricultural agricultural lands, all exception of agricultural
lands, all other natural other natural resources lands, all other natural
resources shall not be shall not be alienated. resources shall not be
alienated. The The exploration, alienated. The
exploration, development, and exploration,
development and utilization of natural development, and
utilization of natural resources shall be under utilization of natural
resources shall be under the full control and resources shall be under
the full control and supervision of the State. the full control and
supervision of the State. Such activities may be supervision of the State.
Such activities may be directly undertaken by The State may directly
directly undertaken by the State, or it may enter undertake such activities
the state, or it may enter into co-production, joint or it may enter into co-
into co-production, joint venture, production- production, joint venture,
venture, production sharing agreements with or production-sharing
sharing agreements with Filipino citizens or agreements with Filipino
Filipino citizens or corporations or citizens, or corporations
corporations or associations at least or associations at least
associations sixty per sixty per cent of whose sixty per centum of
cent of whose voting voting stock or whose capital is owned
stock or controlling controlling interest is by such citizens. Such
interest is owned by owned by such citizens. agreements may be for
such citizens for a Such agreements shall a period not exceeding
period of not more than be for a period of twenty-five years,
twenty-five years, twenty-five years, renewable for not more
renewable for not more renewable for not more than twenty-five years,
than twenty-five years than twenty-five years, and under such terms
and under such terms and under such term and conditions as may
and conditions as may and conditions as may be provided by law. In
be provided by law. In be provided by law. In case of water rights for
case as to water rights cases of water rights for irrigation, water supply,
for irrigation, water irrigation, water supply, fisheries, or industrial
supply, fisheries, or fisheries or industrial uses other than the
industrial uses other uses other than the development of water
than the development of development for water power, beneficial use
water power, beneficial power, beneficial use may be the measure
use may be the measure may be the measure and limit of the grant.
and limit of the grant. and limit of the grant.
The State shall protect
The National Assembly The Congress may by the nation's marine
may by law allow small law allow small-scale wealth in its archipelagic
scale utilization of utilization of natural waters, territorial sea,
natural resources by resources by Filipino and exclusive economic
Filipino citizens. citizens, as well as zone, and reserve its
cooperative fish farming use and enjoyment
The National Assembly, in rivers, lakes, bays, exclusively to Filipino
may, by two-thirds vote and lagoons. citizens.
of all its members by
special law provide the The President with the The Congress may, by
terms and conditions concurrence of law, allow small-scale
under which a foreign- Congress, by special utilization of natural
owned corporation may law, shall provide the resources by Filipino
68

enter into agreements terms and conditions citizens, as well as


with the government under which a foreign- cooperative fish farming,
involving either owned corporation may with priority to
technical or financial enter into agreements subsistence fishermen
assistance for large- with the government and fish-workers in
scale exploration, involving either rivers, lakes, bays, and
development, or technical or financial lagoons.
utilization of natural assistance for large-
resources. [Emphasis scale exploration, The President may enter
supplied.] development, and into agreements with
utilization of natural foreign-owned
resources. [Emphasis corporations
supplied.] involving either
technical or financial
assistance for large-
scale exploration,
development, and
utilization of minerals,
petroleum, and other
mineral oils according to
the general terms and
conditions provided by
law, based on real
contributions to the
economic growth and
general welfare of the
country. In such
agreements, the State
shall promote the
development and use of
local scientific and
technical resources.
[Emphasis supplied.]

The President shall


notify the Congress of
every contract entered
into in accordance with
this provision, within
thirty days from its
execution.

The insights of the proponents of the U.P. Law draft are, therefore, instructive in interpreting the
phrase "technical or financial assistance."

In his position paper entitled Service Contracts: Old Wine in New Bottles?, Professor Pacifico A.
Agabin, who was a member of the working group that prepared the U.P. Law draft, criticized service
contracts for they "lodge exclusive management and control of the enterprise to the service
contractor, which is reminiscent of the old concession regime. Thus, notwithstanding the provision of
the Constitution that natural resources belong to the State, and that these shall not be alienated, the
service contract system renders nugatory the constitutional provisions cited." 244 He elaborates:

Looking at the Philippine model, we can discern the following vestiges of the concession regime, thus:
69

1. Bidding of a selected area, or leasing the choice of the area to the interested party and
then negotiating the terms and conditions of the contract; (Sec. 5, P.D. 87)

2. Management of the enterprise vested on the contractor, including operation of the field if
petroleum is discovered; (Sec. 8, P.D. 87)

3. Control of production and other matters such as expansion and development; (Sec. 8)

4. Responsibility for downstream operations – marketing, distribution, and processing may be


with the contractor (Sec. 8);

5. Ownership of equipment, machinery, fixed assets, and other properties remain with
contractor (Sec. 12, P.D. 87);

6. Repatriation of capital and retention of profits abroad guaranteed to the contractor (Sec.
13, P.D. 87); and

7. While title to the petroleum discovered may nominally be in the name of the government,
the contractor has almost unfettered control over its disposition and sale, and even the
domestic requirements of the country is relegated to a pro rata basis (Sec. 8).

In short, our version of the service contract is just a rehash of the old concession regime x x x. Some
people have pulled an old rabbit out of a magician's hat, and foisted it upon us as a new and different
animal.

The service contract as we know it here is antithetical to the principle of sovereignty over our natural
resources restated in the same article of the [1973] Constitution containing the provision for service
contracts. If the service contractor happens to be a foreign corporation, the contract would also run
counter to the constitutional provision on nationalization or Filipinization, of the exploitation of our
natural resources.245 [Emphasis supplied. Underscoring in the original.]

Professor Merlin M. Magallona, also a member of the working group, was harsher in his reproach of
the system:

x x x the nationalistic phraseology of the 1935 [Constitution] was retained by the [1973] Charter, but
the essence of nationalism was reduced to hollow rhetoric. The 1973 Charter still provided that the
exploitation or development of the country's natural resources be limited to Filipino citizens or
corporations owned or controlled by them. However, the martial-law Constitution allowed them, once
these resources are in their name, to enter into service contracts with foreign investors for financial,
technical, management, or other forms of assistance. Since foreign investors have the capital
resources, the actual exploitation and development, as well as the effective disposition, of the
country's natural resources, would be under their direction, and control, relegating the Filipino
investors to the role of second-rate partners in joint ventures.

Through the instrumentality of the service contract, the 1973 Constitution had legitimized at the
highest level of state policy that which was prohibited under the 1973 Constitution, namely: the
exploitation of the country's natural resources by foreign nationals. The drastic impact of [this]
constitutional change becomes more pronounced when it is considered that the active party to any
service contract may be a corporation wholly owned by foreign interests. In such a case, the
citizenship requirement is completely set aside, permitting foreign corporations to obtain actual
possession, control, and [enjoyment] of the country's natural resources.246 [Emphasis supplied.]

Accordingly, Professor Agabin recommends that:

Recognizing the service contract for what it is, we have to expunge it from the Constitution and
reaffirm ownership over our natural resources. That is the only way we can exercise effective control
over our natural resources.
70

This should not mean complete isolation of the country's natural resources from foreign investment.
Other contract forms which are less derogatory to our sovereignty and control over natural resources
– like technical assistance agreements, financial assistance [agreements], co-production agreements,
joint ventures, production-sharing – could still be utilized and adopted without violating constitutional
provisions. In other words, we can adopt contract forms which recognize and assert our sovereignty
and ownership over natural resources, and where the foreign entity is just a pure contractor instead of
the beneficial owner of our economic resources.247 [Emphasis supplied.]

Still another member of the working group, Professor Eduardo Labitag, proposed that:

2. Service contracts as practiced under the 1973 Constitution should be discouraged, instead the
government may be allowed, subject to authorization by special law passed by an extraordinary
majority to enter into either technical or financial assistance. This is justified by the fact that as
presently worded in the 1973 Constitution, a service contract gives full control over the contract area
to the service contractor, for him to work, manage and dispose of the proceeds or production. It was a
subterfuge to get around the nationality requirement of the constitution.248[Emphasis supplied.]

In the annotations on the proposed Article on National Economy and Patrimony, the U.P. Law draft
summarized the rationale therefor, thus:

5. The last paragraph is a modification of the service contract provision found in Section 9, Article XIV
of the 1973 Constitution as amended. This 1973 provision shattered the framework of nationalism in
our fundamental law (see Magallona, "Nationalism and its Subversion in the Constitution"). Through
the service contract, the 1973 Constitution had legitimized that which was prohibited under the 1935
constitution—the exploitation of the country's natural resources by foreign nationals. Through the
service contract, acts prohibited by the Anti-Dummy Law were recognized as legitimate
arrangements. Service contracts lodge exclusive management and control of the enterprise to the
service contractor, not unlike the old concession regime where the concessionaire had complete
control over the country's natural resources, having been given exclusive and plenary rights to exploit
a particular resource and, in effect, having been assured of ownership of that resource at the point of
extraction (see Agabin, "Service Contracts: Old Wine in New Bottles"). Service contracts, hence, are
antithetical to the principle of sovereignty over our natural resources, as well as the constitutional
provision on nationalization or Filipinization of the exploitation of our natural resources.

Under the proposed provision, only technical assistance or financial assistance agreements may be
entered into, and only for large-scale activities. These are contract forms which recognize and assert
our sovereignty and ownership over natural resources since the foreign entity is just a pure contractor
and not a beneficial owner of our economic resources. The proposal recognizes the need for capital
and technology to develop our natural resources without sacrificing our sovereignty and control over
such resources by the safeguard of a special law which requires two-thirds vote of all the members of
the Legislature. This will ensure that such agreements will be debated upon exhaustively and
thoroughly in the National Assembly to avert prejudice to the nation.249 [Emphasis supplied.]

The U.P. Law draft proponents viewed service contracts under the 1973 Constitution as grants of
beneficial ownership of the country's natural resources to foreign owned corporations. While, in
theory, the State owns these natural resources – and Filipino citizens, their beneficiaries – service
contracts actually vested foreigners with the right to dispose, explore for, develop, exploit, and utilize
the same. Foreigners, not Filipinos, became the beneficiaries of Philippine natural resources. This
arrangement is clearly incompatible with the constitutional ideal of nationalization of natural
resources, with the Regalian doctrine, and on a broader perspective, with Philippine sovereignty.

The proponents nevertheless acknowledged the need for capital and technical know-how in the large-
scale exploitation, development and utilization of natural resources – the second paragraph of the
proposed draft itself being an admission of such scarcity. Hence, they recommended a compromise to
reconcile the nationalistic provisions dating back to the 1935 Constitution, which reserved all natural
resources exclusively to Filipinos, and the more liberal 1973 Constitution, which allowed foreigners to
participate in these resources through service contracts. Such a compromise called for the adoption
of a new system in the exploration, development, and utilization of natural resources in the form of
71

technical agreements or financial agreements which, necessarily, are distinct concepts from service
contracts.

The replacement of "service contracts" with "agreements… involving either technical or financial
assistance," as well as the deletion of the phrase "management or other forms of assistance,"
assumes greater significance when note is taken that the U.P. Law draft proposed other equally
crucial changes that were obviously heeded by the CONCOM. These include the abrogation of the
concession system and the adoption of new "options" for the State in the exploration, development,
and utilization of natural resources. The proponents deemed these changes to be more consistent
with the State's ownership of, and its "full control and supervision" (a phrase also employed by the
framers) over, such resources. The Project explained:

3. In line with the State ownership of natural resources, the State should take a more active role in the
exploration, development, and utilization of natural resources, than the present practice of granting
licenses, concessions, or leases – hence the provision that said activities shall be under the full
control and supervision of the State. There are three major schemes by which the State could
undertake these activities: first, directly by itself; second, by virtue of co-production, joint venture,
production sharing agreements with Filipino citizens or corporations or associations sixty per cent
(60%) of the voting stock or controlling interests of which are owned by such citizens; or third, with a
foreign-owned corporation, in cases of large-scale exploration, development, or utilization of natural
resources through agreements involving either technical or financial assistance only. x x x.

At present, under the licensing concession or lease schemes, the government benefits from such
benefits only through fees, charges, ad valorem taxes and income taxes of the exploiters of our
natural resources. Such benefits are very minimal compared with the enormous profits reaped by
theses licensees, grantees, concessionaires. Moreover, some of them disregard the conservation of
natural resources and do not protect the environment from degradation. The proposed role of the
State will enable it to a greater share in the profits – it can also actively husband its natural resources
and engage in developmental programs that will be beneficial to them.

4. Aside from the three major schemes for the exploration, development, and utilization of our natural
resources, the State may, by law, allow Filipino citizens to explore, develop, utilize natural resources
in small-scale. This is in recognition of the plight of marginal fishermen, forest dwellers, gold panners,
and others similarly situated who exploit our natural resources for their daily sustenance and
survival.250

Professor Agabin, in particular, after taking pains to illustrate the similarities between the two systems,
concluded that the service contract regime was but a "rehash" of the concession system. "Old wine in
new bottles," as he put it. The rejection of the service contract regime, therefore, is in consonance
with the abolition of the concession system.

In light of the deliberations of the CONCOM, the text of the Constitution, and the adoption of other
proposed changes, there is no doubt that the framers considered and shared the intent of the U.P.
Law proponents in employing the phrase "agreements . . . involving either technical or financial
assistance."

While certain commissioners may have mentioned the term "service contracts" during the CONCOM
deliberations, they may not have been necessarily referring to the concept of service contracts under
the 1973 Constitution. As noted earlier, "service contracts" is a term that assumes different meanings
to different people.251 The commissioners may have been using the term loosely, and not in its
technical and legal sense, to refer, in general, to agreements concerning natural resources entered
into by the Government with foreign corporations. These loose statements do not necessarily
translate to the adoption of the 1973 Constitution provision allowing service contracts.

It is true that, as shown in the earlier quoted portions of the proceedings in CONCOM, in response to
Sr. Tan's question, Commissioner Villegas commented that, other than congressional notification, the
only difference between "future" and "past" "service contracts" is the requirement of a general law as
there were no laws previously authorizing the same.252 However, such remark is far outweighed by
72

his more categorical statement in his exchange with Commissioner Quesada that the draft article
"does not permit foreign investors to participate" in the nation's natural resources – which was exactly
what service contracts did – except to provide "technical or financial assistance."253

In the case of the other commissioners, Commissioner Nolledo himself clarified in his work that the
present charter prohibits service contracts.254 Commissioner Gascon was not totally averse to foreign
participation, but favored stricter restrictions in the form of majority congressional concurrence. 255 On
the other hand, Commissioners Garcia and Tadeo may have veered to the extreme side of the
spectrum and their objections may be interpreted as votes against any foreign participation in our
natural resources whatsoever.

WMCP cites Opinion No. 75, s. 1987,256 and Opinion No. 175, s. 1990257 of the Secretary of Justice,
expressing the view that a financial or technical assistance agreement "is no different in concept" from
the service contract allowed under the 1973 Constitution. This Court is not, however, bound by this
interpretation. When an administrative or executive agency renders an opinion or issues a statement
of policy, it merely interprets a pre-existing law; and the administrative interpretation of the law is at
best advisory, for it is the courts that finally determine what the law means.258

In any case, the constitutional provision allowing the President to enter into FTAAs with foreign-owned
corporations is an exception to the rule that participation in the nation's natural resources is reserved
exclusively to Filipinos. Accordingly, such provision must be construed strictly against their enjoyment
by non-Filipinos. As Commissioner Villegas emphasized, the provision is "very
restrictive."259 Commissioner Nolledo also remarked that "entering into service contracts is an
exception to the rule on protection of natural resources for the interest of the nation and, therefore,
being an exception, it should be subject, whenever possible, to stringent rules." 260 Indeed, exceptions
should be strictly but reasonably construed; they extend only so far as their language fairly warrants
and all doubts should be resolved in favor of the general provision rather than the exception. 261

With the foregoing discussion in mind, this Court finds that R.A. No. 7942 is invalid insofar as said Act
authorizes service contracts. Although the statute employs the phrase "financial and technical
agreements" in accordance with the 1987 Constitution, it actually treats these agreements as service
contracts that grant beneficial ownership to foreign contractors contrary to the fundamental law.

Section 33, which is found under Chapter VI (Financial or Technical Assistance Agreement) of R.A.
No. 7942 states:

SEC. 33. Eligibility.—Any qualified person with technical and financial capability to undertake large-
scale exploration, development, and utilization of mineral resources in the Philippines may enter into a
financial or technical assistance agreement directly with the Government through the Department.
[Emphasis supplied.]

"Exploration," as defined by R.A. No. 7942,

means the searching or prospecting for mineral resources by geological, geochemical or geophysical
surveys, remote sensing, test pitting, trending, drilling, shaft sinking, tunneling or any other means for
the purpose of determining the existence, extent, quantity and quality thereof and the feasibility of
mining them for profit.262

A legally organized foreign-owned corporation may be granted an exploration permit,263 which vests it
with the right to conduct exploration for all minerals in specified areas,264 i.e., to enter, occupy and
explore the same.265Eventually, the foreign-owned corporation, as such permittee, may apply for a
financial and technical assistance agreement.266

"Development" is the work undertaken to explore and prepare an ore body or a mineral deposit for
mining, including the construction of necessary infrastructure and related facilities. 267

"Utilization" "means the extraction or disposition of minerals." 268 A stipulation that the proponent shall
dispose of the minerals and byproducts produced at the highest price and more advantageous terms
73

and conditions as provided for under the implementing rules and regulations is required to be
incorporated in every FTAA.269

A foreign-owned/-controlled corporation may likewise be granted a mineral processing


permit.270 "Mineral processing" is the milling, beneficiation or upgrading of ores or minerals and rocks
or by similar means to convert the same into marketable products.271

An FTAA contractor makes a warranty that the mining operations shall be conducted in accordance
with the provisions of R.A. No. 7942 and its implementing rules 272 and for work programs and
minimum expenditures and commitments.273 And it obliges itself to furnish the Government records of
geologic, accounting, and other relevant data for its mining operation.274

"Mining operation," as the law defines it, means mining activities involving exploration, feasibility,
development, utilization, and processing.275

The underlying assumption in all these provisions is that the foreign contractor manages the mineral
resources, just like the foreign contractor in a service contract.

Furthermore, Chapter XII of the Act grants foreign contractors in FTAAs the same auxiliary mining
rights that it grants contractors in mineral agreements (MPSA, CA and JV).276 Parenthetically,
Sections 72 to 75 use the term "contractor," without distinguishing between FTAA and mineral
agreement contractors. And so does "holders of mining rights" in Section 76. A foreign contractor may
even convert its FTAA into a mineral agreement if the economic viability of the contract area is found
to be inadequate to justify large-scale mining operations,277 provided that it reduces its equity in the
corporation, partnership, association or cooperative to forty percent (40%).278

Finally, under the Act, an FTAA contractor warrants that it "has or has access to all the financing,
managerial, and technical expertise. . . ."279 This suggests that an FTAA contractor is bound to
provide some management assistance – a form of assistance that has been eliminated and,
therefore, proscribed by the present Charter.

By allowing foreign contractors to manage or operate all the aspects of the mining operation, the
above-cited provisions of R.A. No. 7942 have in effect conveyed beneficial ownership over the
nation's mineral resources to these contractors, leaving the State with nothing but bare title thereto.

Moreover, the same provisions, whether by design or inadvertence, permit a circumvention of the
constitutionally ordained 60%-40% capitalization requirement for corporations or associations
engaged in the exploitation, development and utilization of Philippine natural resources.

In sum, the Court finds the following provisions of R.A. No. 7942 to be violative of Section 2, Article
XII of the Constitution:

(1) The proviso in Section 3 (aq), which defines "qualified person," to wit:

Provided, That a legally organized foreign-owned corporation shall be deemed a qualified


person for purposes of granting an exploration permit, financial or technical assistance
agreement or mineral processing permit.

(2) Section 23,280 which specifies the rights and obligations of an exploration permittee,
insofar as said section applies to a financial or technical assistance agreement,

(3) Section 33, which prescribes the eligibility of a contractor in a financial or technical
assistance agreement;

(4) Section 35,281 which enumerates the terms and conditions for every financial or technical
assistance agreement;
74

(5) Section 39,282 which allows the contractor in a financial and technical assistance
agreement to convert the same into a mineral production-sharing agreement;

(6) Section 56,283 which authorizes the issuance of a mineral processing permit to a
contractor in a financial and technical assistance agreement;

The following provisions of the same Act are likewise void as they are dependent on the foregoing
provisions and cannot stand on their own:

(1) Section 3 (g),284 which defines the term "contractor," insofar as it applies to a financial or
technical assistance agreement.

Section 34,285 which prescribes the maximum contract area in a financial or technical
assistance agreements;

Section 36,286 which allows negotiations for financial or technical assistance agreements;

Section 37,287 which prescribes the procedure for filing and evaluation of financial or technical
assistance agreement proposals;

Section 38,288 which limits the term of financial or technical assistance agreements;

Section 40,289 which allows the assignment or transfer of financial or technical assistance
agreements;

Section 41,290 which allows the withdrawal of the contractor in an FTAA;

The second and third paragraphs of Section 81,291 which provide for the Government's share
in a financial and technical assistance agreement; and

Section 90,292 which provides for incentives to contractors in FTAAs insofar as it applies to
said contractors;

When the parts of the statute are so mutually dependent and connected as conditions,
considerations, inducements, or compensations for each other, as to warrant a belief that the
legislature intended them as a whole, and that if all could not be carried into effect, the legislature
would not pass the residue independently, then, if some parts are unconstitutional, all the provisions
which are thus dependent, conditional, or connected, must fall with them. 293

There can be little doubt that the WMCP FTAA itself is a service contract.

Section 1.3 of the WMCP FTAA grants WMCP "the exclusive right to explore, exploit, utilise[,] process
and dispose of all Minerals products and by-products thereof that may be produced from the Contract
Area."294 The FTAA also imbues WMCP with the following rights:

(b) to extract and carry away any Mineral samples from the Contract area for the purpose of
conducting tests and studies in respect thereof;

(c) to determine the mining and treatment processes to be utilised during the
Development/Operating Period and the project facilities to be constructed during the
Development and Construction Period;

(d) have the right of possession of the Contract Area, with full right of ingress and egress and
the right to occupy the same, subject to the provisions of Presidential Decree No. 512 (if
applicable) and not be prevented from entry into private ands by surface owners and/or
occupants thereof when prospecting, exploring and exploiting for minerals therein;
75

xxx

(f) to construct roadways, mining, drainage, power generation and transmission facilities and
all other types of works on the Contract Area;

(g) to erect, install or place any type of improvements, supplies, machinery and other
equipment relating to the Mining Operations and to use, sell or otherwise dispose of, modify,
remove or diminish any and all parts thereof;

(h) enjoy, subject to pertinent laws, rules and regulations and the rights of third Parties,
easement rights and the use of timber, sand, clay, stone, water and other natural resources in
the Contract Area without cost for the purposes of the Mining Operations;

xxx

(i) have the right to mortgage, charge or encumber all or part of its interest and obligations
under this Agreement, the plant, equipment and infrastructure and the Minerals produced
from the Mining Operations;

x x x. 295

All materials, equipment, plant and other installations erected or placed on the Contract Area remain
the property of WMCP, which has the right to deal with and remove such items within twelve months
from the termination of the FTAA.296

Pursuant to Section 1.2 of the FTAA, WMCP shall provide "[all] financing, technology, management
and personnel necessary for the Mining Operations." The mining company binds itself to "perform all
Mining Operations . . . providing all necessary services, technology and financing in connection
therewith,"297 and to "furnish all materials, labour, equipment and other installations that may be
required for carrying on all Mining Operations."298> WMCP may make expansions, improvements and
replacements of the mining facilities and may add such new facilities as it considers necessary for the
mining operations.299

These contractual stipulations, taken together, grant WMCP beneficial ownership over natural
resources that properly belong to the State and are intended for the benefit of its citizens. These
stipulations are abhorrent to the 1987 Constitution. They are precisely the vices that the fundamental
law seeks to avoid, the evils that it aims to suppress. Consequently, the contract from which they
spring must be struck down.

In arguing against the annulment of the FTAA, WMCP invokes the Agreement on the Promotion and
Protection of Investments between the Philippine and Australian Governments, which was signed in
Manila on January 25, 1995 and which entered into force on December 8, 1995.

x x x. Article 2 (1) of said treaty states that it applies to investments whenever made and thus the fact
that [WMCP's] FTAA was entered into prior to the entry into force of the treaty does not preclude the
Philippine Government from protecting [WMCP's] investment in [that] FTAA. Likewise, Article 3 (1) of
the treaty provides that "Each Party shall encourage and promote investments in its area by investors
of the other Party and shall [admit] such investments in accordance with its Constitution, Laws,
regulations and investment policies" and in Article 3 (2), it states that "Each Party shall ensure that
investments are accorded fair and equitable treatment." The latter stipulation indicates that it was
intended to impose an obligation upon a Party to afford fair and equitable treatment to the
investments of the other Party and that a failure to provide such treatment by or under the laws of the
Party may constitute a breach of the treaty. Simply stated, the Philippines could not, under said treaty,
rely upon the inadequacies of its own laws to deprive an Australian investor (like [WMCP]) of fair and
equitable treatment by invalidating [WMCP's] FTAA without likewise nullifying the service contracts
entered into before the enactment of RA 7942 such as those mentioned in PD 87 or EO 279.
76

This becomes more significant in the light of the fact that [WMCP's] FTAA was executed not by a
mere Filipino citizen, but by the Philippine Government itself, through its President no less, which, in
entering into said treaty is assumed to be aware of the existing Philippine laws on service contracts
over the exploration, development and utilization of natural resources. The execution of the FTAA by
the Philippine Government assures the Australian Government that the FTAA is in accordance with
existing Philippine laws.300 [Emphasis and italics by private respondents.]

The invalidation of the subject FTAA, it is argued, would constitute a breach of said treaty which, in
turn, would amount to a violation of Section 3, Article II of the Constitution adopting the generally
accepted principles of international law as part of the law of the land. One of these generally accepted
principles is pacta sunt servanda, which requires the performance in good faith of treaty obligations.

Even assuming arguendo that WMCP is correct in its interpretation of the treaty and its assertion that
"the Philippines could not . . . deprive an Australian investor (like [WMCP]) of fair and equitable
treatment by invalidating [WMCP's] FTAA without likewise nullifying the service contracts entered into
before the enactment of RA 7942 . . .," the annulment of the FTAA would not constitute a breach of
the treaty invoked. For this decision herein invalidating the subject FTAA forms part of the legal
system of the Philippines.301 The equal protection clause302 guarantees that such decision shall apply
to all contracts belonging to the same class, hence, upholding rather than violating, the "fair and
equitable treatment" stipulation in said treaty.

One other matter requires clarification. Petitioners contend that, consistent with the provisions of
Section 2, Article XII of the Constitution, the President may enter into agreements involving "either
technical or financial assistance" only. The agreement in question, however, is a technical and
financial assistance agreement.

Petitioners' contention does not lie. To adhere to the literal language of the Constitution would lead to
absurd consequences.303 As WMCP correctly put it:

x x x such a theory of petitioners would compel the government (through the President) to enter into
contract with two (2) foreign-owned corporations, one for financial assistance agreement and with the
other, for technical assistance over one and the same mining area or land; or to execute two (2)
contracts with only one foreign-owned corporation which has the capability to provide both financial
and technical assistance, one for financial assistance and another for technical assistance, over the
same mining area. Such an absurd result is definitely not sanctioned under the canons of
constitutional construction.304 [Underscoring in the original.]

Surely, the framers of the 1987 Charter did not contemplate such an absurd result from their use of
"either/or." A constitution is not to be interpreted as demanding the impossible or the impracticable;
and unreasonable or absurd consequences, if possible, should be avoided.305 Courts are not to give
words a meaning that would lead to absurd or unreasonable consequences and a literal interpretation
is to be rejected if it would be unjust or lead to absurd results. 306 That is a strong argument against its
adoption.307 Accordingly, petitioners' interpretation must be rejected.

The foregoing discussion has rendered unnecessary the resolution of the other issues raised by the
petition.

WHEREFORE, the petition is GRANTED. The Court hereby declares unconstitutional and void:

(1) The following provisions of Republic Act No. 7942:

(a) The proviso in Section 3 (aq),

(b) Section 23,

(c) Section 33 to 41,


77

(d) Section 56,

(e) The second and third paragraphs of Section 81, and

(f) Section 90.

(2) All provisions of Department of Environment and Natural Resources Administrative Order
96-40, s. 1996 which are not in conformity with this Decision, and

(3) The Financial and Technical Assistance Agreement between the Government of the
Republic of the Philippines and WMC Philippines, Inc.

SO ORDERED.

Davide, Jr., C.J., Puno, Quisumbing, Carpio, Corona, Callejo, Sr., and Tinga. JJ., concur.
Vitug, J., see Separate Opinion.
Panganiban, J., see Separate Opinion.
Ynares-Santiago, Sandoval-Gutierrez and Austria-Martinez, JJ., joins J., Panganiban's separate
opinion.
Azcuna, no part, one of the parties was a client.

Footnotes

1 Appears as "Nequito" in the caption of the Petition but "Nequinto" in the body. (Rollo, p. 12.)

2 As appears in the body of the Petition. (Id., at 13.) The caption of the petition does not
include Louel A. Peria as one of the petitioners but the name of his father Elpidio V. Peria
appears therein.

3Appears as "Kaisahan Tungo sa Kaunlaran ng Kanayunan at Repormang Pansakahan


(KAISAHAN)" in the caption of the Petition by "Philippine Kaisahan Tungo sa Kaunlaran ng
Kanayunan at Repormang Pansakahan (KAISAHAN)" in the body. (Id., at 14.)

4Erroneously designated in the Petition as "Western Mining Philippines Corporation." (Id., at


212.) Subsequently, WMC (Philippines), Inc. was renamed "Tampakan Mineral Resources
Corporation." (Id., at 778.)

5An Act Instituting A New System of Mineral Resources Exploration, Development, Utilization
and Conservation.

6Authorizing the Secretary of Environment and Natural Resources to Negotiate and Conclude
Joint Venture, Co-Production, or Production-Sharing Agreements for the Exploration,
Development and Utilization of Mineral Resources, and Prescribing the Guidelines for such
Agreements and those Agreements involving Technical or Financial Assistance by Foreign-
Owned Corporations for Large-Scale Exploration, Development and Utilization of Minerals.

7 Exec. Order No. 279 (1987), sec. 4.

8 Rep. Act No. 7942 (1995), sec. 15.


78

9 Id., sec. 26 (a)-(c).

10 Id., sec. 29.

11 Id., sec. 30.

12 Id., sec. 31.

13 Id., sec. 32.

14 Id., ch. VI.

15 Id., secs. 27 and 33 in relation to sec. 3 (aq).

16 Id., sec. 72.

17 Id., sec. 73.

18 Id., sec. 75.

19 Id., sec. 74.

20 Id., sec. 76.

21 Id., ch. XIII.

22 Id., secs. 20-22.

23 Id., secs. 43, 45.

24 Id., secs. 46-49, 51-52.

25 Id., ch. IX.

26 Id., ch. X.

27 Id., ch. XI.

28 Id., ch. XIV.

29 Id., ch. XV.

30 Id., ch. XVI.

31 Id., ch. XIX.

32 Id., ch. XVII.

33 Section 116, R.A. No. 7942 provides that the Act "shall take effect thirty (30) days following
its complete publication in two (2) newspapers of general circulation in the Philippines."

34 WMCP FTAA, sec. 4.1.

35 Rollo, p. 22.
79

36 Ibid.

37 Ibid.

38Ibid. The number has since risen to 129 applications when the petitioners filed their Reply.
(Rollo, p. 363.)

39 Id., at 22.

40 Id., at 23-24.

41 Id., at 52-53. Emphasis and underscoring supplied.

42 WMCP FTAA, p. 2.

43 Rollo, p. 220.

44 Id., at 754.

45 Vide Note 4.

46 Rollo, p. 754.

47 Id., at 755.

48 Id., at 761-763.

49 Id., at 764-776.

50 Id., at 782-786.

51 Docketed as C.A.-G. R. No. 74161.

52 G.R. No. 153885, entitled Lepanto Consolidated Mining Company v. WMC Resources
International Pty. Ltd., et al., decided September 24, 2003 and G.R. No. 156214, entitled
Lepanto Mining Company v. WMC Resources International Pty. Ltd., WMC (Philippines), Inc.,
Southcot Mining Corporation, Tampakan Mining Corporation and Sagittarius Mines, Inc.,
decided September 23, 2003.

53Section 12, Rule 43 of the Rules of Court, invoked by private respondent, states, " The
appeal shall not stay the award, judgment, final order or resolution sought to be reviewed
unless the Court of Appeals shall direct otherwise upon such terms as it may deem just."

54WMCP's Reply (dated May 6, 2003) to Petitioners' Comment (to the Manifestation and
Supplemental Manifestation), p. 3.

55 Ibid.

56 Ibid.

57WMCP's Reply (dated May 6, 2003) to Petitioners' Comment (to the Manifestation and
Supplemental Manifestation), p. 4.
80

58Philippine Constitution Association v. Enriquez, 235 SCRA 506 (1994); National Economic
Protectionism Association v. Ongpin, 171 SCRA 657 (1989); Dumlao v. COMELEC, 95 SCRA
392 (1980).

59 Dumlao v. COMELEC, supra.

60 Board of Optometry v. Colet, 260 SCRA 88 (1996).

61 Dumlao v. COMELEC, supra.

62 Subic Bay Metropolitan Authority v. Commission on Elections, 262 SCRA 492 (1996).

63 Angara v. Electoral Commission, 63 Phil. 139 (1936).

64
Integrated Bar of the Philippines v. Zamora, 338 SCRA 81, 100 (2000); Dumlao v.
COMELEC, supra; People v. Vera, 65 Phil. 56 (1937).

65 Dumlao v. COMELEC, supra.

66 Integrated Bar of the Philippines v. Zamora, supra.

67
Ermita-Malate Hotel and Motel Operators Association, Inc. v. City Mayor of Manila¸ 21
SCRA 449 (1967).

68Petitioners Roberto P. Amloy, Raqim L. Dabie, Simeon H. Dolojo, Imelda Gandon, Leny B.
Gusanan, Marcelo L. Gusanan, Quintol A. Labuayan, Lomingges Laway, and Benita P.
Tacuayan.

69Petitioners F'long Agutin M. Dabie, Mario L. Mangcal, Alden S. Tusan, Sr. Susuan O.
Bolanio, OND, Lolita G. Demonteverde, Benjie L. Nequinto, Rose Lilia S. Romano and
Amparo S. Yap.

70 Rollo, p. 6.

71 Id. at 337, citing Malabanan v. Gaw Ching, 181 SCRA 84 (1990).

72 246 SCRA 540 (1995).

73 People v. Vera, supra.

74 Militante v. Court of Appeals, 330 SCRA 318 (2000).

75 Ibid.

76 Cruz v. Secretary of Environment and Natural Resources, 347 SCRA 128 (2000), Kapunan,
J., Separate Opinion. [Emphasis supplied.]

77 Joya v. Presidential Commission on Good Government, 225 SCRA 568 (1993).

78 Integrated Bar of the Philippines v. Zamora, supra.

79 J. Bernas, S.J., The 1987 Constitution of the Philippines: A Commentary 1009 (1996).

80
Cruz v. Secretary of Environment and Natural Resources, supra, Kapunan, J., Separate
Opinion.
81

81 Id., Puno, J., Separate Opinion, and Panganiban, J., Separate Opinion.

82 Cariño v. Insular Government, 212 US 449, 53 L.Ed. 595 (1909). For instance, Law 14,
Title 12, Book 4 of the Recopilacion de Leyes de las Indias proclaimed:

We having acquired full sovereignty over the Indies, and all lands, territories, and
possessions not heretofore ceded away by our royal predecessors, or by us, or in our
name, still pertaining to the royal crown and patrimony, it is our will that all lands
which are held without proper and true deeds of grant be restored to us according as
they belong to us, in order that after reserving before all what to us or to our viceroys,
audiencias, and governors may seem necessary for public squares, ways, pastures,
and commons in those places which are peopled, taking into consideration not only
their present condition, but also their future and their probable increase, and after
distributing to the natives what may be necessary for tillage and pasturage,
confirming them in what they now have and giving them more if necessary, all the
rest of said lands may remain free and unencumbered for us to dispose of as we may
wish.

83Republic v. Court of Appeals, 160 SCRA 228 (1988). It has been noted, however, that "the
prohibition in the [1935] Constitution against alienation by the state of mineral lands and
minerals is not properly a part of the Regalian doctrine but a separate national policy
designed to conserve our mineral resources and prevent the state from being deprived of
such minerals as are essential to national defense." (A. Noblejas, Philippine Law on Natural
Resources 126-127 [1959 ed.], citing V. Francisco, The New Mining Law.)

84Cruz v. Secretary of Environment and Natural Resources, supra, Kapunan, J., Separate
Opinion, citing A. Noblejas, Philippine Law on Natural Resources 6 (1961). Noblejas
continues:

Thus, they asserted their right of ownership over mines and minerals or precious
metals, golds, and silver as distinct from the right of ownership of the land in which
the minerals were found. Thus, when on a piece of land mining was more valuable
than agriculture, the sovereign retained ownership of mines although the land has
been alienated to private ownership. Gradually, the right to the ownership of minerals
was extended to base metals. If the sovereign did not exploit the minerals, they grant
or sell it as a right separate from the land. (Id., at 6.)

85In the unpublished case of Lawrence v. Garduño (L-10942, quoted in V. Francisco,


Philippine Law on Natural Resources 14-15 [1956]), this Court observed:

The principle underlying Spanish legislation on mines is that these are subject to the
eminent domain of the state. The Spanish law of July 7, 1867, amended by the law of
March 4, 1868, in article 2 says: "The ownership of the substances enumerated in the
preceding article (among them those of inflammable nature), belong[s] to the state,
and they cannot be disposed of without the government authority."

The first Spanish mining law promulgated for these Islands (Decree of Superior Civil
Government of January 28, 1864), in its Article I, says: "The supreme ownership of
mines throughout the kingdom belong[s] to the crown and to the king. They shall not
be exploited except by persons who obtained special grant from this superior
government and by those who may secure it thereafter, subject to this regulation."

Article 2 of the royal decree on ownership of mines in the Philippine Islands, dated
May 14, 1867, which was the law in force at the time of the cession of these Islands
to the Government of the United States, says: "The ownership of the substances
enumerated in the preceding article (among them those of inflammable nature)
belongs to the state, and they cannot be disposed of without an authorization issued
by the Superior Civil Governor."
82

Furthermore, all those laws contained provisions regulating the manner of


prospecting, locating and exploring mines in private property by persons other than
the owner of the land as well as the granting of concessions, which goes to show that
private ownership of the land did not include, without express grant, the mines that
might be found therein.

Analogous provisions are found in the Civil Code of Spain determining the ownership
of mines. In its Article 339 (Article 420, New Civil Code) enumerating properties of
public ownership, the mines are included, until specially granted to private individuals.
In its article 350 (Art. 437, New Civil Code) declaring that the proprietor of any parcel
of land is the owner of its surface and of everything under it, an exception is made as
far as mining laws are concerned. Then in speaking of minerals, the Code in its
articles 426 and 427 (Art. 519, New Civil Code) provides rules governing the digging
of pits by third persons on private-owned lands for the purpose of prospecting for
minerals.

86 Atok Big-Wedge Mining Co. v. Intermediate Appellate Court, 261 SCRA 528 (1996).

87 Ibid.

88
Cruz v. Secretary of Environment and Natural Resources, supra, Kapunan, J., Separate
Opinion.

89 Ibid.

90 McDaniel v. Apacible and Cuisia, 42 Phil. 749 (1922).

91 Noblejas, supra, at 5.

92 V. M. A. Dimagiba, Service Contract Concepts in Energy, 57 Phil. L. J. 307, 313 (1982).

93P. A. Agabin, Service Contracts: Old Wine in New Bottles?, in II Draft Proposal of the 1986
U.P. Law Constitution Project 3.

94 Id., at 2-3.

95 Id., at 3.

96 Ibid.

97 Ibid.

98 Ibid.

99An Act to Provide for the Exploration, Location and Lease of Lands Containing Petroleum
and other Mineral Oils and Gas in the Philippine Islands.

100 An Act to Provide for the Leasing and Development of Coal Lands in the Philippine
Islands.

101 Agabin, supra, at 3.

102 People v. Linsangan, 62 Phil. 646 (1935).

103 Ibid.
83

104 Ibid.

105 Ibid.

106 Ibid.

107 Atok Big-Wedge Mining Co. v. Intermediate Appellate Court, supra.

108 Bernas, S.J., supra, at 1009-1010, citing Lee Hong Hok v. David, 48 SCRA 372 (1972).

109 II J. Aruego, The Framing of the Philippine Constitution 592 (1949).

110 Id., at 600-601.

111 Id., at 604. Delegate Aruego expounds:

At the time of the framing of the Philippine Constitution, Filipino capital had been
known to be rather shy. Filipinos hesitated as a general rule to invest a considerable
sum of their capital for the development, exploitation, and utilization of the natural
resources of the country. They had not as yet been so used to corporate enterprises
as the peoples of the West. This general apathy, the delegates knew, would mean
the retardation of the development of the natural resources, unless foreign capital
would be encouraged to come in and help in that development. They knew that the
nationalization of the natural resources would certainly not encourage the investment
of foreign capital into them. But there was a general feeling in the Convention that it
was better to have such development retarded or even postponed altogether until
such time when the Filipinos would be ready and willing to undertake it rather than
permit the natural resources to be placed under the ownership or control of foreigners
in order that they might be immediately developed, with the Filipinos of the future
serving not as owners but at most as tenants or workers under foreign masters. By all
means, the delegates believed, the natural resources should be conserved for
Filipino posterity.

The nationalization of natural resources was also intended as an instrument of


national defense. The Convention felt that to permit foreigner to own or control the
natural resources would be to weaken the national defense. It would be making
possible the gradual extension of foreign influence into our politics, thereby
increasing the possibility of foreign control. x x x.

Not only these. The nationalization of the natural resources, it was believed, would
prevent making the Philippines a source of international conflicts with the consequent
danger to its internal security and independence. For unless the natural resources
were nationalized, with the nationals of foreign countries having the opportunity to
own or control them, conflicts of interest among them might arise inviting danger to
the safety and independence of the nation. (Id., at 605-606.)

112
Palting v. San Jose Petroleum Inc., 18 SCRA 924 (1966); Republic v. Quasha, 46 SCRA
160 (1972).

113 Atok Big-Wedge Mining Co. v. Intermediate Appellate Court, supra.

114 Article VI thereof provided:

1. The disposition, exploitation, development and utilization of all agricultural, timber,


and mineral lands of the public domain, waters, minerals, coal, petroleum and other
mineral oils, all forces and sources of potential energy, and other natural resources of
either Party, and the operation of public utilities, shall, if open to any person, be open
84

to citizens of the other Party and to all forms of business enterprise owned or
controlled directly or indirectly, by citizens of such other Party in the same manner as
to and under the same conditions imposed upon citizens or corporations or
associations owned or controlled by citizens of the Party granting the right.

2. The rights provided for in Paragraph 1 may be exercised x x x in the case of


citizens of the United States, with respect to natural resources in the public domain in
the Philippines, only through the medium of a corporation organized under the laws of
the Philippines and at least 60% of the capital stock of which is owned or controlled
by citizens of the United States x x x.

3. The United States of America reserves the rights of the several States of the
United States to limit the extent to which citizens or corporations or associations
owned or controlled by citizens of the Philippines may engage in the activities
specified in this Article. The Republic of the Philippines reserves the power to deny
any of the rights specified in this Article to citizens of the United States who are
citizens of States, or to corporations or associations at least 60% of whose capital
stock or capital is owned or controlled by citizens of States, which deny like rights to
citizens of the Philippines, or to corporations or associations which ore owned or
controlled by citizens of the Philippines x x x.

115An Act to Promote the Exploration, Development, Exploitation, and Utilization of the
Petroleum Resources of the Philippines; to Encourage the Conservation of such Petroleum
Resources; to Authorize the Secretary of Agriculture and Natural Resources to Create an
Administration Unit and a Technical Board in the Bureau of Mines; to Appropriate Funds
therefor; and for other purposes.

116 Rep. Act No. 387 (1949), as amended, art. 10 (b).

117 Id., art. 10 (c).

118 Id., art. 5.

119
Id., art. 31. The same provision recognized the rights of American citizens under the Parity
Amendment:

During the effectivity and subject to the provisions of the ordinance appended to the
Constitution of the Philippines, citizens of the United States and all forms of business
enterprises owned and controlled, directly or indirectly, by citizens of the United
States shall enjoy the same rights and obligations under the provisions of this Act in
the same manner as to, and under the same conditions imposed upon, citizens of the
Philippines or corporations or associations owned or controlled by citizens of the
Philippines.

120 Id., art. 10.

121 Id., art. 3.

122 Id., art. 9.

123 Ibid.

124 Rep. Act No. 387 (1949), as amended, art. 8.

125 Id., art. 25.

126 Id., art. 47.


85

127 Id., art. 60.

128Id., art. 64. Article 49, R.A. No. 387 originally imposed an annual exploration tax on
exploration concessionaires but this provision was repealed by Section 1, R.A. No. 4304.

129 Francisco, supra, at 103.

130 Rep. Act No. 387 (1949), as amended, art. 65.

131 Francisco, supra, at 103.

132 Rep. Act No. 387 (1949), as amended, art. 90 (b) 3.

133 Id., art. 90 (b) 4.

134 Id., art. 93-A.

135 Id., art. 93.

136 Ibid.

137 Rep. Act No. 387 (1949), as amended, art. 94.

138 Id., art. 106.

139 Id., art. 95.

140 Ibid.

141 Rep. Act No. 387 (1949), as amended, art. 95 (e).

142Dimagiba, supra, at 315, citing Fabrikant, Oil Discovery and Technical Change in
Southeast Asia, Legal Aspects of Production Sharing Contracts in the Indonesian Petroleum
Industry, 101-102, sections 13C.24 and 13C.25 (1972).

143 Agabin, supra, at 4.

144 Dimagiba, supra, at 318.

145
Amending Presidential Decree No. 8 issued on October 2, 1972, and Promulgating an
Amended Act to Promote the Discovery and Production of Indigenous Petroleum and
Appropriate Funds Therefor.

146 Pres. Decree No. 87 (1972), sec. 4.

147 Agabin, supra, at 6.

148M. Magallona, Service Contracts in Philippine Natural Resources, 9 World Bull. 1, 4


(1993).

149 Pres. Decree No. 87 (1972), sec. 6.

150 Id., sec. 4.

151 Id., sec. 6.


86

152 Id., sec. 7.

153 Id., sec. 8.

154 Ibid.

155 Ibid.

156 Pres. Decree No. 87 (1972), sec. 9.

157 Id., sec. 12.

158 Id., sec. 13.

159Dimagiba draws the following comparison between the service contract scheme and the
concession system:

In both the concession system and the service contract scheme, work and financial
obligations are required of the developer. Under Republic Act No. 387 and
Presidential Decree No. 87, the concessionaire and the service contractors are
extracted certain taxes in favor of the government. In both arrangements, the
explorationist/developer is given incentives in the form of tax exemptions in the
importation or disposition of machinery, equipment, materials and spare parts needed
in petroleum operations.

The concessionaire and the service contractor are required to keep in their files
valuable data and information and may be required to submit need technological or
accounting reports to the Government. Duly authorized representatives of the
Government could, under the law, inspect or audit the books of accounts of the
contract holder.

In both systems, signature, discovery or production bonuses may be given by the


developer to the host Government.

The concession system, however, differs considerably from the service contract
system in important areas of the operations. In the concession system, the
Government merely receives fixed royalty which is a certain percentage of the crude
oil produced or other units of measure, regardless of whether the concession holder
makes profits or not. This is not so in the service contract system. A certain
percentage of the gross production is set aside for recoverable expenditures by the
contractor. Of the net proceeds the parties are entitled percentages of share that will
accrue to each of them.

In the royalty system, the concessionaire may be discouraged to produce more for
the reason that since the royalty paid to the host country is closely linked to the
volume of production, the greater the produce, the more amount or royalty would be
allocated to the Government. This is not so in the production sharing system. The
share of the Government depends largely on the net proceeds of production after
reimbursing the service contractor of its recoverable expenses.

As a general rule, the Government plays a passive role in the concession system,
more particularly, interested in receiving royalties from the concessionaire. In the
production-sharing arrangement, the Government plays a more active role in the
management and monitoring of oil operations and requires the service contractor
entertain obligations designed to bring more economic and technological benefits to
the host country. (Dimagiba, supra, at 330-331.)
87

160 Agabin, supra, at 6.

161
The antecedents leading to the Proclamation are narrated in Javellana v. Executive
Secretary, 50 SCRA 55 (1973):

On March 16, 1967, Congress of the Philippines passed Resolution No. 2, which was
amended by Resolution No. 4, of said body, adopted on June 17, 1969, calling a
convention to propose amendments to the Constitution of the Philippines. Said
Resolution No. 2, as amended, was implemented by Republic Act No. 6132 approved
on August 24, 1970, pursuant to the provisions of which the election of delegates to
said convention was held on November 10, 1970, and the 1971 Convention began to
perform its functions on June 1, 1971. While the Convention was in session on
September 21, 1972, the President issued Proclamation No. 1081 placing the entire
Philippines under Martial Law. On November 29, 1972, the President of the
Philippines issued Presidential Decree No. 73, submitting to the Filipino people for
ratification or rejection the Constitution of the Republic of the Philippines proposed by
the 1971 Constitutional Convention, and appropriating funds therefor, as well as
setting the plebiscite for such ratification on January 15, 1973.

On January 17, 1973, the President issued Proclamation No. 1102 certifying and
proclaiming that the Constitution proposed by the 1971 Constitutional Convention
"has been ratified by an overwhelming majority of all the votes cast by the members
of all the Barangays (Citizens Assemblies) throughout the Philippines, and has
thereby come into effect."

162 Bernas, S.J., supra, at 1016, Note 28, citing Session of November 25, 1972.

163
Agabin, supra, at 1, quoting Sanvictores, The Economic Provisions in the 1973
Constitution, in Espiritu, 1979 Philconsa Reader on Constitutional and Policy Issues 449.

164 Bernas, S.J., supra, at 1016, Note 28, citing Session of November 25, 1972.

165 Ibid.

166 Ibid.

167 Allowing Citizens of the Philippines or Corporations or Associations at least Sixty Per
Centum of the Capital of which is Owned by such Citizens to Enter into Service Contracts
with Foreign Persons, Corporations for the Exploration, Development, Exploitation or
Utilization of Lands of the Public Domain, Amending for the purpose certain provisions of
Commonwealth Act No. 141.

168 Pres. Decree No. 151 (1973), sec. 1.

169Providing for A Modernized System of Administration and Disposition of Mineral Lands and
to Promote and Encourage the Development and Exploitation thereof.

170 Revising and Consolidating All Laws and Decrees Affecting Fishing and Fisheries.

171 Pres. Decree No. 704 (1975), sec. 21.

172Revising Presidential Decree No. 389, otherwise known as The Forestry Reform Code of
the Philippines.

173 Pres. Decree No. 705 (1975), sec. 62.

174 An Act to Promote the Exploration and Development of Geothermal Resources.


88

175 Magallona, supra, at 6.

176Declaring a National Policy to Implement the Reforms Mandated by the People, Protecting
their Basic Rights, Adopting a Provisional Constitution, and Providing for an Orderly
Transition to a Government under a New Constitution.

177 Const., art. XVIII, sec. 27; De Leon v. Esguerra, 153 SCRA 602 (1987).

178 Miners Association of the Philippines, Inc. v. Factoran, Jr., 240 SCRA 100 (1995).

179 Ibid.

180 Ibid.

181 J. Bernas, S.J., The Intent of the 1986 Constitution Writers 812 (1995).

182 Miners Association of the Philippines, Inc. v. Factoran, Jr., supra.

183 III Records of the Constitutional Commission 255.

184 Id., at 355-356.

185 Const. (1986), art. II, sec. 1.

186
Cruz v. Secretary of Environment and Natural Resources, supra, Puno, J., Separate
Opinion.

187 Rep. Act No. 7942 (1995), sec. 9.

188SEC. 82. Allocation of Government Share.—The Government share as referred to in the


preceding sections shall be shared and allocated in accordance with Sections 290 and 292 of
Republic Act No. 7160 otherwise known as the Local Government Code of 1991. In case the
development and utilization of mineral resources is undertaken by a government-owned or -
controlled corporation, the sharing and allocation shall be in accordance with Sections 291
and 292 of the said Code.

189 An Act Creating A People's Small-Scale Mining Program and for other purposes.

190 Rep. Act No. 7942 (1995), sec. 42.

191 Id., secs. 3 (ab) and 26.

192"Contractor" means a qualified person acting alone or in consortium who is a party to a


mineral agreement or to a financial or technical assistance agreement. (Id., sec. 3[g].)

193"Contract area" means land or body water delineated for purposes of exploration,
development, or utilization of the minerals found therein. (Id., sec. 3[f].)

194"Gross output" means the actual market value of minerals or mineral products from its
mining area as defined in the National Internal Revenue Code (Id., sec. 3[v]).

195 Id., sec. 26 (a).

196An Act Reducing Excise Tax Rates on Metallic and Non-Metallic Minerals and Quarry
Resources, amending for the purpose Section 151 (a) of the National Internal Revenue Code,
as amended.
89

197 Rep. Act No. 7942 (1995), sec. (80).

198 Id., Sec. 26 (b).

199"Mineral resource" means any concentration of minerals/rocks with potential economic


value. (Id., sec. 3[ad].)

200 Id., sec. 26 (c).

201 Ibid.

202 Id., sec. 3 (h).

203 Id., sec. 3 (x).

204 Id., sec. 26, last par.

205 Id., sec. 27.

206 Id., sec. 3 (aq).

207 Id., sec. 3 (r).

208 Id., sec. 33.

209 Id., sec. 3 (t).

210 Id., sec. 3 (aq).

211 The maximum areas in cases of mineral agreements are prescribed in Section 28 as
follows:

SEC. 28. Maximum Areas for Mineral Agreement. – The maximum area that a
qualified person may hold at any time under a mineral agreement shall be:

(a) Onshore, in any one province –

(1) For individuals, ten (10) blocks; and

(2) For partnerships, cooperatives, associations, or corporations, one


hundred (100) blocks.

(b) Onshore, in the entire Philippines –

(1) For individuals, twenty (20) blocks; and

(2) For partnerships, cooperatives, associations, or corporations, two


hundred (200) blocks.

(c) Offshore, in the entire Philippines –

(1) For individuals, fifty (50) blocks;

(2) For partnerships, cooperatives, associations, or corporations, five


hundred (500) blocks; and
90

(3) For the exclusive economic area, a larger area to be determined


by the Secretary.

The maximum areas mentioned above that a contractor may hold under a
mineral agreement shall not include mining/quarry areas under operating
agreements between the contractor and a
claimowner/lessee/permittee/licensee entered into under Presidential Decree
No. 463.

On the other hand, Section 34, which governs the maximum area for FTAAs
provides:

SEC. 34. Maximum Contract Area. – The maximum contract area that may be
granted per qualified person, subject to relinquishment shall be:

(a) 1,000 meridional blocks onshore;

(b) 4,000 meridional blocks offshore; or

(c) Combinations of (a) and (b) provided that it shall not exceed the maximum
limits for onshore and offshore areas.

212 Id., sec. 33.

213 Id., sec. 81.

214 Kapatiran v. Tan, 163 SCRA 371 (1988).

215
Providing for the Publication of Laws either in the Official Gazette or in a Newspaper of
General Circulation in the Philippines as a Requirement for their Effectivity.

216Section 1, E.O. No. 200 was subsequently incorporated in the Administrative Code of 1987
(Executive Order No. 292 as Section 18, Chapter 5 (Operation and Effect of Laws), Book 1
(Sovereignty and General Administration).

217 136 SCRA 27 (1985).

218 Manila Prince Hotel v. Government Service Insurance System, 267 SCRA 408 (1997).

219 Const., art. 3, sec. 1.

220 83 O.G. (Suppl.) 3528-115 to 3528-117 (August 1987).

221 Miners Association of the Philippines, Inc. v. Factoran, Jr., supra.

222 Petitioners note in their Memorandum that the FTAA:

x x x guarantees that wholly foreign owned [WMCP] entered into the FTAA in order to
facilitate "the large scale exploration, development and commercial exploitation of
mineral deposits that may be found to exist within the Contract area." [Section 1.1] As
a contractor it also has the "exclusive right to explore, exploit, utilize, process and
dispose of all mineral products and by-products thereof that may be derived or
produced from the Contract Area." [Section 1.3] Thus, it is divided into an "exploration
and feasibility phase" [Section 3.2 (a)] and a "construction, development and
production phase." [Section 3. 2 (b).]
91

Thus, it is this wholly foreign owned corporation that, among other things:

(a) operates within a prescribed contract area [Section 4],

(b) opts to apply for a Mining Production Sharing Agreement [Section 4.2],

(c) relinquishes control over portions thereof at their own choice [Section 4.6],

(d) submits work programs, incurs expenditures, and makes reports during
the exploration period [Section 5],

(e) submits a Declaration of Mining Feasibility [Sections 5.4 and 5.5],

(f) during the development period, determines the timetable, submits work
programs, provides the reports and determines and executes expansions,
modifications, improvements and replacements of new mining facilities within
the area [Section 6],

(g) complies with the conditions for environmental protection and industrial
safety, posts the necessary bonds and makes representations and warranties
to the government [Section 10.5].

The contract subsists for an initial term of twenty-five (25) years from the date of its
effectivity [Section 3.1] and renewable for a further period of twenty-five years under
the same terms and conditions upon application by private respondent [Section 3.3].
(Rollo, pp. 458-459.)

223 H. C. Black, Handbook on the Construction and Interpretation of the Laws § 8.

224 Ibid.

225 J. M. Tuason & Co., Inc. v. Land Tenure Association, 31 SCRA 413 (1970).

226 Rollo, p. 580.

227 Ibid. Emphasis supplied.

228People v. Manantan, 115 Phil. 657 (1962); Commission on Audit of the Province of Cebu
v. Province of Cebu, 371 SCRA 196 (2001).

229 Rollo, p. 569.

230 III Record of the Constitutional Commission 351-352.

231 V Record of the Constitutional Commission 844.

232 Id., at 841.

233 Id., at 842.

234 Id. at 844.

235
Vide Cherey v. Long Beach, 282 NY 382, 26 NE 2d 945, 127 ALR 1210 (1940), cited in 16
Am Jur 2d Constitutional Law §79.
92

236 Civil Liberties Union v. Executive Secretary, 194 SCRA 317, 325 (1991).

237 III Record of the Constitutional Commission 278.

238 Id., at 316-317.

239 III Record of the Constitutional Commission 358-359.

240 Vera v. Avelino, 77 Phil. 192 (1946).

241 J. Nolledo, The New Constitution of the Philippines Annotated 924-926 (1990).

242Resolution to Incorporate in the New Constitution an Article on National Economy and


Patrimony.

243The Chair of the Committee on National Economy and Patrimony, alluded to it in the
discussion on the capitalization requirement:

MR. VILLEGAS. We just had a long discussion with the members of the team from
the UP Law Center who provided us a draft. The phrase that is contained here which
we adopted from the UP draft is "60 percent of voting stock." (III Record of the
Constitutional Commission 255.)

Likewise, in explaining the reasons for the deletion of the term "exploitation":

MR. VILLEGAS. Madam President, following the recommendation in the UP draft, we


omitted "exploitation" first of all because it is believed to be subsumed under
"development" and secondly because it has a derogatory connotation. (Id., at 358.)

244 Id., at 12.

245 Id., at 15-16.

246M. Magallona, Nationalism and Its Subversion in the Constitution 5, in II Draft Proposal of
the 1986 U.P. Law Constitution Project.

247 Agabin, supra, at 16.

248E. Labitag, Philippine Natural Resources: Some Problems and Perspectives 17 in II Draft
Proposal of the 1986 U.P. Law Constitution Project.

249 I Draft Proposal of the 1986 U.P. Law Constitution Project 11-13.

250 Id., at 9-11. Professor Labitag also suggests that:

x x x. The concession regime of natural resources disposition should be discontinued.


Instead the State shall enter into such arrangements and agreements like co-
production, joint ventures, etc. as shall bring about effective control and a larger
share in the proceeds, harvest or production. (Labitag, supra, at 17.)

251 Vide Note 147.

252Vide Note 230. The question was posed before the Jamir amendment and subsequent
proposals introducing other limitations.
93

Comm. Villegas' response that there was no requirement in the 1973 Constitution for
a law to govern service contracts and that, in fact, there were then no such laws is
inaccurate. The 1973 Charter required similar legislative approval, although it did not
specify the form it should take: "The Batasang Pambansa, in the national interest,
may allow such citizens… to enter into service contracts…." As previously noted,
however, laws authorizing service contracts were actually enacted by presidential
decree.

253 Vide Note 238.

254 Vide Note 241.

255 Vide Note 231.

256 Dated July 28, 1987.

257 Dated October 3, 1990.

258 Peralta v. Civil Service Commission, 212 SCRA 425 (1992).

259 Vide Note 238.

260 III Record of the Constitutional Commission 354.

261 Salaysay v. Castro, 98 Phil. 364 (1956).

262 Rep. Act No. 7942 (1995), sec. 3 (q).

263 Id., sec. 3 (aq).

264 Id., sec. 20.

265 Id., sec. 23, first par.

266 Id., sec. 23, last par.

267 Id., sec. 3 (j).

268 Id., sec. 3 (az).

269 Id., sec. 35 (m).

270 Id., secs. 3 (aq) and 56.

271 Id., sec. 3 (y).

272 Id., sec. 35 (g).

273 Id., sec. 35 (h).

274 Id., sec. 35 (l).

275 Id., sec. 3 (af).


94

276 SEC. 72. Timber Rights.—Any provision of the law to the contrary notwithstanding, a
contractor may be granted a right to cut trees or timber within his mining area as may be
necessary for his mining operations subject to forestry laws, rules and regulations: Provided,
That if the land covered by the mining area is already covered by exiting timber concessions,
the volume of timber needed and the manner of cutting and removal thereof shall be
determined by the mines regional director, upon consultation with the contractor, the timber
concessionaire/permittee and the Forest Management Bureau of the Department: Provided,
further, That in case of disagreement between the contractor and the timber concessionaire,
the matter shall be submitted to the Secretary whose decision shall be final. The contractor
shall perform reforestation work within his mining area in accordance with forestry laws, rules
and regulations. [Emphasis supplied.]

SEC. 73. Water Rights.—A contractor shall have water rights for mining operations
upon approval of application with the appropriate government agency in accordance
with existing water laws, rules and regulations promulgated thereunder: Provided,
That water rights already granted or vested through long use, recognized and
acknowledged by local customs, laws and decisions of courts shall not thereby be
impaired: Provided, further, That the Government reserves the right to regulate water
rights and the reasonable and equitable distribution of water supply so as to prevent
the monopoly of the use thereof. [Emphasis supplied.]

SEC. 74. Right to Possess Explosives.—A contractor/exploration permittee shall


have the right to possess and use explosives within his contract/permit area as may
be necessary for his mining operations upon approval of an application with the
appropriate government agency in accordance with existing laws, rules and
regulations promulgated thereunder: Provided, That the Government reserves the
right to regulate and control the explosive accessories to ensure safe mining
operations. [Emphasis supplied.]

SEC. 75. Easement Rights.—When mining areas are so situated that for purposes of
more convenient mining operations it is necessary to build, construct or install on the
mining areas or lands owned, occupied or leased by other persons, such
infrastructure as roads, railroads, mills, waste dump sites, tailings ponds,
warehouses, staging or storage areas and port facilities, tramways, runways, airports,
electric transmission, telephone or telegraph lines, dams and their normal flood and
catchment areas, sites for water wells, ditches, canals, new river beds, pipelines,
flumes, cuts, shafts, tunnels, or mills, the contractor, upon payment of just
compensation, shall be entitled to enter and occupy said mining areas or lands.
[Emphasis supplied.]

SEC. 76. Entry into Private Lands and Concession Areas.—Subject to prior
notification, holders of mining rights shall not be prevented from entry into private
lands and concession areas by surface owners, occupants, or concessionaires when
conducting mining operations therein: Provided, That any damage done to the
property of the surface owner, occupant, or concessionaire as a consequence of
such operations shall be properly compensated as may be bee provided for in the
implementing rules and regulations: Provided, further, That to guarantee such
compensation, the person authorized to conduct mining operation shall, prior thereto,
post a bond with the regional director based on the type of properties, the prevailing
prices in and around the area where the mining operations are to be conducted, with
surety or sureties satisfactory to the regional director. [Emphasis supplied.]

277 Id., sec. 39, first par.

278 Id., sec. 39, second par.

279 Id., sec. 35 (e).


95

280 SEC. 23. Rights and Obligations of the Permittee.—x x x.

The permittee may apply for a mineral production sharing agreement, joint venture
agreement, co-production agreement or financial or technical assistance agreement
over the permit area, which application shall be granted if the permittee meets the
necessary qualifications and the terms and conditions of any such agreement:
Provided, That the exploration period covered by the exploration period of the mineral
agreement or financial or technical assistance agreement.

281SEC. 35. Terms and Conditions. — The following terms, conditions, and warranties shall
be incorporated in the financial or technical assistance agreement, to wit:

(a) A firm commitment in the form of a sworn statement, of an amount corresponding


to the expenditure obligation that will be invested in the contract area: Provided, That
such amount shall be subject to changes as may be provided for in the rules and
regulations of this Act;

(b) A financial guarantee bond shall be posted in favor of the Government in an


amount equivalent to the expenditure obligation of the applicant for any year;

(c) Submission of proof of technical competence, such as, but not limited to, its track
record in mineral resource exploration, development, and utilization; details of
technology to be employed in the proposed operation; and details of technical
personnel to undertake the operation;

(d) Representations and warranties that the applicant has all the qualifications and
none of the disqualifications for entering into the agreement;

(e) Representations and warranties that the contractor has or has access to all the
financing, managerial and technical expertise and, if circumstances demand, the
technology required to promptly and effectively carry out the objectives of the
agreement with the understanding to timely deploy these resources under its
supervision pursuant to the periodic work programs and related budgets, when
proper, providing an exploration period up to two (2) years, extendible for another two
(2) years but subject to annual review by the Secretary in accordance with the
implementing rules and regulations of this Act, and further, subject to the
relinquishment obligations;

(f) Representations and warranties that, except for paymets for dispositions for its
equity, foreign investments in local enterprises which are qualified for repatriation,
and local supplier's credits and such other generally accepted and permissible
financial schemes for raising funds for valid business purposes, the conractor shall
not raise any form of financing from domestic sources of funds, whether in Philippine
or foreign currency, for conducting its mining operations for and in the contract area;

(g) The mining operations shall be conducted in accordance with the provisions of
this Act and its implementing rules and regulations;

(h) Work programs and minimum expenditures commitments;

(i) Preferential use of local goods and services to the maximum extent practicable;

(j) A stipulation that the contractors are obligated to give preference to Filipinos in all
types of mining employment for which they are qualified and that technology shall be
transferred to the same;
96

(k) Requiring the proponent to effectively use appropriate anti-pollution technology


and facilities to protect the environment and to restore or rehabilitate mined out areas
and other areas affected by mine tailings and other forms of pollution or destruction;

(l) The contractors shall furnish the Government records of geologic, accounting, and
other relevant data for its mining operations, and that book of accounts and records
shall be open for inspection by the government;

(m) Requiring the proponent to dispose of the minerals and byproducts produced
under a financial or technical assistance agreement at the highest price and more
advantageous terms and conditions as provided for under the rules and regulations of
this Act;

(n) Provide for consultation and arbitration with respect to the interpretation and
implementation of the terms and conditions of the agreements; and

(o) Such other terms and conditions consistent with the Constitution and with this Act
as the Secretary may deem to be for the best interest of the State and the welfare of
the Filipino people.

282SEC. 39. Option to Convert into a Mineral Agreement. — The contractor has the option to
convert the financial or technical assistance agreement to a mineral agreement at any time
during the term of the agreement, if the economic viability of the contract area is found to be
inadequate to justify large-scale mining operations, after proper notice to the Secretary as
provided for under the implementing rules and regulations; Provided, That the mineral
agreement shall only be for the remaining period of the original agreement.

In the case of a foreign contractor, it shall reduce its equity to forty percent (40%) in
the corporation, partnership, association, or cooperative. Upon compliance with this
requirement by the contractor, the Secretary shall approve the conversion and
execute the mineral production-sharing agreement.

283SEC. 56. Eligibility of Foreign-owned/-controlled Corporation.—A foreign owned/ -


controlled corporation may be granted a mineral processing permit.

SEC. 3. Definition of Terms. – As used in and for purposes of this Act, the following terms,
284

whether in singular or plural, shall mean:

xxx

(g) "Contractor" means a qualified person acting alone or in consortium who is a party
to a mineral agreement or to a financial or technical assistance agreement.

285SEC. 34. Maximum Contract Area. — The maximum contract area that may be granted
per qualified person, subject to relinquishment shall be:

(a) 1,000 meridional blocks onshore;

(b) 4,000 meridional blocks offshore; or

(c) Combinations of (a) and (b) provided that it shall not exceed the maximum limits
for onshore and offshore areas.

286SEC. 36. Negotiations. — A financial or technical assistance agreement shall be


negotiated by the Department and executed and approved by the President. The President
shall notify Congress of all financial or technical assistance agreements within thirty (30) days
from execution and approval thereof.
97

287SEC. 37. Filing and Evaluation of Financial or Technical Assistance Agreement Proposals.
— All financial or technical assistance agreement proposals shall be filed with the Bureau
after payment of the required processing fees. If the proposal is found to be sufficient and
meritorious in form and substance after evaluation, it shall be recorded with the appropriate
government agency to give the proponent the prior right to the area covered by such
proposal: Provided, That existing mineral agreements, financial or technical assistance
agreements and other mining rights are not impaired or prejudiced thereby. The Secretary
shall recommend its approval to the President.

288SEC. 38. Term of Financial or Technical Assistance Agreement. — A financial or technical


assistance agreement shall have a term not exceeding twenty-five (25) years to start from the
execution thereof, renewable for not more than twenty-five (25) years under such terms and
conditions as may be provided by law.

289 SEC. 40. Assignment/Transfer. — A financial or technical assistance agreement may be


assigned or transferred, in whole or in part, to a qualified person subject to the prior approval
of the President: Provided, That the President shall notify Congress of every financial or
technical assistance agreement assigned or converted in accordance with this provision
within thirty (30) days from the date of the approval thereof.

290SEC. 41. Withdrawal from Financial or Technical Assistance Agreement. — The contractor
shall manifest in writing to the Secretary his intention to withdraw from the agreement, if in his
judgment the mining project is no longer economically feasible, even after he has exerted
reasonable diligence to remedy the cause or the situation. The Secretary may accept the
withdrawal: Provided, That the contractor has complied or satisfied all his financial, fiscal or
legal obligations.

291 SEC. 81. Government Share in Other Mineral Agreements.—x x x.

The Government share in financial or technical assistance agreement shall consist of,
among other things, the contractor's corporate income tax, excise tax, special
allowance, withholding tax due from the contractor's foreign stockholders arising from
dividend or interest payments to the said foreign stockholder in case of a foreign
national and all such other taxes, duties and fees as provided for under existing laws.

The collection of Government share in financial or technical assistance agreement


shall commence after the financial or technical assistance agreement contractor has
fully recovered its pre-operating expenses, exploration, and development
expenditures, inclusive.

292SEC. 90. Incentives.—The contractors in mineral agreements, and financial or technical


assistance agreements shall be entitled to the applicable fiscal and non-fiscal incentives as
provided for under Executive Order No. 226, otherwise known as the Omnibus Investments
Code of 1987: Provided, That holders of exploration permits may register with the Board of
Investments and be entitled to the fiscal incentives granted under the said Code for the
duration of the permits or extensions thereof: Provided, further, That mining activities shall
always be included in the investment priorities plan.

293 Lidasan v. Commission on Elections, 21 SCRA 496 (1967).

294 Vide also WMCP FTAA, sec. 10.2 (a).

295 WMCP, sec. 10.2.

296 Id., sec. 11.

297 Id., sec. 10.1(a).


98

298 Id., sec. 10.1(c).

299 Id., sec. 6.4.

300 Rollo, pp. 563-564.

301 Civil Code, art. 8.

302 Const., art III, sec. 1.

303 Vide Note 223.

304 Rollo, p. 243.

305 Civil Liberties Union v. Executive Secretary, supra.

306 Automotive Parts & Equipment Company, Inc. v. Lingad, 30 SCRA 248 (1969).

307 Ibid.

ON RECONSIDERATION (G.R. No. 127882, December 1, 2004)

EN BANC

G.R. No. 127882 December 1, 2004

LA BUGAL-B'LAAN TRIBAL ASSOCIATION, INC., Represented by its Chairman F'LONG


MIGUEL M. LUMAYONG; WIGBERTO E. TAÑADA; PONCIANO BENNAGEN; JAIME TADEO;
RENATO R. CONSTANTINO JR.; F'LONG AGUSTIN M. DABIE; ROBERTO P. AMLOY; RAQIM L.
DABIE; SIMEON H. DOLOJO; IMELDA M. GANDON; LENY B. GUSANAN; MARCELO L.
GUSANAN; QUINTOL A. LABUAYAN; LOMINGGES D. LAWAY; BENITA P. TACUAYAN; Minors
JOLY L. BUGOY, Represented by His Father UNDERO D. BUGOY and ROGER M. DADING;
Represented by His Father ANTONIO L. DADING; ROMY M. LAGARO, Represented by His
Father TOTING A. LAGARO; MIKENY JONG B. LUMAYONG, Represented by His Father
MIGUEL M. LUMAYONG; RENE T. MIGUEL, Represented by His Mother EDITHA T. MIGUEL;
ALDEMAR L. SAL, Represented by His Father DANNY M. SAL; DAISY RECARSE, Represented
by Her Mother LYDIA S. SANTOS; EDWARD M. EMUY; ALAN P. MAMPARAIR; MARIO L.
MANGCAL; ALDEN S. TUSAN; AMPARO S. YAP; VIRGILIO CULAR; MARVIC M.V.F. LEONEN;
JULIA REGINA CULAR, GIAN CARLO CULAR, VIRGILIO CULAR JR., Represented by Their
Father VIRGILIO CULAR; PAUL ANTONIO P. VILLAMOR, Represented by His Parents JOSE
VILLAMOR and ELIZABETH PUA-VILLAMOR; ANA GININA R. TALJA, Represented by Her
Father MARIO JOSE B. TALJA; SHARMAINE R. CUNANAN, Represented by Her Father
ALFREDO M. CUNANAN; ANTONIO JOSE A. VITUG III, Represented by His Mother ANNALIZA
A. VITUG, LEAN D. NARVADEZ, Represented by His Father MANUEL E. NARVADEZ JR.;
ROSERIO MARALAG LINGATING, Represented by Her Father RIO OLIMPIO A. LINGATING;
MARIO JOSE B. TALJA; DAVID E. DE VERA; MARIA MILAGROS L. SAN JOSE; Sr. SUSAN O.
BOLANIO, OND; LOLITA G. DEMONTEVERDE; BENJIE L. NEQUINTO;1 ROSE LILIA S.
ROMANO; ROBERTO S. VERZOLA; EDUARDO AURELIO C. REYES; LEAN LOUEL A. PERIA,
Represented by His Father ELPIDIO V. PERIA;2 GREEN FORUM PHILIPPINES; GREEN FORUM
WESTERN VISAYAS (GF-WV); ENVIRONMENTAL LEGAL ASSISTANCE CENTER (ELAC);
KAISAHAN TUNGO SA KAUNLARAN NG KANAYUNAN AT REPORMANG PANSAKAHAN
99

(KAISAHAN);3 PARTNERSHIP FOR AGRARIAN REFORM and RURAL DEVELOPMENT


SERVICES, INC. (PARRDS); PHILIPPINE PARTNERSHIP FOR THE DEVELOPMENT OF HUMAN
RESOURCES IN THE RURAL AREAS, INC. (PHILDHRRA); WOMEN'S LEGAL BUREAU (WLB);
CENTER FOR ALTERNATIVE DEVELOPMENT INITIATIVES, INC. (CADI); UPLAND
DEVELOPMENT INSTITUTE (UDI); KINAIYAHAN FOUNDATION, INC.; SENTRO NG
ALTERNATIBONG LINGAP PANLIGAL (SALIGAN); and LEGAL RIGHTS AND NATURAL
RESOURCES CENTER, INC. (LRC), petitioners,
vs.
VICTOR O. RAMOS, Secretary, Department of Environment and Natural Resources (DENR);
HORACIO RAMOS, Director, Mines and Geosciences Bureau (MGB-DENR); RUBEN TORRES,
Executive Secretary; and WMC (PHILIPPINES), INC.,4 respondents.

RESOLUTION

PANGANIBAN, J.:

All mineral resources are owned by the State. Their exploration, development and utilization (EDU)
must always be subject to the full control and supervision of the State. More specifically, given the
inadequacy of Filipino capital and technology in large-scale EDU activities, the State may secure the
help of foreign companies in all relevant matters -- especially financial and technical assistance --
provided that, at all times, the State maintains its right of full control. The foreign assistor or contractor
assumes all financial, technical and entrepreneurial risks in the EDU activities; hence, it may be given
reasonable management, operational, marketing, audit and other prerogatives to protect its
investments and to enable the business to succeed.

Full control is not anathematic to day-to-day management by the contractor, provided that the State
retains the power to direct overall strategy; and to set aside, reverse or modify plans and actions of
the contractor. The idea of full control is similar to that which is exercised by the board of directors of
a private corporation: the performance of managerial, operational, financial, marketing and other
functions may be delegated to subordinate officers or given to contractual entities, but the board
retains full residual control of the business.

Who or what organ of government actually exercises this power of control on behalf of the State? The
Constitution is crystal clear: the President. Indeed, the Chief Executive is the official constitutionally
mandated to "enter into agreements with foreign owned corporations." On the other hand, Congress
may review the action of the President once it is notified of "every contract entered into in accordance
with this [constitutional] provision within thirty days from its execution." In contrast to this express
mandate of the President and Congress in the EDU of natural resources, Article XII of the Constitution
is silent on the role of the judiciary. However, should the President and/or Congress gravely abuse
their discretion in this regard, the courts may -- in a proper case -- exercise their residual duty under
Article VIII. Clearly then, the judiciary should not inordinately interfere in the exercise of this
presidential power of control over the EDU of our natural resources.

The Constitution should be read in broad, life-giving strokes. It should not be used to strangulate
economic growth or to serve narrow, parochial interests. Rather, it should be construed to grant the
President and Congress sufficient discretion and reasonable leeway to enable them to attract foreign
investments and expertise, as well as to secure for our people and our posterity the blessings of
prosperity and peace.

On the basis of this control standard, this Court upholds the constitutionality of the Philippine Mining
Law, its Implementing Rules and Regulations -- insofar as they relate to financial and technical
agreements -- as well as the subject Financial and Technical Assistance Agreement (FTAA). 5
100

Background

The Petition for Prohibition and Mandamus before the Court challenges the constitutionality of (1)
Republic Act No. [RA] 7942 (The Philippine Mining Act of 1995); (2) its Implementing Rules and
Regulations (DENR Administrative Order No. [DAO] 96-40); and (3) the FTAA dated March 30,
1995,6 executed by the government with Western Mining Corporation (Philippines), Inc. (WMCP). 7

On January 27, 2004, the Court en banc promulgated its Decision8 granting the Petition and declaring
the unconstitutionality of certain provisions of RA 7942, DAO 96-40, as well as of the entire FTAA
executed between the government and WMCP, mainly on the finding that FTAAs are service
contracts prohibited by the 1987 Constitution.

The Decision struck down the subject FTAA for being similar to service contracts, 9 which, though
permitted under the 1973 Constitution,10 were subsequently denounced for being antithetical to the
principle of sovereignty over our natural resources, because they allowed foreign control over the
exploitation of our natural resources, to the prejudice of the Filipino nation.

The Decision quoted several legal scholars and authors who had criticized service contracts for, inter
alia, vesting in the foreign contractor exclusive management and control of the enterprise, including
operation of the field in the event petroleum was discovered; control of production, expansion and
development; nearly unfettered control over the disposition and sale of the products
discovered/extracted; effective ownership of the natural resource at the point of extraction; and
beneficial ownership of our economic resources. According to the Decision, the 1987 Constitution
(Section 2 of Article XII) effectively banned such service contracts.

Subsequently, respondents filed separate Motions for Reconsideration. In a Resolution dated March
9, 2004, the Court required petitioners to comment thereon. In the Resolution of June 8, 2004, it set
the case for Oral Argument on June 29, 2004.

After hearing the opposing sides, the Court required the parties to submit their respective Memoranda
in amplification of their arguments. In a Resolution issued later the same day, June 29, 2004, the
Court noted, inter alia, the Manifestation and Motion (in lieu of comment) filed by the Office of the
Solicitor General (OSG) on behalf of public respondents. The OSG said that it was not interposing
any objection to the Motion for Intervention filed by the Chamber of Mines of the Philippines, Inc.
(CMP) and was in fact joining and adopting the latter's Motion for Reconsideration.

Memoranda were accordingly filed by the intervenor as well as by petitioners, public respondents, and
private respondent, dwelling at length on the three issues discussed below. Later, WMCP submitted
its Reply Memorandum, while the OSG -- in obedience to an Order of this Court -- filed a Compliance
submitting copies of more FTAAs entered into by the government.

Three Issues Identified by the Court

During the Oral Argument, the Court identified the three issues to be resolved in the present
controversy, as follows:

1. Has the case been rendered moot by the sale of WMC shares in WMCP to Sagittarius (60 percent
of Sagittarius' equity is owned by Filipinos and/or Filipino-owned corporations while 40 percent is
owned by Indophil Resources NL, an Australian company) and by the subsequent transfer and
registration of the FTAA from WMCP to Sagittarius?

2. Assuming that the case has been rendered moot, would it still be proper to resolve the
constitutionality of the assailed provisions of the Mining Law, DAO 96-40 and the WMCP FTAA?
101

3. What is the proper interpretation of the phrase Agreements Involving Either Technical or Financial
Assistancecontained in paragraph 4 of Section 2 of Article XII of the Constitution?

Should the Motion for Reconsideration Be Granted?

Respondents' and intervenor's Motions for Reconsideration should be granted, for the reasons
discussed below. The foregoing three issues identified by the Court shall now be taken up seriatim.

First Issue:

Mootness

In declaring unconstitutional certain provisions of RA 7942, DAO 96-40, and the WMCP FTAA, the
majority Decision agreed with petitioners' contention that the subject FTAA had been executed in
violation of Section 2 of Article XII of the 1987 Constitution. According to petitioners, the FTAAs
entered into by the government with foreign-owned corporations are limited by the fourth paragraph of
the said provision to agreements involving only technical or financial assistance for large-scale
exploration, development and utilization of minerals, petroleum and other mineral oils. Furthermore,
the foreign contractor is allegedly permitted by the FTAA in question to fully manage and control the
mining operations and, therefore, to acquire "beneficial ownership" of our mineral resources.

The Decision merely shrugged off the Manifestation by WMPC informing the Court (1) that on January
23, 2001, WMC had sold all its shares in WMCP to Sagittarius Mines, Inc., 60 percent of whose equity
was held by Filipinos; and (2) that the assailed FTAA had likewise been transferred from WMCP to
Sagittarius.11 The ponencia declared that the instant case had not been rendered moot by the transfer
and registration of the FTAA to a Filipino-owned corporation, and that the validity of the said transfer
remained in dispute and awaited final judicial determination.12Patently therefore, the Decision is
anchored on the assumption that WMCP had remained a foreign corporation.

The crux of this issue of mootness is the fact that WMCP, at the time it entered into the
FTAA, happened to be wholly owned by WMC Resources International Pty., Ltd. (WMC), which in
turn was a wholly owned subsidiary of Western Mining Corporation Holdings Ltd., a publicly listed
major Australian mining and exploration company.

The nullity of the FTAA was obviously premised upon the contractor being a foreign corporation. Had
the FTAA been originally issued to a Filipino-owned corporation, there would have been no
constitutionality issue to speak of. Upon the other hand, the conveyance of the WMCP FTAA to a
Filipino corporation can be likened to the sale of land to a foreigner who subsequently acquires
Filipino citizenship, or who later resells the same land to a Filipino citizen. The conveyance would be
validated, as the property in question would no longer be owned by a disqualified vendee.

And, inasmuch as the FTAA is to be implemented now by a Filipino corporation, it is no longer


possible for the Court to declare it unconstitutional. The case pending in the Court of Appeals is a
dispute between two Filipino companies (Sagittarius and Lepanto), both claiming the right to purchase
the foreign shares in WMCP. So, regardless of which side eventually wins, the FTAA would still be in
the hands of a qualified Filipino company. Considering that there is no longer any justiciable
controversy, the plea to nullify the Mining Law has become a virtual petition for declaratory relief, over
which this Court has no original jurisdiction.

In their Final Memorandum, however, petitioners argue that the case has not become moot,
considering the invalidity of the alleged sale of the shares in WMCP from WMC to Sagittarius, and of
the transfer of the FTAA from WMCP to Sagittarius, resulting in the change of contractor in the FTAA
in question. And even assuming that the said transfers were valid, there still exists an actual case
predicated on the invalidity of RA 7942 and its Implementing Rules and Regulations (DAO 96-40).
Presently, we shall discuss petitioners' objections to the transfer of both the shares and the FTAA. We
shall take up the alleged invalidity of RA 7942 and DAO 96-40 later on in the discussion of the third
issue.
102

No Transgression of the Constitution


by the Transfer of the WMCP Shares

Petitioners claim, first, that the alleged invalidity of the transfer of the WMCP shares to Sagittarius
violates the fourth paragraph of Section 2 of Article XII of the Constitution; second, that it is contrary to
the provisions of the WMCP FTAA itself; and third, that the sale of the shares is suspect and should
therefore be the subject of a case in which its validity may properly be litigated.

On the first ground, petitioners assert that paragraph 4 of Section 2 of Article XII permits the
government to enter into FTAAs only with foreign-owned corporations. Petitioners insist that the first
paragraph of this constitutional provision limits the participation of Filipino corporations in the
exploration, development and utilization of natural resources to only three species of contracts --
production sharing, co-production and joint venture -- to the exclusion of all other arrangements or
variations thereof, and the WMCP FTAA may therefore not be validly assumed and implemented by
Sagittarius. In short, petitioners claim that a Filipino corporation is not allowed by the Constitution to
enter into an FTAA with the government.

However, a textual analysis of the first paragraph of Section 2 of Article XII does not support
petitioners' argument. The pertinent part of the said provision states: "Sec. 2. x x x The exploration,
development and utilization of natural resources shall be under the full control and supervision of the
State. The State may directly undertake such activities, or it may enter into co-production, joint
venture, or production-sharing agreements with Filipino citizens, or corporations or associations at
least sixty per centum of whose capital is owned by such citizens. x x x." Nowhere in the provision is
there any express limitation or restriction insofar as arrangements other than the three
aforementioned contractual schemes are concerned.

Neither can one reasonably discern any implied stricture to that effect. Besides, there is no basis to
believe that the framers of the Constitution, a majority of whom were obviously concerned with
furthering the development and utilization of the country's natural resources, could have wanted to
restrict Filipino participation in that area. This point is clear, especially in the light of the overarching
constitutional principle of giving preference and priority to Filipinos and Filipino corporations in the
development of our natural resources.

Besides, even assuming (purely for argument's sake) that a constitutional limitation barring Filipino
corporations from holding and implementing an FTAA actually exists, nevertheless, such provision
would apply only to the transfer of the FTAA to Sagittarius, but definitely not to the sale of WMC's
equity stake in WMCP to Sagittarius. Otherwise, an unreasonable curtailment of property rights
without due process of law would ensue. Petitioners' argument must therefore fail.

FTAA Not Intended


Solely for Foreign Corporation

Equally barren of merit is the second ground cited by petitioners -- that the FTAA was intended to
apply solely to a foreign corporation, as can allegedly be seen from the provisions therein. They
manage to cite only one WMCP FTAA provision that can be regarded as clearly intended to apply
only to a foreign contractor: Section 12, which provides for international commercial arbitration under
the auspices of the International Chamber of Commerce, after local remedies are exhausted. This
provision, however, does not necessarily imply that the WMCP FTAA cannot be transferred to and
assumed by a Filipino corporation like Sagittarius, in which event the said provision should simply be
disregarded as a superfluity.

No Need for a Separate


Litigation of the Sale of Shares

Petitioners claim as third ground the "suspicious" sale of shares from WMC to Sagittarius; hence, the
need to litigate it in a separate case. Section 40 of RA 7942 (the Mining Law) allegedly requires the
President's prior approval of a transfer.
103

A re-reading of the said provision, however, leads to a different conclusion. "Sec.


40. Assignment/Transfer -- A financial or technical assistance agreement may be assigned or
transferred, in whole or in part, to a qualified person subject to the prior approval of the President:
Provided, That the President shall notify Congress of every financial or technical assistance
agreement assigned or converted in accordance with this provision within thirty (30) days from the
date of the approval thereof."

Section 40 expressly applies to the assignment or transfer of the FTAA, not to the sale and transfer of
shares of stock in WMCP. Moreover, when the transferee of an FTAA is another foreign corporation,
there is a logical application of the requirement of prior approval by the President of the Republic and
notification to Congress in the event of assignment or transfer of an FTAA. In this situation, such
approval and notification are appropriate safeguards, considering that the new contractor is the
subject of a foreign government.

On the other hand, when the transferee of the FTAA happens to be a Filipino corporation, the need
for such safeguard is not critical; hence, the lack of prior approval and notification may not be deemed
fatal as to render the transfer invalid. Besides, it is not as if approval by the President is entirely
absent in this instance. As pointed out by private respondent in its Memorandum,13 the issue of
approval is the subject of one of the cases brought by Lepanto against Sagittarius in GR No. 162331.
That case involved the review of the Decision of the Court of Appeals dated November 21, 2003 in
CA-GR SP No. 74161, which affirmed the DENR Order dated December 31, 2001 and the Decision of
the Office of the President dated July 23, 2002, both approving the assignment of the WMCP FTAA to
Sagittarius.

Petitioners also question the sale price and the financial capacity of the transferee. According to the
Deed of Absolute Sale dated January 23, 2001, executed between WMC and Sagittarius, the price of
the WMCP shares was fixed at US$9,875,000, equivalent to P553 million at an exchange rate of 56:1.
Sagittarius had an authorized capital stock of P250 million and a paid up capital of P60 million.
Therefore, at the time of approval of the sale by the DENR, the debt-to-equity ratio of the transferee
was over 9:1 -- hardly ideal for an FTAA contractor, according to petitioners.

However, private respondents counter that the Deed of Sale specifically provides that the payment of
the purchase price would take place only after Sagittarius' commencement of commercial production
from mining operations, if at all. Consequently, under the circumstances, we believe it would not be
reasonable to conclude, as petitioners did, that the transferee's high debt-to-equity ratio per se
necessarily carried negative implications for the enterprise; and it would certainly be improper to
invalidate the sale on that basis, as petitioners propose.

FTAA Not Void,


Thus Transferrable

To bolster further their claim that the case is not moot, petitioners insist that the FTAA is void and,
hence cannot be transferred; and that its transfer does not operate to cure the constitutional infirmity
that is inherent in it; neither will a change in the circumstances of one of the parties serve to ratify the
void contract.

While the discussion in their Final Memorandum was skimpy, petitioners in their Comment (on the
MR) did ratiocinate that this Court had declared the FTAA to be void because, at the time it was
executed with WMCP, the latter was a fully foreign-owned corporation, in which the former vested full
control and management with respect to the exploration, development and utilization of mineral
resources, contrary to the provisions of paragraph 4 of Section 2 of Article XII of the Constitution. And
since the FTAA was per se void, no valid right could be transferred; neither could it be ratified, so
petitioners conclude.

Petitioners have assumed as fact that which has yet to be established. First and foremost, the
Decision of this Court declaring the FTAA void has not yet become final. That was precisely the
reason the Court still heard Oral Argument in this case. Second, the FTAA does not vest in the foreign
corporation full control and supervision over the exploration, development and utilization of mineral
104

resources, to the exclusion of the government. This point will be dealt with in greater detail below; but
for now, suffice it to say that a perusal of the FTAA provisions will prove that the government has
effective overall direction and control of the mining operations, including marketing and product
pricing, and that the contractor's work programs and budgets are subject to its review and approval or
disapproval.

As will be detailed later on, the government does not have to micro-manage the mining operations
and dip its hands into the day-to-day management of the enterprise in order to be considered as
having overall control and direction. Besides, for practical and pragmatic reasons, there is a need for
government agencies to delegate certain aspects of the management work to the contractor. Thus the
basis for declaring the FTAA void still has to be revisited, reexamined and reconsidered.

Petitioners sniff at the citation of Chavez v. Public Estates Authority,14 and Halili v. CA,15 claiming that
the doctrines in these cases are wholly inapplicable to the instant case.

Chavez clearly teaches: "Thus, the Court has ruled consistently that where a Filipino citizen sells land
to an alien who later sells the land to a Filipino, the invalidity of the first transfer is corrected by the
subsequent sale to a citizen. Similarly, where the alien who buys the land subsequently acquires
Philippine citizenship, the sale is validated since the purpose of the constitutional ban to limit land
ownership to Filipinos has been achieved. In short, the law disregards the constitutional
disqualification of the buyer to hold land if the land is subsequently transferred to a qualified party, or
the buyer himself becomes a qualified party."16

In their Comment, petitioners contend that in Chavez and Halili, the object of the transfer (the land)
was not what was assailed for alleged unconstitutionality. Rather, it was the transaction that was
assailed; hence subsequent compliance with constitutional provisions would cure its infirmity. In
contrast, in the instant case it is the FTAA itself, the object of the transfer, that is being assailed as
invalid and unconstitutional. So, petitioners claim that the subsequent transfer of a void FTAA to a
Filipino corporation would not cure the defect.

Petitioners are confusing themselves. The present Petition has been filed, precisely because the
grantee of the FTAA was a wholly owned subsidiary of a foreign corporation. It cannot be gainsaid
that anyone would have asserted that the same FTAA was void if it had at the outset been issued to a
Filipino corporation. The FTAA, therefore, is not per se defective or unconstitutional. It was
questioned only because it had been issued to an allegedly non-qualified, foreign-owned corporation.

We believe that this case is clearly analogous to Halili, in which the land acquired by a non-Filipino
was re-conveyed to a qualified vendee and the original transaction was thereby cured.
Paraphrasing Halili, the same rationale applies to the instant case: assuming arguendo the invalidity
of its prior grant to a foreign corporation, the disputed FTAA -- being now held by a Filipino
corporation -- can no longer be assailed; the objective of the constitutional provision -- to keep the
exploration, development and utilization of our natural resources in Filipino hands -- has been served.

More accurately speaking, the present situation is one degree better than that obtaining in Halili, in
which the original sale to a non-Filipino was clearly and indisputably violative of the constitutional
prohibition and thus void ab initio. In the present case, the issuance/grant of the subject FTAA to the
then foreign-owned WMCP was not illegal, void or unconstitutional at the time. The matter had to be
brought to court, precisely for adjudication as to whether the FTAA and the Mining Law had indeed
violated the Constitution. Since, up to this point, the decision of this Court declaring the FTAA void
has yet to become final, to all intents and purposes, the FTAA must be deemed valid and
constitutional.17

At bottom, we find completely outlandish petitioners' contention that an FTAA could be entered into by
the government only with a foreign corporation, never with a Filipino enterprise. Indeed, the
nationalistic provisions of the Constitution are all anchored on the protection of Filipino interests. How
petitioners can now argue that foreigners have the exclusive right to FTAAs totally overturns the entire
basis of the Petition -- preference for the Filipino in the exploration, development and utilization of our
105

natural resources. It does not take deep knowledge of law and logic to understand that what the
Constitution grants to foreigners should be equally available to Filipinos.

Second Issue:

Whether the Court Can Still Decide the Case,


Even Assuming It Is Moot

All the protagonists are in agreement that the Court has jurisdiction to decide this controversy, even
assuming it to be moot.

Petitioners stress the following points. First, while a case becomes moot and academic when "there is
no more actual controversy between the parties or no useful purpose can be served in passing upon
the merits,"18 what is at issue in the instant case is not only the validity of the WMCP FTAA, but also
the constitutionality of RA 7942 and its Implementing Rules and Regulations. Second, the acts of
private respondent cannot operate to cure the law of its alleged unconstitutionality or to divest this
Court of its jurisdiction to decide. Third, the Constitution imposes upon the Supreme Court the duty to
declare invalid any law that offends the Constitution.

Petitioners also argue that no amendatory laws have been passed to make the Mining Act of 1995
conform to constitutional strictures (assuming that, at present, it does not); that public respondents will
continue to implement and enforce the statute until this Court rules otherwise; and that the said law
continues to be the source of legal authority in accepting, processing and approving numerous
applications for mining rights.

Indeed, it appears that as of June 30, 2002, some 43 FTAA applications had been filed with the Mines
and Geosciences Bureau (MGB), with an aggregate area of 2,064,908.65 hectares -- spread over
Luzon, the Visayas and Mindanao19 -- applied for. It may be a bit far-fetched to assert, as petitioners
do, that each and every FTAA that was entered into under the provisions of the Mining Act "invites
potential litigation" for as long as the constitutional issues are not resolved with finality.
Nevertheless, we must concede that there exists the distinct possibility that one or more of the future
FTAAs will be the subject of yet another suit grounded on constitutional issues.

But of equal if not greater significance is the cloud of uncertainty hanging over the mining industry,
which is even now scaring away foreign investments. Attesting to this climate of anxiety is the fact that
the Chamber of Mines of the Philippines saw the urgent need to intervene in the case and to present
its position during the Oral Argument; and that Secretary General Romulo Neri of the National
Economic Development Authority (NEDA) requested this Court to allow him to speak, during that Oral
Argument, on the economic consequences of the Decision of January 27, 2004. 20

We are convinced. We now agree that the Court must recognize the exceptional character of the
situation and the paramount public interest involved, as well as the necessity for a ruling to put an end
to the uncertainties plaguing the mining industry and the affected communities as a result of doubts
cast upon the constitutionality and validity of the Mining Act, the subject FTAA and future FTAAs, and
the need to avert a multiplicity of suits. Paraphrasing Gonzales v. Commission on Elections,21 it is
evident that strong reasons of public policy demand that the constitutionality issue be resolved now.22

In further support of the immediate resolution of the constitutionality issue, public respondents
cite Acop v. Guingona,23 to the effect that the courts will decide a question -- otherwise moot and
academic -- if it is "capable of repetition, yet evading review."24 Public respondents ask the Court to
avoid a situation in which the constitutionality issue may again arise with respect to another FTAA, the
resolution of which may not be achieved until after it has become too late for our mining industry to
grow out of its infancy. They also recall Salonga v. Cruz Paño,25 in which this Court declared
that "(t)he Court also has the duty to formulate guiding and controlling constitutional principles,
precepts, doctrines or rules. It has the symbolic function of educating the bench and bar on the extent
of protection given by constitutional guarantees. x x x."
106

The mootness of the case in relation to the WMCP FTAA led the undersigned ponente to state in his
dissent to the Decision that there was no more justiciable controversy and the plea to nullify the
Mining Law has become a virtual petition for declaratory relief.26 The entry of the Chamber of Mines of
the Philippines, Inc., however, has put into focus the seriousness of the allegations of
unconstitutionality of RA 7942 and DAO 96-40 which converts the case to one for prohibition27 in the
enforcement of the said law and regulations.

Indeed, this CMP entry brings to fore that the real issue in this case is whether paragraph 4 of Section
2 of Article XII of the Constitution is contravened by RA 7942 and DAO 96-40, not whether it was
violated by specific acts implementing RA 7942 and DAO 96-40. "[W]hen an act of the legislative
department is seriously alleged to have infringed the Constitution, settling the controversy becomes
the duty of this Court. By the mere enactment of the questioned law or the approval of the challenged
action, the dispute is said to have ripened into a judicial controversy even without any other overt
act."28 This ruling can be traced from Tañada v. Angara,29 in which the Court said:

"In seeking to nullify an act of the Philippine Senate on the ground that it contravenes the
Constitution, the petition no doubt raises a justiciable controversy. Where an action of the
legislative branch is seriously alleged to have infringed the Constitution, it becomes not only
the right but in fact the duty of the judiciary to settle the dispute.

xxxxxxxxx

"As this Court has repeatedly and firmly emphasized in many cases, it will not shirk, digress
from or abandon its sacred duty and authority to uphold the Constitution in matters that
involve grave abuse of discretion brought before it in appropriate cases, committed by any
officer, agency, instrumentality or department of the government."30

Additionally, the entry of CMP into this case has also effectively forestalled any possible objections
arising from the standing or legal interest of the original parties.

For all the foregoing reasons, we believe that the Court should proceed to a resolution of the
constitutional issues in this case.

Third Issue:

The Proper Interpretation of the Constitutional Phrase


"Agreements Involving Either Technical or Financial Assistance"

The constitutional provision at the nucleus of the controversy is paragraph 4 of Section 2 of Article XII
of the 1987 Constitution. In order to appreciate its context, Section 2 is reproduced in full:

"Sec. 2. All lands of the public domain, waters, minerals, coal, petroleum, and other mineral
oils, all forces of potential energy, fisheries, forests or timber, wildlife, flora and fauna, and
other natural resources are owned by the State. With the exception of agricultural lands, all
other natural resources shall not be alienated. The exploration, development and utilization of
natural resources shall be under the full control and supervision of the State. The State may
directly undertake such activities, or it may enter into co-production, joint venture or
production-sharing agreements with Filipino citizens or corporations or associations at least
sixty per centum of whose capital is owned by such citizens. Such agreements may be for a
period not exceeding twenty-five years, renewable for not more than twenty-five years, and
under such terms and conditions as may be provided by law. In cases of water rights for
irrigation, water supply, fisheries, or industrial uses other than the development of water
power, beneficial use may be the measure and limit of the grant.

"The State shall protect the nation's marine wealth in its archipelagic waters, territorial sea,
and exclusive economic zone, and reserve its use and enjoyment exclusively to Filipino
citizens.
107

"The Congress may, by law, allow small-scale utilization of natural resources by Filipino
citizens, as well as cooperative fish farming, with priority to subsistence fishermen and fish-
workers in rivers, lakes, bays and lagoons.

"The President may enter into agreements with foreign-owned corporations involving either
technical or financial assistance for large-scale exploration, development, and
utilization of minerals, petroleum, and other mineral oils according to the general terms
and conditions provided by law, based on real contributions to the economic growth and
general welfare of the country. In such agreements, the State shall promote the development
and use of local scientific and technical resources.

"The President shall notify the Congress of every contract entered into in accordance with this
provision, within thirty days from its execution."31

No Restriction of Meaning by
a Verba Legis Interpretation

To interpret the foregoing provision, petitioners adamantly assert that the language of the Constitution
should prevail; that the primary method of interpreting it is to seek the ordinary meaning of the words
used in its provisions. They rely on rulings of this Court, such as the following:

"The fundamental principle in constitutional construction however is that the primary source
from which to ascertain constitutional intent or purpose is the language of the provision itself.
The presumption is that the words in which the constitutional provisions are couched express
the objective sought to be attained. In other words, verba legis prevails. Only when the
meaning of the words used is unclear and equivocal should resort be made to extraneous
aids of construction and interpretation, such as the proceedings of the Constitutional
Commission or Convention to shed light on and ascertain the true intent or purpose of the
provision being construed."32

Very recently, in Francisco v. The House of Representatives,33 this Court indeed had the occasion to
reiterate the well-settled principles of constitutional construction:

"First, verba legis, that is, wherever possible, the words used in the Constitution must be
given their ordinary meaning except where technical terms are employed. x x x.

xxxxxxxxx

"Second, where there is ambiguity, ratio legis est anima. The words of the Constitution should
be interpreted in accordance with the intent of its framers. x x x.

xxxxxxxxx

"Finally, ut magis valeat quam pereat. The Constitution is to be interpreted as a whole."34

For ease of reference and in consonance with verba legis, we reconstruct and stratify the aforequoted
Section 2 as follows:

1. All natural resources are owned by the State. Except for agricultural lands, natural
resources cannot be alienated by the State.

2. The exploration, development and utilization (EDU) of natural resources shall be under the
full control and supervision of the State.

3. The State may undertake these EDU activities through either of the following:

(a) By itself directly and solely


108

(b) By (i) co-production; (ii) joint venture; or (iii) production sharing agreements with
Filipino citizens or corporations, at least 60 percent of the capital of which is owned
by such citizens

4. Small-scale utilization of natural resources may be allowed by law in favor of Filipino


citizens.

5. For large-scale EDU of minerals, petroleum and other mineral oils, the President may enter
into "agreements with foreign-owned corporations involving either technical or financial
assistance according to the general terms and conditions provided by law x x x."

Note that in all the three foregoing mining activities -- exploration, development and utilization -- the
State may undertake such EDU activities by itself or in tandem with Filipinos or Filipino corporations,
except in two instances: first, in small-scale utilization of natural resources, which Filipinos may be
allowed by law to undertake; and second, in large-scale EDU of minerals, petroleum and mineral oils,
which may be undertaken by the State via "agreementswith foreign-owned corporations involving
either technical or financial assistance" as provided by law.

Petitioners claim that the phrase "agreements x x x involving either technical or financial
assistance" simply means technical assistance or financial assistance agreements, nothing more and
nothing else. They insist that there is no ambiguity in the phrase, and that a plain reading of
paragraph 4 quoted above leads to the inescapable conclusion that what a foreign-owned corporation
may enter into with the government is merely an agreement for eitherfinancial or technical
assistance only, for the large-scale exploration, development and utilization of minerals, petroleum
and other mineral oils; such a limitation, they argue, excludes foreign management and operation of a
mining enterprise.35

This restrictive interpretation, petitioners believe, is in line with the general policy enunciated by the
Constitution reserving to Filipino citizens and corporations the use and enjoyment of the country's
natural resources. They maintain that this Court's Decision36 of January 27, 2004 correctly declared
the WMCP FTAA, along with pertinent provisions of RA 7942, void for allowing a foreign contractor to
have direct and exclusive management of a mining enterprise. Allowing such a privilege not only runs
counter to the "full control and supervision" that the State is constitutionally mandated to exercise over
the exploration, development and utilization of the country's natural resources; doing so also vests in
the foreign company "beneficial ownership" of our mineral resources. It will be recalled that the
Decision of January 27, 2004 zeroed in on "management or other forms of assistance" or other
activities associated with the "service contracts" of the martial law regime, since "the management or
operation of mining activities by foreign contractors, which is the primary feature of service contracts,
was precisely the evil that the drafters of the 1987 Constitution sought to eradicate."

On the other hand, the intervenor37 and public respondents argue that the FTAA allowed by paragraph
4 is not merely an agreement for supplying limited and specific financial or technical services to the
State. Rather, such FTAA is a comprehensive agreement for the foreign-owned
corporation's integrated exploration, development and utilization of mineral, petroleum or other
mineral oils on a large-scale basis. The agreement, therefore, authorizes the foreign contractor's
rendition of a whole range of integrated and comprehensive services, ranging from the discovery to
the development, utilization and production of minerals or petroleum products.

We do not see how applying a strictly literal or verba legis interpretation of paragraph 4 could
inexorably lead to the conclusions arrived at in the ponencia. First, the drafters' choice of words --
their use of the phrase agreements x x x involving either technical or financial assistance -- does not
indicate the intent to exclude other modes of assistance. The drafters opted to use involving when
they could have simply said agreements for financial or technical assistance, if that was their intention
to begin with. In this case, the limitation would be very clear and no further debate would ensue.

In contrast, the use of the word "involving" signifies the possibility of the inclusion of other forms
of assistance or activities having to do with, otherwise related to or compatible with financial or
technical assistance. The word "involving" as used in this context has three connotations that can be
109

differentiated thus: one, the sense of "concerning," "having to do with," or "affecting"; two, "entailing,"
"requiring," "implying" or "necessitating"; and three, "including," "containing" or "comprising."38

Plainly, none of the three connotations convey a sense of exclusivity. Moreover, the word "involving,"
when understood in the sense of "including," as in including technical or financial
assistance, necessarily implies that there are activities other than those that are being included. In
other words, if an agreement includes technical or financial assistance, there is apart from such
assistance -- something else already in, and covered or may be covered by, the said agreement.

In short, it allows for the possibility that matters, other than those explicitly mentioned, could be made
part of the agreement. Thus, we are now led to the conclusion that the use of the word "involving"
implies that these agreements with foreign corporations are not limited to mere financial or technical
assistance. The difference in sense becomes very apparent when we juxtapose
"agreements for technical or financial assistance" against "agreements including technical or
financial assistance." This much is unalterably clear in a verba legis approach.

Second, if the real intention of the drafters was to confine foreign corporations to financial or technical
assistance and nothing more, their language would have certainly been so unmistakably restrictive
and stringent as to leave no doubt in anyone's mind about their true intent. For example, they would
have used the sentence foreign corporations are absolutely prohibited from involvement in the
management or operation of mining or similar ventures or words of similar import. A search for such
stringent wording yields negative results. Thus, we come to the inevitable conclusion that there
was a conscious and deliberate decision to avoid the use of restrictive wording that bespeaks
an intent not to use the expression "agreements x x x involving either technical or financial
assistance" in an exclusionary and limiting manner.

Deletion of "Service Contracts" to


Avoid Pitfalls of Previous Constitutions,
Not to Ban Service Contracts Per Se

Third, we do not see how a verba legis approach leads to the conclusion that "the management or
operation of mining activities by foreign contractors, which is the primary feature of service contracts,
was precisely the evil that the drafters of the 1987 Constitution sought to eradicate." Nowhere in the
above-quoted Section can be discerned the objective to keep out of foreign hands the management
or operation of mining activities or the plan to eradicate service contracts as these were understood in
the 1973 Constitution. Still, petitioners maintain that the deletion or omission from the 1987
Constitution of the term "service contracts" found in the 1973 Constitution sufficiently proves the
drafters' intent to exclude foreigners from the management of the affected enterprises.

To our mind, however, such intent cannot be definitively and conclusively established from the mere
failure to carry the same expression or term over to the new Constitution, absent a more specific,
explicit and unequivocal statement to that effect. What petitioners seek (a complete ban on foreign
participation in the management of mining operations, as previously allowed by the earlier
Constitutions) is nothing short of bringing about a momentous sea change in the economic and
developmental policies; and the fundamentally capitalist, free-enterprise philosophy of our
government. We cannot imagine such a radical shift being undertaken by our government, to the
great prejudice of the mining sector in particular and our economy in general, merely on the basis of
the omission of the terms service contract from or the failure to carry them over to the new
Constitution. There has to be a much more definite and even unarguable basis for such a drastic
reversal of policies.

Fourth, a literal and restrictive interpretation of paragraph 4, such as that proposed by petitioners,
suffers from certain internal logical inconsistencies that generate ambiguities in the understanding of
the provision. As the intervenor pointed out, there has never been any constitutional or statutory
provision that reserved to Filipino citizens or corporations, at least 60 percent of which is Filipino-
owned, the rendition of financial or technical assistance to companies engaged in mining or the
development of any other natural resource. The taking out of foreign-currency or peso-denominated
loans or any other kind of financial assistance, as well as the rendition of technical assistance --
whether to the State or to any other entity in the Philippines -- has never been restricted in favor of
110

Filipino citizens or corporations having a certain minimum percentage of Filipino equity. Such a
restriction would certainly be preposterous and unnecessary. As a matter of fact, financial, and even
technical assistance, regardless of the nationality of its source, would be welcomed in the mining
industry anytime with open arms, on account of the dearth of local capital and the need to continually
update technological know-how and improve technical skills.

There was therefore no need for a constitutional provision specifically allowing foreign-owned
corporations to render financial or technical assistance, whether in respect of mining or some other
resource development or commercial activity in the Philippines. The last point needs to be
emphasized: if merely financial or technical assistance agreements are allowed, there would
be no need to limit them to large-scale mining operations, as there would be far greater need
for them in the smaller-scale mining activities (and even in non-mining areas). Obviously, the
provision in question was intended to refer to agreements other than those for mere financial
or technical assistance.

In like manner, there would be no need to require the President of the Republic to report to Congress,
if only financial or technical assistance agreements are involved. Such agreements are in the nature
of foreign loans that -- pursuant to Section 20 of Article VII39 of the 1987 Constitution -- the President
may contract or guarantee, merely with the prior concurrence of the Monetary Board. In turn, the
Board is required to report to Congress within thirty days from the end of every quarter of the calendar
year, not thirty days after the agreement is entered into.

And if paragraph 4 permits only agreements for loans and other forms of financial, or technical
assistance, what is the point of requiring that they be based on real contributions to the economic
growth and general welfare of the country? For instance, how is one to measure and assess the "real
contributions" to the "economic growth" and "general welfare" of the country that may ensue from a
foreign-currency loan agreement or a technical-assistance agreement for, say, the refurbishing of an
existing power generating plant for a mining operation somewhere in Mindanao? Such a criterion
would make more sense when applied to a major business investment in a principal sector of the
industry.

The conclusion is clear and inescapable -- a verba legis construction shows that paragraph 4 is not to
be understood as one limited only to foreign loans (or other forms of financial support) and to
technical assistance. There is definitely more to it than that. These are provisions permitting
participation by foreign companies; requiring the President's report to Congress; and using,
as yardstick, contributions based on economic growth and general welfare. These were
neither accidentally inserted into the Constitution nor carelessly cobbled together by the
drafters in lip service to shallow nationalism. The provisions patently have significance and
usefulness in a context that allows agreements with foreign companies to include more than mere
financial or technical assistance.

Fifth, it is argued that Section 2 of Article XII authorizes nothing more than a rendition of specific and
limited financial service or technical assistance by a foreign company. This argument begs the
question "To whom or for whom would it be rendered"? or Who is being assisted? If the answer is
"The State," then it necessarily implies that the State itself is the one directly and solely undertaking
the large-scale exploration, development and utilization of a mineral resource, so it follows that the
State must itself bear the liability and cost of repaying the financing sourced from the foreign lender
and/or of paying compensation to the foreign entity rendering technical assistance.

However, it is of common knowledge, and of judicial notice as well, that the government is and has for
many many years been financially strapped, to the point that even the most essential services have
suffered serious curtailments -- education and health care, for instance, not to mention judicial
services -- have had to make do with inadequate budgetary allocations. Thus, government has had to
resort to build-operate-transfer and similar arrangements with the private sector, in order to get vital
infrastructure projects built without any governmental outlay.

The very recent brouhaha over the gargantuan "fiscal crisis" or "budget deficit" merely confirms what
the ordinary citizen has suspected all along. After the reality check, one will have to admit the
implausibility of a direct undertaking -- by the State itself -- of large-scale exploration, development
111

and utilization of minerals, petroleum and other mineral oils. Such an undertaking entails not only
humongous capital requirements, but also the attendant risk of never finding and developing
economically viable quantities of minerals, petroleum and other mineral oils. 40

It is equally difficult to imagine that such a provision restricting foreign companies to the rendition of
only financial or technical assistance to the government was deliberately crafted by the drafters of the
Constitution, who were all well aware of the capital-intensive and technology-oriented nature of large-
scale mineral or petroleum extraction and the country's deficiency in precisely those areas. 41 To say
so would be tantamount to asserting that the provision was purposely designed to ladle the large-
scale development and utilization of mineral, petroleum and related resources with impossible
conditions; and to remain forever and permanently "reserved" for future generations of Filipinos.

A More Reasonable Look


at the Charter's Plain Language

Sixth, we shall now look closer at the plain language of the Charter and examining the logical
inferences. The drafters chose to emphasize and highlight agreements x x x involving either technical
or financial assistance in relation to foreign corporations' participation in large-scale EDU. The
inclusion of this clause on "technical or financial assistance" recognizes the fact that foreign business
entities and multinational corporations are the ones with the resources and know-how to provide
technical and/or financial assistance of the magnitude and type required for large-scale exploration,
development and utilization of these resources.

The drafters -- whose ranks included many academicians, economists, businessmen, lawyers,
politicians and government officials -- were not unfamiliar with the practices of foreign corporations
and multinationals.

Neither were they so naïve as to believe that these entities would provide "assistance" without
conditionalities or some quid pro quo. Definitely, as business persons well know and as a matter of
judicial notice, this matter is not just a question of signing a promissory note or executing a technology
transfer agreement. Foreign corporations usually require that they be given a say in the management,
for instance, of day-to-day operations of the joint venture. They would demand the appointment of
their own men as, for example, operations managers, technical experts, quality control heads, internal
auditors or comptrollers. Furthermore, they would probably require seats on the Board of Directors --
all these to ensure the success of the enterprise and the repayment of the loans and other financial
assistance and to make certain that the funding and the technology they supply would not go to
waste. Ultimately, they would also want to protect their business reputation and bottom lines. 42

In short, the drafters will have to be credited with enough pragmatism and savvy to know that these
foreign entities will not enter into such "agreements involving assistance" without requiring
arrangements for the protection of their investments, gains and benefits.

Thus, by specifying such "agreements involving assistance," the drafters necessarily gave implied
assent to everything that these agreements necessarily entailed; or that could reasonably be deemed
necessary to make them tenable and effective, including management authority with respect to the
day-to-day operations of the enterprise and measures for the protection of the interests of the foreign
corporation, PROVIDED THAT Philippine sovereignty over natural resources and full control over the
enterprise undertaking the EDU activities remain firmly in the State.

Petitioners' Theory Deflated by the


Absence of Closing-Out Rules or Guidelines

Seventh and final point regarding the plain-language approach, one of the practical difficulties that
results from it is the fact that there is nothing by way of transitory provisions that would serve to
confirm the theory that the omission of the term "service contract" from the 1987 Constitution signaled
the demise of service contracts.
112

The framers knew at the time they were deliberating that there were various service contracts extant
and in force and effect, including those in the petroleum industry. Many of these service contracts
were long-term (25 years) and had several more years to run. If they had meant to ban service
contracts altogether, they would have had to provide for the termination or pretermination of the
existing contracts. Accordingly, they would have supplied the specifics and the when and how of
effecting the extinguishment of these existing contracts (or at least the mechanics for determining
them); and of putting in place the means to address the just claims of the contractors for
compensation for their investments, lost opportunities, and so on, if not for the recovery thereof.

If the framers had intended to put an end to service contracts, they would have at least left specific
instructions to Congress to deal with these closing-out issues, perhaps by way of general guidelines
and a timeline within which to carry them out. The following are some extant examples of such
transitory guidelines set forth in Article XVIII of our Constitution:

"Section 23. Advertising entities affected by paragraph (2), Section 11 of Article XVI of this
Constitution shall have five years from its ratification to comply on a graduated and
proportionate basis with the minimum Filipino ownership requirement therein.

xxxxxxxxx

"Section 25. After the expiration in 1991 of the Agreement between the Republic of the
Philippines and the United States of America concerning military bases, foreign military
bases, troops, or facilities shall not be allowed in the Philippines except under a treaty duly
concurred in by the Senate and, when the Congress so requires, ratified by a majority of the
votes cast by the people in a national referendum held for that purpose, and recognized as a
treaty by the other contracting State.

"Section 26. The authority to issue sequestration or freeze orders under Proclamation No. 3
dated March 25, 1986 in relation to the recovery of ill-gotten wealth shall remain operative for
not more than eighteen months after the ratification of this Constitution. However, in the
national interest, as certified by the President, the Congress may extend such period.

A sequestration or freeze order shall be issued only upon showing of a prima facie case. The
order and the list of the sequestered or frozen properties shall forthwith be registered with the
proper court. For orders issued before the ratification of this Constitution, the corresponding
judicial action or proceeding shall be filed within six months from its ratification. For those
issued after such ratification, the judicial action or proceeding shall be commenced within six
months from the issuance thereof.

The sequestration or freeze order is deemed automatically lifted if no judicial action or


proceeding is commenced as herein provided." 43]

It is inconceivable that the drafters of the Constitution would leave such an important matter -- an
expression of sovereignty as it were -- indefinitely hanging in the air in a formless and ineffective
state. Indeed, the complete absence of even a general framework only serves to further deflate
petitioners' theory, like a child's balloon losing its air.

Under the circumstances, the logical inconsistencies resulting from petitioners' literal and purely verba
legisapproach to paragraph 4 of Section 2 of Article XII compel a resort to other aids to interpretation.

Petitioners' Posture Also Negated


by Ratio Legis Et Anima

Thus, in order to resolve the inconsistencies, incongruities and ambiguities encountered and to supply
the deficiencies of the plain-language approach, there is a need for recourse to the proceedings of the
1986 Constitutional Commission. There is a need for ratio legis et anima.
113

Service Contracts Not


"Deconstitutionalized"

Pertinent portions of the deliberations of the members of the Constitutional Commission (ConCom)
conclusively show that they discussed agreements involving either technical or financial assistance in
the same breadth as service contracts and used the terms interchangeably. The following exchange
between Commissioner Jamir (sponsor of the provision) and Commissioner Suarez irrefutably proves
that the "agreements involving technical or financial assistance" were none other than service
contracts.

THE PRESIDENT. Commissioner Jamir is recognized. We are still on Section 3.

MR. JAMIR. Yes, Madam President. With respect to the second paragraph of Section 3, my
amendment by substitution reads: THE PRESIDENT MAY ENTER INTO AGREEMENTS
WITH FOREIGN-OWNED CORPORATIONS INVOLVING EITHER TECHNICAL OR
FINANCIAL ASSISTANCE FOR LARGE-SCALE EXPLORATION, DEVELOPMENT AND
UTILIZATION OF NATURAL RESOURCES ACCORDING TO THE TERMS AND
CONDITIONS PROVIDED BY LAW.

MR. VILLEGAS. The Committee accepts the amendment. Commissioner Suarez will give the
background.

MR. JAMIR. Thank you.

THE PRESIDENT. Commissioner Suarez is recognized.

MR. SUAREZ. Thank you, Madam President.

Will Commissioner Jamir answer a few clarificatory questions?

MR. JAMIR. Yes, Madam President.

MR. SUAREZ. This particular portion of the section has reference to what was popularly
known before as service contracts, among other things, is that correct?

MR. JAMIR. Yes, Madam President.

MR. SUAREZ. As it is formulated, the President may enter into service contracts but subject
to the guidelines that may be promulgated by Congress?

MR. JAMIR. That is correct.

MR. SUAREZ. Therefore, that aspect of negotiation and consummation will fall on the
President, not upon Congress?

MR. JAMIR. That is also correct, Madam President.

MR. SUAREZ. Except that all of these contracts, service or otherwise, must be made
strictly in accordance with guidelines prescribed by Congress?

MR. JAMIR. That is also correct.

MR. SUAREZ. And the Gentleman is thinking in terms of a law that uniformly covers
situations of the same nature?

MR. JAMIR. That is 100 percent correct.


114

MR. SUAREZ. I thank the Commissioner.

MR. JAMIR. Thank you very much.44

The following exchange leaves no doubt that the commissioners knew exactly what they were dealing
with: service contracts.

THE PRESIDENT. Commissioner Gascon is recognized.

MR. GASCON. Commissioner Jamir had proposed an amendment with regard to


special service contractswhich was accepted by the Committee. Since the Committee has
accepted it, I would like to ask some questions.

THE PRESIDENT. Commissioner Gascon may proceed.

MR. GASCON. As it is proposed now, such service contracts will be entered into by the
President with the guidelines of a general law on service contract to be enacted by
Congress. Is that correct?

MR. VILLEGAS. The Commissioner is right, Madam President.

MR. GASCON. According to the original proposal, if the President were to enter into a
particular agreement, he would need the concurrence of Congress. Now that it has been
changed by the proposal of Commissioner Jamir in that Congress will set the general law to
which the President shall comply, the President will, therefore, not need the concurrence of
Congress every time he enters into service contracts. Is that correct?

MR. VILLEGAS. That is right.

MR. GASCON. The proposed amendment of Commissioner Jamir is in indirect contrast to my


proposed amendment, so I would like to object and present my proposed amendment to the
body.

xxxxxxxxx

MR. GASCON. Yes, it will be up to the body.

I feel that the general law to be set by Congress as regard service contract
agreements which the President will enter into might be too general or since we do not know
the content yet of such a law, it might be that certain agreements will be detrimental to the
interest of the Filipinos. This is in direct contrast to my proposal which provides that there be
effective constraints in the implementation of service contracts.

So instead of a general law to be passed by Congress to serve as a guideline to the


President when entering into service contract agreements, I propose that every service
contract entered into by the President would need the concurrence of Congress, so as to
assure the Filipinos of their interests with regard to the issue in Section 3 on all lands of the
public domain. My alternative amendment, which we will discuss later, reads: THAT THE
PRESIDENT SHALL ENTER INTO SUCH AGREEMENTS ONLY WITH THE
CONCURRENCE OF TWO-THIRDS VOTE OF ALL THE MEMBERS OF CONGRESS
SITTING SEPARATELY.

xxxxxxxxx

MR. BENGZON. The reason we made that shift is that we realized the original proposal could
breed corruption. By the way, this is not just confined to service contracts but also
to financial assistance. If we are going to make every single contract subject to the
115

concurrence of Congress – which, according to the Commissioner's amendment is the


concurrence of two-thirds of Congress voting separately – then (1) there is a very great
chance that each contract will be different from another; and (2) there is a great temptation
that it would breed corruption because of the great lobbying that is going to happen. And we
do not want to subject our legislature to that.

Now, to answer the Commissioner's apprehension, by "general law," we do not mean


statements of motherhood. Congress can build all the restrictions that it wishes into that
general law so that every contract entered into by the President under that specific area will
have to be uniform. The President has no choice but to follow all the guidelines that will be
provided by law.

MR. GASCON. But my basic problem is that we do not know as of yet the contents of such a
general law as to how much constraints there will be in it. And to my mind, although the
Committee's contention that the regular concurrence from Congress would subject Congress
to extensive lobbying, I think that is a risk we will have to take since Congress is a body of
representatives of the people whose membership will be changing regularly as there will be
changing circumstances every time certain agreements are made. It would be best then to
keep in tab and attuned to the interest of the Filipino people, whenever the President enters
into any agreement with regard to such an important matter as technical or financial
assistance for large-scale exploration, development and utilization of natural
resources or service contracts, the people's elected representatives should be on top of it.

xxxxxxxxx

MR. OPLE. Madam President, we do not need to suspend the session. If Commissioner
Gascon needs a few minutes, I can fill up the remaining time while he completes his
proposed amendment. I just wanted to ask Commissioner Jamir whether he would entertain a
minor amendment to his amendment, and it reads as follows: THE PRESIDENT SHALL
SUBSEQUENTLY NOTIFY CONGRESS OF EVERY SERVICE CONTRACT ENTERED
INTO IN ACCORDANCE WITH THE GENERAL LAW. I think the reason is, if I may state it
briefly, as Commissioner Bengzon said, Congress can always change the general law later
on to conform to new perceptions of standards that should be built into service contracts.
But the only way Congress can do this is if there were a notification requirement from the
Office of the President that such service contracts had been entered into, subject then to the
scrutiny of the Members of Congress. This pertains to a situation where the service
contracts are already entered into, and all that this amendment seeks is the reporting
requirement from the Office of the President. Will Commissioner Jamir entertain that?

MR. JAMIR. I will gladly do so, if it is still within my power.

MR. VILLEGAS. Yes, the Committee accepts the amendment.

xxxxxxxxx

SR. TAN. Madam President, may I ask a question?

THE PRESIDENT. Commissioner Tan is recognized.

SR. TAN. Am I correct in thinking that the only difference between these future service
contracts and the past service contracts under Mr. Marcos is the general law to be enacted
by the legislature and the notification of Congress by the President? That is the only
difference, is it not?

MR. VILLEGAS. That is right.

SR. TAN. So those are the safeguards.


116

MR. VILLEGAS. Yes. There was no law at all governing service contracts before.

SR. TAN. Thank you, Madam President.45

More Than Mere Financial


and Technical Assistance
Entailed by the Agreements

The clear words of Commissioner Jose N. Nolledo quoted below explicitly and eloquently
demonstrate that the drafters knew that the agreements with foreign corporations were going to entail
not mere technical or financial assistance but, rather, foreign investment in and management of an
enterprise involved in large-scale exploration, development and utilization of minerals, petroleum, and
other mineral oils.

THE PRESIDENT. Commissioner Nolledo is recognized.

MR. NOLLEDO. Madam President, I have the permission of the Acting Floor Leader to speak
for only two minutes in favor of the amendment of Commissioner Gascon.

THE PRESIDENT. Commissioner Nolledo may proceed.

MR. NOLLEDO. With due respect to the members of the Committee and Commissioner
Jamir, I am in favor of the objection of Commissioner Gascon.

Madam President, I was one of those who refused to sign the 1973 Constitution, and
one of the reasons is that there were many provisions in the Transitory Provisions
therein that favored aliens. I was shocked when I read a provision
authorizing service contracts while we, in this Constitutional Commission, provided
for Filipino control of the economy. We are, therefore, providing for exceptional
instances where aliens may circumvent Filipino control of our economy. And one way
of circumventing the rule in favor of Filipino control of the economy is to
recognize service contracts.

As far as I am concerned, if I should have my own way, I am for the complete


deletion of this provision. However, we are presenting a compromise in the sense
that we are requiring a two-thirds vote of all the Members of Congress as a
safeguard. I think we should not mistrust the future Members of Congress by saying
that the purpose of this provision is to avoid corruption. We cannot claim that they are
less patriotic than we are. I think the Members of this Commission should know that
entering into service contracts is an exception to the rule on protection of natural
resources for the interest of the nation, and therefore, being an exception it should be
subject, whenever possible, to stringent rules. It seems to me that we are liberalizing
the rules in favor of aliens.

I say these things with a heavy heart, Madam President. I do not claim to be a
nationalist, but I love my country. Although we need investments, we must adopt
safeguards that are truly reflective of the sentiments of the people and not mere
cosmetic safeguards as they now appear in the Jamir amendment. (Applause)

Thank you, Madam President.46

Another excerpt, featuring then Commissioner (now Chief Justice) Hilario G. Davide Jr., indicates the
limitations of the scope of such service contracts -- they are valid only in regard to minerals,
petroleum and other mineral oils, not to all natural resources.

THE PRESIDENT. Commissioner Davide is recognized.


117

MR. DAVIDE. Thank you, Madam President. This is an amendment to the Jamir amendment
and also to the Ople amendment. I propose to delete "NATURAL RESOURCES" and
substitute it with the following: MINERALS, PETROLEUM AND OTHER MINERAL OILS. On
the Ople amendment, I propose to add: THE NOTIFICATION TO CONGRESS SHALL BE
WITHIN THIRTY DAYS FROM THE EXECUTION OF THE SERVICE CONTRACT.

THE PRESIDENT. What does the Committee say with respect to the first amendment in lieu
of "NATURAL RESOURCES"?

MR. VILLEGAS. Could Commissioner Davide explain that?

MR. DAVIDE. Madam President, with the use of "NATURAL RESOURCES" here, it would
necessarily include all lands of the public domain, our marine resources, forests, parks and so
on. So we would like to limit the scope of these service contracts to those areas really
where these may be needed, the exploitation, development and exploration of minerals,
petroleum and other mineral oils. And so, we believe that we should really, if we want to
grant service contracts at all, limit the same to only those particular areas where Filipino
capital may not be sufficient, and not to all natural resources.

MR. SUAREZ. Just a point of clarification again, Madam President. When the Commissioner
made those enumerations and specifications, I suppose he deliberately did not include
"agricultural land"?

MR. DAVIDE. That is precisely the reason we have to enumerate what these resources are
into which service contracts may enter. So, beyond the reach of any service contract will
be lands of the public domain, timberlands, forests, marine resources, fauna and flora, wildlife
and national parks.47

After the Jamir amendment was voted upon and approved by a vote of 21 to 10 with 2 abstentions,
Commissioner Davide made the following statement, which is very relevant to our quest:

THE PRESIDENT. Commissioner Davide is recognized.

MR. DAVIDE. I am very glad that Commissioner Padilla emphasized minerals, petroleum and
mineral oils. The Commission has just approved the possible foreign entry into the
development, exploration and utilization of these minerals, petroleum and other mineral oils
by virtue of the Jamir amendment. I voted in favor of the Jamir amendment because it will
eventually give way to vesting in exclusively Filipino citizens and corporations wholly owned
by Filipino citizens the right to utilize the other natural resources. This means that as a matter
of policy, natural resources should be utilized and exploited only by Filipino citizens or
corporations wholly owned by such citizens. But by virtue of the Jamir amendment, since we
feel that Filipino capital may not be enough for the development and utilization of minerals,
petroleum and other mineral oils, the President can enter into service contracts with foreign
corporations precisely for the development and utilization of such resources. And so, there is
nothing to fear that we will stagnate in the development of minerals, petroleum and mineral
oils because we now allow service contracts. x x x."48

The foregoing are mere fragments of the framers' lengthy discussions of the provision dealing
with agreements x x x involving either technical or financial assistance, which ultimately became
paragraph 4 of Section 2 of Article XII of the Constitution. Beyond any doubt, the members of the
ConCom were actually debating about the martial-law-era service contracts for which they were
crafting appropriate safeguards.

In the voting that led to the approval of Article XII by the ConCom, the explanations given by
Commissioners Gascon, Garcia and Tadeo indicated that they had voted to reject this provision on
account of their objections to the "constitutionalization" of the "service contract" concept.
118

Mr. Gascon said, "I felt that if we would constitutionalize any provision on service contracts, this
should always be with the concurrence of Congress and not guided only by a general law to be
promulgated by Congress."49 Mr. Garcia explained, "Service contracts are given constitutional
legitimization in Sec. 3, even when they have been proven to be inimical to the interests of the nation,
providing, as they do, the legal loophole for the exploitation of our natural resources for the benefit of
foreign interests."50 Likewise, Mr. Tadeo cited inter alia the fact that service contracts continued to
subsist, enabling foreign interests to benefit from our natural resources.51 It was hardly likely that
these gentlemen would have objected so strenuously, had the provision called for mere
technical or financial assistance and nothing more.

The deliberations of the ConCom and some commissioners' explanation of their votes leave no room
for doubt that the service contract concept precisely underpinned the commissioners' understanding
of the "agreements involving either technical or financial assistance."

Summation of the
Concom Deliberations

At this point, we sum up the matters established, based on a careful reading of the ConCom
deliberations, as follows:

· In their deliberations on what was to become paragraph 4, the framers used the
term service contracts in referring to agreements x x x involving either technical or financial
assistance.

· They spoke of service contracts as the concept was understood in the 1973 Constitution.

· It was obvious from their discussions that they were not about to ban or eradicate service
contracts.

· Instead, they were plainly crafting provisions to put in place safeguards that would eliminate
or minimize the abuses prevalent during the marital law regime. In brief, they were going to
permit service contracts with foreign corporations as contractors, but with safety measures to
prevent abuses, as an exception to the general norm established in the first paragraph of
Section 2 of Article XII. This provision reserves or limits to Filipino citizens -- and corporations
at least 60 percent of which is owned by such citizens -- the exploration, development and
utilization of natural resources.

· This provision was prompted by the perceived insufficiency of Filipino capital and the felt
need for foreign investments in the EDU of minerals and petroleum resources.

· The framers for the most part debated about the sort of safeguards that would be
considered adequate and reasonable. But some of them, having more "radical" leanings,
wanted to ban service contracts altogether; for them, the provision would permit aliens to
exploit and benefit from the nation's natural resources, which they felt should be reserved
only for Filipinos.

· In the explanation of their votes, the individual commissioners were heard by the entire
body. They sounded off their individual opinions, openly enunciated their philosophies, and
supported or attacked the provisions with fervor. Everyone's viewpoint was heard.

· In the final voting, the Article on the National Economy and Patrimony -- including paragraph
4 allowing service contracts with foreign corporations as an exception to the general norm in
paragraph 1 of Section 2 of the same article -- was resoundingly approved by a vote of 32 to
7, with 2 abstentions.

Agreements Involving Technical


119

or Financial Assistance Are

Service Contracts With Safeguards

From the foregoing, we are impelled to conclude that the phrase agreements involving either technical
or financial assistance, referred to in paragraph 4, are in fact service contracts. But unlike those of the
1973 variety, the new ones are between foreign corporations acting as contractors on the one hand;
and on the other, the government as principal or "owner" of the works. In the new service contracts,
the foreign contractors provide capital, technology and technical know-how, and managerial expertise
in the creation and operation of large-scale mining/extractive enterprises; and the government,
through its agencies (DENR, MGB), actively exercises control and supervision over the entire
operation.

Such service contracts may be entered into only with respect to minerals, petroleum and other
mineral oils. The grant thereof is subject to several safeguards, among which are these requirements:

(1) The service contract shall be crafted in accordance with a general law that will set
standard or uniform terms, conditions and requirements, presumably to attain a certain
uniformity in provisions and avoid the possible insertion of terms disadvantageous to the
country.

(2) The President shall be the signatory for the government because, supposedly before an
agreement is presented to the President for signature, it will have been vetted several times
over at different levels to ensure that it conforms to law and can withstand public scrutiny.

(3) Within thirty days of the executed agreement, the President shall report it to Congress to
give that branch of government an opportunity to look over the agreement and interpose
timely objections, if any.

Use of the Record of the

ConCom to Ascertain Intent

At this juncture, we shall address, rather than gloss over, the use of the "framers' intent" approach,
and the criticism hurled by petitioners who quote a ruling of this Court:

"While it is permissible in this jurisdiction to consult the debates and proceedings of the
constitutional convention in order to arrive at the reason and purpose of the resulting
Constitution, resort thereto may be had only when other guides fail as said proceedings are
powerless to vary the terms of the Constitution when the meaning is clear. Debates in the
constitutional convention 'are of value as showing the views of the individual members, and
as indicating the reason for their votes, but they give us no light as to the views of the large
majority who did not talk, much less the mass of our fellow citizens whose votes at the polls
gave that instrument the force of fundamental law. We think it safer to construe the
constitution from what appears upon its face.' The proper interpretation therefore depends
more on how it was understood by the people adopting it than in the framers' understanding
thereof."52

The notion that the deliberations reflect only the views of those members who spoke out and not the
views of the majority who remained silent should be clarified. We must never forget that those who
spoke out were heard by those who remained silent and did not react. If the latter were silent because
they happened not to be present at the time, they are presumed to have read the minutes and kept
abreast of the deliberations. By remaining silent, they are deemed to have signified their assent to
and/or conformity with at least some of the views propounded or their lack of objections thereto. It was
incumbent upon them, as representatives of the entire Filipino people, to follow the deliberations
closely and to speak their minds on the matter if they did not see eye to eye with the proponents of
the draft provisions.
120

In any event, each and every one of the commissioners had the opportunity to speak out and to vote
on the matter. Moreover, the individual explanations of votes are on record, and they show where
each delegate stood on the issues. In sum, we cannot completely denigrate the value or
usefulness of the record of the ConCom, simply because certain members chose not to speak
out.

It is contended that the deliberations therein did not necessarily reflect the thinking of the voting
population that participated in the referendum and ratified the Constitution. Verily, whether we like it or
not, it is a bit too much to assume that every one of those who voted to ratify the proposed Charter did
so only after carefully reading and mulling over it, provision by provision.

Likewise, it appears rather extravagant to assume that every one of those who did in fact bother to
read the draft Charter actually understood the import of its provisions, much less analyzed it vis-à-vis
the previous Constitutions. We believe that in reality, a good percentage of those who voted in favor
of it did so more out of faith and trust. For them, it was the product of the hard work and careful
deliberation of a group of intelligent, dedicated and trustworthy men and women of integrity and
conviction, whose love of country and fidelity to duty could not be questioned.

In short, a large proportion of the voters voted "yes" because the drafters, or a majority of them,
endorsed the proposed Constitution. What this fact translates to is the inescapable conclusion that
many of the voters in the referendum did not form their own isolated judgment about the draft Charter,
much less about particular provisions therein. They only relied or fell back and acted upon the
favorable endorsement or recommendation of the framers as a group. In other words, by voting yes,
they may be deemed to have signified their voluntary adoption of the understanding and interpretation
of the delegates with respect to the proposed Charter and its particular provisions. "If it's good enough
for them, it's good enough for me;" or, in many instances, "If it's good enough for President Cory
Aquino, it's good enough for me."

And even for those who voted based on their own individual assessment of the proposed Charter,
there is no evidence available to indicate that their assessment or understanding of its provisions was
in fact different from that of the drafters. This unwritten assumption seems to be petitioners' as well.
For all we know, this segment of voters must have read and understood the provisions of the
Constitution in the same way the framers had, an assumption that would account for the favorable
votes.

Fundamentally speaking, in the process of rewriting the Charter, the members of the ConCom as a
group were supposed to represent the entire Filipino people. Thus, we cannot but regard their views
as being very much indicative of the thinking of the people with respect to the matters deliberated
upon and to the Charter as a whole.

It is therefore reasonable and unavoidable to make the following conclusion, based on the
above arguments. As written by the framers and ratified and adopted by the people, the
Constitution allows the continued use of service contracts with foreign corporations -- as
contractors who would invest in and operate and manage extractive enterprises, subject to the
full control and supervision of the State -- sans the abuses of the past regime. The purpose is
clear: to develop and utilize our mineral, petroleum and other resources on a large scale for
the immediate and tangible benefit of the Filipino people.

In view of the foregoing discussion, we should reverse the Decision of January 27, 2004, and in fact
now hold a view different from that of the Decision, which had these findings: (a) paragraph 4 of
Section 2 of Article XII limits foreign involvement in the local mining industry to agreements strictly for
either financial or technical assistance only; (b) the same paragraph precludes agreements that grant
to foreign corporations the management of local mining operations, as such agreements are
purportedly in the nature of service contracts as these were understood under the 1973 Constitution;
(c) these service contracts were supposedly "de-constitutionalized" and proscribed by the omission of
the term service contracts from the 1987 Constitution; (d) since the WMCP FTAA contains provisions
permitting the foreign contractor to manage the concern, the said FTAA is invalid for being a
prohibited service contract; and (e) provisions of RA 7942 and DAO 96-40, which likewise grant
managerial authority to the foreign contractor, are also invalid and unconstitutional.
121

Ultimate Test: State's "Control"


Determinative of Constitutionality

But we are not yet at the end of our quest. Far from it. It seems that we are confronted with a possible
collision of constitutional provisions. On the one hand, paragraph 1 of Section 2 of Article XII explicitly
mandates the State to exercise "full control and supervision" over the exploration, development and
utilization of natural resources. On the other hand, paragraph 4 permits safeguarded service contracts
with foreign contractors. Normally, pursuant thereto, the contractors exercise management
prerogatives over the mining operations and the enterprise as a whole. There is thus a legitimate
ground to be concerned that either the State's full control and supervision may rule out any exercise
of management authority by the foreign contractor; or, the other way around, allowing the foreign
contractor full management prerogatives may ultimately negate the State's full control and
supervision.

Ut Magis Valeat
Quam Pereat

Under the third principle of constitutional construction laid down in Francisco -- ut magis valeat quam
pereat -- every part of the Constitution is to be given effect, and the Constitution is to be read and
understood as a harmonious whole. Thus, "full control and supervision" by the State must be
understood as one that does not preclude the legitimate exercise of management prerogatives by the
foreign contractor. Before any further discussion, we must stress the primacy and supremacy of the
principle of sovereignty and State control and supervision over all aspects of exploration,
development and utilization of the country's natural resources, as mandated in the first paragraph of
Section 2 of Article XII.

But in the next breadth we have to point out that "full control and supervision" cannot be taken literally
to mean that the State controls and supervises everything involved, down to the minutest details, and
makes all decisions required in the mining operations. This strained concept of control and
supervision over the mining enterprise would render impossible the legitimate exercise by the
contractors of a reasonable degree of management prerogative and authority necessary and
indispensable to their proper functioning.

For one thing, such an interpretation would discourage foreign entry into large-scale exploration,
development and utilization activities; and result in the unmitigated stagnation of this sector, to the
detriment of our nation's development. This scenario renders paragraph 4 inoperative and useless.
And as respondents have correctly pointed out, the government does not have to micro-manage the
mining operations and dip its hands into the day-to-day affairs of the enterprise in order for it to be
considered as having full control and supervision.

The concept of control53 adopted in Section 2 of Article XII must be taken to mean less than
dictatorial, all-encompassing control; but nevertheless sufficient to give the State the power to direct,
restrain, regulate and govern the affairs of the extractive enterprises. Control by the State may be on
a macro level, through the establishment of policies, guidelines, regulations, industry standards and
similar measures that would enable the government to control the conduct of affairs in various
enterprises and restrain activities deemed not desirable or beneficial.

The end in view is ensuring that these enterprises contribute to the economic development and
general welfare of the country, conserve the environment, and uplift the well-being of the affected
local communities. Such a concept of control would be compatible with permitting the foreign
contractor sufficient and reasonable management authority over the enterprise it invested in, in order
to ensure that it is operating efficiently and profitably, to protect its investments and to enable it to
succeed.

The question to be answered, then, is whether RA 7942 and its Implementing Rules enable the
government to exercise that degree of control sufficient to direct and regulate the conduct of
affairs of individual enterprises and restrain undesirable activities.
122

On the resolution of these questions will depend the validity and constitutionality of certain provisions
of the Philippine Mining Act of 1995 (RA 7942) and its Implementing Rules and Regulations (DAO 96-
40), as well as the WMCP FTAA.

Indeed, petitioners charge54 that RA 7942, as well as its Implementing Rules and Regulations, makes
it possible for FTAA contracts to cede full control and management of mining enterprises over to fully
foreign-owned corporations, with the result that the State is allegedly reduced to a passive regulator
dependent on submitted plans and reports, with weak review and audit powers. The State does not
supposedly act as the owner of the natural resources for and on behalf of the Filipino people; it
practically has little effective say in the decisions made by the enterprise. Petitioners then conclude
that the law, the implementing regulations, and the WMCP FTAA cede "beneficial ownership" of the
mineral resources to the foreign contractor.

A careful scrutiny of the provisions of RA 7942 and its Implementing Rules belies petitioners' claims.
Paraphrasing the Constitution, Section 4 of the statute clearly affirms the State's control thus:

"Sec. 4. Ownership of Mineral Resources. – Mineral resources are owned by the State and
the exploration, development, utilization and processing thereof shall be under its full control
and supervision. The State may directly undertake such activities or it may enter into mineral
agreements with contractors.

"The State shall recognize and protect the rights of the indigenous cultural communities to
their ancestral lands as provided for by the Constitution."

The aforequoted provision is substantively reiterated in Section 2 of DAO 96-40 as follows:

"Sec. 2. Declaration of Policy. All mineral resources in public and private lands within the
territory and exclusive economic zone of the Republic of the Philippines are owned by the
State. It shall be the responsibility of the State to promote their rational exploration,
development, utilization and conservation through the combined efforts of the Government
and private sector in order to enhance national growth in a way that effectively safeguards the
environment and protects the rights of affected communities."

Sufficient Control Over Mining


Operations Vested in the State
by RA 7942 and DAO 96-40

RA 7942 provides for the State's control and supervision over mining operations. The following
provisions thereof establish the mechanism of inspection and visitorial rights over mining operations
and institute reportorial requirements in this manner:

1. Sec. 8 which provides for the DENR's power of over-all supervision and periodic review for
"the conservation, management, development and proper use of the State's mineral
resources";

2. Sec. 9 which authorizes the Mines and Geosciences Bureau (MGB) under the DENR to
exercise "direct charge in the administration and disposition of mineral resources", and
empowers the MGB to "monitor the compliance by the contractor of the terms and conditions
of the mineral agreements", "confiscate surety and performance bonds", and deputize
whenever necessary any member or unit of the Phil. National Police, barangay, duly
registered non-governmental organization (NGO) or any qualified person to police mining
activities;

3. Sec. 66 which vests in the Regional Director "exclusive jurisdiction over safety inspections
of all installations, whether surface or underground", utilized in mining operations.

4. Sec. 35, which incorporates into all FTAAs the following terms, conditions and warranties:
123

"(g) Mining operations shall be conducted in accordance with the provisions of the Act
and its IRR.

"(h) Work programs and minimum expenditures commitments.

xxxxxxxxx

"(k) Requiring proponent to effectively use appropriate anti-pollution technology and


facilities to protect the environment and restore or rehabilitate mined-out areas.

"(l) The contractors shall furnish the Government records of geologic, accounting and
other relevant data for its mining operation, and that books of accounts and records
shall be open for inspection by the government. x x x.

"(m) Requiring the proponent to dispose of the minerals at the highest price and more
advantageous terms and conditions.

"(n) x x x x x x x x x

"(o) Such other terms and conditions consistent with the Constitution and with this Act
as the Secretary may deem to be for the best interest of the State and the welfare of
the Filipino people."

The foregoing provisions of Section 35 of RA 7942 are also reflected and


implemented in Section 56 (g), (h), (l), (m) and (n) of the Implementing Rules, DAO
96-40.

Moreover, RA 7942 and DAO 96-40 also provide various stipulations confirming the government's
control over mining enterprises:

· The contractor is to relinquish to the government those portions of the contract area not
needed for mining operations and not covered by any declaration of mining feasibility (Section
35-e, RA 7942; Section 60, DAO 96-40).

· The contractor must comply with the provisions pertaining to mine safety, health and
environmental protection (Chapter XI, RA 7942; Chapters XV and XVI, DAO 96-40).

· For violation of any of its terms and conditions, government may cancel an FTAA. (Chapter
XVII, RA 7942; Chapter XXIV, DAO 96-40).

· An FTAA contractor is obliged to open its books of accounts and records for inspection by
the government (Section 56-m, DAO 96-40).

· An FTAA contractor has to dispose of the minerals and by-products at the highest market
price and register with the MGB a copy of the sales agreement (Section 56-n, DAO 96-40).

· MGB is mandated to monitor the contractor's compliance with the terms and conditions of
the FTAA; and to deputize, when necessary, any member or unit of the Philippine National
Police, the barangay or a DENR-accredited nongovernmental organization to police mining
activities (Section 7-d and -f, DAO 96-40).

· An FTAA cannot be transferred or assigned without prior approval by the President (Section
40, RA 7942; Section 66, DAO 96-40).

· A mining project under an FTAA cannot proceed to the construction/development/utilization


stage, unless its Declaration of Mining Project Feasibility has been approved by government
(Section 24, RA 7942).
124

· The Declaration of Mining Project Feasibility filed by the contractor cannot be approved
without submission of the following documents:

1. Approved mining project feasibility study (Section 53-d, DAO 96-40)

2. Approved three-year work program (Section 53-a-4, DAO 96-40)

3. Environmental compliance certificate (Section 70, RA 7942)

4. Approved environmental protection and enhancement program (Section 69, RA


7942)

5. Approval by the Sangguniang Panlalawigan/Bayan/Barangay (Section 70, RA


7942; Section 27, RA 7160)

6. Free and prior informed consent by the indigenous peoples concerned, including
payment of royalties through a Memorandum of Agreement (Section 16, RA 7942;
Section 59, RA 8371)

· The FTAA contractor is obliged to assist in the development of its mining community,
promotion of the general welfare of its inhabitants, and development of science and mining
technology (Section 57, RA 7942).

· The FTAA contractor is obliged to submit reports (on quarterly, semi-annual or annual basis
as the case may be; per Section 270, DAO 96-40), pertaining to the following:

1. Exploration

2. Drilling

3. Mineral resources and reserves

4. Energy consumption

5. Production

6. Sales and marketing

7. Employment

8. Payment of taxes, royalties, fees and other Government Shares

9. Mine safety, health and environment

10. Land use

11. Social development

12. Explosives consumption

· An FTAA pertaining to areas within government reservations cannot be granted without a


written clearance from the government agencies concerned (Section 19, RA 7942; Section
54, DAO 96-40).
125

· An FTAA contractor is required to post a financial guarantee bond in favor of the


government in an amount equivalent to its expenditures obligations for any particular year.
This requirement is apart from the representations and warranties of the contractor that it has
access to all the financing, managerial and technical expertise and technology necessary to
carry out the objectives of the FTAA (Section 35-b, -e, and -f, RA 7942).

· Other reports to be submitted by the contractor, as required under DAO 96-40, are as
follows: an environmental report on the rehabilitation of the mined-out area and/or mine
waste/tailing covered area, and anti-pollution measures undertaken (Section 35-a-2); annual
reports of the mining operations and records of geologic accounting (Section 56-m); annual
progress reports and final report of exploration activities (Section 56-2).

· Other programs required to be submitted by the contractor, pursuant to DAO 96-40, are the
following: a safety and health program (Section 144); an environmental work program
(Section 168); an annual environmental protection and enhancement program (Section 171).

The foregoing gamut of requirements, regulations, restrictions and limitations imposed upon the FTAA
contractor by the statute and regulations easily overturns petitioners' contention. The setup under RA
7942 and DAO 96-40 hardly relegates the State to the role of a "passive regulator" dependent on
submitted plans and reports. On the contrary, the government agencies concerned are empowered to
approve or disapprove -- hence, to influence, direct and change -- the various work programs and the
corresponding minimum expenditure commitments for each of the exploration, development and
utilization phases of the mining enterprise.

Once these plans and reports are approved, the contractor is bound to comply with its commitments
therein. Figures for mineral production and sales are regularly monitored and subjected to
government review, in order to ensure that the products and by-products are disposed of at the best
prices possible; even copies of sales agreements have to be submitted to and registered with MGB.
And the contractor is mandated to open its books of accounts and records for scrutiny, so as to
enable the State to determine if the government share has been fully paid.

The State may likewise compel the contractor's compliance with mandatory requirements on mine
safety, health and environmental protection, and the use of anti-pollution technology and facilities.
Moreover, the contractor is also obligated to assist in the development of the mining community and
to pay royalties to the indigenous peoples concerned.

Cancellation of the FTAA may be the penalty for violation of any of its terms and conditions and/or
noncompliance with statutes or regulations. This general, all-around, multipurpose sanction is no
trifling matter, especially to a contractor who may have yet to recover the tens or hundreds of millions
of dollars sunk into a mining project.

Overall, considering the provisions of the statute and the regulations just discussed, we believe that
the State definitely possesses the means by which it can have the ultimate word in the operation of
the enterprise, set directions and objectives, and detect deviations and noncompliance by the
contractor; likewise, it has the capability to enforce compliance and to impose sanctions, should the
occasion therefor arise.

In other words, the FTAA contractor is not free to do whatever it pleases and get away with it;
on the contrary, it will have to follow the government line if it wants to stay in the enterprise.
Ineluctably then, RA 7942 and DAO 96-40 vest in the government more than a sufficient degree
of control and supervision over the conduct of mining operations.

Section 3(aq) of RA 7942


Not Unconstitutional

An objection has been expressed that Section 3(aq)55 of RA 7942 -- which allows a foreign contractor
to apply for and hold an exploration permit -- is unconstitutional. The reasoning is that Section 2 of
Article XII of the Constitution does not allow foreign-owned corporations to undertake mining
126

operations directly. They may act only as contractors of the State under an FTAA; and the State, as
the party directly undertaking exploitation of its natural resources, must hold through the government
all exploration permits and similar authorizations. Hence, Section 3(aq), in permitting foreign-owned
corporations to hold exploration permits, is unconstitutional.

The objection, however, is not well-founded. While the Constitution mandates the State to exercise
full control and supervision over the exploitation of mineral resources, nowhere does it require the
government to hold all exploration permits and similar authorizations. In fact, there is no prohibition at
all against foreign or local corporations or contractors holding exploration permits. The reason is not
hard to see.

Pursuant to Section 20 of RA 7942, an exploration permit merely grants to a qualified person the right
to conduct exploration for all minerals in specified areas. Such a permit does not amount to an
authorization to extract and carry off the mineral resources that may be discovered. This phase
involves nothing but expenditures for exploring the contract area and locating the mineral bodies. As
no extraction is involved, there are no revenues or incomes to speak of. In short, the exploration
permit is an authorization for the grantee to spend its own funds on exploration programs that are pre-
approved by the government, without any right to recover anything should no minerals in commercial
quantities be discovered. The State risks nothing and loses nothing by granting these permits to local
or foreign firms; in fact, it stands to gain in the form of data generated by the exploration activities.

Pursuant to Section 24 of RA 7942, an exploration permit grantee who determines the commercial
viability of a mining area may, within the term of the permit, file with the MGB a declaration of mining
project feasibility accompanied by a work program for development. The approval of the mining
project feasibility and compliance with other requirements of RA 7942 vests in the grantee the
exclusive right to an MPSA or any other mineral agreement, or to an FTAA.

Thus, the permit grantee may apply for an MPSA, a joint venture agreement, a co-production
agreement, or an FTAA over the permit area, and the application shall be approved if the permit
grantee meets the necessary qualifications and the terms and conditions of any such agreement.
Therefore, the contractor will be in a position to extract minerals and earn revenues only when the
MPSA or another mineral agreement, or an FTAA, is granted. At that point, the contractor's rights and
obligations will be covered by an FTAA or a mineral agreement.

But prior to the issuance of such FTAA or mineral agreement, the exploration permit grantee (or
prospective contractor) cannot yet be deemed to have entered into any contract or agreement with
the State, and the grantee would definitely need to have some document or instrument as evidence of
its right to conduct exploration works within the specified area. This need is met by the exploration
permit issued pursuant to Sections 3(aq), 20 and 23 of RA 7942.

In brief, the exploration permit serves a practical and legitimate purpose in that it protects the
interests and preserves the rights of the exploration permit grantee (the would-be contractor) -
- foreign or local -- during the period of time that it is spending heavily on exploration works,
without yet being able to earn revenues to recoup any of its investments and
expenditures. Minus this permit and the protection it affords, the exploration works and expenditures
may end up benefiting only claim-jumpers. Such a possibility tends to discourage investors and
contractors. Thus, Section 3(aq) of RA 7942 may not be deemed unconstitutional.

The Terms of the WMCP FTAA

A Deference to State Control

A perusal of the WMCP FTAA also reveals a slew of stipulations providing for State control and
supervision:

1. The contractor is obligated to account for the value of production and sale of minerals
(Clause 1.4).
127

2. The contractor's work program, activities and budgets must be approved by/on behalf of
the State (Clause 2.1).

3. The DENR secretary has the power to extend the exploration period (Clause 3.2-a).

4. Approval by the State is necessary for incorporating lands into the FTAA contract area
(Clause 4.3-c).

5. The Bureau of Forest Development is vested with discretion in regard to approving the
inclusion of forest reserves as part of the FTAA contract area (Clause 4.5).

6. The contractor is obliged to relinquish periodically parts of the contract area not needed for
exploration and development (Clause 4.6).

7. A Declaration of Mining Feasibility must be submitted for approval by the State (Clause
4.6-b).

8. The contractor is obligated to report to the State its exploration activities (Clause 4.9).

9. The contractor is required to obtain State approval of its work programs for the succeeding
two-year periods, containing the proposed work activities and expenditures budget related to
exploration (Clause 5.1).

10. The contractor is required to obtain State approval for its proposed expenditures for
exploration activities (Clause 5.2).

11. The contractor is required to submit an annual report on geological, geophysical,


geochemical and other information relating to its explorations within the FTAA area (Clause
5.3-a).

12. The contractor is to submit within six months after expiration of exploration period a final
report on all its findings in the contract area (Clause 5.3-b).

13. The contractor, after conducting feasibility studies, shall submit a declaration of mining
feasibility, along with a description of the area to be developed and mined, a description of
the proposed mining operations and the technology to be employed, and a proposed work
program for the development phase, for approval by the DENR secretary (Clause 5.4).

14. The contractor is obliged to complete the development of the mine, including construction
of the production facilities, within the period stated in the approved work program (Clause
6.1).

15. The contractor is obligated to submit for approval of the DENR secretary a work program
covering each period of three fiscal years (Clause 6.2).

16. The contractor is to submit reports to the DENR secretary on the production, ore
reserves, work accomplished and work in progress, profile of its work force and management
staff, and other technical information (Clause 6.3).

17. Any expansions, modifications, improvements and replacements of mining facilities shall
be subject to the approval of the secretary (Clause 6.4).

18. The State has control with respect to the amount of funds that the contractor may borrow
within the Philippines (Clause 7.2).

19. The State has supervisory power with respect to technical, financial and marketing issues
(Clause 10.1-a).
128

20. The contractor is required to ensure 60 percent Filipino equity in the contractor, within ten
years of recovering specified expenditures, unless not so required by subsequent legislation
(Clause 10.1).

21. The State has the right to terminate the FTAA for the contractor's unremedied substantial
breach thereof (Clause 13.2);

22. The State's approval is needed for any assignment of the FTAA by the contractor to an
entity other than an affiliate (Clause 14.1).

We should elaborate a little on the work programs and budgets, and what they mean with respect to
the State's ability to exercise full control and effective supervision over the enterprise. For instance,
throughout the initial five-year exploration and feasibility phase of the project, the contractor is
mandated by Clause 5.1 of the WMCP FTAA to submit a series of work programs (copy furnished the
director of MGB) to the DENR secretary for approval. The programs will detail the contractor's
proposed exploration activities and budget covering each subsequent period of two fiscal years.

In other words, the concerned government officials will be informed beforehand of the proposed
exploration activities and expenditures of the contractor for each succeeding two-year period, with the
right to approve/disapprove them or require changes or adjustments therein if deemed necessary.

Likewise, under Clause 5.2(a), the amount that the contractor was supposed to spend for exploration
activities during the first contract year of the exploration period was fixed at not less than P24 million;
and then for the succeeding years, the amount shall be as agreed between the DENR secretary and
the contractor prior to the commencement of each subsequent fiscal year. If no such agreement is
arrived upon, the previous year's expenditure commitment shall apply.

This provision alone grants the government through the DENR secretary a very big say in the
exploration phase of the project. This fact is not something to be taken lightly, considering that
the government has absolutely no contribution to the exploration expenditures or work activities and
yet is given veto power over such a critical aspect of the project. We cannot but construe as very
significant such a degree of control over the project and, resultantly, over the mining enterprise itself.

Following its exploration activities or feasibility studies, if the contractor believes that any part of the
contract area is likely to contain an economic mineral resource, it shall submit to the DENR secretary
a declaration of mining feasibility (per Clause 5.4 of the FTAA), together with a technical description of
the area delineated for development and production, a description of the proposed mining operations
including the technology to be used, a work program for development, an environmental impact
statement, and a description of the contributions to the economic and general welfare of the country
to be generated by the mining operations (pursuant to Clause 5.5).

The work program for development is subject to the approval of the DENR secretary. Upon its
approval, the contractor must comply with it and complete the development of the mine, including the
construction of production facilities and installation of machinery and equipment, within the period
provided in the approved work program for development (per Clause 6.1).

Thus, notably, the development phase of the project is likewise subject to the control and supervision
of the government. It cannot be emphasized enough that the proper and timely construction and
deployment of the production facilities and the development of the mine are of pivotal significance to
the success of the mining venture. Any missteps here will potentially be very costly to remedy. Hence,
the submission of the work program for development to the DENR secretary for approval is
particularly noteworthy, considering that so many millions of dollars worth of investments -- courtesy
of the contractor -- are made to depend on the State's consideration and action.

Throughout the operating period, the contractor is required to submit to the DENR secretary for
approval, copy furnished the director of MGB, work programs covering each period of three fiscal
years (per Clause 6.2). During the same period (per Clause 6.3), the contractor is mandated to submit
various quarterly and annual reports to the DENR secretary, copy furnished the director of MGB, on
129

the tonnages of production in terms of ores and concentrates, with corresponding grades, values and
destinations; reports of sales; total ore reserves, total tonnage of ores, work accomplished and work
in progress (installations and facilities related to mining operations), investments made or committed,
and so on and so forth.

Under Section VIII, during the period of mining operations, the contractor is also required to submit to
the DENR secretary (copy furnished the director of MGB) the work program and corresponding
budget for the contract area, describing the mining operations that are proposed to be carried out
during the period covered. The secretary is, of course, entitled to grant or deny approval of any work
program or budget and/or propose revisions thereto. Once the program/budget has been approved,
the contractor shall comply therewith.

In sum, the above provisions of the WMCP FTAA taken together, far from constituting a surrender of
control and a grant of beneficial ownership of mineral resources to the contractor in question, bestow
upon the State more than adequate control and supervision over the activities of the
contractor and the enterprise.

No Surrender of Control
Under the WMCP FTAA

Petitioners, however, take aim at Clause 8.2, 8.3, and 8.5 of the WMCP FTAA which, they say,
amount to a relinquishment of control by the State, since it "cannot truly impose its own discretion" in
respect of the submitted work programs.

"8.2. The Secretary shall be deemed to have approved any Work Programme or Budget or
variation thereofsubmitted by the Contractor unless within sixty (60) days after submission by
the Contractor the Secretary gives notice declining such approval or proposing a revision of
certain features and specifying its reasons therefor ('the Rejection Notice').

8.3. If the Secretary gives a Rejection Notice, the Parties shall promptly meet and endeavor
to agree on amendments to the Work Programme or Budget. If the Secretary and the
Contractor fail to agree on the proposed revision within 30 days from delivery of the Rejection
Notice then the Work Programme or Budget or variation thereof proposed by the Contractor
shall be deemed approved, so as not to unnecessarily delay the performance of the
Agreement.

8.4. x x x x x x x x x

8.5. So far as is practicable, the Contractor shall comply with any approved Work Programme
and Budget. It is recognized by the Secretary and the Contractor that the details of any Work
Programmes or Budgets may require changes in the light of changing circumstances. The
Contractor may make such changes without approval of the Secretary provided they do not
change the general objective of any Work Programme, nor entail a downward variance of
more than twenty per centum (20percent) of the relevant Budget. All other variations to an
approved Work Programme or Budget shall be submitted for approval of the Secretary."

From the provisions quoted above, petitioners generalize by asserting that the government does not
participate in making critical decisions regarding the operations of the mining firm. Furthermore, while
the State can require the submission of work programs and budgets, the decision of the contractor will
still prevail, if the parties have a difference of opinion with regard to matters affecting operations and
management.

We hold, however, that the foregoing provisions do not manifest a relinquishment of control. For
instance, Clause 8.2 merely provides a mechanism for preventing the business or mining operations
from grinding to a complete halt as a result of possibly over-long and unjustified delays in the
government's handling, processing and approval of submitted work programs and budgets. Anyway,
the provision does give the DENR secretary more than sufficient time (60 days) to react to submitted
130

work programs and budgets. It cannot be supposed that proper grounds for objecting thereto, if any
exist, cannot be discovered within a period of two months.

On the other hand, Clause 8.3 seeks to provide a temporary, stop-gap solution in the event a
disagreement over the submitted work program or budget arises between the State and the contractor
and results in a stalemate or impasse, in order that there will be no unreasonably long delays in the
performance of the works.

These temporary or stop-gap solutions are not necessarily evil or wrong. Neither does it follow that
the government will inexorably be aggrieved if and when these temporary remedies come into
play. First, avoidance of long delays in these situations will undoubtedly redound to the benefit of the
State as well as the contractor. Second, who is to say that the work program or budget proposed by
the contractor and deemed approved under Clause 8.3 would not be the better or more reasonable or
more effective alternative? The contractor, being the "insider," as it were, may be said to be in a better
position than the State -- an outsider looking in -- to determine what work program or budget would be
appropriate, more effective, or more suitable under the circumstances.

All things considered, we take exception to the characterization of the DENR secretary as a
subservient nonentity whom the contractor can overrule at will, on account of Clause 8.3. And neither
is it true that under the same clause, the DENR secretary has no authority whatsoever to disapprove
the work program. As Respondent WMCP reasoned in its Reply-Memorandum, the State -- despite
Clause 8.3 -- still has control over the contract area and it may, as sovereign authority, prohibit work
thereon until the dispute is resolved. And ultimately, the State may terminate the agreement, pursuant
to Clause 13.2 of the same FTAA, citing substantial breach thereof. Hence, it clearly retains full and
effective control of the exploitation of the mineral resources.

On the other hand, Clause 8.5 is merely an acknowledgment of the parties' need for flexibility, given
that no one can accurately forecast under all circumstances, or predict how situations may change.
Hence, while approved work programs and budgets are to be followed and complied with as far as
practicable, there may be instances in which changes will have to be effected, and effected rapidly,
since events may take shape and unfold with suddenness and urgency. Thus, Clause 8.5 allows the
contractor to move ahead and make changes without the express or implicit approval of the DENR
secretary. Such changes are, however, subject to certain conditions that will serve to limit or restrict
the variance and prevent the contractor from straying very far from what has been approved.

Clause 8.5 provides the contractor a certain amount of flexibility to meet unexpected situations, while
still guaranteeing that the approved work programs and budgets are not abandoned altogether.
Clause 8.5 does not constitute proof that the State has relinquished control. And ultimately, should
there be disagreement with the actions taken by the contractor in this instance as well as under
Clause 8.3 discussed above, the DENR secretary may resort to cancellation/termination of the FTAA
as the ultimate sanction.

Discretion to Select Contract


Area Not an Abdication of Control

Next, petitioners complain that the contractor has full discretion to select -- and the government has
no say whatsoever as to -- the parts of the contract area to be relinquished pursuant to Clause 4.6 of
the WMCP FTAA.56This clause, however, does not constitute abdication of control. Rather, it is a
mere acknowledgment of the fact that the contractor will have determined, after appropriate
exploration works, which portions of the contract area do not contain minerals in commercial
quantities sufficient to justify developing the same and ought therefore to be relinquished. The State
cannot just substitute its judgment for that of the contractor and dictate upon the latter which areas to
give up.

Moreover, we can be certain that the contractor's self-interest will propel proper and efficient
relinquishment. According to private respondent,57 a mining company tries to relinquish as much non-
mineral areas as soon as possible, because the annual occupation fees paid to the government are
based on the total hectarage of the contract area, net of the areas relinquished. Thus, the larger the
131

remaining area, the heftier the amount of occupation fees to be paid by the contractor. Accordingly,
relinquishment is not an issue, given that the contractor will not want to pay the annual occupation
fees on the non-mineral parts of its contract area. Neither will it want to relinquish promising sites,
which other contractors may subsequently pick up.

Government Not a Subcontractor

Petitioners further maintain that the contractor can compel the government to exercise its power of
eminent domain to acquire surface areas within the contract area for the contractor's use. Clause 10.2
(e) of the WMCP FTAA provides that the government agrees that the contractor shall "(e) have the
right to require the Government at the Contractor's own cost, to purchase or acquire surface areas for
and on behalf of the Contractor at such price and terms as may be acceptable to the contractor. At
the termination of this Agreement such areas shall be sold by public auction or tender and the
Contractor shall be entitled to reimbursement of the costs of acquisition and maintenance, adjusted
for inflation, from the proceeds of sale."

According to petitioners, "government becomes a subcontractor to the contractor" and may, on


account of this provision, be compelled "to make use of its power of eminent domain, not for public
purposes but on behalf of a private party, i.e., the contractor." Moreover, the power of the courts to
determine the amount corresponding to the constitutional requirement of just compensation has
allegedly also been contracted away by the government, on account of the latter's commitment that
the acquisition shall be at such terms as may be acceptable to the contractor.

However, private respondent has proffered a logical explanation for the provision.58 Section 10.2(e)
contemplates a situation applicable to foreign-owned corporations. WMCP, at the time of the
execution of the FTAA, was a foreign-owned corporation and therefore not qualified to own land. As
contractor, it has at some future date to construct the infrastructure -- the mine processing plant, the
camp site, the tailings dam, and other infrastructure -- needed for the large-scale mining operations. It
will then have to identify and pinpoint, within the FTAA contract area, the particular surface areas with
favorable topography deemed ideal for such infrastructure and will need to acquire the surface rights.
The State owns the mineral deposits in the earth, and is also qualified to own land.

Section 10.2(e) sets forth the mechanism whereby the foreign-owned contractor, disqualified to own
land, identifies to the government the specific surface areas within the FTAA contract area to be
acquired for the mine infrastructure. The government then acquires ownership of the surface land
areas on behalf of the contractor, in order to enable the latter to proceed to fully implement the FTAA.

The contractor, of course, shoulders the purchase price of the land. Hence, the provision allows it,
after termination of the FTAA, to be reimbursed from proceeds of the sale of the surface areas, which
the government will dispose of through public bidding. It should be noted that this provision will not be
applicable to Sagittarius as the present FTAA contractor, since it is a Filipino corporation qualified to
own and hold land. As such, it may therefore freely negotiate with the surface rights owners and
acquire the surface property in its own right.

Clearly, petitioners have needlessly jumped to unwarranted conclusions, without being aware of the
rationale for the said provision. That provision does not call for the exercise of the power of eminent
domain -- and determination of just compensation is not an issue -- as much as it calls for a qualified
party to acquire the surface rights on behalf of a foreign-owned contractor.

Rather than having the foreign contractor act through a dummy corporation, having the State do the
purchasing is a better alternative. This will at least cause the government to be aware of such
transaction/s and foster transparency in the contractor's dealings with the local property owners. The
government, then, will not act as a subcontractor of the contractor; rather, it will facilitate the
transaction and enable the parties to avoid a technical violation of the Anti-Dummy Law.

Absence of Provision
Requiring Sale at Posted
Prices Not Problematic
132

The supposed absence of any provision in the WMCP FTAA directly and explicitly requiring the
contractor to sell the mineral products at posted or market prices is not a problem. Apart from Clause
1.4 of the FTAA obligating the contractor to account for the total value of mineral production and the
sale of minerals, we can also look to Section 35 of RA 7942, which incorporates into all FTAAs certain
terms, conditions and warranties, including the following:

"(l) The contractors shall furnish the Government records of geologic, accounting and other
relevant data for its mining operation, and that books of accounts and records shall be open
for inspection by the government.x x x

(m) Requiring the proponent to dispose of the minerals at the highest price and more
advantageous terms and conditions."

For that matter, Section 56(n) of DAO 99-56 specifically obligates an FTAA contractor to dispose of
the minerals and by-products at the highest market price and to register with the MGB a copy of the
sales agreement. After all, the provisions of prevailing statutes as well as rules and regulations are
deemed written into contracts.

Contractor's Right to Mortgage


Not Objectionable Per Se

Petitioners also question the absolute right of the contractor under Clause 10.2 (l) to mortgage and
encumber not only its rights and interests in the FTAA and the infrastructure and improvements
introduced, but also the mineral products extracted. Private respondents do not touch on this matter,
but we believe that this provision may have to do with the conditions imposed by the creditor-banks of
the then foreign contractor WMCP to secure the lendings made or to be made to the latter. Ordinarily,
banks lend not only on the security of mortgages on fixed assets, but also on encumbrances of goods
produced that can easily be sold and converted into cash that can be applied to the repayment of
loans. Banks even lend on the security of accounts receivable that are collectible within 90 days.59

It is not uncommon to find that a debtor corporation has executed deeds of assignment "by way of
security" over the production for the next twelve months and/or the proceeds of the sale thereof -- or
the corresponding accounts receivable, if sold on terms -- in favor of its creditor-banks. Such deeds
may include authorizing the creditors to sell the products themselves and to collect the sales
proceeds and/or the accounts receivable.

Seen in this context, Clause 10.2(l) is not something out of the ordinary or objectionable. In any case,
as will be explained below, even if it is allowed to mortgage or encumber the mineral end-products
themselves, the contractor is not freed of its obligation to pay the government its basic and additional
shares in the net mining revenue, which is the essential thing to consider.

In brief, the alarum raised over the contractor's right to mortgage the minerals is simply unwarranted.
Just the same, the contractor must account for the value of mineral production and the sales
proceeds therefrom. Likewise, under the WMCP FTAA, the government remains entitled to its sixty
percent share in the net mining revenues of the contractor. The latter's right to mortgage the minerals
does not negate the State's right to receive its share of net mining revenues.

Shareholders Free to Sell Their Stocks

Petitioners likewise criticize Clause 10.2(k), which gives the contractor authority "to change its equity
structure at any time." This provision may seem somewhat unusual, but considering that WMCP then
was 100 percent foreign-owned, any change would mean that such percentage would either stay
unaltered or be decreased in favor of Filipino ownership. Moreover, the foreign-held shares may
change hands freely. Such eventuality is as it should be.

We believe it is not necessary for government to attempt to limit or restrict the freedom of the
shareholders in the contractor to freely transfer, dispose of or encumber their shareholdings,
consonant with the unfettered exercise of their business judgment and discretion. Rather, what is
133

critical is that, regardless of the identity, nationality and percentage ownership of the various
shareholders of the contractor -- and regardless of whether these shareholders decide to take the
company public, float bonds and other fixed-income instruments, or allow the creditor-banks to take
an equity position in the company -- the foreign-owned contractor is always in a position to render the
services required under the FTAA, under the direction and control of the government.

Contractor's Right to Ask


For Amendment Not Absolute

With respect to Clauses 10.4(e) and (i), petitioners complain that these provisions bind government to
allow amendments to the FTAA if required by banks and other financial institutions as part of the
conditions for new lendings. However, we do not find anything wrong with Clause 10.4(e), which only
states that "if the Contractor seeks to obtain financing contemplated herein from banks or other
financial institutions, (the Government shall) cooperate with the Contractor in such efforts provided
that such financing arrangements will in no event reduce the Contractor's obligations or the
Government's rights hereunder." The colatilla obviously safeguards the State's interests; if breached,
it will give the government cause to object to the proposed amendments.

On the other hand, Clause 10.4(i) provides that "the Government shall favourably consider any
request from [the] Contractor for amendments of this Agreement which are necessary in order for the
Contractor to successfully obtain the financing." Petitioners see in this provision a complete
renunciation of control. We disagree.

The proviso does not say that the government shall grant any request for amendment. Clause 10.4(i)
only obliges the State to favorably consider any such request, which is not at all unreasonable, as it is
not equivalent to saying that the government must automatically consent to it. This provision should
be read together with the rest of the FTAA provisions instituting government control and supervision
over the mining enterprise. The clause should not be given an interpretation that enables the
contractor to wiggle out of the restrictions imposed upon it by merely suggesting that certain
amendments are requested by the lenders.

Rather, it is up to the contractor to prove to the government that the requested changes to the FTAA
are indispensable, as they enable the contractor to obtain the needed financing; that without such
contract changes, the funders would absolutely refuse to extend the loan; that there are no other
sources of financing available to the contractor (a very unlikely scenario); and that without the needed
financing, the execution of the work programs will not proceed. But the bottom line is, in the exercise
of its power of control, the government has the final say on whether to approve or disapprove such
requested amendments to the FTAA. In short, approval thereof is not mandatory on the part of the
government.

In fine, the foregoing evaluation and analysis of the aforementioned FTAA provisions
sufficiently overturns petitioners' litany of objections to and criticisms of the State's alleged
lack of control.

Financial Benefits Not


Surrendered to the Contractor

One of the main reasons certain provisions of RA 7942 were struck down was the finding mentioned
in the Decision that beneficial ownership of the mineral resources had been conveyed to the
contractor. This finding was based on the underlying assumption, common to the said provisions, that
the foreign contractor manages the mineral resources in the same way that foreign contractors in
service contracts used to. "By allowing foreign contractors to manage or operate all the aspects of the
mining operation, the above-cited provisions of R.A. No. 7942 have in effect conveyed beneficial
ownership over the nation's mineral resources to these contractors, leaving the State with nothing
but bare title thereto."60 As the WMCP FTAA contained similar provisions deemed by the ponente to
be abhorrent to the Constitution, the Decision struck down the Contract as well.
134

Beneficial ownership has been defined as ownership recognized by law and capable of being
enforced in the courts at the suit of the beneficial owner. 61 Black's Law Dictionary indicates that the
term is used in two senses: first, to indicate the interest of a beneficiary in trust property (also called
"equitable ownership"); and second, to refer to the power of a corporate shareholder to buy or sell the
shares, though the shareholder is not registered in the corporation's books as the owner. 62 Usually,
beneficial ownership is distinguished from naked ownership, which is the enjoyment of all the benefits
and privileges of ownership, as against possession of the bare title to property.

An assiduous examination of the WMCP FTAA uncovers no indication that it confers upon WMCP
ownership, beneficial or otherwise, of the mining property it is to develop, the minerals to be
produced, or the proceeds of their sale, which can be legally asserted and enforced as against the
State.

As public respondents correctly point out, any interest the contractor may have in the proceeds of the
mining operation is merely the equivalent of the consideration the government has undertaken to pay
for its services. All lawful contracts require such mutual prestations, and the WMCP FTAA is no
different. The contractor commits to perform certain services for the government in respect of the
mining operation, and in turn it is to be compensated out of the net mining revenues generated from
the sale of mineral products. What would be objectionable is a contractual provision that unduly
benefits the contractor far in excess of the service rendered or value delivered, if any, in exchange
therefor.

A careful perusal of the statute itself and its implementing rules reveals that neither RA 7942 nor DAO
99-56 can be said to convey beneficial ownership of any mineral resource or product to any foreign
FTAA contractor.

Equitable Sharing
of Financial Benefits

On the contrary, DAO 99-56, entitled "Guidelines Establishing the Fiscal Regime of Financial or
Technical Assistance Agreements" aims to ensure an equitable sharing of the benefits derived from
mineral resources. These benefits are to be equitably shared among the government (national and
local), the FTAA contractor, and the affected communities. The purpose is to ensure sustainable
mineral resources development; and a fair, equitable, competitive and stable investment regime for
the large-scale exploration, development and commercial utilization of minerals. The general
framework or concept followed in crafting the fiscal regime of the FTAA is based on the principle that
the government expects real contributions to the economic growth and general welfare of the country,
while the contractor expects a reasonable return on its investments in the project.63

Specifically, under the fiscal regime, the government's expectation is, inter alia, the receipt of its share
from the taxes and fees normally paid by a mining enterprise. On the other hand, the FTAA contractor
is granted by the government certain fiscal and non-fiscal incentives64 to help support the former's
cash flow during the most critical phase (cost recovery) and to make the Philippines competitive with
other mineral-producing countries. After the contractor has recovered its initial investment, it will pay
all the normal taxes and fees comprising the basic share of the government, plus an additional share
for the government based on the options and formulae set forth in DAO 99-56.

The said DAO spells out the financial benefits the government will receive from an FTAA, referred to
as "the Government Share," composed of a basic government share and an additional
government share.

The basic government share is comprised of all direct taxes, fees and royalties, as well as other
payments made by the contractor during the term of the FTAA. These are amounts paid directly to (i)
the national government (through the Bureau of Internal Revenue, Bureau of Customs, Mines &
Geosciences Bureau and other national government agencies imposing taxes or fees), (ii) the local
government units where the mining activity is conducted, and (iii) persons and communities directly
affected by the mining project. The major taxes and other payments constituting the basic government
share are enumerated below:65
135

Payments to the National Government:

· Excise tax on minerals - 2 percent of the gross output of mining operations

· Contractor' income tax - maximum of 32 percent of taxable income for corporations

· Customs duties and fees on imported capital equipment -the rate is set by the Tariff
and Customs Code (3-7 percent for chemicals; 3-10 percent for explosives; 3-15
percent for mechanical and electrical equipment; and 3-10 percent for vehicles,
aircraft and vessels

· VAT on imported equipment, goods and services – 10 percent of value

· Royalties due the government on minerals extracted from mineral reservations, if


applicable – 5 percent of the actual market value of the minerals produced

· Documentary stamp tax - the rate depends on the type of transaction

· Capital gains tax on traded stocks - 5 to 10 percent of the value of the shares

· Withholding tax on interest payments on foreign loans -15 percent of the amount of
interest

· Withholding tax on dividend payments to foreign stockholders – 15 percent of the


dividend

· Wharfage and port fees

· Licensing fees (for example, radio permit, firearms permit, professional fees)

· Other national taxes and fees.

Payments to Local Governments:

· Local business tax - a maximum of 2 percent of gross sales or receipts (the rate
varies among local government units)

· Real property tax - 2 percent of the fair market value of the property, based on an
assessment level set by the local government

· Special education levy - 1 percent of the basis used for the real property tax

· Occupation fees - PhP50 per hectare per year; PhP100 per hectare per year if
located in a mineral reservation

· Community tax - maximum of PhP10,500 per year

· All other local government taxes, fees and imposts as of the effective date of the
FTAA - the rate and the type depend on the local government

Other Payments:

· Royalty to indigenous cultural communities, if any – 1 percent of gross output from


mining operations
136

· Special allowance - payment to claim owners and surface rights holders

Apart from the basic share, an additional government share is also collected from the FTAA
contractor in accordance with the second paragraph of Section 81 of RA 7942, which provides that
the government share shall be comprised of, among other things, certain taxes, duties and fees. The
subject proviso reads:

"The Government share in a financial or technical assistance agreement shall consist of, among
other things, the contractor's corporate income tax, excise tax, special allowance, withholding tax
due from the contractor's foreign stockholders arising from dividend or interest payments to the said
foreign stockholder in case of a foreign national, and all such other taxes, duties and fees as provided
for under existing laws." (Bold types supplied.)

The government, through the DENR and the MGB, has interpreted the insertion of the phrase among
other things as signifying that the government is entitled to an "additional government share" to be
paid by the contractor apart from the "basic share," in order to attain a fifty-fifty sharing of net benefits
from mining.

The additional government share is computed by using one of three options or schemes presented
in DAO 99-56: (1) a fifty-fifty sharing in the cumulative present value of cash flows; (2) the share
based on excess profits; and (3) the sharing based on the cumulative net mining revenue. The
particular formula to be applied will be selected by the contractor, with a written notice to the
government prior to the commencement of the development and construction phase of the mining
project.66

Proceeds from the government shares arising from an FTAA contract are distributed to and received
by the different levels of government in the following proportions:

National Government 50 percent

Provincial 10 percent
Government

Municipal 20 percent
Government

Affected Barangays 20 percent

The portion of revenues remaining after the deduction of the basic and additional government shares
is what goes to the contractor.

Government's Share in an
FTAA Not Consisting Solely
of Taxes, Duties and Fees

In connection with the foregoing discussion on the basic and additional government shares, it is
pertinent at this juncture to mention the criticism leveled at the second paragraph of Section 81 of RA
7942, quoted earlier. The said proviso has been denounced, because, allegedly, the State's share in
FTAAs with foreign contractors has been limited to taxes, fees and duties only; in effect, the State has
been deprived of a share in the after-tax income of the enterprise. In the face of this allegation, one
has to consider that the law does not define the term among other things; and the Office of the
Solicitor General, in its Motion for Reconsideration, appears to have erroneously claimed that the
phrase refers to indirect taxes.

The law provides no definition of the term among other things, for the reason that Congress
deliberately avoided setting unnecessary limitations as to what may constitute compensation to the
137

State for the exploitation and use of mineral resources. But the inclusion of that phrase clearly and
unmistakably reveals the legislative intent to have the State collect more than just the usual taxes,
duties and fees. Certainly, there is nothing in that phrase -- or in the second paragraph of Section 81 -
- that would suggest that such phrase should be interpreted as referring only to taxes, duties, fees
and the like.

Precisely for that reason, to fulfill the legislative intent behind the inclusion of the phrase among other
things in the second paragraph of Section 81,67 the DENR structured and formulated in DAO 99-56
the said additional government share. Such a share was to consist not of taxes, but of a share in
the earnings or cash flows of the mining enterprise. The additional government share was to be
paid by the contractor on top of the basic share, so as to achieve a fifty-fifty sharing -- between the
government and the contractor -- of net benefits from mining. In the Ramos-DeVera paper, the
explanation of the three options or formulas68 -- presented in DAO 99-56 for the computation of the
additional government share -- serves to debunk the claim that the government's take from an FTAA
consists solely of taxes, fees and duties.

Unfortunately, the Office of the Solicitor General -- although in possession of the relevant data -- failed
to fully replicate or echo the pertinent elucidation in the Ramos-DeVera paper regarding the three
schemes or options for computing the additional government share presented in DAO 99-56. Had due
care been taken by the OSG, the Court would have been duly apprised of the real nature and
particulars of the additional share.

But, perhaps, on account of the esoteric discussion in the Ramos-DeVera paper, and the even more
abstruse mathematical jargon employed in DAO 99-56, the OSG omitted any mention of the three
options. Instead, the OSG skipped to a side discussion of the effect of indirect taxes, which
had nothing at all to do with the additional government share, to begin with. Unfortunately, this move
created the wrong impression, pointed out in Justice Antonio T. Carpio's Opinion, that the OSG had
taken the position that the additional government share consisted of indirect taxes.

In any event, what is quite evident is the fact that the additional government share, as formulated,
has nothing to do with taxes -- direct or indirect -- or with duties, fees or charges. To repeat, it is over
and above the basic government share composed of taxes and duties. Simply put, the additional
share may be (a) an amount that will result in a 50-50 sharing of the cumulative present value of
the cash flows69 of the enterprise; (b) an amount equivalent to 25 percent of the additional or excess
profits of the enterprise, reckoned against a benchmark return on investments; or (c) an amount that
will result in a fifty-fifty sharing of the cumulative net mining revenue from the end of the recovery
period up to the taxable year in question. The contractor is required to select one of the three options
or formulae for computing the additional share, an option it will apply to all of its mining operations.

As used above, "net mining revenue" is defined as the gross output from mining operations for a
calendar year, less deductible expenses (inclusive of taxes, duties and fees). Such revenue would
roughly be equivalent to "taxable income" or income before income tax. Definitely, as compared with,
say, calculating the additional government share on the basis of net income (after income tax), the
net mining revenue is a better and much more reasonable basis for such computation, as it gives a
truer picture of the profitability of the company.

To demonstrate that the three options or formulations will operate as intended, Messrs. Ramos and
de Vera also performed some quantifications of the government share via a financial modeling of
each of the three options discussed above. They found that the government would get the highest
share from the option that is based on the net mining revenue, as compared with the other two
options, considering only the basic and the additional shares; and that, even though production rate
decreases, the government share will actually increase when the net mining revenue and the
additional profit-based options are used.

Furthermore, it should be noted that the three options or formulae do not yet take into account the
indirect taxes70and other financial contributions71 of mining projects. These indirect taxes and other
contributions are real and actual benefits enjoyed by the Filipino people and/or government. Now, if
some of the quantifiable items are taken into account in the computations, the financial modeling
would show that the total government share increases to 60 percent or higher -- in one instance, as
138

much as 77 percent and even 89 percent -- of the net present value of total benefits from the project.
As noted in the Ramos-DeVera paper, these results are not at all shabby, considering that the
contractor puts in all the capital requirements and assumes all the risks, without the government
having to contribute or risk anything.

Despite the foregoing explanation, Justice Carpio still insisted during the Court's deliberations that the
phrase among other things refers only to taxes, duties and fees. We are bewildered by his position.
On the one hand, he condemns the Mining Law for allegedly limiting the government's benefits only to
taxes, duties and fees; and on the other, he refuses to allow the State to benefit from the correct and
proper interpretation of the DENR/MGB. To remove all doubts then, we hold that the State's share is
not limited to taxes, duties and fees only and that the DENR/MGB interpretation of the phrase among
other things is correct. Definitely, this DENR/MGB interpretation is not only legally sound, but also
greatly advantageous to the government.

One last point on the subject. The legislature acted judiciously in not defining the terms among other
things and, instead, leaving it to the agencies concerned to devise and develop the various modes of
arriving at a reasonable and fair amount for the additional government share. As can be seen from
DAO 99-56, the agencies concerned did an admirable job of conceiving and developing not just one
formula, but three different formulae for arriving at the additional government share. Each of these
options is quite fair and reasonable; and, as Messrs. Ramos and De Vera stated, other alternatives or
schemes for a possible improvement of the fiscal regime for FTAAs are also being studied by the
government.

Besides, not locking into a fixed definition of the term among other things will ultimately be more
beneficial to the government, as it will have that innate flexibility to adjust to and cope with rapidly
changing circumstances, particularly those in the international markets. Such flexibility is especially
significant for the government in terms of helping our mining enterprises remain competitive in world
markets despite challenging and shifting economic scenarios.

In conclusion, we stress that we do not share the view that in FTAAs with foreign contractors
under RA 7942, the government's share is limited to taxes, fees and duties. Consequently, we
find the attacks on the second paragraph of Section 81 of RA 7942 totally unwarranted.

Collections Not Made Uncertain


by the Third Paragraph of Section 81

The third or last paragraph of Section 8172 provides that the government share in FTAAs shall be
collected when the contractor shall have recovered its pre-operating expenses and exploration and
development expenditures. The objection has been advanced that, on account of the proviso, the
collection of the State's share is not even certain, as there is no time limit in RA 7942 for this grace
period or recovery period.

We believe that Congress did not set any time limit for the grace period, preferring to leave it to the
concerned agencies, which are, on account of their technical expertise and training, in a better
position to determine the appropriate durations for such recovery periods. After all, these recovery
periods are determined, to a great extent, by technical and technological factors peculiar to the mining
industry. Besides, with developments and advances in technology and in the geosciences, we cannot
discount the possibility of shorter recovery periods. At any rate, the concerned agencies have not
been remiss in this area. The 1995 and 1996 Implementing Rules and Regulations of RA 7942
specify that the period of recovery, reckoned from the date of commercial operation, shall be for a
period not exceeding five years, or until the date of actual recovery, whichever comes earlier.

Approval of Pre-Operating
Expenses Required by RA 7942

Still, RA 7942 is criticized for allegedly not requiring government approval of pre-operating,
exploration and development expenses of the foreign contractors, who are in effect given unfettered
discretion to determine the amounts of such expenses. Supposedly, nothing prevents the contractors
139

from recording such expenses in amounts equal to the mining revenues anticipated for the first 10 or
15 years of commercial production, with the result that the share of the State will be zero for the first
10 or 15 years. Moreover, under the circumstances, the government would be unable to say when it
would start to receive its share under the FTAA.

We believe that the argument is based on incorrect information as well as speculation. Obviously,
certain crucial provisions in the Mining Law were overlooked. Section 23, dealing with the rights and
obligations of the exploration permit grantee, states: "The permittee shall undertake exploration work
on the area as specified by its permit based on an approved work program." The next proviso
reads: "Any expenditure in excess of the yearly budget of the approved work program may be carried
forward and credited to the succeeding years covering the duration of the permit. x x x." (underscoring
supplied)

Clearly, even at the stage of application for an exploration permit, the applicant is required to submit --
for approval by the government -- a proposed work program for exploration, containing a yearly
budget of proposed expenditures. The State has the opportunity to pass upon (and approve or reject)
such proposed expenditures, with the foreknowledge that -- if approved -- these will subsequently be
recorded as pre-operating expenses that the contractor will have to recoup over the grace period.
That is not all.

Under Section 24, an exploration permit holder who determines the commercial viability of a project
covering a mining area may, within the term of the permit, file with the Mines and Geosciences
Bureau a declaration of mining project feasibility. This declaration is to be accompanied by a work
program for development for the Bureau's approval, the necessary prelude for entering into an FTAA,
a mineral production sharing agreement (MPSA), or some other mineral agreement. At this stage, too,
the government obviously has the opportunity to approve or reject the proposed work program and
budgeted expenditures for development works on the project. Such expenditures will ultimately
become the pre-operating and development costs that will have to be recovered by the contractor.

Naturally, with the submission of approved work programs and budgets for the exploration and the
development/construction phases, the government will be able to scrutinize and approve or
reject such expenditures. It will be well-informed as to the amounts of pre-operating and other
expenses that the contractor may legitimately recover and the approximate period of time needed to
effect such a recovery. There is therefore no way the contractor can just randomly post any amount of
pre-operating expenses and expect to recover the same.

The aforecited provisions on approved work programs and budgets have counterparts in Section 35,
which deals with the terms and conditions exclusively applicable to FTAAs. The said provision
requires certain terms and conditions to be incorporated into FTAAs; among them, "a firm
commitment x x x of an amount corresponding to the expenditure obligation that will be invested in the
contract area" and "representations and warranties x x x to timely deploy these [financing, managerial
and technical expertise and technological] resources under its supervision pursuant to the periodic
work programs and related budgets x x x," as well as "work programs and minimum expenditures
commitments." (underscoring supplied)

Unarguably, given the provisions of Section 35, the State has every opportunity to pass upon the
proposed expenditures under an FTAA and approve or reject them. It has access to all the
information it may need in order to determine in advance the amounts of pre-operating and
developmental expenses that will have to be recovered by the contractor and the amount of time
needed for such recovery.

In summary, we cannot agree that the third or last paragraph of Section 81 of RA 7942 is in any
manner unconstitutional.

No Deprivation of Beneficial Rights

It is also claimed that aside from the second and the third paragraphs of Section 81 (discussed
above), Sections 80, 84 and 112 of RA 7942 also operate to deprive the State of beneficial rights of
140

ownership over mineral resources; and give them away for free to private business enterprises
(including foreign owned corporations). Likewise, the said provisions have been construed as
constituting, together with Section 81, an ingenious attempt to resurrect the old and discredited
system of "license, concession or lease."

Specifically, Section 80 is condemned for limiting the State's share in a mineral production-sharing
agreement (MPSA) to just the excise tax on the mineral product. Under Section 151(A) of the Tax
Code, such tax is only 2 percent of the market value of the gross output of the minerals.
The colatilla in Section 84, the portion considered offensive to the Constitution, reiterates the same
limitation made in Section 80.73

It should be pointed out that Section 80 and the colatilla in Section 84 pertain only to MPSAs and
have no application to FTAAs. These particular statutory provisions do not come within the issues that
were defined and delineated by this Court during the Oral Argument -- particularly the third issue,
which pertained exclusively to FTAAs. Neither did the parties argue upon them in their pleadings.
Hence, this Court cannot make any pronouncement in this case regarding the constitutionality of
Sections 80 and 84 without violating the fundamental rules of due process. Indeed, the two provisos
will have to await another case specifically placing them in issue.

On the other hand, Section 11274 is disparaged for allegedly reverting FTAAs and all mineral
agreements to the old and discredited "license, concession or lease" system. This Section states in
relevant part that "the provisions of Chapter XIV [which includes Sections 80 to 82] on government
share in mineral production-sharing agreement x x x shall immediately govern and apply to a mining
lessee or contractor." (underscoring supplied) This provision is construed as signifying that the 2
percent excise tax which, pursuant to Section 80, comprises the government share in MPSAs shall
now also constitute the government share in FTAAs -- as well as in co-production agreements and
joint venture agreements -- to the exclusion of revenues of any other nature or from any other source.

Apart from the fact that Section 112 likewise does not come within the issues delineated by this Court
during the Oral Argument, and was never touched upon by the parties in their pleadings, it must also
be noted that the criticism hurled against this Section is rooted in unwarranted conclusions made
without considering other relevant provisions in the statute. Whether Section 112 may properly apply
to co-production or joint venture agreements, the fact of the matter is that it cannot be made to apply
to FTAAs.

First, Section 112 does not specifically mention or refer to FTAAs; the only reason it is being applied
to them at all is the fact that it happens to use the word "contractor." Hence, it is a bit of a stretch to
insist that it covers FTAAs as well. Second, mineral agreements, of which there are three types --
MPSAs, co-production agreements, and joint venture agreements -- are covered by Chapter V of RA
7942. On the other hand, FTAAs are covered by and in fact are the subject of Chapter VI, an entirely
different chapter altogether. The law obviously intends to treat them as a breed apart from mineral
agreements, since Section 35 (found in Chapter VI) creates a long list of specific terms, conditions,
commitments, representations and warranties -- which have not been made applicable to mineral
agreements -- to be incorporated into FTAAs.

Third, under Section 39, the FTAA contractor is given the option to "downgrade" -- to convert the
FTAA into a mineral agreement at any time during the term if the economic viability of the contract
area is inadequate to sustain large-scale mining operations. Thus, there is no reason to think that the
law through Section 112 intends to exact from FTAA contractors merely the same government share
(a 2 percent excise tax) that it apparently demands from contractors under the three forms of mineral
agreements. In brief, Section 112 does not apply to FTAAs.

Notwithstanding the foregoing explanation, Justices Carpio and Morales maintain that the Court must
rule now on the constitutionality of Sections 80, 84 and 112, allegedly because the WMCP FTAA
contains a provision which grants the contractor unbridled and "automatic" authority to convert the
FTAA into an MPSA; and should such conversion happen, the State would be prejudiced since its
share would be limited to the 2 percent excise tax. Justice Carpio adds that there are five MPSAs
already signed just awaiting the judgment of this Court on respondents' and intervenor's Motions for
Reconsideration. We hold however that, at this point, this argument is based on pure speculation. The
141

Court cannot rule on mere surmises and hypothetical assumptions, without firm factual anchor. We
repeat: basic due process requires that we hear the parties who have a real legal interest in the
MPSAs (i.e. the parties who executed them) before these MPSAs can be reviewed, or worse, struck
down by the Court. Anything less than that requirement would be arbitrary and capricious.

In any event, the conversion of the present FTAA into an MPSA is problematic. First, the contractor
must comply with the law, particularly Section 39 of RA 7942; inter alia, it must convincingly show that
the "economic viability of the contract is found to be inadequate to justify large-scale mining
operations;" second, it must contend with the President's exercise of the power of State control over
the EDU of natural resources; and third, it will have to risk a possible declaration of the
unconstitutionality (in a proper case) of Sections 80, 84 and 112.

The first requirement is not as simple as it looks. Section 39 contemplates a situation in which an
FTAA has already been executed and entered into, and is presumably being implemented, when the
contractor "discovers" that the mineral ore reserves in the contract area are not sufficient to justify
large-scale mining, and thus the contractor requests the conversion of the FTAA into an MPSA. The
contractor in effect needs to explain why, despite its exploration activities, including the conduct of
various geologic and other scientific tests and procedures in the contract area, it was unable to
determine correctly the mineral ore reserves and the economic viability of the area. The contractor
must explain why, after conducting such exploration activities, it decided to file a declaration of mining
feasibility, and to apply for an FTAA, thereby leading the State to believe that the area could sustain
large-scale mining. The contractor must justify fully why its earlier findings, based on scientific
procedures, tests and data, turned out to be wrong, or were way off. It must likewise prove that
its new findings, also based on scientific tests and procedures, are correct. Right away, this puts the
contractor's technical capabilities and expertise into serious doubt. We wonder if anyone would relish
being in this situation. The State could even question and challenge the contractor's qualification and
competence to continue the activity under an MPSA.

All in all, while there may be cogent grounds to assail the aforecited Sections, this Court -- on
considerations of due process -- cannot rule upon them here. Anyway, if later on these
Sections are declared unconstitutional, such declaration will not affect the other portions
since they are clearly separable from the rest.

Our Mineral Resources Not


Given Away for Free by RA 7942

Nevertheless, if only to disabuse our minds, we should address the contention that our mineral
resources are effectively given away for free by the law (RA 7942) in general and by Sections 80, 81,
84 and 112 in particular.

Foreign contractors do not just waltz into town one day and leave the next, taking away mineral
resources without paying anything. In order to get at the minerals, they have to invest huge sums of
money (tens or hundreds of millions of dollars) in exploration works first. If the exploration proves
unsuccessful, all the cash spent thereon will not be returned to the foreign investors; rather, those
funds will have been infused into the local economy, to remain there permanently. The benefits
therefrom cannot be simply ignored. And assuming that the foreign contractors are successful in
finding ore bodies that are viable for commercial exploitation, they do not just pluck out the minerals
and cart them off. They have first to build camp sites and roadways; dig mine shafts and connecting
tunnels; prepare tailing ponds, storage areas and vehicle depots; install their machinery and
equipment, generator sets, pumps, water tanks and sewer systems, and so on.

In short, they need to expend a great deal more of their funds for facilities, equipment and supplies,
fuel, salaries of local labor and technical staff, and other operating expenses. In the meantime, they
also have to pay taxes,75 duties, fees, and royalties. All told, the exploration, pre-feasibility, feasibility,
development and construction phases together add up to as many as eleven years. 76 The contractors
have to continually shell out funds for the duration of over a decade, before they can commence
commercial production from which they would eventually derive revenues. All that money translates
into a lot of "pump-priming" for the local economy.
142

Granted that the contractors are allowed subsequently to recover their pre-operating expenses, still,
that eventuality will happen only after they shall have first put out the cash and fueled the economy.
Moreover, in the process of recouping their investments and costs, the foreign contractors do not
actually pull out the money from the economy. Rather, they recover or recoup their investments out of
actual commercial production by not paying a portion of the basic government share corresponding to
national taxes, along with the additional government share, for a period of not more than five
years77 counted from the commencement of commercial production.

It must be noted that there can be no recovery without commencing actual commercial production. In
the meantime that the contractors are recouping costs, they need to continue operating; in order to do
so, they have to disburse money to meet their various needs. In short, money is continually infused
into the economy.

The foregoing discussion should serve to rid us of the mistaken belief that, since the foreign
contractors are allowed to recover their investments and costs, the end result is that they practically
get the minerals for free, which leaves the Filipino people none the better for it.

All Businesses Entitled


to Cost Recovery

Let it be put on record that not only foreign contractors, but all businessmen and all business entities
in general, have to recoup their investments and costs. That is one of the first things a student learns
in business school. Regardless of its nationality, and whether or not a business entity has a five-year
cost recovery period, it will -- must -- have to recoup its investments, one way or another. This is just
common business sense. Recovery of investments is absolutely indispensable for business survival;
and business survival ensures soundness of the economy, which is critical and contributory to the
general welfare of the people. Even government corporations must recoup their investments in order
to survive and continue in operation. And, as the preceding discussion has shown, there is no
business that gets ahead or earns profits without any cost to it.

It must also be stressed that, though the State owns vast mineral wealth, such wealth is not readily
accessible or transformable into usable and negotiable currency without the intervention of the
credible mining companies. Those untapped mineral resources, hidden beneath tons of earth and
rock, may as well not be there for all the good they do us right now. They have first to be extracted
and converted into marketable form, and the country needs the foreign contractor's funds, technology
and know-how for that.

After about eleven years of pre-operation and another five years for cost recovery, the foreign
contractors will have just broken even. Is it likely that they would at that point stop their operations and
leave? Certainly not. They have yet to make profits. Thus, for the remainder of the contract term, they
must strive to maintain profitability. During this period, they pay the whole of the basic government
share and the additional government share which, taken together with indirect taxes and other
contributions, amount to approximately 60 percent or more of the entire financial benefits generated
by the mining venture.

In sum, we can hardly talk about foreign contractors taking our mineral resources for free. It takes a
lot of hard cash to even begin to do what they do. And what they do in this country ultimately benefits
the local economy, grows businesses, generates employment, and creates infrastructure, as
discussed above. Hence, we definitely disagree with the sweeping claim that no FTAA under Section
81 will ever make any real contribution to the growth of the economy or to the general welfare of the
country. This is not a plea for foreign contractors. Rather, this is a question of focusing the judicial
spotlight squarely on all the pertinent facts as they bear upon the issue at hand, in order to avoid
leaping precipitately to ill-conceived conclusions not solidly grounded upon fact.

Repatriation of After-Tax Income

Another objection points to the alleged failure of the Mining Law to ensure real contributions to the
economic growth and general welfare of the country, as mandated by Section 2 of Article XII of the
143

Constitution. Pursuant to Section 81 of the law, the entire after-tax income arising from the
exploitation of mineral resources owned by the State supposedly belongs to the foreign contractors,
which will naturally repatriate the said after-tax income to their home countries, thereby resulting in no
real contribution to the economic growth of this country. Clearly, this contention is premised on
erroneous assumptions.

First, as already discussed in detail hereinabove, the concerned agencies have correctly interpreted
the second paragraph of Section 81 of RA 7942 to mean that the government is entitled to an
additional share, to be computed based on any one of the following factors: net mining revenues, the
present value of the cash flows, or excess profits reckoned against a benchmark rate of return on
investments. So it is not correct to say that all of the after-tax income will accrue to the foreign FTAA
contractor, as the government effectively receives a significant portion thereof.

Second, the foreign contractors can hardly "repatriate the entire after-tax income to their home
countries." Even a bit of knowledge of corporate finance will show that it will be impossible to maintain
a business as a "going concern" if the entire "net profit" earned in any particular year will be taken out
and repatriated. The "net income" figure reflected in the bottom line is a mere accounting figure not
necessarily corresponding to cash in the bank, or other quick assets. In order to produce and set
aside cash in an amount equivalent to the bottom line figure, one may need to sell off assets or
immediately collect receivables or liquidate short-term investments; but doing so may very likely
disrupt normal business operations.

In terms of cash flows, the funds corresponding to the net income as of a particular point in time
are actually in usein the normal course of business operations. Pulling out such net income disrupts
the cash flows and cash position of the enterprise and, depending on the amount being taken out,
could seriously cripple or endanger the normal operations and financial health of the business
enterprise. In short, no sane business person, concerned with maintaining the mining
enterprise as a going concern and keeping a foothold in its market, can afford to repatriate the
entire after-tax income to the home country.

The State's Receipt of Sixty


Percent of an FTAA Contractor's
After-Tax Income Not Mandatory

We now come to the next objection which runs this way: In FTAAs with a foreign contractor, the State
must receive at least 60 percent of the after-tax income from the exploitation of its mineral resources.
This share is the equivalent of the constitutional requirement that at least 60 percent of the capital,
and hence 60 percent of the income, of mining companies should remain in Filipino hands.

First, we fail to see how we can properly conclude that the Constitution mandates the State to extract
at least 60 percent of the after-tax income from a mining company run by a foreign contractor. The
argument is that the Charter requires the State's partner in a co-production agreement, joint venture
agreement or MPSA to be a Filipino corporation (at least 60 percent owned by Filipino citizens).

We question the logic of this reasoning, premised on a supposedly parallel or analogous situation. We
are, after all, dealing with an essentially different equation, one that involves different elements. The
Charter did not intend to fix an iron-clad rule on the 60 percent share, applicable to all
situations at all times and in all circumstances.If ever such was the intention of the framers, they
would have spelt it out in black and white. Verba legis will serve to dispel unwarranted and untenable
conclusions.

Second, if we would bother to do the math, we might better appreciate the impact (and
reasonableness) of what we are demanding of the foreign contractor. Let us use
a simplified illustration. Let us base it on gross revenues of, say, P500. After deducting operating
expenses, but prior to income tax, suppose a mining firm makes a taxable incomeof P100. A
corporate income tax of 32 percent results in P32 of taxable income going to the government, leaving
the mining firm with P68. Government then takes 60 percent thereof, equivalent to P40.80, leaving
only P27.20 for the mining firm.
144

At this point the government has pocketed P32.00 plus P40.80, or a total of P72.80 for every P100 of
taxable income, leaving the mining firm with only P27.20. But that is not all. The government has also
taken 2 percent excise tax "off the top," equivalent to another P10. Under the minimum 60 percent
proposal, the government nets around P82.80 (not counting other taxes, duties, fees and charges)
from a taxable income of P100 (assuming gross revenues of P500, for purposes of illustration). On
the other hand, the foreign contractor, which provided all the capital, equipment and labor, and took
all the entrepreneurial risks -- receives P27.20. One cannot but wonder whether such a distribution is
even remotely equitable and reasonable, considering the nature of the mining business. The amount
of P82.80 out of P100.00 is really a lot – it does not matter that we call part of it excise tax or income
tax, and another portion thereof income from exploitation of mineral resources. Some might think it
wonderful to be able to take the lion's share of the benefits. But we have to ask ourselves if we are
really serious in attracting the investments that are the indispensable and key element in generating
the monetary benefits of which we wish to take the lion's share. Fairness is a credo not only in law,
but also in business.

Third, the 60 percent rule in the petroleum industry cannot be insisted upon at all times in the mining
business. The reason happens to be the fact that in petroleum operations, the bulk of expenditures is
in exploration, but once the contractor has found and tapped into the deposit, subsequent investments
and expenditures are relatively minimal. The crude (or gas) keeps gushing out, and the work entailed
is just a matter of piping, transporting and storing. Not so in mineral mining. The ore body does not
pop out on its own. Even after it has been located, the contractor must continually invest in
machineries and expend funds to dig and build tunnels in order to access and extract the minerals
from underneath hundreds of tons of earth and rock.

As already stated, the numerous intrinsic differences involved in their respective operations and
requirements, cost structures and investment needs render it highly inappropriate to use petroleum
operations FTAAs as benchmarks for mining FTAAs. Verily, we cannot just ignore the realities of
the distinctly different situations and stubbornly insist on the "minimum 60 percent."

The Mining and the Oil Industries


Different From Each Other

To stress, there is no independent showing that the taking of at least a 60 percent share in the after-
tax income of a mining company operated by a foreign contractor is fair and reasonable under most if
not all circumstances. The fact that some petroleum companies like Shell acceded to such
percentage of sharing does not ipso facto mean that it is per se reasonable and applicable to non-
petroleum situations (that is, mining companies) as well. We can take judicial notice of the fact that
there are, after all, numerous intrinsic differences involved in their respective operations and
equipment or technological requirements, costs structures and capital investment needs, and product
pricing and markets.

There is no showing, for instance, that mining companies can readily cope with a 60 percent
government share in the same way petroleum companies apparently can. What we have is a
suggestion to enforce the 60 percent quota on the basis of a disjointed analogy. The only factor
common to the two disparate situations is the extraction of natural resources.

Indeed, we should take note of the fact that Congress made a distinction between mining firms and
petroleum companies. In Republic Act No. 7729 -- "An Act Reducing the Excise Tax Rates on Metallic
and Non-Metallic Minerals and Quarry Resources, Amending for the Purpose Section 151(a) of the
National Internal Revenue Code, as amended" -- the lawmakers fixed the excise tax rate on metallic
and non-metallic minerals at two percent of the actual market value of the annual gross output at the
time of removal. However, in the case of petroleum, the lawmakers set the excise tax rate for the first
taxable sale at fifteen percent of the fair international market price thereof.

There must have been a very sound reason that impelled Congress to impose two very dissimilar
excise tax rate. We cannot assume, without proof, that our honorable legislators acted arbitrarily,
capriciously and whimsically in this instance. We cannot just ignore the reality of two distinctly
different situations and stubbornly insist on going "minimum 60 percent."
145

To repeat, the mere fact that gas and oil exploration contracts grant the State 60 percent of the net
revenues does not necessarily imply that mining contracts should likewise yield a minimum of 60
percent for the State. Jumping to that erroneous conclusion is like comparing apples with oranges.
The exploration, development and utilization of gas and oil are simply different from those of mineral
resources.

To stress again, the main risk in gas and oil is in the exploration. But once oil in commercial quantities
is struck and the wells are put in place, the risk is relatively over and black gold simply flows out
continuously with comparativelyless need for fresh investments and technology.

On the other hand, even if minerals are found in viable quantities, there is still need for continuous
fresh capital and expertise to dig the mineral ores from the mines. Just because deposits of mineral
ores are found in one area is no guarantee that an equal amount can be found in the adjacent areas.
There are simply continuing risks and need for more capital, expertise and industry all the time.

Note, however, that the indirect benefits -- apart from the cash revenues -- are much more in the
mineral industry. As mines are explored and extracted, vast employment is created, roads and other
infrastructure are built, and other multiplier effects arise. On the other hand, once oil wells start
producing, there is less need for employment. Roads and other public works need not be constructed
continuously. In fine, there is no basis for saying that government revenues from the oil industry and
from the mineral industries are to be identical all the time.

Fourth, to our mind, the proffered "minimum 60 percent" suggestion tends to limit the flexibility and tie
the hands of government, ultimately hampering the country's competitiveness in the international
market, to the detriment of the Filipino people. This "you-have-to-give-us-60-percent-of-after-tax-
income-or-we-don't-do- business-with-you" approach is quite perilous. True, this situation may not
seem too unpalatable to the foreign contractor during good years, when international market prices
are up and the mining firm manages to keep its costs in check. However, under unfavorable economic
and business conditions, with costs spiraling skywards and minerals prices plummeting, a mining firm
may consider itself lucky to make just minimal profits.

The inflexible, carved-in-granite demand for a 60 percent government share may spell the end of the
mining venture, scare away potential investors, and thereby further worsen the already dismal
economic scenario. Moreover, such an unbending or unyielding policy prevents the government from
responding appropriately to changing economic conditions and shifting market forces. This inflexibility
further renders our country less attractive as an investment option compared with other countries.

And fifth, for this Court to decree imperiously that the government's share should be not less than 60
percent of the after-tax income of FTAA contractors at all times is nothing short of dictating upon the
government. The result, ironically, is that the State ends up losing control. To avoid compromising the
State's full control and supervision over the exploitation of mineral resources, this Court must back off
from insisting upon a "minimum 60 percent" rule. It is sufficient that the State has the power and
means, should it so decide, to get a 60 percent share (or more) in the contractor's net mining
revenues or after-tax income, or whatever other basis the government may decide to use in reckoning
its share. It is not necessary for it to do so in every case, regardless of circumstances.

In fact, the government must be trusted, must be accorded the liberty and the utmost flexibility to deal,
negotiate and transact with contractors and third parties as it sees fit; and upon terms that it
ascertains to be most favorable or most acceptable under the circumstances, even if it means
agreeing to less than 60 percent. Nothing must prevent the State from agreeing to a share less than
that, should it be deemed fit; otherwise the State will be deprived of full control over mineral
exploitation that the Charter has vested in it.

To stress again, there is simply no constitutional or legal provision fixing the minimum share of the
government in an FTAA at 60 percent of the net profit. For this Court to decree such minimum is to
wade into judicial legislation, and thereby inordinately impinge on the control power of the State. Let it
be clear: the Court is not against the grant of more benefits to the State; in fact, the more the better. If
during the FTAA negotiations, the President can secure 60 percent, 78 or even 90 percent, then all the
146

better for our people. But, if under the peculiar circumstances of a specific contract, the President
could secure only 50 percent or 55 percent, so be it. Needless to say, the President will have to report
(and be responsible for) the specific FTAA to Congress, and eventually to the people.

Finally, if it should later be found that the share agreed to is grossly disadvantageous to the
government, the officials responsible for entering into such a contract on its behalf will have to answer
to the courts for their malfeasance. And the contract provision voided. But this Court would abuse its
own authority should it force the government's hand to adopt the 60 percent demand of some of our
esteemed colleagues.

Capital and Expertise Provided,


Yet All Risks Assumed by Contractor

Here, we will repeat what has not been emphasized and appreciated enough: the fact that the
contractor in an FTAA provides all the needed capital, technical and managerial expertise, and
technology required to undertake the project.

In regard to the WMCP FTAA, the then foreign-owned WMCP as contractor committed, at the very
outset, to make capital investments of up to US$50 million in that single mining project. WMCP claims
to have already poured in well over P800 million into the country as of February 1998, with more in
the pipeline. These resources, valued in the tens or hundreds of millions of dollars, are invested in a
mining project that provides no assurance whatsoever that any part of the investment will be
ultimately recouped.

At the same time, the contractor must comply with legally imposed environmental standards and the
social obligations, for which it also commits to make significant expenditures of funds. Throughout, the
contractor assumes all the risks79 of the business, as mentioned earlier. These risks are indeed very
high, considering that the rate of success in exploration is extremely low. The probability of finding
any mineral or petroleum in commercially viable quantities is estimated to be about 1:1,000 only. On
that slim chance rides the contractor's hope of recouping investments and generating profits. And
when the contractor has recouped its initial investments in the project, the government share
increases to sixty percent of net benefits -- without the State ever being in peril of incurring costs,
expenses and losses.

And even in the worst possible scenario -- an absence of commercial quantities of minerals to justify
development -- the contractor would already have spent several million pesos for exploration works,
before arriving at the point in which it can make that determination and decide to cut its losses. In fact,
during the first year alone of the exploration period, the contractor was already committed to spend
not less than P24 million. The FTAA therefore clearly ensures benefits for the local economy,
courtesy of the contractor.

All in all, this setup cannot be regarded as disadvantageous to the State or the Filipino people;
it certainly cannot be said to convey beneficial ownership of our mineral resources to foreign
contractors.

Deductions Allowed by the


WMCP FTAA Reasonable

Petitioners question whether the State's weak control might render the sharing arrangements
ineffective. They cite the so-called "suspicious" deductions allowed by the WMCP FTAA in arriving at
the net mining revenue, which is the basis for computing the government share. The WMCP FTAA,
for instance, allows expenditures for "development within and outside the Contract Area relating to the
Mining Operations,"80 "consulting fees incurred both inside and outside the Philippines for work related
directly to the Mining Operations,"81 and "the establishment and administration of field offices including
administrative overheads incurred within and outside the Philippines which are properly allocatable to
the Mining Operations and reasonably related to the performance of the Contractor's obligations and
exercise of its rights under this Agreement."82
147

It is quite well known, however, that mining companies do perform some marketing activities abroad
in respect of selling their mineral products and by-products. Hence, it would not be improper to allow
the deduction of reasonable consulting fees incurred abroad, as well as administrative expenses and
overheads related to marketing offices also located abroad -- provided that these deductions are
directly related or properly allocatable to the mining operations and reasonably related to the
performance of the contractor's obligations and exercise of its rights. In any event, more facts are
needed. Until we see how these provisions actually operate, mere "suspicions" will not suffice to
propel this Court into taking action.

Section 7.9 of the WMCP FTAA


Invalid and Disadvantageous

Having defended the WMCP FTAA, we shall now turn to two defective provisos. Let us start with
Section 7.9 of the WMCP FTAA. While Section 7.7 gives the government a 60 percent share in the
net mining revenues of WMCP from the commencement of commercial production, Section 7.9
deprives the government of part or all of the said 60 percent. Under the latter provision, should
WMCP's foreign shareholders -- who originally owned 100 percent of the equity -- sell 60 percent or
more of its outstanding capital stock to a Filipino citizen or corporation, the State loses its right to
receive its 60 percent share in net mining revenues under Section 7.7.

Section 7.9 provides:

The percentage of Net Mining Revenues payable to the Government pursuant to Clause 7.7
shall be reduced by 1percent of Net Mining Revenues for every 1percent ownership interest
in the Contractor (i.e., WMCP) held by a Qualified Entity.83

Evidently, what Section 7.7 grants to the State is taken away in the next breath by Section 7.9 without
any offsetting compensation to the State. Thus, in reality, the State has no vested right to receive any
income from the FTAA for the exploitation of its mineral resources. Worse, it would seem that what is
given to the State in Section 7.7 is by mere tolerance of WMCP's foreign stockholders, who can at
any time cut off the government's entire 60 percent share. They can do so by simply selling 60
percent of WMCP's outstanding capital stock to a Philippine citizen or corporation. Moreover, the
proceeds of such sale will of course accrue to the foreign stockholders of WMCP, not to the State.

The sale of 60 percent of WMCP's outstanding equity to a corporation that is 60 percent Filipino-
owned and 40 percent foreign-owned will still trigger the operation of Section 7.9. Effectively, the
State will lose its right to receive all 60 percent of the net mining revenues of WMCP; and foreign
stockholders will own beneficially up to 64 percent of WMCP, consisting of the remaining 40 percent
foreign equity therein, plus the 24 percent pro-rata share in the buyer-corporation.84

In fact, the January 23, 2001 sale by WMCP's foreign stockholder of the entire outstanding equity in
WMCP to Sagittarius Mines, Inc. -- a domestic corporation at least 60 percent Filipino owned -- may
be deemed to have automatically triggered the operation of Section 7.9, without need of further action
by any party, and removed the State's right to receive the 60 percent share in net mining revenues.

At bottom, Section 7.9 has the effect of depriving the State of its 60 percent share in the net mining
revenues of WMCP without any offset or compensation whatsoever. It is possible that the inclusion of
the offending provision was initially prompted by the desire to provide some form of incentive for the
principal foreign stockholder in WMCP to eventually reduce its equity position and ultimately divest in
favor of Filipino citizens and corporations. However, as finally structured, Section 7.9 has the
deleterious effect of depriving government of the entire 60 percent share in WMCP's net mining
revenues, without any form of compensation whatsoever. Such an outcome is completely
unacceptable.

The whole point of developing the nation's natural resources is to benefit the Filipino people, future
generations included. And the State as sovereign and custodian of the nation's natural wealth is
mandated to protect, conserve, preserve and develop that part of the national patrimony for their
benefit. Hence, the Charter lays great emphasis on "real contributions to the economic growth and
148

general welfare of the country"85 as essential guiding principles to be kept in mind when negotiating
the terms and conditions of FTAAs.

Earlier, we held (1) that the State must be accorded the liberty and the utmost flexibility to deal,
negotiate and transact with contractors and third parties as it sees fit, and upon terms that it
ascertains to be most favorable or most acceptable under the circumstances, even if that should
mean agreeing to less than 60 percent; (2) that it is not necessary for the State to extract a 60 percent
share in every case and regardless of circumstances; and (3) that should the State be prevented from
agreeing to a share less than 60 percent as it deems fit, it will be deprived of the full control over
mineral exploitation that the Charter has vested in it.

That full control is obviously not an end in itself; it exists and subsists precisely because of the need to
serve and protect the national interest. In this instance, national interest finds particular application in
the protection of the national patrimony and the development and exploitation of the country's mineral
resources for the benefit of the Filipino people and the enhancement of economic growth and the
general welfare of the country. Undoubtedly, such full control can be misused and abused, as we
now witness.

Section 7.9 of the WMCP FTAA effectively gives away the State's share of net mining revenues
(provided for in Section 7.7) without anything in exchange. Moreover, this outcome constitutes unjust
enrichment on the part of the local and foreign stockholders of WMCP. By their mere divestment of up
to 60 percent equity in WMCP in favor of Filipino citizens and/or corporations, the local and foreign
stockholders get a windfall. Their share in the net mining revenues of WMCP is automatically
increased, without their having to pay the government anything for it. In short, the provision in
question is without a doubt grossly disadvantageous to the government, detrimental to the interests of
the Filipino people, and violative of public policy.

Moreover, it has been reiterated in numerous decisions 86 that the parties to a contract may establish
any agreements, terms and conditions that they deem convenient; but these should not be contrary to
law, morals, good customs, public order or public policy. 87 Being precisely violative of anti-graft
provisions and contrary to public policy, Section 7.9 must therefore be stricken off as invalid.

Whether the government officials concerned acceded to that provision by sheer mistake or with full
awareness of the ill consequences, is of no moment. It is hornbook doctrine that the principle of
estoppel does not operate against the government for the act of its agents, 88 and that it is never
estopped by any mistake or error on their part.89 It is therefore possible and proper to rectify the
situation at this time. Moreover, we may also say that the FTAA in question does not involve mere
contractual rights; being impressed as it is with public interest, the contractual provisions and
stipulations must yield to the common good and the national interest.

Since the offending provision is very much separable90 from Section 7.7 and the rest of the FTAA, the
deletion of Section 7.9 can be done without affecting or requiring the invalidation of the WMCP FTAA
itself. Such a deletion will preserve for the government its due share of the benefits. This way, the
mandates of the Constitution are complied with and the interests of the government fully protected,
while the business operations of the contractor are not needlessly disrupted.

Section 7.8(e) of the WMCP FTAA


Also Invalid and Disadvantageous

Section 7.8(e) of the WMCP FTAA is likewise invalid. It provides thus:

"7.8 The Government Share shall be deemed to include all of the following sums:

"(a) all Government taxes, fees, levies, costs, imposts, duties and royalties including
excise tax, corporate income tax, customs duty, sales tax, value added tax,
occupation and regulatory fees, Government controlled price stabilization schemes,
any other form of Government backed schemes, any tax on dividend payments by
the Contractor or its Affiliates in respect of revenues from the Mining Operations and
149

any tax on interest on domestic and foreign loans or other financial arrangements or
accommodations, including loans extended to the Contractor by its stockholders;

"(b) any payments to local and regional government, including taxes, fees, levies,
costs, imposts, duties, royalties, occupation and regulatory fees and infrastructure
contributions;

"(c) any payments to landowners, surface rights holders, occupiers, indigenous


people or Claimowners;

"(d) costs and expenses of fulfilling the Contractor's obligations to contribute to


national development in accordance with Clause 10.1(i) (1) and 10.1(i) (2);

"(e) an amount equivalent to whatever benefits that may be extended in the future by
the Government to the Contractor or to financial or technical assistance agreement
contractors in general;

"(f) all of the foregoing items which have not previously been offset against the
Government Share in an earlier Fiscal Year, adjusted for inflation." (underscoring
supplied)

Section 7.8(e) is out of place in the FTAA. It makes no sense why, for instance, money spent by the
government for the benefit of the contractor in building roads leading to the mine site should still be
deductible from the State's share in net mining revenues. Allowing this deduction results in benefiting
the contractor twice over. It constitutes unjust enrichment on the part of the contractor at the expense
of the government, since the latter is effectively being made to pay twice for the same item. 91 For
being grossly disadvantageous and prejudicial to the government and contrary to public policy,
Section 7.8(e) is undoubtedly invalid and must be declared to be without effect. Fortunately, this
provision can also easily be stricken off without affecting the rest of the FTAA.

Nothing Left Over


After Deductions?

In connection with Section 7.8, an objection has been raised: Specified in Section 7.8 are numerous
items of deduction from the State's 60 percent share. After taking these into account, will the State
ever receive anything for its ownership of the mineral resources?

We are confident that under normal circumstances, the answer will be yes. If we examine the various
items of "deduction" listed in Section 7.8 of the WMCP FTAA, we will find that they correspond closely
to the components or elements of the basic government share established in DAO 99-56, as
discussed in the earlier part of this Opinion.

Likewise, the balance of the government's 60 percent share -- after netting out the items of deduction
listed in Section 7.8 --corresponds closely to the additional government share provided for in DAO
99-56 which, we once again stress, has nothing at all to do with indirect taxes. The Ramos-DeVera
paper92 concisely presents the fiscal contribution of an FTAA under DAO 99-56 in this equation:

Receipts from an FTAA = basic gov't share + add'l gov't share

Transposed into a similar equation, the fiscal payments system from the WMCP FTAA assumes the
following formulation:

Government's 60 percent share in net mining revenues of WMCP = items listed in Sec. 7.8 of
the FTAA + balance of Gov't share, payable 4 months from the end of the fiscal year

It should become apparent that the fiscal arrangement under the WMCP FTAA is very similar to that
under DAO 99-56, with the "balance of government share payable 4 months from end of fiscal year"
150

being the equivalent of the additional government share computed in accordance with the "net-
mining-revenue-based option" under DAO 99-56, as discussed above. As we have emphasized
earlier, we find each of the three options for computing the additional government share -- as
presented in DAO 99-56 -- to be sound and reasonable.

We therefore conclude that there is nothing inherently wrong in the fiscal regime of the WMCP
FTAA, and certainly nothing to warrant the invalidation of the FTAA in its entirety.

Section 3.3 of the WMCP


FTAA Constitutional

Section 3.3 of the WMCP FTAA is assailed for violating supposed constitutional restrictions on the
term of FTAAs. The provision in question reads:

"3.3 This Agreement shall be renewed by the Government for a further period of twenty-five
(25) years under the same terms and conditions provided that the Contractor lodges a
request for renewal with the Government not less than sixty (60) days prior to the expiry of
the initial term of this Agreement and provided that the Contractor is not in breach of any of
the requirements of this Agreement."

Allegedly, the above provision runs afoul of Section 2 of Article XII of the 1987 Constitution, which
states:

"Sec. 2. All lands of the public domain, waters, minerals, coal, petroleum, and other mineral
oils, all forces of potential energy, fisheries, forests or timber, wildlife, flora and fauna, and
other natural resources are owned by the State. With the exception of agricultural lands, all
other natural resources shall not be alienated. The exploration, development and utilization of
natural resources shall be under the full control and supervision of the State. The State may
directly undertake such activities, or it may enter into co-production, joint venture or
production-sharing agreements with Filipino citizens or corporations or associations at least
sixty per centum of whose capital is owned by such citizens. Such agreements may be for a
period not exceeding twenty-five years, renewable for not more than twenty-five years,
and under such terms and conditions as may be provided by law. In cases of water
rights for irrigation, water supply, fisheries, or industrial uses other than the development of
water power, beneficial use may be the measure and limit of the grant.

"The State shall protect the nation's marine wealth in its archipelagic waters, territorial sea,
and exclusive economic zone, and reserve its use and enjoyment exclusively to Filipino
citizens.

"The Congress may, by law, allow small-scale utilization of natural resources by Filipino
citizens, as well as cooperative fish farming, with priority to subsistence fishermen and fish-
workers in rivers, lakes, bays and lagoons.

"The President may enter into agreements with foreign-owned corporations involving either
technical or financial assistance for large-scale exploration, development, and utilization of
minerals, petroleum, and other mineral oils according to the general terms and conditions
provided by law, based on real contributions to the economic growth and general welfare of
the country. In such agreements, the State shall promote the development and use of local
scientific and technical resources.

"The President shall notify the Congress of every contract entered into in accordance with this
provision, within thirty days from its execution."93

We hold that the term limitation of twenty-five years does not apply to FTAAs. The reason is that the
above provision is found within paragraph 1 of Section 2 of Article XII, which refers to mineral
agreements -- co-production agreements, joint venture agreements and mineral production-sharing
agreements -- which the government may enter into with Filipino citizens and corporations, at least 60
151

percent owned by Filipino citizens. The word "such" clearly refers to these three mineral agreements -
- CPAs, JVAs and MPSAs -- not to FTAAs.

Specifically, FTAAs are covered by paragraphs 4 and 5 of Section 2 of Article XII of the Constitution.
It will be noted that there are no term limitations provided for in the said paragraphs dealing with
FTAAs. This shows that FTAAs are sui generis, in a class of their own. This omission was obviously a
deliberate move on the part of the framers. They probably realized that FTAAs would be different in
many ways from MPSAs, JVAs and CPAs. The reason the framers did not fix term limitations
applicable to FTAAs is that they preferred to leave the matter to the discretion of the legislature and/or
the agencies involved in implementing the laws pertaining to FTAAs, in order to give the latter enough
flexibility and elbow room to meet changing circumstances.

Note also that, as previously stated, the exploratory phrases of an FTAA lasts up to eleven years.
Thereafter, a few more years would be gobbled up in start-up operations. It may take fifteen years
before an FTAA contractor can start earning profits. And thus, the period of 25 years may really be
short for an FTAA. Consider too that in this kind of agreement, the contractor assumes all
entrepreneurial risks. If no commercial quantities of minerals are found, the contractor bears all
financial losses. To compensate for this long gestation period and extra business risks, it would not be
totally unreasonable to allow it to continue EDU activities for another twenty five years.

In any event, the complaint is that, in essence, Section 3.3 gives the contractor the power to compel
the government to renew the WMCP FTAA for another 25 years and deprives the State of any say on
whether to renew the contract.

While we agree that Section 3.3 could have been worded so as to prevent it from favoring the
contractor, this provision does not violate any constitutional limits, since the said term limitation does
not apply at all to FTAAs. Neither can the provision be deemed in any manner to be illegal, as no law
is being violated thereby. It is certainly not illegal for the government to waive its option to refuse the
renewal of a commercial contract.

Verily, the government did not have to agree to Section 3.3. It could have said "No" to the stipulation,
but it did not. It appears that, in the process of negotiations, the other contracting party was able to
convince the government to agree to the renewal terms. Under the circumstances, it does not seem
proper for this Court to intervene and step in to undo what might have perhaps been a possible
miscalculation on the part of the State. If government believes that it is or will be aggrieved by the
effects of Section 3.3, the remedy is the renegotiation of the provision in order to provide the State the
option to not renew the FTAA.

Financial Benefits for Foreigners


Not Forbidden by the Constitution

Before leaving this subject matter, we find it necessary for us to rid ourselves of the false belief that
the Constitution somehow forbids foreign-owned corporations from deriving financial benefits from the
development of our natural or mineral resources.

The Constitution has never prohibited foreign corporations from acquiring and enjoying "beneficial
interest" in the development of Philippine natural resources. The State itself need not directly
undertake exploration, development, and utilization activities. Alternatively, the Constitution authorizes
the government to enter into joint venture agreements (JVAs), co-production agreements (CPAs) and
mineral production sharing agreements (MPSAs) with contractors who are Filipino citizens or
corporations that are at least 60 percent Filipino-owned. They may do the actual "dirty work" -- the
mining operations.

In the case of a 60 percent Filipino-owned corporation, the 40 percent individual and/or


corporate non-Filipino stakeholders obviously participate in the beneficial interest derived from the
development and utilization of our natural resources. They may receive by way of dividends, up to 40
percent of the contractor's earnings from the mining project. Likewise, they may have a say in the
decisions of the board of directors, since they are entitled to representation therein to the extent of
152

their equity participation, which the Constitution permits to be up to 40 percent of the contractor's
equity. Hence, the non-Filipino stakeholders may in that manner also participate in the management
of the contractor's natural resource development work. All of this is permitted by our Constitution, for
any natural resource, and without limitation even in regard to the magnitude of the mining project or
operations (see paragraph 1 of Section 2 of Article XII).

It is clear, then, that there is nothing inherently wrong with or constitutionally objectionable about the
idea of foreign individuals and entities having or enjoying "beneficial interest" in -- and participating in
the management of operations relative to -- the exploration, development and utilization of our natural
resources.

FTAA More Advantageous


Than Other Schemes
Like CPA, JVA and MPSA

A final point on the subject of beneficial interest. We believe the FTAA is a more advantageous
proposition for the government as compared with other agreements permitted by the Constitution. In a
CPA that the government enters into with one or more contractors, the government shall provide
inputs to the mining operations other than the mineral resource itself. 94

In a JVA, a JV company is organized by the government and the contractor, with both parties having
equity shares (investments); and the contractor is granted the exclusive right to conduct mining
operations and to extract minerals found in the area.95 On the other hand, in an MPSA, the
government grants the contractor the exclusive right to conduct mining operations within the contract
area and shares in the gross output; and the contractor provides the necessary financing, technology,
management and manpower.

The point being made here is that, in two of the three types of agreements under consideration,
the government has to ante up some risk capital for the enterprise. In other words, government funds
(public moneys) are withdrawn from other possible uses, put to work in the venture and placed at risk
in case the venture fails. This notwithstanding, management and control of the operations of the
enterprise are -- in all three arrangements -- in the hands of the contractor, with the government being
mainly a silent partner. The three types of agreement mentioned above apply to any natural resource,
without limitation and regardless of the size or magnitude of the project or operations.

In contrast to the foregoing arrangements, and pursuant to paragraph 4 of Section 2 of Article XII, the
FTAA is limited to large-scale projects and only for minerals, petroleum and other mineral oils. Here,
the Constitution removes the 40 percent cap on foreign ownership and allows the foreign corporation
to own up to 100 percent of the equity. Filipino capital may not be sufficient on account of the size of
the project, so the foreign entity may have to ante up all the risk capital.

Correlatively, the foreign stakeholder bears up to 100 percent of the risk of loss if the project fails. In
respect of the particular FTAA granted to it, WMCP (then 100 percent foreign owned) was
responsible, as contractor, for providing the entire equity, including all the inputs for the project. It was
to bear 100 percent of the risk of loss if the project failed, but its maximum potential "beneficial
interest" consisted only of 40 percent of the net beneficial interest, because the other 60 percent is
the share of the government, which will never be exposed to any risk of loss whatsoever.

In consonance with the degree of risk assumed, the FTAA vested in WMCP the day-to-day
management of the mining operations. Still such management is subject to the overall control and
supervision of the State in terms of regular reporting, approvals of work programs and budgets, and
so on.

So, one needs to consider in relative terms, the costs of inputs for, degree of risk attendant to, and
benefits derived or to be derived from a CPA, a JVA or an MPSA vis-à-vis those pertaining to an
FTAA. It may not be realistically asserted that the foreign grantee of an FTAA is being unduly favored
or benefited as compared with a foreign stakeholder in a corporation holding a CPA, a JVA or an
153

MPSA. Seen the other way around, the government is definitely better off with an FTAA than a CPA,
a JVA or an MPSA.

Developmental Policy on the Mining Industry

During the Oral Argument and in their Final Memorandum, petitioners repeatedly urged the Court to
consider whether mining as an industry and economic activity deserved to be accorded priority,
preference and government support as against, say, agriculture and other activities in which Filipinos
and the Philippines may have an "economic advantage." For instance, a recent US study96 reportedly
examined the economic performance of all local US counties that were dependent on mining and 20
percent of whose labor earnings between 1970 and 2000 came from mining enterprises.

The study -- covering 100 US counties in 25 states dependent on mining -- showed that per capita
income grew about 30 percent less in mining-dependent communities in the 1980s and 25 percent
less for the entire period 1980 to 2000; the level of per capita income was also lower. Therefore, given
the slower rate of growth, the gap between these and other local counties increased.

Petitioners invite attention to the OXFAM America Report's warning to developing nations that mining
brings with it serious economic problems, including increased regional inequality, unemployment and
poverty. They also cite the final report97 of the Extractive Industries Review project commissioned by
the World Bank (the WB-EIR Report), which warns of environmental degradation, social disruption,
conflict, and uneven sharing of benefits with local communities that bear the negative social and
environmental impact. The Report suggests that countries need to decide on the best way to exploit
their natural resources, in order to maximize the value added from the development of their resources
and ensure that they are on the path to sustainable development once the resources run out.

Whatever priority or preference may be given to mining vis-à-vis other economic or non-economic
activities is a question of policy that the President and Congress will have to address; it is not for this
Court to decide. This Court declares what the Constitution and the laws say, interprets only when
necessary, and refrains from delving into matters of policy.

Suffice it to say that the State control accorded by the Constitution over mining activities assures a
proper balancing of interests. More pointedly, such control will enable the President to demand the
best mining practices and the use of the best available technologies to protect the environment and to
rehabilitate mined-out areas. Indeed, under the Mining Law, the government can ensure the
protection of the environment during and after mining. It can likewise provide for the mechanisms to
protect the rights of indigenous communities, and thereby mold a more socially-responsive, culturally-
sensitive and sustainable mining industry.

Early on during the launching of the Presidential Mineral Industry Environmental Awards on February
6, 1997, then President Fidel V. Ramos captured the essence of balanced and sustainable mining in
these words:

"Long term, high profit mining translates into higher revenues for government, more decent
jobs for the population, more raw materials to feed the engines of downstream and allied
industries, and improved chances of human resource and countryside development by
creating self-reliant communities away from urban centers.

xxxxxxxxx

"Against a fragile and finite environment, it is sustainability that holds the key. In sustainable
mining, we take a middle ground where both production and protection goals are balanced,
and where parties-in-interest come to terms."

Neither has the present leadership been remiss in addressing the concerns of sustainable mining
operations. Recently, on January 16, 2004 and April 20, 2004, President Gloria Macapagal Arroyo
issued Executive Orders Nos. 270 and 270-A, respectively, "to promote responsible mineral
resources exploration, development and utilization, in order to enhance economic growth, in a manner
154

that adheres to the principles of sustainable development and with due regard for justice and equity,
sensitivity to the culture of the Filipino people and respect for Philippine sovereignty." 98

REFUTATION OF DISSENTS

The Court will now take up a number of other specific points raised in the dissents of Justices Carpio
and Morales.

1. Justice Morales introduced us to Hugh Morgan, former president and chief executive officer of
Western Mining Corporation (WMC) and former president of the Australian Mining Industry Council,
who spearheaded the vociferous opposition to the filing by aboriginal peoples of native title claims
against mining companies in Australia in the aftermath of the landmark Mabo decision by the
Australian High Court. According to sources quoted by our esteemed colleague, Morgan was also
a racist and a bigot. In the course of protesting Mabo, Morgan allegedly uttered derogatory remarks
belittling the aboriginal culture and race.

An unwritten caveat of this introduction is that this Court should be careful not to permit the entry
of the likes of Hugh Morgan and his hordes of alleged racist-bigots at WMC. With all due respect,
such scare tactics should have no place in the discussion of this case. We are deliberating on the
constitutionality of RA 7942, DAO 96-40 and the FTAA originally granted to WMCP, which had been
transferred to Sagittarius Mining, a Filipino corporation. We are not discussing the apparition of white
Anglo-Saxon racists/bigots massing at our gates.

2. On the proper interpretation of the phrase agreements involving either technical or financial
assistance, Justice Morales points out that at times we "conveniently omitted" the use of the
disjunctive either…or, which according to her denotes restriction; hence the phrase must be deemed
to connote restriction and limitation.

But, as Justice Carpio himself pointed out during the Oral Argument, the disjunctive phrase either
technical or financial assistance would, strictly speaking, literally mean that a foreign contractor
may provide only one or the other, but not both. And if both technical and financial assistance were
required for a project, the State would have to deal with at least two different foreign contractors --
one for financial and the other for technical assistance. And following on that, a foreign contractor,
though very much qualified to provide both kinds of assistance, would nevertheless be prohibited from
providing one kind as soon as it shall have agreed to provide the other.

But if the Court should follow this restrictive and literal construction, can we really find two (or more)
contractors who are willing to participate in one single project -- one to provide the "financial
assistance" only and the other the "technical assistance" exclusively; it would be excellent if these two
or more contractors happen to be willing and are able to cooperate and work closely together on the
same project (even if they are otherwise competitors). And it would be superb if no conflicts would
arise between or among them in the entire course of the contract. But what are the chances things will
turn out this way in the real world? To think that the framers deliberately imposed this kind of
restriction is to say that they were either exceedingly optimistic, or incredibly naïve. This begs the
question -- What laudable objective or purpose could possibly be served by such strict and restrictive
literal interpretation?

3. Citing Oposa v. Factoran Jr., Justice Morales claims that a service contract is not a contract or
property right which merits protection by the due process clause of the Constitution, but merely a
license or privilege which may be validly revoked, rescinded or withdrawn by executive action
whenever dictated by public interest or public welfare.

Oposa cites Tan v. Director of Forestry and Ysmael v. Deputy Executive Secretary as authority. The
latter cases dealt specifically with timber licenses only. Oposa allegedly reiterated that a license is
merely a permit or privilege to do what otherwise would be unlawful, and is not a contract between the
authority, federal, state or municipal, granting it and the person to whom it is granted; neither is it
property or a property right, nor does it create a vested right; nor is it taxation. Thus this Court held
that the granting of license does not create irrevocable rights, neither is it property or property rights.
155

Should Oposa be deemed applicable to the case at bar, on the argument that natural resources are
also involved in this situation? We do not think so. A grantee of a timber license, permit or license
agreement gets to cut the timber already growing on the surface; it need not dig up tons of earth to
get at the logs. In a logging concession, the investment of the licensee is not as substantial as the
investment of a large-scale mining contractor. If a timber license were revoked, the licensee packs up
its gear and moves to a new area applied for, and starts over; what it leaves behind are mainly the
trails leading to the logging site.

In contrast, the mining contractor will have sunk a great deal of money (tens of millions of dollars) into
the ground, so to speak, for exploration activities, for development of the mine site and infrastructure,
and for the actual excavation and extraction of minerals, including the extensive tunneling work to
reach the ore body. The cancellation of the mining contract will utterly deprive the contractor of its
investments (i.e., prevent recovery of investments), most of which cannot be pulled out.

To say that an FTAA is just like a mere timber license or permit and does not involve contract or
property rights which merit protection by the due process clause of the Constitution, and may
therefore be revoked or cancelled in the blink of an eye, is to adopt a well-nigh confiscatory stance; at
the very least, it is downright dismissive of the property rights of businesspersons and corporate
entities that have investments in the mining industry, whose investments, operations and
expenditures do contribute to the general welfare of the people, the coffers of government, and the
strength of the economy. Such a pronouncement will surely discourage investments (local and
foreign) which are critically needed to fuel the engine of economic growth and move this country out
of the rut of poverty. In sum, Oposa is not applicable.

4. Justice Morales adverts to the supposedly "clear intention" of the framers of the Constitution to
reserve our natural resources exclusively for the Filipino people. She then quoted from the records of
the ConCom deliberations a passage in which then Commissioner Davide explained his vote, arguing
in the process that aliens ought not be allowed to participate in the enjoyment of our natural
resources. One passage does not suffice to capture the tenor or substance of the entire extensive
deliberations of the commissioners, or to reveal the clear intention of the framers as a group. A re-
reading of the entire deliberations (quoted here earlier) is necessary if we are to understand the true
intent of the framers.

5. Since 1935, the Filipino people, through their Constitution, have decided that the retardation or
delay in the exploration, development or utilization of the nation's natural resources is merely
secondary to the protection and preservation of their ownership of the natural resources, so says
Justice Morales, citing Aruego. If it is true that the framers of the 1987 Constitution did not care much
about alleviating the retardation or delay in the development and utilization of our natural
resources, why did they bother to write paragraph 4 at all? Were they merely paying lip service to
large-scale exploration, development and utilization? They could have just completely ignored the
subject matter and left it to be dealt with through a future constitutional amendment. But we have to
harmonize every part of the Constitution and to interpret each provision in a manner that would give
life and meaning to it and to the rest of the provisions. It is obvious that a literal interpretation of
paragraph 4 will render it utterly inutile and inoperative.

6. According to Justice Morales, the deliberations of the Constitutional Commission do not support our
contention that the framers, by specifying such agreements involving financial or technical assistance,
necessarily gave implied assent to everything that these agreements implicitly entailed, or that could
reasonably be deemed necessary to make them tenable and effective, including management
authority in the day-to-day operations. As proof thereof, she quotes one single passage from the
ConCom deliberations, consisting of an exchange among Commissioners Tingson, Garcia and
Monsod.

However, the quoted exchange does not serve to contradict our argument; it even bolsters it. Comm.
Christian Monsod was quoted as saying: "xxx I think we have to make a distinction that it is not really
realistic to say that we will borrow on our own terms. Maybe we can say that we inherited unjust
loans, and we would like to repay these on terms that are not prejudicial to our own growth. But the
general statement that we should only borrow on our own terms is a bit unrealistic." Comm. Monsod is
one who knew whereof he spoke.
156

7. Justice Morales also declares that the optimal time for the conversion of an FTAA into an MPSA is
after completion of the exploration phase and just before undertaking the development and
construction phase, on account of the fact that the requirement for a minimum investment of $50
million is applicable only during the development, construction and utilization phase, but not during
the exploration phase, when the foreign contractor need merely comply with minimum ground
expenditures. Thus by converting, the foreign contractor maximizes its profits by avoiding its
obligation to make the minimum investment of $50 million.

This argument forgets that the foreign contractor is in the game precisely to make money. In order to
come anywhere near profitability, the contractor must first extract and sell the mineral ore. In order to
do that, it must also develop and construct the mining facilities, set up its machineries and equipment
and dig the tunnels to get to the deposit. The contractor is thus compelled to expend funds in order to
make profits. If it decides to cut back on investments and expenditures, it will necessarily sacrifice the
pace of development and utilization; it will necessarily sacrifice the amount of profits it can make from
the mining operations. In fact, at certain less-than-optimal levels of operation, the stream of revenues
generated may not even be enough to cover variable expenses, let alone overhead expenses; this is
a dismal situation anyone would want to avoid. In order to make money, one has to spend money.
This truism applies to the mining industry as well.

8. Mortgaging the minerals to secure a foreign FTAA contractor's obligations is anomalous, according
to Justice Morales since the contractor was from the beginning obliged to provide all financing needed
for the mining operations. However, the mortgaging of minerals by the contractor does not necessarily
signify that the contractor is unable to provide all financing required for the project, or that it does not
have the financial capability to undertake large-scale operations. Mortgaging of mineral products, just
like the assignment (by way of security) of manufactured goods and goods in inventory, and the
assignment of receivables, is an ordinary requirement of banks, even in the case of clients with more
than sufficient financial resources. And nowadays, even the richest and best managed corporations
make use of bank credit facilities -- it does not necessarily signify that they do not have the financial
resources or are unable to provide the financing on their own; it is just a manner of maximizing the
use of their funds.

9. Does the contractor in reality acquire the surface rights "for free," by virtue of the fact that it is
entitled to reimbursement for the costs of acquisition and maintenance, adjusted for inflation? We
think not. The "reimbursement" is possible only at the end of the term of the contract, when the
surface rights will no longer be needed, and the land previously acquired will have to be disposed of,
in which case the contractor gets reimbursement from the sales proceeds. The contractor has to pay
out the acquisition price for the land. That money will belong to the seller of the land. Only if and when
the land is finally sold off will the contractor get any reimbursement. In other words, the contractor will
have been cash-out for the entire duration of the term of the contract -- 25 or 50 years, depending. If
we calculate the cost of money at say 12 percent per annum, that is the cost or opportunity loss to the
contractor, in addition to the amount of the acquisition price. 12 percent per annum for 50 years is 600
percent; this, without any compounding yet. The cost of money is therefore at least 600 percent of the
original acquisition cost; it is in addition to the acquisition cost. "For free"? Not by a long shot.

10. The contractor will acquire and hold up to 5,000 hectares? We doubt it. The acquisition by the
State of land for the contractor is just to enable the contractor to establish its mine site, build its
facilities, establish a tailings pond, set up its machinery and equipment, and dig mine shafts and
tunnels, etc. It is impossible that the surface requirement will aggregate 5,000 hectares. Much of the
operations will consist of the tunneling and digging underground, which will not require possessing or
using any land surface. 5,000 hectares is way too much for the needs of a mining operator. It simply
will not spend its cash to acquire property that it will not need; the cash may be better employed for
the actual mining operations, to yield a profit.

11. Justice Carpio claims that the phrase among other things (found in the second paragraph of
Section 81 of the Mining Act) is being incorrectly treated as a delegation of legislative power to the
DENR secretary to issue DAO 99-56 and prescribe the formulae therein on the State's share from
mining operations. He adds that the phrase among other things was not intended as a delegation of
legislative power to the DENR secretary, much less could it be deemed a valid delegation of
legislative power, since there is nothing in the second paragraph of Section 81 which can be said to
157

grant any delegated legislative power to the DENR secretary. And even if there were, such delegation
would be void, for lack of any standards by which the delegated power shall be exercised.

While there is nothing in the second paragraph of Section 81 which can directly be construed as a
delegation of legislative power to the DENR secretary, it does not mean that DAO 99-56 is invalid per
se, or that the secretary acted without any authority or jurisdiction in issuing DAO 99-56. As we stated
earlier in our Prologue, "Who or what organ of government actually exercises this power of control on
behalf of the State? The Constitution is crystal clear: the President. Indeed, the Chief Executive is the
official constitutionally mandated to 'enter into agreements with foreign owned corporations.' On the
other hand, Congress may review the action of the President once it is notified of 'every contract
entered into in accordance with this [constitutional] provision within thirty days from its execution.'"It is
the President who is constitutionally mandated to enter into FTAAs with foreign corporations, and in
doing so, it is within the President's prerogative to specify certain terms and conditions of the
FTAAs, for example, the fiscal regime of FTAAs -- i.e., the sharing of the net mining revenues
between the contractor and the State.

Being the President's alter ego with respect to the control and supervision of the mining industry, the
DENR secretary, acting for the President, is necessarily clothed with the requisite authority and power
to draw up guidelines delineating certain terms and conditions, and specifying therein the terms of
sharing of benefits from mining, to be applicable to FTAAs in general. It is important to remember that
DAO 99-56 has been in existence for almost six years, and has not been amended or revoked by the
President.

The issuance of DAO 99-56 did not involve the exercise of delegated legislative power. The
legislature did not delegate the power to determine the nature, extent and composition of the items
that would come under the phrase among other things. The legislature's power pertains to the
imposition of taxes, duties and fees. This power was not delegated to the DENR secretary. But the
power to negotiate and enter into FTAAs was withheld from Congress, and reserved for the President.
In determining the sharing of mining benefits, i.e., in specifying what the phrase among other
things include, the President (through the secretary acting in his/her behalf) was not determining the
amount or rate of taxes, duties and fees, but rather the amount of INCOME to be derived from
minerals to be extracted and sold, income which belongs to the State as owner of the mineral
resources. We may say that, in the second paragraph of Section 81, the legislature in a sense
intruded partially into the President's sphere of authority when the former provided that

"The Government share in financial or technical assistance agreement shall consist of,
among other things, the contractor's corporate income tax, excise tax, special allowance,
withholding tax due from the contractor's foreign stockholders arising from dividend or interest
payments to the said foreign stockholder in case of a foreign national and all such other
taxes, duties and fees as provided for under existing laws." (Italics supplied)

But it did not usurp the President's authority since the provision merely included the enumerated
items as part of the government share, without foreclosing or in any way preventing (as in fact
Congress could not validly prevent) the President from determining what constitutes the State's
compensation derived from FTAAs. In this case, the President in effect directed the inclusion or
addition of "other things," viz., INCOME for the owner of the resources, in the government's share,
while adopting the items enumerated by Congress as part of the government share also.

12. Justice Carpio's insistence on applying the ejusdem generis rule of statutory construction to the
phrase among other things is therefore useless, and must fall by the wayside. There is no point trying
to construe that phrase in relation to the enumeration of taxes, duties and fees found in paragraph 2
of Section 81, precisely because "the constitutional power to prescribe the sharing of mining
income between the State and mining companies,"to quote Justice Carpio pursuant to an FTAA
is constitutionally lodged with the President, not with Congress. It thus makes no sense to
persist in giving the phrase among other things a restricted meaning referring only to taxes, duties
and fees.

13. Strangely, Justice Carpio claims that the DENR secretary can change the formulae in DAO 99-56
any time even without the approval of the President, and the secretary is the sole authority to
158

determine the amount of consideration that the State shall receive in an FTAA, because Section 5 of
the DAO states that "xxx any amendment of an FTAA other than the provision on fiscal regime shall
require the negotiation with the Negotiation Panel and the recommendation of the Secretary for
approval of the President xxx". Allegedly, because of that provision, if an amendment in the FTAA
involves non-fiscal matters, the amendment requires approval of the President, but if the amendment
involves a change in the fiscal regime, the DENR secretary has the final authority, and approval of the
President may be dispensed with; hence the secretary is more powerful than the President.

We believe there is some distortion resulting from the quoted provision being taken out of context.
Section 5 of DAO 99-56 reads as follows:

"Section 5. Status of Existing FTAAs. All FTAAs approved prior to the effectivity of this
Administrative Order shall remain valid and be recognized by the Government: Provided, That
should a Contractor desire to amend its FTAA, it shall do so by filing a Letter of Intent (LOI) to
the Secretary thru the Director. Provided, further, That if the Contractor desires to amend the
fiscal regime of its FTAA, it may do so by seeking for the amendment of its FTAA's whole
fiscal regime by adopting the fiscal regime provided hereof: Provided, finally, That any
amendment of an FTAA other than the provision on fiscal regime shall require the negotiation
with the Negotiating Panel and the recommendation of the Secretary for approval of the
President of the Republic of the Philippines." (underscoring supplied)

It looks like another case of misapprehension. The proviso being objected to by Justice Carpio is
actually preceded by a phrase that requires a contractor desiring to amend the fiscal regime of its
FTAA, to amend the same by adopting the fiscal regime prescribed in DAO 99-56 -- i.e., solely in that
manner, and in no other. Obviously, since DAO 99-56 was issued by the secretary under the
authority and with the presumed approval of the President, the amendment of an FTAA by
merely adopting the fiscal regime prescribed in said DAO 99-56 (and nothing more) need not
have the express clearance of the President anymore. It is as if the same had been pre-approved.
We cannot fathom the complaint that that makes the secretary more powerful than the President, or
that the former is trying to hide things from the President or Congress.

14. Based on the first sentence of Section 5 of DAO 99-56, which states "[A]ll FTAAs approved prior
to the effectivity of this Administrative Order shall remain valid and be recognized by the
Government", Justice Carpio concludes that said Administrative Order allegedly exempts FTAAs
approved prior to its effectivity -- like the WMCP FTAA -- from having to pay the State any share from
their mining income, apart from taxes, duties and fees.

We disagree. What we see in black and white is the statement that the FTAAs approved before the
DAO came into effect are to continue to be valid and will be recognized by the State. Nothing is said
about their fiscal regimes. Certainly, there is no basis to claim that the contractors under said FTAAs
were being exempted from paying the government a share in their mining incomes.

For the record, the WMCP FTAA is NOT and has never been exempt from paying the government
share. The WMCP FTAA has its own fiscal regime -- Section 7.7 -- which gives the government
a 60 percent share in the net mining revenues of WMCP from the commencement of
commercial production.

For that very reason, we have never said that DAO 99-56 is the basis for claiming that the WMCP
FTAA has a consideration. Hence, we find quite out of place Justice Carpio's statement that ironically,
DAO 99-56, the very authority cited to support the claim that the WMCP FTAA has a consideration,
does not apply to the WMCP FTAA. By its own express terms, DAO 99-56 does not apply to FTAAs
executed before the issuance of DAO 99-56, like the WMCP FTAA. The majority's position has
allegedly no leg to stand on since even DAO 99-56, assuming it is valid, cannot save the WMCP
FTAA from want of consideration. Even assuming arguendo that DAO 99-56 does not apply to the
WMCP FTAA, nevertheless, the WMCP FTAA has its own fiscal regime, found in Section 7.7 thereof.
Hence, there is no such thing as "want of consideration" here.
159

Still more startling is this claim: The majority supposedly agrees that the provisions of the WMCP
FTAA, which grant a sham consideration to the State, are void. Since the majority agrees that the
WMCP FTAA has a sham consideration, the WMCP FTAA thus lacks the third element of a valid
contract. The Decision should declare the WMCP FTAA void for want of consideration unless it treats
the contract as an MPSA under Section 80. Indeed the only recourse of WMCP to save the validity of
its contract is to convert it into an MPSA.

To clarify, we said that Sections 7.9 and 7.8(e) of the WMCP FTAA are provisions grossly
disadvantageous to government and detrimental to the interests of the Filipino people, as well as
violative of public policy, and must therefore be stricken off as invalid. Since the offending provisions
are very much separable from Section 7.7 and the rest of the FTAA, the deletion of Sections 7.9 and
7.8(e) can be done without affecting or requiring the invalidation of the WMCP FTAA itself, and such
deletion will preserve for government its due share of the 60 percent benefits. Therefore, the WMCP
FTAA is NOT bereft of a valid consideration (assuming for the nonce that indeed this is the
"consideration" of the FTAA).

SUMMATION

To conclude, a summary of the key points discussed above is now in order.

The Meaning of "Agreements Involving


Either Technical or Financial Assistance"

Applying familiar principles of constitutional construction to the phrase agreements involving either
technical or financial assistance, the framers' choice of words does not indicate the intent to exclude
other modes of assistance, but rather implies that there are other things being included or possibly
being made part of the agreement, apart from financial or technical assistance. The drafters avoided
the use of restrictive and stringent phraseology; a verba legis scrutiny of Section 2 of Article XII of the
Constitution discloses not even a hint of a desire to prohibit foreign involvement in the management or
operation of mining activities, or to eradicate service contracts. Such moves would necessarily imply
an underlying drastic shift in fundamental economic and developmental policies of the State. That
change requires a much more definite and irrefutable basis than mere omission of the words "service
contract" from the new Constitution.

Furthermore, a literal and restrictive interpretation of this paragraph leads to logical inconsistencies. A
constitutional provision specifically allowing foreign-owned corporations to render financial or
technical assistance in respect of mining or any other commercial activity was clearly unnecessary;
the provision was meant to refer to more than mere financial or technical assistance.

Also, if paragraph 4 permits only agreements for financial or technical assistance, there would be no
point in requiring that they be "based on real contributions to the economic growth and general
welfare of the country." And considering that there were various long-term service contracts still in
force and effect at the time the new Charter was being drafted, the absence of any transitory
provisions to govern the termination and closing-out of the then existing service contracts strongly
militates against the theory that the mere omission of "service contracts" signaled their prohibition by
the new Constitution.

Resort to the deliberations of the Constitutional Commission is therefore unavoidable, and a careful
scrutiny thereof conclusively shows that the ConCom members discussed agreements involving
either technical or financial assistance in the same sense as service contracts and used the terms
interchangeably. The drafters in fact knew that the agreements with foreign corporations were going
to entail not mere technical or financial assistance but, rather, foreign investment in and management
of an enterprise for large-scale exploration, development and utilization of minerals.

The framers spoke about service contracts as the concept was understood in the 1973 Constitution. It
is obvious from their discussions that they did not intend to ban or eradicate service contracts.
Instead, they were intent on crafting provisions to put in place safeguards that would eliminate or
minimize the abuses prevalent during the martial law regime. In brief, they were going to permit
160

service contracts with foreign corporations as contractors, but with safety measures to
prevent abuses, as an exception to the general norm established in the first paragraph of
Section 2 of Article XII, which reserves or limits to Filipino citizens and corporations at least 60
percent owned by such citizens the exploration, development and utilization of mineral or
petroleum resources. This was prompted by the perceived insufficiency of Filipino capital and the
felt need for foreign expertise in the EDU of mineral resources.

Despite strong opposition from some ConCom members during the final voting, the Article on the
National Economy and Patrimony -- including paragraph 4 allowing service contracts with foreign
corporations as an exception to the general norm in paragraph 1 of Section 2 of the same Article --
was resoundingly and overwhelmingly approved.

The drafters, many of whom were economists, academicians, lawyers, businesspersons and
politicians knew that foreign entities will not enter into agreements involving assistance without
requiring measures of protection to ensure the success of the venture and repayment of their
investments, loans and other financial assistance, and ultimately to protect the business reputation of
the foreign corporations. The drafters, by specifying such agreements involving assistance,
necessarily gave implied assent to everything that these agreements entailed or that could reasonably
be deemed necessary to make them tenable and effective -- including management authority with
respect to the day-to-day operations of the enterprise, and measures for the protection of the interests
of the foreign corporation, at least to the extent that they are consistent with Philippine sovereignty
over natural resources, the constitutional requirement of State control, and beneficial ownership of
natural resources remaining vested in the State.

From the foregoing, it is clear that agreements involving either technical or financial
assistance referred to in paragraph 4 are in fact service contracts, but such new service contracts are
between foreign corporations acting as contractors on the one hand, and on the other hand
government as principal or "owner" (of the works), whereby the foreign contractor provides the capital,
technology and technical know-how, and managerial expertise in the creation and operation of the
large-scale mining/extractive enterprise, and government through its agencies (DENR, MGB) actively
exercises full control and supervision over the entire enterprise.

Such service contracts may be entered into only with respect to minerals, petroleum and other
mineral oils. The grant of such service contracts is subject to several safeguards, among them: (1)
that the service contract be crafted in accordance with a general law setting standard or uniform
terms, conditions and requirements; (2) the President be the signatory for the government; and (3) the
President report the executed agreement to Congress within thirty days.

Ultimate Test: Full State Control

To repeat, the primacy of the principle of the State's sovereign ownership of all mineral resources,
and its full control and supervision over all aspects of exploration, development and utilization of
natural resources must be upheld. But "full control and supervision" cannot be taken literally to mean
that the State controls and supervises everything down to the minutest details and makes all required
actions, as this would render impossible the legitimate exercise by the contractor of a reasonable
degree of management prerogative and authority, indispensable to the proper functioning of the
mining enterprise. Also, government need not micro-manage mining operations and day-to-day affairs
of the enterprise in order to be considered as exercising full control and supervision.

Control, as utilized in Section 2 of Article XII, must be taken to mean a degree of control sufficient to
enable the State to direct, restrain, regulate and govern the affairs of the extractive enterprises.
Control by the State may be on a macro level, through the establishment of policies, guidelines,
regulations, industry standards and similar measures that would enable government to regulate the
conduct of affairs in various enterprises, and restrain activities deemed not desirable or beneficial,
with the end in view of ensuring that these enterprises contribute to the economic development and
general welfare of the country, conserve the environment, and uplift the well-being of the local
affected communities. Such a degree of control would be compatible with permitting the foreign
contractor sufficient and reasonable management authority over the enterprise it has invested in, to
ensure efficient and profitable operation.
161

Government Granted Full Control


by RA 7942 and DAO 96-40

Baseless are petitioners' sweeping claims that RA 7942 and its Implementing Rules and Regulations
make it possible for FTAA contracts to cede full control and management of mining enterprises over
to fully foreign owned corporations. Equally wobbly is the assertion that the State is reduced to a
passive regulator dependent on submitted plans and reports, with weak review and audit powers and
little say in the decision-making of the enterprise, for which reasons "beneficial ownership" of the
mineral resources is allegedly ceded to the foreign contractor.

As discussed hereinabove, the State's full control and supervision over mining operations are ensured
through the following provisions in RA 7942: Sections 8, 9, 16, 19, 24, 35[(b), (e), (f), (g), (h), (k), (l),
(m) and (o)], 40, 57, 66, 69, 70, and Chapters XI and XVII; as well as the following provisions of DAO
96-40: Sections7[(d) and (f)], 35(a-2), 53[(a-4) and (d)], 54, 56[(g), (h), (l), (m) and (n)], 56(2), 60, 66,
144, 168, 171 and 270, and also Chapters XV, XVI and XXIV.

Through the foregoing provisions, the government agencies concerned are empowered to approve or
disapprove -- hence, in a position to influence, direct, and change -- the various work programs and
the corresponding minimum expenditure commitments for each of the exploration, development and
utilization phases of the enterprise. Once they have been approved, the contractor's compliance with
its commitments therein will be monitored. Figures for mineral production and sales are regularly
monitored and subjected to government review, to ensure that the products and by-products are
disposed of at the best prices; copies of sales agreements have to be submitted to and registered
with MGB.

The contractor is mandated to open its books of accounts and records for scrutiny, to enable the State
to determine that the government share has been fully paid. The State may likewise compel
compliance by the contractor with mandatory requirements on mine safety, health and environmental
protection, and the use of anti-pollution technology and facilities. The contractor is also obligated to
assist the development of the mining community, and pay royalties to the indigenous peoples
concerned. And violation of any of the FTAA's terms and conditions, and/or non-compliance with
statutes or regulations, may be penalized by cancellation of the FTAA. Such sanction is significant to
a contractor who may have yet to recover the tens or hundreds of millions of dollars sunk into a
mining project.

Overall, the State definitely has a pivotal say in the operation of the individual enterprises, and can set
directions and objectives, detect deviations and non-compliances by the contractor, and enforce
compliance and impose sanctions should the occasion arise. Hence, RA 7942 and DAO 96-40 vest in
government more than a sufficient degree of control and supervision over the conduct of mining
operations.

Section 3(aq) of RA 7942 was objected to as being unconstitutional for allowing a foreign contractor to
apply for and hold an exploration permit. During the exploration phase, the permit grantee (and
prospective contractor) is spending and investing heavily in exploration activities without yet being
able to extract minerals and generate revenues. The exploration permit issued under Sections 3(aq),
20 and 23 of RA 7942, which allows exploration but not extraction, serves to protect the interests and
rights of the exploration permit grantee (and would-be contractor), foreign or local. Otherwise, the
exploration works already conducted, and expenditures already made, may end up only benefiting
claim-jumpers. Thus, Section 3(aq) of RA 7942 is not unconstitutional.

WMCP FTAA Likewise Gives the


State Full Control and Supervision

The WMCP FTAA obligates the contractor to account for the value of production and sale of minerals
(Clause 1.4); requires that the contractor's work program, activities and budgets be approved by the
State (Clause 2.1); gives the DENR secretary power to extend the exploration period (Clause 3.2-a);
requires approval by the State for incorporation of lands into the contract area (Clause 4.3-c); requires
Bureau of Forest Development approval for inclusion of forest reserves as part of the FTAA contract
162

area (Clause 4.5); obligates the contractor to periodically relinquish parts of the contract area not
needed for exploration and development (Clause 4.6); requires submission of a declaration of mining
feasibility for approval by the State (Clause 4.6-b); obligates the contractor to report to the State the
results of its exploration activities (Clause 4.9); requires the contractor to obtain State approval for its
work programs for the succeeding two year periods, containing the proposed work activities and
expenditures budget related to exploration (Clause 5.1); requires the contractor to obtain State
approval for its proposed expenditures for exploration activities (Clause 5.2); requires the contractor
to submit an annual report on geological, geophysical, geochemical and other information relating to
its explorations within the FTAA area (Clause 5.3-a); requires the contractor to submit within six
months after expiration of exploration period a final report on all its findings in the contract area
(Clause 5.3-b); requires the contractor after conducting feasibility studies to submit a declaration of
mining feasibility, along with a description of the area to be developed and mined, a description of the
proposed mining operations and the technology to be employed, and the proposed work program for
the development phase, for approval by the DENR secretary (Clause 5.4); obligates the contractor to
complete the development of the mine, including construction of the production facilities, within the
period stated in the approved work program (Clause 6.1); requires the contractor to submit for
approval a work program covering each period of three fiscal years (Clause 6.2); requires the
contractor to submit reports to the secretary on the production, ore reserves, work accomplished and
work in progress, profile of its work force and management staff, and other technical information
(Clause 6.3); subjects any expansions, modifications, improvements and replacements of mining
facilities to the approval of the secretary (Clause 6.4); subjects to State control the amount of funds
that the contractor may borrow within the Philippines (Clause 7.2); subjects to State supervisory
power any technical, financial and marketing issues (Clause 10.1-a); obligates the contractor to
ensure 60 percent Filipino equity in the contractor within ten years of recovering specified
expenditures unless not so required by subsequent legislation (Clause 10.1); gives the State the right
to terminate the FTAA for unremedied substantial breach thereof by the contractor (Clause 13.2);
requires State approval for any assignment of the FTAA by the contractor to an entity other than an
affiliate (Clause 14.1).

In short, the aforementioned provisions of the WMCP FTAA, far from constituting a surrender of
control and a grant of beneficial ownership of mineral resources to the contractor in question, vest the
State with control and supervision over practically all aspects of the operations of the FTAA
contractor, including the charging of pre-operating and operating expenses, and the disposition of
mineral products.

There is likewise no relinquishment of control on account of specific provisions of the WMCP FTAA.
Clause 8.2 provides a mechanism to prevent the mining operations from grinding to a complete halt
as a result of possible delays of more than 60 days in the government's processing and approval of
submitted work programs and budgets. Clause 8.3 seeks to provide a temporary, stop-gap solution in
case a disagreement between the State and the contractor (over the proposed work program or
budget submitted by the contractor) should result in a deadlock or impasse, to avoid unreasonably
long delays in the performance of the works.

The State, despite Clause 8.3, still has control over the contract area, and it may, as sovereign
authority, prohibit work thereon until the dispute is resolved, or it may terminate the FTAA, citing
substantial breach thereof. Hence, the State clearly retains full and effective control.

Clause 8.5, which allows the contractor to make changes to approved work programs and budgets
without the prior approval of the DENR secretary, subject to certain limitations with respect to the
variance/s, merely provides the contractor a certain amount of flexibility to meet unexpected
situations, while still guaranteeing that the approved work programs and budgets are not abandoned
altogether. And if the secretary disagrees with the actions taken by the contractor in this instance, he
may also resort to cancellation/termination of the FTAA as the ultimate sanction.

Clause 4.6 of the WMCP FTAA gives the contractor discretion to select parts of the contract area to
be relinquished. The State is not in a position to substitute its judgment for that of the contractor, who
knows exactly which portions of the contract area do not contain minerals in commercial quantities
and should be relinquished. Also, since the annual occupation fees paid to government are based on
163

the total hectarage of the contract area, net of the areas relinquished, the contractor's self-interest will
assure proper and efficient relinquishment.

Clause 10.2(e) of the WMCP FTAA does not mean that the contractor can compel government to use
its power of eminent domain. It contemplates a situation in which the contractor is a foreign-owned
corporation, hence, not qualified to own land. The contractor identifies the surface areas needed for it
to construct the infrastructure for mining operations, and the State then acquires the surface rights on
behalf of the former. The provision does not call for the exercise of the power of eminent domain (or
determination of just compensation); it seeks to avoid a violation of the anti-dummy law.

Clause 10.2(l) of the WMCP FTAA giving the contractor the right to mortgage and encumber the
mineral products extracted may have been a result of conditions imposed by creditor-banks to secure
the loan obligations of WMCP. Banks lend also upon the security of encumbrances on goods
produced, which can be easily sold and converted into cash and applied to the repayment of loans.
Thus, Clause 10.2(l) is not something out of the ordinary. Neither is it objectionable, because even
though the contractor is allowed to mortgage or encumber the mineral end-products themselves, the
contractor is not thereby relieved of its obligation to pay the government its basic and additional
shares in the net mining revenue. The contractor's ability to mortgage the minerals does not negate
the State's right to receive its share of net mining revenues.

Clause 10.2(k) which gives the contractor authority "to change its equity structure at any time," means
that WMCP, which was then 100 percent foreign owned, could permit Filipino equity ownership.
Moreover, what is important is that the contractor, regardless of its ownership, is always in a position
to render the services required under the FTAA, under the direction and control of the government.

Clauses 10.4(e) and (i) bind government to allow amendments to the FTAA if required by banks and
other financial institutions as part of the conditions of new lendings. There is nothing objectionable
here, since Clause 10.4(e) also provides that such financing arrangements should in no event reduce
the contractor's obligations or the government's rights under the FTAA. Clause 10.4(i) provides that
government shall "favourably consider" any request for amendments of this agreement necessary for
the contractor to successfully obtain financing. There is no renunciation of control, as the proviso does
not say that government shall automatically grant any such request. Also, it is up to the contractor to
prove the need for the requested changes. The government always has the final say on whether to
approve or disapprove such requests.

In fine, the FTAA provisions do not reduce or abdicate State control.

No Surrender of Financial Benefits

The second paragraph of Section 81 of RA 7942 has been denounced for allegedly limiting the
State's share in FTAAs with foreign contractors to just taxes, fees and duties, and depriving the State
of a share in the after-tax income of the enterprise. However, the inclusion of the phrase "among
other things" in the second paragraph of Section 81 clearly and unmistakably reveals the legislative
intent to have the State collect more than just the usual taxes, duties and fees.

Thus, DAO 99-56, the "Guidelines Establishing the Fiscal Regime of Financial or Technical
Assistance Agreements," spells out the financial benefits government will receive from an FTAA, as
consisting of not only a basic government share, comprised of all direct taxes, fees and royalties, as
well as other payments made by the contractor during the term of the FTAA, but also an additional
government share, being a share in the earnings or cash flows of the mining enterprise, so as
to achieve a fifty-fifty sharing of net benefits from mining between the government and the contractor.

The additional government share is computed using one of three (3) options or schemes detailed in
DAO 99-56, viz., (1) the fifty-fifty sharing of cumulative present value of cash flows; (2) the excess
profit-related additional government share; and (3) the additional sharing based on the cumulative net
mining revenue. Whichever option or computation is used, the additional government share has
nothing to do with taxes, duties, fees or charges. The portion of revenues remaining after the
deduction of the basic and additional government shares is what goes to the contractor.
164

The basic government share and the additional government share do not yet take into account the
indirect taxes and other financial contributions of mining projects, which are real and actual benefits
enjoyed by the Filipino people; if these are taken into account, total government share increases to 60
percent or higher (as much as 77 percent, and 89 percent in one instance) of the net present value of
total benefits from the project.

The third or last paragraph of Section 81 of RA 7942 is slammed for deferring the payment of the
government share in FTAAs until after the contractor shall have recovered its pre-operating expenses,
exploration and development expenditures. Allegedly, the collection of the State's share is rendered
uncertain, as there is no time limit in RA 7942 for this grace period or recovery period. But although
RA 7942 did not limit the grace period, the concerned agencies (DENR and MGB) in formulating the
1995 and 1996 Implementing Rules and Regulations provided that the period of recovery, reckoned
from the date of commercial operation, shall be for a period not exceeding five years, or until the date
of actual recovery, whichever comes earlier.

And since RA 7942 allegedly does not require government approval for the pre-operating, exploration
and development expenses of the foreign contractors, it is feared that such expenses could be
bloated to wipe out mining revenues anticipated for 10 years, with the result that the State's share is
zero for the first 10 years. However, the argument is based on incorrect information.

Under Section 23 of RA 7942, the applicant for exploration permit is required to submit a proposed
work program for exploration, containing a yearly budget of proposed expenditures, which the State
passes upon and either approves or rejects; if approved, the same will subsequently be recorded as
pre-operating expenses that the contractor will have to recoup over the grace period.

Under Section 24, when an exploration permittee files with the MGB a declaration of mining project
feasibility, it must submit a work program for development, with corresponding budget, for approval by
the Bureau, before government may grant an FTAA or MPSA or other mineral agreements; again,
government has the opportunity to approve or reject the proposed work program and budgeted
expenditures for development works, which will become the pre-operating and development costs that
will have to be recovered. Government is able to know ahead of time the amounts of pre-operating
and other expenses to be recovered, and the approximate period of time needed therefor. The
aforecited provisions have counterparts in Section 35, which deals with the terms and conditions
exclusively applicable to FTAAs. In sum, the third or last paragraph of Section 81 of RA 7942 cannot
be deemed defective.

Section 80 of RA 7942 allegedly limits the State's share in a mineral production-sharing agreement
(MPSA) to just the excise tax on the mineral product, i.e., only 2 percent of market value of the
minerals. The colatilla in Section 84 reiterates the same limitation in Section 80. However, these two
provisions pertain only to MPSAs, and have no application to FTAAs. These particular
provisions do not come within the issues defined by this Court. Hence, on due process
grounds, no pronouncement can be made in this case in respect of the constitutionality of
Sections 80 and 84.

Section 112 is disparaged for reverting FTAAs and all mineral agreements to the old "license,
concession or lease" system, because it allegedly effectively reduces the government share in FTAAs
to just the 2 percent excise tax which pursuant to Section 80 comprises the government share in
MPSAs. However, Section 112 likewise does not come within the issues delineated by this Court, and
was never touched upon by the parties in their pleadings. Moreover, Section 112 may not properly
apply to FTAAs. The mining law obviously meant to treat FTAAs as a breed apart from mineral
agreements. There is absolutely no basis to believe that the law intends to exact from FTAA
contractors merely the same government share (i.e., the 2 percent excise tax) that it apparently
demands from contractors under the three forms of mineral agreements.

While there is ground to believe that Sections 80, 84 and 112 are indeed unconstitutional, they cannot
be ruled upon here. In any event, they are separable; thus, a later finding of nullity will not affect the
rest of RA 7942.
165

In fine, the challenged provisions of RA 7942 cannot be said to surrender financial benefits
from an FTAA to the foreign contractors.

Moreover, there is no concrete basis for the view that, in FTAAs with a foreign contractor, the State
must receive at least 60 percent of the after-tax income from the exploitation of its mineral resources,
and that such share is the equivalent of the constitutional requirement that at least 60 percent of the
capital, and hence 60 percent of the income, of mining companies should remain in Filipino hands.
Even if the State is entitled to a 60 percent share from other mineral agreements (CPA, JVA and
MPSA), that would not create a parallel or analogous situation for FTAAs. We are dealing with an
essentially different equation. Here we have the old apples and oranges syndrome.

The Charter did not intend to fix an iron-clad rule of 60 percent share, applicable to all situations,
regardless of circumstances. There is no indication of such an intention on the part of the framers.
Moreover, the terms and conditions of petroleum FTAAs cannot serve as standards for mineral mining
FTAAs, because the technical and operational requirements, cost structures and investment
needs of off-shore petroleum exploration and drilling companies do not have the remotest
resemblance to those of on-shore mining companies.

To take the position that government's share must be not less than 60 percent of after-tax income of
FTAA contractors is nothing short of this Court dictating upon the government. The State resultantly
ends up losing control. To avoid compromising the State's full control and supervision over the
exploitation of mineral resources, there must be no attempt to impose a "minimum 60 percent" rule. It
is sufficient that the State has the power and means, should it so decide, to get a 60 percent share (or
greater); and it is not necessary that the State does so in every case.

Invalid Provisions of the WMCP FTAA

Section 7.9 of the WMCP FTAA clearly renders illusory the State's 60 percent share of WMCP's
revenues. Under Section 7.9, should WMCP's foreign stockholders (who originally owned 100 percent
of the equity) sell 60 percent or more of their equity to a Filipino citizen or corporation, the State loses
its right to receive its share in net mining revenues under Section 7.7, without any offsetting
compensation to the State. And what is given to the State in Section 7.7 is by mere tolerance of
WMCP's foreign stockholders, who can at any time cut off the government's entire share by simply
selling 60 percent of WMCP's equity to a Philippine citizen or corporation.

In fact, the sale by WMCP's foreign stockholder on January 23, 2001 of the entire outstanding equity
in WMCP to Sagittarius Mines, Inc., a domestic corporation at least 60 percent Filipino owned, can be
deemed to have automatically triggered the operation of Section 7.9 and removed the State's right to
receive its 60 percent share. Section 7.9 of the WMCP FTAA has effectively given away the State's
share without anything in exchange.

Moreover, it constitutes unjust enrichment on the part of the local and foreign stockholders in WMCP,
because by the mere act of divestment, the local and foreign stockholders get a windfall, as their
share in the net mining revenues of WMCP is automatically increased, without having to pay anything
for it.

Being grossly disadvantageous to government and detrimental to the Filipino people, as well as
violative of public policy, Section 7.9 must therefore be stricken off as invalid. The FTAA in question
does not involve mere contractual rights but, being impressed as it is with public interest, the
contractual provisions and stipulations must yield to the common good and the national interest. Since
the offending provision is very much separable from the rest of the FTAA, the deletion of Section 7.9
can be done without affecting or requiring the invalidation of the entire WMCP FTAA itself.

Section 7.8(e) of the WMCP FTAA likewise is invalid, since by allowing the sums spent by
government for the benefit of the contractor to be deductible from the State's share in net mining
revenues, it results in benefiting the contractor twice over. This constitutes unjust enrichment on the
part of the contractor, at the expense of government. For being grossly disadvantageous and
166

prejudicial to government and contrary to public policy, Section 7.8(e) must also be declared without
effect. It may likewise be stricken off without affecting the rest of the FTAA.

EPILOGUE

AFTER ALL IS SAID AND DONE, it is clear that there is unanimous agreement in the Court upon the
key principle that the State must exercise full control and supervision over the exploration,
development and utilization of mineral resources.

The crux of the controversy is the amount of discretion to be accorded the Executive Department,
particularly the President of the Republic, in respect of negotiations over the terms of FTAAs,
particularly when it comes to the government share of financial benefits from FTAAs. The Court
believes that it is not unconstitutional to allow a wide degree of discretion to the Chief Executive,
given the nature and complexity of such agreements, the humongous amounts of capital and
financing required for large-scale mining operations, the complicated technology needed, and the
intricacies of international trade, coupled with the State's need to maintain flexibility in its dealings, in
order to preserve and enhance our country's competitiveness in world markets.

We are all, in one way or another, sorely affected by the recently reported scandals involving
corruption in high places, duplicity in the negotiation of multi-billion peso government contracts, huge
payoffs to government officials, and other malfeasances; and perhaps, there is the desire to see some
measures put in place to prevent further abuse. However, dictating upon the President what
minimum share to get from an FTAA is not the solution.It sets a bad precedent since such a
move institutionalizes the very reduction if not deprivation of the State's control. The remedy may be
worse than the problem it was meant to address. In any event, provisions in such future agreements
which may be suspected to be grossly disadvantageous or detrimental to government may be
challenged in court, and the culprits haled before the bar of justice.

Verily, under the doctrine of separation of powers and due respect for co-equal and coordinate
branches of government, this Court must restrain itself from intruding into policy matters and must
allow the President and Congress maximum discretion in using the resources of our country and in
securing the assistance of foreign groups to eradicate the grinding poverty of our people and answer
their cry for viable employment opportunities in the country.

"The judiciary is loath to interfere with the due exercise by coequal branches of government of their
official functions."99 As aptly spelled out seven decades ago by Justice George Malcolm, "Just as the
Supreme Court, as the guardian of constitutional rights, should not sanction usurpations by any other
department of government, so should it as strictly confine its own sphere of influence to the powers
expressly or by implication conferred on it by the Organic Act."100 Let the development of the mining
industry be the responsibility of the political branches of government. And let not this Court interfere
inordinately and unnecessarily.

The Constitution of the Philippines is the supreme law of the land. It is the repository of all the
aspirations and hopes of all the people. We fully sympathize with the plight of Petitioner La Bugal
B'laan and other tribal groups, and commend their efforts to uplift their communities. However, we
cannot justify the invalidation of an otherwise constitutional statute along with its implementing rules,
or the nullification of an otherwise legal and binding FTAA contract.

We must never forget that it is not only our less privileged brethren in tribal and cultural communities
who deserve the attention of this Court; rather, all parties concerned -- including the State itself, the
contractor (whether Filipino or foreign), and the vast majority of our citizens -- equally deserve the
protection of the law and of this Court. To stress, the benefits to be derived by the State from mining
activities must ultimately serve the great majority of our fellow citizens. They have as much right and
interest in the proper and well-ordered development and utilization of the country's mineral resources
as the petitioners.

Whether we consider the near term or take the longer view, we cannot overemphasize the need for
an appropriate balancing of interests and needs -- the need to develop our stagnating mining
167

industry and extract what NEDA Secretary Romulo Neri estimates is some US$840 billion (approx.
PhP47.04 trillion) worth of mineral wealth lying hidden in the ground, in order to jumpstart our
floundering economy on the one hand, and on the other, the need to enhance our nationalistic
aspirations, protect our indigenous communities, and prevent irreversible ecological damage.

This Court cannot but be mindful that any decision rendered in this case will ultimately impact not only
the cultural communities which lodged the instant Petition, and not only the larger community of the
Filipino people now struggling to survive amidst a fiscal/budgetary deficit, ever increasing prices of
fuel, food, and essential commodities and services, the shrinking value of the local currency, and a
government hamstrung in its delivery of basic services by a severe lack of resources, but also
countless future generations of Filipinos.

For this latter group of Filipinos yet to be born, their eventual access to education, health care and
basic services, their overall level of well-being, the very shape of their lives are even now being
determined and affected partly by the policies and directions being adopted and implemented by
government today. And in part by the this Resolution rendered by this Court today.

Verily, the mineral wealth and natural resources of this country are meant to benefit not merely a
select group of people living in the areas locally affected by mining activities, but the entire Filipino
nation, present and future, to whom the mineral wealth really belong. This Court has therefore
weighed carefully the rights and interests of all concerned, and decided for the greater good of the
greatest number. JUSTICE FOR ALL, not just for some; JUSTICE FOR THE PRESENT AND THE
FUTURE, not just for the here and now.

WHEREFORE, the Court RESOLVES to GRANT the respondents' and the intervenors' Motions for
Reconsideration; to REVERSE and SET ASIDE this Court's January 27, 2004 Decision;
to DISMISS the Petition; and to issue this new judgment declaring CONSTITUTIONAL (1) Republic
Act No. 7942 (the Philippine Mining Law), (2) its Implementing Rules and Regulations contained in
DENR Administrative Order (DAO) No. 9640 -- insofar as they relate to financial and technical
assistance agreements referred to in paragraph 4 of Section 2 of Article XII of the Constitution; and
(3) the Financial and Technical Assistance Agreement (FTAA) dated March 30, 1995 executed by the
government and Western Mining Corporation Philippines Inc. (WMCP), except Sections 7.8 and 7.9 of
the subject FTAA which are hereby INVALIDATED for being contrary to public policy and for being
grossly disadvantageous to the government.

SO ORDERED.

Davide Jr., C.J., Sandoval-Gutierrez, Austria-Martinez, and Garcia, JJ., concur.


Puno, J., in the result and votes to invalidate sections 3.3; 7.8 and 7.9 of the WMC FTAA.
Quisumbing, J., in the result.
Ynares-Santiago, J., joins dissenting opinion of J. Antonio Carpio & J. Conchita C. Morales.
Carpio, and Carpio-Morales, JJ., see dissenting opinion.
Corona, J., certifies he voted affirmatively with the majority and he was allowed to do so although he
is on leave.
Callejo, Sr., J., concurs to the dissenting opinion of J. Carpio.
Azcuna, J., took no part-same reason.
Tinga, and Chico-Nazario, JJ., concur with a separate opinion.

CONCURRING OPINION

CHICO-NAZARIO, J.:

I concur in the well-reasoned ponencia of my esteemed colleague Mr. Justice Artemio V. Panganiban.
I feel obligated, however, to add the following observations:
168

I. RE "FULL CONTROL AND SUPERVISION"

With all due respect, I believe that the issue of unconstitutionality of Republic Act No. 7942, its
implementing rules, and the Financial Assistance Agreement between the Philippine Government and
WMPC (Philippines) Inc. (WMPC FTAA) executed pursuant to Rep. Act No. 7942 hinges, to a large
extent, on the interpretation of the phrase in Section 2, Article XII of the 1987 Constitution, which
states:

(T)he exploration, development, and utilization of natural resources shall be under the full
control and supervision of the State. x x x. (Emphasis supplied)

Construing said phrase vis-à-vis the entire provision, it appears from the deliberations in the
Constitutional Commission that the term "control" does not have the meaning it ordinarily has in
political law which is the power of a superior to substitute his judgment for that of an inferior.1 Thus –

MR. NOLLEDO: Suppose a judicial entity is given the power to exploit natural resources and,
of course, there are decisions made by the governing board of that judicial entity, can the
state change the decisions of the governing board of that entity based on the words "full
control".

MR. VILLEGAS: If it is within the context of the contract, I think the State cannot violate the
laws of the land.2

Moreover, "full control and supervision" does not mean that foreign stockholders cannot be legally
elected as members of the board of a corporation doing business under, say, a co-production, joint
venture or profit-sharing agreement, 40% of whose capital is foreign owned. Otherwise, and as
Commissioner Romulo declared, it would be unfair to the foreign stockholder 3 and, per Commissioner
Padilla, "refusing them a voice in management would make a co-production, joint venture and
production sharing illusory."4

It is apparently for the foregoing reasons that there was a disapproval of the amendment proposed by
Commissioner, now Mr. Chief Justice Davide, that the governing and managing bodies of such
corporations shall be vested exclusively in citizens of the Philippines 5 so that control of all
corporations involved in the business of utilizing our natural resources would always be in Filipino
hands.

The disapproval must be juxtaposed with the fact that a provision substantially similar to the proposed
Davide amendment was approved with regard to educational institutions, viz:

Section 4 (2). Educational institutions, other than those established by religious groups and
mission boards, shall be owned solely by citizens of the Philippines or corporations or
associations at least sixty per centum of the capital of which is owned by such citizens. The
Congress may, however, require increased Filipino equity participation in all educational
institutions.

The control and administration of educational institutions shall be vested in citizens of the
Philippines. (Emphasis supplied)

From the foregoing, it can be clearly inferred that it was NOT the intention of the framers of the
Constitution to deprive governing boards of domestic corporations with non-Filipino members, the
right to control and administer the corporation that explores, develops and utilizes natural resources
insofar as agreements with the State for co-production, joint venture and production-sharing are
concerned, otherwise the Davide amendment would have been approved and, like the prohibition in
above-quoted Section 4(2), Article XIV, control and supervision of all business involved in the
exploration and development of mineral resources would have been left solely in Filipino hands.
169

Accordingly, to the extent that the corporate board governs and manages the operations for the
exploration and use of natural resources, to that extent the "full control and supervision" thereof by the
State is diminished.

In effect, therefore, when the State enters into such agreements as provided in the Constitution, it
allows itself to surrender part of its sovereign right to full control and supervision of said activities, the
State having the right to partly surrender the exercise of sovereign powers under the doctrine of auto-
limitation.6

If foreigners (under joint ventures etc.) have a say in the management of the business of utilizing
natural resources as corporate directors of domestic corporations, there is no justification for holding
that foreign corporations who put in considerably large amounts of money under agreements involving
either technical or financial assistance for large scale exploration, development and utilization of
minerals, petroleum and other mineral oils are prohibited from managing such business.

Indeed, to say that the Constitution requires the State to have full and total control and supervision of
the exploration, development and utilization of minerals when undertaken in a large scale under
agreements with foreign corporations involving huge amounts of money is to divorce oneself from
reality. As Mr. Justice Panganiban said, no firm would invest funds in such enterprise unless it has a
say in the management of the business.

To paraphrase this Court in one of its landmark cases, the fundamental law does not intend an
impossible undertaking.7 It must therefore be presumed that the Constitution did not at all intend an
interpretation of Section 2, Article XII which deprives the foreign corporation engaged in large scale
mining activities a measure of control in the management and operation of such activities, and in said
manner, remove from the realm of the possible the enterprise the Constitution envisions thereunder.

This brings me to the final point raised by my esteemed colleague, Mme. Justice Conchita Carpio
Morales, that it is of no moment that the declaration of Rep. Act No. 7942 may discourage foreign
assistance and/or retard or delay the exploration, development or utilization of the nation's natural
resources as the Filipino people, as early as the 1935 Constitution, have determined such matters as
secondary to the protection and preservation of their ownership of these natural resources. With due
respect, I find such proposition not legally justifiable as it looks backward to the justification in the
1935 Constitution instead of forward under the 1987 Constitution which expressly allows foreign
participation in the exploration, development or utilization of the nation's marine wealth to allow the
State to take advantage of foreign funding or technical assistance. As long as the means employed by
such foreign assistance result in real contributions to the economic growth of our country and
enhance the general welfare of our people, the development of our mineral resources by and through
foreign corporations, such FTAAs are not unconstitutional.

II. RE: REQUIREMENT THAT FTAAs MUST BE "BASED


ON REAL CONTRIBUTIONS TO THE ECONOMIC GROWTH
AND GENERAL WELFARE OF THE COUNTRY"

The policy behind Rep. Act No. 7942 is to promote the "rational exploration, development, utilization
and conservation" of the State-owned mineral resources "through the combined efforts of government
and the private sector in order to enhance national growth in a way that effectively safe-guards the
environment and protect the rights of affected communities".8 This policy, with reference specifically to
FTAAs, is in keeping with the constitutional precept that FTAAs must be based on real contributions
to the economic growth and general welfare of the country. As has been said, "a statute derives its
vitality from the purpose for which it is enacted and to construe it in a manner that disregards or
defeats such purpose is to nullify or destroy the law."9 In this regard, much has been said about the
alleged unconstitutionality of Section 81 of Rep. Act No. 7942 as it allegedly allows for the waiver of
the State's right to receive income from the exploitation of its mineral resources as it limits the State's
share in FTAAs with foreign contractors to taxes, duties and fees. For clarity, the provision states –

SEC. 81. Government Share in Other Mineral Agreements. -- The share of the Government in
co-production and joint-venture agreements shall be negotiated by the Government and the
170

contractor taking into consideration the: (a) capital investment of the project, (b) risks
involved, (c) contribution of the project to the economy, and (d) other factors that will provide
for a fair and equitable sharing between the Government and the contractor. The Government
shall also be entitled to compensations for its other contributions which shall be agreed upon
by the parties, and shall consist, among other things, the contractor's income tax, excise tax,
special allowance, withholding tax due from the contractor's foreign stockholders, arising from
dividend or interest payments to the said foreign stockholders, in case of a foreign national,
and all such other taxes, duties and fees as provided for under existing laws.

The Government share in financial or technical assistance agreement shall consist of, among
other things,the contractor's corporate income tax, excise tax, special allowance, withholding
tax due from the contractor's foreign stockholders arising from dividend or interest payments
to the said foreign stockholder in case of foreign national and all such other taxes, duties
and fees as provided for under existing laws.

The collection of Government share in financial or technical assistance agreement shall


commence after the financial or technical assistance agreement contractor has fully
recovered its pre-operating expenses, exploration, and development expenditures, inclusive.
(Emphasis supplied)

The controversy revolves around the proper interpretation of "among other things" stated in the
second paragraph of Section 81. Mr. Justice Carpio is of the opinion that "among other things" could
only mean "among other taxes", referring to the unnamed "other taxes, duties, and fees as provided
for under existing laws" contained in the last clause of Section 81, paragraph 2. If such were the
correct interpretation, then truly, the provision is unconstitutional as a sharing based only on taxes
cannot be considered as contributing to the economic growth and general welfare of the country. I am
bothered, however, by the interpretation that the phrase "among other things" refers to "and all such
other taxes, duties and fees as provided for under existing laws" since it would render the former
phrase superfluous. In other words, there would have been no need to include the phrase "among
other things" if all it means is "all other taxes" since the latter is already expressly stated in the
provision. As it is a truism that all terms/phrases used in a statute has relevance to the object of the
law, then I find the view of Mr. Justice Panganiban – that "all other things" means "additional
government share" in the form of "earnings or cash flow of the mining enterprise" as interpreted by the
DENR -- more compelling. Besides, such an interpretation would affirm the constitutionality of the
provision which would then be in keeping with the rudimentary principle that a law shall not be
declared invalid unless the conflict with the Constitution is clear beyond reasonable doubt. 10 To justify
nullification of a law, there must be a clear and unequivocal breach of the Constitution, not a doubtful
and argumentative implication.11

Finally, I wish to stress that it would appear that the constitutional mandate that large-scale mining
activities under FTAAs must be based on real contributions to the economic growth and general
welfare of the country is both a standard for the statute required to implement subject provision as
well as the vehicle for the exercise of the State's resultant residual control and supervision of the
mining activities.

In all FTAAs, the State is deemed to reserve its right to control the end to be achieved so that real
contributions to the economy can be realized and, in the final analysis, the business will redound to
the general welfare of the country.

However, the question of whether or not the FTAA will, in fact, redound to the general welfare of the
public involves a "judgment call" by our policy makers who are answerable to our people during the
appropriate electoral exercises and are not subject to judicial pronouncements based on grave abuse
of discretion.12

For the foregoing reasons, I vote to grant the motion for reconsideration.
171

DISSENTING OPINION

CARPIO, J.:

I dissent and vote to deny respondents' motions for reconsideration. I find that Section 3(aq), Section
39, Section 80, the second paragraph of Section 81, the proviso in Section 84, and the first proviso in
Section 112 of Republic Act No. 79421 ("RA 7942") violate Section 2, Article XII of the 1987
Constitution and are therefore unconstitutional.

In essence, these provisions of RA 7942 waive the State's ownership rights under the
Constitution over mineral resources. These provisions also abdicate the State's constitutional
duty to control and supervise fully the exploitation of mineral resources.

A. The Threshold Issue for Resolution

Petitioners claim that respondent Department of Environment and Natural Resources Secretary Victor
O. Ramos, in issuing the rules to implement RA 7942, gravely abused his discretion amounting to lack
or excess of jurisdiction. Petitioners assert that RA 7942 is unconstitutional for the following reasons:

1. RA 7942 "allows fully foreign owned corporations to explore, develop, utilize and exploit
mineral resources in a manner contrary to Section 2, paragraph 4, Article XII of the
Constitution";

2. RA 7942 "allows enjoyment by foreign citizens as well as fully foreign owned corporations
of the nation's marine wealth contrary to Section 2, paragraph 2 of Article XII of the
Constitution";

3. RA 7942 "violates Section 1, Article III of the Constitution";

4. RA 7942 "allows priority to foreign and fully foreign owned corporations in the exploration,
development and utilization of mineral resources contrary to Article XII of the Constitution";

5. RA 7942 "allows the inequitable sharing of wealth contrary to Section 1, paragraph 1,


and Section 2, paragraph 4, Article XII of the Constitution."2 (Emphasis supplied)

Petitioners also assail the validity of the Financial and Technical Assistance Agreement between the
Philippine Government and WMCP (Philippines), Inc. dated 2 March 19953 ("WMCP FTAA") for
violation of Section 2, Article XII of the 1987 Constitution.

The issues that petitioners raise boil down to whether RA 7942 and the WMCP FTAA violate
Section 2, Article XII of the 1987 Constitution.

B. The Constitutional Declaration and Mandate

Section 2, Article XII of the 1987 Constitution4 provides as follows:

All x x x minerals, x x x petroleum, and other mineral oils, x x x and other natural resources
are owned by the State. x x x The exploration, development, and utilization of natural
resources shall be under the full control and supervision of the State. x x x. (Emphasis
supplied)

Two basic principles flow from this constitutional provision. First, the Constitution vests in the State
ownership of all mineral resources. Second, the Constitution mandates the State to exercise full
control and supervisionover the exploitation of mineral resources.

The first principle reiterates the Regalian doctrine, which established State ownership of natural
resources since the arrival of the Spaniards in the Philippines in the 16 th century. The 1935, 1973 and
172

1987 Constitutions incorporate the Regalian doctrine.5 The State, as owner of the nation's natural
resources, exercises the attributes of ownership over its natural resources. 6 An important attribute of
ownership is the right to receive the income from any commercial exploitation of the natural
resources.7

The second principle insures that the benefits of State ownership of natural resources accrue to the
Filipino people. The framers of the 1987 Constitution introduced the second principle to avoid the
adverse effects of the "license, concession or lease"8 system of exploitation under the 1935 and 1973
Constitutions.9 The "license, concession or lease" system enriched the private concessionaires who
controlled the exploitation of natural resources. However, the "license, concession or lease" system
left the Filipino people impoverished, starkly exemplified by the nation's denuded forests whose
exploitation did not benefit the Filipino people.

The framers of the 1987 Constitution clearly intended to abandon the "license, concession or lease"
system prevailing under the 1935 and 1973 Constitutions. This exchange in the deliberations of the
Constitutional Commission reveals this clear intent:

MR. DAVIDE: Thank you, Mr. Vice-President. I would like to seek some clarifications.

MR. VILLEGAS: Yes.

MR. DAVIDE: Under the proposal, I notice that except for the lands of the public domain, all
the other natural resources cannot be alienated and in respect to lands of the public domain,
private corporations with the required ownership by Filipino citizens can only lease the
same. Necessarily, insofar as other natural resources are concerned, it would only be
the State which can exploit, develop, explore and utilize the same. However, the State
may enter into a joint venture, co-production or production-sharing. Is that not
correct?

MR. VILLEGAS: Yes.

MR. DAVIDE: Consequently, henceforth upon the approval of this Constitution, no


timber or forest concessions, permits or authorization can be exclusively granted to
any citizen of the Philippines nor to any corporation qualified to acquire lands of the
public domain?

MR. VILLEGAS: Would Commissioner Monsod like to comment on that? I think his answer is
"yes."

MR. DAVIDE: So, what will happen now to licenses or concessions earlier granted by the
Philippine government to private corporations or to Filipino citizens? Would they be deemed
repealed?

MR. VILLEGAS: This is not applied retroactively. They will be respected. 10 (Emphasis
supplied)

To carry out this intent, the 1987 Constitution uses a different phraseology from that used in the 1935
and 1973 Constitutions. The previous Constitutions used the phrase "license, concession or lease" in
referring to exploitation of natural resources. The 1987 Constitution uses the phrase "co-production,
joint venture or production-sharing agreements," with "full control and supervision" by the State. The
change in language was a clear rejection of the old system of "license, concession or lease."

The 1935 and 1973 Constitutions also used the words "belong to" in stating the Regalian doctrine,
thus declaring that natural resources "belong to the State." The 1987 Constitution uses the word
"owned," thus prescribing that natural resources are "owned" by the State. In using the word "owned,"
the 1987 Constitution emphasizes the attributes of ownership, among which is the right to the income
of the property owned.11
173

The State as owner of the natural resources must receive income from the exploitation of its natural
resources. The payment of taxes, fees and charges, derived from the taxing or police power of
the State, is not a substitute.The State is duty bound to secure for the Filipino people a fair share of
the income from any exploitation of the nation's precious and exhaustible natural resources. As
explained succinctly by a textbook writer:

Under the former licensing, concession, or lease schemes, the government benefited from
such activities only through fees, charges and taxes. Such benefits were very minimal
compared with the enormous profits reaped by the licensees, concessionaires or lessees who
had control over the particular resources over which they had been given exclusive right to
exploit. Moreover, some of them disregarded the conservation of natural resources. With the
new role, the State will be able to obtain a greater share in the profits. It can also actively
husband our natural resources and engage in development programs that will be beneficial to
the nation.12 (Emphasis supplied)

Thus, the 1987 Constitution commands the State to exercise full control and supervision over the
exploitation of natural resources to insure that the State receives its fair share of the income.
In Miners Association of the Philippines v. Hon. Factoran, Jr., et al.,13 the Court ruled that "the
old system of exploration, development and utilization of natural resources through 'license,
concession or lease' x x x has been disallowed by Article XII, Section 2 of the 1987
Constitution." The Court explained:

Upon the effectivity of the 1987 Constitution on February 2, 1987, the State assumed a
more dynamic role in the exploration, development and utilization of the natural
resources of the country. Article XII, Section 2 of the said Charter explicitly ordains that the
exploration, development and utilization of natural resources shall be under the full control
and supervision of the State. Consonant therewith, the exploration, development and
utilization of natural resources may be undertaken by means of direct act of the State, or it
may opt to enter into co-production, joint venture, or production-sharing agreements, or it may
enter into agreements with foreign-owned corporations involving either technical or financial
assistance for large-scale exploration, development, and utilization of minerals, petroleum,
and other mineral oils according to the general terms and conditions provided by law, based
on real contributions to the economic growth and general welfare of the country.
(Emphasis supplied)

The old system of "license, concession or lease" which merely gave the State a pittance in the form of
taxes, fees and charges is now buried in history. Any attempt to resurrect it is unconstitutional and
deserves outright rejection by this Court.

The Constitution prohibits the alienation of all natural resources except agricultural lands.14 The
Constitution, however, allows the State to exploit commercially its natural resources and sell the
marketable products from such exploitation. This the State may do through a co-production, joint
venture or production-sharing arrangement with companies at least 60% Filipino owned. The
necessary implication is that the State, as owner of the natural resources, must receive a fair share
of the income from such commercial operation. The State may receive its share of the net income in
cash or in kind.

The State may also directly exploit its natural resources in either of two ways. The State may set up
its own company to engage in the exploitation of natural resources. Alternatively, the State may enter
into a financial or technical assistance agreement ("FTAA") with private companies who act as
contractors of the State. The State may seek from such contractors either financial or technical
assistance, or both, depending on the State's own needs. Under an FTAA, the contractor, foreign or
local, manages the contracted work or operations to the extent of its financial or technical contribution,
subject to the State's control and supervision.

Except in large-scale exploitation of certain minerals, the State's contractors must be 60% Filipino
owned companies. The State pays such contractors, for their technical services or financial
assistance, a share of the income from the exploitation of the natural resources. The State retains the
remainder of the income after paying the Filipino owned contractor.
174

In large-scale exploitation of minerals, petroleum and other mineral oils, the Constitution allows the
State to contract with "foreign-owned corporations" under an FTAA. This is still a direct
exploitation by the State but using a foreign instead of a local contractor. However, the Constitution
requires that the participation of foreign contractors must make a real contribution to the national
economy and the general welfare. The State pays the foreign contractor, for its technical services or
financial assistance, a share of the income from the exploitation of the minerals, petroleum or other
mineral oils. The State retains the rest of the income after paying the foreign contractor.

Whether the FTAA contractor is local or foreign, the State must retain its fair share of the income from
the exploitation of the natural resources that it owns. To insure it retains its fair share of the income,
the State must exercise full control and supervision over the exploitation of its natural resources. And
whether the FTAA contractor is local or foreign, the State is directly undertaking the exploitation of
its natural resources, with the FTAA contractor providing technical services or financing to the State.
Since the State is directly undertaking the exploitation, all exploration permits and similar
authorizations are in the name of the Philippine Government, which then authorizes the
contractor to act on its behalf.

The State exercises full control and supervision over the mining operations in the Philippines of the
foreign contractor. However, the State does not exercise control and supervision over the foreign
contractor itself or its board of directors. The State does not also exercise any control or supervision
over the foreign contractor's mining operations in other countries, or even its non-mining operations in
the Philippines. There is no conflict of power between the State and the foreign contractor's board of
directors. By entering into an FTAA, the foreign contractor, through its board of directors, agrees to
manage the contracted work or operations to the extent of its financial or technical contribution
subject to the State's control and supervision.

No government should contract with a corporation, local or foreign, to exploit commercially the
nation's natural resources without the State receiving any income as owner of the natural resources.
Natural resources are non-renewable and exhaustible assets of the State. Certainly, no government
in its right mind should give away for free its natural resources to private business enterprises, local or
foreign, amidst widespread poverty among its people.

In sum, two basic constitutional principles govern the exploitation of natural resources in the country.
First, the State owns the country's natural resources and must benefit as owner from any exploitation
of its natural resources. Second, to insure that it receives its fair share as owner of the natural
resources, the State must exercise full control and supervision over the exploitation of its natural
resources.

We shall subject RA 7942 to constitutional scrutiny based on these two basic principles.

C. Waiver of Beneficial Rights from Ownership of Mineral Resources

RA 7942 contains five provisions which waive the State's right to receive income from the
exploitation of its mineral resources. These provisions are Sections 39, 80, 81, 84 and 112:

Section 39. Option to Convert into a Mineral Agreement. — The contractor has the option
to convert the financial or technical assistance agreement to a mineral agreement at
any time during the term of the agreement, if the economic viability of the contract area
is found to be inadequate to justify large-scale mining operations, after proper notice to
the Secretary as provided for under the implementing rules and regulations: Provided, That
the mineral agreement shall only be for the remaining period of the original agreement.

In the case of a foreign contractor, it shall reduce its equity to forty percent (40%) in the
corporation, partnership, association, or cooperative. Upon compliance with this
requirement by the contractor, the Secretary shall approve the conversion and execute
the mineral production-sharing agreement.
175

Section 80. Government Share in Mineral Production Sharing Agreement. — The total
government share in a mineral production sharing agreement shall be the excise tax
on mineral products as provided in Republic Act No. 7729, amending Section 151(a) of the
National Internal Revenue Code, as amended.

Section 81. Government Share in Other Mineral Agreements. — The share of the
Government in co-production and joint-venture agreements shall be negotiated by the
Government and the contractor taking into consideration the: (a) capital investment of the
project, (b) risks involved, (c) contribution of the project to the economy, and (d) other factors
that will provide for a fair and equitable sharing between the Government and the contractor.
The Government shall also be entitled to compensation for its other contributions which shall
be agreed upon by the parties, and shall consist, among other things, the contractor's income
tax, excise tax, special allowance, withholding tax due from the contractor's foreign
stockholders arising from dividend or interest payments to the said foreign stockholders, in
case of a foreign national, and all such other taxes, duties and fees as provided for under
existing laws.

The Government share in financial or technical assistance agreement shall consist of,
among other things, the contractor's corporate income tax, excise tax, special
allowance, withholding tax due from the contractor's foreign stockholders arising from
dividend or interest payments to the said foreign stockholder in case of a foreign
national and all such other taxes, duties and fees as provided for under existing laws.

The collection of Government share in financial or technical assistance agreement


shall commence after the financial or technical assistance agreement contractor has
fully recovered its pre-operating expenses, exploration, and development
expenditures, inclusive.

Section 84. Excise Tax on Mineral Products. — The contractor shall be liable to pay the
excise tax on mineral products as provided for under Section 151 of the National Internal
Revenue Code: Provided, however, That with respect to a mineral production sharing
agreement, the excise tax on mineral products shall be the government share under
said agreement.

Section 112. Non-impairment of Existing Mining/Quarrying Rights. - All valid and existing
mining lease contracts, permits/licenses, leases pending renewal, mineral production–
sharing agreements granted under Executive Order No. 279, at the date of effectivity of this
Act, shall remain valid x x x Provided, That the provisions of Chapter XIV15 on
government share in mineral production-sharing agreement x x x shall immediately
govern and apply to a mining lessee or contractor unless the mining lessee or contractor
indicates his intention to the Secretary, in writing, not to avail of said provisions: x x x.

(Emphasis supplied)

Section 80 of RA 7942 limits to the excise tax the State's share in a mineral production-sharing
agreement ("MPSA"). Section 80 expressly states that the excise tax on mineral products shall
constitute the "total government share in a mineral production sharing agreement." Under
Section 151(A) of the Tax Code, this excise tax on metallic and non-metallic minerals is only 2% of
the market value, as follows:

Section 151. Mineral Products. —

(A) Rates of Tax. — There shall be levied, assessed and collected on minerals, mineral
products and quarry resources, excise tax as follows:

(1) On coal and coke, a tax of Ten pesos (P10.00) per metric ton;
176

(2) On all nonmetallic minerals and quarry resources, a tax of two percent (2%) based on the
actual market value of the gross output thereof at the time of removal, in the case of those
locally extracted or produced; or the value used by the Bureau of Customs in determining
tariff and customs duties, net of excise tax and value-added tax, in the case of importation.

xxx

(3) On all metallic minerals, a tax based on the actual market value of the gross output
thereof at the time of removal, in the case of those locally extracted or produced; or the value
used by the Bureau of Customs in determining tariff and customs duties, net of excise tax and
value-added tax, in the case of importation, in accordance with the following schedule:

(a) Copper and other metallic minerals:

(i) On the first three (3) years upon the effectivity of Republic Act No. 7729, one
percent (1%);

(ii) On the fourth and the fifth years, one and a half percent (1½%); and

(iii) On the sixth year and thereafter, two percent (2%).

(b) Gold and chromite, two percent (2%).

x x x. (Emphasis supplied)

Section 80 of RA 7942 does not allow the State to receive any income as owner of the mineral
resources.The proviso in Section 84 of RA 7942 reiterates this when it states that "the excise tax on
mineral products shall be the government share under said agreement."16 The State receives
only an excise tax flowing from its taxing power, not from its ownership of the mineral resources. The
excise tax is imposed not only on mineral products, but also on alcohol, tobacco and
automobiles17 produced by companies that do not exploit natural resources owned by the State. The
excise tax is not payment for the exploitation of the State's natural resources, but payment for the
"privilege of engaging in business."18 Clearly, under Section 80 of RA 7942, the State does not
receive as owner of the mineral resources any income from the exploitation of its mineral
resources.

The second paragraph of Section 81 of RA 7942 also limits the State's share in FTAAs with foreign
contractors to taxes, duties and fees. Section 81 of RA 7942 provides that the State's share in FTAAs
with foreign contractors –

shall consist of, among other things, the contractor's corporate income tax, excise tax,
special allowance, withholding tax due from the contractor's foreign stockholders arising from
dividend or interest payments to the said foreign stockholder in case of a foreign national and
all such other taxes, duties and fees as provided for under existing laws. (Emphasis supplied)

RA 7942 does not explain the phrase "among other things." The Solicitor General states correctly that
the phrase refers to taxes.19 The phrase is an ejusdem generis phrase, and means "among other
taxes, duties and fees" since the items specifically enumerated are all taxes, duties and fees. The last
phrase "all such other taxes, duties and fees as provided for under existing laws" at the end of the
sentence clarifies further that the phrase "among other things" refers to taxes, duties and fees.

The second paragraph of Section 81 does not require the Government and the foreign FTAA
contractor to negotiate the State's share. In contrast, the first paragraph of Section 81 expressly
provides that the "share of the Government in co-production and joint-venture agreements shall be
negotiated by the Government and the contractor" which is 60% Filipino owned.
177

In a co-production or joint venture agreement, the Government contributes other inputs or equity in
addition to its mineral resources.20 Thus, the first paragraph of Section 81 requires the Government
and the 60% Filipino owned company to negotiate the State's share. However, in an FTAA with a
foreign contractor under the second paragraph of Section 81, the Government's contribution is only
the mineral resources. Section 81 does not require the Government and the foreign contractor to
negotiate the State's share from the net proceeds because there is no share for the State. Section 81
does not recognize the State's contribution of mineral resources as worthy of any share of the
net proceeds from the mining operations.

Thus, in FTAAs with foreign contractors under RA 7942, the State's share is limited to taxes,
fees and duties. The taxes include "withholding tax due from the contractor's foreign stockholders
arising from dividend or interest payments." All these taxes, fees and duties are imposed pursuant to
the State's taxing power. The tax on income, including dividend and interest income, is imposed on all
taxpayers whether or not they are stockholders of mining companies. These taxes, fees and duties
are not contractual payments to the State as owner of the mineral resources but are mandatory
exactions based on the taxing power of the State.

Section 112 of RA 7942 is another provision that violates Section 2, Article XII of the 1987
Constitution. Section 112 "immediately" reverts all mineral agreements to the old and discredited
"license, concession or lease" system outlawed by the 1987 Constitution. Section 112 states that "the
provisions of Chapter XIV21 on government share in mineral production-sharing agreement x x
x shall immediately govern and apply to a mining lessee or contractor." The contractor, local or
foreign, will now pay only the "government share in a mineral production-sharing
agreement" under RA 7942. Section 80 of RA 7942, which specifically governs MPSAs, limits
the "government share" solely to the excise tax on mineral products - 2% on metallic and non-
metallic minerals and 3% on indigenous petroleum.

In allowing the payment of the excise tax as the only share of the government in any mineral
agreement, whether co-production, joint venture or production-sharing, Section 112 of RA 7942
reinstates the old "license, concession or lease" system where the State receives only minimal taxes,
duties and fees. This clearly violates Section 2, Article XII of the Constitution and is therefore
unconstitutional. Section 112 of RA 7942 is a sweeping negation of the clear letter and intent of the
1987 Constitution that the exploitation of the State's natural resources must benefit primarily the
Filipino people.

Of course, Section 112 gives contractors the option not to avail of the benefit of Section 112. This is
in the guise that the enactment of RA 7942 shall not impair pre-existing mining rights, as the heading
of Section 112 states. It is doubtful, however, if any contractor of sound mind would refuse to receive
100% rather than only 40% of the net proceeds from the exploitation of minerals under the FTAA.

Another provision that violates Section 2, Article XII of the Constitution is Section 39 of RA 7942.
Section 39 grants the foreign contractor the option to convert the FTAA into a "mineral production-
sharing agreement" if the foreign contractor finds that the mineral deposits do not justify large-scale
mining operations. Section 39 of RA 7942 operates to deprive the State of income from the mining
operations and limits the State to the excise tax on mineral products.

Section 39 grants the foreign contractor the option to revert to the "license, concession or lease"
system which the 1987 Constitution has banned. The only requirement for the exercise of the option
is for the foreign contractor to divest 60% of its equity to a Philippine citizen or to a corporation 60%
Filipino owned. Section 39 states, "Upon compliance with this requirement by the contractor, the
Secretary shall approve the conversion and execute the mineral production-sharing
agreement." The foreign contractor only needs to give "proper notice to the Secretary as provided for
under the implementing rules and regulations" if the contractor finds the contract area not viable for
large-scale mining. Thus, Section 39 of RA 7942 is unconstitutional.

Sections 39, 80, 81, 84 and 112 of RA 7942 operate to deprive the State of the beneficial rights
arising from its ownership of mineral resources. What Section 2, Article XII of the 1987 Constitution
vests in absolute ownership to the State, Sections 80, 81, 84 and 112 of RA 7942 take away and give
for free to private business enterprises, including foreign-owned companies.
178

The legislature has discretion whether to tax a business or product. If the legislature chooses to tax a
business or product, it is free to determine the rate or amount of the tax, provided it is not
confiscatory.22 The legislature has the discretion to impose merely a 2% excise tax on mineral
products. Courts cannot inquire into the wisdom of the amount of such tax, no matter how meager it
may be. This discretion of the legislature emanates from the State's taxing power, a power vested
solely in the legislature.

However, the legislature has no power to waive for free the benefits accruing to the State from its
ownership of mineral resources. Absent considerations of social justice, the legislature has no power
to give away for free what forms part of the national patrimony of the State. Any surrender by the
legislature of the nation's mineral resources, especially to foreign private enterprises, is repugnant to
the concept of national patrimony. Mineral resources form part of the national patrimony under Article
XII (National Economy and Patrimony) of the 1987 Constitution.

Under the last paragraph of Section 81, the collection of the State's so-called "share" (consisting of
taxes) in FTAAs with foreign contractors is not even certain. This paragraph provides that the State's
"share x x x shall commence after the financial or technical assistance agreement contractor has fully
recovered its pre-operating expenses, exploration, and development expenditures." There is no time
limit in RA 7942 for this grace period when the collection of the State's "share" does not run. 23

RA 7942 itself does not require government approval for the pre-operating, exploration and
development expenses of the foreign contractor. The determination of the amount of pre-operating,
exploration and development expenses is left solely to the discretion of the foreign contractor. Nothing
prevents the foreign contractor from recording pre-operating, exploration and development expenses
equal to the mining revenues it anticipates for the first 10 years. If that happens, the State's share is
ZERO for the first 10 years.

The Government cannot tell the Filipino people when the State will start to receive its "share"
(consisting of taxes) in mining revenues under the FTAA. The Executive Department cannot correct
these deficiencies in RA 7942 through remedial implementing rules. The correction involves
substantive legislation, not merely filling in the implementing details of the law.

Taxes, fees and duties cannot constitute payment for the State's share as owner of the mineral
resources. This was the mode of payment used under the old system of "license, concession or
lease" which the 1987 Constitution abrogated. Obviously, Sections 80, 81, 84 and 112 of RA 7942
constitute an ingenious attempt to resurrect the old and discredited system, which the 1987
Constitution has now outlawed. Under the 1987 Constitution, the State must receive its fair share
as owner of the mineral resources, separate from taxes, fees and duties paid by taxpayers. The
legislature may waive taxes, fees and duties, but it cannot waive the State's share in mining
operations.

Any law waiving for free the State's right to the benefits arising from its ownership of mineral
resources is unconstitutional. Such law negates Section 2, Article XII of the 1987 Constitution vesting
ownership of mineral resources in the State. Such law will not contribute to "economic growth and the
general welfare of the country" as required in the fourth paragraph of Section 2. Thus, in waiving the
State's income from the exploitation of mineral resources, Section 80, the second paragraph of
Section 81, the proviso in Section 84, and Section 112 of RA 7942 violate the Constitution and are
therefore void.

D. Abdication of the State's Duty to Control and Supervise


Fully the Exploitation of Mineral Resources

The 1987 Constitution commands the State to exercise "full control and supervision" over the
exploitation of natural resources. The purpose of this mandatory directive is to insure that the State
receives its fair share in the exploitation of natural resources. The framers of the Constitution were
determined to avoid the disastrous mistakes of the past. Under the old system of "license, concession
or lease," the State gave full control to the concessionaires who enriched themselves while paying the
State minimal taxes, fees and charges.
179

Under the 1987 Constitution, for a co-production, joint venture or production-sharing agreement to be
valid the State must exercise full control and supervision over the mining operations. This means that
the State should approve all capital and operating expenses in the exploitation of the natural
resources. Approval of capital expenses determines how much capital is recoverable by the mining
contractor. Approval of operating expenses determines the reasonable amounts deductible from the
annual income from mining operations. Such approvals are essential because the net income from
mining operations, which is the basis of the State's share, depends on the allowable amount of capital
and operating expenses. There is approval of capital and operating expenses when the State
approves them, or if the State disapproves them and a dispute arises, when their final allowance is
subject to arbitration.

The provisions of RA 7942 on MPSAs and FTAAs do not give the State any control and supervision
over mining operations. The reason is obvious. The State's so-called "share" in a mineral production-
sharing agreement under Section 80 is limited solely to the excise tax on mineral products. This
excise tax is based on the market value of the mineral product determined without reference to the
capital or operating expenses of the mining contractor.

Likewise, the State's "share" in an FTAA under Section 81 has no relation to the capital or operating
expenses of the foreign contractor. The State's "share" constitutes the same excise tax on mineral
products, in addition to other direct and indirect taxes. The basis of the excise tax is the selling price
of the mineral product. Hence, there is no reason for the State to approve or disapprove the capital or
operating expenses of the mining contractor. Consequently, RA 7942 does not give the State any
control and supervision over mining operations contrary to the express command of the Constitution.
This makes Section 80, the second paragraph of Section 81, the proviso in Section 84, and Section
112 of RA 7942 unconstitutional.

E. RA 7942 Will Not Contribute to Economic


Growth or General Welfare of the Country

The fourth paragraph of Section 2, Article XII of the 1987 Constitution requires that FTAAs with
foreign contractors must make "real contributions to the economic growth and general welfare of
the country." Under Section 81 of RA 7942, all the net proceeds arising from the exploitation of
mineral resources accrue to the foreign contractor even if the State owns the mineral resources. The
foreign contractor will naturally repatriate the entire after-tax net proceeds to its home country.
Sections 94(a) and 94(b) of RA 7942 guarantee the foreign contractor the right to repatriate its after-
tax net proceeds, as well as its entire capital investment, after the termination of its mining operations
in the country.24

Clearly, no FTAA under Section 81 will ever make any real contribution to the growth of the economy
or to the general welfare of the country. The foreign contractor, after it ceases to operate in the
country, can even remit to its home country the scrap value of its capital equipment. Thus, the second
paragraph of Section 81 of RA 7942 is unconstitutional for failure to meet the constitutional
requirement that the FTAA with a foreign contractor should make a real contribution to the national
economy and general welfare.

F. Example of FTAA that Complies with Section 2, Article XII of the 1987 Constitution

The Solicitor General warns that declaring unconstitutional RA 7942 or its provisions will endanger the
Philippine Government's contract with the foreign contractor extracting petroleum in Malampaya,
Palawan.25 On the contrary, the FTAA with the foreign petroleum contractor meets the essential
constitutional requirements since the State receives a fair share of the income from the petroleum
operations. The State also exercises control and supervision over the exploitation of the petroleum.
The petroleum FTAA provides enough safeguards to insure that the petroleum operations will make a
real contribution to the national economy and general welfare.

The Service Contract dated 11 December 1990 between the Philippine Government as the first party,
and Occidental Philippines, Inc. and Shell Exploration B.V. as the second party26 ("Occidental-Shell
180

FTAA"), covering offshore exploitation of petroleum in Northwest Palawan, contains the following
provisions:

a. There is express recognition that the "conduct of Petroleum Operations shall be under
the full control and supervision of the Office of Energy Affairs," 27 now Department of
Energy ("DOE"), and that the "CONTRACTOR shall undertake and execute the Petroleum
Operations contemplated hereunder under the full control and supervision of the
OFFICE OF ENERGY AFFAIRS;"28

b. The State receives 60% of the net proceeds from the petroleum operations, while the
foreign contractor receives the remaining 40%;29

c. The DOE has a right to inspect and audit every year the foreign contractor's books and
accounts relating to the petroleum operations, and object in writing to any expense
(operating and capital expenses)30within 60 days from completion of the audit, and if
there is no amicable settlement, the dispute goes to arbitration; 31

d. The operating expenses in any year cannot exceed 70% of the gross proceeds from the
sale of petroleum in the same year, and any excess may be carried over in succeeding
years;32

e. The Bureau of Internal Revenue ("BIR") can inspect and examine all the accounts, books
and records of the foreign contractor relating to the petroleum operations upon 24 hours
written notice;33

f. The petroleum output is sold at posted or market prices;34

g. The foreign contractor pays the 32% Philippine corporate income tax on its 40% share of
the net proceeds, including withholding tax on dividends or remittances of profits. 35 (Emphasis
supplied)

The Occidental-Shell FTAA gives the State its fair share of the income from the petroleum operations
of the foreign contractor. There is no question that the State receives its rightful share, amounting to
60% of the net proceeds,in recognition of its ownership of the petroleum resources. In addition,
Occidental-Shell's 40% share in the net proceeds is subject to the 32% Philippine income tax. The
Occidental-Shell FTAA also gives the State, through the DOE and BIR, full control and supervision
over the petroleum operations of the foreign contractor. The foreign contractor can recover only
the capital and operating expenses approved by the DOE or by the arbitral panel. 36 The
Occidental-Shell FTAA also contains other safeguards to protect the interest of the State as owner of
the petroleum resources. While the foreign contractor manages the contracted work or operations to
the extent of its financial or technical contribution, there are sufficient safeguards in the FTAA to
insure compliance with the constitutional requirements. The terms of the Occidental-Shell FTAA are
fair to the State and to Occidental-Shell.

In FTAAs with a foreign contractor, the State must receive at least 60% percent of the net proceeds
from the exploitation of its mineral resources. This share is the equivalent of the constitutional
requirement that at least 60% of the capital, and hence 60% of the income, of mining companies
should remain in Filipino hands. Intervenor CMP and even respondent WMCP agree that the State
has a 60% interest in the mining operations under an FTAA with a foreign
contractor. Intervenor CMP asserts that the Philippine Government "stands in the place of the 60%
Filipino-owned company."37 Intervenor CMP also states that "the contractor will get 40% of the
financial benefits,"38 admitting that the State, which is the owner of the mineral resources, will retain
the remaining 60% of the net proceeds.

Respondent WMCP likewise admits that the 60%-40% "sharing ratio between the Philippine
Government and the Contractor is also in accordance with the 60%-40% equity requirement for
Filipino-owned corporations."39 Respondent WMCP even adds that the 60%-40% sharing ratio
is "in line with the intent behind Section 2 of Article XII that the Filipino people, as represented
181

by the State, benefit primarily from the exploration, development, and utilization of the
Philippines' natural resources."40 If the State has a 60% interest in the mining operations under an
FTAA, then it must retain at least 60% of the net proceeds.

Otherwise, there is no sense exploiting the State's natural resources if all or a major part of the profits
are remitted abroad, precluding any real contribution to the national economy or the general welfare.
The constitutional requirement of full control and supervision necessarily means that the State must
receive the income that corresponds to the party exercising full control, and this logically means a
majority of the income.

The Occidental-Shell FTAA satisfies these constitutional requirements because the State receives
60% of the net proceeds and exercises full control and supervision of the petroleum operations. The
State's right to receive 60% of the net proceeds and its exercise of full control and supervision are the
essential constitutional requirements for the validity of any FTAA. The name given to the contract is
immaterial – whether a "Service Contract" or any other name - provided these two essential
constitutional requirements are present. Thus, the designation of the Occidental-Shell FTAA as a
"Service Contract" is inconsequential since the two essential constitutional requirements for the
validity of the contract as an FTAA are present.

With the State's right to receive 60% of the net proceeds, coupled with its control and supervision, the
petroleum operations in the Occidental-Shell FTAA are legally and in fact 60% owned and controlled
by Filipinos. Indeed, the State is directly undertaking the petroleum exploitation with Occidental-
Shell as the foreign contractor. The Occidental-Shell FTAA does not provide for the issuance of
exploration permits to Occidental-Shell precisely because the State itself is directly undertaking the
petroleum exploitation.

Section 3(aq) of RA 7942 allows the foreign contractor to hold the exploration permit under the FTAA.
However, Section 2, Article XII of the 1987 Constitution does not allow foreign owned corporations to
undertake directly mining operations. Foreign owned corporations can only act as contractors of the
State under the FTAA, which is one method for the State to undertake directly the exploitation of its
natural resources. The State, as the party directly undertaking the exploitation of its natural resources,
must hold through the Government all exploration permits and similar authorizations. Section 3(aq) of
RA 7942, in allowing foreign owned corporations to hold exploration permits, is unconstitutional.

The Occidental-Shell FTAA, involving a far riskier offshore venture than land-based mining
operations, is a modelfor emulation if foreign contractors want to comply with the constitutional
requirements. Section 112 of RA 7942, however, negates the benefits of the State from the
Occidental-Shell FTAA.

Occidental-Shell can invoke Section 112 of RA 7942 and deny the State its 60% share of the net
proceeds from the exploitation of petroleum. Section 112 allows the foreign contractor to pay only
the "government share in a mineral production-sharing agreement" under RA 7942. Section 80
of RA 7942 on MPSAs limits the "government share" solely to the excise tax – 2% on metallic and
non-metallic mineral products and 3% on petroleum. Section 112 of RA 7942 is unconstitutional since
it is contrary to Section 2, Article XII of the 1987 Constitution.

G. The WMCP FTAA Violates Section 2, Article XII of the 1987 Constitution

The WMCP FTAA41 ostensibly gives the State 60% share of the net mining revenue. In reality, this
60% share is illusory. Section 7.7 of the WMCP FTAA provides that:

From the Commencement of Commercial Production, the Contractor shall pay a


government share of sixty per centum (60%) of Net Mining Revenues, calculated in
accordance with the following provisions (the Government Share). The Contractor shall be
entitled to retain the balance of all revenues from the Mining Operations. (Emphasis supplied)
182

However, under Section 7.9 of the WMCP FTAA, if WMCP's foreign stockholders sell 60% of their
equity to a Philippine citizen or corporation, the State loses its right to receive its 60% share of the net
mining revenues under Section 7.7. Thus, Section 7.9 provides:

The percentage of Net Mining Revenues payable to the Government pursuant to


Clause 7.7 shall be reduced by 1% of Net Mining Revenues for every 1% ownership
interest in the Contractor held by a Qualified Entity. (Emphasis supplied)

What Section 7.7 gives to the State, Section 7.9 takes away without any offsetting compensation to
the State. In reality, the State has no vested right to receive any income from the exploitation of its
mineral resources. What the WMCP FTAA gives to the State in Section 7.7 is merely by
tolerance of WMCP's foreign stockholders, who can at anytime cut off the State's entire 60%
share by selling 60% of WMCP's equity to a Philippine citizen or corporation.42 The proceeds of
such sale do not accrue to the State but belong entirely to the foreign stockholders of WMCP.

Section 2.1 of the WMCP FTAA defines a "Qualified Entity" to include a corporation 60% Filipino
owned and 40% foreign owned.43 WMCP's foreign stockholders can sell 60% of WMCP's equity to
such corporation and the sale will still trigger the operation of Section 7.9 of the WMCP FTAA. Thus,
the State will receive ZERO percent of the income but the foreign stockholders will own beneficially
64% of WMCP, consisting of their remaining 40% equity and 24% pro-rata share in the buyer-
corporation. WMCP will then invoke Section 39 of RA 7942 allowing it to convert the FTAA into an
MPSA, thus subjecting WMCP to pay only 2% excise tax on mineral products in lieu of sharing its
mining income with the State. This violates Section 2, Article XII of the 1987 Constitution requiring
that only corporations "at least sixty per centum of whose capital is owned by such citizens" can enter
into co-production, joint venture or production-sharing agreements with the State.

The State, as owner of the mineral resources, must receive a fair share of the income from any
commercial exploitation of its mineral resources. Mineral resources form part of the national
patrimony, and so are the net proceeds from such resources. The Legislature or Executive
Department cannot waive the State's right to receive a fair share of the income from such mineral
resources.

The intervenor Chamber of Mines of the Philippines ("CMP") admits that under an FTAA with a
foreign contractor, the Philippine Government "stands in the place of the 60% Filipino owned
company" and hence must retain 60% of the net proceeds. Thus, intervenor CMP concedes that:

x x x In other words, in the FTAA situation, the Government stands in the place of the
60% Filipino-owned company, and the 100% foreign-owned contractor company takes all
the risks of failure to find a commercially viable large-scale ore body or oil deposit, for
which the contractor will get 40% of the financial benefits.44 (Emphasis supplied)

For this reason, intervenor CMP asserts that the "contractor's stipulated share under the WMCP
FTAA is limited to a maximum of 40% of the net production."45 Intervenor CMP further insists
that "60% of its (contractor's) net returns from mining, if any, will go to the Government under
the WMCP FTAA."46Intervenor CMP, however, fails to consider that the Government's 60% share
is illusory because under Section 7.9 of the WMCP FTAA the foreign stockholders of WMCP can
reduce at any time to ZERO percent the Government's share.

If WMCP's foreign stockholders do not immediately sell 60% of WMCP's equity to a Philippine citizen
or corporation, the State in the meantime receives its 60% share. However, under Section 7.10 of the
WMCP FTAA, the State shall receive its share "after the offsetting of the items referred to in
Clauses 7.8 and 7.9," namely:

7.8. The Government Share shall be deemed to include all of the following sums:

(a) all Government taxes, fees, levies, costs, imposts, duties and royalties including
excise tax, corporate income tax, customs duty, sales tax, value added tax,
occupation and regulatory fees, Government controlled price stabilization schemes,
183

any other form of Government backed schemes, any tax on dividend payments by
the Contractor or its Affiliates in respect of revenues from the Mining Operations and
any tax on interest on domestic and foreign loans or other financial arrangements or
accommodation, including loans extended to the Contractor by its stockholders;

(b) any payments to local and regional government, including taxes, fees, levies,
costs, imposts, duties, royalties, occupation and regulatory fees and infrastructure
contributions;

(c) any payments to landowners, surface rights holders, occupiers, indigenous people
or Claim-owners;

(d) costs and expenses of fulfilling the Contractor's obligations to contribute to


national development in accordance with Clause 10.1(i)(1) and 10.1(i)(2);

(e) an amount equivalent to whatever benefits that may be extended in the future by
the Government to the Contractor or to financial or technical assistance agreement
contractors in general;

(f) all of the foregoing items which have not previously been offset against the
Government Share in an earlier Fiscal year, adjusted for inflation.

7.9. The percentage of Net Mining Revenues payable to the Government pursuant to Clause
7.7 shall be reduced by 1% of Net Mining Revenues for every 1% ownership interest in the
Contractor held by a Qualified Entity.

It makes no sense why under Section 7.8(e) money spent by the Government for the benefit of the
contractor, like building roads leading to the mine site, is deductible from the State's 60% share of the
Net Mining Revenues. Unless of course the purpose is solely to reduce further the State's share
regardless of any reason. In any event, the numerous deductions from the State's 60% share make
one wonder if the State will ever receive anything for its ownership of the mineral resources. Even
assuming the State will receive something, the foreign stockholders of WMCP can at anytime take it
away by selling 60% of WMCP's equity to a Philippine citizen or corporation.

In short, the State does not have any right to any share in the net income from the mining operations
under the WMCP FTAA. The stipulated 60% share of the Government is illusory. The State is left to
collect only the 2% excise tax as its sole share from the mining operations.

Indeed, on 23 January 2001, WMCP's foreign stockholders sold 100% of WMCP's equity to
Sagittarius Mines, Inc., a domestic corporation 60% Filipino owned and 40% foreign owned. 47 This
sale automatically triggered the operation of Section 7.9 of the WMCP FTAA reducing the
State's share in the Net Mining Revenues to ZERO percent without any offsetting
compensation to the State. Thus, as of now, the State has no right under the WMCP FTAA to
receive any share in the mining revenues of the contractor, even though the State owns the mineral
resources being exploited under the WMCP FTAA.

Intervenor CMP anchors its arguments on the erroneous interpretation that the WMCP FTAA gives
the State 60% of the net income of the foreign contractor. Thus, intervenor CMP states that "60% of
its (WMCP's) net returns from mining, if any, will go to the Government under the WMCP
FTAA."48 This basic error in interpretation leads intervenor CMP to erroneous conclusions of law and
fact.

Like intervenor CMP, respondent WMCP also maintains that under the WMCP FTAA, the State
is "guaranteed" a 60% share of the foreign contractor's Net Mining Revenues. Respondent WMCP
contends, after quoting Section 7.7 of the WMCP FTAA, that:

In other words, the State is guaranteed a sixty per centum (60%) share of the Mining
Revenues, or 60% of the actual fruits of the endeavor. This is in line with the intent
184

behind Section 2 of Article XII that the Filipino people, as represented by the State,
benefit primarily from the exploration, development, and utilization of the Philippines'
natural resources.

Incidentally, this sharing ratio between the Philippine Government and the
Contractor is also in accordance with the 60%-40% equity requirement for Filipino-
owned corporations in Paragraph 1 of Section 2 of Article XII. 49 (Italics and
underscoring in the original)

This so-called "guarantee" is a sham. Respondent WMCP gravely misleads this Court. Section 7.9 of
the WMCP FTAA provides that the State's share "shall be reduced by 1% of Net Mining Revenues
for every 1% ownership interest in the Contractor held by a Qualified Entity." This reduction is
without any offsetting compensation to the State and constitutes a waiver of the State's share to
WMCP's foreign stockholders. The Executive Department cannot give away for free, especially to
foreigners, what forms part of the national patrimony. This negates the constitutionally mandated
State ownership of mineral resources for the benefit of the Filipino people.

WMCP's stockholders may also invoke Section 112 of RA 7942 allowing a mining contractor to pay
the State's share in accordance with Section 80 of RA 7942. WMCP will end up paying only the 2%
excise tax to the Philippine Government for the exploitation of the mineral resources the State
owns. In short, the old and discredited system of "license, concession or lease" will govern the
WMCP FTAA.

The WMCP FTAA is also emphatic in stating that WMCP shall have exclusive right to exploit,
utilize, process and dispose of all mineral products produced under the WMCP FTAA. Section
1.3 of the WMCP FTAA provides:

The Contractor shall have the exclusive right to explore, exploit, utilise, process and dispose
of all Mineral products and by-products thereof that may be derived or produced from the
Contract Area but shall not, by virtue only of this Agreement, acquire any title to lands
encompassed within the Contract Area.

Under the WMCP FTAA, the contractor has exclusive right to exploit, utilize and process the
mineral resources to the exclusion of third parties and even the Philippine Government. Since
WMCP's right is exclusive, the Government has no participation in approving the operating expenses
of the foreign contractor relating to the exploitation, utilization, and processing of mineral resources.
The Government will have to accept whatever operating expenses the contractor decides to incur in
exploiting, utilizing and processing mineral resources.

Under the WMCP FTAA, the contractor has exclusive right to dispose of the minerals recovered in
the mining operations. This means that the contractor can sell the minerals to any buyer, local or
foreign, at the price and terms the contractor chooses without any intervention from the State. There
is no requirement in the WMCP FTAA that the contractor must sell the minerals at posted or market
prices. The contractor has the sole right to "mortgage, charge or encumber" the "Minerals produced
from the Mining Operations."50

Section 8.3 of the WMCP FTAA also makes a sham of the DENR Secretary's authority to approve the
foreign contractor's Work Program. Section 8.3 provides:

If the Secretary gives a Rejection Notice the Parties shall promptly meet and endeavour to
agree on amendments to the Work Program or budget. If the Secretary and the Contractor
fail to agree on the proposed revision within 30 days from delivery of the Rejection
Notice then the Work Programme or Budget or variation thereof proposed by the
Contractor shall be deemed approved, so as not to unnecessarily delay the performance of
the Agreement. (Emphasis supplied)

The DENR Secretary is the representative of the State which owns the mineral resources. The DENR
Secretary implements the mining laws, including RA 7942. Section 8.3, however, treats the DENR
185

Secretary like a subservient non-entity whom the contractor can overrule at will. Under Section 8.3 of
the WMCP FTAA, the DENR Secretary has no authority whatsoever to disapprove the Work Program.
This is not what the Constitution means by full control and supervision by the State of mining
operations.

Section 10.4(i) of the WMCP FTAA compels the Philippine Government to agree to any request
by the foreign contractor to amend the WMCP FTAA to satisfy the conditions of creditors of the
contractor. Thus, Section 10.4(i) states:

(i) the Government shall favourably consider any request, from Contractor for
amendments of this Agreement which are necessary in order for the Contractor to
successfully obtain the financing;

x x x. (Emphasis supplied)

This provision requires the Government to favorably consider any request from the contractor - which
means that the Government must render a response favorable to the contractor. In effect, the
contractor has the right to amend the WMCP FTAA even against the will of the Philippine Government
just so the contractor can borrow money from banks.

True, the preceding Section 10.4(e) of the WMCP FTAA provides that "such financing arrangements
will in no event reduce the Contractor's obligations or the Government's rights." However, Section
10.4(i) binds the Government to agree to any future amendment requested by the foreign contractor
even if the Government does not agree with the wisdom of the amendment. This provision is contrary
to the State's full control and supervision in the exploitation of mineral resources.

Clearly, under the WMCP FTAA the State has no full control and supervision over the mining
operations of the contractor. Provisions in the WMCP FTAA that grant the State full control and
supervision are negated by other provisions that take away such control and supervision.

The WMCP FTAA also violates the constitutional limits on the term of an FTAA. Section 2, Article XII
of the 1987 Constitution limits the term of a mineral agreement to "a period not exceeding twenty-
five years, renewable for not more than twenty-five years, and under such terms and
conditions as may be provided by law." The original term cannot exceed 25 years, and at the end
of such term, either the Government or the contracting party may decide not to renew the mineral
agreement. However, both the Government and the contracting party may also decide to renew the
agreement, in which case the renewal cannot exceed another 25 years. What is essential is that
either party has the option to renew or not to renew the mineral agreement at the end of the original
term.

However, Section 3.3 of the WMCP FTAA binds the Philippine Government to an ironclad 50-year
term. Section 3.3 compels the Government to renew the FTAA for another 25 years after the
original 25-year term expires.Thus, Section 3.3 states:

This Agreement shall be renewed by the Government for a further period of twenty-five
(25) years under the same terms and conditions provided that the Contractor lodges a
request for a renewal with the Government not less than sixty (60) days prior to the expiry of
the initial term of this Agreement and provided that the Contractor is not in breach of any of
the requirements of this Agreement. (Emphasis supplied)

Under Section 3.3, the contractor has the option to renew or not to renew the agreement. The
Government has no such option and must renew the agreement once the contractor makes a request
for renewal. Section 3.3 violates the constitutional limits because it binds the Government to a 50-year
FTAA at the sole option of the contractor.

H. Arguments of the Solicitor General and the NEDA Secretary


186

The Solicitor General states that the "basic share" of the State in FTAAs involving large-scale
exploitation of minerals, petroleum and other mineral oils –

x x x consists of all direct taxes, fees and royalties, as well as other payments made by the
Contractor during the term of the FTAA. The amounts are paid to the (i) national government,
(ii) local governments, and (iii) persons directly affected by the mining project. Some of the
major taxes paid are as follows Section 3(g) of DAO-99-56:

A. Payments to National Government

· Excise tax on minerals – 2% of gross output of mining operations

· Contractor's income tax – 32% of taxable income for corporation

· Customs duties and fees - rate is set by Tariff and Customs Code

· VAT on imported equipment, goods and services - 10% of value

· Royalty on minerals extracted from mineral reservations, if applicable – 5% of the


actual market value of the minerals produced

· Documentary stamp tax – rate depends on the type of transaction

· Capital gains tax on traded stocks – 5 to 10% of the value

· Tax on interest payments on foreign loans – 15% of the interest

· Tax on foreign stockholders dividends - 15% of the dividend

· Wharfage and port fees

· Licensing fees (e.g., radio permit, firearms permit, professional fees)

B. Payments to Local Governments

· Local business tax - maximum of 2% of gross sale or receipt

· Real property tax - 2% of the fair market value of property based on an assessment
level set by the local government

· Local business tax - maximum of 2% of gross sale or receipt

· Special education levy - 1% of the basis used in real property tax

· Occupation tax - 50 pesos per hectare per year; 100 pesos per hectare per year if
located in a mineral concession

· Community tax - 10,500 pesos maximum per year

· Other local taxes and fees - rate and type depends on the local government

C. Other Payments

· Royalty to indigenous cultural communities, if any - not less than 1% of the gross
output from mining operations
187

· Special allowance – payment to claim owners or surface right owners

The Solicitor General argues that the phrase "among other things" in the second paragraph of
Section 81 of RA 7942 means that the State "is entitled to an additional government share to be
paid by the Contractor." The Solicitor General explains:

An additional government share is collected from an FTAA contractor to fulfill the intent of
Section 81 of RA No. 7942, to wit:

Sec. 81. The Government share in an FTAA shall consist of, among other things, the
Contractor's corporate income tax, excise tax, special allowance, withholding tax due
from the Contractor's foreign stockholders arising from dividends or interest payments
to the said foreign stockholders in case of a foreign-owned corporation and all such
other taxes, duties and fees as provided for in existing laws. (Underscoring supplied)

The phrase "among other things" indicates that the Government is entitled to an additional
share to be paid by the Contractor, aside from the basic share in order to achieve the fifty-fifty
sharing of net benefits from mining.

By including indirect taxes and other financial contributions in the form of fuel tax;
employees' payroll and fringe benefits; various withholding taxes on royalties to land
owners and claim owners, and employees' income; value added tax on local goods,
equipment, supplies and services; and expenditures for social infrastructures in the
mine site (hospitals, schools, etc.) and development of host and neighboring
communities, geosciences and mining technology, the government share will be in the
range of 60% or more of the total financial benefits. (Bold and underscoring in the
original)

The Solicitor General enumerates this "additional government share" as "indirect taxes and other
financial contributions in the form of fuel tax; employees' payroll and fringe benefits; various
withholding taxes on royalties to land owners and claim owners, and employees' income;
value added tax on local goods, equipment, supplies and services; x x x." The Solicitor
General's argument merely confirms that under Section 81 of RA 7942 the State only receives taxes,
duties and fees under the FTAA. The State does not receive, as owner of the mineral resources, any
income from the mining operations of the contractor.

In short, the "basic share" of the State consists of direct taxes by the national and local
governments. The "additional share" of the State consists of indirect taxes including even fringe
benefits to employees and compensation to private surface right owners. Direct and indirect
taxes, however, are impositions by the taxing authority, a burden borne by all taxpayers whether or
not they exploit the State's mineral resources. Fringe benefits of employees are compensation for
services rendered under an employer-employee relationship. Compensation to surface right owners is
payment for the damage suffered by private landowners arising from the mining operations. All these
direct and indirect taxes, as well as other expenses of the contractor, do not constitute
payment for the share of the State as owner of the mineral resources.

Clearly, the so-called "share" of the State consists only of direct and indirect taxes, as well as other
operating expenses not even payable to the State. The Solicitor General in effect concedes that
under the second paragraph of Section 81, the State does not receive any share of the net proceeds
from the mining operations of the FTAA contractor. Despite this, the Solicitor General insists that the
State remains the owner of the mineral resources and exercises full control over the mining
operations of the FTAA contractor. The Solicitor General has redefined the civil law concept of
ownership,51 by giving the owner full control in the exploitation of the property he owns but denying
him the fruits or income from such exploitation. The only satisfaction of the owner is that the FTAA
contractor pays taxes to the Government.

However, even this psychological satisfaction is dubious. Under the third paragraph of Section 81 of
RA 7942, the "collection of Government share in financial and technical assistance agreement shall
188

commence after the financial and technical assistance agreement contractor has fully recovered its
pre-operating expenses, exploration, and development expenditures, inclusive." This provision does
not defer the collection of the State's "share," but prevents the accrual of the State's "share" until the
contractor has fully recovered all its pre-operating, exploration and development expenditures. This
provision exempts for an undefined period the contractor from all existing taxes that are part
of the Government's so-called "share" under Section 81.52 The Solicitor General has interpreted
these taxes to include "other national taxes and fees" as well as "other local taxes and fees."

Secretary Romulo L. Neri of the National Economic and Development Authority ("NEDA") has warned
this Court of the supposed dire repercussions to the nation's long-term economic growth if this Court
declares the assailed provisions of RA 7942 unconstitutional.53 Under the Constitution, the NEDA is
the "independent (economic) planning agency of the government." 54 However, in this case the NEDA
Secretary has joined the chorus of the foreign chambers of commerce to uphold the validity of RA
7942 as essential to entice foreign investors to exploit the nation's mineral resources.

We cannot fault the foreign chambers of commerce for driving a hard bargain to maximize the profits
of foreign investors. We are, however, saddened that the NEDA Secretary is willing to give away for
free to foreign investors the State's share of the income from its ownership of mineral resources. If the
NEDA Secretary owns the mineral resources instead of the State, will he allow the foreign contractor
to exploit his mineral resources for free, the only obligation of the foreign contractor being to pay taxes
to the Government?

Secretary Neri claims that the potential tax collection from the mining industry alone is P57 billion as
against the present collection of P2 billion. Secretary Neri adds that the potential tax collection from
incremental activities linked to mining is another P100 billion, thus putting the total potential tax
collection from mining and related industries at P157 billion.55 Secretary Neri also estimates
the "potential mining wealth in the Philippines" at P47 trillion or US$840 billion, 15 times our total
foreign debt of US$56 billion.56

If all that the State will receive from its P47 trillion potential mineral wealth is the P157 billion in direct
and indirect taxes, then the State will truly receive only a pittance. The P157 billion in taxes constitute
a mere .33% or a third of 1% of the total mineral wealth of P47 trillion. Even if the P157 billion is
collected annually over 25 years, the original term of an FTAA, the total tax collection will amount to
only P3.92 trillion, or a mere 8.35% of the total mineral wealth. The rest of the country's mineral
wealth will flow out of the country if foreign contractors exploit our mineral resources under FTAAs
pursuant to RA 7942.

Secretary Neri also warns that foreign investors who have acquired local cement factories in the last
ten years will find their investments illegal if the Court declares unconstitutional the assailed
provisions of RA 7942.57 Such specious arguments deserve scant consideration. Cement
manufacturing is not a nationalized activity. Hence, foreigners can own 100% of cement companies in
this country. When the foreign investors acquired the local cement factories, they spun off the quarry
operations into separate companies 60% owned by Filipino citizens. The foreign investors knew the
constitutional requirements of holding quarry permits.

Besides, the quarrying requirement of cement companies is just a simple surface mining of limestone.
Such activity does not constitute large-scale exploitation of mineral resources. It definitely cannot
qualify for FTAAs with foreign contractors under the fourth paragraph of Section 2, Article XII of the
Constitution. Obviously, only a company at least 60% Filipino owned can engage in such mining
activity.

The offshore Occidental-Shell FTAA shows that even in riskier ventures involving far more capital
investments, the State can negotiate and secure at least 60% of the net proceeds from the
exploitation of mineral resources. Foreign contractors like Occidental-Shell are willing to pay the State
60% of the net proceeds from petroleum operations, in addition to paying the Government the 32%
corporate income tax on its 40% share of the net proceeds. Even intervenor CMP and respondent
WMCP agree that the State has a 60% interest in mining operations under an FTAA. I simply
cannot fathom why the NEDA Secretary is willing to accept a ZERO percent share in the income from
the exploitation of inland mineral resources.
189

FTAAs like the WMCP FTAA, which gives the State an illusory 60% share of the net proceeds from
mining revenues, will only impoverish further the Filipino people. The nation's potential mineral wealth
of P47 trillion will contribute to economic development only if the bulk of the wealth remains in the
country, not if remitted abroad by foreign contractors.

I. Refutation of Arguments of Majority Opinion

The majority opinion advances the following arguments:

1. DENR Department Administrative Order No. 56-99 ("DAO 56-99") is the basis for
determining the State's share in the mining income of the foreign FTAA contractor. The DENR
Secretary issued DAO 56-99 pursuant to the phrase "among other things" in Section 81 of
RA 7942. The majority opinion claims that the phrase "among other things" "clearly and
unmistakably reveals the legislative intent to have the State collect more than just the
usual taxes, duties and fees." The majority opinion anchors on the phrase "among other
things" its argument that RA 7942 allows the State to collect a share in the mining income of
the foreign FTAA contractor, in addition to taxes, duties and fees. Thus, on the phrase
"among other things" depends whether the State and the Filipino people are entitled
under RA 7942 to share in the vast mineral wealth of the nation, estimated by NEDA
at P47 trillion or US$840 billion.

2. FTAAs, like the WMCP FTAA, are not subject to the term limit in Section 2, Article XII
of the 1987 Constitution. In short, while co-production, joint venture and production-sharing
agreements cannot exceed 25 years, renewable for another 25 years, as provided in Section
2, Article XII of the 1987 Constitution, the WMCP FTAA is not governed by the constitutional
limitation. The majority opinion states that the "constitutional term limitations do not apply
to FTAAs." Thus, the majority opinion upholds the validity of Section 3.3 of the WMCP FTAA
providing for a 50-year term at the sole option of WMCP.

3. Section 112 of RA 7942, placing "all valid and existing" mining agreements under the
fiscal regime prescribed in Section 80 of RA 7942, does not apply to FTAAs. Thus, the
majority opinion states, "[W]hether Section 112 may properly apply to co-production or
joint venture agreements, the fact of the matter is that it cannot be made to apply to
FTAAs."

4. Foreign FTAA contractors and even foreign corporations can hold exploration permits,
despite Section 2, Article XII of the 1987 Constitution reserving to Philippine citizens and to
corporations 60% Filipino owned the "exploration, development and utilization of natural
resources." Thus, the majority opinion states that "there is no prohibition at all
against foreign or local corporations or contractors holding exploration permits."

5. The Constitution does not require that the State's share in FTAAs or other mineral
agreements should be at least 60% of the net mining revenues. Thus, the majority opinion
states that "the Charter did not intend to fix an iron-clad rule on the 60 percent share,
applicable to all situations at all times and in all circumstances."

I respond to the arguments of the majority opinion.

1. DAO 99-56 as Basis for Government's Share in FTAAs

The main thrust of my separate opinion is that mineral agreements under RA 7942, whether FTAAs
under Section 81 or MPSAs under Section 80, do not allow the State to receive any share from the
income of mining companies. The State can collect only taxes, duties and fees from mining
companies.

The majority opinion, however, points to the phrase "among other things" in the second paragraph
of Section 81 as the authority of the State to collect in FTAAs a share in the mining income separate
from taxes, duties and fees. The majority opinion can point to no other provision in RA 7942 allowing
190

the State to collect any share. The majority opinion admits that limiting the State's share in any
mineral agreement to taxes, duties and fees is unconstitutional. Thus, the majority opinion's case
rises or falls on whether the phrase "among other things" allows the State to collect from
FTAA contractors any income in addition to taxes, duties and fees.

In the case of MPSAs, the majority opinion cannot point to any provision in RA 7942 allowing the
State to collect any share in MPSAs separate from taxes, duties and fees. The language of Section
80 is so crystal clear – "the total government share in a mineral production sharing agreement
shall be the excise tax on mineral products" - that there is no dispute whatsoever about it. The
majority opinion merely states that the constitutionality of Section 80 is not in issue in the present
case. Section 81, the constitutionality of which the majority opinion admits is in issue here, is
intertwined with Sections 39, 80, 84 and 112. Resolving the constitutionality of Section 81 necessarily
involves a determination of the constitutionality of Sections 39, 80, 84 and 112.

The WMCP FTAA, the constitutionality of which is certainly in issue, is governed not only by Section
81 but also by Sections 39, 80 and 112. The reason is that the WMCP FTAA is a reversible contract
that gives WMCP the absolute option at anytime to convert the FTAA into an MPSA. In short, the
WMCP FTAA is like a single coin with two sides - one an FTAA and the other an MPSA.

a. The Integrated Intent, Plan and Structure of RA 7942

The clear intent of RA 7942 is to limit the State's share from mining operations to taxes, duties and
fees, unless the State contributes equity in addition to the mineral resources. RA 7942 does not
recognize the mere contribution of mineral resources as entitling the State to receive a share in the
net mining revenues separate from taxes, duties and fees. Thus, Section 80 expressly states that
the "total government share in a mineral production sharing agreement shall be the excise tax
on mineral products." Section 84 reiterates this by stating that "with respect to mineral
production sharing agreement, the excise tax on mineral products shall be the government
share under said agreement." The only share of the State in an MPSA is the excise tax. Ironically,
Sections 80 and 84 disallow the State from sharing in the production or income, even as the contract
itself is called a mineral production sharing agreement.

In co-production and joint venture agreements, where the State contributes equity in addition to the
mineral resources, the first paragraph of Section 81 expressly requires that "the share of the
government x x x shall be negotiated by the Government and the contractor." However, in
FTAAs where the State contributes only its mineral resources, the second paragraph of Section 81
states –

The Government share in financial or technical assistance agreement shall consist of, among
other things, the contractor's corporate income tax, excise tax, special allowance, withholding
tax due from the contractor's foreign stockholders arising from dividend or interest payments
to the said foreign stockholder in case of a foreign national and all such other taxes, duties
and fees as provided for under existing laws.

All the items enumerated in the second paragraph of Section 81 as comprising the "Government
share" refer totaxes, duties and fees. The phrase "all such other taxes, duties and fees as
provided for under existing laws" makes this clear.

Section 112 places "all valid and existing mining" agreements "at the date of effectivity" of RA
7942 under the fiscal regime prescribed in Section 80. Section 112 expressly states that
the "government share in mineral production sharing agreement x x x shall immediately
govern and apply to a mining lessee or contractor."Section 112 provides:

Section 112. Non-impairment of Existing Mining/Quarrying Rights. — All valid and existing
mining lease contracts, permits/licenses, leases pending renewal, mineral production-
sharing agreements granted under Executive Order No. 279, at the date of effectivity of
this Act, shall remain valid, shall not be impaired, and shall be recognized by the
Government: Provided, That the provisions of Chapter XIV on government share in
191

mineral production-sharing agreement and of Chapter XVI on incentives of this


Act shall immediately govern and apply to a mining lessee or contractor unless the
mining lessee or contractor indicates his intention to the secretary, in writing, not to avail of
said provisions: Provided, further, That no renewal of mining lease contracts shall be made
after the expiration of its term: Provided, finally, That such leases, production-sharing
agreements, financial or technical assistance agreements shall comply with the applicable
provisions of this Act and its implementing rules and regulations. (Emphasis supplied)

Thus, Section 112 requires "all" FTAAs and MPSAs, as of the date of effectivity of RA 7942, to pay
only the excise tax - 2% on metallic and non-metallic minerals and 3% on petroleum 58 - instead of the
stipulated mining income sharing, if any, in their respective FTAAs or MPSAs.

This means that Section 112 applies even to the Occidental-Shell FTAA, which was executed
before the enactment of RA 7942. This reduces the State's share in the Malampaya gas
extraction from 60% of net proceeds to 3% of the market price of the gas as provided in
Section 80 of RA 7942 in relation to Section 151 of the National Internal Revenue Code. This is
disastrous to the national economy because Malampaya under the original Occidental-Shell
FTAA generates annually some US$0.5 billion to the National Treasury.

Section 112 applies to all agreements executed "under Executive Order No. 279." The WMCP
FTAA expressly states in its Section 1.1, "This Agreement is a Financial & Technical Assistance
Agreement entered into pursuant to Executive Order No. 279." Thus, Section 112 applies to the
WMCP FTAA.

Section 39 of RA 7942 grants the FTAA contractor the "option to convert" the FTAA into an
MPSA "at any time during the term" of the FTAA if the contract areas are not economically viable
for large-scale mining. Once the contractor reduces its foreign equity to not more than 40%, the
Secretary "shall approve the conversion and execute the mineral production sharing
agreement. Thus, Section 39 provides:

Section 39. Option to Convert into a Mineral Agreement. — The contractor has the option
to convert the financial or technical assistance agreement to a mineral agreement at
any time during the term of the agreement, if the economic viability of the contract area is
found to be inadequate to justify large-scale mining operations, after proper notice to the
Secretary as provided for under the implementing rules and regulations: Provided, That the
mineral agreement shall only be for the remaining period of the original agreement.

In the case of a foreign contractor, it shall reduce its equity to forty percent (40%) in the corporation,
partnership, association, or cooperative. Upon compliance with this requirement by the
contractor, the Secretary shall approve the conversion and execute the mineral production-
sharing agreement. (Emphasis supplied)

The only requirement in the second paragraph of Section 39 is that the FTAA contractor shall reduce
its foreign equity to 40%. The second paragraph states, "Upon compliance with this requirement,
the Secretary shall approve the conversion and execute the mineral production sharing
agreement." The determination of the economic viability of the contract area for large-scale mining,
which is left to the foreign contractor with "proper notice" only to the DENR Secretary, is not even
made a condition for the conversion.

Under Section 3(aq) of RA 7942, the foreign contractor holds the exploration permit and conducts the
physical exploration. The foreign contractor controls the release of the technical data on the mineral
resources. The foreign contractor can easily justify the non-viability of the contract area for large-scale
mining. The Philippine Government will have to depend on the foreign contractor for technical
data on whether the contract area is viable for large-scale mining. Obviously, such a situation
gives the foreign contractor actual control in determining whether the contract area is viable for large-
scale mining.
192

The conversion from an FTAA into an MPSA is solely at the will of the foreign contractor because the
contractor can choose at any time to sell 60% of its equity to a Philippine citizen. The price or
consideration for the sale of the contractor's 60% equity does not go to the State but to the foreign
stockholders of the contractor. Under Section 80 of RA 7942, once the FTAA is converted into an
MPSA the only share of the State is the 2% excise tax on mineral products. Thus, under RA 7942
the FTAA contractor has the absolute option to pay the State only the 2% excise tax, despite
any other stipulated consideration in the FTAA.

Clearly, Sections 3(aq), 39, 80, 81, 84 and 112 are tightly integrated under a single intent, plan and
structure: unless the State contributes equity in addition to the mineral resources, the State shall
receive only taxes, duties and fees. The State's contribution of mineral resources is not sufficient to
entitle the State to receive any income from the mining operations separate from taxes, duties and
fees.

b. The Meaning of the Phrase "Among Other Things"

As far as the State and the Filipino people are concerned, the most important part of an FTAA is the
consideration: how much will the State receive from the exploitation of its non-renewable and
exhaustible mineral resources?

Section 81 of RA 7942 does not require the foreign FTAA contractor to pay the State any share from
the mining income apart from taxes, duties and fees. The second paragraph of Section 81, just like
Section 80, only allows the State to collect taxes, duties and fees as the State's share from the mining
operations. The intent of RA 7942 is that the State cannot share in the income from mining
operations, separate from taxes, duties and fees, based only on the mineral resources that the State
contributes to the mining operations.

This is also the position of the Solicitor General – that the State's share under Section 81 refers only
to direct and indirect taxes. Thus, the Solicitor General agrees that Section 81 does not allow
the State to collect any share from the mining income separate from taxes, duties and
fees. The majority opinion agrees that Section 81 is unconstitutional if it does not require the foreign
FTAA contractor to pay the State any share of the net mining income apart from taxes, duties and
fees.

However, the majority opinion says that the phrase "among other things" in Section 81 is the
authority to require the FTAA contractor to pay a consideration separate from taxes, duties and fees.
The majority opinion cites the phrase "among other things" as the source of power of the DENR
Secretary to adopt DAO 56-9959prescribing the formulae on the State's share from mining
operations separate from taxes, duties and fees.

In short, the majority opinion says that the phrase "among other things" is a delegation of legislative
power to the DENR Secretary to adopt the formulae on the share of the State from mining
operations. The issue now is whether the phrase "among other things" in the second
paragraph of Section 81 is intended as a delegation of legislative power to the DENR
Secretary. If so, the issue turns on whether it is a valid delegation of legislative power. I
reproduce again the second paragraph of Section 81 for easy reference:

The Government share in financial or technical assistance agreement shall consist of, among
other things,the contractor's corporate income tax, excise tax, special allowance,
withholding tax due from the contractor's foreign stockholders arising from dividend or
interest payments to the said foreign stockholder in case of a foreign national and all such
other taxes, duties and fees as provided for under existing laws. (Emphasis supplied)

Section 81 of RA 7942 does not delegate any legislative power to the DENR Secretary to adopt the
formulae in determining the share of the State. There is absolutely no language in the second
paragraph of Section 81 granting the DENR Secretary any delegated legislative power. Thus,
the DENR Secretary acted without authority or jurisdiction in issuing DAO 56-99 based on a supposed
delegated power in the second paragraph of Section 81. This makes DAO 56-99 void.
193

Even assuming, for the sake of argument, that there is language in Section 81 delegating legislative
power to the DENR Secretary to adopt the formulae in DAO 56-99, such delegation is void. Section
81 has no standards by which the delegated power shall be exercised. There is no specification on
the minimum or maximum share that the State must receive from mining operations under FTAAs. No
parameters on the extent of the delegated power to the DENR Secretary are found in Section 81.
Neither were such parameters ever discussed even remotely by Congress when it enacted RA 7942.

In sharp contrast, the first paragraph of the same Section 81, in prescribing the State's share in co-
production and joint venture agreements, expressly specifies the standards in determining the
State's share as follows: "(a) capital investment of the project, (b) risks involved, (c) contribution of the
project to the economy, and (d) other factors that will provide for a fair and equitable sharing between
the Government and the contractor." The reason for the absence of similar standards in the
succeeding paragraph of Section 81 in determining the State's share in FTAAs is obvious - the State's
share in FTAAs is limited solely to taxes, duties and fees. Thus, such standards are inapplicable and
irrelevant.

The majority opinion now makes the formulae in DAO 56-99 the heart and soul of RA 7942 because
the formulae supposedly determine the consideration of the FTAA. The consideration is the most
important part of the FTAA as far as the State and Filipino people are concerned. The formulae in
DAO 56-99 derive life solely from the phrase "among other things." DAO 56-99 itself states that it is
issued "[P]ursuant to Section 81 and other pertinent provisions of Republic Act No. 7942." Without the
phrase "among other things," the majority opinion could not point to any other provision in RA 7942 to
support the existence of the formulae in DAO 56-99.

Thus, the phrase "among other things" determines whether the FTAA has the third element of a
valid contract – the commercial value or consideration that the State will receive. The majority opinion
in effect says that Congress made the wealth and even the future prosperity of the nation to depend
on the phrase "among other things."

The DENR Secretary can change the formulae in DAO 56-99 any time even without the approval of
the President or Congress. The DENR Secretary is the sole authority to determine the amount of
consideration that the State shall receive in an FTAA. Section 5 of DAO 56-99 states:

x x x any amendment of an FTAA other than the provision on fiscal regime shall require
the negotiation with the Negotiation Panel and the recommendation of the Secretary for
approval of the President of the Republic of the Philippines. (Emphasis supplied)

Under Section 5, if the amendment in the FTAA involves non-fiscal matters, the amendment requires
the approval of the President. However, if the amendment involves a change in the fiscal regime –
referring to the consideration of the FTAA - the DENR Secretary has the final authority and approval
of the President is not required. This makes the DENR Secretary more powerful than the President.

Section 5 of DAO 56-99 violates paragraphs 4 and 5 of Section 2, Article XII of the 1987 Constitution
mandating that the President shall approve all FTAAs and send copies of all approved FTAAs to
Congress. The consideration of the FTAA is the most important part of the FTAA as far as the State
and the Filipino people are concerned. The DENR Secretary, in issuing DAO 56-99, has arrogated
to himself the power to approve FTAAs, a power vested by the Constitution solely in the
President. By not even informing the President of changes in the fiscal regime and thus preventing
such changes from reaching Congress, DAO 56-99 even seeks to hide changes in the fiscal regime
from Congress. By its provisions alone, DAO 56-99 is clearly unconstitutional and void.

Section 5 of DAO 56-99 also states that "[A]ll FTAAs approved prior to the effectivity of this
Administrative Order shall remain valid and be recognized by the Government." This means that
the fiscal regime of an FTAA executed prior to the effectivity of DAO 56-99 "shall remain valid and be
recognized." If the earlier FTAA provides for a fiscal regime different from DAO 56-99, then the fiscal
regime in the earlier FTAA shall prevail. In effect, DAO 56-99 exempts an FTAA approved prior to its
effectivity from paying the State the share prescribed in the formulae under DAO 56-99 if the earlier
FTAA provides for a different fiscal regime. Such is the case of the WMCP FTAA.
194

Based on the majority opinion's position that the 1987 Constitution requires payment in addition to
taxes, duties and fees, this makes DAO 56-99 unconstitutional and void. DAO 56-99 does not require
prior FTAAs to pay the State the share prescribed in the formulae under DAO 56-99 even if the
consideration in the prior FTAAs is limited only to taxes, duties and fees. DAO 56-99 recognizes such
payment of taxes, duties and fees as a "valid" consideration. Certainly, the DENR Secretary has no
authority to exempt foreign FTAA contractors from a constitutional requirement. Not even Congress or
the President can do so.

Ironically, DAO 56-99, the very authority the majority opinion cites to support its claim that the WMCP
FTAA has a consideration, does not apply to the WMCP FTAA. By its own express terms, DAO 56-
99 does not apply to FTAAs executed before the issuance of DAO 56-99, like the WMCP
FTAA. The majority opinion's position has no leg to stand on since even DAO 56-99, assuming it is
valid, cannot save the WMCP FTAA from want of consideration.

The formulae prescribed in DAO 56-99 are totally alien to the phrase "among other things." There is
no relationship whatsoever between the phrase "among other things" and the highly esoteric formulae
prescribed in DAO 56-99. No one in this Court can assure the Filipino people that the formulae in
DAO 56-99 will guarantee the State 60%, or 30% or even 10% of the net proceeds from the mining
operations. And yet the majority opinion trumpets DAO 56-99 as the savior of Section 81 from certain
constitutional infirmity.

The majority opinion gives the stamp of approval and legitimacy on DAO 56-99. This assumes that
the majority understand fully the formulae in DAO 56-99. Can the majority tell the Court and the
Filipino people the minimum share that the State will receive under the formulae in DAO 56-99? The
formulae in DAO 56-99 are fuzzy since they do not guarantee the minimum share of the State,
unlike the clear and specific income sharing provisions in the Occidental-Shell FTAA or in the case
of Consolidated Mines, Inc. v. Court of Tax Appeals.60

The Solicitor General asserts that the phrase "among other things" refers to indirect taxes, an
interpretation that contradicts the DENR Secretary's interpretation under DAO 56-99. The Solicitor
General is correct. The ejusdem generis rule of statutory interpretation applies squarely to the
phrase "among other things."

In Philippine Bank of Communications v. Court of Appeals,61 the Court held:

Under the rule of ejusdem generis, where a description of things of a particular class or kind
is 'accompanied by words of a generic character, the generic words will usually be limited to
things of a kindred nature with those particularly enumerated x x x.'

In Grapilon v. Municipal Council of Cigara,62 the Court construed the general word "absence" in the
phrase "absence, suspension or other temporary disability of the mayor" in Section 2195 of the
Revised Administrative Code as "on the same level as 'suspension' and 'other forms of temporary
disability'." The Court quoted with approval the following Opinion of the Secretary of Interior:

The phrase 'other temporary disability' found in section 2195 of the Code, follows the words
'absence' and 'suspension' and is used as a modifier of the two preceding words, under the
principle of statutory construction known as ejusdem generis.

In City of Manila v. Entote,63 the Court ruled that broad expressions such as "and all
others" or "any others" or "other matters," when accompanied by an enumeration of items of the
same kind or class, "are usually to be restricted to persons or things of the same kind or class with
those specifically named" in the enumeration. Thus, the Court held:

In our jurisdiction, this Court in Ollada vs. Court of Tax Appeals, et al. applied the rule of
"ejusdem generis" to construe the purview of a general phrase "other matters" appearing
after an enumeration of specific cases decided by the Collector of Internal Revenue and
appealable to the Court of Tax Appeals found in section 7, paragraph 1, of Republic Act No.
1125, and it held that in order that a matter may come under said general clause, it is
195

necessary that it belongs to the same kind or class of cases therein specifically enumerated.
(Emphasis supplied)

The four requisites of the ejusdem generis rule64 are present in the phrase "among other
things" as appearing in Section 81 of RA 7942. First, the general phrase "among other things" is
accompanied by an enumeration of specific items, namely, "the contractor's corporate income tax,
excise tax, special allowance, withholding tax due from the contractor's foreign stockholders
arising from dividend or interest payments to the said foreign stockholder in case of a foreign national
and all such other taxes, duties and fees as provided for under existing laws." Second, all the items
enumerated are of the same kind or class - they are all taxes, duties and fees. Third, the enumeration
of the specific items is not exhaustive because "all such other taxes, duties and fees" are included.
Thus, the enumeration of specific items is merely illustrative. Fourth, there is no indication of
legislative intent to give the general phrase "among other things" a broader meaning. On the
contrary, the legislative intent of RA 7942 is to limit the State's share from mining operations to taxes,
duties and fees.

In short, the phrase "among other things" refers to taxes, duties and fees. The phrase "among
other things" is even followed at the end of the sentence by the phrase "and all such other taxes,
duties, and fees," reinforcing even more the restriction of the phrase "among other things" to
taxes, duties and fees. The function of the phrase "and such other taxes, duties and fees" is to clarify
that the taxes enumerated are not exhaustive but merely illustrative.

c. Formulae in DAO 56-99 a Mere Creation of DENR

The majority opinion praises the DENR for "conceiving and developing" the formulae in DAO 56-
99. Thus, the majority opinion states:

As can be seen from DAO 56-99, the agencies concerned did an admirable job
of conceiving and developing not just one formula, but three different formulas for
arriving at the additional government share. (Emphasis supplied)

Indeed, we credit the DENR for conceiving and developing on their own the formulae in DAO 56-
99. The formulae are the creation of DENR, not of Congress.

The DENR conceived and developed the formulae to save Section 81 not only from constitutional
infirmity, but also from blatantly depriving the State and Filipino people from any share in the income
of mining companies. However, the DENR's admittedly "admirable job" cannot amend Section 81 of
RA 7942. The DENR has no legislative power to correct constitutional infirmities in RA 7942. The
DENR does not also possess the constitutional power to prescribe the sharing of mining income
between the State and mining companies, the act the DENR attempts to do in adopting DAO 56-99.

d. DAO 56-99 is an Exercise in Futility

Even assuming arguendo the majority opinion is correct that the phrase "among other things"
constitutes sufficient legal basis to issue DAO 56-99, the FTAA contractor can still prevent the State
from collecting any share of the mining income. By invoking Section 39 of RA 7942 giving the foreign
FTAA contractor the option to convert the FTAA into an MPSA, the FTAA contractor can easily
place itself outside the scope of DAO 56-99 which expressly applies only to FTAAs.

Also, by invoking Section 112, the foreign contractor need not even convert its FTAA into a mineral
production agreement to place its contract under Section 80 and outside of Section 81. Section 112
automatically and immediately places all FTAAs under the fiscal regime applicable to MPSAs, forcing
the State to collect only the 2% excise tax. Thus, DAO 56-99 is an exercise in futility. This now
compels the Court to resolve the constitutionality of Sections 39 and 112 of RA 7942 in the present
case.

e. Congress Prescribes the Terms and Conditions of FTAAs.


196

In a last-ditch attempt to justify the constitutionality of DAO 56-99, the majority opinion now claims
that the President has the prerogative to prescribe the terms and conditions of FTAAs,
including the fiscal regime of FTAAs. The majority opinion states:

x x x It is the President who is constitutionally mandated to enter into FTAAs with foreign
corporations, and in doing so, it is within the President's prerogative to specify certain terms
and conditions of the FTAAs, for example, the fiscal regime of FTAAs - i.e., the sharing of
the net revenues between the contractor and the State. (Emphasis in the original;
underscoring supplied)

The majority opinion is re-writing the 1987 Constitution and even RA 7942. Paragraph 4, Section 2,
Article XII of the 1987 Constitution expressly provides:

The President may enter into agreements with foreign-owned corporations involving either
technical or financial assistance for large-scale exploration, development, and utilization of
minerals, petroleum, and other mineral oils according to the general terms and conditions
provided by law, x x x. (Emphasis supplied)

Clearly, the 1987 Constitution mandates that the President may enter into FTAAs only "according to
the general terms and conditions provided by law." There is no doubt whatsoever that it is
Congress that prescribes the terms and conditions of FTAAs, not the President as the majority
opinion claims. The 1987 Constitution mandates the President to comply with the terms and
conditions prescribed by Congress for FTAAs.

Indeed, RA 7942 stipulates the terms and conditions for FTAAs. Section 35 of RA 7942 provides that
the "following terms, conditions, and warranties shall be incorporated in the financial or
technical assistance agreement to wit: x x x." Section 38 of RA 7942 expressly limits an FTAA to
a "term not exceeding twenty-five (25) years,"which is one of the issues in the present case.

The majority opinion claims that the President has the power to prescribe "the fiscal regime of
FTAAs – i.e., the sharing of the net mining revenues between the contractor and the
State." This claim of the majority opinion renders the entire Chapter XIV of RA 7942 an act of
usurpation by Congress of Presidential power. Chapter XIV – entitled "Government Share" -
prescribes the fiscal regimes of MPSAs and FTAAs. The constitutionality of Sections 80 and 81 of
Chapter XIV - whether the fiscal regimes prescribed in these sections of RA 7942 comply with the
1987 Constitution - is the threshold issue in this case.

The majority opinion seeks to uphold the constitutionality of Section 81 of RA 7942, an act of
Congress prescribing the fiscal regime of FTAAs. If it is the President who has the constitutional
authority to prescribe the fiscal regime of FTAAs, then Section 81 is unconstitutional for being a
usurpation by Congress of a Presidential power. The majority opinion not only re-writes the 1987
Constitution, it also contradicts itself.

That is not all. By claiming that the President has the prerogative to prescribe the fiscal regime of
FTAAs, the majority opinion contradicts its basic theory that DAO 56-99 draws life from the
phrase "among other things" in Section 81 of RA 7942. Apparently, the majority opinion is no longer
confident of its position that DAO 56-99 draws life from the phrase "among other things." The
majority opinion now invokes a non-existent Presidential power that directly collides with the express
constitutional power of Congress to prescribe the "general terms and conditions" of FTAAs.

f. Sections 80 and 84 of RA 7942 are Void on their Face

Definitely, Section 80 of RA 7942 is constitutionally infirm even based on the reasoning of the majority
opinion. The majority opinion agrees that the 1987 Constitution requires the mining contractor to pay
the State "more than just the usual taxes, duties and fees." Under Section 80, the excise tax – 2%
for metallic and non-metallic minerals and 3% for petroleum - is the only and total share of the State
from mining operations. Section 80 provides:
197

Section 80. Government Share in Mineral Production Sharing Agreement. — The total
government share in a mineral production sharing agreement shall be the excise tax
on mineral products as provided in Republic Act No. 7729, amending Section 151(a) of the
National Internal Revenue Code, as amended. (Emphasis supplied)

Section 80 has no ifs or buts. Section 84 even reiterates Section 80 that "with respect to a mineral
production sharing agreement, the excise tax on mineral products shall be the government
share under said agreement." There is no ejusdem generis phrase like "among other things" in
Section 80 that the majority opinion can cling on to save it from constitutional infirmity. DAO 56-99,
the magic wand of the majority opinion, expressly applies only to FTAAs and not to MPSAs. By any
legal yardstick, even by the arguments of the majority opinion, Sections 80 and 84 are void and
unconstitutional.

g. Necessity of Resolving Constitutionality of Sections 39, 80 and 84

The majority opinion states that the constitutionality of Sections 80 and 84 of RA 7942 is not in issue
in the present case. The majority opinion forgets that petitioners have assailed the constitutionality of
RA 7942 and the WMCP FTAA for violation of Section 2, Article XII of the 1987 Constitution.
Petitioner specifically assails the "inequitable sharing of wealth" in the WMCP FTAA, which
petitioners assert is "contrary to Section 1, paragraph 1, and Section 2, paragraph 4, Article XII
of the Constitution."

Section 9.1 of the WMCP FTAA grants WMCP the absolute option, by mere notice to the DENR
Secretary, to convert the FTAA into an MPSA under Section 80. The "sharing of wealth" in Section 80
is "inequitable" and "contrary to x x x Section 2, paragraph 4, Article XII of the Constitution" because
the State will only collect the 2% excise tax in an MPSA. Such a pittance of a sharing will not make
any "real contributions to the economic growth and general welfare of the country" as required in
paragraph 4, Section 2, Article XII of the 1987 Constitution.

Section 39 of RA 7942 also grants foreign FTAA contractors the option, by mere notice to the DENR
Secretary, to convert their FTAAs into MPSAs under Section 80. Necessarily, the constitutionality of
the WMCP FTAA must be resolved in conjunction with Section 80 of RA 7942.

The WMCP FTAA is like a coin with two sides, one side is an FTAA, and the other an MPSA. By mere
notice to the DENR Secretary, WMCP can convert the contract from an FTAA to an MPSA, a copy of
which, complete with all terms and conditions, is annexed to the WMCP FTAA.65 The DENR
Secretary has no option but to sign the annexed MPSA. There are only two conditions to WMCP's
exercise of this option: the reduction of foreign equity in WMCP to 40%, and notice to the DENR
Secretary. The first condition is already fulfilled since all the equity of WMCP is now owned by a
corporation 60% Filipino owned. The notice to the DENR Secretary is solely at the will of WMCP.

What this Court is staring at right now is a dual contract - an FTAA which, by mere notice to the
DENR Secretary, immediately becomes an MPSA. The majority opinion agrees that the provisions of
the WMCP FTAA, which grant a sham consideration to the State, are void. Since the majority
opinion agrees that the WMCP FTAA has a sham consideration, the WMCP FTAA thus lacks
the third element of a valid contract. The majority opinion should declare the WMCP FTAA
void for want of consideration unless the majority opinion treats the contract as an MPSA
under Section 80. Indeed, the only recourse of WMCP to save the validity of its contract is to convert
it into an MPSA.

Thus, with the absence of consideration in the WMCP FTAA, what is actually before this Court is an
MPSA. This squarely puts in issue whether an MPSA is constitutional if the only consideration or
payment to the State is the 2% excise tax as provided in Section 80 of RA 7942.

The basic constitutional infirmity of the WMCP FTAA is the absence of a fair consideration to the
State as owner of the mineral resources. Petitioners call this the "inequitable sharing of wealth." The
constitutionality of the consideration for the WMCP FTAA cannot be resolved without determining the
198

validity of both Sections 80 and 81 of RA 7942 because the consideration for the WMCP FTAA is
anchored on both Sections 80 and 81.

The majority opinion refuses to face the issue of whether the WMCP contract can validly rely on
Section 80 for its consideration. If this issue is not resolved now, then the WMCP FTAA has no
consideration. The majority opinion admits that the consideration in the WMCP FTAA granting the
State 60% share in the mining revenues is a sham and thus void ab initio.

Strangely, the majority opinion claims that the share of the State in the mining revenues is not the
principal consideration of the FTAA. The majority opinion claims that the principal consideration of
the FTAA is the "development" of the minerals by the foreign contractor. The foreign contractor can
bring equipment to the mine site, tunnel the mines, and construct underground rails to bring the
minerals to the surface - in short develop the mines. What will the State and the Filipino people
benefit from such activities unless they receive a share of the mining proceeds? After the minerals are
exhausted, those equipment, tunnels and rails would be dilapidated and even obsolete. Besides,
those equipment belong to the foreign contractor even after the expiration of the FTAA.

Plainly, even a businessman with limited experience will not agree that the principal consideration in
an FTAA, as far as the State and Filipino people are concerned, is the development of the mines. It is
obvious why the majority opinion will not accept that the principal consideration is the share of the
State in the mining proceeds. Otherwise, the majority opinion will have to admit that the WMCP FTAA
lacks the third element of a valid contract - the consideration. This will compel the majority opinion to
admit that the WMCP FTAA is void ab initio.

The only way for the majority opinion to save the WMCP FTAA from nullity is to treat it as an MPSA
and thus apply Section 80 of RA 7942. This puts in issue the constitutionality of Section 80. The
majority opinion, however, refuses to treat the WMCP FTAA as an MPSA. Thus, the WMCP FTAA still
lacks a valid consideration. However, the majority opinion insists that the WMCP FTAA is valid.

If the majority opinion puts the constitutionality of Section 80 in issue, the majority opinion will have to
declare Section 80 unconstitutional. The majority opinion agrees that the 1987 Constitution requires
the State to collect "more than the usual taxes, duties and fees." Section 80 indisputably limits the
State to collect only the excise tax and nothing more.

The equivocal stance of the majority opinion will not put an end to this litigation. Once WMCP
converts its FTAA into an MPSA to avoid paying "more than the usual taxes, duties and fees,"
petitioners will immediately question the validity of WMCP's MPSA as well as the constitutionality of
Section 80. The case will end up again in this Court on the same issue of whether there is a valid
consideration for such MPSA, which necessarily involves a determination of the constitutionality of
Section 80. Clearly, this Court has no recourse but to decide now the constitutionality of Section 80.

As the Solicitor General reported in his Compliance dated 20 October 2004, the DENR has signed
five MPSAs with different parties.66 These five MPSAs uniformly contain the following provision:

Share of the Government - The Government Share shall be the excise tax on mineral
products at the time of removal and at the rate provided for in Republic Act No. 7729
amending Section 151(a) of the National Internal Revenue Code, as amended, as well
as other taxes, duties, and fees levied by existing laws. (Emphasis supplied)

If the constitutionality of Section 80 is not resolved now, these five MPSAs, including the WMCP
FTAA once converted into an MPSA, will remain in limbo. There will be no implementation of these
MPSAs until the Court finally resolves this constitutional issue.

Even if evaded now, the constitutionality of Section 80 will certainly resurface, resulting in a repeat of
this litigation, most probably even between the same parties. To avoid unnecessary delay, this Court
must rule now on the constitutionality of Section 80 of RA 7942.

2. The Constitutional Term Limit Applies to FTAAs


199

Section 3.3 of the WMCP FTAA provides a fixed contract term of 50 years at the option of WMCP.
Thus, Section 3.3 provides:

This Agreement shall be renewed by the Government for a further period of twenty-five
(25) years under the same terms and conditions provided that the Contractor lodges a
request for a renewal with the Government not less than sixty (60) days prior to the expiry of
the initial term of this Agreement and provided that the Contractor is not in breach of any of
the requirements of this Agreement. (Emphasis supplied)

This provision grants WMCP the absolute right to extend the first 25-year term of the FTAA to
another 25-year term upon mere lodging of a request or notice to the Philippine
Government. WMCP has the absolute right to extend the term of the FTAA to 50 years and all that
the Government can do is to acquiesce to the wish of WMCP.

Section 3.3 of the WMCP FTAA is void because it violates Section 2, Article XII of the 1987
Constitution, the first paragraph of which provides:

All lands of the public domain, waters, minerals, coal, petroleum, and other mineral oils, all
forces of potential energy, fisheries, forests or timber, wildlife, flora and fauna, and other
natural resources are owned by the State. With the exception of agricultural lands, all other
natural resources shall not be alienated. The exploration, development, and utilization of
natural resources shall be under the full control and supervision of the State. The State may
directly undertake such activities, or it may enter into co-production, joint venture, or
production-sharing agreements with Filipino citizens, or corporations or associations at least
sixty per centumof whose capital is owned by such citizens. Such agreements may be for a
period not exceeding twenty-five years, renewable for not more than twenty-five years,
and under such terms and conditions as may be provided by law. In cases of water
rights for irrigation, water supply, fisheries, or industrial uses other than the development of
water power, beneficial use may be the measure and limit of the grant. (Emphasis supplied)

The majority opinion, however, makes the startling assertion that FTAAs are not covered by the term
limit under Section 2, Article XII of the 1987 Constitution. The majority opinion states:

I believe that the constitutional term limits do not apply to FTAAs. The reason is that the
above provision is found within paragraph 1 of Section 2 of Article XII, which refers to
mineral agreements – co-production agreements, joint venture agreements and mineral
production sharing agreements - which the government may enter into with Filipino citizens
and corporations, at least 60 percent owned by Filipino citizens. (Emphasis supplied)

If the term limit does not apply to FTAAs because the term limit is found in the first paragraph of
Section 2, then the other limitations in the same first paragraph of Section 2 do not also apply to
FTAAs. These limitations are three: first, that the State owns the natural resources; second, except for
agricultural lands, natural resources shall not be alienated; third, the State shall exercise full control
and supervision in the exploitation of natural resources. Under the majority opinion's
interpretation, these three limitations will no longer apply to FTAAs, leading to patently absurd
results. The majority opinion will also contradict its own admission that even in FTAAs the State must
exercise full control and supervision in the exploitation of natural resources.

Section 2, Article XII of the 1987 Constitution is a consolidation of Sections 8 and 9, Article XIV of
the 1973 Constitution, which state:

Section 8. All lands of public domain, waters, minerals, coal, petroleum and other mineral oils,
all forces of potential energy, fisheries, wildlife, and other natural resources of the Philippines
belong to the State. With the exception of agricultural, industrial or commercial, residential, or
resettlement lands of the public domain, natural resources shall not be alienated, and no
license, concession, or lease for the exploration, or utilization of any of the natural resources
shall be granted for a period exceeding twenty-five years, except as to water rights for
200

irrigation, water supply, fisheries, or industrial uses other than development of water power, in
which cases, beneficial use may be the measure and the limit of the grant.

Section 9. The disposition, exploration, development, exploitation, or utilization of any of the


natural resources of the Philippines shall be limited to citizens of the Philippines, or to
corporations or associations at least sixty per centum of the capital which is owned by such
citizens. The Batasang Pambansa, in the national interest, may allow such citizens,
corporations or associations to enter into service contracts for financial, technical,
management, or other forms of assistance with any foreign person or entity for the
exploration, or utilization of any of the natural resources. Existing valid and binding service
contracts for financial, technical, management, or other forms of assistance are hereby
recognized as such.

Section 9, Article XIV of the 1973 Constitution, a one-paragraph section, contained the provision
reserving the exploration, development and utilization of natural resources to Philippine
citizens or corporations 60% Filipino owned as well as the provision on FTAAs. The provision
on the 25-year term limit was found in the preceding Section 8 of Article XIV. If the 25-year term limit
under the 1973 Constitution did not apply to FTAAs, then it should not also have applied to non-FTAA
mining contracts, an interpretation that is obviously wrong. Thus, the term limit in Section 8, Article
XIV of the 1973 Constitution necessarily applied to both non-FTAA mining contracts and FTAAs in
Section 9.

What the framers of the 1987 Constitution did was to consolidate Sections 8 and 9, Article XIV of the
1973 Constitution into one section, the present Section 2, Article XII of the 1987 Constitution. The
consolidation necessitated re-arranging the sentences and paragraphs without any intention of
destroying their unity and coherence. Certainly, the consolidation did not mean that the FTAAs are no
longer subject to the 25-year term limit. If anything, the consolidation merely strengthened the need,
following the rules of statutory construction, to read and interpret together all the paragraphs, and
even the sentences, of Section 2, Article XII of the 1987 Constitution.

In his book The 1987 Constitution of the Republic of the Philippines: A Commentary, Father
Joaquin G. Bernas, S.J., who was a leading member of the 1986 Constitutional Commission,
discussed the limitations on the exploitation of natural resources. Father Bernas states:

4. Other limitations

Agreements for the exploitation of the natural resources can have a life of only twenty-
five years. This twenty-five year limit dates back to the 1935 Constitution and is considered
to be a "reasonable time to attract capital, local and foreign, and to enable them to recover
their investment and make a profit. The twenty-five year limit on the exploitation of natural
resources is not applicable to "water rights for irrigation, water supply, fisheries, or industrial
uses other than the development of water power." In these cases, "beneficial use may be the
measure and the limit of the grant." But in the case of water rights for water power, the
twenty-five year limit is applicable."67 (Emphasis supplied)

The 1935, 1973 and 1987 Constitutions all limit the exploitation of natural resources to 25-year terms.
They also limit franchises for public utilities, leases of alienable lands of public domain, and water
rights for power development to 25-year terms. If a different term is intended, the Constitution
expressly says so as in water rights for uses other than power development. Under the 1973 and
1987 Constitutions, there is no separate term for FTAAs other than the 25-year term for the
exploitation of natural resources.

The WMCP FTAA draws life from Executive Order No. 279 issued on 25 July 1987 by then President
Corazon C. Aquino when she still exercised legislative powers. Section 1.1 of the WMCP FTAA
expressly states, "This Agreement is a Financial & Technical Assistance Agreement entered
into pursuant to Executive Order No. 279." Section 7 of Executive Order No. 279 provides:
201

Section 7. All provisions of Presidential Decree No. 463, as amended, other existing
mining laws, and their implementing rules and regulations, or parts thereof, which are not
inconsistent with the provisions of this Executive Order, shall continue in force and
effect. (Emphasis supplied)

Section 40 of Presidential Decree No. 463 ("PD 463"), as amended by Presidential Decree No. 1385,
provides:

Section 40. Issuance of Mining Lease Contracts - x x x After the mining claim has been
verified as to its mineral contents and its actual location on the ground as determined through
reports submitted to the Director, the Secretary shall approve and issue the
corresponding mining lease contract, which shall be for a period not exceeding
twenty-five (25) years, renewable upon the expiration thereof for another period not
exceeding twenty-five (25) years under such terms and conditions as provided by
law. (Emphasis supplied)

Thus, at the time of execution of the WMCP FTAA, statutory law limited the term of all mining
contracts to 25-year terms. PD 463 merely implemented the mandate of the 1973 Constitution on the
25-year term limit, which is the same 25-year term limit in the 1987 Constitution. Under Section 7 of
Executive Order No. 279, Section 40 of PD 463 limiting mining contracts to a 25-year term
applies to the WMCP FTAA. Therefore, Section 3.3 of the WMCP FTAA providing for a 50-year
term is void.

Then President Aquino also issued Executive Order No. 211 on 10 July 1987, a bare 17 days before
issuing Executive Order No. 279. Section 3 of Executive Order No. 211 states:

Section 3. The processing, evaluation and approval of all mining applications, declarations of
locations, operating agreements and service contracts as provided for in Section 2 above,
shall be governed by Presidential Decree No. 463, as amended, other existing mining laws,
and their implementing rules and regulations: Provided, However, that the privileges
granted as well as the terms and conditions thereof shall be subject to any and all
modifications or alterations which Congress may adopt pursuant to Section 2, Article
XII of the 1987 Constitution. (Emphasis supplied)

Section 3 of Executive Order No. 211 applies to the WMCP FTAA which was executed on 22 March
1995, more than seven years after the issuance of Executive Order No. 211. Subsequently, Congress
enacted RA 7942 to prescribe new terms and conditions for all mineral agreements. RA 7942 took
effect on 9 April 1995.

RA 7942 governs the WMCP FTAA because Executive Order No. 211 expressly makes mining
agreements like the WMCP FTAA subject to "any and all modifications or alterations which
Congress may adopt pursuant to Section 2, Article XII of the 1987 Constitution." Section 38 of
RA 7942 provides for a 25-year term limit specifically for FTAAs, thus:

Section 38. Term of Financial or Technical Assistance Agreement. — A financial or


technical assistance agreement shall have a term not exceeding twenty-five (25) years
to start from the execution thereof, renewable for not more than twenty-five (25) years
under such terms and conditions as may be provided by law. (Emphasis supplied)

Thus, the 25-year term limit specifically for FTAAs in Section 38 of RA 7942 applies to the WMCP
FTAA. Again, Section 3.3 of the WMCP FTAA providing for a 50-year term is void.

What is clear from the foregoing is that the 25-year statutory term limit on mining contracts is merely
an implementation of the 25-year constitutional term limit, whether under the 1935, 1973 or 1987
Constitutions. The majority opinion's assertion that the 25-year term in the first paragraph of Section
2, Article XII of the 1987 Constitutions does not apply to FTAAs is obviously wrong.

3. Section 112 of RA 7942 Applies to the WMCP FTAA


202

The majority opinion insists that Section 112 of RA 7942 does not apply to the WMCP FTAA. Section
112 provides:

Section 112. Non-impairment of Existing Mining/Quarrying Rights. — All valid and existing
mining lease contracts, permits/licenses, leases pending renewal, mineral production-
sharing agreements granted under Executive Order No. 279, at the date of effectivity of
this Act, shall remain valid, shall not be impaired, and shall be recognized by the
Government: Provided, That the provisions of Chapter XIV on government share in
mineral production-sharing agreement and of Chapter XVI on incentives of this
Act shall immediately govern and apply to a mining lessee or contractor unless the
mining lessee or contractor indicates his intention to the secretary, in writing, not to avail of
said provisions: Provided, further, That no renewal of mining lease contracts shall be made
after the expiration of its term: Provided, finally, That such leases, production-sharing
agreements, financial or technical assistance agreements shall comply with the applicable
provisions of this Act and its implementing rules and regulations. (Emphasis supplied)

Section 112 "immediately" applies the fiscal regime under Section 80 on "mineral production sharing
agreement" to "all valid and existing mining" contracts, including those "granted under Executive
Order No. 279." If Section 112 applies to the WMCP FTAA, then the WMCP FTAA is subject only
to the 2% excise tax under Section 80 as the "total share" of the Philippine Government.

The majority opinion states, "Whether Section 112 may properly apply to co-production or joint
venture agreements, the fact of the matter is that it cannot be made to apply to FTAAs." This
position of the majority opinion is understandable. If Section 112 applies to FTAAs, the majority
opinion would have to rule on the constitutionality of Section 80 of RA 7942. The majority opinion
already agrees that the 1987 Constitution requires the FTAA contractor to pay the State "more than
the usual taxes, duties and fees." If Section 112 applies to FTAAs, the majority opinion would have no
choice but declare unconstitutional Section 80.

Thus, the majority opinion insists that Section 112 "cannot be made to apply to FTAAs." This
insistence of the majority opinion collides with the very clear and plain language of Section
112 of RA 7942 and Section 1.1 of the WMCP FTAA. This insistence of the majority opinion will
lead to absurd results.

First, Section 112 of RA 7942 speaks of "all valid and existing mining" contracts. The phrase "all
valid and existing mining" contracts means the entire or total mining contracts in existence "at the
date of effectivity" of RA 7942 without exception. The word "all" negates any exception. This
certainly includes the WMCP FTAA, unless the majority opinion concedes that the WMCP FTAA is
not a mining contract, or if it is, that it is not a valid contract.

Second, the last proviso of Section 112 itself expressly states that "financial or technical
assistance agreementsshall comply with the applicable provisions of this Act and its
implementing rules and regulations." There is no shadow of doubt whatsoever that Section 112, by
its own plain, clear and indisputable language, commands that FTAAs shall comply with RA 7942. I
truly cannot fathom how the majority opinion can assert that Section 112 cannot apply to FTAAs.

Third, Section 112 expressly refers to Chapters XIV and XVI of RA 7942. Chapter XIV refers to the
"Government Share" and covers Sections 80, 81 and 82 of RA 7942. Section 81, as the majority
opinion concedes, applies to FTAAs. Chapter XVI refers to "Incentives" and covers Section 90 to
94 of RA 7942. Section 90 states that the "contractors in mineral agreements, and financial
technical and assistance agreements shall be entitled to the fiscal and non-fiscal incentives as
provided under Executive Order No. 226 x x x." Clearly, Section 112 applies to FTAAs.

Fourth, Section 1.1 of the WMCP FTAA expressly states, "This Agreement is a Financial &
Technical Assistance Agreement entered into pursuant to Executive Order No. 279." Section
112 states in unequivocal language that "all valid and existing" agreements "granted under
Executive Order No. 279" are immediately placed under the fiscal regime of MPSAs. In short, mining
agreements granted under Executive Order No. 279 are expressly among the agreements
203

included in Section 112 and placed under the fiscal regime prescribed in Section 80. There is no
doubt whatsoever that Section 112 applies to the WMCP FTAA which was "entered into pursuant to
Executive Order No. 279."

Fifth, Section 3 of Executive Order No. 211 expressly subjects all mining contracts executed by the
Executive Department to the terms and conditions of new mining laws that Congress might enact in
the future. Thus, Section 3 of Executive Order No. 211 states:

Section 3. The processing, evaluation and approval of all mining applications, declarations of
locations, operating agreements and service contracts as provided for in Section 2 above,
shall be governed by Presidential Decree No. 463, as amended, other existing mining laws,
and their implementing rules and regulations: Provided, However, that the privileges
granted as well as the terms and conditions thereof shall be subject to any and all
modifications or alterations which Congress may adopt pursuant to Section 2, Article
XII of the 1987 Constitution. (Emphasis supplied)

There is no dispute that Executive Order No. 211, issued prior to the execution of the WMCP FTAA,
applies to the WMCP FTAA. There is also no dispute that RA 7942 took effect after the issuance of
Executive Order No. 211 and after the execution of the WMCP FTAA. Therefore, Section 112 of RA
7942 applies specifically to the WMCP FTAA.

Indeed, it is plain to see why Section 112 of RA 7942 applies to FTAAs, like the WMCP FTAA, that
were executed prior to the enactment of RA 7942. Section 112 is found in Chapter XX of RA 7942 on
"Transitory and Miscellaneous Provisions." The title of Section 112 refers to the "[N]on-impairment of
Existing Mining Quarrying Rights." RA 7942 is the general law governing all kinds of mineral
agreements, including FTAAs. In fact, Chapter VI of RA 7942, covering nine sections, deals
exclusively on FTAAs. The fiscal regime in FTAAs executed prior to the enactment of RA 7942 may
differ from the fiscal regime prescribed in RA 7942. Hence, Section 112 provides the transitory
provisions to resolve differences in the fiscal regimes, ostensibly to avoid impairment of contract
obligations. Clearly, Section 112 applies to FTAAs.

There are no ifs or buts in Section 112. The plain, simple and clear language of Section 112 makes
FTAAs, like the WMCP FTAA, subject to Section 112. We repeat the express words of Section 112 -

(1) "All valid and existing mining lease contracts x x x mineral production-sharing
agreements granted under Executive Order No. 279, at the date of effectivity of this Act
x x x."

(2) the "x x x government share in mineral production- sharing agreement x x x


shall immediately govern and apply to a mining lessee or contractor x x x."

(3) "financial or technical assistance agreements shall comply with the applicable
provisions of this Act and its implementing rules and regulations."

With such clear and unequivocal language, how can the majority opinion blithely state that Section
112 "cannot be made to apply to FTAAs"? It defies common sense, simple logic and plain English
to assert that Section 112 does not apply to FTAAs. It defies the fundamental rule of statutory
construction as repeated again and again in jurisprudence:

Time and time again, it has been repeatedly declared by this Court that where the law speaks
in clear and categorical language, there is no room for interpretation. There is only room for
application.68

For nothing is better settled than that the first and fundamental duty of courts is to apply the
law as they find it, not as they like it to be. Fidelity to such a task precludes construction or
interpretation, unless application is impossible or inadequate without it. 69
204

Where the law is clear and unambiguous, it must be taken to mean exactly what it says and
the court has no choice but to see to it that its mandate is obeyed. 70

If Section 112 of RA 7942 does not apply to FTAAs as the majority opinion asserts, what will
govern FTAAs executed before the enactment of RA 7942, like the WMCP FTAA? Section 112
expressly addresses FTAAs executed before the enactment of RA 7942, requiring these earlier
FTAAs to comply with the provisions of RA 7942 and its implementing rules. Executive Order No. 211,
issued seven years before the execution of the WMCP FTAA, requires all FTAAs subsequently
executed to comply with the terms and conditions of any future mining law that Congress may enact.
That law is RA 7942 which took effect after the execution of the WMCP FTAA.

The majority opinion allows the WMCP FTAA to become sui generis, an FTAA outside the scope of
RA 7942 which expressly governs "all" mining agreements, whether MPSAs or FTAAs. This means
that the WMCP FTAA is not even governed by Section 81 of RA 7942 and its phrase "among other
things," which the majority opinion claims is the authority to subject the WMCP FTAA to the payment
of consideration that is "more than the usual taxes, duties and fees."

This makes the majority opinion's position self-contradictory and inutile. The majority opinion claims
that the WMCP FTAA is subject to the phrase "among other things" in Section 81. At the same time,
the majority opinion asserts that Section 112, which requires earlier FTAAs to comply with Section 81
and other provisions of RA 7942, does not apply to the WMCP FTAA. The majority opinion is caught
in a web of self-contradictions.

This exemption by the majority opinion of the WMCP FTAA from Section 112 is judicial class
legislation.Why is the WMCP FTAA so special that the majority opinion wants it exempted from
Section 112 of RA 7942? Why are only "all" other FTAAs subject to the terms and conditions of RA
7942 and not the WMCP FTAA?

4. Foreign Corporations and Contractors Cannot Hold Exploration Permits

The majority opinion states that "there is no prohibition at all against foreign or local
corporations or contractors holding exploration permits." This is another assertion of the majority
opinion that directly collides with the plain language of the 1987 Constitution.

Section 2, Article XII of the 1987 Constitution expressly reserves to Philippine citizens and
corporations 60% Filipino owned the "exploration, development and utilization of natural
resources." The majority opinion rationalizes its assertion in this manner:

Pursuant to Section 20 of RA 7942, an exploration permit merely grants to a qualified


person the right to conduct exploration for minerals in specified areas. Such a permit
does not amount to an authorization to extract and carry off the mineral resources that
may be discovered. x x x. (Italics in original)

The issue is not whether an exploration permit allows a foreign contractor or corporation to extract
mineral resources, for apparently by its language alone a mere exploration permit does not. There is
no dispute that an exploration permit merely means authority to explore, not to extract. The issue is
whether the issuance of an exploration permit to a foreign contractor violates the constitutional
limitation that only Philippine citizens or corporations 60% Filipino owned can engage in
the "exploration x x x of natural resources."

The plain language of Section 2, Article XII of the 1987 Constitution clearly limits to Philippine citizens
or to corporations 60% Filipino owned the right to engage in the "exploration x x x of natural
resources." To engage in "exploration" is simply to explore, not to develop, utilize or
extract. To engage in exploration one must secure an exploration permit. The mere issuance of the
exploration permit is the authority to engage in the exploration of natural resources.

This activity of exploration, which requires an exploration permit, is a reserved activity not allowed to
foreign contractors or foreign corporations. Foreign contractors and foreign corporations cannot
205

secure exploration permits because they cannot engage in the exploration of natural resources. If, as
the majority opinion asserts, foreign contractors or foreign corporations can secure and hold
exploration permits, then they can engage in the "exploration x x x of natural resources." This
violates Section 2, Article XII of the 1987 Constitution.

Consequently, Section 3(aq) of RA 7942, which provides that "a legally organized foreign-owned
corporation shall be deemed a qualified person for purposes of granting an exploration permit," is void
and unconstitutional.

However, the State may directly undertake to explore, develop and utilize the natural resources. To
do this the State may contract a foreign corporation to conduct the physical act of exploration in the
State's behalf, as in an FTAA. In such a case, the foreign FTAA contractor is merely an agent of the
State which holds the right to explore. No exploration permit is given to the foreign contractor because
it is the State that is directly undertaking the exploration, development and utilization of the natural
resources.

The requirement reserving "exploration x x x of natural resources" to Philippine citizens or to


corporations 60% Filipino owned is not a matter of constitutional whim. The State cannot allow foreign
corporations, except as contractual agents under the full control and supervision of the State, to
explore our natural resources because information derived from such exploration may have national
security implications.

If a Chinese company from the People's Republic of China is allowed to explore for oil and gas in the
Spratlys, the technical information obtained by the Chinese company may only bolster the resolve of
the Chinese Government to hold on to their occupied reefs in the Spratlys despite these reefs being
within the Exclusive Economic Zone of the Philippines. Certainly, we cannot expect the Chinese
company to disclose to the Philippine Government the important technical data obtained from such
exploration.

In Africa, foreign mining companies who have explored the mineral resources of certain countries shift
their support back and forth between government and rebel forces depending on who can give them
better terms in exploiting the mineral resources. Technical data obtained from mineral exploration
have triggered or fueled wars and rebellions in many countries. The right to explore mineral resources
is not a trivial matter as the majority opinion would want us to believe.

Even if the foreign companies come from countries with no territorial dispute with the Philippines, can
we expect them to disclose fully to the Philippine Government all the technical data they obtain on our
mineral resources? These foreign companies know that the Philippine Government will use the very
same data in negotiating from them a higher share of the mining revenues. Why will the foreign
companies give to the Philippine Government technical data justifying a higher share for the
Philippine Government and a lower share for the foreign companies? The framers of the 1935, 1973
and 1986 Constitutions were acutely aware of this problem. That is why the 1987 Constitution not only
reserves the "exploration x x x of natural resources" to Philippine citizens and to corporations 60%
Filipino owned, it also now requires the State to exercise "full control and supervision" over the
"exploration x xx of natural resources."

5. The State is Entitled to 60% Share in the Net Mining Revenues

The majority opinion claims that the Constitution does not require that the State's share in FTAAs or
other mineral agreements should be at least 60% of the net mining revenues. Thus, the majority
opinion states that "the Charter did not intend to fix an iron-clad rule on the 60 percent share,
applicable to all situations at all times and in all circumstances."

The majority opinion makes this claim despite the express admission by intervenor CMP and
respondent WMCP that the State, as owner of the natural resources, is entitled to 60% of the net
mining revenues. The intervenor CMP admits that under an FTAA, the Philippine
Government "stands in the place of the 60% Filipino owned company" and hence must retain
60% of the net income. Thus, intervenor CMP concedes that:
206

x x x In other words, in the FTAA situation, the Government stands in the place of the
60% Filipino-owned company, and the 100% foreign-owned contractor company takes all
the risks of failure to find a commercially viable large-scale ore body or oil deposit, for
which the contractor will get 40% of the financial benefits.71 (Emphasis supplied)

As applied to the WMCP FTAA, intervenor CMP asserts that the "contractor's stipulated share
under the WMCP FTAA is limited to a maximum of 40% of the net production."72 Intervenor CMP
further insists that "60% of its (contractor's) net returns from mining, if any, will go to the
Government under the WMCP FTAA."73

Like intervenor CMP, respondent WMCP also maintains that under an FTAA, the State
is "guaranteed" a 60% share of the foreign contractor's Net Mining Revenues. Respondent WMCP
admits that:

In other words, the State is guaranteed a sixty per centum (60%) share of the Mining
Revenues, or 60% of the actual fruits of the endeavor. This is in line with the intent
behind Section 2 of Article XII that the Filipino people, as represented by the State,
benefit primarily from the exploration, development, and utilization of the Philippines'
natural resources.

Incidentally, this sharing ratio between the Philippine Government and the Contractor
is also in accordance with the 60%-40% equity requirement for Filipino-owned
corporations in Paragraph 1 of Section 2 of Article XII.74 (Emphasis supplied)

In short, the entire mining industry, as represented by intervenor CMP, is willing to pay the State a
share equivalent to 60% of the net mining revenues. Even the foreign contractor WMCP agrees to
pay the State 60% of its net mining revenues, albeit dishonestly.

However, the majority opinion refuses to accept that the State is entitled to what the entire mining
industry is willing to pay the State. Incredibly, the majority opinion claims that "there is no
independent showing that the taking of at least 60 percent share in the after-tax income of a
mining company operated by a foreign contractor is fair and reasonable under most if not all
circumstances." Despite the willingness of the entire mining industry to pay the State a 60% share
without exception, the majority opinion insists that such sharing is not fair and reasonable to the
mining industry "under most if not all circumstances." What is the basis of the majority opinion in
saying this when the entire mining industry already admits, concedes and accepts that the State is
entitled, without exception, to 60% of the net mining revenues?

Oddly, the majority opinion cites only the personal experience of the ponente, who had previously
"been engaged in private business for many years." The majority opinion even states, in insisting that
the State should receive less than 60% share, that "[F]airness is a credo not only in law, but also
in business." The majority opinion cannot be more popish than the Pope. The majority
opinion ponente's business judgment cannot supplant the unanimous business judgment of the entire
mining industry, as manifested by intervenor CMP before this Court. What is obvious is that it is not
fair to deprive the Filipino people, many of whom live in hand to mouth existence, of what is legally
their share of the national patrimony, in light of the willingness of the entire mining industry to pay the
Filipino people their rightful share.

The majority opinion gives a "simplified illustration" to show that the State does not deserve a 60%
share of the net proceeds from mining revenues. The majority opinion states:

x x x Let us base it on gross revenues of, say, P500. After deducting operating expenses, but
prior to income tax, suppose a mining makes a taxable income of P100. A corporate income
tax of 32 percent results in P32 of taxable income going to the government, leaving the
mining firm with P68. Government then takes 60 percent thereof, equivalent to P40.80,
leaving only P27.20 for the mining firm.
207

The majority opinion's "simplified illustration" is indeed too simplified because it does not even
consider the exploration, development and capital expenses. The majority opinion's "simplified
illustration" deducts from gross revenues only "operating expenses." This is an egregious error that
makes this "simplified illustration" misleading. Exploration, development and other capital expenses
constitute a huge part of the deductions from gross revenues. In the early years of commercial
production, the exploration, development and capital expenses, if not subject to a cap or limitation,
can wipe out the gross revenues.

The majority opinion's operating expenses are not even taken from mining industry rates. One can
even zero out the taxable income by simply jacking up the operating expenses. A "simplified
illustration" of an income statement of an operating mining company, omitting the deduction of
amortized capital expenses, serves no purpose whatsoever. What is important is the return on the
investment of the foreign contractor. The absolute amount that goes to the contractor may be smaller
than what goes to the State. However, the amount that goes to the contractor may be a hundred
times its investment. This can only be determined if the capital expenditures of the contractor are
taken into account.

Under an FTAA, the State is directly undertaking the exploitation of mineral resources. The net
proceeds are not subject to income tax since there is no separate taxable entity. The State is an entity
but not a taxable corporate entity. The State does not pay income tax to itself, and even if it does, it is
just a book entry since it is the payor and payee at the same time. Only the 40% share of the FTAA
contractor is subject to the 32% corporate income tax. On this score alone, the majority opinion's
"simplified illustration" is wrong.

Intervenor CMP and respondent WMCP are correct in anchoring on Section 2, Article XII of the 1987
Constitution their admission that the State is entitled to 60% of the net mining revenues. Their
common position is based on the Constitution, existing laws and industry practice.

First, the State owns the mineral resources. To the owner of the mineral resources belongs the
income from any exploitation of the mineral resources. The owner may share its income with the
contractor as compensation to the contractor, which is an agent of the owner. The industry practice is
the owner receives an equal or larger share of the income as against the share of the contractor or
agent.

In the Occidental-Shell FTAA covering Malampaya, where the contractor contributed all the capital
and technology, the State receives 60% of the net proceeds. In addition, Occidental-Shell's 40%
share is subject to the 32% Philippine income tax. Occidental-Shell's US$2 billion investment75 in
Malampaya is by far the single biggest foreign investment in the Philippines. The offshore Malampaya
gas extraction is also by far more capital intensive and riskier than land-based mineral extraction.
Over the 20-year life of the natural gas reserves, the State will receive US$8-10 billion76 from its share
in the Occidental-Shell FTAA.

In Consolidated Mines, Inc. v. Court of Tax Appeals,77 a case decided under the 1973
Constitution, Consolidated Mines, the concessionaire of the mines, shared equally the net mining
income with Benguet Consolidated Mines, the mining operator or contractor. Thus, as quoted
in Consolidated Mines, the agreement between the concessionaire and operator stated:

X. After Benguet has been fully reimbursed for its expenditures, advances and disbursements
as aforesaid the net profits from the operation shall be divided between Benguet and
Consolidated share and share alike, it being understood however, that the net profits as the
term is used in this agreement shall be computed by deducting from gross income all
operating expenses and all disbursements of any nature whatsoever as may be made in
order to carry out the terms of this agreement. (Emphasis supplied)

Incidentally, in Consolidated Mines the State did not receive any share in the net mining income
because of the "license, concession or lease" system under the 1935 and 1973 Constitutions. The
State and the Filipino people received only taxes, duties and fees.
208

Second, the State exercises "full control and supervision" over the exploitation of mineral
resources. "Full control" as used in the Constitution means more than ordinary majority control. In
corporate practice, ordinary control of a corporation means a simple majority control, or at least 50%
plus one of the total voting stock. In contrast, full or total control means two-thirds of the voting
stock, which enables the owner of the two-thirds equity to amend any provision in the charter of the
corporation. However, since foreigners can own up to 40% of the equity of mining companies, "full
control" cannot exceed the control corresponding to the State's 60% equity. Thus, the State's share in
the net proceeds of mining companies should correspond to its 60% interest and control in mining
companies.

Third, Section 2, Article XII of the 1987 Constitution requires that the FTAA must make "real
contributions to the economic growth and general welfare of the country." As respondent
WMCP aptly admits, "the intent behind Section 2 of Article XII (is) that the Filipino people, as
represented by the State, (shall) benefit primarily from the exploration, development, and
utilization of the Philippines' natural resources." For the Filipino people to benefit primarily from
the exploitation of natural resources, and for FTAAs to make real contributions to the national
economy, the majority of the net proceeds from mining operations must accrue to the State.

Fourth, the 1987 Constitution ordains the State to "conserve and develop our patrimony." The
nation's mineral resources are part of our national patrimony. The State can "conserve" our mineral
resources only if the majority of the net proceeds from the exploitation of mineral resources accrue to
the State.

In sum, only the majority opinion refuses to accept that the State has a right to receive at least 60% of
the net proceeds from mining operations. The principal parties involved in this case do not object that
the State shall receive such share. The entire mining industry and respondent WMCP admit that the
State is entitled to a 60% share of the net proceeds. The State, represented by the Government, will
certainly not object to such share.

More than anything else, the intent and language of the 1987 Constitution require that the State
receive the bulk of the income from mining operations. Only Congress, through a law, may allow a
share lesser than 60% if certain compelling conditions are present. Congress may authorize the
President to make such determination subject to standards and limitations that Congress shall
prescribe.

The majority opinion wants to give the President the absolute discretion to determine the State's
share from mining revenues. The President will be hard put accepting anything less than 60% of the
net proceeds. If the President accepts less than 60%, the President is open to a charge of entering
into a manifestly and grossly disadvantageous contract to the Government because the entire mining
industry, including WMCP, has already agreed to pay 60% of the net proceeds to the State. The only
way to avoid this is for Congress to enact a law providing for the conditions when the State may
receive less than 60% of the net proceeds.

Conclusion

Let us assume that one of the Justices of this Court is the owner of mineral resources – say gold
reserves. A foreigner offers to extract the gold and pay for all development, capital and operating
expenses. How much will the good Justice demand as his or her share of the gold extracted by the
foreigner? If the Justice follows the Malampaya precedent, he or she will demand a 60% share of the
net proceeds. If the Justice follows the manifestation of intervenor CMP and respondent WMCP
before this Court, he or she will also demand a 60% share in the net proceeds. If the Justice follows
the Consolidated Mines precedent, he or she will demand no less than 50% of the net proceeds. In
either case, the 2% excise tax on the gold extracted is part of the operating expenses to be paid by
the foreigner but deducted from the gross proceeds.

Now, under the Regalian doctrine the State, not the Justice, owns the gold reserves. How much
should the State demand from the foreigner as the State's share of the gold that is extracted? If we
209

follow Sections 39, 80, 81, 84 and 112 of RA 7942, the State will receive only 2% excise tax as
its "total share" from the gold that is extracted.

Is this fair to the State and the Filipino people, many of whom live below the poverty line? Is this what
the 1987 Constitution mandates when it says that (a) the State must conserve and develop the
nation's patrimony, (b) the State owns all the natural resources, (c) the State must exercise full control
and supervision over the exploitation of its natural resources, and (d) FTAAs must make real
contributions to the national economy and the general welfare?

How this Court decides the present case will determine largely whether our country will remain poor,
or whether we can progress as a nation. Based on NEDA's estimates, the total mineral wealth of the
nation is P47 trillion, or US$840 billion. This is 15 times more than our US$56 billion foreign debt. Can
this Court in conscience agree that the State will receive only 2% of the P47 trillion mineral
wealth of the nation?

In Miners Association, this Court ruled that the 1987 Constitution has abandoned the old system of
"license, concession or lease" and instead installed full State control and supervision over the
exploitation of natural resources. No amount of dire warnings or media publicity should intimidate this
Court into resurrecting the old and discredited system that has caused the denudation of almost all of
the nation's virgin forests without any visible benefit to the Filipino people.

The framers of the 1987 Constitution have wisely instituted the new system to prevent a repeat of the
denudation of our forestlands that did not even make any real contribution to the economic growth of
the nation. This Court must do its solemn duty to uphold the intent and letter of the Constitution and,
in the words of the Preamble of the 1987 Constitution, "conserve and develop our patrimony" for the
benefit of the Filipino people.

This Court cannot trivialize the Filipino people's right to be the primary beneficiary of the nation's
mineral resources by ruling that the phrase "among other things" is sufficient to insure that FTAAs
will "make real contributions to the economic growth and general welfare of the country." This
Court cannot tell the Filipino people that the phrase "among other things" is sufficient to "preserve
and develop the national patrimony." This Court cannot tell the Filipino people that the phrase
"among other things" means that they will receive the bulk of mining revenues.

This Court cannot tell the Filipino people that Congress deliberately used the phrase "among other
things" to guarantee that the Filipino people will receive their equitable share from mining revenues
of foreign contractors. This Court cannot tell the Filipino people that with the phrase "among other
things," this Court has protected the national interest as mandated by the 1987 Constitution.

I therefore vote to deny the motions for reconsideration. I vote to declare unconstitutional Section
3(aq), Section 39, Section 80, the second paragraph of Section 81, the proviso in Section 84, and the
first proviso in Section 112 of RA 7942 for violation of Section 2, Article XII of the 1987 Constitution. In
issuing the rules to implement these void provisions of RA 7942, DENR Secretary Victor O. Ramos
gravely abused his discretion amounting to lack or excess of jurisdiction.

I also vote to declare unconstitutional the present WMCP FTAA for violation of the same Section 2,
Article XII of the 1987 Constitution. However, WMCP may negotiate with the Philippine Government
for a new mineral agreement covering the same area consistent with this Decision.

DISSENTING OPINION

CARPIO MORALES, J.:

Regrettably, a majority of the members of this Court has voted to reverse its January 27, 2004
Decision in La Bugal-B'Laan Tribal Association, Inc. v. Ramos1 by which it declared certain
210

provisions2 of the Mining Act of 19953 on Financial or Technical Assistance Agreements (FTAAs), the
related provisions of Department of Environment and Natural Resources Administrative Order 96-40
(DAO No. 96-40), and the March 22, 1995 Financial and Technical Assistance Agreement (FTAA)
executed between the Government of the Republic of the Philippines and WMC Philippines, Inc.
(WMCP) in violation of Section 2, Article XII of the Constitution.

Because I find that: (1) the "agreements … involving either technical or financial assistance"
contemplated by the fourth paragraph of Section 2, Article XII of the 1987 Constitution are distinct and
dissimilar from the "service contracts" under the 1973 Constitution; and (2) these certain provisions of
the Mining Act, its implementing rules, and the WMCP FTAA unconstitutionally convey beneficial
ownership and control over Philippine mineral and petroleum resources to foreign contractors, I most
respectfully dissent.

Antecedents

By motion, private respondent WMCP seeks a reconsideration of this Court's Decision, it arguing
essentially that FTAAs are the same as service contracts which were sanctioned under the 1973
Constitution.

By Resolution of June 22, 2004, this Court, upon motion,4 impleaded Philippine Chamber of Mines
(PCM), as respondent-in-intervention. Intervenor PCM argues that the "agreements" referred to in
paragraph 4 of Section 2, Article XII of the Constitution were intended to involve or include the
"service contracts" provided for in the 1973 Constitution.

The parties were, on June 29, 2004, heard on oral arguments during which two major issues were
tackled: first, the proper interpretation of the phrase "agreements… involving either technical or
financial assistance" in Section 2, Article XII of the Constitution, and second, mootness.

Thereafter, the parties submitted their respective memoranda, as required by Resolution of this Court.
However, despite the verbal request of Associate Justice Artemio V. Panganiban during the oral
arguments,5 intervenor PCM failed to submit along with its memorandum any documents to establish
international mining practices, particularly in developing countries.

Issues for Resolution

The majority opinion holds that the resolution of the Motions for Reconsideration in this case should
be confined to the issues taken up during the oral arguments on June 29, 2004. These were: (1) the
proper interpretation of the phrase "agreements… involving either technical or financial assistance" in
Section 2, Article XII of the Constitution, and (2) mootness.

It further holds that the issue of whether the Mining Act and the WMCP FTAA are manifestly
disadvantageous to the government could not be passed upon because the same was supposedly
not raised in the original petition.

These rulings, while well intentioned, cannot be accepted.

First, there is no rule of procedure, whether in Rule 52 or elsewhere, which restricts the resolution of a
case to the issues taken up in the oral arguments. The reason is obvious. The issues for resolution in
any given case are determined by the conflicting arguments of the parties as set forth in their
pleadings. On the other hand, the matters to be taken up in an oral argument may be limited, by order
of the court, to only such points as the court may deem necessary. Thus, Section 1 of Rule 49
provides:

Section 1. When allowed. – At its own instance or upon motion of a party, the court may hear
the parties in oral argument on the merits of a case, or on any material incident in
connection therewith.
211

The oral argument shall be limited to such matters as the court may specify in its order
or resolution(Emphasis supplied)

A narrow delimitation of matters to be taken up during oral argument is a matter of practical necessity
since often not all the relevant issues can be thoroughly discussed without unduly imposing on the
time of the Court. However, unlike a pre-trial order,6 the delimitation does not control or limit the
issues to be resolved. These issues may be subject matter of the parties' memoranda, as in this case.

Second, as noted in the Decision,7 the issue of whether the Mining Act and the WMCP FTAA afford
the State a just share in the proceeds of its natural resources was in fact raised by the petitioners, viz:

Petitioners claim that the DENR Secretary acted without or in excess of jurisdiction:

x x x in signing and promulgating DENR Administrative Order No. 96-40 implementing


Republic Act No. 7942, the latter being unconstitutional in that it allows fully foreign owned
corporations to explore, develop, utilize and exploit mineral resources in a manner contrary to
Section 2, paragraph 4, Article XII of the Constitution;

II

x x x in signing and promulgating DENR Administrative Order No. 96-40 implementing


Republic Act No. 7942, the latter being unconstitutional in that it allows the taking of private
property without the determination of public use and for just compensation;

III

x x x in signing and promulgating DENR Administrative Order No. 96-40 implementing


Republic Act No. 7942, the latter being unconstitutional in that it violates Sec. 1, Art. III of the
Constitution;

IV

x x x in signing and promulgating DENR Administrative Order No. 96-40 implementing


Republic Act No. 7942, the latter being unconstitutional in that it allows enjoyment by foreign
citizens as well as fully foreign owned corporations of the nation's marine wealth contrary to
Section 2, paragraph 2 of Article XII of the Constitution;

x x x in signing and promulgating DENR Administrative Order No. 96-40 implementing


Republic Act No. 7942, the latter being unconstitutional in that it allows priority to foreign and
fully foreign owned corporations in the exploration, development and utilization of mineral
resources contrary to Article XII of the Constitution;

VI

x x x in signing and promulgating DENR Administrative Order No. 96-40 implementing


Republic Act No. 7942, the latter being unconstitutional in that it allows the inequitable
sharing of wealth contrary to Sections [sic] 1, paragraph 1, and Section 2, paragraph
4[,] [Article XII] of the Constitution;

VII

x x x in recommending approval of and implementing the Financial and Technical Assistance


Agreement between the President of the Republic of the Philippines and Western Mining
212

Corporation Philippines Inc. because the same is illegal and unconstitutional. 8 (Emphasis and
underscoring supplied)

Indeed, this Court expressly passed upon this issue in the Decision when it held that:

With the foregoing discussion in mind, this Court finds that R.A. No. 7942 is invalid insofar as
said Act authorizes service contracts. Although the statute employs the phrase "financial and
technical agreements" in accordance with the 1987 Constitution, it actually treats these
agreements as service contracts that grant beneficial ownership to foreign
contractors contrary to the fundamental law.9 (Emphasis and underscoring supplied)

Moreover, the issue of whether the State is deprived of its just share in the proceeds from mining was
touched upon by the parties in their memoranda. Thus, respondent WMCP argues that:

Section 10.2 (a) of the COLUMBIO FTAA does not prohibit the State from partaking of the fruits
of the exploration. In fact, Section 7.7 of the COLUMBIO FTAA provides:

"7.7 Government Share

From the Commencement of Commercial Production, the Contractor shall pay a


government share of sixty per centum (60%) of Net Mining Revenues, calculated in
accordance with the following provisions (the "Government Share"). The Contractor
shall be entitled to retain the balance of all revenues from the Mining Operations."

In other words, the State is guaranteed a sixty per centum (60%) share of the Net Mining
Revenues, or 60% of the actual fruits of the endeavor. This is in line with the intent behind
Section 2 of Article XII that the Filipino people, as represented by the State, benefit
primarily from the exploration, development, and utilization of the Philippines' natural
resources. 10 (Emphasis and underscoring supplied)

while the petitioners, for their part, claim:

For instance, government share is computed on the basis of net mining revenue. Net mining
revenue is gross mining revenue less, among others, deductible expenses. Some of the
allowable deductions from the base amount to be used to compute government share
are suspicious. The WMCP FTAA contract, for instance, allows expenditures for
development "outside the Contract Area," consulting fees for work done "outside the
Philippines," and the "establishment and administration of field offices including administrative
overheads incurred within and outside the Philippines."

xxx

One mischief inherent in past service contracts was the practice of transfer pricing. UNCTAD
defines this as the "pricing of transfers of goods, services and other assets within a TNC
network." If government does not control the exploration, development and utilization
of natural resources, then the intra-transnational corporation pricing of expenditures
may not become transparent. 11 (Emphasis supplied; footnotes omitted)

In fine, the majority opinion skirts an issue raised in the original Petition for Prohibition and
Mandamus, passed upon in its Decision of January 27, 2004 and argued by the parties in the present
Motion for Reconsideration.

Instead, I find that the myriad arguments raised by the parties may be grouped according to two broad
categories: first, the arguments pertaining to the constitutionality of FTAA provisions of the Mining Act;
and second, those pertaining to the validity of the WMCP FTAA. Within these categories, the following
issues are submitted for resolution: (1) whether in invalidating certain provisions of the Mining Act a
non-justiciable political question is passed upon; (2) whether the FTAAs contemplated in Section 2,
213

Article XII of the 1987 Constitution are identical to, or inclusive of, the "service contracts" provided for
in the 1973 Constitution; (3) whether the declaration of the unconstitutionality of certain provisions of
the Mining Act should be reconsidered; (4) whether the question of validity of the WMCP FTAA was
rendered moot before the promulgation of the Decision; and (5) whether the decision to declare the
WMCP FTAA unconstitutional and void should be reconsidered.

Following the foregoing framework of analysis, I now proceed to resolve the issues raised in the
motion for reconsideration.

Constitutionality of the Philippine Mining Act of 1995

The issues presented constitute


justiciable questions.

Contrary to the posture of respondent WMCP, this Court did not tread on a political question in
rendering its Decision of January 27, 2004.

The Constitution delineates the parameters of the powers of the legislative, the executive and the
judiciary.12Whether the first and second great departments of government exceeded those
parameters is the function of the third.13 Thus, the Constitution defines judicial power to include "the
duty… to determine whether or not there has been a grave abuse of discretion amounting to lack or
excess of jurisdiction on the part of any branch or instrumentality of the Government." 14

Judicial power does not extend to political questions, which are concerned with issues dependent
upon the wisdom, not the legality, of a particular measure.15 The reason is that, under our system of
government, policy issues are within the domain of the political branches of government and of the
people themselves as the repository of all state power.16 In short, the judiciary does not settle policy
issues.17

The distinction between a truly political question and an ostensible one lies in the answer to the
question of whether there are constitutionally imposed limits on powers or functions conferred upon
political bodies.18 If there are constitutionally imposed limits, then the issue is justiciable, and a court
is duty-bound to examine whether the branch or instrumentality of the government properly acted
within those limits.19

Respondent WMCP argues that the "exploration, development, and utilization of natural resources
are matters of policy, in other words, political matters or questions," over which this Court has no
jurisdiction.

Respondent is mistaken. The questions involved in this case are not political. The provisions of
paragraph 4, Section 2 of Article XII of the Constitution, including the phrase "agreements… involving
either technical or financial assistance," incorporate limitations 20 on the scope of such agreements or
FTAAs. Consequently, they constitute limitations on the powers of the legislative to determine their
terms, as well as the powers of the Executive to enter into them. In its Decision, this Court found that,
by enacting the objectionable portions of the Mining Act and in entering into the subject FTAA, the
Congress and the President went beyond the constitutionally delimited scope of such agreements and
thereby transgressed the boundaries of their constitutional powers.

The "agreements" contemplated in paragraph 4, Section 2,


Article XII of the Constitution are distinct and dissimilar from the old "service contracts."

The majority and respondents share a common thesis: that the fourth paragraph of Sec. 2, Article XII
contemplates not only financial or technical assistance but, just like the service contracts which were
allowed under the 1973 Constitution, management assistance as well.
214

The constitutional provision in dispute reads:

Art. XII

National Economy and Patrimony

xxx

Sec. 2. All lands of the public domain, waters, minerals, coal, petroleum, and other mineral
oils, all forces of potential energy, fisheries, forests or timber, wildlife, flora and fauna, and
other natural resources are owned by the State. With the exception of agricultural lands, all
other natural resources shall not be alienated. The exploration, development, and utilization
of natural resources shall be under the full control and supervisionof the State. The State
may directly undertake such activities or it may enter into co-production, joint venture, or
production-sharing agreements with Filipino citizens, or corporations or associations at least
sixty per centum of whose capital is owned by such citizens. Such agreements may be for a
period not exceeding twenty-five years, renewable for not more than twenty-five years, and
under such terms and conditions as may be provided by law. In cases of water rights for
irrigation, water supply, fisheries, or industrial uses other than the development of water
power, beneficial use may be the measure and limit of the grant.

The State shall protect the nation's marine wealth in its archipelagic waters, territorial sea,
and exclusive economic zone, and reserve its use and enjoyment exclusively to Filipino
citizens.

The Congress may, by law, allow small-scale utilization of natural resources by Filipino
citizens, as well as cooperative fish farming, with priority to subsistence fishermen and fish
workers in rivers, lakes, bays, and lagoons.

The President may enter into agreements with foreign-owned


corporations involving either technical or financial assistance for large-scale
exploration, development, and utilization of minerals, petroleum, and other mineral oils
according to the general terms and conditions provided by law, based on real
contributions to the economic growth and general welfare of the country. In such
agreements, the State shall promote the development and use of local scientific and
technical resources.

The President shall notify the Congress of every contract entered into in accordance
with this provision, within thirty days from its execution. (Emphasis and underscoring
supplied)

Its counterpart provision in Article XIV of the 1973 Constitution authorized "service contracts" as
follows:

Sec. 9. The disposition, exploration, development, exploitation, or utilization of any of the


natural resources of the Philippines shall be limited to citizens, or to corporations or
associations at least sixty per centum of which is owned by such citizens. The Batasang
Pambansa, in the national interest, may allow such citizens, corporations or
associations to enter into service contracts for financial, technical, management, or
other forms of assistance with any person or entity for the exploration, development,
exploration, or utilization of any of the natural resources. Existing valid and binding
service contracts for financial, technical, management, or other forms of assistance are
hereby recognized as such. (Emphasis and underscoring supplied)

Respondent WMCP contends that the fourth paragraph of Section 2 is an exception to the rule that
participation in the country's natural resources is reserved to Filipinos. 21 It hastens to add, however,
that the word "may" therein is permissive not restrictive; 22 and that consistent with the provision's
permissive nature, the word "involving" therein should be construed to mean "to include," such that
215

the assistance by foreign corporations should not be confined to technical or financial, but also to
management forms.23 And it notes that the Constitution used "involving" instead of such restrictive
terms as "solely," "only," or "limited to."24

To the Office of the Solicitor General (OSG), the intent behind the fourth paragraph is to prevent the
practice under the 1973 Constitution of allowing foreigners to circumvent the capitalization
requirement,25 as well as to address the absence of a governing law that led to the abuse of service
contracts.26 The phrase "technical or financial" is merely for emphasis, the OSG adds, that it is
descriptive, not definitive, of the forms of assistance that the State needs and which foreign
corporations may provide in the large-scale exploration, development and utilization of the specified
resources.27 Furthermore, the OSG contends that the denomination of the subject FTAA as a
"financial and technical assistance agreement" is a misnomer and should more properly be called
"agreements for large-scale exploration, development, and utilization of minerals, petroleum, and
other mineral oils."28 It argues that the President has broad discretion to enter into any agreement,
regardless of the scope of assistance, with foreign corporations.29 Driving its point, the OSG poses: If
the framers of the Constitution intended to limit the service of foreign corporations to "passive
assistance," such as simple loan agreements, why confine them to large-scale ventures?30 Why does
the Constitution require that such agreements be based on real contributions to economic growth and
general welfare of the country?31 Why the condition in the last paragraph of Section 2 that the
President report to Congress?32 Finally, the OSG asserts that these requirements would be
superfluous if the assistance to be rendered were merely technical or financial. 33 And that it would
make more sense if the phrase "agreements… involving technical or financial assistance" were
construed to mean the same concept as the service contracts under the 1973 Constitution.

The OSG's contentions are complemented by intervenor PCM which maintains that the FTAA "is an
agreement for [the] rendition of a whole range of services of an integrated and comprehensive
character, ranging from discovery through development and utilization and production of minerals or
petroleum by the foreign-owned corporation."34 In fine, intervenor posits that the change in
phraseology in the 1987 Constitution does not relate to the substance of the agreement,35 otherwise,
the State itself would be compelled to conduct the exploration, development and utilization of natural
resources, ventures that it is ill-equipped to undertake.36

Primary Concepts in Article XII of the Constitution

Before passing upon the foregoing arguments and for better clarity, it may be helpful to first examine
the concepts of (a) "beneficial ownership," (b) "full control and supervision," and (c) "real contributions
to the economic growth and general welfare of the country" which are at the heart of Section 2, Article
XII of the Constitution.

Beneficial Ownership

Beneficial ownership, as the plain meaning of the words implies, refers to the right to the gains,
rewards and advantages generated by the property.37

The concept is not new, but in fact is well entrenched in the law of trusts. 38 Thus, while the trustee
holds the legal title to or ownership of the property entrusted to him, he is nevertheless not the
beneficial owner. Rather, he holds and administers the property for the benefit of another, called the
beneficiary or the cestui que trust. Hence, the profits realized from the administration and
management of the property by the trustee, who is the "naked owner," less any lawful fees due to the
latter, accrue to the cestui que trust, who is the "beneficial" or "equitable" owner.39

The foregoing concepts are directly applicable to the statement in Section 2, Article XII of the
Constitution that "[a]ll lands of the public domain, waters, minerals, coal, petroleum, and other mineral
oils, all forces of potential energy, fisheries, forests or timber, wildlife, flora and fauna, and
other natural resources are owned by the State."

The words "owned" and "State" should both be understood on two levels. "Owned" or "ownership"
refers to both the legal title to and the beneficial ownership of the natural resources. Similarly, "State"
216

should be understood as denoting both the body politic making up the Republic of the Philippines, i.e.,
the Filipino people, as well as the Government which represents them and acts on their behalf.

Thus, the phrase "natural resources are owned by the State" simultaneously vests the legal title to the
nation's natural resources in the Government, and the beneficial ownership of these resources in the
sovereign Filipino people, from whom all governmental authority emanates.40

On this point, petitioners and respondent WMCP appear to be in rare agreement. Thus, petitioners, in
their Memorandum state:

xxx With respect to exploration, development and utilization of mineral resources, the State
should not merely be concerned about passing laws. It is expected that it holds these
natural resources covered in Article XII, Section 2 in dominium and in trust for [the]
Filipino people.41 (Emphasis and underscoring supplied; italics in the original)

Respondent WMCP is even more emphatic:

The Regalian Doctrine, as embodied under the Constitution, is a recognition that sovereignty
resides in the Filipino people, and the prime duty of government or the State is to serve and
protect the people. Thus, the ownership of natural resources by the State under Section
2, Article XII of the Constitution is actually a beneficial trust in favor of the Filipino
people.

Stated differently, it is the Filipino people who own the nation's natural resources, and
the State is merely the guardian-in-trust therof.42 (Emphasis and underscoring supplied;
italics in the original; citations omitted)

Clearly, in the exploration, development and utilization of the nation's natural resources, the
Government is in a position analogous to a trustee, holding title to and managing these resources for
the benefit of the Filipino people, including future generations.43 As the trustee of the sovereign, the
Government has a fiduciary duty to ensure that the gains, rewards and advantages generated by the
Philippines' natural resources accrue to the benefit of the Filipino people. Corollary to this, the
Government cannot, without violating its sacred trust, enter into any agreement or arrangement which
effectively deprives the Filipino people of their beneficial ownership of these resources – e.g., when it
enters into an agreement whereby the vast majority of the resources, or the profit generated from the
resources, is bargained away in favor of a foreign entity.

Full Control and Supervision

In the context of its role as trustee, the Government's "full control and supervision" over the
exploration, development and utilization of the nation's natural resources, in its most basic and
fundamental sense, is accomplished by maintaining a position whereby it can carry out its fiduciary
duty to protect the beneficial interest of its cestui que trust in these resources.

Significantly, Section 2, Article XII of the Constitution provides that the Government may undertake
the exploration, development and utilization of these resources by itself or together with a third
party.44 In the first case, where no third party is involved, the Government's "full control and
supervision" over the resources is easily achieved. In the second case, where the third party may
naturally be expected to seek participation in the operation of the venture and ask for compensation in
proportion to its contribution(s), the Government must still maintain a position vis-à-vis its third party
partner whereby it can adequately protect the interest of the Filipino people, who are the beneficial
owners of the resources.

By way of concrete example, the Government may enter into a joint venture agreement45 with a third
party to explore, develop or utilize certain natural resources through a jointly owned corporation,
wherein the government has the controlling interest. Under this arrangement, the Government would
clearly be in a position to protect the interest of the beneficial owners of the natural resources.
217

In the alternative, as suggested by the OSG,46 the Government may be allowed one or more directors
(holding nominal shares) on the governing board and executive committee(s) of the private
corporation contracted to undertake mining activities in behalf of the government. Depending on the
by-laws of the private corporation, strategic representation of the Government in its governing board
and executive committee(s) may afford sufficient protection to the interest of the people.

However, Section 2, Article XII of the Constitution does not limit the options available to the
Government, when dealing with prospective mining partners, to joint ventures or representation in the
contractor's board of directors. To be sure, the provision states that the Government may enter into
"co-production, joint venture, or production-sharing agreements with Filipino citizens, or corporations
or associations," or, for large scale exploration, development and utilization, "agreements with foreign-
owned corporations involving either technical or financial assistance." But whatever form the
agreement entered into by the Government and its third party partner(s) may take, the same must
contain, as an absolute minimum, provisions that ensure that the Government can effectively
perform its fiduciary duty to safeguard the beneficial interest of the Filipino people in their natural
resources, as mandated by the Constitution.

Real Contributions to the Economy


and the General Welfare of the Country

Section 2, Article XII likewise requires that "agreements … involving financial or technical assistance"
be "based on real contributions to the economic growth and general welfare of the country." This
provision articulates the value which the Constitution places on natural resources, and recognizes
their potential benefits. It likewise acknowledges the fact that the impact of mining operations is not
confined to the economy but, perhaps to a greater extent, affects Philippine society as a whole as
well.

"Minerals, petroleum and other mineral oils," are part of the non-renewable wealth of the Filipino
people. By pursuing large scale exploration, development and utilization of these resources, the State
would be allowing the consumption or exhaustion of these resources, and thus deprive future Filipino
generations the enjoyment thereof. Mining – especially large-scale mining – often results in the
displacement of local residents. Its negative effects on the environment are well-documented.47

Thus, for benefits from the exploration, development and utilization of these resources to be real, they
must yield profits over and above 1) the capital and operating costs incurred, 2) the resulting damage
to the environment, and 3) the social costs to the people who are immediately and adversely affected
thereby.

Moreover, the State must ensure that the real benefits from the utilization of these resources
are sufficient to offset the corresponding loss of these resources to future generations. Real
benefits are intergenerational benefits because the motherland's natural resources are the birthright
not only of the present generation of Filipinos but of future generations as well. 48

The requirement of real benefit is applicable even when the exploration, development and utilization
are being undertaken directly by the Government or with the aid of Filipinos or Filipino corporations.
But it takes on greater significance when a foreign entity is involved. In the latter instance, the foreign
entity would naturally expect to be compensated for its assistance. In that event, it is inescapable that
a foreigner would be benefiting from an activity (i.e. mining) which also results in numerous, serious
and long term harmful consequences to the environment and to Philippine society.

Moreover, as recognized by the 1935 Constitutional Convention, foreign involvement in the


exploitation of Philippine natural resources has serious implications on national security. As recounted
by delegate Jose Aruego:

The nationalization of the natural resources was also intended as an instrument of national
defense. The Convention felt that to permit foreigners to own or control the natural
resources would be to weaken the national defense. It would be making possible the
218

gradual extension of foreign influence into our politics, thereby increasing the
possibility of foreign control. xxx

Not only these. The nationalization of the natural resources, it was believed, would
prevent making the Philippines a source of international conflicts with the consequent
danger to its internal security and independence. For unless the natural resources were
nationalized, with the nationals of foreign countries having the opportunity to own or control
them, conflicts of interest among them might arise inviting danger to the safety and
independence of the nation.49 (Emphasis supplied)

Significantly, and contrary to the posture of the OSG, it is immaterial whether the foreign involvement
takes the form of "active" participation in the mining concern or "passive" assistance such as a foreign
mining loan or the licensing of mining technology. Whether the foreign involvement is passive or
active, the fact remains that the foreigner will expect to be compensated and, as a necessary
consequence, a fraction of the gains, rewards and advantages generated by Philippine natural
resources will be diverted to foreign hands even as the long term pernicious "side effects" of the
mining activity will be borne solely by the Filipino people.

Under such circumstances, the Executive, in determining whether or not to avail of the assistance of a
foreign corporation in the large scale exploration, development and utilization of Philippine natural
resources, must carefully weigh the costs and benefits if it is to faithfully discharge its fiduciary duty to
protect the beneficial interest of the Filipino people in these resources.

These same considerations likewise explain why the last paragraph of Section 2 mandates that the
President "notify the Congress of every contract entered into in accordance with this provision, within
thirty days from its execution." The Constitution requires that the Legislative branch, which is
perceived to be more broadly representative of the people and therefore more immediately sensitive
to their concerns, be given a timely opportunity to scrutinize and evaluate the Executive's decision.

With these concepts in mind, I now turn to what I believe to be the proper interpretation of
"agreements… involving either technical or financial assistance" in paragraph 4 of Section 2, Article
XII of the Constitution.

Construction of paragraph 4, Section 2,


Article XII of the Constitution

The suggestion that the avoidance of the term "service contracts" in the fourth paragraph is to prevent
the circumvention, prevalent under the 1973 Constitution, of the 60-40 capital requirement does not
persuade, it being too narrow an interpretation of that provision. If that were the only purpose in the
change of phraseology, this Court reiterates, there would have been no need to replace the term
"service contracts" with "agreements… involving either technical or financial assistance."

The loophole in the 1973 Constitution that sanctioned dummyism is easily plugged by the provision in
the present Constitution that the President, not Congress or the Batasan Pambansa (under the 1973
Constitution), may enter into either technical or financial agreements with foreign corporations. The
framers then could have easily employed the more traditional term "service contracts" in designating
the agreements contemplated, and thus obviated confusion, especially since the term was employed
by the legal system then prevailing50 and had a settled acceptation.

The other proffered raison d'être of the fourth paragraph, i.e. to address the absence of a governing
law that led to the abuse of service contracts, is equally unpersuasive. In truth, there were a host of
laws governing service contracts pertaining to various natural resources, as this Court noted when it
traced the history of Section 2, Article XII in its Decision.51

Respondent WMCP nevertheless correctly states that the fourth paragraph establishes an exception
to the rule limiting the exploration, development and utilization of the nation's natural resources to
Filipinos. As an exception, however, it is illogical to deduce that the provision should be
219

interpreted liberally, not restrictively. It bears repeating that the provision, being an exception,
should be strictly construed against foreign participation.

In any case, the constitutional provision allowing the President to enter into FTAAs with
foreign-owned corporations is an exception to the rule that participation in the nation's natural
resources is reserved exclusively to Filipinos. Accordingly, such provision must be construed
strictly against their enjoyment by non-Filipinos. As Commissioner Villegas emphasized,
the provision is "very restrictive." Commissioner Nolledo also remarked that "entering
into service contracts is an exception to the rule on protection of natural resources for
the interest of the nation and, therefore, being an exception, it should be subject,
whenever possible, to stringent rules." Indeed, exceptions should be strictly but
reasonably construed; they extend only so far as their language fairly warrants and all doubts
should be resolved in favor of the general provision rather than the exception.52 (Emphasis
and underscoring supplied; citations omitted).

That the fourth paragraph employs the word "may" does not make it non-restrictive. Indeed, "may"
does make the provision permissive, but only as opposed to mandatory, 53 and operates to confer
discretion upon a party.54 Thus, as used in the fourth paragraph, "may" provides the President with
the option to enter into FTAAs. It is, however, not incumbent upon the President to do so for, as
owner of the natural resources, the "State [itself] may directly undertake such activities." 55 If the
President opts to exercise the prerogative to enter into FTAAs, the agreement must conform to the
restrictions laid down by Section 2, including the scope of the assistance, which must be limited to
financial or technical forms.

"May" in the fourth paragraph, therefore, should be understood in the same sense as it is used in the
first paragraph, that is, that the State "may enter into… agreements with Filipino citizens, or
corporations or association at least sixty per centum of whose capital is owned by such citizens."

The majority, however, opines that the "agreements involving either technical or financial assistance"
referred to in paragraph 4 of Section 2 of Article XII of the 1987 Constitution are indeed service
contracts. In support of this conclusion, the majority maintains that the use of the phrase
"agreements… involving either technical or financial assistance" does not indicate the intent to
exclude other modes of assistance because the use of the word "involving" signifies the possibility of
the inclusion of other forms of assistance or activities. And it proffers that the word "involving" has
three connotations that can be differentiated as follows: (1) the sense of concerning, having to do
with, or affecting; (2) entailing, requiring, implying or necessitating; (3) including, containing or
comprising. None of these three connotations, it is contended, convey a sense of exclusivity. Thus, it
concludes that had the framers intended to exclude other forms of assistance, they would have simply
said "agreements for technical or financial assistance" as opposed to "agreements including technical
or financial assistance."

To interpret the term "involving" in the fourth paragraph to mean "including," as the majority contends,
would run counter to the restrictive spirit of the provision. Notably, the 1987 Constitution uses
"involving" not "including." As admitted in the majority opinion, the word "involve" may also mean
concerning, having to do with or affecting. Following the majority opinion's own methodology of
substitution, "agreements… involving either technical or financial assistance" means
"agreements…concerning either technical or financial assistance." And the word "concerning"
according to Webster's Third New International Dictionary means "regarding", "respecting" or "about."
To reiterate, these terms indicate exclusivity. More tellingly, the 1987 Constitution not only deleted the
term "management" in the 1973 Constitution, but also the catch-all phrase "or other forms of
assistance,"56 thus reinforcing the exclusivity of "either technical or financial assistance."

That the fourth paragraph does not employ the terms "solely," "only," or "limited to" to qualify "either
technical or financial assistance" does not detract from the provision's restrictive nature. Moreover,
the majority opinion's illustration conveniently omits "either… or." As Senior Associate Justice
Reynato S. Puno pointed out during the oral arguments, the use of the disjunctive "either… or"
denotes restriction.57
220

According to the Penguin Dictionary, the word "either" may be used as (1) an adjective or (2) a
pronoun or (3) a conjunction or (4) an adverb. As an adjective, the word "either" means (1) any one of
two; one or the other; or (2) one and the other; each. As a pronoun, the word "either" means the one
or the other. As a conjunction, the word "either" is used before two or more sentence elements
of the same class or function joined usually by "or" to indicate what immediately follows is the
first of two or more alternatives. Lastly, as an adverb, "either" is used for emphasis after a negative
or implied negation (i.e. for that matter or likewise). The traditional rule holds that "either" should be
used only to refer to one of two items and that "any" is required when more than two items are
involved.58 However, modern English usage has relaxed this rule when "either" is used as a
conjunction.59 Thus, the word "either" may indicate the choice between two or more possibilities.

"Either" in paragraph 4, section 2, Article XII, is clearly used as a conjunction, joining two (and only
two) concepts – financial and technical. The use of the word "either" clearly limits the President to
only two possibilities, financial and technical assistance. Other forms of assistance are plainly not
allowed, since only the words "financial and technical" follow the word "either."

In accordance with the intent of the provision, "agreements… involving either technical or financial" is
deemed restrictive and not just descriptive. It is a condition, a limitation, not a mere description.

The OSG's suggestion that the President may enter into "any" agreement, the scope of which may go
beyond technical or financial assistance, with a foreign-owned corporation, does not impress. The first
paragraph of Section 2 limits contracts with Filipino citizens or corporations to co-production, joint
venture or production-sharing agreements. To subscribe to the OSG's theory would allow foreign-
owned corporations participation in the country's natural resources equal to, perhaps even greater
than, that of Filipino citizens or corporations.

The OSG cites the Separate Opinion of Justice Jose C. Vitug, now retired, who proposed that, on the
premise that the State itself may undertake the exploration, development and utilization of natural
resources, a foreign-owned corporation may engage in such activities in behalf of the State:

The Constitution has not prohibited the State from itself exploring, developing, or utilizing the
country's natural resources, and, for this purpose, it may, I submit, enter into the necessary
agreements with individuals or entities in the pursuit of a feasible operation.

The fundamental law is deemed written in every contract. The FTAA entered into by the
government and WMCP recognizes this vital principle. Thus, two of the agreement's clauses
provide:

"WHEREAS, the 1987 Constitution of the Republic of the Philippines provides in


Article XII, Section 2 that all lands of the public domain, waters, minerals, coal,
petroleum, and other natural resources are owned by the State, and that the
exploration, development and utilization of natural resources shall be under the full
control and supervision of the State; and

"WHEREAS, the Constitution further provides that the Government may enter into
agreements with foreign-owned corporations involving either technical or financial
assistance for large scale exploration, development and utilization of minerals."

The assailed contract or its provisions must then be read in conformity with abovementioned
constitutional mandate. Hence, Section 10.2(a) of the FTAA, for instance, which states that
"the Contractor shall have the exclusive right to explore for, exploit, utilize, process, market,
export and dispose of all minerals and products and by-products thereof that may be derived
or produced from the Contract Area and to otherwise conduct Mining Operations in the
Contract Area in accordance with the terms and conditions hereof," must be taken to mean
that the foregoing rights are to be exercised by WMCP for and in behalf of the State and that
WMCP, as the Contractor, would be bound to carry out the terms and conditions of the
agreement acting for and in behalf of the State. In exchange for the financial and technical
221

assistance, inclusive of its services, the Contractor enjoys an exclusivity of the contract and a
corresponding compensation therefor.60 (Underscoring supplied).

This proposition must be rejected since it sanctions the circumvention, if not outright violation, of the
fourth paragraph by allowing foreign corporations to render more than technical or financial
assistance on the pretext that it is an agent of the State. Quando aliquid prohibitur ex directo,
prohibitur et per obliquum. What is prohibited directly is prohibited indirectly.61 Further, the proposition
lends itself to mischievous consequences. If followed to its logical conclusion, nothing would stop the
State from engaging the services of a foreign corporation to undertake in its behalf the exploration,
development and utilization of all other natural resources, not just "minerals, petroleum and mineral
oils," even on a small scale, not just "large-scale."

The present Constitution restricts foreign involvement to large-scale activities because the idea is to
limit the participation of foreign corporations only to areas where they are needed.

MS. QUESADA. Going back to Section 3, the section suggests that:

The exploration, development, and utilization of natural resources … may be directly


undertaken by the State, or it may enter into co-production, joint venture or production-
sharing agreement with … corporations or associations at least sixty percent of whose voting
stock or controlling interest is owned by such citizens.

Lines 25 to 30 on the other hand, suggest that in the large-scale exploration, development
and utilization of natural resources, the President with the concurrence of Congress may
enter into agreements with foreign-owned corporations even for technical or financial
assistance.

I wonder if this first part of Section 3 contradicts the second part. I am raising this
point for fear that foreign investors will use their enormous capital resources to
facilitate the actual exploitation or exploration, development and effective disposition
of our natural resources to the detriment of Filipino investors. I am not saying that we
should not consider borrowing money from foreign sources. What I refer to is that
foreign interest should be allowed to participate only to the extent that they lend us
money and give us technical assistance with the appropriate government permit. In
this way, we can insure the enjoyment of our natural resources by out people.

MR. VILLEGAS. Actually, the second provision about the President does not permit
foreign investors to participate. It is only technical or financial assistance – they do not
own anything – but on conditions that have to be determined by law with the concurrence of
Congress. So, it is very restrictive.

If the Commissioner will remember, this removes the possibility for service
contracts which we said yesterday were avenues used in the previous regime to go
around the 60-40 requirement.62 (Emphasis and underscoring supplied)

The intent is to allow Filipinos to benefit from Filipino resources.

MR. DAVIDE. May I be allowed to explain the proposal?

MR. MAAMBONG. Subject to the three-minute rule, Madam President.

MR. DAVIDE. It will not take me three minutes.

The Commission had just approved the Preamble. In the Preamble we clearly sated there
that the Filipino people are sovereign and that one of the objectives for the creation or
establishment of a government is to conserve and develop the national patrimony. The
implication is that the national patrimony or our natural resources are exclusively
222

reserved for the Filipino people. No alien must be allowed to enjoy, exploit and develop
our natural resources. As a matter of fact, that principle proceeds from the fact that our
natural resources are gifts from God to the Filipino people and it would be a breach of
that special blessing from God if we will allow aliens to exploit our natural resources.

I voted in favor of the Jamir proposal because it is not really exploitation that we grant
ed to the aliencorporations but only for them to render financial or technical assistanc
e. It is not for them to enjoyour natural resources. Madam President, our natural
resources are depleting; our population is increasing by leaps and bounds. Fifty years from
now, if we will allow these aliens to exploit our natural resources, there will be no more natural
resources for the next generations of Filipinos. It may last long if we will begin now. Since
1935 the aliens have been allowed to enjoy to a certain extent the exploitation of our natural
resources, and we became victims of foreign dominance and control. The aliens are
interested in coming to the Philippines because they would like to enjoy the bounty of nature
exclusively intended for the Filipinos by God.

And so I appeal to all, for the sake of the future generations, that if we have to pray in the
Preamble "to preserve and develop the national patrimony for the sovereign Filipino people
and for the generations to come," we must at this time decide once and for all that our natural
resources must be reserved only to Filipino citizens.

Thank you.63 (Emphasis and underscoring supplied)

The intent loses all significance if foreign-owned corporations are likewise allowed to participate even
in small or medium-scale ventures.

Thus, in keeping with the clear intent and rationale of the Constitution, financial or technical
assistance by foreign corporations are allowable only where there is no Filipino or Filipino-owned
corporation (including corporations at least 60% of the capital of which are owned by Filipinos) which
can provide the same or similar assistance.

To reiterate, the over-arching letter and intent of the Constitution is to reserve the exploration,
development and utilization of natural resources to Filipinos.

The justification for foreign involvement in the exploration, development and utilization of natural
resources was that Filipino nationals or corporations may not possess the necessary capital, technical
knowledge or technology to mount a large scale undertaking. In the words of the "Draft of the 1986
U.P. Law Constitution Project" (U.P. Law Draft) which was taken into consideration during the
deliberation of the CONCOM:64

Under the proposed provision, only technical assistance or financial assistance


agreements may be entered into, and only for large-scale activities. These are contract
forms which recognize and assert our sovereignty and ownership over natural
resources since the foreign entity is just a pure contractor and not a beneficial owner
of our economic resources. The proposal recognizes the need for capital and
technology to develop our natural resources without sacrificing our sovereignty and
control over such resources65 x x x (Emphasis and underscoring supplied)

Thus, the contention that Section 2, Article XII allows for any agreement for assistance by a foreign
corporation "so long as such assistance requires specialized knowledge or skills, and are related to
the exploration, development and utilization of mineral resources" is erroneous. 66

Where a foreign corporation does not offer financial or technological assistance beyond the
capabilities of its Philippine counterparts, an FTAA with such a corporation would be highly
questionable. Similarly, where the scope of the undertaking does not qualify as "large scale," an
FTAA with a foreign corporation is equally suspect.
223

"Agreements" in Section 2, Article XII


do not include "service contracts."

This Court's ruling in the Decision under reconsideration that the agreements involving either
technical or financial assistance contemplated by the 1987 Constitution are different and dissimilar
from the service contracts under the 1973 Constitution must thus be affirmed. That there is this
difference, as noted in the Decision, is gathered from the change in phraseology. 67 There was no
need to employ strongly prohibitory language, like that found in the Bill of Rights. 68 For the framers to
expressly prohibit "management and other forms of assistance" would be redundant inasmuch as the
elimination of such phrase serves the same purpose. The deletion is simply too significant to ignore
and speaks just as profoundly – it is an outright rejection.

It bears noting that the fourth paragraph does not employ the same language adopted in the first
paragraph, which specifically denominates the agreements that the State may enter into with Filipinos
or Filipino-owned corporations. The fourth paragraph does not state "The President may also enter
into co-production, joint venture, or production-sharing agreements with foreign-owned
corporations for large-scale exploration, development, and utilization of minerals, petroleum, and
other mineral oils…." On the other hand, the fourth paragraph cannot be construed as a grant of
boundless discretion to the President to enter into any agreement regardless of the scope of
assistance because it would result in a bias against Filipino citizens and corporations.

On this point, the following observations from the U.P. Law Draft on the odious and objectionable
features of service contracts bear restating:

5. The last paragraph is a modification of the service contract provision found in Section 9,
Article XIV of the 1973 Constitution as amended. This 1973 provision shattered the
framework of nationalism in our fundamental law (see Magallona, "Nationalism and its
Subversion in the Constitution"). Through the service contract, the 1973 Constitution had
legitimized that which was prohibited under the 1935 constitution—the exploitation of
the country's natural resources by foreign nationals. Through the service contract, acts
prohibited by the Anti-Dummy Law were recognized as legitimate arrangements. Service
contracts lodge exclusive management and control of the enterprise to the service
contractor, not unlike the old concession regime where the concessionaire had
complete control over the country's natural resources, having been given exclusive
and plenary rights to exploit a particular resource and, in effect, having been assured
of ownership of that resource at the point of extraction (see Agabin, "Service Contracts:
Old Wine in New Bottles"). Service contracts, hence, are antithetical to the principle of
sovereignty over our natural resources, as well as the constitutional provision on
nationalization or Filipinization of the exploitation of our natural resources. 69 (Emphasis
supplied)

Furthermore, Professor Pacifico A. Agabin, a member of the working group of the U.P. Law
Constitution Project and now counsel for intervenor PCM, stated in his position paper:

Recognizing the service contract for what it


is, we have to expunge it from the Constitution and reaffirm ownership over our natural
resources. That is the only way we can exercise effective control over ournatural resources.

This should not mean complete isolation of the country's natural resources from foreign
investment. Other contract forms which
are less derogatory to our sovereignty and control over natural resources – like technical
assistance agreements, financial assistance [agreements], co-production agreements,
joint ventures, production-sharing [agreements] – could still be utilized and adopted without
violating constitutional provisions. In other words, we can adopt contract forms which
recognize and assert our sovereignty and ownership over natural resources, and where the
entity is just a pure contractor instead of the beneficial owner of our economic
resources.70 (Emphasis & underscoring supplied),
224

indicating that the proposed financial or technical assistance agreements are contract
forms different from the 1973 Constitution service contracts.

Thus the phrase "agreements with foreign-owned corporations involving either technical or financial
assistance" in Section 2, Article XII of the Constitution must be interpreted as restricting foreign
involvement in the exploration, development and utilization of natural resources to large
scale undertakings requiring foreign financial or technicalassistance and not, as alleged by
respondents, inclusive of any possible agreement under the sun.

The majority however argues that the deletion or omission from the 1987 Constitution of the term
"service contracts" found in the 1973 Constitution does not sufficiently prove the drafters' intent to
exclude foreigners from management since such intent cannot be definitively and conclusively
established. This argument overlooks three basic principles of statutory construction.

First, casus omisus pro omisso habendus est.71 As recently as 2001 in Commission on Audit of the
Province of Cebu v. Province of Cebu,72 this Court held that a person, object or thing omitted from an
enumeration must be held to have been omitted intentionally.73 That there is a difference between
technical or financial assistance contemplated by the 1987 Constitution and the service contracts
under the 1973 Constitution is gathered from the omission of the phrase "management or other forms
of assistance."

As earlier noted, the phrase "service contracts" has been deleted in the 1987 Constitution's
Article on National Economy and Patrimony. If the CONCOM intended to retain the concept of
service contracts under the 1973 Constitution, it would have simply adopted the old
terminology ("service contracts") instead of employing new and unfamiliar terms
("agreements…involving either technical or financial assistance.") Such a difference
between the language of a provision in a revised constitution and that of a similar
provision in the preceding constitution is viewed as indicative of a difference in
purpose. If, as respondents suggest, the concept of "technical or financial assistance"
agreements is identical to that of "service contracts," the CONCOM would not have bothered
to fit the same dog with a new collar. To uphold respondents' theory would reduce the first to
a mere euphemism for the second render the change in phraseology
meaningless.74 (Emphasis and underscoring supplied; citation omitted)

Second, expressio unius est exclusion alterius.75 The express mention of one person, thing, act, or
consequence excludes all others.76

Third and lastly, expressium facit cessare tacitum.77 What is expressed puts an end to that which is
implied.78 Since the constitutional provision, by its terms, is expressly limited to financial or technical
agreements, it may not, by interpretation or construction, be extended to other forms of assistance.

These three principles of statutory construction, derived from the well-settled principle of verba legis,
proceed from the premise that the Constitutional Commission would not have made specific
enumerations in the provision if it had the intention not to restrict its meaning and confine its terms to
those expressly mentioned. And this Court may not, in the guise of interpretation, enlarge the scope
of a constitutional provision and include therein situations not provided nor intended by the framers.
To do so would be to do violence to the very language of the Constitution, the same Constitution
which this Court has sworn to uphold.

The majority counters, however, that service contracts were not de-constitutionalized since the
deliberations of the members of the Constitutional Commission conclusively show that they discussed
agreements involving either technical or financial assistance in the same breath as service contracts
and used the terms interchangeably. This argument merely echoes that of private respondent WMCP
which had already been addressed in this Court's Decision of January 27, 2004, (the Decision) viz:

While certain commissioners may have mentioned the term "service contracts" during the
CONCOM deliberations, they may not have been necessarily referring to the concept of
service contracts under the 1973 Constitution. As noted earlier "service contracts" is a
225

term that assumes different meanings to different people. The commissioners may
have been using the term loosely, and not in its technical and legal sense, to refer, in
general, to agreements concerning natural resources entered into by the Government
with foreign corporations. These loose statements do not necessarily translate to the
adoption of the 1973 Constitution provision allowing service contracts.

It is true that, as shown in the earlier quoted portions of the proceedings in [the] CONCOM, in
response to Sr. Tan's question, Commissioner Villegas commented that, other than
congressional notification, the only difference between "future" and "past" "service contracts"
is the requirement of a general law as there were no laws previously authorizing the
same.79 However, such remark is far outweighed by his more categorical statement in
his exchange with Commissioner Quesada that the draft article "does not permit
foreign investors to participate" in the nation's natural resources – which was exactly
what service contracts did – except to provide "technical or financial assistance."

In the case of the other commissioners, Commissioner Nolledo himself clarified in his work
that the present charter prohibits service contracts. Commissioner Gascon was not totally
averse to foreign participation, but favored stricter restrictions in the form of majority
congressional concurrence. On the other hand, Commissioners Garcia and Tadeo may have
veered to the extreme side of the spectrum and their objections may be interpreted as votes
against any foreign participation in our natural resources whatsoever.80(Emphasis and
underscoring supplied; citations omitted)

In fact, the opinion of Commissioner Nolledo in his textbook which is cited in this Court's January 27,
2004 Decision should leave no doubt as to the intention of the framers to eliminate service contracts
altogether.

Are service contracts allowed under the new Constitution? No. Under the new Constitution,
foreign investors (fully alien-owned) can NOT participate in Filipino enterprises except to
provide: (1) Technical Assistance for highly technical enterprises; and (2) Financial
Assistance for large-scale enterprises.

The intention of this provision, as well as other provisions on foreign investments, is to


prevent the practice (prevalent in the Marcos government) of skirting the 60/40 equation using
the cover of service contracts.81

Next, the majority opinion asserts that if the framers had meant to ban service contracts altogether,
they would have provided for the termination or pre-termination of the existing service contracts.

There was no need for a constitutional provision to govern the termination or pre-termination of
existing service contracts since the intention of the framers was to apply the rule banning service
contracts prospectively.

MR. DAVIDE. Under the proposal, I notice that except for the lands of the public domain, all
other natural resources cannot be alienated and in respect to lands of the public domain,
private corporations with the required ownership by Filipino citizens can only lease the same.
Necessarily, insofar as other natural resources are concerned, it would only be the State
which can exploit, develop, explore and utilize the same. However, the State may enter into a
joint venture, coproduction (sic) or production-sharing. Is that not correct?

MR. VILLEGAS. Yes.

MR. DAVIDE. Consequently, henceforth upon the approval of this Constitution, no timber or
forest concessions, permits or authorization can be exclusively granted to any citizen of the
Philippines nor to any corporation qualified to acquire lands of the public domain?

MR. VILLEGAS. Would Commissioner Monsod like to comment on that? I think his answer is
"yes."
226

MR. DAVIDE. So, what will happen now to licenses or concessions earlier granted by the
Philippine government to private corporations or to Filipino citizens? Would they be deemed
repealed?

MR. VILLEGAS. This is not applied retroactively. They will be respected.

MR. DAVIDE. In effect, they will be deemed repealed?

MR. VILLEGAS. No.82 (Emphasis and underscoring supplied)

Besides, a service contract is only a license or privilege, not a contract or property right which merits
protection by the due process clause of the Constitution. Thus in the landmark case of Oposa v.
Factoran, Jr,83 this Court held:

xx
x Needless to say, all licenses may thus be revoked or rescinded by executive action. It
is not acontract, property or a property right protected by the due process clause of th
e Constitution. In Tan vs. Director of Forestry, this Court held:

"x x x A timber license is an instrument by which the State regulates the utilization and
disposition of forest resources to the end that public welfare is promoted. A timber license is
not a contract within the purview of the due process clause; it is only a license or privilege,
which can be validly withdrawn whenever dictated by public interest or public welfare
as in this case.

'A license is merely a permit or privilege to do what otherwise would be unlawful, and
is not a contract between the authority, federal, state, or municipal, granting it and the
person to whom it is granted; neither is it property or a property right, nor does it
create a vested right; nor is it taxation' Thus, this Court held that the granting
of license does not create irrevocable rights, neither is it property or property
rights."

We reiterated this pronouncement in Felipe Ysmael, Jr. & Co, Inc. vs. Deputy Executive Secretary:

"x x x Timber licenses, permits and license agreements are the principal instruments by which
the State regulates the utilization and disposition of forest resources to the end that public
welfare is promoted. And it can hardly be gainsaid that they merely evidence a privilege
granted by the State to qualified entities, and do not vest in the latter a permanent or
irrevocable right to the particular concession area and the forest products therein. They may
be validly amended, modified, replaced or rescinded by the Chief Executive when
national interests so require. Thus, they are not deemed contracts within the purview
of the due process clause."

Since timber licenses are not contracts, the non-impairment clause which reads:

"SEC 10. No law impairing, the obligation of contracts shall be passed."

cannot be invoked.

In the second place, even if it is to be assumed that the same are contracts, the instant case does not
involve a law or even an executive issuance declaring the cancellation or modification of existing
timber licenses. Hence, the non-impairment clause cannot as yet be invoked. Nevertheless, granting
further that a law has actually been passed mandating cancellations or modifications, the same
cannot still be stigmatized as a violation of the non-impairment clause. This is because by its very
nature and purpose, such a law could have only been passed in the exercise of the police power of
the state for the purpose of advancing the right of the people to a balanced and healthful ecology,
227

promoting their health and enhancing the general welfare. In Abe vs. Foster Wheeler Corp., this Court
stated:

"The freedom of contract, under our system of government, is not meant to be absolute. The
same is understood to be subject to reasonable legislative regulation aimed at the promotion
of public health, moral, safety and welfare. In other words, the constitutional guaranty of
non-impairment of obligations of contract is limited by the exercise of the police power
of the State, in the interest of public health, safety, moral and general welfare."

The reason for this is emphatically set forth in Nebia vs. New York quoted in Philippine American Life
Insurance Co. vs. Auditor General, to wit:

"Under our form of government the use of property and the making of contracts are normally
matters of private and not of public concern. The general rule is that both shall be free of
governmental interference. But neither property rights nor contract rights are absolute; for
government cannot exist if the citizen may at will use his property to the detriment of his
fellows, or exercise his freedom of contract to work them harm. Equally fundamental with the
private right is that of the public to regulate it in the common interest."

In short, the non-


impairment clause must yield to the police power of the state.84 (Emphasis and
underscoring supplied; citations omitted)

The majority however argues that Oposa is not applicable since the investment in a logging
concession is not as substantial an investment as that of a large scale mining contractor. Such a
contention is patently absurd. Taken to its logical conclusion, the majority would have this Court
exempt firms in highly capital intensive industries from the exercise of police power simply to protect
their investment. That would mean that the legislature would, for example, be powerless to revoke or
amend legislative franchises of public utilities, such as power and telecommunications firms, which no
doubt require huge sums of capital.

The majority opinion then proffers that the framers of the Constitution were pragmatic enough to know
that foreign entities would not enter into such agreements without requiring arrangements for the
protection of their investments, gains, and benefits or other forms of conditionalities. It goes on to
argue that "by specifying such 'agreements involving assistance,' the framers of the Constitution
necessarily gave implied assent to everything that these agreements necessarily entailed; or that
could reasonably be deemed necessary to make them tenable and effective, including management
authority with respect to the day-to-day operations of the enterprise and measures for the protection
of the interests of the foreign corporation."

The deliberations of the Constitutional Commission, however, do not support the immediately
foregoing contentions.

MR. TINGSON. Within the purview of what the Gentleman is saying, would he welcome
friendly foreigners to lend us their technical expertise in helping develop our country?

MR. GARCIA. Part 2 of this proposal, Filipino control of the economy, in fact, says that the
entry of foreign capital, technology and business enterprises into the national economy shall
be effectively regulated to ensure the protection of the interest of our people.

In other words, we welcome them but on our own terms. This is very similar to our
position on loans. We welcome loans as long as they are paid on our own terms, on
our ability to pay, not on their terms.For example, the case of Peru is instructive. They
decided first to develop and grow, and were willing to pay only 10 percent of their foreign
exchange earnings. That, I think, is a very commendable position given the economic
situation of a country such as Peru. The Philippines is a similar case, especially when we
realize that the foreign debt was made by a government that was bankrupt in its desire to
serve the people.
228

MR. MONSOD. Mr. Vice-President, I think we have to make a distinction that it is not really
realistic to say that we will borrow on our own terms. Maybe we can say that we inherited
unjust loans, and we would like to repay these on terms that are not prejudicial to our own
growth. But the general statement that we should only borrow on our own terms is a bit
unrealistic.

MR. GARCIA. Excuse me. The point I am trying to make is that we do not have to
borrow. If we have to borrow, it must be on our terms. In other words, banks do not
lend out of the goodness of their hearts. Banks lend to make a profit.

MR. TINGSON. Mr. Vice-President, I think the trouble in our country is that we have
forgotten the scriptural injunction that the borrower becomes a slave to the lender.
That is the trouble with our country; we have borrowed and borrowed but we forget
that we become slaves to those who lend us.85 (Emphasis and underscoring supplied)

By public respondent's information, "[t]he potential mining wealth in the Philippines is estimated at
$840 billion or P47 trillion or 10 times our annual GDP, and 15 times our total foreign debt of $56
billion. Globally, the Philippines ranks third in gold, fourth in copper, fifth in nickel and sixth in
chromite."86 With such high concentration of valuable minerals coupled with the Filipino people's
willingness to protect and preserve ownership of their natural resources at the expense of retarding or
postponing the exploration, development, and utilization of these resources, the Philippines clearly
has the superior bargaining position and should be able to dictate its terms. No foreign entity should
be able to bully the Philippines and intimidate the Government into conceding to certain conditions
incompatible with the Constitution.

Extent of foreign corporation's


participation in the management of an FTAA

Foreign-owned corporations, however, are not precluded from a limited participation in the
management of the exploration, development and utilization of natural resources.

Some degree of participation by the contractor in management, to assure the proper application of its
investment and/or to facilitate the technical assistance and transfer of technology may be unavoidable
and not necessarily undesirable. Thus, there is merit in respondent WMCP's contention, to which
even petitioners conceded during the oral arguments, that a foreign-owned corporation is not
prevented from having limited participation in the management assistance or participation so long as
it is incidental to the financial or technical assistance being rendered:

JUSTICE PANGANIBAN:

Alright. Going back to verba legis, you say that the FTAA's are limited to financial or
technical assistance only.

ATTY. LEONEN:

Either financial or technical assistance, yes your Honor.

ATTY. LEONEN:

Full management, your Honor.

JUSTICE PANGANIBAN:

Full management is excluded.

ATTY. LEONEN:
229

Yes your Honor.

JUSTICE PANGANIBAN:

But incidental management to protect the financial or technical assistance


should be allowed.

ATTY. LEONEN:

If a mining company would get the technical expertise to bring in drilling rig
your Honor, and that is the sole contract, then we cannot imagine a situation
were it is not the technicians that we will do the actual drilling your Honor, but
for the entire contract area your Honor as it is now in the FTAA then I think that
would be different.

JUSTICE PANGANIBAN:

Yes I agree. In other words, the words financial or technical may include parts
of management, isn't it? Its reasonable in other words if I may re state it, it's
reasonable to expect that entities,foreign entities who don't know anything about
this country, well that is an exaggeration, who know not too much about this country,
would not just extend money, period. They would want to have a say a little bit of
say management and sometimes even in auditing of the company, isn't it
reasonable to expect.

ATTY. LEONEN:

I would qualify my answer your Honor with management of what your Honor. It
means if it's for development and utilization of the minerals.

JUSTICE PANGANIBAN:

No.

ATTY. LEONEN:

Yes your Honor, but if it's management of sub-contracted activity like a symposium
then that would be all right your Honor. Mining companies do symposiums also.

JUSTICE PANGANIBAN:

Management to protect their own investments, whether it be technical or


financial.

ATTY. LEONEN:

Their investment, your Honor, which cannot be the entire mining


operation from my perspective, your Honor.

JUSTICE PANGANIBAN:

Yes I agree because there is the Constitutional provision of control and


supervision, full control and supervision to the State.

ATTY. LEONEN:
230

And Filipino corporations your Honor.

JUSTICE PANGANIBAN:

Or even Filipino corporation, the full control and supervision is still with the State.

ATTY. LEONEN:

Yes your Honor.

JUSTICE PANGANIBAN:

Even with Filipino citizens being the contractors, full control and supervision is still
with the State.

ATTY. LEONEN:

Yes, your Honor.

JUSTICE PANGANIBAN:

In all these contract full control and supervision is with the State.

ATTY. LEONEN:

Yes your Honor and we can only hope that the State is responsive to the people we
represent.

xxx

JUSTICE PANGANIBAN:

Yes, yes. Can it also not be said reading that the Constitution that the safeguards on
contracts with foreigners was left by the Constitutional Commission or by Constitution
itself to Congress to craft out.

ATTY. LEONEN:

I can accept your Honor that there was a province of power that was given to
Congress, but it was delimited by the fact, that they removed the word
management and other arrangement and put the words either financial and
technical.

JUSTICE PANGANIBAN:

Yes but you just admitted earlier that these two words would also include some
form of management or other things to protect the investment or the
technology being put by the foreign company.

ATTY. LEONEN:

Yes your Honor for so long as it's not the entire.

JUSTICE PANGANIBAN:
231

Yes, yes provided the State does not lose control and supervision, isn't it?

ATTY. LEONEN:

Yes your Honor.87 (Emphasis and underscoring supplied)

Thus, the degree of the foreign corporation's participation in the management of the mining concern is
co-extensive with and strictly limited to the degree of financial or technical assistance extended. The
scope of the assistance defines the limits of the participation in management.

However, to whatever extent the foreign corporation's incidental participation in the management of
the mining concern may be, full control and supervision, sufficient to protect the interest of the
Filipino people, over all aspects of mining operations must be retained by the
Government. While this does not necessarily mean that the Government must assume the role of a
back seat driver, actively second guessing every decision made by the foreign corporation, it does
mean that sufficient safeguards must be incorporated into the FTAA to insure that the people's
beneficial interest in their natural resources are protected at all times.

Moreover, the foreign contractor's limited participation in management, as the Court held in its
Decision, should not effectively grant foreign-owned corporations beneficial ownership over the
natural resources.

The opinion, submitted by the OSG, of Bernardo M. Villegas, who was a Member of the Constitutional
Commission and Chair of its Committee on National Economy and Patrimony, is not inconsistent with
the foregoing conclusion. Commissioner Villegas opined:

The phrase "service contracts" contained in the 1973 Constitution was deleted in the 1987
Constitution because there was the general perception among the Concom members that it
was used during the Marcos regime as an instrument to circumvent the 60-40 limit in favor of
Filipino ownership. There was also the impression that the inclusion of the word
"management" in the description of the service contract concept in the 1973 Constitution was
tantamount to ownership by the foreign partner.

The majority of the Concom members, however, recognized the vital need of the Philippine
economy for foreign capital and technology in the exploitation of natural resources to benefit
Filipinos, especially the poor in the countryside where the mining sites are located. For this
reason, the majority voted for "agreements involving financial or technical assistance" or
FTAA.

I maintain that the majority who voted Yes to this FTAA provision realized that an FTAA
involved more than borrowing money and/or buying technology from foreigners. If an FTAA
involved only a loan and/or purchase of technology, there would not have been a need for a
constitutional provision because existing laws in the Philippines more than adequately
regulate these transactions.

It can be deducted from the various comments of both those who voted Yes and No to the
FTAA provision that an FTAA also involves the participation in management of the foreign
partner. What was then assumed in 1986 is now even clearer in the way business
organizations have evolved in the last decade or so under the modern concept of good
governance. There are numerous stakeholders in a business other than the stockholders or
equity owners who participate actively in the management of a business enterprise. Not only
do creditors and suppliers demand representation in boards of directors. There are also other
so-called independent directors who actively participate in management.

In summary, the word "management" was deleted from the description of the FTAA
because some CONCOM delegates identified management with beneficial
ownership. In order not to prolong the debate, those in favor of the FTAA provision agreed
not to include the word management. But from what has been discussed above, it was clear
232

in the minds of those who voted YES that the FTAA included more than just a loan and/or
purchase of technology from foreigners but necessarily allowed the active
participation of the foreign partners in the management of the enterprise engaged in
the exploitation of natural resources.88 (Emphasis supplied).

Under no circumstances should the execution of an FTAA be tantamount to the grant of a roving
commission whereby a foreign contractor is given blanket and unfettered discretion to do whatever it
deems necessary – denude watersheds, divert sources of water, drive communities from their homes
– in pursuit of its pecuniary goals.

Nor should the scope of an FTAA be broadened to include "managerial assistance." As discussed
extensively in the Decision,89 "managerial assistance" – a euphemism by which full control and
beneficial ownership of natural resources were vested in foreigners – is part and parcel of the martial
law era "service contracts" and the old "concession regime" which the 1987 Constitution has
consigned to the dust bin of history.

The elimination of the phrase "service contracts" effectuates another purpose. Intervenor PCM agrees
that the Constitution tries to veer away from the old concession system, 90 which vested foreign-owned
corporations control and beneficial ownership over Philippine natural resources. Hence, the 1987
Constitution also deleted the provision in the 1935 and 1973 Constitutions authorizing the State to
grant licenses, concessions, or leases for the exploration, exploitation, development, or utilization of
natural resources.91

Prof. Agabin had no flattering words for the concession system, which he described in his position
paper as follows:

Under the concession system, the concessionaire makes a direct equity investment for the
purpose of exploiting a particular natural resource within a given area. Thus, the concession
amounts to a complete control by the concessionaire over the country's natural
resource, for it is given exclusive and plenary rights to exploit a particular resource
and is in effect assured ownership of that resource at the point of extraction. In
consideration for the right to exploit a natural resource, the concessionaire either pays rent or
royalty which is a fixed percentage of the gross proceeds. But looking beyond the legal
significance of the concession regime, we can see that there are functional implications
which give the concessionaire great economic power arising from its exclusive equity
holding. This includes, first, appropriation of the returns of the undertaking, subject to a
modest royalty; second, exclusive management of the project; third, control of
production of the natural resource, such as volume of production, expansion, research
and development; and fourth, exclusive responsibility for downstream operations, like
processing, marketing, and distribution. In short, even if nominally, the state is the
sovereign and owner of the natural resource being exploited, it has been shorn of all
elements of control over such natural resource because of the exclusive nature of the
contractual regime of the concession. The concession system, investing as it does
ownership of natural resources, constitutes a consistent inconsistency with the principle
embodied in our Constitution that natural resources belong to the State and shall not be
alienated, not to mention the fact that the concession was the bedrock of the colonial system
in the exploitation of natural resources.92 (Underscoring in the original)

Vestiges of the concession system endured in the service contract regime, including the vesting on
the contractor of the management of the enterprise, as well as the control of production and other
matters, such as expansion and development. 93 Also, while title to the resource discovered was
nominally in the name of the government, the contractor had almost unfettered control over its
disposition and sale.94

The salutary intent of the 1987 Constitution notwithstanding, these stubborn features of the
concession system persist in the Mining Act of 1995. The statute allows a foreign-owned corporation
to carry out mining operations,95which includes the conduct of exploration,96 development97 and
utilization98 of the resources.99 The same law grants foreign contractors auxiliary mining rights, i.e.,
timber rights,100 water rights,101 the right to possess explosives,102 easement rights,103 and entry into
233

private lands and concession areas.104 These are the very same rights granted under the old
concession and service contract systems.

The majority opinion proposes two alternative standards of Government control over FTAA
operations. Thus, in the opening paragraphs it states:

Full control is not anathema to day-to-day management by the contractor, provided that the
State retains the power to direct overall strategy; and to set aside, reverse, or modify
plans and actions of the contractor. The idea of full control is similar to that which is
exercised by the board of directors of a private corporation x x x (Emphasis and
underscoring supplied)

However, the majority opinion subsequently substantially reduces the scope of its definition of
"control" in this wise:

The concept of control adopted in Section 2 of Article XII must be taken to mean less than
dictatorial, all-encompassing control; but nevertheless sufficient to give the State the
power to direct, restrain, regulate and govern the affairs of the extractive
enterprises. Control by the State may be on a macro level, through the establishment of
policies, guidelines, regulations, industry standards and similar measures that would
enable the government to control the conduct of affairs in various enterprises and
restrain activities deemed not desirable or beneficial. (Emphasis and underscoring
supplied; citations omitted; italics in the original)

This second definition is apparently analogous to regulatory control which the Government is
automatically presumed to exercise over all business activities by virtue of the Police Power. This
definition of the "full control and supervision" mandated by Section 2, Article XII of the Constitution
strikes a discordant and unconvincing chord as it gives no effect to the mandated "full" character of
the State's control but merely places it at par with any other business activity or industry regulated by
the Government.

But even under this second and more limited concept of regulatory control, the provisions of the
Mining Act pertaining to FTAAs do not pass the test of constitutionality.

To be sure, the majority opinion cites a litany of documents, plans, reports and records which the
foreign FTAA contractor is obliged to submit or make available under the Mining Act and DAO 96-40.
However, the mere fact that the Act requires the submission of work programs and minimum
expenditure commitments105 does not provide adequate protection. These were also required under
the old concession106 and service contract107 systems, but did not serve to place full control and
supervision of the country's natural resources in the hands of the Government.

Conspicuously absent from the Mining Act are effective means by which the Government can protect
the beneficial interest of the Filipino people in the exploration, development and utilization of their
resources. It appears from the provisions of the Mining Act that the Government, once it has
determined that a foreign corporation is eligible for an FTAA and enters into such an agreement, has
very little say in the corporation's actual operations.

Thus, when pressed to identify the mechanism by which the Government can administratively compel
compliance with the foregoing requirements as well as the other terms and conditions of the Mining
Act, DAO 96-40 and DAO 99-56, the majority can only point to the cancellation of the agreement(s)
and/or the incentives concerned under Section 95 to 99 of the Mining Act: 108

CHAPTER XVII

Ground for Cancellation, Revocation, and Termination

SECTION 95. Late or Non-filing of Requirements. — Failure of the permittee or contractor to


comply with any of the requirements provided in this Act or in its implementing rules and
234

regulations, without a valid reason, shall be sufficient ground for the suspension of any permit
or agreement provided under this Act.

SECTION 96. Violation of the Terms and Conditions of Permit or Agreements. — Violation of
the terms and conditions of the permits or agreements shall be a sufficient ground for
cancellation of the same.

SECTION 97. Non-payment of Taxes and Fees. — Failure to pay taxes and fees due the
Government for two (2) consecutive years shall cause the cancellation of the exploration
permit, mineral agreement, financial or technical assistance agreement and other agreements
and the re-opening of the area subject thereof to new applicants.

SECTION 98. Suspension or Cancellation of Tax Incentives and Credits. — Failure to abide
by the terms and conditions of tax incentives and credits shall cause the suspension or
cancellation of said incentives and credits.

SECTION 99. Falsehood or Omission of Facts in the Statement — All statements made in the
exploration permit, mining agreement and financial or technical assistance agreement shall
be considered as conditions and essential parts thereof and any falsehood in said statements
or omission of facts therein which may alter, change or affect substantially the facts set forth
in said statements may cause the revocation and termination of the exploration permit, mining
agreement and financial or technical assistance agreement.

An examination of the foregoing fails to impress. For instance, how does cancellation of the FTAA
under Section 97 for nonpayment of taxes and fees (comprising the "basic share" of the government)
for two consecutive years facilitate the collection of the unpaid taxes and fees? How does it preserve
and protect the beneficial interest of the Filipino people? For that matter, how does the DENR
administratively compel compliance with the anti-pollution and other requirements?109 If minerals are
found to have been sold overseas at less than the most advantageous market prices, how does the
DENR obtain satisfaction from the offending foreign FTAA contractor for the difference?

In sum, the enforcement provisions of the Mining Act and its Implementing Rules are scarcely
effective, and, worse, perceptibly less than the analogous provisions of other Government Regulatory
Agencies.

For instance, the Bangko Sentral Ng Pilipinas, the Central Monetary Authority mandated by the
Constitution to exercise supervision (but not full control and supervision) over banks,110 is empowered
to (1) appoint a conservator with such powers as shall be deemed necessary to take charge of the
assets, liabilities and management of a bank or quasi-bank;111 (2) under certain well defined
conditions, summarily and without need for prior hearing forbid a bank from doing business in the
Philippines and appoint the Philippine Deposit Insurance Corporation as receiver; 112 and (3) impose a
number of administrative sanctions such as (a) fines not to exceed P30,000 per day for each violation,
(b) suspension of a bank's rediscounting privileges, (c) suspension of lending or foreign exchange
operations or authority to accept new deposits or make new investments, (d) suspension of interbank
clearing privileges, and (e) revocation of quasi-banking license.113

Similarly, to give effect to the Constitutional mandate to afford full protection to labor, 114 the Labor
Code115 grants the Secretary of Labor the power to (1) issue compliance orders to give effect to the
labor standards provisions of the Code;116 and (2) enjoin an intended or impending strike or lockout by
assuming jurisdiction over a labor dispute in an industry determined to be indispensable to the
national interest.117

Under the Tax Code, the Commissioner of Internal Revenue has the power to (1) temporarily suspend
the business operations of a taxpayer found to have committed certain specified violations; 118 (2)
order the constructive distraint of the property of a taxpayer;119 and (3) impose the summary remedies
of distraint of personal property and or levy on real property for nonpayment of taxes.120
235

In comparison, the Mining Act and its Implementing Rules conspicuously fail to provide the DENR
with anything remotely analogous to the foregoing regulatory and enforcement powers of other
government agencies.

In fine, the provisions of the Mining Act and its Implementing Rules give scarcely more than
lip service to the constitutional mandate for the State to exercise full control and supervision
over the exploration, development and utilization of Philippine Natural Resources. Evaluated
as a whole and in comparison with other government agencies, the provisions of the Mining
Act and its Implementing Rules fail to meet even the reduced standard of effective regulatory
control over mining operations. In effect, they abdicate control over mining operations in favor
of the foreign FTAA contractor. For this reason, the provisions of the Mining Act, insofar as
they pertain to FTAA contracts, must be declared unconstitutional and void.

The majority opinion vigorously asserts that it is the Chief Executive who exercises the power of
control on behalf of the State.

This only begs the question. How does President effectively enforce the terms and conditions of an
FTAA? What specific powers are subsumed within the constitutionally mandated "power of control?"
On these particular matters the majority opinion, like the Mining Act, is silent.

Provisions of the Mining Act pertaining to FTAAs


void for conveying beneficial ownership of
Philippine mineral resources to foreign contractors

An examination of the Mining Act reveals that the law grants the lion's share of the proceeds of the
mining operation to the foreign corporation. Thus the second and third paragraphs of Section 81 of
the law provide:

SECTION 81. Government Share in Other Mineral Agreements. — x x x

The Government share in financial or technical assistance agreement shall consist of,
among other things, the contractor's corporate income tax, excise tax, special
allowance, withholding tax due from the contractor's foreign stockholders arising from
dividend or interest payments to the said foreign stockholder in case of a foreign national and
all such other taxes, duties and fees as provided for under existing laws.

The collection of Government share in financial or technical assistance agreement shall


commence after the financial or technical assistance agreement contractor has fully
recovered its pre-operating expenses, exploration, and development expenditures,
inclusive. (Emphasis supplied)

Under the foregoing provisions, the Government does not receive a share in the proceeds of the
mining operation. All it receives are taxes and fees from the foreign corporation, just as in the old
concession121 and service contract122 regimes. The collection of taxes and fees cannot be considered
a return on the resources mined corresponding to beneficial ownership of the Filipino people. Taxes
are collected under the State's power to generate funds to finance the needs of the citizenry and to
advance the common weal.123 They are not a return on investment or property. Similarly, fees are
imposed under the police power primarily for purposes of regulation.124Again, they do not correspond
to a return on investment or property.

Even more galling is the stipulation in the above-quoted third paragraph that the Government's share
(composed only of taxes and fees) shall not be collected until after the foreign corporation has "fully
recovered its pre-operating expenses, exploration, and development expenditures, inclusive." In one
breath this provision virtually guarantees the foreigner a return on his investment while simultaneously
leaving the Government's (and People's) share to chance.
236

It is, therefore, clearly evident that the foregoing provisions of the Mining Act effectively transfer the
beneficial ownership over the resources covered by the agreement to a foreigner, in contravention of
the letter and spirit of the Constitution.

Consequently, the assailed Decision inescapably concluded that:

The underlying assumption in all these provisions is that the foreign contractor manages the
mineral resources, just like the foreign contractor in a service contract. 125

The Mining Act gives the foreign-owned corporation virtually complete control, not mere
"incidental" participation in management, over the entire operations.

The law is thus at its core a retention of the concession system. It still grants beneficial
ownership of the natural resources to the foreign contractor and does little to affirm the
State's ownership over them, and its supervision and control over their exploration,
development and utilization.

While agreeing that the Constitution vests the beneficial ownership of Philippine minerals with the
Filipino people, entitling them to gains, rewards and advantages generated by these minerals, the
majority opinion nevertheless maintains that the Mining Act, as implemented by DENR Administrative
Order 99-56126 (DAO 99-56), is constitutional as, so it claims, it does not "convey beneficial ownership
of any mineral resource or product to any foreign FTAA contractor." The majority opinion adds that the
State's share, as expounded by DAO 99-56, amounts to "real contributions to the economic growth
and general welfare of the country," at the same time allowing the contractor to recover "a reasonable
return on its investments in the project."

Under DAO 99-56, the "government's share" in an FTAA is divided into (1) a "basic government
share" composed of a number of taxes and fees127 and (2) an "additional government
share"128 computed according to one of three possible methods – (a) a 50-50 sharing in the
cumulative present value of cash flows,129 (b) a profit related additional government share130 or (c) an
additional share based on the cumulative net mining revenue131 – at the option of the contractor.

Thus, the majority opinion claims that the total government share, equal to the sum of the "basic
government share" and the "additional government share," will achieve "a fifty-fifty sharing – between
the government and the contractor – of net benefits from mining."

This claim is misleading and meaningless for two reasons:

First, as priorly discussed, the taxes and fees which make up the government's "basic share"
cannot be considered a return on the resources mined corresponding to the beneficial
ownership of the Filipino people. Again, they do not correspond to a return on investment or
property.

Second, and more importantly, the provisions of the Mining Act effectively allow the foreign
contractor to circumvent all the provisions of DAO 99-56, including its intended "50-50
sharing" of the net benefits from mining, and reduce government's total share to as low as
TWO percent (2%) of the value of the minerals mined.

The foreign contractor can do this because Section 39 of the Mining Act allows it to convert its FTAA
into a Mineral Production-Sharing Agreement (MPSA) by the simple expedient of reducing its equity in
the corporation undertaking the FTAA to 40%:

SECTION 39. Option to Convert into a Mineral Agreement. — The contractor has the
option to convert the financial or technical assistance agreement to a mineral
agreement at any time during the term of the agreement, if the economic viability of the
contract area is found to be inadequate to justify large-scale mining operations, after proper
notice to the Secretary as provided for under the implementing rules and regulations:
237

Provided, That the mineral agreement shall only be for the remaining period of the original
agreement.

In the case of a foreign contractor, it shall reduce its equity to forty percent (40%) in
the corporation, partnership, association, or cooperative. Upon compliance with this
requirement by the contractor, the Secretary shall approve the conversion and execute
the mineral production-sharing agreement.(Emphasis and underscoring supplied)

And under Section 80 of the Mining Act, in connection with Section 151(a) of the National Internal
Revenue Code132(Tax Code), the TOTAL GOVERNMENT SHARE in an MPSA is ONLY TWO
PERCENT (2%) of the value of the minerals. Section 80 of the Mining Act provides:

SECTION 80. Government Share in Mineral Production Sharing Agreement. — The total
government share in a mineral production sharing agreement shall be the excise tax
on mineral products as provided inRepublic Act No. 7729, amending Section 151(a) of
the National Internal Revenue Code, as amended. (Emphasis supplied)

While Section 151(a) of the Tax Code reads:

Sec. 151. Mineral Products. — (a) Rates of Tax. — There shall be levied, assessed and
collected on mineral, mineral products and quarry resources, excise tax as follows:

(1) On coal and coke, a tax of ten pesos (P10.00) per metric ton.

(2) On non-metallic minerals and quarry resources, a tax of two percent (2%) based on
the actual market value of the annual gross output thereof at the time of removal, in the case
of those locally extracted or produced; or the value used by the Bureau of Customs in
determining tariff and customs duties, net of excise tax and value-added tax, in the case of
importation.

(3) On all metallic minerals, a tax based on the actual market value of the gross output
thereof at the time of removal, in the case of those locally extracted or produced; or the value
used by the Bureau of Customs in determining tariff and customs duties, net of excise tax and
value-added tax, in the case of importation, in accordance with the following schedule:

(a) Copper and other metallic minerals:

(i) On the first three (3) years upon the effectivity of this Act, one percent
(1%);

(ii) On the fourth and fifth year, one and a half percent (1 1/2%); and

(iii) On the sixth year and thereafter, two percent (2%)

(b) Gold and chromite, two percent (2%)

(4) On indigenous petroleum, a tax of fifteen percent (15%) of the fair international market
price thereof, on the first taxable sale, such tax to be paid by the buyer or purchaser within 15
days from the date of actual or constructive delivery to the said buyer or purchaser. The
phrase 'first taxable sale, barter, exchange or similar transaction' means the transfer of
indigenous petroleum in its original state to a first taxable transferee. The fair international
market price shall be determined in consultation with an appropriate government agency.

For the purpose of this subsection, 'indigenous petroleum' shall include locally extracted
mineral oil, hydrocarbon gas, bitumen, crude asphalt, mineral gas and all other similar or
naturally associated substances with the exception of coal, peat, bituminous shale and/or
stratified mineral deposits. (Emphasis supplied)
238

By taking advantage of the foregoing provisions and selling 60% of its equity to a Filipino corporation
(such as any of the members of respondent-in-intervention Philippine Chamber of Mines) a foreign
contractor can easily reduce the total government's share (held in trust for the benefit of the Filipino
People) in the minerals mined to a paltry 2% while maintaining a 40% beneficial interest in the same.

What is more, if the Filipino corporation acquiring the foreign contractor's stake is itself 60% Filipino-
owned and 40% foreign-owned (a "60-40" Filipino corporation such as Sagittarius Mines, the putative
purchaser of WMC's 100% equity in WMCP), then the total beneficial interest of foreigners in the
mineral output of the mining concern would constitute a majority of 64%133 while the beneficial
ownership of Filipinos would, at most,134 amount to 36% – 34% for the Filipino stockholders of the 60-
40 Filipino corporation and 2% for the Government (in trust for the Filipino People).

The foregoing scheme, provided for in the Mining Act itself, is no different and indeed is virtually
identical to that embodied in Section 7.9 of the WMCP FTAA which the majority opinion itself
found to be "without a doubt grossly disadvantageous to the government, detrimental to the
interests of the Filipino people, and violative of public policy:"

x x x While Section 7.7 gives the government a 60 percent share in the net mining revenues
of WMCP from the commencement of commercial production; Section 7.9 deprives the
government of part or all of the said 60 percent. Under the latter provision, should
WMCP's foreign shareholders – who originally owned 100 percent of the equity – sell 60
percent or more of its outstanding capital stock to a Filipino citizen or corporation, the State
loses its right to receive its 60 percent share in net mining revenues under Section 7.7.

Section 7.9 provides

The percentage of Net Mining Revenues payable to the Government pursuant to Clause 7.7
shall be reduced by 1percent of Net Mining Revenues for every 1percent ownership interest
in the Contractor (i.e., WMCP) held by a Qualified Entity.

Evidently, what Section 7.7 grants to the State is taken away in the next breath by Section
7.9 without any offsetting compensation to the State. Thus, in reality, the State has no
vested right to receive any income from the FTAA for the exploration of its mineral
resources. Worse, it would seem that what is given to the State in Section 7.7 is by
mere tolerance of WMCP's foreign stockholders, who can at any time cut off the
government's entire 60 percent share. They can do so by simply selling 60 percent of
WMCP's outstanding stock to a Philippine citizen or corporation. Moreover, the
proceeds of such sale will of course accrue to the foreign stockholders of WMCP, not
to the State.

The sale of 60 percent of WMCP's outstanding equity to a corporation that is 60 percent


Filipino-owned and 40 percent foreign-owned will still trigger the operation of Section
7.9. Effectively, the State will lose its right to receive all 60 percent of the net mining
revenues of WMCP; and foreign stockholders will own beneficially up to 64 percent of
WMCP, consisting of the remaining 40percent foreign equity therein, plus the 24
percent pro-rata share in the buyer-corporation.

xxx

At bottom, Section 7.9 has the effect of depriving the State of its 60 percent share in the net
mining revenues of WMCP without any offset or compensation whatsoever. It is possible
that the inclusion of the offending provision was initially prompted by the desire to
provide some form of incentive for the principal foreign stockholder in WMCP to
eventually reduce its equity position and ultimately divest itself thereof in favor of
Filipino citizens and corporations. However, as finally structured, Section 7.9 has the
deleterious effect of depriving government of the entire 60 percent share in WMCP's
net mining revenues, without any form of compensation whatsoever. Such an outcome
is completely unacceptable.
239

The whole point of developing the nation's natural resources is to benefit the Filipino people,
future generations included. And the State as sovereign and custodian of the nation's natural
wealth is mandated to protect, conserve, preserve and develop that part of the national
patrimony for their benefit. Hence, the Charter lays great emphasis on "real contributions to
the economic growth and general welfare of the country" [Footnote 75 of the Dissent omitted]
as essential guiding principles to be kept in mind when negotiating the terms and conditions
of FTAAs.

xxx

Section 7.9 of the WMCP FTAA effectively gives away the State's share of net
mining revenues (provided for in Section 7.7) without anything in exchange. Moreover, this
outcome constitutes unjust enrichment on the part of local and foreign stockholders of
WMCP. By their mere divestment of up to 60 percent equity in WMCP in favor of Filipino
citizens and/or corporations, the local and foreign stockholders get a windfall. Their share in
the net mining revenues of WMCP is automatically increased, without their having to pay the
government anything for it. In short, the provision in question is without a doubt grossly
disadvantageous to the government, detrimental to the interests of the Filipino people,
and violative of public policy. (Emphasis supplied; italics and underscoring in the original;
footnotes omitted)

The foregoing disquisition is directly applicable to the provisions of the Mining Act. By selling 60% of
its outstanding equity to a 60% Filipino-owned and 40% foreign-owned corporation, the foreign
contractor can readily convert its FTAA into an MPSA. Effectively, the State's share in the net
benefits from mining will be automatically and drastically reduced from the theoretical 50%
anticipated under DAO 99-56 to merely 2%. What is given to the State by Section 81 and DAO
99-56 is all but eliminated by Sections 39 and 80. At the same time, foreign stockholders will
beneficially own up to 64% of the mining concern, consisting of the remaining 40% foreign
equity therein plus the 24% pro-rata share in the buyer-corporation.

It is possible that, like Section 7.9 of the WMCP FTAA, Section 39 of the Mining Act was intended to
provide some form of incentive for the foreign FTAA contractor to eventually reduce its equity position
and ultimately divest itself thereof in favor of Filipino citizens and corporations. However, the net
effect is to allow the Filipino people to be robbed of their just share in Philippine mineral
resources. Such an outcome is completely unacceptable and cannot be sanctioned by this
Court.

By this simple conversion, which may be availed of at any time, the local and foreign stockholders will
obtain a windfall at the expense of the Government, which is the trustee of the Filipino people. The
share of these stockholders in the net mining revenues from Philippine resources will be automatically
increased without their having to pay the government anything in exchange.

On this basis alone, and despite whatever other differences of opinion might exist, the majority must
concede that the provisions of the Mining Act are grossly disadvantageous to the government,
detrimental to the interests of the Filipino people, and violative of Section 2, Article XII of the
Constitution.

En passant, it is significant to note that Section 39 of the Mining Act allows an FTAA holder to covert
its agreement to an MPSA "at any time during the term of the agreement."

As any reasonable person with a modicum of business experience can readily determine, the optimal
time for the foreign contractor to convert its FTAA into an MPSA is after the completion of the
exploration phase and just before undertaking the development, construction and utilization phase.
This is because under Section 56 (a) of DAO 40-96, the requirement for a minimum investment of
Fifty Million U.S. Dollars (US$ 50,000,000.00)135 is only applicable during the development,
construction and utilization phase and NOT during the exploration phase where the foreign contractor
need only comply with the stipulated minimum ground expenditures:
240

SECTION 56. Terms and Conditions of an FTAA. — The following terms, conditions and
warranties shall be incorporated in the FTAA, namely:

a. A firm commitment, in the form of a sworn statement during the existence of the
Agreement, that the Contractor shall comply with minimum ground expenditures during
the exploration and pre-feasibility periods as follows:

Year US $/Hectare

12

22

38

48

5 18

6 23

and a minimum investment of Fifty Million US Dollars ($50,000,000.00) or its Philippine


Peso equivalent in the case of Filipino Contractor for infrastructure and development
in the contract area. If a Temporary/Special Exploration Permit has been issued prior to the
approval of an FTAA, the exploration expenditures incurred shall form part of the
expenditures during the first year of the exploration period of the FTAA.

In the event that the Contractor exceeds the minimum expenditure requirement in any one (1)
year, the amount in excess may be carried forward and deducted from the minimum
expenditure required in the subsequent year. In case the minimum ground expenditure
commitment for a given year is not met for justifiable reasons as determined by the
Bureau/concerned Regional Office, the unexpended amount may be spent on the subsequent
year(s) of the exploration period. (Emphasis supplied)

By converting its FTAA to an MPSA just before undertaking development, construction and utilization
activities, a foreign contractor further maximizes its profits by avoiding its obligation to make a
minimum investment of US$ 50,000,000.00. Assuming an exploration term of 6 years, it will have paid
out only a little over US$ 2.4 million136 in minimum ground expenditures.

Clearly, under the terms and provisions of the Mining Act, even the promised influx of tens of
millions of dollars in direct foreign investments is merely hypothetical and ultimately illusory.

Grant of Exploration Permits to Foreign


Corporations is Unconstitutional

The majority is also convinced that Section 3(aq) of the Mining Act, defining foreign corporations as a
qualified entity for the purposes of granting exploration permits, is "not unconstitutional."

The questioned provision reads:

SECTION 3. Definition of Terms. — As used in and for purposes of this Act, the following
terms, whether in singular or plural, shall mean:

xxx
241

(aq) "Qualified person" means any citizen of the Philippines with capacity to contract, or a
corporation, partnership, association, or cooperative organized or authorized for the purpose
of engaging in mining, with technical and financial capability to undertake mineral resources
development and duly registered in accordance with law at least sixty per centum (60%) of
the capital of which is owned by citizens of the Philippines: Provided, That a legally
organized foreign-owned corporation shall be deemed a qualified person for purposes
of granting an exploration permit, financial or technical assistance agreement or mineral
processing permit. (Emphasis supplied)

In support of its contention that the above-quoted provision does not offend against the Constitution,
the majority opinion states that: (1) "there is no prohibition at all against foreign or local corporations
or contractors holding exploration permits;" and (2) an "exploration permit serves a practical and
legitimate purpose in that it protects the interests and preserves the rights of the exploration permit
grantee x x x during the period of time that it is spending heavily on exploration works, without yet
being able to earn revenues x x x."

The majority opinion also characterizes an exploration permit as "an authorization for the grantee to
spend its funds on exploration programs that are pre-approved by the government." And it comments
that "[t]he State risks nothing and loses nothing by granting these permits" to foreign firms.

These contentions fail for two obvious reasons.

First, setting aside for the moment all disagreements pertaining to the construction of Section 2,
Article XII of the Constitution, the following, at the very least, may be said to have been conclusively
determined by this Court: (1) the only constitutionally sanctioned method by which a foreign entity
may participate in the natural resources of the Philippines is by virtue of paragraph 4 of Section 2,
Article XII of the Constitution; (2) said provision requires that an agreement be entered into (3)
between the President and the foreign corporation (4) for the large-scale exploration, development,
and utilization of minerals, petroleum, and other mineral oils (5) according to the general terms and
conditions provided by law, (6) based on real contributions to the economic growth and general
welfare of the country; (7) such agreements will promote the development and use of local scientific
and technical resources; and (8) the President shall notify the Congress of every contract entered into
in accordance with this provision, within thirty days from its execution.

However, by the majority opinion's express admission, the grant of an exploration permit does not
even contemplate the entry into an agreement between the State and the applicant foreign
corporation since "prior to the issuance of such FTAA or mineral agreement, the exploration permit
grantee (or prospective contractor) cannot yet be deemed to have entered into any contract or
agreement with the State."

Consequently, the grant of an exploration permit – which is not an agreement – cannot possibly be
construed as being favorably sanctioned by paragraph 4 of Section 2, Article XII of the
Constitution which refers to "agreements … involving either financial or technical assistance." Not
falling within the exception embodied in paragraph 4 of Section 2, Article XII of the Constitution, the
grant of such a permit to a foreign corporation is prohibited and the proviso providing for such grant in
Section 3 (aq) of the Mining Act is void for being unconstitutional.

Second, given the foregoing discussion on the circumvention of the State's share in an FTAA, it is
clearly evident that to allow the grant of exploration permits to foreign corporations is to allow the
whole-sale circumvention of the entire system of FTAAs mandated by the Constitution.

For Chapter IV of the Mining Act on Exploration Permits grants to the permit holder, including foreign
corporations, the principal rights conferred on an FTAA contractor during the exploration phase,
including (1) the right to enter, occupy and explore the permit area under Section 23,137 and (2)
the exclusive right to an MPSA or other mineral agreements or FTAAs upon the filing of a Declaration
of Mining Project Feasibility under Sections 23 and 24;138 but requires none of the obligations of an
FTAA – not even the obligation under Section 56 of DAO 40-96 to pay the minimum ground
expenditures during the exploration and feasibility period.139
242

Thus, all that a foreign mining company need do to further maximize its profits and further reduce the
Government's revenue from mining operations is to apply for an exploration permit and content itself
with the "smaller" permit area of 400 meridional blocks onshore (which itself is not small considering
that it is equivalent to 32,400 hectares or 324,000,000 square meters).140 It is not obligated to pay any
minimum ground expenditures during the exploration period.

Should it discover minerals in commercial quantities, it can circumvent the Fiscal Regime in DAO 99-
56 by divesting 60% of its equity in favor of a Philippine corporation and opting to enter into an MPSA.
By doing so it automatically reduces the Government's TOTAL SHARE to merely 2% of value of the
minerals mined by operation of Section 81.

And if the Philippine corporation to which it divested its 60% foreign equity is itself a 60-40 Philippine
Corporation, then the beneficial interest of foreigners in the minerals mined would be a minimum of
64%.

In light of the foregoing, Section 3 (aq), in so far as it allows the granting of exploration permits to
foreign corporations, is patently unconstitutional, hence, null and void.

II

Invalidity of the WMCP FTAA Sale of foreign


interest in WMCP to a Filipino corporation
did not render the case moot and academic.

Respondent WMCP, now renamed Tampakan Mineral Resources Corporation, submits that the case
has been rendered moot since "[e]xcept for the nominal shares of directors, 100% of TMRC's share
are now owned by Sagittarius Mines, which is a Filipino-owned corporation. More than 60% of the
equity of Sagittarius is owned by Filipinos or Filipino-owned corporations."141 This Court initially
reserved judgment on this issue.142

Petitioner invokes by analogy the rule that where land is invalidly transferred to an alien who
subsequently becomes a Filipino citizen or transfers it to one, the infirmity in the original transaction is
considered cured and the title of the transferee is rendered valid, citing Halili v. Court of
Appeals.143 The rationale for this rule is that if the ban on aliens from acquiring lands is to preserve
the nation's lands for future generations of Filipinos, that aim or purpose would not be thwarted but
achieved by making lawful the acquisition of real estate by Filipino citizens. 144

Respondent WMCP's analogy is fallacious. Whether the legal title to the corporate vehicle holding the
FTAA has been transferred from a foreigner to a Filipino is irrelevant. What is relevant is whether a
foreigner has improperly and illegally obtained an FTAA and has therefore benefited from the
exploration, development or utilization of Philippine natural resources in a manner contrary to the
provisions of the Constitution.

As above-stated the doctrine enunciated in Halili is based on the premise that the purpose of the
Constitution in prohibiting alien ownership of agricultural land is to retain the ownership or legal
title of the land in the hands of Filipinos. This purpose is not identical or even analogous to that in
Section 2, Article XII of the Constitution. As priorly discussed, the primary purpose of the provisions
on National Patrimony is to preserve to the Filipino people the beneficial ownership of their natural
resources – i.e. the right to the gains, rewards and advantages generated by their natural resources.
Except under the terms of Section 2, Article XII, foreigners are prohibited from involving themselves in
the exploration, development or utilization of these resources, much less from profiting from them.

Divestment by a foreigner of an illegally acquired right to mine Philippine resources does not alter the
illegal character of the right being divested or sold. Indeed, such divestment or sale is obviously a
method by which the foreigner may derive pecuniary benefit from his unlawful act since he receives
payment for his illegally acquired interest in the country's natural resources.
243

To rule otherwise would be to condone, even to invite, foreign entities to obtain Philippine mining
interests in violation of the Constitution with the assurance that they can escape liability and at the
same time make a tidy sum by later selling these interests to Filipinos. This is nothing less than
allowing foreign speculation in Philippine natural resources. Worse, there is the very real possibility
that these foreign entities may intentionally inflate the value of their illegally–acquired mineral rights to
the detriment of their Filipino purchasers as the past Bre-X scandal145 and recent Shell oil reserve
controversy146 vividly illustrate.

To allow a foreigner to profit from illegally obtained mining rights or FTAAs subverts and circumvents
the letter and intent of Article XII of the Constitution. It facilitates rather than prevents the rape and
plunder of the nation's natural resources by unscrupulous neo-colonial entities. It thwarts, rather than
achieves, the purpose of the fundamental law.

As applied to the facts of this case, respondent WMCP, in essence, claims that now that the operation
and management of the WMCP FTAA is in the hands of a Filipino company, no serious question as to
the FTAA's validity need arise.

On the contrary, this very fact – that WMC has sold its 100% interest in WMCP to a Filipino company
for US$10,000,000.00 – directly leads to some very serious questions concerning the WMCP FTAA
and its validity. First, if a Filipino corporation is capable of undertaking the terms of the FTAA, why
was an agreement with a foreign owned corporation entered into in the first place? Second, does not
the fact that, as alleged by petitioners147 and admitted by respondent WMCP,148 Sagittarius, WMCP's
putative new owner, is capitalized at less than half the purchase price 149 of WMC's shares in WMCP,
a strong indication that Sagittarius is merely acting as the dummy of WMC? Third, if indeed WMCP
has, to date, spent US$40,000,000.00 in the implementation of the FTAA, as it claims,150 why did
WMC sell 100% of its shares in WMCP for only US$10,000,000.00? Finally, considering that, as
emphasized by WMCP,151 "payment of the purchase price by Sagittarius to WMC will come only after
the commencement of commercial production," hasn't WMC effectively acquired a beneficial interest
in any minerals mined in the FTAA area to the extent of US$10,000,000.00? If so, is the acquisition of
such a beneficial interest by a foreign corporation permitted under our Constitution?

Succinctly put, the question remains: What is the validity of the FTAA by which WMC, a fully foreign
owned corporation, has acquired a more than half billion peso152 interest in Philippine mineral
resources located in a contract area of 99,387 (alleged to have later been reduced to
30,000)153 hectares of land spread across the four provinces of South Cotabato, Sultan Kudarat,
Davao del Sur and North Cotabato?

Clearly then, the issues of this case have not been rendered moot by the sale of WMC's 100%
interest in WMCP to a Filipino corporation, whether the latter be Sagittarius or Lepanto. If the FTAA is
held to be valid under the Constitution, then the sale is valid and, more importantly, WMC's
US$10,000,000.00 interest in Philippine mineral deposit, arising as it did from the sale and its prior
100% ownership of WMCP, is likewise valid. However, if the FTAA is held to be invalid, then neither
WMC's interest nor the sale which gave rise to said interest is valid for no foreigner may profit from
the natural resources of the Republic of the Philippines in a manner contrary to the terms of
the Philippine Constitution. If held unconstitutional, the WMCP FTAA is void ab initio for being
contrary to the fundamental law and no rights may arise from it, either in favor of WMC or its Filipino
transferee.

Evidently, the transfer of the shares in WMCP from WMC Resources International Pty. Ltd. (WMC), a
foreign-owned corporation, to a Filipino-owned one, whether Sagittarius or Lepanto, now presently
engaged in a dispute over said shares,154 did not "cure" the FTAA nor moot the petition at bar. On the
contrary, it is the Decision in this case that rendered those pending cases moot for the invalidation of
the FTAA leaves Sagittarius and Lepanto with nothing to dispute.

Terms of the WMCP FTAA are


contrary to the Constitution and
render said FTAA null and void.
244

The WMCP FTAA is clearly contrary to the agreements provided for in Section 2, Article XII of the
Constitution. In the Decision under reconsideration, this Court observed:

Section 1.3 of the WMCP FTAA grants WMCP "the exclusive right to explore, exploit, utilise[,]
process and dispose of all Minerals products and by-products thereof that may be produced
from the Contract Area." The FTAA also imbues WMCP with the following rights:

(b) to extract and carry away any Mineral samples from the Contract area for the purpose of
conducting tests and studies in respect thereof;

(c) to determine the mining and treatment processes to be utilized during the
Development/Operating Period and the project facilities to be constructed during the
Development and Construction Period;

(d) have the right of possession of the Contract Area, with full right of ingress and egress and
the right to occupy the same, subject to the provisions of Presidential Decree No. 512 (if
applicable) and not be prevented from entry into private lands by surface owners and/or
occupants thereof when prospecting, exploring and exploiting for minerals therein;

xxx

(f) to construct roadways, mining, drainage, power generation and transmission facilities and
all other types of works on the Contract Area;

(g) to erect, install or place any type of improvements, supplies, machinery and other
equipment relating to the Mining Operations and to use, sell or otherwise dispose of, modify,
remove or diminish any and all parts thereof;

(h) enjoy, subject to pertinent laws, rules and regulations and the rights of third Parties,
easement rights and the use of timber, sand, clay, stone, water and other natural resources in
the Contract Area without cost for the purposes of the Mining Operations;

xxx

(l) have the right to mortgage, charge or encumber all or part of its interest and obligations
under this Agreement, the plant, equipment and infrastructure and the Minerals produced
from the Mining Operations;

x x x.

All materials, equipment, plant and other installations erected or placed on the Contract Area
remain the property of WMCP, which has the right to deal with and remove such items within
twelve months from the termination of the FTAA.

Pursuant to Section 1.2 of the FTAA, WMCP shall provide "[all] financing, technology,
management and personnel necessary for the Mining Operations." The mining company
binds itself to "perform all Mining Operations . . . providing all necessary services, technology
and financing in connection therewith," and to "furnish all materials, labour, equipment and
other installations that may be required for carrying on all Mining Operations." WMCP may
make expansions, improvements and replacements of the mining facilities and may add such
new facilities as it considers necessary for the mining operations.

These contractual stipulations, taken together, grant WMCP beneficial ownership over natural
resources that properly belong to the State and are intended for the benefit of its citizens.
These stipulations are abhorrent to the 1987 Constitution. They are precisely the vices that
the fundamental law seeks to avoid, the evils that it aims to suppress. Consequently, the
contract from which they spring must be struck down.155 (Citations omitted)
245

Indeed, save for the fact that the contract covers a larger area, the subject FTAA is actually a mineral
production sharing agreement. Respondent WMCP admitted as much in its Memorandum.156 The first
paragraph of Section 2, Article XII of the Constitution, however, allows this type of agreement only
with Filipino citizens or corporations.

That the subject FTAA is void for having an unlawful cause bears reaffirmation. In onerous contracts
the cause is understood to be, for each contracting party, the prestation or promise of a thing or
service by the other.157 On the part of WMCP, a foreign-owned corporation, the cause was to extend
not only technical or financial assistance but management assistance as well. The management
prerogatives contemplated by the FTAA are not merely incidental to the two other forms of
assistance, but virtually grant WMCP full control over its mining operations. Thus, in Section 8.3 158 of
the FTAA, in case of a dispute between the DENR and WMCP, it is WMCP's decision which will
prevail.

The questioned FTAA also grants beneficial ownership over Philippine natural resources to WMCP,
which is prohibited from entering into such contracts not only by the fourth paragraph of Section 2,
Article XII of the Constitution, but also by the first paragraph, the FTAA practically being a production-
sharing agreement reserved to Filipinos.

Contracts whose cause is contrary to law or public policy are inexistent and void from the
beginning.159 They produce no effect whatsoever.160 They cannot be ratified,161 and so cannot the
WMCP FTAA.

The terms of the WMCP FTAA effectively give away


the Beneficial Ownership of Philippine minerals

As previously observed, the majority opinion finds Section 7.9. of the WMCP FTAA to be "grossly
disadvantageous to the government, detrimental to the interests of the Filipino people, and violative of
public policy" since it "effectively gives away the State's share of net mining revenues (provided for in
Section 7.7) without anything in exchange."

It likewise finds Section 7.8(e) of the WMCP FTAA to be invalid. Said provision states:

7.8 The Government Share shall be deemed to include all of the following sums:

xxx

(e) an amount equivalent to whatever benefits that may be extended in the


future by the Government to the Contractor or to financial or technical
assistance agreement contractors in general. (Emphasis supplied)

And in its own estimation:

Section 7.8(e) is out of place in the FTAA. This provision does not make any sense why, for
instance, money spent by the government for the benefit of the contractor in building roads
leading to the mine site should still be deductible from the State's share in net mining
revenues. Allowing this deduction results in benefiting the contractor twice over. To do
so would constitute unjust enrichment on the part of the contractor at the expense of
the government, since the latter is effectively being made to pay twice for the same
item. For being grossly disadvantageous and prejudicial to the government and
contrary to public policy, Section 7.8(e) is undoubtedly invalid and must be declared to
be without effect. xxx (Emphasis supplied; citations omitted; underscore in the original)

The foregoing estimation notwithstanding, the majority opinion declines to invalidate the WMCP FTAA
on the theory that Section 7.9 and 7.8 are separable from the rest of the agreement, which may
supposedly be given effect without the offending provisions.
246

As previously discussed, the same deleterious results are easily achieved by the foreign contractor's
conversion of its FTAA into an MPSA under the provisions of the Mining Act. Hence, merely striking
out Sections 7.9 and 7.8(e) of the WMCP FTAA will not suffice; the provisions pertaining to FTAAs in
the Mining Act must be stricken out for being unconstitutional as well.

Moreover, Section 7.8 (e) and 7.9 are not the only provisions of the WMCP FTAA which convey
beneficial ownership of mineral resources to a foreign corporation.

Under Section 10.2 (l) of the WMCP FTAA, the foreign FTAA contractor shall have the right to
mortgage and encumber, not only its rights and interests in the FTAA, but the very minerals
themselves:

10.2 Rights of Contractor

The Government agrees that the Contractor shall:-

xxx

(l) have the right to mortgage, charge or encumber all or part of its interest and obligations
under this Agreement, the plant, equipment and infrastructure and the Minerals produced
from the Mining Operations; (Emphasis supplied)

Although respondents did not proffer their own explanation, the majority opinion theorizes that the
foregoing provision is necessitated by the conditions that may be imposed by creditor-banks on the
FTAA contractor:

xxx I believe that this provision may have to do with the conditions imposed by the creditor-
banks of the then foreign contractor WMCP to secure the lendings made to the latter.
Ordinarily, banks lend not only on the security of mortgages on fixed assets, but also on
encumbrances of goods produced that can easily be sold and converted into cash that can be
applied to the repayment of loans. Banks even lend on the security
of accounts receivable that are collectible within 90 days. (Citations omitted; underscore in
the original)

It, however, overlooks the provision of Art. 2085 of the Civil Code which enumerates the essential
requisites of a contract of mortgage:

Art. 2085. The following requisites are essential to the contracts of pledge
and mortgage:

(1) That they be constituted to secure the fulfillment of a principal obligation;

(2) That the pledgor or mortgagor be the absolute owner of the thing pledged or
mortgaged;

(3) That the persons constituting the pledge or mortgage have the free disposal of their
property, and in the absence thereof, that they be legally authorized for the purpose.

Third persons who are not parties to the principal obligation may secure the latter by pledging
or mortgaging their own property. (Emphasis and underscoring supplied)

From the foregoing provision of law, it is abundantly clear


that only the absolute owner of the minerals has theright to mortgage the same, and under Sec
tion 2, Article XII of the Constitution the absolute owner of theminerals is none other than the
State. While the foreign FTAA contractor may have an interest in the proceeds of the minerals, it does
not acquire ownership over the minerals themselves.
247

Put differently, the act of mortgaging the minerals is an act of ownership, which, under the
Constitution, is reserved solely to the State. In purporting to grant such power to a foreign FTAA
contractor, Section 10.2 (l) of the WMCP FTAA clearly runs afoul of the Constitution.

Moreover, it bears noting that to encumber natural resources of the State to secure a foreign FTAA
contractor's obligations is anomalous since Section 1.2 of the WMCP FTAA provides that
"[a]ll financing, technology, management and personnel necessary for the Mining Operations shall be
provided by the Contractor."

Indeed, even the provisions of the Mining Act, irredeemably flawed though they may be, require that
the FTAA contractor have the financial capability to undertake the large-scale exploration,
development and utilization of mineral resources in the Philippines; 162 and, specifically, that the
contractor warrant that it has or has access to all the financing required to promptly and effectively
carry out the objectives of the FTAA.163

Under Section 10.2 (e) of the WMCP FTAA, the foreign FTAA Contractor has the power to require the
Government to acquire surface rights in its behalf at such price and terms acceptable to it:

10.2 Rights of Contractor

The Government agrees that the Contractor shall:-

xxx

(e) have the right to require the Government at the Contractor's own cost, to purchase
or acquire surface areas for and on behalf of the Contractor at such price and terms as
may be acceptable to the Contractor. At the termination of this Agreement such areas shall
be sold by public auction or tender and the Contractor shall be entitled to reimbursement
of the costs of acquisition and maintenance, adjusted for inflation, from the proceeds of
sale; (Emphasis supplied)

Petitioners, in their Memorandum, point out that pursuant to the foregoing, the foreign FTAA
contractor may compel the Government to exercise its power of eminent domain to acquire the title to
the land under which the minerals are located for and in its behalf.

The majority opinion, however, readily accepts the explanation proffered by respondent WMCP, thus:

Section 10.2 (e) sets forth the mechanism whereby the foreign-owned contractor, disqualified
to own land, identifies to the government the specific surface areas within the FTAA contract
area to be acquired for the mine infrastructure. The government then acquires ownership of
the surface land areas on behalf of the contractor, in order to enable the latter to proceed to
fully implement the FTAA.

The contractor, of course, shoulders the purchase price of the land. Hence, the provision
allows it, after the termination of the FTAA to be reimbursed from proceeds of the sale of the
surface areas, which the government will dispose of through public bidding.

And it concludes that "the provision does not call for the exercise of the power of eminent domain"
and the determination of just compensation.

The foregoing arguments are specious.

First, the provision in question clearly contemplates a situation where the surface area is not already
owned by the Government – i.e. when the land over which the minerals are located is owned by some
private person.
248

Second, the logical solution in that situation is not, as asserted by respondent WMCP, to have the
Government purchase or acquire the land, but for the foreign FTAA contractor to negotiate a lease
over the property with the private owner.

Third, it is plain that the foreign FTAA contractor would only avail of Section 10.2 (e) if, for some
reason or another, it is unable to lease the land in question at the price it is willing to pay. In that
situation, it would have the power under Section 10.2 (e) to compel the State, as the only entity which
can legally compel the landowner to involuntarily part with his property, to acquire the land at a price
dictated by the foreign FTAA contractor.

Clearly, the State's power of eminent domain is very much related to the practical workings of Section
10.2 (e) of the WMCP FTAA. It is the very instrument by which the contractor assures itself that it can
obtain the "surface right" to the property at a price of its own choosing. Moreover, under Section 60 of
DAO 40-96, the contractor may, after final relinquishment, hold up to 5,000 hectares of land in this
manner.

More. While the foreign FTAA contractor advances the purchase price for the property, in reality it
acquires the "surface right" for free since under the same provision of the WMCP FTAA it is entitled to
reimbursement of the costs of acquisition and maintenance, adjusted for inflation. And as if the
foregoing were not enough, when read together with Section 3.3,164 the foreign FTAA contractor
would have the right to hold the "surface area" for a maximum of 50 years, at its option.

In sum, by virtue of Sections 10.2 (e) and 3.3. of the WMCP


FTAA, the foreign FTAA contractor is given thepower to hold inalienable mineral land of up to
5,000 hectares, with the assistance of the State's power ofeminent domain, free of charge, for
a period of up to 50 years in contravention of Section 3, Article XII of theConstitution:

Section 3. Lands of the public domain are classified into agricultural, forest or timber, mineral
lands, and national parks. Agricultural lands of the public domain may be further classified by
law according to the uses which they may be devoted. Alienable lands of the public
domain shall be limited to agricultural lands. Private corporations or associations may
not hold such alienable lands of the public domain except by lease, for a period not
exceeding twenty-five years, renewable for not more than twenty-five years, and not to
exceed one thousand hectares in area. Citizens of the Philippines may lease not more than
five hundred hectares, or acquire not more than twelve hectares thereof by purchase,
homestead, or grant.

Taking into account the requirements of conservation, ecology, and development, and subject
to the requirements of agrarian reform, the Congress shall determine, by law, the size of
lands of the public domain which may be acquired, developed, held, or leased and the
conditions therefor. (Emphasis supplied)

Taken together, the foregoing provisions of the WMCP FTAA amount to a conveyance to a foreign
corporation of the beneficial ownership of both the minerals and the surface rights to the same in
contravention of the clear provisions of the Constitution.

The majority opinion posits that "[t]he acquisition by the State of land for the contractor is just to
enable the contractor to establish its mine site, build its facilities, establish a tailings pond, set up its
machinery and equipment, and dig mine shafts and tunnels, etc." It thus concludes that "5,000
hectares is way too much for the needs of a mining operator."

Evidently, the majority opinion does not take into account open pit mining. Open pit or opencut
mining, as differentiated from methods that require tunneling into the earth, is a method of extracting
minerals by their removal from an open pit or borrow;165 it is a mine working in which excavation is
performed from the surface.166 It entails a surface mining operation in which blocks of earth are dug
from the surface to extract the ore contained in them. During the mining process, the surface of the
land is excavated forming a deeper and deeper pit until the end of mining operations. 167 It is used
extensively in mining metal ores, copper, gold, iron, aluminum168 – the very minerals which the
249

Philippines is believed to possess in vast quantities; and is considered the most cost-effective mining
method.169

Furthermore, considering that FTAAs deal with large scale exploration, development and utilization of
mineral resources and that the original contract area of the WMCP FTAA was 99,387 hectares, an
open pit mining operation covering a total of 5,000 hectares is not outside the realm of possibility.

In any event, regardless of what the majority opinion considers "way too much" (or too little), it is
undisputed that under Section 60 of DAO 40-96, which is among the enactments under review, the
contractor may, after final relinquishment, hold up to 5,000 hectares of land. And, under Section 3.3.
of the WMCP FTAA, it may do so for a term of 25 years automatically renewable for another 25 years,
at the option of the contractor.

The majority opinion also argues that, although entitled to reimbursement of its acquisition cost at the
end of the contract term, the FTAA contractor does not acquire its surface rights for free since "the
contractor will have been cash-out for the entire duration of the term of the contract – 25 to 50 years,
depending," thereby foregoing any interest income he might have earned. This is the "opportunity
cost" of the contractor's decision to use its money to acquire the surface rights instead of leaving it in
the bank.

The majority opinion does not consider the fact that "opportunity cost" is more theoretical rather than
actual and, for that reason, is not an allowable deduction from gross income in an income statement.
In layman's terms it is equivalent to "the value of the chickens that might have been hatched if only
the cook had not scrambled the eggs." Neither does it consider the fact that the contractor's foregone
interest income does not find its way to the pockets of either the previous land owner (in this case, the
Bugal B'Laans) or the State.

But even if the contractor does incur some opportunity cost in holding the surface rights for 35 to 50
years. The fact remains that, under the terms of the WMCP FTAA, the contractor is given the
power to hold inalienable mineral land of up to 5,000 hectares, with the assistance of the
State's power of eminent domain for a period of up to 50 years in contravention of Section 3,
Article XII of the Constitution.

Clearly, Section 3 and 10.2 (e) of the WMCP FTAA in conjunction with Section 60 of DAO 40-96,
amount to a conveyance to a foreign corporation of the beneficial ownership of both the minerals and
the surface rights over the same, in contravention of the clear provisions of the Constitution.

The terms of the WMCP FTAA abdicate all control over the
mining operation in favor of the foreign FTAA contractor

The majority opinion's defense of the constitutionality of Section 8.1, 8.2, 8.3 of the WMCP FTAA is
similarly unpersuasive. These Sections provide:

8.1 The Secretary shall be deemed to have approved any Work Programme or Budget
or variation thereof submitted by the Contractor unless within sixty (60) days after
submission by the Contractor the Secretary gives notice declining such approval or
proposing a revision of certain features and specifying its reasons therefore ("the
Rejection Notice").

8.2 If the Secretary gives a Rejection Notice the Parties shall promptly meet and endeavour
to agree on amendments to the Work Programme or budget. If the Secretary and the
Contractor fail to agree on the proposed revision within 30 days from delivery of the
Rejection Notice then the Work Programme or Budget or variation thereof proposed by
the Contractor shall be deemed approved so as not to unnecessarily delay the
performance of this Agreement.

Even measured against the majority opinion's standards of control – i.e. either (1) the power to set
aside, reverse, or modify plans and actions of the contractor; or (2) regulatory control – the foregoing
250

provisions cannot pass muster. This is because, by virtue of the foregoing provisions, the foreign
FTAA contractor has unfettered discretion to countermand the orders of its putative regulator, the
DENR.

Contrary to the majority's assertions, the foregoing provisions do not provide merely temporary or
stop-gap solutions. The determination of the FTAA contractor permanently reverses the "Rejection
Notice" of the DENRsince, by the majority opinion's own admission, there is no available remedy for
the DENR under the agreement except to seek the cancellation of the same.

Indeed, the justification for the foregoing provisions is revealing:

xxx First, avoidance of long delays in these situations will undoubtedly redound to the benefit
of the State as well as to the contractor. Second, who is to say that the work program or
budget proposed by the contractor and deemed approved under Clause 8.3 would not
be the better or more reasonable or more effective alternative? The contractor, being
the "insider," as it were, may be said to be in a better position than the State – an
outsider looking in – to determine what work program or budget would be appropriate,
more effective, or more suitable under the circumstances. (Emphasis and underscoring
supplied)

Both reasons tacitly rely on the unstated assumption that the interest of the foreign FTAA contractor
and that of the Government are identical. They are not.

Private businesses, including large foreign-owned corporations brimming with capital and technical
expertise, are primarily concerned with maximizing the pecuniary returns to their owners or
shareholders. To this extent, they can be relied upon to pursue the most efficient courses of action
which maximize their profits at the lowest possible cost.

The Government, on the other hand, is mandated to concern itself with more than just narrow self-
interest. With respect to the nation's natural wealth, as the majority opinion points out, the
Government is mandated to preserve, protect and even maximize the beneficial interest of the Filipino
people in their natural resources. Moreover, it is directed to ensure that the large-scale exploration,
development and utilization of these resources results in real contributions to the economic growth
and general welfare of the nation. To achieve these broader goals, the Constitution mandates that the
State exercise full control and supervision over the exploration, development and utilization of the
country's natural resources.

However, taking the majority opinion's reasoning to its logical conclusion, the business "insider's
opinion" would always be superior to the Government's administrative or regulatory determination with
respect to mining operations. Consequently, it is the foreign contractor's opinion that should always
prevail. Ultimately, this means that, at least for the majority, foreign private business interests
outweigh those of the State – at least with respect to the conduct of mining operations.

Indeed, in what other industry can the person regulated permanently overrule the administrative
determinations of the regulatory agency?

To any reasonable mind, the absence of an effective means to enforce even administrative
determinations over an FTAA contractor, except to terminate the contract itself, falls far too short of
the concept of "full control and supervision" as to cause the offending FTAA to fall outside the ambit of
Section 2, Article XII of the Constitution.

Verily, viewed in its entirety, the WMCP FTAA cannot withstand a rigid constitutional scrutiny
since, by its provisions, it conveys both the beneficial ownership of Philippine minerals and
control over their exploration, development and utilization to a foreign corporation. Being
contrary to both the letter and intent of Section 2, Article XII of the Constitution, the WMCP
FTAA must be declared void and of no effect whatsoever.

A Final Note
251

For over 350 years, the natural resources of this nation have been under the control and domination
of foreign powers – whether political or corporate. Philippine mineral wealth, viciously wrenched from
the bosom of the motherland, has enriched foreign shores while the Filipino people, to whom such
wealth justly belongs, have remained impoverished and unrecompensed.

Time and time again the Filipino people have sought an end to this intolerable situation. From 1935
they have struggled to assert their legal control and ownership over their patrimony only to have their
efforts repeatedly subverted – first, by the parity amendment to the 1935 Constitution and
subsequently by the service contract provision in the 1973 Constitution.

It is not surprising that an industry, overly dependent on foreign support and now in decline, should
implore this Court to reverse itself if only to perpetuate its otherwise economically unsustainable
conduct. It is even understandable, however regrettable, that a government, strapped for cash and in
the midst of a self-proclaimed fiscal crisis, would be inclined to turn a blind eye to the consequences
of unconstitutional legislation in the hope, however false or empty, of obtaining fabulous amounts of
hard currency.

But these considerations should not outweigh the Constitution.

As always, the one overriding consideration of this Court should be the will of the sovereign Filipino
people as embodied in their Constitution. The Constitution which gives life to and empowers this
Court. The same Constitution to which the members of this Court have sworn their unshakable loyalty
and their unwavering fidelity.

Now, the unmistakable letter and intent of the 1987 Constitution notwithstanding, the majority of this
Court has chosen to reverse its earlier Decision which, to me, would once again open the doors to
foreign control and ownership of Philippine natural resources. The task of reclaiming Filipino control
over Philippine natural resources now belongs to another generation.

ACCORDINGLY, I vote to deny respondents' Motions for Reconsideration.

SEPARATE OPINION

TINGA, J.:

The Constitution was crafted by men and women of divergent backgrounds and varying ideologies.
Understandably, the resultant document is accommodative of these distinct, at times competing
philosophies. Untidy as any mélange would seem, our fundamental law nevertheless hearkens to the
core democratic ethos over and above the obvious inconveniences it spawns.

However, when the task of judicial construction of the Constitution comes to fore, clarity is demanded
from this Court. In turn, there is a need to balance and reconcile the diverse views that animate the
provisions of the Constitution, so as to effectuate its true worth as an instrument of national unity and
progress.

The variances and consequent challenges are vividly reflected in Article XII of the Constitution on
National Patrimony, in a manner akin to Article II on Declaration of Principles and State Policies.
Some of the provisions impress as protectionist, yet there is also an undisguised accommodation of
liberal economic policies. Section 2, Article XII,1 the provision key to this case, is one such Janus-
faced creature. It seems to close the door on foreign handling of our natural resources, but at the
same time it leaves open a window for alien participation in some aspects. The central question
before us is how wide is the entry of opportunity created by the provision.
252

My vote on the motions for reconsideration is hinged on a renewed exegesis of Section 22 of Article
XII in conjunction with the proper understanding of the nature of the power vested on the President
under Section 2. It has to be appreciated in relation to the inherent functions of the executive branch
of government.

The Contract-Making Power of the President

While all government authority emanates from the people, the breadth and depth of such authority are
not brought to bear by direct popular action, but through representative government in accord with the
principles of republicanism.3 By investiture of the Constitution, the function of executive power is
parceled solely to the duly elected President.4 The Constitution contains several express
manifestations of executive power, such as the provision on control over all executive departments,
bureaus and offices,5 as well as the so-called "Commander-in-Chief" clause.6

Yet it has likewise been recognized in this jurisdiction that "executive power" is not limited to such
powers as are expressly granted by the Constitution. Marcos v. Manglapus7 concedes that the
President has powers other than those expressly stated under the Constitution, 8 and thus implies that
these powers may be exercised without being derivative from constitutional authority. 9 The
precedental value of Marcos v. Manglapus may be controvertible,10 but the cogency of its analysis of
the scope of executive power is indisputable. Neither is the concept of plenary executive power novel,
as discussed by Justice Irene Cortes in her ponencia:

It has been advanced that whatever power inherent in the government that is neither
legislative nor judicial has to be executive. Thus, in the landmark decision of Springer v.
Government of the Philippine Islands, 277 U.S. 189 (1928), on the issue of who between the
Governor-General of the Philippines and the Legislature may vote the shares of stock held by
the Government to elect directors in the National Coal Company and the Philippine National
Bank, the U.S. Supreme Court, in upholding the power of the Governor-General to do so,
said:

. . . Here the members of the legislature who constitute a majority of the "board" and
"committee" respectively, are not charged with the performance of any legislative
functions or with the doing of anything which is in aid of performance of any such
functions by the legislature. Putting aside for the moment the question whether the
duties devolved upon these members are vested by the Organic Act in the Governor-
General, it is clear that they are not legislative in character, and still more clear that
they are not judicial. The fact that they do not fall within the authority of either of these
two constitutes logical ground for concluding that they do fall within that of the
remaining one among which the powers of government are divided . . . [At 202-203;
emphasis supplied.]

We are not unmindful of Justice Holmes' strong dissent. But in his enduring words of dissent
we find reinforcement for the view that it would indeed be a folly to construe the powers of a
branch of government to embrace only what are specifically mentioned in the Constitution:

The great ordinances of the Constitution do not establish and divide fields of black
and white. Even the more specific of them are found to terminate in a penumbra
shading gradually from one extreme to the other. . . .

xxx xxx xxx

It does not seem to need argument to show that however we may disguise it by
veiling words we do not and cannot carry out the distinction between legislative and
executive action with mathematical precision and divide the branches into watertight
compartments, were it ever so desirable to do so, which I am far from believing that it
is, or that the Constitution requires.[At 210-211.]11
253

Such general power has not been diminished notwithstanding the avowed intent of some of the
framers of the 1987 Constitution to limit the powers of the President as a reaction to abuses under
President Marcos, for as the Court noted, "the result was a limitation of the specific powers of the
President, particularly those relating to the commander-in-chief clause, but not a diminution of the
general grant of executive power."12 The critical perspective of this case should spring from a
recognition of this elemental fact.

Undeniably, the particular power now in question is expressly provided for by Section 2, Article XII of
the Constitution. Still, it originates from the concept of executive power that is not explicitly provided
for by the Constitution. As a necessary incident of the functions of the executive office, it can be
concluded that the President has the authority to enter into contracts in behalf of the State in matters
which are not denied him or her or not otherwise assigned to the other great branches of government,
even if such general power is not categorically recognized in the Constitution. Among these traditional
functions of the executive branch is the power to determine economic policy.

As once noted by Justice Feliciano, the Republic of the Philippines is itself a body corporate and
juridical person vested with the full panoply of powers and attributes which are compendiously
described as "legal personality."13 As "Chief of State" the President is also regarded as the head of
this body corporate,14 and thus is capacitated to represent the State when engaging with other
entities. Such executive function, in theory, does not require a constitutional provision, or even a
Constitution, in order to be operative. It is a power possessed by every duly constituted presidency
starting with Aguinaldo's. This faculty is complementary to the traditional regard of a Head of State as
emblematic of the State he/she represents.

The power to contract in behalf of the State is clearly an executive function, as opposed to legislative
or judicial. This is easily discernible through the process of exclusion. The other branches of
government — the legislative and the judiciary — are not similarly capacitated since their core
functions pertain to legislating and adjudicating respectively.

However, I am not making any pretense that such executive power to contract is unimpeachable or
limitless. The Constitution frowns on unchecked executive power, mandating in broad strokes, the
power of judicial review15 and legislative oversight.16 The Constitution itself may expressly restrict the
exercise of any sort of executive function. Section 2 undeniably constrains the exercise of the
executive power to contract in several regards.

Constitutional Limitations under Section 2, Article XII

What are the express limitations under Section 2 on the power of the executive to contract with
foreign corporations regarding the exploration, development and utilization of our natural resources?

There are two fundamental restrictions, both of which are asserted in the second paragraph of
Section 2. These are that the State retains legal ownership of all natural resources,17 and that the
State shall have full control and supervision over the exploration, development and utilization of
natural resources.18 These key postulates are facially broad and warrant clarification. They also
predicate several specific restrictions laid down in the fourth paragraph of Section 2 on the power of
the President to enter into agreements with foreign corporations. These specific limitations are as
follows:

First, the natural resources that may be subject of the agreement are a limited class, particularly
minerals, petroleum, and other mineral oils. Among the natural resources which are excluded from
these agreements are lands of the public domain, waters, coal, fisheries, forests or timbers, wildlife,
flora and fauna. Most notable of the exclusions are forests and timbers which are in all respects
expressly limited to Filipinos.

It is noteworthy that a previous version of the fourth paragraph of Section 2 deliberated upon during
the 1987 Constitutional Commission allowed agreements with foreign-owned corporations with
respect to all classes of natural resources.19 However, on the initiative of Commissioner (now Chief
Justice) Davide, the provision was amended to limit the scope of such agreements to minerals,
254

petroleum and other mineral oils, which Commissioner Davide recognized as "those particular areas
where Filipino capital may not be sufficient."20

The exclusion of timber resources from the scope of financial/technical assistance


agreements marks a significant distinction from the service contracts of old. This does not
come as a surprise, considering well-reported abuses under the old regime of issuing timber
licensing agreements, which numbered in the thousands prior to the 1987 Constitution. On the
other hand, no similar extensive collateral damage has been reported for the petroleum and
mining industry, capital-intensive industries whose potential for government revenues in
billions of pesos has long been sought after by the State.21 Hence, the variance in treatment
from the timber industry and the rest of the natural resources.

Second, these agreements with foreign-owned corporations can only be entered into for only large-
scale exploration, development and utilization of minerals, petroleum, and other mineral oils.

Third, it is only the President who may enter into these agreements. This is another pronounced
change from the 1973 Constitution, which allowed private persons to enter into service contracts with
foreign corporations.

Fourth, these agreements must be in accord with the general terms and conditions provided by law.
This proviso by itself, and more so when taken together, as it should, with another provision, 22 entails
legislative intervention and affirmance in the exercise of this executive power. While it is the President
who enters into these contracts, he/she must act within such terms and conditions as may be
prescribed by Congress through legislation. The value of legislative input as a means of influencing
policy should not be discounted. Policy initiatives grounded on particular economic ideologies may
find enactment through legislation when approved by the necessary majorities in Congress.
Legislative work includes consultative processes with persons of diverse interests, assuring that
economic decisions need not be made solely from an ivory tower. There is also the possible sanction
of repudiation by the voters of legislators who prove insensate to the economic concerns of their
constituents.

Fifth, the President is mandated to base the decision of entering into these agreements on "real
contributions to the economic growth and general welfare of the country." In terms of real limitations,
this condition has admittedly little effect. The discretion as to whether or not to enter into these
agreements is vested solely by the Constitution in the President, and such exercise of discretion,
pertaining as it does to the political wisdom of a co-equal branch, generally deserves respect from the
courts.

The above conditionalities, particularly the first three, effect the desire of the framers of the 1987
Constitution to limit foreign participation in natural resource-oriented enterprises. They provide a vivid
contrast to the 1973 Constitution, which permitted private persons to enter into service contracts for
financial, technical, management, or other forms of assistance with any person or entity, including
foreigners, and for the exploration or utilization of any of the natural resources. 23 These requisites
imposed by the 1987 Constitution, which are significantly more onerous than those laid down in the
1973 Constitution, warrant obeisance by the executive branch and recognition by this Court.

Not Strictly Technical or Financial Assistance

The Court's previous Decision, now for reconsideration, insisted on another restriction purportedly
imposed by the fourth paragraph of Section 2. It is argued that foreign–owned corporations are
allowed to render only technical or financial assistance in the large-scale exploration, development
and utilization of minerals, petroleum and mineral oils. This conservative view is premised on the
sentiment that the Constitution limits foreign involvement only to areas where they are needed, the
overpowering intent being to allow Filipinos to benefit from Filipino resources. 24Towards that end, the
perception arises that the power of the executive to enter into agreements with foreign-owned
corporations is an executive privilege, hampered by the limitations that generally attach to the grant of
privileges.
255

On the fundamental nature of this power, I harbor an entirely different view. The actual art of
governing under our Constitution does not and cannot conform to judicial definitions of the power of
any of its branches based on isolated clauses or even single articles torn from context.25 The
previously adopted approach is rigidly formalist, and impervious to the traditional prerogatives of
executive power.

As I stated earlier, the executive authority to contract is a right emanating from traditional executive
functions, and is connected with the power of the executive branch to determine economic
policy. Hence, the proper approach in interpreting Section 2, Article XII is to tilt in favor of
asserting the right rather than view the provision as a limitation on a privilege. To subscribe to
the Court's previous view will necessitate adopting as a fundamental premise that absent an
express grant of power, the executive branch has no capacity to contract since such capacity
arises from a privilege.

Had the provision been worded to state that the President may enter into agreements for technical or
financial assistance only, then this unambiguous limitation should be affirmed. Yet the Constitution
does not express such an intent. The controversial provision is crafted in such a way that allows any
type of agreement, so long as they involve either technical or financial assistance. In fact, the
provision does not restrict the scope of the agreement so as to pertain exclusively either to technical
or financial assistance.

The Constitution, in allowing foreign participation specifically in the large scale exploration,
development and utilization of natural resources, is cognizant of the sad truth that such activities
entail significant outlay of capital and advanced technological know-how that domestic corporations
may not yet have.26 The provision expressly adverts to "technical" and "financial" assistance in
recognition of the reality that these two facets are the indispensable requisites to qualify
foreign participants in the exploration, development, and utilization of mineral and petroleum
resources.

Had the framers chosen to restrict all aspects of all mining activities to domestic persons, the real fear
would have materialized that our mineral reserves could remain untapped for a significant period of
time, owing to the paucity of venture capital. There was a real option to heed dogmatic guns who
insisted that the mineral resources remain unutilized until the day when the domestic mining industry
becomes capacitated to undertake the extraction without need of foreign aid. Obviously, the more
pragmatic view won the day.

If indeed the foreign entity is limited only to technical or financial participation, the implication
is that it is up to the State to do all the rest. Considering the lack of know-how and financial
capital, matters which were appreciated by the framers of the Constitution, this intended effect
is preposterous. Even the State itself would hesitate to undertake such extractive activities
owing to the intensive capital and extensive training such enterprise would entail. By allowing
this expansive set-up under Section 2, the Constitution enables the minimization of risk on the
part of the State should it desire to undertake large-scale mineral extractive activities. The pay-
off though, understandably, is an atypical cession of several State prerogatives in the
development of its mineral and petroleum resources.

Perhaps there is need to be explicit and incisive about the implications of Section 2. The word
"assistance," shorn of context, implies a charitable grant offered without any quid pro quo attached.
Unconditional foreign aid may be more prevalent this day and age with the acceptance of the notion
that there are base minimum standards of decent living which all persons are entitled to. However,
such concept is alien to the mining industry. There is no such entity as an International Benevolent
Association for Extraction of Minerals. If "assistance" is to be restrictively interpreted according to
ordinary parlance, no entity would be interested in undertaking this regulated industry.

Any decision by any enterprise to assist in the exploration, development or utilization of mineral
resources does not arise from a philanthropic impulse. It is a pure and simple investment, and one
that is not engaged in unless there is the expectation or hope of a reasonable return. I hasten to add
that the deliberate incorporation of the fourth paragraph of Section 2 has created a window of
opportunity for foreign investments in the extractive enterprises involving petroleum and other mineral
256

oils, subject of course to limitations under the law. The term may prove discomfiting to the
ideologically committed, the sentimental nationalist or the visceral oppositionist. Still, the notion is not
inconsistent with the general power of the executive to enter into agreements for the purpose of
enticing foreign investments.

Why then the term "assistance?" Apart from its apparent political palatability in comparison with
"investment," as intimated before, the term is useful in underscoring the essential facets of the
foreign investment which is assistance in the financial or technical areas, as well as the
fundamental limitations and conditionalities of the investment. What is allowed is participation,
though limited, by foreign corporations which in turn are entitled to expect a return on their
investments.

The Court had earlier premised the invalidity of several provisions of the Mining Act on the argument
that those provisions authorized service contracts. But while the 1987 Constitution does not utilize
the term "service contracts," it actually contemplates a broader expanse of agreements
beyond mere contracts for services rendered. Still, although the provision sanctions a more
numerous class of agreements, these are subjected to more stringent restrictions than what had been
allowed under the 1973 Constitution. Thus, the test should be whether the law and the contract
take away the State's full control and supervision over the exploration, development and
utilization of the country's mineral resources and negate or defeat the State's ownership
thereof.

In line with the test, Section 2 should be accorded a liberal interpretation so as to recognize this
fundamental prerogative of the presidency. Such "liberal interpretation" does not equate to a
wholesale concession of mining resources to foreigners, much less to an atmosphere of
complaisance, whether from their perception or the Filipinos.' The fourth paragraph sets specific
limitations on the exercise by the President of this contract-making power. On the other hand, the
second paragraph of Section 2 lays down the fundamental limitations which likewise may not be
countermanded.

On the basis of the foregoing discussion, and as a necessary consequence of my view that the
agreements under Section 2 are not strictly limited to financial or technical assistance, I would
consider the following questioned provisions of Republic Act No. 7942 as valid —Sections 3 (g), 34 to
38, 40 to 41, 56 and 90. These provisions were struck down on the premise that they allowed the
constitution of "service contracts," an agreement which to my mind is still within the contemplation of
Section 2, Article XII.

State Ownership over Mineral and Petroleum Resources

There is need to clarify the specific meaning of these general limitations arising from the State's
assertion of ownership, full control and supervision.

In respect to the petition, the question of ownership has become material to the proper share the
State should receive from the exploration, development and utilization of mineral resources. I perceive
that all the members of the Court agree that such profit may not be limited to only such revenue
derived from the taxation of the mining activities. Since the right of the State to obtain a share in the
net proceeds and not merely through taxes arises as an attribute of ownership unequivocally reserved
by the Constitution for the State, such right may not be proscribed either by legislative provision or
contractual stipulation.

Yet it should be conceded that the State has the right to enter into an agreement concerning such
profits. There are, as probably should be, political consequences if the President opts to surrender all
of the State's profits to a foreign corporation, yet in bare theory, the right to bargain profits pertains to
the wisdom of a political act not ordinarily justiciable before this Court. Still, the overriding adherence
of the Constitution to the regalian doctrine should be given due respect, and an interpretation allowing
"beneficial ownership" by the foreign corporation should not be favored.
257

For purposes of the present judicial review, I would consider it prudent to limit myself to conceding
that the Court had previously erred in invalidating certain provisions of Rep. Act No. 7942 and the
WMC FTAA on the mistaken notion that the law and the agreement cede beneficial ownership of
mineral resources to a foreign corporation.

Section 4 of Rep. Act No. 7942 expressly recognizes State ownership over mineral resources, though
it is silent on the operational terms of such ownership. Of course, such general submission would not
be in itself curative of whatever contraventions to State ownership are contained in the same law;
hence, the need for deeper inquiry.

The dissenters wish to strike down the second paragraph of Section 81 of Rep. Act No. 7942 because
it purportedly precludes the Government from obtaining profits under the agreement from sources
other than its share in taxation. However, as the ponencia points out, the phrase "among other things"
sufficiently allows the government from demanding a share in the cash flow or earnings of the mining
enterprise. A contrary view is anchored on a rule of statutory construction that concludes that "among
other things" refers only to taxes. Yet, there is also a rule of construction that laws should be
interpreted with a view of upholding rather than destroying it. Thus, the ponencia's formulation, which
achieves the result of the minority without need of statutory invalidation, is highly preferable.

The provisions of Rep. Act No. 7942 which authorize the conversion of a financial or technical
assistance into a mineral production sharing agreement (MPSA) turned out to be just as controversial.
In this regard, the minority wishes to strike down Section 39, which in conjunction with Sections 80
and 84 of the law would purportedly allow such conversion, in that it would effectively limit the
government share in the profits to only the excise tax on mineral products under internal revenue law.

These concerns are valid and raise troubling questions. Yet equally troubling is that the Court is being
called upon to rule on a premature question. There is no such creature yet as an FTAA converted into
an MPSA, and so there is no occasion that calls for the application of Sections 39, 80 and 84. I do not
subscribe to judicial pre-emptive strikes, as they preclude the application of still undisclosed
considerations which may prove illuminating and even crucial to the proper disposition of the case. By
seeking invalidation of these "MPSA provisions," the Court is also asked to strike down an enactment
of a co-equal branch which has not given rise to an actual case or controversy. After all, such
enactment deserves due respect from this branch of government. Assuming that the provisions are
indeed invalid, the Court will not hesitate, at the proper time, to strike them down or at least impose a
proper interpretation that does not run afoul of the Constitution.27 However, in the absence of any
actual attempt to convert an FTAA to an MPSA, the time is not now.

I likewise agree with the ponencia that Section 7.9 deprives the State of its rightful share as an
incident of ownership without offsetting compensation. The provisions of the FTAA are fair game for
judicial review considering their present applicability. In fact, the invalidation of Section 7.9 becomes
even more proper now under the circumstances since the provision has become effectual considering
the sale of the foreign equity in WMCP to a domestic corporation. It is within the competence of this
Court to invalidate Section 7.9 here and now. For that matter, Section 7.8(e) of the FTAA may be
similarly invalidated as it can already serve to unduly deprive the Government of its proper share by
allowing double recovery by WMC.

"Full Control and Supervision" of the State

The matter of "full control and supervision" emerges just as controversial. Does this grant of power
mandate that the State exercise management over the activity, or exclude the exercise of managerial
control by the foreign corporation?

I don't think it proper to construe the word "full" as implying that such control or supervision may not
be at all yielded or delegated, for reasons I shall elaborate upon. Instead, "full" should be read as
pertaining to the encompassing scope of the concerns of the State relating to the extractive
enterprises on which it may interfere or impose its will.
258

It must be conceded that whichever party obtains managerial control must be allowed considerable
elbow room in the exercise of management prerogatives. Management is in the most informed
position to make resources productive in the pursuit of the enterprise's objectives. 28 In this age of
specialization, corporations have benefited with the devolution of operational control to specialists,
rather than generalists. The era of the buccaneer entrepreneur chartering his industry solely on gut
feel is over. The vagaries of international finance have dictated that prudent capitalists cede to the
opinion of their experts who are hired because they trained within their particular fields to know better
than the persons who employ them. The Constitution does not prescribe a particular manner of
management; thus, we can conclude that the State is not compelled to adopt outmoded methods that
could tend to minimize profits.

Still, the question as to who should exercise management is best left to the parties of the agreement,
namely the President and the foreign corporations. They would be in the best position to determine
who is best qualified to exert managerial control. This prerogative of management can be exercised
by the State if it so insists and the co-parties agree, and the wisdom of such arrogation is ultimately a
policy question this Court has little control over. And even if the State cedes management to a
different entity such as the foreign corporation, it has the duty to safeguard that the actual
exercise of managerial power does not contravene our laws and public policy.

There is barely any support of the view that only the State may exert managerial control. Even the
minority concede that these foreign corporations are not precluded from participating in the
management of the project. I think it unwise to construe "full supervision and control" to the effect that
the State's assent or opinion is necessary before any day-to-day operational questions may be
resolved. There is neither an express rule to that effect, nor any law of construction that necessitates
such interpretation. Ideally of course, the most qualified party should be allowed to manage the
enterprise, and we should not allow an interpretation that compels a possibly unsuited entity, such as
the State, to operationalize the business.29 Such a limited construction would be inconvenient and
absurd,30 not to mention potentially wasteful.

The Constitution itself concedes that the State may not have the best sense as to how to undertake
large-scale exploration, development and utilization of mineral and petroleum resources. This is
evinced by the allowance of foreign technical assistance and foreign participation in the extractive
enterprise. Had the Constitution recognized that the State was supremely qualified to undertake the
operational aspects of the activity, then it could have phrased the provision in such a way that would
strictly limit the foreign participation to monetary investment or a financial grant of assistance.

The absence of an express provision on management permits consideration of the following sensible
critique on yielding too many management prerogatives to a remote overseer such as the State. An
early United Nations report once noted that while it is theoretically possible to endow a government
department with a high degree of operating flexibility, it is in practice difficult to do so. 31 It has been
proposed that the further away a decision-maker is to the market, the higher the information cost, or
the opportunity cost to the gaining of information.32 Remoteness can be achieved through the layering
of bureaucratic structure, and because of the information loss that accompanies the transmission of
information and judgments from lower levels of the hierarchy to higher levels, the ultimate basis of a
decision may be misleading at best and erroneous at worst.33

The same conclusion arises from the view that what the provision authorizes is foreign investment.
The foreign player necessarily at least has a reasonable say in how the mining venture is run. The
interest of the investor in seeing that the investment is not wasted should be recognized not only as a
right available to the investor, but from the broader view that such say would lead to a more prudent
management of the project. It must be noted that mineral and petroleum resources are non-
renewable, thus a paramount interest arises to ensure against wasteful exploitation.

Next for consideration is the situation, as in this case, if management is ceded to the foreign
corporation, or even to a private domestic corporation for that matter. What should be the proper
dichotomy, if any, between the private entity's exercise of managerial control, and the State's full
control and supervision?
259

The President may insist on conditions into the agreement pertaining to the State's degree of control
and supervision in the mining activity. This was certainly done with the WMC FTAA, which is replete
with stipulations delineating the State's control which are judicially enforceable, imposed presumably
at the President's call. But the FTAA itself is not the only vehicle by which State control and
supervision is exercised. These can similarly be enforced through statutes, as well as executive or
administrative issuances. The Mining Act itself is an expression of State control and supervision,
implemented in coordination with the executive and legislative branches.

As a general point, I believe that State control and supervision is unconstitutionally yielded if either of
the Mining Act or the FTAA precludes the application of the laws and regulations of the Philippines,
enunciatory as they are of State policy. Neither the Mining Act nor the WMC FTAA are flawed in that
regard. The agreements under contemplation are not beyond the ambit of our regular laws, or
regulatory enactments pertaining to such areas as environmental concerns. Violations of these laws
uttered in the name of the FTAA are punishable in this jurisdiction.

Still, the fact that the Constitution requires "full control and supervision" indicates an expectation of a
more activist role on the part of the State in the operations of the mining enterprise, perhaps to the
prejudice of the laissez-fairecapitalist. Most importantly, the State cannot abdicate its traditional
functions by contractual limitations. It could compel the mining operations to comply with existing
environmental regulations, as well as with future issuances. It may compel the foreign corporation
payment of all assessable levies. It may evict officers of the foreign corporation for violation of
immigration laws. It may preclude mining operations that affect prerogatives granted by law to
indigenous peoples. It could restrict particular mining operations which are established to be disasters
or nuisances to the affected communities. The power of the State to enforce its police powers needs
no statutory grant and are certainly not limited either by the Mining Act or the WMC FTAA.

As to "business decisions," I think that the State may exercise control for the purpose of ensuring
profit of the enterprise as a whole. This may involve visitorial activity, the conduct of periodic audits,
and such powers normally attributed to an overseer of a business. Just as the foreign corporation is
expected to guard against waste of financial capital, the State is expected likewise to guard against
the waste of resource capital.

I might as well add that, in my view, the constitutional objective of maintaining full control and
supervision over the exploration, development and utilization of the country's mineral resources in the
State would be best served by the creation of a public corporation for the development and utilization
of these resources, accountable to the State for all actions in its behalf. The device of a corporation
properly utilized provides sufficient protection to the State's interests while affording flexibility and
efficiency in the conduct of mining operations.34

The creation of a public corporation could remedy a number of potential problems regarding full State
control and supervision of extractive activities concerning our mineral resources by entities which
have the funds and/or technical know-how but which cannot have a great degree of control and
supervision over such activities. Persons knowledgeable and competent in mining operations may sit
in the corporation's board of directors and craft policies which implement and further concretize the
broad aims of R.A. No. 7942, taking into consideration the nature of the mining industry. The Board
would also be in charge of studying existing contracts for mining activities, and approving proposed
contracts. The Board may also employ corporate officers and employees to take charge of the day-to-
day operations of the mining activities pursuant to the corporation's contracts with other entities.

Under such a scheme, the perceived abdication by the State of control and supervision over mining
activities in favor of the foreign entities rendering financial and/or technical assistance would be
greatly diminished. It would be the public corporation which would principally undertake mining
activities and contract with foreign entities for financial and/or technical assistance if necessary. The
foreign contractor in such cases would not have the power to determine the course of the project or
the major policies involved therein because these functions would belong to the public corporation as
the agent of the State.
260

A public corporation would also have the additional benefit of compelling the input of not only the
executive branch, but also that of the legislative. Such executive-legislative coordination is necessary
since public corporations may only be created through statute.

Section 3.3 of WMC FTAA Constitutional

Finally, it is argued that Section 3.3 of the WMC FTAA violates paragraph 1, Section 2, Article XII of
the Constitution, which imposes a limitation on the term of mineral agreements. I agree with
the ponencia that the constitutional provision does not pertain to FTAAs. It is clear from reading
Section 1 that the agreements limited in term therein are co-production agreements, joint venture
agreements, and mineral production-sharing agreements, which are all referred to in Section 1, and
not the FTAAs mentioned only in Section 4. Accordingly, Section 3.3 of the WMC FTAA is not infirm.

Epilogue

Behind the legal issues presented by the petition are fundamental policy questions from which highly
opinionated views can develop, even from the members of this Court. The promise brought about by
the large-scale exploitation of our mineral and petroleum resources may bring in much needed
revenue, but Filipinos should properly inquire at what cost. As a Filipino, I am distressed whenever
the government crosses the line from cooperation to subservience to foreign partners in development.
Popular Western wisdom aside, what is good for General Motors is not necessarily good for the
country. The propagation of a foreign-influenced mining industry may lead to a whole slew of social
problems35 which shall be exacerbated if the government is complicit, either through active
participation or benign neglect, to abuses committed by the mining industry against the Filipino
people. Unlike the foreign corporation, the bottom line which the State should consider is not found
below a ledger, but in the socio-economic dynamic that will confront the government as a result of the
large-scale mining venture. Political capital is more fickle than financial capital.

Still, the right to vote I exercise today is that as of a member of the Court, and not that of the general
electorate. The limits of judicial power would exasperate any well-meaning judge who feels duty-
bound to affirm a constitutionally valid law or principle he or she may otherwise disagree with. My
views on how the government should act are segregate from my view on whether the government has
the power to act at all.

My conclusions are borne out of a close textual analysis of Section 2 in light of my fundamental
understanding of the constitutional powers of the executive branch. This is in line with my perception
of the judicial duty as being limited to charting the scope and boundaries of the law. The philosophy of
inclusiveness that drives my interpretation of Section 2 is bolstered not because it might lead to
benefits to the economy, but because it gives due regard to the discretion of the Executive to
determine what is good for the economy. This judicial attitude may not always ensure the economic
good. But before we carve that judicial path out of what we believe are good intentions, restraint is
imperative out of due deference to our co-equal branches, since the duty of formulating and
implementing economic policies falls exclusively within their purview.

In view of the foregoing, I concur with the opinion of Justice Panganiban.

Footnotes

1 Spelled as "Nequito" in the caption of the Petition, but "Nequinto" in the body. Rollo, p. 12.

2 Asspelled in the body of the Petition. Id., p. 13. The caption of the Petition does not include
Louel A. Peria as one of the petitioners; only the name of his father, Elpidio V. Peria, appears
therein.
261

3 Statedas "Kaisahan Tungo sa Kaunlaran at Repormang Pansakahan (KAISAHAN)" in the


caption of the Petition, but "Philippine Kaisahan Tungo sa Kaunlaran at Repormang
Pansakahan (KAISAHAN)" in the body. Id., p. 14.

4 Erroneously designated in the Petition as "Western Mining Philippines Corporation." Id., p.


212.

5 Thisis dependent upon the discussion, infra, of the invalidity of Sections 7.8(e) and 7.9 of
the subject FTAA, for violation of the Civil Code and the Anti-Graft Law -- these provisions
being contrary to public policy and grossly disadvantageous to the government.

6 TheFTAA is for the exploration, development and commercial exploitation of mineral


deposits in South Cotabato, Sultan Kudarat, Davao del Sur and North Cotabato, covering an
area of 99,387 hectares.

7 At
the time of execution of the subject FTAA in 1995, WMCP was owned by WMC
Resources International Pty., Ltd. (WMC) -- "a wholly owned subsidiary of Western Mining
Corporation Holdings Limited, a publicly listed major Australian mining and exploration
company." See WMCP FTAA, p. 2.

On Jan. 23, 2001, WMC sold all its shares in WMCP to Sagittarius Mines, Inc. (Sagittarius), a
corporation organized under Philippine laws, 60% the equity of which is owned by Filipino
citizens or Filipino-owned corporations and 40% by Indophil Resources, NL, an Australian
company. WMCP was then renamed "Tampakan Mineral Resources Corporation," and now it
claims that by virtue of the sale and transfer of shares, it has ceased to be connected in any
way with WMC. On account of such sale and transfer of shares, the then DENR Secretary
approved by Order dated Dec. 18, 2001 the transfer and registration of the subject FTAA from
WMCP to Sagittarius (Tampakan). Lepanto Consolidated Mining Co., which was interested in
acquiring the shares in WMCP, appealed this Order of the DENR Secretary, but the Office of
the President, and subsequently, the Court of Appeals (CA), upheld said Order.

8 Penned by the esteemed Justice Conchita Carpio Morales, the Decision was promulgated
on a vote of 8-5-1. Chief Justice Davide and Justices Puno, Quisumbing, Carpio, Corona,
Callejo, and Tinga concurred. Justices Santiago, Gutierrez, and Martinez joined the Dissent
of Justice Panganiban, while Justice Vitug wrote a separate Dissent. Justice Azcuna took no
part.

9 Promulgated on Dec. 31, 1972, Presidential Decree No. 87 (PD 87, otherwise known as
"The Oil Exploration and Development Act of 1972" in §1 thereof) permitted the government
to explore for and produce indigenous petroleum through service contracts. A service
contract has been defined as a contractual arrangement for engaging in the exploitation and
development of petroleum, mineral, energy, land and other natural resources, whereby a
government or an agency thereof, or a private person granted a right or privilege by said
government, authorizes the other party -- the service contractor -- to engage or participate in
the exercise of such right or the enjoyment of the privilege, by providing financial or technical
resources, undertaking the exploitation or production of a given resource, or directly
managing the productive enterprise, operations of the exploration and exploitation of the
resources, or the disposition or marketing of said resources. See Prof. M. Magallona, Service
Contracts in Philippine Natural Resources, 9 World Bulletin 1, 4 (1993).

Under PD 87, the service contractor undertook and managed the petroleum operations
subject to government oversight. The service contractor was required to be technically
competent and financially capable to undertake the necessary operations, as it provided all
needed services, technology and financing; performed the exploration work obligations; and
assumed all related risks. It could not recover any of its expenditures, if no petroleum was
produced. In the event petroleum is discovered in commercial quantity, the contractor
operated the field for the government. Proceeds of sale of the petroleum produced under the
262

contract were then applied to pay the service fee due the contractor and reimburse it for its
operating expenses incurred.

10 Sec. 9 of Art. XIV (National Economy and Patrimony) of the 1973 Constitution allowed
Filipino citizens, with the approval of the Batasang Pambansa, to enter into service contracts
with any person or entity for the exploration and utilization of natural resources.

"Sec. 9. The disposition, exploration, development, exploitation, or utilization of any of the


natural resources of the Philippines shall be limited to citizens, or to corporations or
associations at least sixty per centum of which is owned by such citizens. The Batasang
Pambansa, in the national interest, may allow such citizens, corporations or associations to
enter into service contracts for financial, technical, management or other forms of assistance
with any person or entity for the exploration or utilization of any of the natural resources.
Existing valid and binding service contracts for financial, technical, management or other
forms of assistance are hereby recognized as such."

The intention behind the provision, according to a delegate, was to promote proper
development of the natural resources, given the lack of Filipino capital and technical skills
needed therefor. The original proposal was to authorize government to enter into such service
contracts with foreign entities, but as finally approved, the provision permitted the Batasang
Pambansa to authorize a citizen or private entity to be party to such contract. Following the
ratification of the 1973 Charter, PD Nos. 151, 463, 704, 705, 1442 were promulgated,
authorizing service contracts for exploration, development, exploitation or utilization of lands
of the public domain; exploration, development, etc. of a lessee's mining claims and the
processing and marketing of the products thereof; production, storage, marketing and
processing of fish and fishery/aquatic products; exploration, development, and utilization of
forest resources; and exploration, development, and exploitation of geothermal resources,
respectively.

11 Renamed Tampakan Mineral Resources Corporation.

12 That is, the Court of Appeals' resolution of the petition for review -- docketed as CA-GR No.
74161 and lodged by Lepanto Consolidated Mining -- of the decision of the Office of the
President which upheld the order of the DENR Secretary approving the transfer and
registration of the FTAA to Sagittarius Mines, Inc.

13 At p. 68.

14 433 Phil. 506, July 9, 2002; 403 SCRA 1, May 6, 2003; and 415 SCRA 403, November 11,
2003.

15 300 Phil. 906, March 12, 1998.

16 Chavez v. Public Estates Authority, 403 SCRA, 1, 28-29, supra, per Carpio, J.

17 Thependency of a motion for reconsideration shall stay the final resolution sought to be
reconsidered. §4 of Rule 52, and §4 of Rule 56B of the Rules of Court.

18 See Enrile v. Senate Electoral Tribunal, GR No. 132986, May 19, 2004.

19 Perthe "List of Financial/Technical Assistance Agreement (FTAA applications)" as of June


30, 2002 prepared by the Mines and Geosciences Bureau's (MGB) Mining Tenements
Management Division, cited in petitioners' Final Memorandum.

20 Insteadof allowing Sec. Gen. Neri to speak during the Oral Argument, the Court in its
Resolution of June 29, 2004 required him to submit his Position Paper through the Office of
263

the Solicitor General. Said paper was made part of the Memorandum of the public
respondents.

21 27 SCRA 853, April 18, 1969.

22 Gonzales v. COMELEC, 137 Phil. 471, 489, April 18, 1969, per Fernando, J. (later CJ.),
citing People v. Vera, 65 Phil. 56, 94, November 16, 1937, per Laurel, J.

23 433Phil. 62, 68, July 2, 2002, citing Alunan III v. Mirasol, 342 Phil. 467, 477, July 31, 1997
and Viola v. Alunan III, 343 Phil. 184, 191, August 15, 1997.

24 Southern
Pacific Terminal Co. v. ICC, 219 US 498, 31 S.Ct. 279, 283, February 20, 1911,
per McKenna, J.

25 134 SCRA 438, 463-464, February 18, 1985, per Gutierrez Jr., J.

26 §1 of Rule 63 of the Rules of Court:

"Section 1. Who may file petition. – Any person interested under a deed, will, contract or other
written instrument, whose rights are affected by a statute, executive order or regulation,
ordinance, or any other governmental regulation may, before breach or violation thereof, bring
an action in the appropriate Regional Trial Court to determine any question of construction or
validity arising, and for a declaration of his rights or duties, thereunder."

27 §2 of Rule 65 of the Rules of Court:

"Section 2. Petition for prohibition. – When the proceedings of any tribunal,


corporation, board, officer or person, whether exercising judicial, quasi-judicial or
ministerial functions, are without or in excess of its or his jurisdiction, or with grave
abuse of discretion amounting to lack or excess of jurisdiction, and there is no appeal
or any other plain, speedy, and adequate remedy in the ordinary course of law, a
person aggrieved thereby may file a verified petition in the proper court, alleging the
facts with certainty and praying that judgment be rendered commanding the
respondent to desist from further proceedings in the action or matter specified
therein, or otherwise granting such incidental reliefs as law and justice may require."

28 Pimentel Jr. v. Aguirre, 391 Phil. 84, 107, July 19, 2000, per Panganiban, J.

29 338 Phil. 546, May 2, 1997.

30 Tañada v. Angara, pp. 47-49, per Panganiban, J. Italics supplied.

31 Emphasis supplied.

32 AngBagong Bayani v. COMELEC, 412 Phil. 308, 338-339, June 26, 2001, per
Panganiban, J., citing JM Tuason & Co., Inc. v. LTA, 31 SCRA 413, 422-423, February 18,
1970, as cited in Agpalo, Statutory Construction (1990), pp. 311 and 313.

33 GRNos. 160261, 160262, 160263, 160277, 160292, 160295, 160310, 160318, 160342,
160343, 160360, 160365, 160370, 160376, 160392, 160397, 160403, and 160405,
November 10, 2003, per Carpio Morales, J.

34 Franciscov. The House of Representatives, 415 SCRA 44, 126-127, November 10, 2003,
per Carpio Morales, J. Citations omitted.

35 During the Oral Argument, petitioner's counsel, Atty. Marvic Leonen conceded that the
foreign contractor may exercise limited management prerogatives to the extent of the
264

financial or technical assistance given. TSN, pp. 181-186. How such "limited management"
can be operationalized was not explained.

36 Inthe January 27, 2004 Decision, this Court held that the fourth paragraph of Section 2 of
Art. XII limits foreign involvement in the local mining industry to agreements strictly for
financial and/or technical assistance only, and precludes agreements which grant to foreign
corporations the management of local mining operations, since the latter agreements are
purportedly in the nature of service contracts, as this concept was understood under the 1973
Constitution. Such contracts were supposedly deconstitutionalized and proscribed by the
omission of the phrase "service contracts" from the 1987 Constitution. Since the WMCP
FTAA contains provisions that permit the contractor's management of the concern, the
Decision struck down the FTAA for being a prohibited service contract. Provisions of RA 7942
which granted managerial authority to the foreign contractor were also declared
unconstitutional.

37 Intervenor's Memorandum, pp. 7, 11 and 12.

38 www.dictionary.com provides the following meanings for "involving":

1. To contain as a part; include.

2. To have as a necessary feature or consequence; entail: was told that the job would involve
travel.

3. To engage as a participant; embroil: involved the bystanders in his dispute with the police.

4. a. To connect closely and often incriminatingly; implicate: evidence that involved the
governor in the scandal.

b. To influence or affect: The matter is serious because it involves your reputation.

5. To occupy or engage the interest of: a story that completely involved me for the rest of the
evening.

6. To make complex or intricate; complicate.

7. To wrap; envelop: a castle that was involved in mist.

8. Archaic. To wind or coil about.

39 Itreads as follows: "Section 20. The President may contract or guarantee foreign loans on
behalf of the Republic of the Philippines with the prior concurrence of the Monetary Board,
and subject to such limitations as may be provided by law. The Monetary Board shall, within
thirty days from the end of every quarter of the calendar year, submit to the Congress a
complete report of its decision on applications for loans to be contracted or guaranteed by the
Government or government-owned and controlled corporations which would have the effect
of increasing the foreign debt, and containing other matters as may be provided by law."

40 According to estimates by the MGB, the success-to-failure ratio of large-scale mining or


hydrocarbon projects is about 1:1,000. It goes without saying that such a miniscule success
ratio hardly encourages the investment of tremendous amounts of risk capital and modern
technology required for the discovery, extraction and treatment of mineral ores, and oil and
gas deposits.

41 TheConstitutional Commission (ConCom) began its work in 1986, three short years after
the assassination in August 21, 1983 of former Senator Benigno "Ninoy" Aquino, Jr. During
the early part of this three-year period, the country underwent a wracking economic crisis
265

characterized by scarcity of funds, capital flight, stringent import controls, grave lack of foreign
exchange needed to fund critical importations of raw materials, panic-buying, hoarding of
commodities, and grave lack of foreign exchange needed to fund critical importations of raw
materials. Many businesses were on the verge of failure and collapse, and many in fact did.
The members of the ConCom were unlikely to forget the critical condition of the Philippine
economy and the penury of its government.

42 Themanagement of every business has two primary objectives. The first is to earn profit.
The second is to stay solvent, that is, to have on hand sufficient cash to pay debts as they fall
due. Other objectives may be targeted, but a business cannot hope to accomplish them,
unless it meets these two basic tests of survival -- operating profitably and staying solvent.
Meigs and Meigs, Accounting: The Basis for Business Decisions (5thed., 1982), p. 11.

43 Art. XVIII, "Transitory Provisions," of the 1987 Constitution.

44 III Record of the Constitutional Commission, p. 348. Emphasis supplied.

45 Id., pp. 349-352. Emphasis supplied.

46 Id., p. 354. Emphasis supplied.

47 Id., pp. 355-356. Emphasis supplied.

48 Id., p. 361. Emphasis supplied.

49 V Records of the Constitutional Commission, p. 845.

50 Id., p. 841.

51 Id., p. 844.

52 Civil
Liberties Union v. Executive Secretary, 194 SCRA 317, 337-338, February 22, 1991,
per Fernan, CJ.

53 The transitive verb 'control' has the following meanings -- to exercise restraining or directing
influence over; to regulate; to have power over; to rule; to govern. The noun 'control' refers to
an act or instance of controlling; the power or authority to guide or manage; and the
regulation of economic activity especially by government directive (as in price controls). From
Merriam-Webster Online, Online Dictionary, www.m-w.com.

54 On p. 2 of the Final Memorandum for Petitioners.

55 Sec. 3(aq) of RA 7942 reads as follows: "aq. Qualified person means any citizen of the
Philippines with capacity to contract, or a corporation, partnership, association, or cooperative
organized or authorized for the purpose of engaging in miring, with technical and financial
capability to undertake mineral resources development and duly registered in accordance
with law at least sixty per centum (60 percent) of the capital of which is owned by citizens of
the Philippines: Provided, That a legally organized foreign-owned corporation shall be
deemed a qualified person for purposes of granting an exploration permit, financial or
technical assistance agreement or mineral processing permit." Underscoring supplied.

56 Per Clause 4.6 of the WMCP FTAA, the contractor is required to relinquish each year
during the exploration period at least ten percent (10%) of the original contract area, by
identifying and dropping from the FTAA coverage those areas which do not have mineral
potentials, in order that by the time actual mining operations commence, the FTAA contract
area shall have been reduced to only 5,000 hectares.
266

57 Memorandum (in support of WMCP's Motion and Supplemental Motion for


Reconsideration), p. 61.

58 Id., pp. 63-64.

59 Accounts receivable may be converted to cash in one of three ways: (1) assignment of
receivables, which is a borrowing arrangement with receivables pledged as security on the
loan; (2) factoring receivables, which is a sale of receivables without recourse for cash to a
third party, usually a bank or other financial institution; and (3) the transfer of receivables with
recourse, which is a hybrid of the other two forms of receivable financing. Smith and
Skousen, Intermediate Accounting, (1992, 11th ed.), pp. 317-321.

Banks usually prefer lending against the security of accounts receivable backed up by
postdated checks. They refer to these facilities as "bills discounting lines."

60 Decision, p. 83; bold types supplied.

61 "Beneficial interest has been defined as the profit, benefit, or advantage resulting from a
contract, or the ownership of an estate as distinct from the legal ownership or
control." Christiansen v. Department of Social Security, 131 P. 2d 189, 191, 15 Wash. 2d 465,
467, November 25, 1942, per Driver, J.

Beneficial use, ownership or interest in property means "such a right to its enjoyment as
exists where the legal title is in one person and the right to such beneficial use or interest is in
another x x x." Montana Catholic Missions v. Missoula County, 26 S Ct. 197, 200, 200 U.S.
118, 127-128, January 2, 1906, per Peckham, J.

62 See p. 1138 thereof.

63 Ramosand De Vera, "The Fiscal Regime of Financial or Technical Assistance


Agreements", p. 2. A photocopy of their paper is attached as Annex 2 to the Motion for
Reconsideration of public respondents.

64 These incentives consist principally of the waiver of national taxes during the cost recovery
period of the FTAA. During such period, the contractor pays only part of the basic
government's share in taxes consisting of local government taxes and fees. These are the
local business tax, real property tax, community tax, occupation fees, regulatory fees, all
other local taxes and fees in force, and royalty payments to indigenous cultural communities,
if any.

These national taxes, however, are not to be paid by the contractor: (i) excise tax on
minerals; (ii) contractor's income tax; (iii) customs duties and fees on imported capital
equipment; (iv) value added tax on purchases of imported equipment, goods and services; (v)
withholding tax on interest payments on foreign loans; (vi) withholding tax on dividends to
foreign stockholders; and (vii) royalties due the government on mineral reservations.

Other incentives to the contractor include those under the Omnibus Investment Code of 1997;
those for the use of pollution control devices and facilities; income tax carry-forward of losses
(five-year net loss carry forward); and income tax accelerated depreciation.

65 See §3(g), DAO 99-56. According to the paper by Messrs. Ramos and De Vera, supra, who
are, respectively, the director of the MGB and chief of the Mineral Economics, Information
and Publication Division of the MGB, majority of the payments listed under Sec. 3(g) are
relatively small in value. The most significant payments in terms of amount are the excise tax,
royalties to mineral reservations and indigenous cultural communities, income tax and real
property tax.
267

66 Per Messrs. Ramos and De Vera, supra, "(t)he term of a successful FTAA may be divided
into a pre-operating period, a cost recovery period and a post recovery period. The pre-
operating period consists of the exploration, pre-feasibility, feasibility, development and
construction phases. The aggregate of this period is a maximum of eleven (11) years. The
cost recovery period, on the other hand, consists of the initial years of commercial operation
where the contractor is allowed to recover its pre-operating expenses. The end of this period
is when the aggregate of the net cash flow from the mining operation becomes equal to the
total pre-operating expenses or a maximum of five (5) years from commencement of
commercial production, whichever comes first. The post recovery period is the remaining term
of the FTAA immediately following the cost recovery period. The additional government share
from an FTAA is collected after the cost recovery period."

67 Ramos and De Vera, supra, pp. 3-4.

68 The
discussion on pp. 4-7 of the Ramos-DeVera report, focusing on the modes of
computation of the additional government share as spelled out in DAO 99-56, is significant:

The phrase "among other things" demands that Government is entitled to additional
share aside from the normal taxes and fees paid during operation. Simple as it was
formulated, the phrase is another challenging task to operationalize. In 1997, the
Philippine government conducted several consultative meetings with various investor
groups, national government agencies concerned with taxation and incentives and
other stakeholders of the mining industry to formulate the possible modes of
determining the additional government share for FTAA. The negotiation took into
consideration the following:

· Capital investment in the project;

· Risks involved;

· Contribution of the project to the economy;

· Contribution of the project to community and local government;

· Technical complexity of the project; and

· Other factors that will provide for a fair and equitable sharing between the
government and the contractor.

During these consultations, some investor groups have repeatedly expressed their
objections to the imposition of an additional government share. However, since
Government is firmly committed to adhere to its interpretation of Section 81 of the
mining law on government share in an FTAA, it decided to push through with the
collection of this additional government share by formally making part of the mining
regulation through the issuance by the Department of Environment and Natural
Resources of Administrative Order No. 99-56 providing for the guidelines in
establishing the fiscal regime of Financial or Technical Assistance Agreements.

There were three schemes for computing the additional government share presented
in the administrative order.

5.1 Net Mining Revenue-Based Option

Net mining revenue means the gross output from mining operations during a calendar
year less deductible expenses. These deductible expenses consist of expenses
incurred by the Contractor directly, reasonably and necessarily related to mining
operations in the contract area during a calendar year, namely:
268

· Mining, milling, transport and handling expenses together with smelting and
refining costs other than smelting and refining costs paid to third parties;

· General and administrative expenses actually incurred by the Contractor in


the Philippines;

· Consulting fees incurred for work related to the project; provided that those
expenses incurred outside of the Philippines are justifiable and allowable
subject to the approval of the Director of Mines and Geosciences Bureau;

· Environmental expenses of the Contractor including such expenses


necessary to fully comply with its environmental obligations;

· Expenses for the development of host and neighboring communities and for
the development of geoscience and mining technology together with training
costs and expenses;

· Royalty payments to claim owners or surface land owners relating to the


Contract Area during the Operating Phase;

· Continuing exploration and mine development expenses within the Contract


Area after the pre-operating period; and

· Interest expenses charged on loans or such other financing-related


expenses incurred by the Contractor subject to limitations in debt/equity ratio
as given in the contract and which shall not be more than the prevailing
international rates charged for similar types of transactions at the time the
financing was arranged, and where such loans are necessary for the
operations; and

· Government taxes, duties and fees.

The additional government share from this option for any year i is the difference
between 50% of the cumulative annual net mining revenues CNi and the cumulative
total government share CGi (basic and additional). The intention is to distribute the
cumulative net mining revenue equally between the Government and the contractor.
It can be expressed through the following formula:

If 50% of CNi < CGi

Additional Government Share = 0

Else, if 50% of CNi > CGi

Additional Gov't Share = (50% x CNi) - CGi

5.2 Cash Flow-Based Option

Project cash flow before financing and tax (CFi) is calculated as follows:

CFi = GO - DE + I - PE - OC

In this formula, GO is the gross output; DE are the deductible expenses; I is the
interest expense; PE is unrecovered pre-operating expense; and OC is on-going
capital expenditures. This option provides that Government gets an additional share
from the project cash flow if the cumulative present value of the previous total
government share collected (basic and additional) is less than 50% of the cumulative
269

present value of the project cash flows. The additional government share AGS is
therefore the difference between 50% and the percentage of the cumulative present
value of total government shares CGA over the cumulative present value of the
project cash flows CP. The cumulative present value of project cash flow for any
year i is given by the following formula:

CPi = CPi-1 x (1.10) + CFi

The factor 1.10 is a future value factor based on the cost of borrowed money with
allowance for inflation of the US dollar. The cumulative present value of the total
government share before additional government share CGB for year i is:

CGBi = CGAi-1 x (1 + Cost of Capital) + BGSi

where CGAi is the cumulative present value of total government share inclusive of the
additional government share during the year is CGAi = CGBi + AGSi.

If CGBi > 50% of CPi :

Additional Government Share = 0

Else, if CGBi < 50% of CPi :

Additional Government Share = (50% x CPi) – CGBi

5.3 Profit-Based Option

This third option provides that Government shall receive an additional share of
twenty-five percent (25%) of the additional or excess profits during a taxable year
when the two-year average ratio of the net income after tax (NIAT) to gross output
(GO) is 0.40 or better. The trigger level of 0.40 ratio is approximately equivalent to a
20% return on investment when computed based on the life of the project. Investors
have indicated that their minimum return on investment before they would invest on a
mining project in the Philippine is 15%. It was agreed upon that a return on
investment below 20% but not lower than 15% is normal profit. If the project reaches
20% or better, there is then an additional or excess profits. The computation of the
0.40 trigger shall be based on a 2-year moving average which is the average of the
previous year's ratio and the current year's ratio. The additional or excess profit is
computed using the following formula:

Additional Profits = [NIAT - (0.40 x GO)] / (1 - ITR)

In the above formula, ITR refers to the prevailing income tax rate applied by the
Bureau of Internal Revenue in computing the income tax of the contractor during a
taxable year.

If the two-year average ratio is less than 0.40:

Additional Government Share = 0

Else, if the two-year average ratio is 0.40 or better:

Additional Government Share = 25% x Excess Profits

The government shares 25% of any marginal profit derived by the contractor at 20%
or higher return on investment.
270

In all of these three options, the basis of computation are all in US dollars based on
prevailing foreign exchange rate at the time the expenses were incurred. Alternatives
or options aside from these three schemes are studied by Government for possible
improvement of the current fiscal system. The basic guideline, however, is that the
total government share should not be less than fifty percent of the sharing.

6. Collection of the Additional Government Share

The term of a successful FTAA may be divided into a pre-operating period, a cost
recovery period and a post recovery period. The pre-operating period consists of the
exploration, pre-feasibility, feasibility, development and construction phases. The
aggregate of this period is a maximum of eleven (11) years. The cost recovery
period, on the other hand, consists of the initial years of commercial operation where
the contractor is allowed to recover its pre-operating expenses. The end of this period
is when the aggregate of the net cash flow from the mining operation becomes equal
to the total pre-operating expenses or a maximum of five (5) years from
commencement of commercial production, whichever comes first. The post recovery
period is the remaining term of the FTAA immediately following the cost recovery
period. The additional government share from an FTAA is collected after the cost
recovery period. (underscoring supplied)

69 The cash flows of a business concern tend to be more accurate and realistic indicia of the
financial capacity of the enterprise, rather than net income or taxable income, which are
arrived at after netting out non-cash items like depreciation, doubtful accounts expense for
probable losses, and write-offs of bad debts.

Cash flows provide relevant information about the cash effects of an entity's operations, and
its investing and financing transactions. Smith and Skousen, supra, p. 184.

70 Some of these indirect taxes are: fuel taxes; withholding tax on payrolls, on royalty
payments to claim owners and surface owners and on royalty payments for technology
transfer; value added tad on local equipment, supplies and services.

71 Other contributions of mining projects include: infrastructure (hospitals, roads, schools,


public markets, churches, and the like) and social development projects; payroll and fringe
benefits (direct and indirect employment); expenditures by the contractor for development of
host and neighboring communities; expenditures for the development of geosciences/mining
technology; expenditures for social infrastructures; and the resulting multiplier effects of
mining operations.

72 The third paragraph of §81, RA 7942 states: "The collection of Government share in
financial or technical assistance agreement shall commence after the financial or technical
assistance agreement contractor has fully recovered its pre-operating expenses, exploration,
and development expenditures, inclusive."

73 §§80 and 84 of RA 7942 are reproduced below:

Sec. 80. Government Share in Mineral Production Sharing Agreement. – The total
government share in a mineral production sharing agreement shall be the excise tax on
mineral products as provided in Republic Act No. 7729, amending Section 151(a) of the
National Internal Revenue Code, as amended.

Sec. 84. Excise Tax on Mineral Products. – The contractor shall be liable to pay the excise
tax on mineral products as provided for under Section 151 of the National Internal Revenue
Code: Provided, however, That with respect to a mineral production sharing agreement, the
excise tax on mineral products shall be the government share under said agreement.
(Underscoring supplied)
271

74 §112 of RA 7942 is reproduced below:

Sec. 112. Non-impairment of Existing Mining/Quarrying Rights. – All valid and


existing mining lease contracts, permits/licenses, leases pending renewal, mineral
production-sharing agreements granted under Exec. Order No. 279, at the date of
effectivity of this Act, shall remain valid, shall not be impaired, and shall be
recognized by the Government: Provided, That the provisions of Chapter XIV on
government share in mineral production sharing agreement and of Chapter XVI on
incentives of this Act shall immediately govern and apply to a mining lessee or
contractor unless the mining lessee or contractor indicates his intention to the
Secretary, in writing, not to avail of such provisions: Provided, further, That no
renewal of mining lease contracts shall be made after the expiration of its term:
Provided, finally, That such leases, production-sharing agreements, financial or
technical assistance agreements shall comply with the applicable provisions of this
Act and its implementing rules and regulations. (Underscoring supplied)

75 Even during the cost recovery period, the contractor will still have to pay a portion of
the basic government share consisting of local government taxes and fees, such as local
business taxes, real property taxes, community taxes, occupation fees, regulatory fees, and
all other local taxes and fees, plus royalty payments to indigenous cultural communities, if
any.

76 Ramos and DeVera, supra, p. 7.

77 Ibid., p. 11. See also §3e of DAO 99-56.

78 JusticeCarpio argues thus: The WMCP FTAA grants the State 60 percent of net profit;
CMP likewise agrees to 60 percent; the Malampaya-Shell FTAA provides for 60 percent also;
so the Court should decree a minimum of 60 percent. Our answer: no law authorizes this
Court to issue such a decree. It is up to the State to negotiate the most advantageous
percentage. This Court cannot be stampeded into the realm of legislation.

79 Clause 1.2 thereof states: "All financing, technology, management and personnel
necessary for the Mining Operations shall be provided by the Contractor in accordance with
the provisions of this Agreement. If no Minerals in commercial quantity are developed and
produced, the Contractor acknowledges that it will not be entitled to reimbursement of its
expenses incurred in conducting the Mining Operations."

80 WMCP FTAA Clause 2.1 (iv), p. 6.

81 Id., Clause 2.1 (v), p. 6.

82 Id., Clause 2.1 (vii), p. 6.

83 "Qualified
Entity" is defined as "an entity that at the relevant time is qualified to enter into a
mineral production sharing agreement with the Government under the laws restricting foreign
ownership and equity in natural resource projects." §2 -- Definitions, WMCP FTAA, p. 10.
(Underscoring supplied.)

Pursuant to §26a in relation to §§3g and 3aq of RA 7942, a contractor in an MPSA should be
a citizen of the Philippines or a corporation at least 60 percent of the capital of which is owned
by citizens of the Philippines.

84 Since we assume that the buyer-corporation, which buys up 60% equity in WMCP, is 60%
Filipino-owned and 40% foreign-owned, therefore, the foreign stockholders in such buyer-
corporation hold 24% beneficial interest in WMCP.
272

85 Fourth paragraph of Sec. 2, Art. XII of the 1987 Constitution.

86 See,for instance, Maestrado v. CA, 327 SCRA 678, 692, March 9, 2000 and Philippine
Telegraph and Telephone Co. v. NLRC, 338 Phil. 1093, 1111, May 23, 1997.

87 Art.1306 of the Civil Code provides: "The contracting parties may establish such
stipulations, clauses, terms and conditions as they may deem convenient, provided they are
not contrary to law, morals, good customs, public order, or public policy.".

88 Republic v. CA, 354 SCRA 148, March 9, 2001, per Ynares-Santiago, J.

89 PhilippineBasketball Association v. CA, 337 SCRA 358, 369, August 8, 2000. Likewise,
§11 of Book I of Chapter 3 of Exec. Order No. 292, otherwise known as "The Administrative
Code of 1987," states: "Sec. 11. The State's Responsibility for Acts of Agents. – (1) The State
shall be legally bound and responsible only through the acts performed in accordance with
the Constitution and the laws by its duly authorized representatives. (2) The State shall not be
bound by the mistakes or errors of its officers or agents in the exercise of their functions."

90Art. 1420 of the Civil Code provides: "In case of a divisible contract, if the illegal terms can
be separated from the legal ones, the latter may be enforced."

91 Sarmiento v. CA, 353 Phil. 834, 853, July 2, 1998.

92 Ramos-DeVera, supra, p. 2.

93 Bold types supplied.

94 §3[h] in relation to §26[b] of RA 7942.

95 §26[c] of RA 7942.

96 OXFAM America Research Report, September 2002.

97 Dated December 2003.

98 §1 of EO 270.

99 Decena v. Malayaon, AM No. RTJ-02-1669, April 14, 2004, per Tinga, J.

100 Manila
Electric Company v. Pasay Transportation, 57 Phil. 600, 605, November 25, 1932,
per Malcolm, J.

CHICO-NAZARIO, J.:

1 Mondano v. Silvosa, GR No. L-7708, 30 May 1955, 97 Phil. 143

2 J. Bernas, S.J. The Intent of the 1987 Constitution Writers, 1995 Ed., p. 812.

3 Id. at 818

4 Ibid.

5 Id. at 817-818.
273

6 In Reagan v. Commission on Internal Revenue (L-26379, 27 December 1969, 30 SCRA


968,973) the Court discussed the concept of auto-limitation in this wise: "It is to be admitted
that any State may by its consent, express or implied, submit to a restriction of its sovereignty
rights. That is the concept of sovereignty as auto-limitation which, in the succinct language of
Jellinek, 'is the property of a state-force due to which it has the exclusive capacity of legal-self
determination and self-restriction.' A State then, if it chooses to, may refrain from the exercise
of what otherwise is illimitable competence." See also Tañada v. Angara, GR No. 118295, 2
May 1997, 272 SCRA 18

7 Cf. Akbayan-Youth v. Commission on Elections, 355 SCRA 318 (2001).

8 Section 2, Rep. Act. No. 7942

9Pilipinas Kao, Inc. vs. Court of Appeals, G.R. No. 105014, 18 December 2001, 372 SCRA
548

10Aris (Phis.) Inc. v. National Labor Relations Commission, G.R. No. 90501, 05 August 1991,
200 SCRA 246

11 Ibid.

12 See Tanada v. Angara, 272 SCRA 18.

CARPIO, J.:

1 Philippine Mining Act of 1995.

2 Rollo, pp. 23–24.

3Ibid., pp. 65-120. Then Executive Secretary Teofisto Guingona, Jr. signed the WMCP FTAA
on behalf of then President Fidel V. Ramos upon recommendation of then DENR Secretary
Angel C. Alcala.

4 Section 2, Article XII of the 1987 Constitution provides in full: "All lands of the public domain,
waters, minerals, coal, petroleum, and other mineral oils, all forces of potential energy,
fisheries, forests or timber, wildlife, flora and fauna, and other natural resources are owned by
the State. With the exception of agricultural lands, all other natural resources shall not be
alienated. The exploration, development, and utilization of natural resources shall be under
the full control and supervision of the State. The State may directly undertake such activities,
or it may enter into co-production, joint venture, or production-sharing agreements with
Filipino citizens, or corporations or associations at least sixty per centum of whose capital is
owned by such citizens. Such agreements may be for a period not exceeding twenty-five
years, renewable for not more than twenty-five years, and under such terms and conditions
as may be provided by law. In cases of water rights for irrigation, water supply, fisheries, or
industrial uses other than the development of water power, beneficial use may be the
measure and limit of the grant.

The State shall protect the nation's marine wealth in its archipelagic waters, territorial sea,
and exclusive economic zone, and reserve its use and enjoyment exclusively to Filipino
citizens.

The Congress may, by law, allow small-scale utilization of natural resources by Filipino
citizens, as well as cooperative fish farming, with priority to subsistence fishermen and
fishworkers in rivers, lakes, bays, and lagoons.

The President may enter into agreements with foreign-owned corporations involving either
technical or financial assistance for large-scale exploration, development, and utilization of
274

minerals, petroleum, and other mineral oils according to the general terms and conditions
provided by law, based on real contributions to the economic growth and general welfare of
the country. In such agreements, the State shall promote the development and use of local
scientific and technical resources.

The President shall notify the Congress of every contract entered into in accordance with this
provision, within thirty days from its execution."

5 Chavez v. Public Estates Authority, 433 Phil. 506 (2002).

6 The only limitation is that the State cannot alienate its natural resources except for
agricultural lands. However, the State can exploit commercially its natural resources and sell
the marketable products from such exploitation. See note 12.

7 Article 441, Civil Code.

8Section 1, Article XIII of the 1935 Constitution; Section 8, Article XIV of the 1973
Constitution.

9 Miners Association of the Philippines v. Hon. Factoran, Jr., et al., 310 Phil. 113 (1995).

10 Records of the Constitutional Commission, Vol. III, p. 260.

11 See note 7.

12 Hector de Leon, PHILIPPINE CONSTITUTIONAL LAW, Vol. 2, p. 804 (1999 Ed.).

13 See note 9.

14Section 2, Article XII of the 1987 Constitution provides in part: "x x x With the exception of
agricultural lands, all other natural resources shall not be alienated."

15 Chapter XIV covers Sections 80 to 82 of RA 7942.

16
The five Mineral Production Sharing Agreements (Annexes A to F) attached to the 20
October 2004 Compliance of the Solicitor General uniformly contain the following provision:

Share of the Government - The Government Share shall be the excise tax on
mineral products at the time of removal and at the rate provided for in Republic
Act No. 7729 amending Section 151(a) of the National Internal Revenue Code,
as amended, as well as other taxes, duties, and fees levied by existing laws.
(Emphasis supplied)

Clearly, the State's share is limited to taxes, duties and fees just like under the old
system of "license, concession or lease." See the (1) Mineral Production Sharing
Agreement between the Republic of the Philippines and Ungay-Malobago Mines, Inc.
and Rapu-Rapu Minerals, Inc. dated 12 September 2000; (2) Mineral Production
Sharing Agreement between the Republic of the Philippines and Ungay-Malobago
Mines, Inc. and TVI Resource Development (Phils.), Inc. dated 17 June 1998; (3)
Mineral Production Sharing Agreement between the Republic of the Philippines and
Base Metals Mineral Resources Corporation (BMMRC) dated 20 November 1997; (4)
Mineral Production Sharing Agreement between the Republic of the Philippines and
Philex Gold Philippines, Inc. dated 29 December 1999 (MPSA No. 148-99XIII); and
(5) Mineral Production Sharing Agreement between the Republic of the Philippines
and Philex Gold Philippines, Inc. dated 29 December 1999 (MPSA No. 149-99-XIII).

17 Sections 144, 145 and 149, National Internal Revenue Code.


275

18 Commissioner of Internal Revenue v. Court of Appeals, 312 Phil. 337 (1995).

19 Memorandum dated 13 July 2004, p. 56.

20 Section 26, RA 7942.

21 Chapter XIV covers Sections 80 to 82 of RA 7942.

22China Banking Corporation v. Court of Appeals, G.R. Nos. 146749 & 147938, 10 June
2003, 403 SCRA 634; City of Baguio v. De Leon, 134 Phil. 912 (1968).

23 The 1995 Implementing Rules and Regulations of RA 7942 attempt to limit the period to
five years. Thus, Section 236 of the Implementing Rules states that the "period of recovery
which is reckoned from the date of commercial operation shall be for a period not exceeding
five years or until the date of actual recovery, whichever comes first." However, the
succeeding sentence of Section 236 also states, "For clarification, the Government's
entitlement to its share shall commence after the FTAA contractor has fully recovered its pre-
operating, exploration and development stage expenses, inclusive and the contractor's
obligations under Chapter XXVII (on Taxes and Fees) of the rules and regulations do not
arise until this time." What the first sentence limits the succeeding sentence cancels. The
1996 Revised Implementing Rules and Regulations of RA 7942 omit the clarificatory
sentence.

24Section 94(a) of RA 7942 guarantees the foreign contractor the "right to repatriate the
entire proceeds of the liquidation of the foreign investment in the currency in which the
investment was originally made and at the exchange rate prevailing at the time of
repatriation." Section 94(b) guarantees the "right to remit earnings from the investment in the
currency in which the foreign investment was originally made and at the exchange rate
prevailing at the time of remittance."

25 Memorandum dated 13 July 2004, p. 65.

26 Annex 8, Compliance of the Solicitor General dated 20 October 2004.

27 Fifth Whereas Clause, Occidental-Shell FTAA.

28 Section 1.1, Occidental-Shell FTAA.

29 Sections 7.3 and 7.4, Occidental-Shell FTAA.

30 Section 2.19, Occidental-Shell FTAA.

31Sections 12.1 and 15.2, Occidental-Shell FTAA; Paragraph 4, Annex B on Accounting


Procedures.

32 Section 7.2, Occidental-Shell FTAA.

33 Section 6.1.i, Occidental-Shell FTAA.

34 Sections 2.16 and 2.17, Occidental-Shell FTAA.

35 Sections 2.24, 6.1.j, 6.3 and 8.1, Occidental-Shell FTAA.

36Under Section 12.1 of the Occidental-Shell FTAA, the three-man arbitral panel consists of
the Philippine Government's nominee, Occidental-Shell's nominee, and a third member
mutually chosen by the nominees of the Government and Occidental-Shell.
276

37 Intervenor CMP's Motion for Reconsideration dated 10 July 2004, p. 22.

38 Ibid.

39 Respondent WMCP's Memorandum dated 15 July 2004, p. 42.

40 Ibid.

41 See note 3.

42The same provision appears in the FTAA between the Republic of the Philippines and
ARIMCO Mining Corporation dated 20 June 1994. ARIMCO, a domestic corporation owned
and controlled by an Australian mining company, does not need to pay the 60% share of the
Philippine Government in the mining revenues if ARIMCO's foreign parent company sells
60% of ARIMCO's equity to a Philippine citizen or to a 60% Filipino owned corporation. In
such event, the share of the Philippine Government in the mining revenues is ZERO percent.
ARIMCO will only pay the Philippine Government the 2% excise tax due on mineral products
under a mineral production sharing agreement. See Annex 5, Compliance of Solicitor General
dated 20 October 2004.

43 Section 2.1 of the WMCP FTAA defines a "Qualified Entity" as an "entity that at the relevant
time is qualified to enter into a mineral production sharing agreement with the Government
under the laws restricting foreign ownership and equity in natural resource projects."

44 Motion for Reconsideration dated 14 July 2004, p. 22.

45 Ibid., p. 20.

46 Ibid., p. 12.

47 Decision dated 27 January 2004.

48 Memorandum dated 14 April 2004, p.12.

49 Memorandum dated 15 July 2004, p. 42.

50 Section 10.2 (l), WMCP FTAA.

51 Article 441, Civil Code.

52Section 2.1 of the WMCP FTAA allows WMCP to recover pre-operating expenses over 10
years from the start of commercial production.

53 Memorandum dated 13 July 2004, p. 65.

54 Section 9, Article XII of the 1987 Constitution.

55 Memorandum dated 13 July 2004, p. 60.

56 Ibid., p. 59.

57 Ibid., p. 65.

58 Section 151, National Internal Revenue Code.


277

59 DENR ADMINISTRATIVE ORDER NO. 56-99

SUBJECT: Guidelines Establishing the Fiscal Regime of Financial or Technical


Assistance Agreements Pursuant to Section 81 and other pertinent provisions of
Republic Act No. 7942, otherwise known as the Philippine Mining Act of 1995 (the
"Mining Act"), the following guidelines establishing the fiscal regime of Financial
or Technical Assistance Agreements (FTAA) are hereby promulgated.

SECTION 1. Scope

This Administrative Order is promulgated to:

a. Establish the fiscal regime for FTAAs which the Government and the FTAA
Contractors shall adopt for the large-scale exploration, development and commercial
utilization of mineral resources in the country; and

b. Provide for the formulation of a Pro Forma FTAA embodying the fiscal regime
established herein and such other terms and conditions as provided in the Mining Act
and the Implementing Rules and Regulations (IRR) of the Mining Act.

SECTION 2. Objectives

The objectives of this Administrative Order are:

a. To achieve an equitable sharing among the Government, both National and Local,
the FTAA Contractor and the concerned communities of the benefits derived from
mineral resources to ensure sustainable mineral resources development; and

b. To ensure a fair, equitable, competitive and stable investment regime for the large
scale exploration, development and commercial utilization of minerals.

SECTION 3. Fiscal Regime of a Financial or Technical Assistance Agreement

The Financial or Technical Assistance Agreement which the Government and the
FTAA Contractor shall enter into shall have a Fiscal Regime embodying the following
provisions:

a. General Principles. The Government Share derived from Mining Operations after
the Date of Commencement of Commercial Production shall be determined in
accordance with this Section.

b. Occupation Fees. Prior to or upon registration of the FTAA and on the same date
every year thereafter, the Contractor shall pay to the concerned Treasurer of the
municipality(ies) or city(ies) the required Occupation Fee over the Contract Area at
the rate provided for by existing laws, rules and regulations.

c. Deductible Expenses. Allowable deductible expenses shall include all the


expenses incurred by the Contractor directly, reasonably and necessarily related to
the Mining Operations in the Contract Area in a Calendar Year during the Operating
Phase. Allowable deductible expenses shall include the following:

1. Mining, milling, transport and handling expenses together with smelting and
refining costs other than smelting and refining costs paid to third parties;

2. General and administrative expenses actually incurred by the Contractor in the


Philippines;
278

3. Consulting fees:

a) incurred within the Philippines for work related to the project

b) incurred outside the Philippines for work related to the project: Provided, That such
fees are justifiable and subject to the approval of the Director.

4. Environmental expenses of the Contractor including such expenses necessary to


fully comply with its environmental obligations as stipulated in the environmental
protection provision of the FTAA and in the IRR;

5. Expenses for the development of host and neighboring communities and for the
development of geoscience and mining technology as stipulated in the FTAA and in
the IRR together with the training costs and expenses referred to in the FTAA;

6. Royalty payments to Claimowners or surface land owners relating to the Contract


Area during the Operating Phase;

7. Continuing exploration and mine development expenses within the Contract Area
after the pre-operating period;

8. Interest expenses charged on loans or such other financing-related expenses


incurred by the Contractor subject to the financing requirement in the FTAA, which
shall not be more than the prevailing international rates charged for similar types of
transactions at the time the financing was arranged, and where such loans are
necessary for the operations; and

9. Government taxes, duties and fees.

Ongoing Capital Expenditures shall be considered as capital expenses subject to


Depreciation Charges.

"Ongoing Capital Expenditures" shall mean expenses for approved acquisitions of


equipment and approved construction of buildings necessary for the Mining
Operations as provided in its approved Mining Project Feasibility Study.

"Depreciation Charges" means the annual non-cash deduction from the Operating
Income for the use of fixed assets that are subject to exhaustion, wear and tear and
obsolescence during their employment in a Mining Operation. Its applicability and
computation are regulated by existing taxation laws, the Mining Act and the IRR.
Incentives relating to depreciation allowance shall be in accordance to the provisions
of the Mining Act and the IRR.

"Operating Income" means the Gross Output less Deductible Expenses, while "Gross
Output" has the meaning ascribed to it in the National Internal Revenue Code.

d. Payment of Government Taxes and Fees. The Contractor shall promptly pay all
the taxes and fees required by the Government in carrying out the activities covered
in the FTAA and in such amount, venue, procedure and time as stipulated by the
particular law and implementing rules and regulations governing such taxes and fees
subject to all rights of objection or review as provided for in relevant laws, rules and
regulations. In case of non-collection as covered by Clause 3-g-1 of this Section, the
Contractor shall follow the prevailing procedures for availment of such non-collection
in accordance with pertinent laws, rules and regulations. Where prevailing orders,
rules and regulations do not fully recognize and implement the provisions covered by
Clause 3-g-1 of this Section, the Government shall exert its best efforts to ensure that
all such orders, rules and regulations are revised or modified accordingly.
279

e. Recovery of Pre-Operating Expenses. Considering the high risk, high cost and
long term nature of Mining Operations, the Contractor is given the opportunity to
recover its Pre-Operating Expenses incurred during the pre-operating period, after
which the Government shall receive its rightful share of the national patrimony. The
Recovery Period, which refers to the period allowed to the Contractor to recover its
Pre-Operating Expenses as provided in the Mining Act and the IRR, shall be for a
maximum of five (5) years or at a date when the aggregate of the Net Cash Flows
from the Mining Operations is equal to the aggregate of its Pre-operating Expenses,
reckoned from the Date of Commencement of Commercial Production, whichever
comes first. The basis for determining the Recovery Period shall be the actual Net
Cash Flows from Mining Operations and actual Pre-Operating Expenses converted
into its US dollar equivalent at the time the expenditure was incurred.

"Net Cash Flow" means the Gross Output less Deductible Expenses, Pre-Operating
Expenses, Ongoing Capital Expenditures and Working Capital charges.

f. Recoverable Pre-Operating Expenses. Pre-Operating Expenses for recovery which


shall be approved by the Secretary upon recommendation of the Director shall
consist of actual expenses and capital expenditures relating to the following:

1. Acquisition, maintenance and administration of any mining or exploration


tenements or agreements covered by the FTAA;

2. Exploration, evaluation, feasibility and environmental studies, production, mining,


milling, processing and rehabilitation,

3. Stockpiling, handling, transport services, utilities and marketing of minerals and


mineral products;

4. Development within the Contract Area relating to the Mining Operations;

5. All Government taxes and fees;

6. Payments made to local Governments and infrastructure contributions;

7. Payments to landowners, surface rights holders, Claimowners, including the


Indigenous Cultural Communities, if any;

8. Expenses incurred in fulfilling the Contractor's obligations to contribute to national


development and training of Philippine personnel;

9. Consulting fees incurred inside and outside the Philippines for work related directly
to the Mining Operations;

10. The establishment and administration of field and regional offices including
administrative overheads incurred within the Philippines which are properly
allocatable to the Mining Operations and directly related to the performance of the
Contractor's obligations and exercise of its rights under the FTAA;

11. Costs incurred in financial development, including interest loans payable within or
outside the Philippines, subject to the financing requirements required in the FTAA
and to a limit on debt-equity ratio of 5:1 for investments equivalent to 200 Million US
Dollars or less, or for the first 200 Million US Dollars of investments in excess of 200
Million US Dollars, or 8:1 for that part of the investment which exceeds 200 Million US
Dollars: Provided, That the interests shall not be more than the prevailing
international rates charged for similar types of transaction at the time the financing
was arranged;
280

12. All costs of constructing and developing the mine incurred before the Date of
Commencement of Commercial Production, including capital and property as
hereinafter defined irrespective as to their means of financing, subject to the
limitations defined by Clause 3-f-11 hereof, and inclusive of the principal obligation
and the interests arising from any Contractor's leasing, hiring, purchasing or similar
financing arrangements including all payments made to Government both National
and Local; and

13. General and administrative expenses actually incurred by the Contractor for the
benefit of the Contract Area.

The foregoing recoverable Pre-Operating Expenses shall be subject to verification of


its actual expenditure by an independent audit recognized by the Government and
chargeable against the Contractor.

g. Government Share.

1. Basic Government Share. The following taxes, fees and other such charges shall
constitute the Basic Government Share:

a) Excise tax on minerals;

b) Contractor's income tax;

c) Customs duties and fees on imported capital equipment;

d) Value added tax on the purchase of imported equipment, goods and services;

e) Withholding tax on interest payments on foreign loans;

f) Withholding tax on dividends to foreign stockholders;

g) Royalties due the Government on Mineral Reservations;

h) Documentary stamps taxes,

i) Capital gains tax;

j) Local business tax;

k) Real property tax,

l) Community tax;

m) Occupation fees;

n) All other local Government taxes, fees and imposts as of the effective date of the
FTAA;

o) Special Allowance, as defined in the Mining Act; and

p) Royalty payments to any Indigenous People(s)/Indigenous Cultural


Community(ies).

From the Effective Date, the foregoing taxes, fees and other such charges
constituting the Basic Government Share, if applicable, shall be paid by the
281

Contractor: Provided, That above items (a) to (g) shall not be collected from the
Contractor upon the date of approval of the Mining Project Feasibility Study up to the
end of the Recovery Period. Any taxes, fees, royalties, allowances or other imposts,
which should not be collected by the Government, but nevertheless paid by the
Contractor and are not refunded by the Government before the end of the next
taxable year, shall be included in the Government Share in the next taxable year. Any
Value-Added Tax refunded or credited shall not form part of Government Share.

2. Additional Government Share. Prior to the commencement of Development and


Construction Phase, the Contractor may select one of the formula for calculating the
Additional Government Share set out below which the Contractor wishes to apply to
all of its Mining Operations and notify the Government in writing of that selection.
Upon the issuance of such notice, the formula so selected shall thereafter apply to all
of the Contractor's Mining Operations.

a) Fifty-Fifty Sharing of the Cumulative Present Value of Cash Flows. The


Government shall collect an Additional Government Share from the Contractor
equivalent to an amount which when aggregated with the cumulative present value of
Government Share during the previous Contract Years and the Basic Government
Share for the current Contract Year is equivalent to a minimum of fifty percent (50%)
of the Cumulative Present Value of Project Cash Flow before financing for the current
Contract Year. as defined below.

Computation. The computation of the Additional Government Share shall commence


immediately after the Recovery Period. If the computation covers a period of less
than one year, the Additional Government Share corresponding to this period shall be
computed pro-rata wherein the Additional Government Share during the year shall be
multiplied by the fraction of the year after recovery. The Additional Government Share
shall be computed as follows:

Project Cash Flow Before Financing and Tax ("CF") for a taxable year shall be
calculated as follows:

CF = GO - DE +I - PE - OC

Cumulative Present Value of Project Cash Flow ("CP") shall be the sum of the
present value of the cumulative present value of project cash flow during the previous
year (CP i-1 x 1.10) and the Project Cash Flow Before Financing and Tax for the
current year ("CF"), and shall be calculated as follows:

CP = (CP i-1 x 1.10)

Cumulative Present Value of Total Government Share Before Additional Government


Share ("CGB") shall be the sum of: the present value of the cumulative present value
of the Total Government Share during the previous year (CGA i-J x 1.10), and the
Basic Government Share for the current year (BGS), and shall be calculated as
follows:

CGB = (CGA i-1 x 1.10) + BGS

The Additional Government Share ("AGS") shall be:

If: CGB > CP x 0.5 then AGS = 0

If CGB < CP x 0.5 then AGS = [CP x 0.5] - CGB

Cumulative Present Value of Total Government Share (CGA):


282

CGA = CGB + AGS

where:

BGS = Basic Government Share shall have the meaning as described in Clause 3-g-
1 hereof;

GO = Gross Output shall have the same meaning as defined in the National Internal
Revenue Code;

DE = Deductible Expenses shall have the meaning as described in Clause 3-c


hereof;

I = Interest payments on loans included in the Deductible Expenses shall be


equivalent to those referred to in Clause 3-c-8 hereof;

PE = unrecovered Pre-Operating Expenses;

OC = On-going Capital Expenditures as defined in Clause 3-c hereof;

CPi-1 = cumulative present value of project cash flow during the previous year; and

CGA = cumulative present value of total Government Share during the previous year.

b) Profit Related Additional Government Share. The Government shall collect an


Additional Government Share from the Contractor based on twenty-five percent
(25%) of the additional profits once the arithmetic average of the ratio of Net Income
After Tax To Gross Output as defined in the National Internal Revenue Code, for the
current and previous taxable years is 0.40 or higher rounded off to the nearest two
decimal places.

Computation. The computation of the Additional Government Share from additional


profit shall commence immediately after the Recovery Period. If the computation
covers a period of less than a year, the additional profit corresponding to this period
shall be computed pro-rata wherein the total additional profit during the year shall be
multiplied by the fraction of the year after recovery.

The additional profit shall be derived from the following formula.:

If the computed average ratio as derived from above is less than 0.40:

Additional Profit = 0

If the computed average ratio is 0.40 or higher:

[NIAT-(0.40xGO)]

Additional Profit = ------------------------------

(1-ITR)

The Additional Government Share from the additional profit is computed using the
following formula:

Additional Government Share


283

From Additional Profit = 25% x Additional Profit

where:

NIAT = Net Income After Tax for the particular taxable year under consideration.

GO = Gross Output from operations during the same taxable year.

ITR = Income Tax Rate applied by the Bureau of Internal Revenue in computing the
income tax of the Contractor during the taxable year.

c) Additional Share Based from the Cumulative Net Mining Revenue. The Additional
Government Share for a given taxable year shall be calculated as follows:

(i) Fifty percent (50%) of cumulative Net Mining Revenue from the end of the
Recovery Period to the end of that taxable year;

LESS

(ii) Cumulative Basic Government Share for that period as calculated under Clause 3-
g-1 hereof;

AND LESS (if applicable)

(iii) Cumulative Additional Government Share in respect of the period commencing at


the end of the Recovery Period and expiring at the end of the taxable year
immediately preceding the taxable year in question.

"Net Mining Revenue" means the Gross Output from Mining Operations during a
Calendar year less Deductible Expenses, plus Government taxes, duties and fees
included as part of Deductible Expenses.

3. Failure to Notify. If the Contractor does not notify the Government within the time
contemplated by Clause 3-g-2 of the formula for calculating the Additional
Government Share which the Contractor wishes to apply to all of its Mining
Operations, the Government shall select and inform the Contractor which option will
apply to the latter.

4. Filing and Payment of Additional Government Share. Payment of the Additional


Government Share shall commence after the Recovery Period. The Additional
Government Share shall be computed, filed and paid to the MGB within fifteen (15)
clays after the filing and payment of the final income tax return during the taxable
year to the Bureau of Internal Revenue. Late filing and payment of the Additional
Government Share shall be subject to the same penalties applicable to late filing of
income tax returns. The Contractor shall furnish the Director a copy of its income tax
return not later than fifteen (15) days after the date of filing.

A record of all transactions relating to the computation of the Additional Government


Share shall be maintained by the Contractor and shall be made available to the
Secretary or his/her authorized representative for audit.

h. Sales and Exportation — The Contractor shall endeavor to dispose of the minerals
and by-products produced in the Contract Area at the highest commercially
achievable market price and lowest commercially achievable commissions and
related fees in the circumstances then prevailing and to negotiate for sales terms and
conditions compatible with world market conditions. The Contractor may enter into
long term sales and marketing contracts or foreign exchange and commodity hedging
284

contracts which the Government acknowledges to be acceptable notwithstanding that


the sale price of minerals may from time to time be lower, or that the terms and
conditions of sales are less favorable, than those available elsewhere.

The Government shall be informed by the Contractor when it enters into a marketing
agreement with both foreign and local buyers. The Contractor shall provide the
Government a copy of the final marketing agreement entered into with buyers subject
to the confidentiality clause of the FTAA.

The Government shall be entitled to check and inspect all sales and exportation of
minerals and/or mineral products including the terms and conditions of all sales
commitments.

Sales commitments with affiliates, if any, shall be made only at prices based on or
equivalent to arm's length sales and in accordance with such terms and conditions at
which such agreement would be made if the parties had not been affiliated, with due
allowance for normal selling discounts or commissions. Such discounts or
commissions allowed the affiliates must be no greater than the prevailing rate so that
such discounts or commissions will not reduce the net proceeds of sales to the
Contractor below those which it would have received if the parties had not been
affiliated. The Contractor shall, subject to confidentiality clause of the FTAA, submit to
the Government evidence of the correctness of the figures used in computing the
prices discounts and commissions, and a copy of the sales contract.

The Contractor undertakes that any mining, processing or treatment of Ore by the
Contractor shall be conducted in accordance with such generally accepted
international standards as are economically and technically feasible, and in
accordance with such standards the Contractor undertakes to use all reasonable
efforts to optimize the mining recovery of Ore from proven reserves and metallurgical
recovery of minerals from the Ore: Provided, That it is economically and technically
feasible to do so.

For purposes of this Clause 3-h, an affiliate of an affiliated company means:

a) any company in which the Contractor holds fifty percent (50%) or more of
the shares;

b) any company which holds fifty percent (50%) or more of the Contractor's
shares;

c) any company affiliated by the same definition in (a) or (b) to an affiliated


company of the Contractor is itself considered an affiliated company for
purposes of the FTAA;

d) any company which, directly or indirectly, is controlled by or controls, or is


under common control by the Contractor;

e) any shareholder or group of shareholders of the Contractor or of an


affiliated company; or

f) any individual or group of individuals in the employment of the Contractor


or of any affiliated company.

Control means the power exercisable, directly or indirectly, to direct or cause the
direction of the management and policies of a company exercised by any other
company and shall include the right to exercise control or power to acquire control
directly or indirectly, over the company's affairs and the power to acquire not less
than fifty percent (50%) of the share capital or voting power of the Contractor. For this
285

purpose, a creditor who lends, directly or indirectly, to the contractor, unless he has
lent money to the Contractor in the ordinary course of money-lending business, may
be deemed to be a Person with power to acquire not less than fifty percent (50%) of
the share capital or voting power of the Contractor if the amount of the total of its loan
is not less than fifty percent (50%) of the total loan capital of the company. cdll

If a person ("x") would not be an affiliate of an affiliated company ("y") on the basis of
the above definition but would be an affiliate if each reference in that definition to "fifty
percent (50%)" was read as a reference to "forty percent (40%)" and the Government
has reasonable grounds for believing that "x" otherwise controls "y" or "x" is otherwise
controlled by "y," then, upon the Contractor being notified in writing by the
Government of that belief and the grounds therefore, "x" and "y" shall be deemed to
be affiliates unless the Contractor is able to produce reasonable evidence to the
contrary.

i. Price or Cost Transfers. The Contractor commits itself not to engage in transactions
involving price or cost transfers in the sale of minerals or mineral products and in the
purchase of input goods and services resulting either in the illegitimate loss or
reduction of Government Share or illegitimate increase in Contractor's share. If the
Contractor engages affiliates or an affiliated company in the sale of its mineral
products or in providing goods, services, loans or other forms of financing hereunder,
it shall do so on terms no less than would be the case with unrelated persons in
arms-length transactions.

SECTION 4. Pro Forma FTAA Contract

The fiscal regime provided herein, and the terms and conditions provided in the
Mining Act and IRR shall be embodied in a Pro Forma FTAA Contract to be prepared
by the Department of Environment and Natural Resources. The Pro Forma FTAA
Contract shall also incorporate such other provisions as the DENR may formulate as
a result of consultations or negotiations conducted for that purpose with concerned
entities.

The Pro Forma FTAA Contract shall be used by the DENR, the Negotiating Panel
and the mining applicant for negotiation of the terms and conditions of the FTAA:
Provided, That the terms and conditions provided in the Pro Forma FTAA Contract
shall be incorporated in each and every FTAA.

SECTION 5. Status of Existing FTAAs

All FTAAs approved prior to the effectivity of this Administrative Order shall
remain valid and be recognized by the Government: Provided, That should a
Contractor desire to amend its FTAA, it shall do so by filing a Letter of Intent
(LOI) to the Secretary thru the Director. Provided, further, That if the Contractor
desires to amend the fiscal regime of its FTAA, it may do so by seeking for the
amendment of its FTAA's whole fiscal regime by adopting the fiscal regime
provided hereof: Provided, finally, That any amendment of an FTAA other than
the provision on fiscal regime shall require the negotiation with the Negotiating
Panel and the recommendation of the Secretary for approval of the President of
the Republic of the Philippines.

SECTION 6. Repealing Clause

All orders and circulars or parts thereof inconsistent with or contrary to the provisions
of this Order are hereby repealed, amended or modified accordingly.

SECTION 7. Effectivity
286

This Order shall take effect fifteen (15) days upon its complete publication in
newspaper of general circulation and fifteen (15) days after registration with the
Office of the National Administrative Register.

(SGD.) ANTONIO H. CERILLES


Secretary

60 G.R. Nos. L-18843 & 18844, 29 August 1974; Supra, note 77.

61 323 Phil. 297 (1996).

62 112 Phil. 24 (1961).

63 156 Phil. 498 (1974).

64 Ruben E. Agpalo, STATUTORY CONSTRUCTION, p. 217 (1998 Ed.), citing Commissioner


of Customs v. Court of Appeals, G.R. No. 33471, 31 January 1972, 43 SCRA 192; Asturias
Sugar Central, Inc. v. Commissioner of Customs, G.R. No. 19337, 30 September 1969, 29
SCRA 617; People v. Kottinger, 45 Phil. 352 (1923).

65 Section IX of the WMCP FTAA, entitled "Option to Convert into MPSA," provides:

9.1 The Contractor may, at any time, give notice to the Secretary of its intention to
convert this Agreement either in whole or in part into one or more Mineral Production
Sharing Agreements in the form of the Agreement annexed hereto in Annexure B ("the
MPSA") over such part or parts of the Contract Area as are specified in the notice.

66The five Mineral Production Sharing Agreements (Annexes A to F) attached to the 20


October 2004 Compliance of the Solicitor General are: (1) Mineral Production Sharing
Agreement between the Republic of the Philippines and Ungay-Malobago Mines, Inc. and
Rapu-Rapu Minerals, Inc. dated 12 September 2000; (2) Mineral Production Sharing
Agreement between the Republic of the Philippines and Ungay-Malobago Mines, Inc. and TVI
Resource Development (Phils.), Inc. dated 17 June 1998; (3) Mineral Production Sharing
Agreement between the Republic of the Philippines and Base Metals Mineral Resources
Corporation (BMMRC) dated 20 November 1997; (4) Mineral Production Sharing Agreement
between the Republic of the Philippines and Philex Gold Philippines, Inc. dated 29 December
1999 (MPSA No. 148-99XIII); and (5) Mineral Production Sharing Agreement between the
Republic of the Philippines and Philex Gold Philippines, Inc. dated 29 December 1999 (MPSA
No. 149-99-XIII).

67 p. 1140, 2003 Edition.

68Cebu Portland Cement Company v. Municipality of Naga, Cebu, et al., 133 Phil. 695
(1968).

69 Resins, Inc. v. Auditor General, 134 Phil. 697 (1968).

70Luzon Surety Co., Inc. v. De Garcia, et al., 140 Phil. 509 (1969); Quijano, et al. v.
Development Bank of the Phils., et al., 146 Phil. 283 (1970); Chartered Bank Employees
Association v. Ople, No. L-44717, 28 August 1985, 138 SCRA 273.

71 Motion for Reconsideration dated 14 July 2004, p. 22.

72 Ibid., p. 20.

73 Ibid., p. 12.
287

74 Memorandum dated 15 July 2004, p. 42.

75 www.malampaya.com

76 Ibid.

77 157 Phil. 608 (1974).

CARPIO MORALES, J.:

1 421 SCRA 148 (2004).

2Section 3 (aq); Section 23; Sections 33-41; Section 56; Section 81, pars. 2-3; and Section
90.

3 Rep. Act No. 7942 (1995).

4In its Motion for Intervention, intervenor PCM alleged that the Court's January 27, 2004
Decision in this case would adversely affect the ability of domestic mining companies to
contract with their foreign counterparts with regard to mining operations beyond the resources
of the local companies. (Rollo, at 2096.)

5 Transcript of Stenographic Notes, June 29, 2004 (TSN) at 129.

6 Rules of Court, Rule 18, sec. 7.

7 La Buga-B'Laan Tribal Association, Inc. v. Ramos, 421 SCRA 148 (2004).

8 Id. at 173-174.

9 Id. at 234.

10Memorandum (In support of WMCP's Motion and Supplemental Motion for


Reconsideration) at 42-43.

11 Final Memorandum for the Petitioners at 9.

12 Angara v. Electoral Commission, 63 Phil. 139, 156-158 (1936).

13 Bengson v. Senate Blue Ribbon Committee, 203 SCRA 767, 775-776 (1991).

14 Const., art. VIII, sec. 1.

15 Tañada v. Cuenco, 103 Phil. 1051, 1067 (1957).

16 Valmonte v. Belmonte, Jr., 170 SCRA 256, 268 (1989).

17 Ibid.

18 Francisco, Jr. v. House of Representatives, 415 SCRA 44, 143-151 (2003).

19 Ibid.

20 Vide: La Bugal-B'Laan Tribal Association, Inc. v. Ramos, supra at 207-208.


288

21 Memorandum for WMCP at 37.

22 Id. at 38.

23 Id. at 39.

24 Ibid.

25 Memorandum for Public Respondents at 34.

26 Id. at 37.

27 Id. at 21.

28 Id. at 22.

29 Rollo at 1373-1374.

30 Memorandum for Public Respondents at 24.

31 Ibid.

32 Id. at 25.

33 Id. at 23.

34 Memorandum for Intervenor at 7.

35 Statement for Intervenor at 1.

36 Memorandum for Intervenor at 9.

37 Vide: Black's Law Dictionary 156 (6th ed., 1991).

38 Article 1440 of the Civil Code provides:

Art. 1440. A person who establishes a trust is called a trustor; one in whom
confidence is reposed as regards property for the benefit of another person is known
as the trustee; and the person for whose benefit the trust has been created is referred
to as the beneficiary.

Justice Jose C. Vitug (ret.) describes a trust relationship as follows:

A trust is a juridical relationship that exists between one person having the equitable
title or beneficial enjoyment of property, real or personal, and another having the legal
title thereto. The person who establishes the trust is the trustor (or grantor); one in
whom confidence is reposed as regards property for the benefit of another person is
known as the trustee (fiduciary), and the person for whose benefit the trust has been
created is referred to as the beneficiary (cestui que trust). The Code has adopted the
principles of the general law of trusts, insofar as they are not in conflict with its
provisions, the Code of Commerce, the Rules of Court and special laws. [III J.C.
Vitug Civil Law 175 (2003); citations omitted]

39 Vide: Black's Law Dictionary 156 (6th ed., 1991).


289

40 Const. art. II, sec. 1.

41 Memorandum for Petitioners at 11.

42 Memorandum for WMCP at 59.

43 Oposa v. Factoran, Jr., 224 SCRA 792, 803 (1993).

44Vide: Miners Association of the Philippines, Inc. v. Factoran, Jr., 240 SCRA 100, 106
(1995).

45 Vide: Rep. Act No. 7942 (1995), sec. 26 (c).

46 Memorandum for Public Respondents at 49.

47For instance an article written by Patricia Thompson describes the 1996 Marcopper
environmental disaster:

Between 2.4 and 4 million tons of tailings solids escaped from an open pit
impoundment at Marcopper's copper mine on the island of Marinduque in the
Philippines on March 24, 1996, when a concrete drainage plug gave way. The
sediment-laden water flowed into the Boac River system at rates of 5 to 10 cubic
meters per second. Although "independent studies by the United Nations and the
Philippine Department of Science and Technology have concluded that the escaped
material is not toxic," the increased sediment load in the Boac River led to substantial
salt and freshwater kills. An impact assessment estimated that ten years would
elapse before freshwater fish would be viable in the river again and predicted a
seventy percent reduction in the "salt water fish catch from the mouth of the Boac
River," however, there are some indications that this initial estimate may be too high.
Although the Boac River itself is not a drinking water source, the release threatened
potable water supplies along the banks of the river and necessitated airdrops of food
and medical supplies. [P. Thompson, II. Mining Criminal Sanctions Sought in
Philippine Mine Tailings Spill, 1996 Colo. J. Int'l Envt'l. l. & Pol'y 54 (1996).]

48 Vide: Oposa v. Factoran, Jr., supra.

49II J. Aruego, The Framing of the Philippine Constitution 605-606 (1949); vide: La Bugal-
B'Laan Tribal Association, Inc. v. Ramos, supra at 192, note 111.

50 Vide: Pres. Decree No. 87 (Amending Presidential Decree No. 8 issued on October 2,
1972, and Promulgating an Amended Act to Promote the Discovery and Production of
Indigenous Petroleum and Appropriate Funds therefor), Pres. Decree No. 151 (Allowing
Citizens of the Philippines or Corporations or Associations at least Sixty Per Centum of the
Capital of which is Owned by such Citizens to Enter into Service Contracts with Foreign
Persons, Corporations for the Exploration, Development, Exploitation or Utilization of Lands
of the Public Domain, amending for the purpose certain provisions of Commonwealth Act No.
141), Pres. Decree No. 463 (Providing for A Modernized System of Administration and
Disposition of Mineral Lands and to Promote and Encourage the Development and
Exploitation thereof), and Pres. Decree No. 1442 (An Act to Promote the Exploration and
Development of Geothermal Resources).

51 La Bugal-B'Laan Tribal Association, Inc. v. Ramos, supra at 199-205 & 233, note 252.

52 Id. at 234.

53 Caltex (Philippines), Inc. v. Court of Appeals, 212 SCRA 448, 463 (1992).
290

54 Capati v. Ocampo, 113 SCRA 794, 796 (1982).

55 Const., art. XII, sec. 2, first par.

56 La Bugal-B'Laan Tribal Association, Inc. v. Ramos, supra at 208 & 218-222.

57 TSN at 37-40.

58 http://dictionary.reference.com/search?q=either

59 Ibid.

60 La Bugal-B'Laan Tribal Association, Inc. v. Ramos, supra at 252-253.

61 Laurel v. Civil Service Commission, 203 SCRA 195, 209 (1991).

62 III Record of the Constitutional Commission 316-317.

63 Id. at 358-359.

64 La Bugal-B'Laan Tribal Association, Inc. v. Ramos, supra at 224.

65 I Draft Proposal of the 1986 U.P. Law Constitution Project, Article XV at 12 -13.

66 La Bugal-B'Laan Tribal Association, Inc. v. Ramos, supra at 217-218.

67 Id. at 208 & 218-222.

68 Vide: Section 1 ("No person shall be deprived of life, liberty or property without due process
of law, nor shall any person be denied of the equal protection of the laws."); Section 4 ("No
law shall be passed abridging the freedom of speech, of expression, or of the press, or the
right of the people peaceably to assemble and petition the government for redress of
grievances."); Section 5 ("No law shall be made respecting an establishment of religion, or
prohibiting the exercise thereof. The free exercise and enjoyment of religious profession and
worship, without discrimination or preference, shall forever be allowed. No religious test shall
be required for the exercise of civil or political rights.")

69 I Draft Proposal of the 1986 U.P. Law Constitution Project, Article XV at 11-12.

70P. A. Agabin, Service Contracts: Old Wines in New Bottles?, II Draft Proposal of the 1986
U.P. Law Constitution Project 16, cited in La Bugal-B'Laan Tribal Association, Inc. v.
Ramos, supra at 229.

71A case omitted is to be held as intentionally omitted. [Black's Law Dictionary 219 (6th ed.,
1991)]

72 371 SCRA 196 (2001).

73 Id. at 205.

74 La Bugal-B'Laan Tribal Association, Inc. v. Ramos, supra at 220.

75 The expression of one thing is the exclusion of another. [Black's Law Dictionary 581
(6th ed., 1991)]
291

76Vide: Canet v. Decena, G.R. No. 155344, January 20, 2004; Commissioner of Internal
Revenue v. Michel J. Lhuiller Pawnshop, Inc., 406 SCRA 178, 186 (2003); National Power
Corporation v. City of Cabanatuan,401 SCRA 259, 280 (2003); Malinias v. Commission on
Elections, 390 SCRA 480, 491 (2002); Integrated Bar of the Philippines v. Zamora, 338
SCRA 81, 109 (2000); People v. Mamac, 332 SCRA 547, 556 (2000); Mathay, Jr. v. Court of
Appeals, 320 SCRA 703, 711 (1999); Miranda v. Abaya, 311 SCRA 617, 624 (1999); City
Government of San Pablo, Laguna v. Reyes, 305 SCRA 353, 361 (1999); Centeno v. Villalon-
Pornillos, 236 SCRA 197, 203 (1994); Phil. American Life Insurance Company v.
Ansaldo, 234 SCRA 509, 515 (1994); Commissioner of Customs v. Court of Tax Appeals, 224
SCRA 665, 669-670 (1993); Ledesma v. Court of Appeals, 211 SCRA 753, 760
(1992); Montoya v. Escayo, 171 SCRA 442, 448 (1989); Singapore Airlines Local Employees
Association v. NLRC, 130 SCRA 472, 479 (1984); Vera v. Fernandez, 89 SCRA 199, 203
(1979); Central Barrio v. City Treasurer of Davao, 23 SCRA 6, 9 (1968); Catuiza v. People, 13
SCRA 538, 542 (1965); Ursal v. Court of Tax Appeals, 101 Phil. 209, 212 (1957); Vega v.
Mun. Board of the City of Iloilo, 94 Phil. 949, 953 (1954); Sotto v. Commission on Elections,
76 Phil. 516, 530 (1946).

77 That which is expressed makes that which is implied to cease. [Black's Law Dictionary 581
(6th ed., 1991)]

78Vide: Canet v. Decena, G.R. No. 155344, January 20, 2004; Malinias v. Commission on
Election 390 SCRA 480, 491 (2002); National Electrification Administration v. Commission on
Audit, 377 SCRA 223, 232 (2002); Espiritu v. Cipriano, 55 SCRA 533, 538 (1974).

79 Comm. Villegas' response that there was no requirement in the 1973 Constitution for a law
to govern service contracts and that, in fact, there were then no such laws is inaccurate. The
1973 Charter required similar legislative approval, although it did not specify the form it
should take: "The Batasang Pambansa, in the national interest, may allow such citizens … to
enter into service contracts …" As previously noted in this Court's Decision of January 27,
2004, however, laws authorizing service contracts were actually enacted by presidential
decree [i.e. Presidential Decree No. 87 (Amending Presidential Decree No. 8 issued on
October 2, 1972, and Promulgating an Amended Act to Promote the Discovery and
Production of Indigenous Petroleum and Appropriate Funds therefore), Pres. Decree No. 151
(Allowing Citizens of the Philippines or Corporations or Associations at least Sixty Per
Centum of the Capital of which is Owned by such Citizens to Enter into Service Contracts
with Foreign Persons, Corporations for the Exploration, Development, Exploitation or
Utilization of Lands of the Public Domain, amending for the purpose certain provisions of
Commonwealth Act No. 141), Pres. Decree No. 463 (Providing for a Modernized System of
Administration and Disposition of Mineral Lands and to Promote and Encourage the
Development and Exploitation thereof), and Pres. Decree No. 1442 (An Act to Promote the
Exploration and Development of Geothermal Resources)]

80 La Bugal-B'Laan Tribal Association, Inc. v. Ramos, supra at 233-234.

81 Id. at 224.

82 III Record of the Constitutional Commission 260.

83 224 SCRA 792 (1993).

84 Id. at 811-813.

85 III Record of the Constitutional Commission 319.

86 Rollo at 2779.

87 TSN at 181-186.
292

88 Memorandum for Public Respondents, Annex 1.

89 La Bugal-B'Laan Tribal Association, Inc. v. Ramos, supra at 227-234.

90 Statement for Intervenor, p. 2.

91 La Bugal-B'Laan Tribal Association, Inc. v. Ramos, supra at 206; vide: Miners Association
of the Philippines v. Factoran, 240 SCRA 100, 104 (1995).

92P. A. Agabin, Service Contracts: Old Wines in New Bottles?, II Draft Proposal of the 1986
U.P. Law Constitution Project 3-4.

93 La Bugal-B'Laan Tribal Association, Inc. v. Ramos, supra at 227-228 citing Agabin, supra,
at 15-16.

94 Ibid.

95 Rep. Act No. 7942 (1995), secs. 35 (g), sec. 3 (af).

96 Id., sec. 3 (q).

97 Id., sec. 3 (j).

98 Id., sec. 3 (az).

99 Id., sec. 33.

100 Id., sec. 72.

101 Id., sec. 73.

102 Id., sec. 74.

103 Id., sec. 75.

104 Id., sec. 76.

105 Id., sec. 35 (h).

106 La Bugal-B'Laan Tribal Association, Inc. v. Ramos, supra at 195.

107 Vide: Pres. Decree No. 87, sec. 8 (c), (e) and (f).

108 The DENR Secretary is also empowered to charge fines for late or non-submission of
reports under Section 111 of the Mining Act, but the majority opinion either overlooked this
provision or considered it too insubstantial to be able to compel enforcement of the law and
its implementing rules.

109Section 108 provides a criminal penalty for violation of the terms and conditions of an
environmental compliance certificate, but this remedy is judicial and not administrative. In any
event, what is the likelihood of a Philippine court acquiring criminal jurisdiction over the
person of the foreign corporate officers of the foreign FTAA contractor who may be
responsible for such violations?

110 Const., art. XII, sec. 20.


293

111 Rep. Act. No. 7653 (1993), sec. 29.

112 Id. sec. 30

113 Id. sec. 37.

114 Const. art. XIII, sec. 3.

115 Pres. Decree No. 442 as amended.

116 Id. art. 128 (b).

117 Id. art. 263 (g).

118 Rep. Act No. 8424 (1997), sec. 115.

119 Id. sec. 206.

120 Id. sec. 207.

121 La Bugal-B'Laan Tribal Association, Inc. v. Ramos, supra at 196.

122 Vide: Pres. Decree No. 87, sec. 8 (k) and sec. 9 (e).

123 National Power Corporation v. Province of Albay, 186 SCRA 198, 207 (1990).

124 Progressive Development Corporation v. Quezon City, 172 SCRA 629, 635 (1989).

125 La Bugal-B'Laan Tribal Association, Inc. v. Ramos, supra at 236.

126
Guidelines Establishing the Fiscal Regime of financial or Technical Assistance
Agreements.

127 Section 3 (g) (1) of DAO 99-56 provides:

Section 3. Fiscal Regime of a Financial or Technical Assistance Agreement

xxx

g. Government Share.

1. Basic Government Share. The following taxes, fees and other such charges shall
constitute the Basic Government Share:

a) Excise tax on minerals;

b) Contractor's income tax;

c) Customs duties and fees on imported capital equipment;

d) Value added tax on the purchase of imported equipment, goods and


services;

e) Withholding tax on interest payments on foreign loans;


294

f) Withholding tax on dividends to foreign stockholders;

g) Royalties due the Government on Mineral Reservations;

h) Documentary stamps taxes;

i) Capital gains tax;

j) Local business tax;

k) Real property tax;

l) Community tax;

m) Occupation fees;

n) All other local Government taxes, fees and imposts as of the effective date
of the FTAA;

o) Special Allowance, as defined in the Mining Act; and

p) Royalty payments to any Indigenous People(s)/Indigenous Cultural


Community (ies).

From the Effective Date, the foregoing taxes, fees and other such charges
constituting the Basic Government Share, if applicable, shall be paid by the
Contractor: Provided, That above items (a) to (g) shall not be collected from the
Contractor upon the date of approval of the Mining Project Feasibility Study up to the
end of the Recovery Period. Any taxes, fees, royalties, allowances or other imposts,
which should not be collected by the Government, but nevertheless paid by the
Contractor and are not refunded by the Government before the end of the next
taxable year, shall be included in the Government Share in the next taxable year. Any
Value-Added Tax refunded or credited shall not form part of Government Share.

128 Section 3 (g) (2) of DAO 99-56 provides:

2. Additional Government Share. Prior to the commencement of Development and


Construction Phase, the Contractor may select one of the formula for calculating the
Additional Government Share set out below which the Contractor wishes to apply to
all of its Mining Operations and notify the Government in writing of that selection.
Upon the issuance of such notice, the formula so selected shall thereafter apply to all
of the Contractor's Mining Operations.

xxx

129 Section 3 (g) (2) (1) of DAO 99-56 provides:

a) Fifty-Fifty Sharing of the Cumulative Present Value of Cash Flows. The


Government shall collect an Additional Government Share from the Contractor
equivalent to an amount which when aggregated with the cumulative present value of
Government Share during the previous Contract Years and the Basic Government
Share for the current Contract Year is equivalent to a minimum of fifty percent (50%)
of the Cumulative Present Value of Project Cash Flow before financing for the current
Contract Year, as defined below.

Computation. The computation of the Additional Government Share shall commence


immediately after the Recovery Period. If the computation covers a period of less
295

than one year, the Additional Government Share corresponding to this period shall be
computed pro-rata wherein the Additional Government Share during the year shall be
multiplied by the fraction of the year after recovery. The Additional Government Share
shall be computed as follows:

Project Cash Flow Before Financing and Tax ("CF") for a taxable year shall be
calculated as follows:

CF = GO - DE + I - PE - OC

Cumulative Present Value of Project Cash Flow ("CP") shall be the sum of the
present value of the cumulative present value of project cash flow during the previous
year (CP i-1 x 1.10) and the Project Cash Flow Before Financing and Tax for the
current year ("CF"), and shall be calculated as follows:

CP = (CP i-1 x 1.10) + CF

Cumulative Present Value of Total Government Share Before Additional Government


Share ("CGB") shall be the sum of: the present value of the cumulative present value
of the Total Government Share during the previous year (CGAi-1 x 1.10), and the
Basic Government Share for the current year (BGS), and shall be calculated as
follows:

CGB = (CGA i-1 x 1.10) + BGS

The Additional Government Share ("AGS") shall be:

If: CGB > CP □ 0.5 then AGS = 0

If: CGB < CP □ 0.5 then AGS = [ CP x 0.5 ] - CGB

Cumulative Present Value of Total Government Share (CGA):

CGA = CGB + AGS

where:

BGS = Basic Government Share shall have the meaning as

described in Clause 3-g-1 hereof;

GO = Gross Output shall have the same meaning as defined in

the National Internal Revenue Code;

DE = Deductible Expenses shall have the meaning as

described in Clause 3-c hereof;

I = Interest payments on loans included in the Deductible

Expenses shall be equivalent to those referred to in Clause 3-c-8 hereof;

PE = unrecovered Pre-Operating Expenses;


296

OC = On-going Capital Expenditures as defined in Clause 3-c

hereof;

CP i-1 = cumulative present value of project cash flow during the

previous year; and

CGAi-1 = cumulative present value of total Government Share

during the previous year.

130 Section 3 (g) (2) (2) of DAO 99-56 provides:

b) Profit Related Additional Government Share. The Government shall collect an


Additional Government Share from the Contractor based on twenty-five percent
(25%) of the additional profits once the arithmetic average of the ratio of Net Income
After Tax To Gross Output as defined in the National Internal Revenue Code, for the
current and previous taxable years is 0.40 or higher rounded off to the nearest two
decimal places.

Computation. The computation of the Additional Government Share from additional


profit shall commence immediately after the Recovery Period. If the computation
covers a period of less than a year, the additional profit corresponding to this period
shall be computed pro-rata wherein the total additional profit during the year shall be
multiplied by the fraction of the year after recovery.

The additional profit shall be derived from the following formula:

If the computed average ratio as derived from above is less than 0.40:

Additional Profit = 0

If the computed average ratio is 0.40 or higher:

[NIAT-(0.40 x GO)]

Additional Profit = ------------------------

( 1 - ITR )

The Additional Government Share from the additional profit is computed using the
following formula:

Additional Government Share

From Additional Profit = 25% x Additional Profit

where:

NIAT = Net Income After Tax for the particular taxable year under consideration.

GO = Gross Output from operations during the same taxable year.


297

ITR = Income Tax Rate applied by the Bureau of Internal Revenue in computing the
income tax of the Contractor during the taxable year.

131 Section 3 (g) (2) (3) of DAO 99-56 provides:

c) Additional Share Based from the Cumulative Net Mining Revenue. The Additional
Government Share for a given taxable year shall be calculated as follows:

(i) Fifty percent (50%) of cumulative Net Mining Revenue from the end of the
Recovery Period to the end of that taxable year;

LESS

(ii) Cumulative Basic Government Share for that period as calculated under Clause 3-
g-1 hereof;

AND LESS (if applicable)

(iii) Cumulative Additional Government Share in respect of the period commencing at


the end of the Recovery Period and expiring at the end of the taxable year
immediately preceding the taxable year in question.

"Net Mining Revenue" means the Gross Output from Mining Operations during a
Calendar year less Deductible Expenses, plus Government taxes, duties and fees
included as part of Deductible Expenses.

132 Republic Act No. 8424 as amended.

133The 40% equity of the foreign stockholders in a 60-40 Filipino corporation would translate
to a 24% (40% x 60%) beneficial interest in the corporation undertaking the MPSA.

134 Of course, the 60% Filipino equity in a 60-40 Filipino corporation could also be held by
another 60-40 Filipino corporation or corporations, further diluting actual Filipino beneficial
interest and increasing foreign beneficial interest.

135 As noted in the Decision (La Bugal-B'Laan Tribal Association, Inc., supra at 212-213),
unlike E.O. 279, the Mining Act does not define "large-scale" in terms of capital expenditure
although this was evidently the way it was understood by the 1986 Constitutional
Commission. (vide: III Records of the Constitutional Commission 255).

In fact, the Mining Act does not categorically define "large-scale" at all. However, a
comparison of the maximum areas for exploration in Section 22 for Exploration
Permits (400 meridional blocks onshore for corporations), Section 28 for Mineral
Agreements (200 meridional blocks for corporations) and Section 34 for FTAAs
(1,000 meridional blocks for corporations) indicates that "large-scale" under the
Mining Act refers to the size of the contract area.

It is only Section 56 of DAO 40-96 that any reference to the US$50,000,000.00


minimum capital investment prescribed by E.O. 279 is made.

136 Applying the formula in Section 56 (a) of DAO 40-96 and assuming: (1) the foreign FTAA
contractor began with the maximum contract area of 1,000 meridional blocks onshore, (2) an
exploration period of 6 years and (3) compliance with Section 60 of DAO 40-96 on
relinquishment of areas covered by FTAA.

The figure for an exploration period of 10 years is US$ 4.8 million. The figure for a 20-
year exploration period is US$ 7.7 million.
298

One meridional block is equivalent to 81 hectares. (Website of the Philippine Mines


and Geosciences Bureau www.mgb.gov.ph/epprimer.htm)

137SECTION 23. Rights and Obligations of the Permittee. — An exploration permit


shall grant to the permittee, his heirs or successors-in-interest, the right to enter,
occupy and explore the area: Provided, That if private or other parties are affected,
the permittee shall first discuss with the said parties the extent, necessity, and
manner of his entry, occupation and exploration and in case of disagreement, a panel
of arbitrators shall resolve the conflict or disagreement.

The permittee shall undertake an exploration work on the area as specified by its
permit based on an approved work program.

Any expenditure in excess of the yearly budget of the approved work program may
be carried forward and credited to the succeeding years covering the duration of the
permit. The Secretary, through the Director, shall promulgate rules and regulations
governing the terms and conditions of the permit.

The permittee may apply for a mineral production sharing agreement, joint
venture agreement, co-production agreement or financial or technical assistance
agreement over the permit area, which application shall be granted if the
permittee meets the necessary qualifications and the terms and conditions of
any such agreement: Provided, That the exploration period covered by the
exploration permit shall be included as part of the exploration period of the mineral
agreement or financial or technical assistance agreement. (Emphasis supplied)

138SECTION 24. Declaration of Mining Project Feasibility. — A holder of an exploration


permit who determines the commercial viability of a project covering a mining area may,
within the term of the permit, file with the Bureau a declaration of mining project feasibility
accompanied by a work program for development. The approval of the mining project
feasibility and compliance with other requirements provided in this Act shall entitle the
holder to an exclusive right to a mineral production sharing agreement or other mineral
agreements or financial or technical assistance agreement. (Emphasis supplied)

139Sections 17-30 of DAO 40-96 on exploration permits contains absolutely no minimum


requirement for ground expenditures, much less the minimum required investment of US$
50,000,000.00 for development, infrastructure and utilization.

140 Vide: note 20.

141 Memorandum for WMCP, p. 2.

142 La Bugal-B'Laan Tribal Association, Inc. v. Ramos, supra at 176.

143287 SCRA 465, 474 (1998). The Constitution prohibits non-Filipinos from acquiring or
holding title to private lands or to lands of the public domain, except only by way of legal
succession.

144 Id. at 475.

145In 1997 Bre-X, a large Canadian mining firm, was found to have inflated the prospective
amount of gold deposits in its Busang, Indonesia mining operation by "salting" and tampering
with gold samples taken from the site. After news of the gold salting scam had broken out,
Bre-X's share price fell by almost 90%. [W. Symonds & M. Shari, 'After Bre-X, Gold's Glow is
Gone' Available at http:// www.businessweek.com/1997/15/b352267.htm]
299

146In January, 2004, 20% of Royal Dutch/Shell's reserves of oil and gas were reclassified
from "proven" to merely "probable" or other even less certain categories. As a result, Shell's
share prices fell by 7% ['Shell shock' Available at
http:// www.economist.co.uk/business/PrinterFriendly.cfm?Story_ID=2354469]

147 Memorandum for Petitioners at 14.

148 Memorandum for WMCP at 67.

149 US$ 4,000,000.00 or approximately P224,000,000.00.

150 Memorandum for WMCP at 16.

151 Id. at 67.

152
At the prevailing rate of exchange, the US$10,000,000.00 selling price of WMC's shares in
WMCP is worth approximately P560,000,000.00.

153 TSN at 155-156; Memorandum for WMCP at 60-61.

154 La Bugal-B'Laan Tribal Association, Inc. v. Ramos, supra at 176.

155 Id. at 243-245.

156 Memorandum for WMCP at 5.

157 Civil Code, art. 1350.

158 Section 8.3 provides:

If the Secretary gives a Rejection Notice the Parties shall promptly meet and
endeavour to agree on amendments to the Work Programme or budget. If the
Secretary and the Contractor fail to agree on the proposed revision within 30
days from delivery of the Rejection Notice then the Work Programme or Budget or
variation thereof proposed by the Contractor shall be deemed approved, so as
not to unnecessarily delay the performance of this Agreement. (Emphasis
supplied; Rollo, p. 92-93.)

159 Civil Code, art. 1409 (1).

160 Id. art. 1352.

161 Id. art. 1409.

162 R.A. No. 7942, sec. 33.

163 Id, sec. 35 (e).

164 3.3. This Agreement shall be renewed by the Government for a further period of twenty-
five (25) years under the same terms and conditions provided that the Contractor lodges a
request for renewal with the Government not less than sixty (60) days prior to the expiry of
the initial terms of this Agreement and provided that the Contractor is not in breach of any of
the requirements of this Agreement.

165 http://en.wikipedia.org/wiki/Open-pit_mining htm.


300

166 Webster's Third New International Dictionary 1579 (1976).

167 http://riot.ieor.berkeley.edu/riot/Applications/OPM/OPMDetails.html.

168 http://www.mine-engineer.com/mining/open_pit.htm; http://en.wikipedia.org/wiki/Open-


pit_mining htm.

169 http://www.mcq.org/roc/en/exploitation/exploitation_2_1_2.html.

TINGA, J.:

1 SECTION 2. All lands of the public domain, waters, minerals, coal, petroleum, and other
mineral oils, all forces of potential energy, fisheries, forests or timber, wildlife, flora and fauna,
and other natural resources are owned by the State. With the exception of agricultural lands,
all other natural resources shall not be alienated. The exploration, development, and
utilization of natural resources shall be under the full control and supervision of the State. The
State may directly undertake such activities, or it may enter into co-production, joint venture,
or production-sharing agreements with Filipino citizens, or corporations or associations at
least sixty per centum of whose capital is owned by such citizens. Such agreements may be
for a period not exceeding twenty-five years, renewable for not more than twenty-five years,
and under such terms and conditions as may be provided by law. In cases of water rights for
irrigation, water supply, fisheries, or industrial uses other than the development of water
power, beneficial use may be the measure and limit of the grant.

The State shall protect the nation's marine wealth in its archipelagic waters, territorial
sea, and exclusive economic zone, and reserve its use and enjoyment exclusively to
Filipino citizens.

The Congress may, by law, allow small-scale utilization of natural resources by


Filipino citizens, as well as cooperative fish farming, with priority to subsistence
fishermen and fishworkers in rivers, lakes, bays, and lagoons.

The President may enter into agreements with foreign-owned corporations


involving either technical or financial assistance for large-scale exploration,
development, and utilization of minerals, petroleum, and other mineral oils
according to the general terms and conditions provided by law, based on real
contributions to the economic growth and general welfare of the country. In
such agreements, the State shall promote the development and use of local
scientific and technical resources.

The President shall notify the Congress of every contract entered into in accordance
with this provision, within thirty days from its execution. (Emphasis supplied)

2Each time Sec. 2 is hereafter mentioned, it is understood to be Sec. 2, Art. XII of the
Constitution.

3 "The Philippines is a democratic and republican State. xxx" See Section 1, Article II,
Constitution. "Republicanism, in so far as it implies the adoption of a representative type of
government, necessarily points to the enfranchised citizen as a particle of popular sovereignty
and as the ultimate source of the established authority." Moya v. Del Fierro, 69 Phil. 199, 204
(1939), See also Badelles v. Cabili, 136 Phil. 383, 395-396 (1969).

4Section 1, Article VII of the Constitution states: "The executive power shall be vested in the
President of the Philippines."
301

5See Section 17, Article VII, Constitution, which reads: "The President shall have control of
all the executive departments, bureaus and offices. He shall ensure that the laws be faithfully
executed."

6 See Section 18, Article VII, Constitution, which begins: "The President shall be the
Commander-in-Chief of all armed forces of the Philippines and whenever it becomes
necessary, he may call out such armed forces to prevent or suppress lawless violence,
invasion or rebellion. xxx"

7 G.R. No. 88211, 27 October 1989, 178 SCRA 760.

8 Id. at 764. Citing the eminent American legal scholar Laurence Tribe, who notes that US
jurisprudence makes clear "that the constitutional concept of inherent power is not a synonym
for power without limit; rather, the concept suggests only that not all powers granted in the
Constitution are themselves exhausted by internal enumeration, so that, within a sphere
properly regarded as one of "executive" power, authority is implied unless there or elsewhere
expressly limited." Ibid.

9Justice Irene Cortes, who penned the Court's decision in Marcos v. Manglapus, has opined
elsewhere on the grant of plenary executive powers on the President, "[who] personifies the
executive branch. There is a unity in the executive branch absent from the two other
branches of government. The president is not the chief of many executives. He is the
executive. His direction of the executive branch can be more immediate and direct than the
United States president because he is given by express provision of the constitution control
over all executive departments, bureaus and offices." I. Cortes, The Philippine Presidency: A
Study of Executive Power, pp. 68-69; cited in Sanlakas v. Executive Secretary et al., G.R.
Nos. 159086, 159103, 159185, 159196, 3 February 2004.

10 "This case is unique. It should not create a precedent, for the case of a dictator forced out
of office and into exile after causing twenty years of political, economic and social havoc in
the country and who within the short space of three years seeks to return, is in a class by
itself." Marcos v. Manglapus, supra note 7, at 682.

11 Id. at 692. See also supra note 8. In light of the U.S. Supreme Court decision in the famed
Steel Seizure case, Youngstown Sheet v. Sawyer, supra note 2, and the competing analyses
of Justice Black (whose "formalist" approach led to rigid categorization of separate legislative,
executive and judicial functions), and Justices Frankfurter and Jackson (who opted for a more
flexible, functional approach), Gunther and Sullivan note that "[m]uch scholarly commentary
on separation of powers has endorsed the functional approach, and cite this following
argument for the "functional" view: "When the Constitution confers power, it confers power on
the three generalist political heads of authority, not on branches as such. [Its] silence about
the shape of the inevitable, actual government was a product both of drafting compromises
and of the explicit purpose to leave Congress free to make whatever arrangements it deemed
'necessary and proper' for the detailed pursuit of government purposes." G. Gunther and K.
Sullivan, Constitutional Law (14th ed., 2001), at 342; citing Strauss, "Formal and Functional
Approaches to Separation of Powers Questions – A Foolish Inconsistency," 72 Corn.L.Rev.
488 (1987).

Another analysis is proferred by Chemerinsky, who acknowledges that the debate on inherent
presidential power has existed "from the earliest days of the country." E. Chemerinsky,
Constitutional Law: Principles and Policies (2nd ed., 2002), at 329. In analyzing the U.S.
Supreme Court's divided opinions in the seminal case of Youngstown Sheet, supra note 2, he
notes that while the majority opinion of Justice Black seems to deny the existence of any
inherent presidential power, the concurring opinions of Justices Douglas, Frankfurter and
Jackson do seem to acknowledge the existence of such power, albeit subject to proscription
by the legislative branch. Chemerinsky also notes that the view of inherent presidential
authority had been affirmed in the earlier case of U.S. v. Curtiss-Wright Export Corporation,
299 U.S. 304 (1936), which pertained to the presidential power to conduct foreign policy. Id.
at 334.
302

12 Ibid. See also Sanlakas v. Executive Secretary; supra note 9.

13 Iron and Steel Authority v. Court of Appeals, 319 Phil. 648, 658 (1995).

14Apropos to the nature of the Filipino presidency is the following comment on the U.S.
presidency by an American historian, "As our Chief of State, and as such the embodiment of
the people's elective will, the President is clad with the prerogative of the office, and
possesses more actual sovereign power than any British king since George III. In his role as
Chief of Foreign Relations, from the beginning he has been the sole organ of the nation in its
external relations, and its sole representative with foreign nations. While the Senate must
advise and consent to any treaty, the President has exclusive initiative in their negotiation."
G.F. Milton, The Use of Presidential Power: 1789-1943 (1980 ed.), at 3.

15Section 1, Article VIII, Constitution enables the courts to determine whether or not there
has been a grave abuse of discretion amounting to lack or excess of jurisdiction on the part of
the executive, a duty which is made easier if there is a specifically prescribed constitutional
standard which warrants obeisance by the executive branch.

16 See Secs. 21 and 22, Art. VI, Const., which read:

Sec. 21. The Senate or the House of Representatives or any of its respective
committees may conduct inquiries in aid of legislation in accordance with its duly
published rules of procedure. The rights of persons appearing in or affected by such
inquiries shall be respected.

Sec. 22. The heads of departments may upon their own initiative, with the consent of
the President, or upon the request of either House, as the rules of each House shall
provide, appear before and be heard by such House or any matter pertaining to their
departments. Written questions shall be submitted to the President of the Senate or
the Speaker of the House of Representatives at least three days before their
scheduled appearance. Interpellations shall not be limited to written questions, but
may cover matters related thereto. When the security of the State or the public
interest so requires and the President so states in writing, the appearance shall be
conducted in executive session.

17 See Section 2, Article XII, Constitution, which states in part, "All lands of the public domain,
waters, minerals, coal, petroleum, and other mineral oils, all forces of potential energy,
fisheries, forests or timber, wildlife, flora and fauna, and other natural resources are owned by
the State." An offshoot of the long-standing Regalian doctrine recognized in this jurisdiction.

18"The exploration, development, and utilization of natural resources shall be under the full
control and supervision of the State." Id.

19 The so-called "Jamir amendment," proposed by Commissioner Alberto M.K. Jamir, which
read "The President may enter into agreements with foreign-owned corporations involving
either technical or financial assistance for large-scale exploration, development and utilization
of natural resources according to the general terms and conditions provided by law based on
real contributions to the long-term growth of the economy." 3 Record of the Constitutional
Commission: Proceedings and Debates (1987), at 351.

20 Id. at 356.

21 Indeed, since 1973 when the service contract system for petroleum was implemented, the
government has earned over 1.882 Billion Pesos and 10.160 Billion Pesos in revenues from
oil and natural gas production, respectively. Based on data provided by the Department of
Energy.
303

22Paragraph 5, Sec. 2, Art. XII. It provides: The President shall notify the Congress of every
contract entered into in accordance with this provision, within thirty days from its execution.

23 See Section 9, Article XIV, 1973 Constitution.

24 Resolution, p. 26.

25Per Jackson, J., concurring, Youngstown Sheet & Tube Co. v. Sawyer, 343 U.S. 579
(1952).

26The following exchanges during the debates of the 1987 Constitutional Commission
indicate that the absence of domestic capital for mineral and petroleum development was
duly considered by the framers.

MR. GASCON. As far as investment is concerned in developing certain priority areas


for our economic development, are there areas where there is much need for foreign
investments?

MR. VILLEGAS. During the public hearings, we heard people from the mining and oil
exploration industries, who presented a very strong case, that foreign investment is
actually indispensable because there is no risk capital available in the Philippines. If
the Gentleman will remember, the figure cited over the last ten years is that P800
million literally went down the drain in oil exploration and up to now, no oil has been
found, and all that money was foreign money. These people asked a rhetorical
question: Can you imagine if that money belonged to Filipinos? 3 Record of the
Constitutional Commission: Proceedings and Debates (1987), at 310.

xxx

MR. DAVIDE. I am very glad that Commissioner Padilla emphasized minerals,


petroleum and mineral oils. The Commission has just approved the possible foreign
entry into the development, exploration and utilization of these minerals, petroleum
and other mineral oils by virtue of the Jamir amendment. I voted in favor of the Jamir
amendment because it will eventually give way to vesting in exclusively Filipino
citizens and corporations wholly owned by Filipino citizens the right to utilize the other
natural resources. This means that as a matter of policy, natural resources should be
utilized and exploited only by Filipino citizens or corporations wholly owned by such
citizens. But by virtue of the Jamir amendment, since we feel that Filipino capital may
not be enough for the development and utilization of minerals, petroleum and other
mineral oils, the President can enter into service contracts with foreign corporations
precisely for the development and utilization of such resources. 3 Record of the
Constitutional Commission: Proceedings and Debates (1987), at 361.

27 Invalidity of provisions which do not adequately assert constitutional rights or prerogatives


need not always be the proper remedy, considering, as Justice Vitug noted in his separate
opinion in this case, that "[t]he fundamental law is deemed written in every contract." Vitug,
J., Separate Opinion, La Bugal-B'laan Tribal Association, Inc. v. Ramos, G.R. No. 127882, 27
January 2004.

28N. Hamilton, The Iron Range Resources and Rehabilitation Board: An Unconstitutional and
Confused Delegation of Executive Power to Legislators, 25 William Mitchell Law Rev. 1204,
1235 (1999).

29The following traditional observation of John Thurston, as cited in a periodical article, bears
noting:

[Thurston] explained that the day-to-day administration of the corporation should be


independent of the executive and the legislature, but "[I]n matters of general and
304

public policy, the corporation must necessarily be subject to executive and legislative
control." In addition to having control over "general and public policy," the executive
and legislature also should monitor the efficiency of the public corporation. However,
Thurston perceived a dilemma in balancing the need "to ensure that the corporation
functions efficiently and without waste," and the problem of "preventing unnecessary
interference with details of administration." xxx Id., at 1231.

30Interpretatio talis in ambiguis simper fienda est, ut evitur inconveniens et absurdum. Where
there is ambiguity, such interpretation as will avoid inconvenience and absurdity is to be
adopted. Cosico v. NLRC, 338 Phil. 1080, 1089 (1997); citing Commissioner of Internal
Revenue v. TMX Sales, Inc., 205 SCRA 184, 188 (1992).

31United Nations Technical Assistance Administration, Some Problems in the Organization


and Administration of Public Enterprise in the Industrial Field 8 (1954), cited in
Hamilton, supra note 35, at 1230. "As long as an enterprise is not clearly differentiated from
other types of governmental activity, strong pressures will be brought to make it conform to
standard government regulations and procedures." Ibid.

32 Id. at 1228.

33 Ibid.

34 The employment of the corporate entity was suggested by Neil W. Hamilton, a Professor of
Regulatory Policy in the William Mitchell College of Law, in his article analyzing the
effectiveness and economic efficiency of a government board for the rehabilitation iron mines
in Minnesota, U.S.A. which were being depleted. Professor Hamilton proffered the view that
the executive and the legislative branches of government would have control over the general
and public policy concerning the operation of iron mines and should monitor the efficiency of
the public corporation created to take care of the operation of iron mines, but the corporation,
through its board of directors and officers, would have control over day-to-day operations.
("The Iron Range Resources and Rehabilitation Board: An Unconstitutional and Confused
Delegation of Executive Power to Legislators," 25 William Mitchell Law Review 1203 [1999] ).

35The following perspective from sectors not affiliated with the business community deserve
contemplation:

"Creating a favorable investment climate for foreign mining companies has led to new
social problems, namely human rights problems and dislocation of indigenous
peoples. The country has experienced incidents of armed violence from mining
guards and military personnel assigned to assist the mining companies. Indigenous
tribes have been displaced as military operations facilitate the entry of corporations
into mining areas. Mining operations are severely infringing on communities and their
livelihoods. In 1996, a mining tailings spill from the Marcopper tailing dam in
Marinduque seriously polluted the Boac River and Calancan Bay on which the local
communities depend." See http://www.foe.org/camps/intl/imf/selling/asia4.html.

"At risk to the peoples of the Philippines is their remaining patrimony and economic
sovereignty. Mining legislation opens up the country to further foreign domination and
control. It perpetuates the semi-feudal, semi-capitalist neocolonial character of the
economy. It is creating mass displacement, especially of indigenous communities and
upland farmers. Foreign companies have an abominable history of creating
environmental disasters as well, and turning virgin forests and clean water sources
and farming lands into wastelands and deserts. They also have a terrible reputation
for excessive exploitation of workers and mass unemployment. Finally, foreign owned
mines will bring militarization as the owners will guard mining areas." B.J. Warden, at
http://www.canadianliberty.bcca/relatedinfo/miningco.html.
305

USERO v CA (G.R. No. 152115, January 26, 2005)

THIRD DIVISION

G.R. No. 152115 January 26, 2005

NIMFA USERO, petitioner,


vs.
COURT OF APPEALS and SPS. HERMINIGILDO & CECILIA POLINAR, respondents.

x--------------------------------x

G.R. No. 155055 January 26, 2005

LUTGARDA R. SAMELA, petitioner,


vs.
COURT OF APPEALS and SPS. HERMINIGILDO & CECILIA POLINAR, respondents.

DECISION

CORONA, J.:

Before this Court are two consolidated petitions for review on certiorari under Rule 45 of the Rules of
Court. The first petition, docketed as G.R. No. 152115, filed by Nimfa Usero, assails the September
19, 2001 decision1 of the Court of Appeals in CA-GR SP No. 64718. The second petition, docketed as
G.R. No. 155055, filed by Lutgarda R. Samela, assails the January 11, 2002 decision2 of the Court of
Appeals in CA-GR SP NO. 64181.

The undisputed facts follow.

Petitioners Lutgarda R. Samela and Nimfa Usero are the owners respectively of lots 1 and 2, Block 5,
Golden Acres Subdivision, Barrio Almanza, Las Piñas City.

Private respondent spouses Polinar are the registered owners of a parcel of land at no. 18 Anahaw
St., Pilar Village, Las Piñas City, behind the lots of petitioners Samela and Usero.

Situated between the lots of the parties is a low-level strip of land, with a stagnant body of water filled
with floating water lilies; abutting and perpendicular to the lot of petitioner Samela, the lot of the
Polinars and the low-level strip of land is the perimeter wall of Pilar Village Subdivision.

Apparently, every time a storm or heavy rains occur, the water in said strip of land rises and the
strong current passing through it causes considerable damage to the house of respondent Polinars.
Frustrated by their predicament, private respondent spouses, on July 30, 1998, erected a concrete
wall on the bank of the low-level strip of land about three meters from their house and rip-rapped the
soil on that portion of the strip of land.

Claiming ownership of the subject strip of land, petitioners Samela and Usero demanded that the
spouses Apolinar stop their construction but the spouses paid no heed, believing the strip to be part of
a creek. Nevertheless, for the sake of peace, the Polinars offered to pay for the land being claimed by
petitioners Samela and Usero. However, the parties failed to settle their differences.

On November 9, 1998, petitioners filed separate complaints for forcible entry against the Polinars at
the Metropolitan Trial Court of Las Piñas City. The case filed by petitioner Samela was docketed as
Civil Case No. 5242, while that of petitioner Usero was docketed as Civil Case No. 5243.
306

In Civil Case No. 5242, petitioner Samela adduced in evidence a copy of her Transfer Certificate of
Title, plan of consolidation, subdivision survey, the tax declaration in her name, and affidavits of
petitioner Usero and a certain Justino Gamela whose property was located beside the perimeter wall
of Pilar Village.

The spouses Polinar, on the other hand, presented in evidence their own TCT; a barangay
certification as to the existence of the creek; a certification from the district engineer that the western
portion of Pilar Village is bound by a tributary of Talon Creek throughout its entire length; boundary
and index map of Pilar Village showing that the village is surrounded by a creek and that the Polinar
property is situated at the edge of said creek; and pictures of the subject strip of land filled with water
lilies.

On March 22, 1999, the trial court rendered a decision in favor of petitioner Samela:

WHEREFORE, the Court hereby renders judgment ordering the defendants to vacate and remove at
their expense the improvements made on the subject lot; ordering the defendants to pay the plaintiff
₱1,000.00 a month as reasonable compensation for the use of the portion encroached from the filing
of the complaint until the same is finally vacated; and to pay plaintiff ₱10,000.00 as reasonable
attorney’s fees plus costs of suit.31ªvvphi1.nét

In a parallel development, the Metropolitan Trial Court, in Civil Case No. 5243, issued an order on
February 29, 2000, directing petitioner Usero and the Polinar spouses to commission a professional
geodetic engineer to conduct a relocation survey and to submit the report to the trial court.

On April 24, 2000, Mariano Flotilde, a licensed geodetic engineer, conducted a relocation survey of
Usero’s property covered by TCT No. T- 29545. The result of the said relocation survey, as stated in
his affidavit, was as follows:

1. That I executed a relocation survey of Lot 2, Block 5, (LRC) PCS-4463 covered by TCT No.
T-29545 registered in the name of Nimfa O. Usero;

2. That according to my survey, I found out that there is no existing creek on the boundary of
the said lot;

3. That based on the relocation plan surveyed by the undersigned, attached herewith,
appearing is the encroachment on the above-mentioned lot by Spouses Herminigildo and
Cecilia Polinar with an area of FORTY THREE (43) SQUARE METERS;

4. That this affidavit was made in compliance with Court Order dated February 23, 2000 of
Metropolitan Trial Court, Las Piñas City, Branch LXXIX.4

On August 25, 2000, the Metropolitan Trial Court decided in favor of petitioner Usero:

WHEREFORE, judgment is hereby rendered in favor of the plaintiff and against the defendants
ordering them:

a) To vacate and remove at their expense the improvement made on the subject lot;

b) To pay the plaintiff ₱1,000.00 a month as reasonable compensation for the portion
encroached from the time of the filing of the complaint until the same is finally vacated;

c) To pay plaintiff ₱10,000.00 as reasonable attorney’s fees plus costs of suit.

SO ORDERED.5

The Polinar spouses appealed the decisions of the two Municipal Trial Courts to the Regional Trial
Court of Las Piñas, Branch 253 which heard the appeals separately.
307

On December 20, 2000, the Regional Trial Court, deciding Civil Case No. 5242, reversed the decision
of the trial court and ordered the dismissal of the complaint. It confirmed the existence of the creek
between the northwestern portion of the lot of petitioner Samela and the southwestern portion of the
lot of the spouses Polinar:

Finding the existence of a creek between the respective properties of the parties, plaintiff-appellee
cannot therefore lay claim of lawful ownership of that portion because the same forms part of public
dominion.1a\^/phi1.net Consequently, she cannot legally stop the defendants-appellants from rip-
rapping the bank of the creek to protect the latter’s property from soil erosion thereby avoiding danger
to their lives and damage to property.

Absent a lawful claim by the plaintiff-appellee over the subject portion of that lot, defendants-
appellants are not duty bound to pay the former compensation for the use of the same. As a result,
they may maintain the said improvements introduced thereon subject to existing laws, rules and
regulations and/or ordinances appurtenant thereto.

WHEREFORE, premises considered, the Decision rendered by Branch 79 of the Metropolitan Trial
Court, Las Piñas is REVERSED. Accordingly, the instant complaint is DISMISSED.

SO ORDERED.6

On March 16, 2001, the Regional Trial Court, in Civil Case No. 5243, also reversed the finding of the
Municipal Trial Court:

From the foregoing, defendants-appellants may maintain the improvements introduced on the subject
portion of the lot subject to existing laws, rules and regulations and/or ordinances pertaining thereto.
Consequently, no compensation may be awarded in favor of the plaintiff-appellee.

WHEREFORE, premises considered, the above-mentioned Decision rendered by Branch 79 of the


Las Piñas City Metropolitan Trial Court is REVERSED. Accordingly, the instant complaint is
DISMISSED.

From the adverse decisions of the Regional Trial Court, petitioners filed their respective petitions for
review on certiorari to the Court of Appeals. Petitioner Samela’s case was docketed as CA-G.R. SP
64181 while that of petitioner Usero was docketed as CA-G.R. SP 64718.1awphi1.nét

Both petitions failed in the CA. Thus the instant consolidated petitions.

The pivotal issue in the case at bar is whether or not the disputed strip of land, allegedly encroached
upon by the spouses Polinar, is the private property of petitioners or part of the creek and therefore
part of the public domain. Clearly this an issue which calls for a review of facts already determined by
the Court of Appeals.

The jurisdiction of the Court in petitions for review on certiorari under Rule 45 of the Rules of Court is
limited to reviewing only errors of law, not of fact, unless the factual findings complained of are devoid
of support by the evidence on record or the assailed judgment is based on a misapprehension of
facts.7 This is obviously not the case here.

A careful scrutiny of the records reveals that the assailed decisions are founded on sufficient
evidence. That the subject strip of land is a creek is evidenced by: (1) a barangay certification that a
creek exists in the disputed strip of land; (2) a certification from the Second Manila Engineering
District, NCR-DPWH, that the western portion of Pilar Village where the subject strip of land is located
is bounded by a tributary of Talon Creek and (3) photographs showing the abundance of water lilies in
the subject strip of land. The Court of Appeals was correct: the fact that water lilies thrive in that strip
of land can only mean that there is a permanent stream of water or creek there.
308

In contrast, petitioners failed to present proof sufficient to support their claim. Petitioners presented
the TCTs of their respective lots to prove that there is no creek between their properties and that of
the Polinars. However, an examination of said TCTs reveals that the descriptions thereon are
incomplete. In petitioner Samela’s TCT No. T-30088, there is no boundary description relative to the
northwest portion of the property pertaining to the site of the creek. Likewise in TCT No. T-22329-A of
the spouses Polinar, the southeast portion which pertains to the site of the creek has no described
boundary. Moreover the tax declaration presented by petitioner is devoid of any entry on the "west
boundary" vis-a-vis the location of the creek. All the pieces of evidence taken together, we can only
conclude that the adjoining portion of these boundaries is in fact a creek and belongs to no one but
the state.

Property is either of public dominion or of private ownership.8 Concomitantly, Article 420 of the Civil
Code provides:

ART. 420. The following things are property of public dominion:

(1) Those intended for public use, such as roads, canals, rivers, torrents, ports and bridges
constructed by the State, banks, shores, roadsteads, and others of similar character;

The phrase "others of similar character" includes a creek which is a recess or an arm of a river. It is
property belonging to the public domain which is not susceptible to private ownership. 9 Being public
water, a creek cannot be registered under the Torrens System in the name of any individual10 .

Accordingly, the Polinar spouses may utilize the rip-rapped portion of the creek to prevent the erosion
of their property.

WHEREFORE, the consolidated petitions are hereby denied. The assailed decisions of the Court of
Appeals in CA-G.R. SP 64181 and CA-G.R. SP 64718 are affirmed in toto.

SO ORDERED.

Panganiban, (Chairman), Sandoval-Gutierrez, Carpio-Morales, and Garcia, JJ., concur.

Footnotes

1Penned by Associate Justice Andres B. Reyes, Jr. and concurred in by Associate Justices
B. A. Adelfuin- De La Cruz and Edgardo P. Cruz of the former Special Fourteenth Division.

2Penned by Associate Justice Romeo J. Callejo Sr. (now Associate Justice of the Supreme
Court) and concurred in by Associate Justices Remedios Salazar-Fernando and Josefina
Guevarra-Salonga of the Twelfth Division.

3 G.R. No. 155055, Rollo, p. 83.

4 G.R. No. 152115, Rollo, pp. 29-30.

5 G.R. No. 152115, Rollo, p. 30.

6 Penned by Judge Jose F. Coibes Jr., Rollo, pp. 123-125.

7Magellan Capital Management Corporation v. Zosa, G.R. No. 129916 , 26 March 2001, 355
SCRA 157.
309

8 Article 419, Civil Code of the Philippines.

9 Maneclang v. IAC, L-66575, 24 May 1988, 161 SCRA 469.

10 Diego v. Court of Appeals, 102 Phil. 494 (1957).

GOVERNMENT v CABANGIS (G.R. No. L-28379, March 27, 1929)

Republic of the Philippines


SUPREME COURT
Manila

EN BANC

G.R. No. L-28379 March 27, 1929

THE GOVERNMENT OF THE PHILIPPINE ISLANDS, applicant-appellant,


vs.
CONSORCIA CABANGIS, ET AL., claimants-appellees.

Attorney-General Jaranilla for appellant.


Abad Santos, Camus & Delgado for appellees.

VILLA-REAL, J.:

The Government of the Philippine Islands appeals to this court from the judgment of the Court of First
Instance of Manila in cadastral proceeding No. 373 of the Court of First Instance of Manila, G. L. R. O.
Cadastral Record No. 373, adjudicating the title and decreeing the registration of lots Nos. 36, 39 and
40, block 3055 of the cadastral survey of the City of Manila in favor of Consuelo, Consorcia, Elvira
and Tomas, surnamed Cabangis, in equal parts, and dismissing the claims presented by the
Government of the Philippine Islands and the City of Manila.

In support of its appeal, the appellant assigns the following alleged errors as committed by the trial
court in its judgment, to wit:

1. The lower court erred in not holding that the lots in question are of the public domain, the
same having been gained from the sea (Manila Bay) by accession, by fillings made by the
Bureau of Public Works and by the construction of the break-water (built by the Bureau of
Navigation) near the mouth of Vitas Estero.

2. The lower court erred in holding that the lots in question formed part of the big parcel of
land belonging to the spouses Maximo Cabangis and Tita Andres, and in holding that these
spouses and their successors in interest have been in continuous, public, peaceful and
uninterrupted possession of said lots up to the time this case came up.

3. The lower court erred in holding that said lots existed before, but that due to the current of
the Pasig River and to the action of the big waves in Manila Bay during the south-west
monsoons, the same disappeared.

4. The lower court erred in adjudicating the registration of the lands in question in the name of
the appellees, and in denying the appellant's motion for a new trial.

A preponderance of the evidence in the record which may properly be taken into consideration in
deciding the case, proves the following facts:
310

Lots 36, 39 and 40, block 3035 of cadastral proceeding No. 71 of the City of Manila, G. L. R. O.
Record No. 373, were formerly a part of a large parcel of land belonging to the predecessor of the
herein claimants and appellees. From the year 1896 said land began to wear away, due to the action
of the waves of Manila Bay, until the year 1901 when the said lots became completely submerged in
water in ordinary tides, and remained in such a state until 1912 when the Government undertook the
dredging of Vitas Estuary in order to facilitate navigation, depositing all the sand and silt taken from
the bed of the estuary on the low lands which were completely covered with water, surrounding that
belonging to the Philippine Manufacturing Company, thereby slowly and gradually forming the lots,
the subject matter of this proceeding.

Up to the month of February, 1927 nobody had declared lot 39 for the purposes of taxation, and it was
only in the year 1926 that Dr. Pedro Gil, in behalf of the claimants and appellees, declared lot No. 40
for such purpose.

In view of the facts just stated, as proved by a preponderance of the evidence, the question arises:
Who owns lots 36, 39 and 40 in question?

The claimants-appellees contend that inasmuch as the said lots once formed a part of a large parcel
of land belonging to their predecessors, whom they succeeded, and their immediate predecessor in
interest, Tomas Cabangis, having taken possession thereof as soon as they were reclaimed, giving
his permission to some fishermen to dry their fishing nets and deposit their bancas thereon, said lots
belong to them.

Article 339, subsection 1, of the Civil Code, reads:

Article 339. Property of public ownership is —

1. That devoted to public use, such as roads, canals, rivers, torrents, ports and bridges
constructed by the State, riverbanks, shorts, roadsteads, and that of a similar character.

xxx xxx xxx

Article 1, case 3, of the Law of Waters of August 3, 1866, provides as follows:

ARTICLE 1. The following are part of the national domain open to public use:

xxx xxx xxx

3. The Shores. By the shore is understood that space covered and uncovered by the
movement of the tide. Its interior or terrestrial limit is the line reached by the highest
equinoctial tides. Where the tides are not appreciable, the shore begins on the land side at
the line reached by the sea during ordinary storms or tempests.

In the case of Aragon vs. Insular Government (19 Phil., 223), with reference to article 339 of the Civil
Code just quoted, this court said:

We should not be understood, by this decision, to hold that in a case of gradual encroachment or
erosion by the ebb and flow of the tide, private property may not become 'property of public
ownership,' as defined in article 339 of the code, where it appears that the owner has to all intents
and purposes abandoned it and permitted it to be totally destroyed, so as to become a part of the
'playa' (shore of the seas), 'rada' (roadstead), or the like. . . .

In the Enciclopedia Juridica Espanola, volume XII, page 558, we read the following:

With relative frequency the opposite phenomenon occurs; that is, the sea advances and
private properties are permanently invaded by the waves, and in this case they become part
of the shore or beach. They then pass to the public domain, but the owner thus dispossessed
311

does not retain any right to the natural products resulting from their new nature; it is a de
facto case of eminent domain, and not subject to indemnity.

Now then , when said land was reclaimed, did the claimants-appellees or their predecessors recover
it as their original property?

As we have seen, the land belonging to the predecessors of the herein claimants-appellees began to
wear way in 1896, owing to the gradual erosion caused by the ebb and flow of the tide, until the year
1901, when the waters of Manila Bay completely submerged a portion of it, included within lots 36, 39
and 40 here in question, remaining thus under water until reclaimed as a result of certain work done
by the Government in 1912. According to the above-cited authorities said portion of land, that is, lots
36, 39 and 40, which was private property, became a part of the public domain. The predecessors of
the herein claimants-appellees could have protected their land by building a retaining wall, with the
consent of competent authority, in 1896 when the waters of the sea began to wear it away, in
accordance with the provisions of Article 29 of the aforecited Law of Waters of August 3, 1866, and
their failure to do so until 1901, when a portion of the same became completely covered by said
waters, remaining thus submerged until 1912, constitutes abandonment.

Now then: The lots under discussion having been reclaimed from the seas as a result of certain work
done by the Government, to whom do they belong?

The answer to this question is found in article 5 of the aforementioned Law of Waters, which is as
follows:

ART. 5. Lands reclaimed from the sea in consequence of works constructed by the State, or
by the provinces, pueblos or private persons, with proper permission, shall become the
property of the party constructing such works, unless otherwise provided by the terms of the
grant of authority.

The fact that from 1912 some fishermen had been drying their fishing nets and depositing
their bancas on lots 36, 39 and 40, by permission of Tomas Cabangis, does not confer on the latter or
his successors the ownership of said lots, because, as they were converted into public land, no
private person could acquire title thereto except in the form and manner established by the law.

In the case of Buzon vs. Insular Government and City of Manila (13 Phil., 324), cited by the claimants-
appellees, this court, admitting the findings and holdings of the lower court, said the following:

If we heed the parol evidence, we find that the seashore was formerly about one
hundred brazas distant from the land in question; that, in the course of time, and by the
removal of a considerable quantity of sand from the shore at the back of the land for the use
of the street car company in filling in Calle Cervantes, the sea water in ordinary tides now
covers part of the land described in the petition.

The fact that certain land, not the bed of a river or of the sea, is covered by sea water during
the period of ordinary high tide, is not a reason established by any law to cause the loss
thereof, especially when, as in the present case, it becomes covered by water owing to
circumstances entirely independent of the will of the owner.

In the case of Director of Lands vs. Aguilar (G.R. No. 22034),1 also cited by the claimants-appellees,
wherein the Government adduced no evidence in support of its contention, the lower court said in
part:

The contention of the claimants Cabangis is to the effect that said lots are a part of the
adjoining land adjudicated to their deceased father, Don Tomas Cabangis, which, for over fifty
years had belonged to their deceased grandmother, Tita Andres, and that, due to certain
312

improvements made in Manila Bay, the waters of the sea covered a large part of the lots
herein claimed.

The Government of the Philippine Islands also claims the ownership of said lots, because, at
ordinary high tide, they are covered by the sea.

Upon petition of the parties, the lower court made an ocular inspection of said lots on
September 12, 1923, and on said inspection found some light material houses built thereon,
and that on that occasion the waters of the sea did not reach the aforesaid lots.

From the evidence adduced at the trial of this cause, it may be inferred that Tita Andres,
during her lifetime was the owner of a rather large parcel of land which was adjudicated by a
decree to her son Tomas Cabangis; the lots now in question are contiguous to that land and
are covered by the waters of the sea at extraordinary high tide; some 50 years before the sea
did not reach said strip of land, and on it were constructed, for the most part, light material
houses, occupied by the tenants of Tita Andres, to whom they paid rent. Upon her death, her
son Tomas Cabangis succeeded to the possession, and his children succeeded him, they
being the present claimants, Consuelo, Jesus, Tomas, and Consorcia Cabangis.

The Government of the Philippine Islands did not adduce any evidence in support of its
contention, with the exception of registry record No. 8147, to show that the lots here in
question were not excluded from the application presented in said proceeding.

It will be seen that in the case of Buzon vs. Insular Government and City of Manila, cited above, the
rise of the waters of the sea that covered the lands there in dispute, was due not to the action of the
tide but to the fact that a large quantity of sand was taken from the sea at the side of said land in
order to fill in Cervantes Street, and this court properly held that because of this act, entirely
independent of the will of the owner of said land, the latter could not lose the ownership thereof, and
the mere fact that the waters of the sea covered it as a result of said act, is not sufficient to convert it
into public land, especially, as the land was high and appropriate for building purposes.

In the case of the Director of Lands vs. Aguilar also cited by the claimants-appellees, the Insular
Government did not present any evidence in support of its contention, thus leaving uncontradicted the
evidence adduced by the claimants Aguilar et al., as to the ownership, possession and occupation of
said lots.

In the instant case the evidence shows that from 1896, the waves of Manila Bay had been gradually
and constantly washing away the sand that formed the lots here in question, until 1901, when the sea
water completely covered them, and thus they remained until the year 1912. In the latter year they
were reclaimed from the sea by filling in with sand and silt extracted from the bed of Vitas Estuary
when the Government dredged said estuary in order to facilitate navigation. Neither the herein
claimants-appellees nor their predecessors did anything to prevent their destruction.

In conclusion, then, we hold that the lots in question having disappeared on account of the gradual
erosion due to the ebb and flow of the tide, and having remained in such a state until they were
reclaimed from the sea by the filling in done by the Government, they are public land. (Aragon vs.
Insular Government, 19 Phil., 223; Francisco vs. Government of the Philippine Islands, 28 Phil., 505).

By virtue whereof, the judgment appealed from is reversed and lots Nos. 36, 39 and 40 of cadastral
proceeding No. 373 of the City of Manila are held to be public land belonging to the Government of
the United States under the administration and control of the Government of the Philippine Islands. So
ordered.

Johnson, Street, Malcolm, Ostrand, Johns and Romualdez, JJ., concur.


313

CEBU ACETYLENE v BERCILLES (G.R. No. L40474, August 29, 1975)

Republic of the Philippines


SUPREME COURT
Manila

SECOND DIVISION

G.R. No. L40474 August 29, 1975

CEBU OXYGEN & ACETYLENE CO., INC., petitioner,


vs.
HON. PASCUAL A. BERCILLES Presiding Judge, Branch XV, 14th Judicial District, and JOSE
L. ESPELETA, Assistant Provincial Fiscal, Province of Cebu, representing the Solicitor
General's Office and the Bureau of Lands, respondents.

Jose Antonio R Conde for petitioner.

Office of the Acting Solicitor General Hugo E. Gutierrez, Jr., Assistant Solicitor General Octavio R.
Ramirez and Trial Attorney David R. Hilario for respondents. .

CONCEPCION, Jr., J.:

This is a petition for the review of the order of the Court of First Instance of Cebu dismissing
petitioner's application for registration of title over a parcel of land situated in the City of Cebu.

The parcel of land sought to be registered was only a portion of M. Borces Street, Mabolo, Cebu City.
On September 23, 1968, the City Council of Cebu, through Resolution No. 2193, approved on
October 3, 1968, declared the terminal portion of M. Borces Street, Mabolo, Cebu City, as an
abandoned road, the same not being included in the City Development Plan.1 Subsequently, on
December 19, 1968, the City Council of Cebu passed Resolution No. 2755, authorizing the Acting
City Mayor to sell the land through a public bidding.2 Pursuant thereto, the lot was awarded to the
herein petitioner being the highest bidder and on March 3, 1969, the City of Cebu, through the Acting
City Mayor, executed a deed of absolute sale to the herein petitioner for a total consideration of
P10,800.00.3 By virtue of the aforesaid deed of absolute sale, the petitioner filed an application with
the Court of First instance of Cebu to have its title to the land registered.4

On June 26, 1974, the Assistant Provincial Fiscal of Cebu filed a motion to dismiss the application on
the ground that the property sought to be registered being a public road intended for public use is
considered part of the public domain and therefore outside the commerce of man. Consequently, it
cannot be subject to registration by any private individual.5

After hearing the parties, on October 11, 1974 the trial court issued an order dismissing the
petitioner's application for registration of title.6 Hence, the instant petition for review.

For the resolution of this case, the petitioner poses the following questions:

(1) Does the City Charter of Cebu City (Republic Act No. 3857) under Section 31,
paragraph 34, give the City of Cebu the valid right to declare a road as abandoned?
and

(2) Does the declaration of the road, as abandoned, make it the patrimonial property
of the City of Cebu which may be the object of a common contract?
314

(1) The pertinent portions of the Revised Charter of Cebu City provides:

Section 31. Legislative Powers. Any provision of law and executive order to the
contrary notwithstanding, the City Council shall have the following legislative powers:

xxx xxx xxx

(34) ...; to close any city road, street or alley, boulevard, avenue, park or square.
Property thus withdrawn from public servitude may be used or conveyed for any
purpose for which other real property belonging to the City may be lawfully used or
conveyed.

From the foregoing, it is undoubtedly clear that the City of Cebu is empowered to close a city road or
street. In the case of Favis vs. City of Baguio,7 where the power of the city Council of Baguio City to
close city streets and to vacate or withdraw the same from public use was similarly assailed, this court
said:

5. So it is, that appellant may not challenge the city council's act of withdrawing a strip
of Lapu-Lapu Street at its dead end from public use and converting the remainder
thereof into an alley. These are acts well within the ambit of the power to close a city
street. The city council, it would seem to us, is the authority competent to determine
whether or not a certain property is still necessary for public use.

Such power to vacate a street or alley is discretionary. And the discretion will not
ordinarily be controlled or interfered with by the courts, absent a plain case of abuse
or fraud or collusion. Faithfulness to the public trust will be presumed. So the fact that
some private interests may be served incidentally will not invalidate the vacation
ordinance.

(2) Since that portion of the city street subject of petitioner's application for registration of title was
withdrawn from public use, it follows that such withdrawn portion becomes patrimonial property which
can be the object of an ordinary contract.

Article 422 of the Civil Code expressly provides that "Property of public dominion, when no longer
intended for public use or for public service, shall form part of the patrimonial property of the State."

Besides, the Revised Charter of the City of Cebu heretofore quoted, in very clear and unequivocal
terms, states that: "Property thus withdrawn from public servitude may be used or conveyed for any
purpose for which other real property belonging to the City may be lawfully used or conveyed."

Accordingly, the withdrawal of the property in question from public use and its subsequent sale to the
petitioner is valid. Hence, the petitioner has a registerable title over the lot in question.

WHEREFORE, the order dated October 11, 1974, rendered by the respondent court in Land Reg.
Case No. N-948, LRC Rec. No. N-44531 is hereby set aside, and the respondent court is hereby
ordered to proceed with the hearing of the petitioner's application for registration of title.

SO ORDERED.

Makalintal, C.J, Fernando, Barredo and Aquino, JJ., concur.

Footnotes

1 Annex A, p. 11, rollo.


315

2 Annex B, p. 12, rollo.

3 Annex C, p. 13, rollo.

4 Annex D, p. 15, rollo.

5 Annex E. p. 18, rollo.

6 Annex F. P. 20, rollo.

7 G.R. No. L-29910, April 25, 1969; SCRA 1060.

Vda De TANTOCO v MUNICIPAL COUNCIL (G.R. No. L-24950, March 25, 1926)

Republic of the Philippines


SUPREME COURT
Manila

EN BANC

G.R. No. L-24950 March 25, 1926

VIUDA DE TAN TOCO, plaintiff-appellant,


vs.
THE MUNICIPAL COUNCIL OF ILOILO, defendant-appellee.

Arroyo & Evangelista for appellant.


Provincial Fiscal Borromeo Veloso for appelle.

VILLAMOR, J.:

It appears from the record that the widow of Tan Toco had sued the municipal council of Iloilo for the
amount of P42,966.40, being the purchase price of two strips of land, one on Calle J. M. Basa
consisting of 592 square meters, and the other on Calle Aldiguer consisting of 59 square meters,
which the municipality of Iloilo had appropriated for widening said street. The Court of First Instance of
Iloilo sentenced the said municipality to pay the plaintiff the amount so claimed, plus the interest, and
the said judgment was on appeal affirmed by this court.1

On account of lack of funds the municipality of Iloilo was unable to pay the said judgment, wherefore
plaintiff had a writ of execution issue against the property of the said municipality, by virtue of which
the sheriff attached two auto trucks used for street sprinkling, one police patrol automobile, the police
stations on Mabini street, and in Molo and Mandurriao and the concrete structures, with the
corresponding lots, used as markets by Iloilo, Molo, and Mandurriao.

After notice of the sale of said property had been made, and a few days before the sale, the provincial
fiscal of Iloilo filed a motion which the Court of First Instance praying that the attachment on the said
property be dissolved, that the said attachment be declared null and void as being illegal and violative
of the rights of the defendant municipality.

Plaintiffs counsel objected o the fiscal's motion but the court, by order of August 12, 1925, declared
the attachment levied upon the aforementioned property of the defendant municipality null and void,
thereby dissolving the said attachment.
316

From this order the plaintiff has appealed by bill of exceptions. The fundamental question raised by
appellant in her four assignments of error is whether or not the property levied upon is exempt from
execution.

The municipal law, section 2165 of the Administrative Code, provides that:

Municipalities are political bodies corporate, and as such are endowed with the faculties of
municipal corporations, to be exercised by and through their respective municipal government
in conformity with law.

It shall be competent for them, in their proper corporate name, to sue and be sued, to contract
and be contracted with, to acquire and hold real and personal property for municipal
purposes, and generally to exercise the powers hereinafter specified or otherwise conferred
upon them by law.

For the purposes of the matter here in question, the Administrative Code does not specify the kind of
property that a municipality may acquire. However, article 343 of the Civil Code divides the property of
provinces and towns (municipalities) into property for public use and patrimonial property. According
to article 344 of the same Code, provincial roads and foot-path, squares, streets, fountains and public
waters, drives and public improvements of general benefit built at the expense of the said towns or
provinces, are property for public use.

All other property possessed by the said towns and provinces is patrimonial and shall be subject to
the provisions of the Civil Code except as provided by special laws.

Commenting upon article 344, Mr. Manresa says that "In accordance with administrative legislation"
(Spanish) we must distinguish, as to the patrimonial property of the towns, "between that a common
benefit and that which is private property of the town. The first differs from property for public use in
that generally its enjoyment is less, as it is limited to neighbors or to a group or class thereof; and,
furthermore, such use, more or less general, is not intrinsic with this kind of property, for by its very
nature it may be enjoyed as though it were private property. The third group, that is, private property,
is used in the name of the town or province by the entities representing it and, like and private
property, giving a source of revenue."

Such distinction, however, is of little practical importance in this jurisdiction in view of the different
principles underlying the functions of a municipality under the American rule. Notwithstanding this, we
believe that the principle governing property of the public domain of the State is applicable to property
for public use of the municipalities as said municipal is similar in character. The principle is that the
property for public use of the State is not within the commerce of man and, consequently, is
inalienable and not subject to prescription. Likewise, property for public of the municipality is not
within the commerce of man so long as it is used by the public and, consequently, said property is
also inalienable.

The American Law is more explicit about this matter as expounded by Mcquilin in Municipal
Corporations, volume 3, paragraph 1160, where he says that:

States statutes often provide the court houses, jails and other buildings owned by
municipalities and the lots on which they stand shall be exempt from attachment and
execution. But independent of express statutory exemption, as a general proposition,
property, real and personal, held by municipal corporations, in trust for the benefit of their
inhabitants, and used for public purposes, is exempt.

For example, public buildings, school houses, streets, squares, parks, wharves, engines and
engine houses, and the like, are not subject to execution. So city waterworks, and a stock of
liquors carried in a town dispensary, are exempt. The reason for the exemption is obvious.
Municipal corporations are created for public purposes and for the good of the citizens in their
aggregate or public capacity. That they may properly discharge such public functions
corporate property and revenues are essential, and to deny them these means the very
317

purpose of their creation would be materially impeded, and in some instances practically
destroy it. Respecting this subject the Supreme Court of Louisiana remarked: "On the first
view of this question there is something very repugnant to the moral sense in the idea that a
municipal corporation should contract debts, and that, having no resources but the taxes
which are due to it, these should not be subjected by legal process to the satisfaction of its
creditors. This consideration, deduced from the principles of moral equity has only given way
to the more enlarged contemplation of the great and paramount interests of public order and
the principles of government."

It is generally held that property owned by a municipality, where not used for a public purpose
but for quasi private purposes, is subject to execution on a judgment against the municipality,
and may be sold. This rule applies to shares of stock owned by a municipal corporation, and
the like. But the mere fact that corporate property held for public uses is being temporarily
used for private purposes does not make it subject execution.

If municipal property exempt from execution is destroyed, the insurance money stands in lieu
thereof and is also exempt.

The members or inhabitants of a municipal corporation proper are not personally liable for the
debts of the municipality, except that in the New England States the individual liability of the
inhabitant is generally maintained.

In Corpus Juris, vol 23, page 355, the following is found:

Where property of a municipal or other public corporation is sough to be subjected to


execution to satisfy judgments recovered against such corporation, the question as to
whether such property is leviable or not is to be determined by the usage and purposes for
which it is held. The rule is that property held for public uses, such as public buildings, streets,
squares parks, promenades, wharves, landing places fire engines, hose and hose carriages,
engine houses, public markets, hospitals, cemeteries, and generally everything held for
governmental purposes, is not subject to levy and sale under execution against such
corporation. The rule also applies to funds in the hands of a public officer. Likewise it has
been held that taxes due to a municipal corporation or country cannot be seized under
execution by a creditor of such corporation. But where a municipal corporation or country
owns in its proprietary, as distinguished from its public or governmental capacity, property not
useful or used for a public purpose but for quasi private purposes, the general rule is that
such property may be seized and sold under execution against the corporation, precisely as
similar property of individuals is seized and sold. But property held for public purposes is not
subject to execution merely because it is temporarily used for private purposes, although if
the public use is wholly abandoned it becomes subject to execution. Whether or not property
held as public property is necessary for the public use is a political, rather than a judicial
question.

In the case of City of New Orleans vs. Louisiana Construction Co., Ltd. (140 U. S., 654; 35 Law. ed.,
556), it was held that a wharf for unloading sugar and molasses, open to the public, was property for
the public use of the City of New Orleans and was not subject to attachment for the payment of the
debts of the said city.

In that case it was proven that the said wharf was a parcel of land adjacent to the Mississippi River
where all shipments of sugar and molasses taken to New Orleans were unloaded.

That city leased the said wharf to the Louisiana Construction Company, Ltd., in order that it might
erect warehouses so that the merchandise upon discharge might not be spoiled by the elements. The
said company was given the privilege of charging certain fees for storing merchandise in the said
warehouses and the public in general had the right to unload sugar and molasses there by paying the
required fees, 10 per cent of which was turned over to the city treasury.
318

The United States Supreme Court on an appeal held that the wharf was public property, that it never
ceased to be such in order to become private property of the city; wherefore the company could not
levy execution upon the wharf in order to collect the amount of the judgment rendered in favor thereof.

In the case of Klein vs. City of New Orleans (98 U. S., 149; 25 Law. ed., 430), the Supreme Court of
the United States that a public wharf on the banks of the Mississippi River was public property and not
subject to execution for the payment of a debt of the City of New Orleans where said wharf was
located.

In this case a parcel of land adjacent to the Mississippi River, which formerly was the shore of the
river and which later enlarged itself by accession, was converted into a wharf by the city for public
use, who charged a certain fee for its use.

It was held that the land was public property as necessary as a public street and was not subject to
execution on account of the debts of the city. It was further held that the fees collected where also
exempt from execution because they were a part of the income of the city.

In the case of Tufexis vs. Olaguera and Municipal Council of Guinobatan (32 Phil., 654), the question
raised was whether for the payment of a debt to a third person by the concessionaire of a public
market, the said public market could be attached and sold at public auction. The Supreme Court held
that:

Even though a creditor is unquestionably entitled to recover out of his debtor's property, yet
when among such property there is included the special right granted by the Government of
usufruct in a building intended for a public service, and when this privilege is closely related to
a service of a public character, such right of the creditor to the collection of a debt owed him
by the debtor who enjoys the said special privilege of usufruct in a public market is not
absolute and may be exercised only through the action of court of justice with respect to the
profits or revenue obtained under the special right of usufruct enjoyed by debtor.

The special concession of the right of usufruct in a public market cannot be attached like any
ordinary right, because that would be to permit a person who has contracted with the state or
with the administrative officials thereof to conduct and manage a service of a public character,
to be substituted, without the knowledge and consent of the administrative authorities, by one
who took no part in the contract, thus giving rise to the possibility of the regular course of a
public service being disturbed by the more or less legal action of a grantee, to the prejudice of
the state and the public interests.

The privilege or franchise granted to a private person to enjoy the usufruct of a public market
cannot lawfully be attached and sold, and a creditor of such person can recover his debt only
out of the income or revenue obtained by the debtor from the enjoyment or usufruct of the
said privilege, in the same manner that the rights of such creditors of a railroad company can
be exercised and their credit collected only out of the gross receipts remaining after deduction
has been made therefrom of the operating expenses of the road. (Law of November 12, 1896,
extended to the overseas provinces by the royal order of August 3, 1886.)

For the reasons contained in the authorities above quoted we believe that this court would have
reached the same conclusion if the debtor had been municipality of Guinobatan and the public market
had been levied upon by virtue of the execution.

It is evident that the movable and immovable property of a municipality, necessary for governmental
purpose, may not be attached and sold for the payment of a judgment against the municipality. The
supreme reason for this rule is the character of the public use to which such kind of property is
devoted. The necessity for government service justifies that the property of public of the municipality
be exempt from execution just as it is necessary to exempt certain property of private individuals in
accordance with section 452 of the Code of Civil Procedure.
319

Even the municipal income, according to the above quoted authorities, is exempt from levy and
execution. In volume 1, page 467, Municipal Corporations by Dillon we find that:

Municipal corporations are instituted by the supreme authority of a state for the public good.
They exercise, by delegation from the legislature, a portion of the sovereign power. The main
object of their creation is to act as administrative agencies for the state, and to provide for the
police and local government of certain designated civil divisions of its territory. To this end
they are invested with certain governmental powers and charged with civil, political, and
municipal duties. To enable them beneficially to exercise these powers and discharge these
duties, they are clothed with the authority to raise revenues, chiefly by taxation, and
subordinately by other modes as by licenses, fines, and penalties. The revenue of the public
corporation is the essential means by which it is enabled to perform its appointed work.
Deprived of its regular and adequate supply of revenue, such a corporation is practically
destroyed and the ends of its erection thwarted. Based upon considerations of this character,
it is the settled doctrine of the law that only the public property but also the taxes and public
revenues of such corporations cannot be seized under execution against them, either in the
treasury or when in transit to it. Judgments rendered for taxes, and the proceeds of such
judgments in the hands of officers of the law, are not subject to execution unless so declared
by statute. The doctrine of the inviolability of the public revenues by the creditor is maintained,
although the corporation is in debt, and has no means of payment but the taxes which it is
authorized to collect.

Another error assigned by counsel for appellant is the holding of the court a quo that the proper
remedy for collecting the judgment in favor of the plaintiff was by way or mandamus.

While this question is not necessarily included in the one which is the subject of this appeal, yet we
believe that the holding of the court, assigned as error by appellant's counsel, is true when, after a
judgment is rendered against a municipality, it has no property subject to execution. This doctrine is
maintained by Dillon (Municipal Corporations, vol. 4, par. 1507, 5th ed.) based upon the decisions of
several States of the Union upholding the same principle and which are cited on page 2679 of the
aforesaid work. In this sense this assignment of error, we believe, is groundless.

By virtue of all the foregoing, the judgment appealed from should be and is hereby affirmed with costs
against the appellant. So ordered.

Avanceña, C. J., Street, Malcolm, Ostrand, Johns, Romualdez and Villa-Real., JJ., concur.

Footnotes

1R. G. No. L-22617, promulgated November 28, 1924, not reported.


320

SALAS v JARENCIO (G.R. No. L-29788, August 30, 1972)

Republic of the Philippines


SUPREME COURT
Manila

EN BANC

G.R. No. L-29788 August 30, 1972

RAFAEL S. SALAS, in his capacity as Executive Secretary; CONRADO F. ESTRELLA, in his


capacity as Governor of the Land Authority; and LORENZO GELLA, in his capacity as Register
of Deeds of Manila, petitioners-appellants,
vs.
HON. HILARION U. JARENCIO, as Presiding Judge of Branch XXIII, Court of First Instance of
Manila; ANTONIO J. VILLEGAS, in his capacity as Mayor of the City of Manila; and the CITY OF
MANILA, respondents-appellees.

Office of the Solicitor General Felix V. Makasiar, Assistant Solicitor-General Antonio A. Torres,
Solicitor Raul I. Goco and Magno B. Pablo & Cipriano A. Tan, Legal Staff, Land Authority for
petitioners-appellants.

Gregorio A. Ejercito and Felix C. Chavez for respondents-appellees.

ESGUERRA, J.:p

This is a petition for review of the decision of the Court of First Instance of Manila, Branch XXIII, in
Civil Case No. 67946, dated September 23, 1968, the dispositive portion of which is as follows:

WHEREFORE, the Court renders judgment declaring Republic Act No. 4118
unconstitutional and invalid in that it deprived the City of Manila of its property without
due process and payment of just compensation. Respondent Executive Secretary
and Governor of the Land Authority are hereby restrained and enjoined from
implementing the provisions of said law. Respondent Register of Deeds of the City of
Manila is ordered to cancel Transfer Certificate of Title No. 80876 which he had
issued in the name of the Land Tenure Administration and reinstate Transfer
Certificate of Title No. 22547 in the name of the City of Manila which he cancelled, if
that is feasible, or issue a new certificate of title for the same parcel of land in the
name of the City of Manila.1

The facts necessary for a clear understanding of this case are as follows:

On February 24, 1919, the 4th Branch of the Court of First Instance of Manila, acting as a land
registration court, rendered judgment in Case No. 18, G.L.R.O. Record No. 111, declaring the City of
Manila the owner in fee simple of a parcel of land known as Lot No. 1, Block 557 of the Cadastral
Survey of the City of Mani1a, containing an area of 9,689.8 square meters, more or less. Pursuant to
said judgment the Register of Deeds of Manila on August 21, 1920, issued in favor of the City of
Manila, Original Certificate of Title No. 4329 covering the aforementioned parcel of land. On various
dates in 1924, the City of Manila sold portions of the aforementioned parcel of land in favor of Pura
Villanueva. As a consequence of the transactions Original Certificate of Title No. 4329 was cancelled
and transfer certificates of title were issued in favor of Pura Villanueva for the portions purchased by
her. When the last sale to Pura Villanueva was effected on August 22, 1924, Transfer Certificate of
321

Title No. 21974 in the name of the City of Manila was cancelled and in lieu thereof Transfer Certificate
of Title (TCT) No. 22547 covering the residue thereof known as Lot 1-B-2-B of Block 557, with an
area of 7,490.10 square meters, was issued in the name of the City of Manila.

On September 21, 1960, the Municipal Board of Manila, presided by then Vice-Mayor Antono J.
Villegas, adopted a resolution requesting His Excellency, the President of the Philippines to consider
the feasibility of declaring the City property bounded by Florida, San Andres, and Nebraska Streets,
under Transfer Certificate of Title Nos. 25545 and 22547, containing a total area of 7,450 square
meters as a patrimonial property of the City of Manila for the purpose of reselling these lots to the
actual occupants thereof.2

The said resolution of the Municipil Board of the City of Manila was officially transmitted to the
President of the Philippines by then Vice-Mayor Antonio J. Villegas on September 21, 1960, with the
information that the same resolution was, on the same date, transmitted to the Senate and House of
Representatives of the Congress of the Philippines.3

During the First Session of the Fifth Congress of the Philippines, House Bill No. 191 was filed in the
House of Representatives by then Congressman Bartolome Cabangbang seeking to declare the
property in question as patrimonial property of the City of Manila, and for other purposes. The
explanatory note of the Bill gave the grounds for its enactment, to wit:

In the particular case of the property subject of this bill, the City of Manila does not
seem to have use thereof as a public communal property. As a matter of fact, a
resolution was adopted by the Municipal Board of Manila at its regular session held
on September 21, 1960, to request the feasibility of declaring the city property
bounded by Florida, San Andres and Nebraska Streets as a patrimonial property of
the City of Manila for the purpose of reselling these lots to the actual occupants
thereof. Therefore, it will be to the best interest of society that the said property be
used in one way or another. Since this property has been occupied for a long time by
the present occupants thereof and since said occupants have expressed their
willingness to buy the said property, it is but proper that the same be sold to them.4

Subsequently, a revised version of the Bill was introduced in the House of Representatives by
Congressmen Manuel Cases, Antonio Raquiza and Nicanor Yñiguez as House Bill No. 1453, with the
following explanatory note:

The accompanying bill seeks to convert one (1) parcel of land in the district of
Malate, which is reserved as communal property into a disposable or alienable
property of the State and to provide its subdivision and sale to bona fide occupants or
tenants.

This parcel of land in question was originally an aggregate part of a piece of land with
an area of 9,689.8 square meters, more or less. ... On September 21, 1960, the
Municipal Board of Manila in its regular session unanimously adopted a resolution
requesting the President of the Philippines and Congress of the Philippines the
feasibility of declaring this property into disposable or alienable property of the State.
There is therefore a precedent that this parcel of land could be subdivided and sold to
bona fide occupants. This parcel of land will not serve any useful public project
because it is bounded on all sides by private properties which were formerly parts of
this lot in question.

Approval of this bill will implement the policy of the Administration of land for the
landless and the Fifth Declaration of Principles of the Constitution, which states that
the promotion of Social Justice to insure the well-being and economic security of all
people should be the concern of the State. We are ready and willing to enact
legislation promoting the social and economic well-being of the people whenever an
opportunity for enacting such kind of legislation arises.
322

In view of the foregoing consideration and to insure fairness and justice to the present bona fide
occupants thereof, approval of this Bill is strongly urged. 5

The Bill having been passed by the House of Representatives, the same was thereafter sent to the
Senate where it was thoroughly discussed, as evidenced by the Congressional Records for May 20,
1964, pertinent portion of which is as follows:

SENATOR FERNANDEZ: Mr. President, it will be re called that when the late Mayor
Lacson was still alive, we approved a similar bill. But afterwards, the late Mayor
Lacson came here and protested against the approval, and the approval was
reconsidered. May I know whether the defect in the bill which we approved, has
already been eliminated in this present bill?

SENATOR TOLENTINO: I understand Mr. President, that that has already been
eliminated and that is why the City of Manila has no more objection to this bill.

SENATOR FERNANDEZ: Mr. President, in view of that manifestation and


considering that Mayor Villegas and Congressman Albert of the Fourth District of
Manila are in favor of the bill. I would not want to pretend to know more what is good
for the City of Manila.

SENATOR TOLENTINO: Mr. President, there being no objection, I move that we


approve this bill on second reading.

PRESIDENT PRO-TEMPORE: The biII is approved on second reading after several


Senetors said aye and nobody said nay.

The bill was passed by the Senate, approved by the President on June 20, 1964, and became
Republic Act No. 4118. It reads as follows:

Lot I-B-2-B of Block 557 of the cadastral survey of the City of Manila, situated in the
District of Malate, City of Manila, which is reserved as communal property, is hereby
converted into disposal or alienable land of the State, to be placed under the disposal
of the Land Tenure Administration. The Land Tenure Administration shall subdivide
the property into small lots, none of which shall exceed one hundred and twenty
square meters in area and sell the same on installment basis to the tenants or bona
fide occupants thereof and to individuals, in the order mentioned: Provided, That no
down payment shall be required of tenants or bona fide occupants who cannot afford
to pay such down payment: Provided, further, That no person can purchase more
than one lot: Provided, furthermore, That if the tenant or bona fide occupant of any
given lot is not able to purchase the same, he shall be given a lease from month to
month until such time that he is able to purchase the lot: Provided, still further, That in
the event of lease the rentals which may be charged shall not exceed eight per cent
per annum of the assessed value of the property leased: And provided, finally, That in
fixing the price of each lot, which shall not exceed twenty pesos per square meter,
the cost of subdivision and survey shall not be included.

Sec. 2. Upon approval of this Act no ejectment proceedings against any tenant or
bona fide occupant of the above lots shall be instituted and any ejectment
proceedings pending in court against any such tenant or bona fide occupant shall be
dismissed upon motion of the defendant: Provided, That any demolition order
directed against any tenant or bona fide occupant shall be lifted.

Sec. 3. Upon approval of this Act, if the tenant or bona fide occupant is in arrears in
the payment of any rentals, the amount legally due shall be liquidated and shall be
payable in twenty-four equal monthly installments from the date of liquidation.
323

Sec. 4. No property acquired by virtue of this Act shall be transferred, sold,


mortgaged, or otherwise disposed of within a period of five years from the date full
ownership thereof has been vested in the purchaser without the consent of the Land
Tenure Administration.

Sec. 5. In the event of the death of the purchaser prior to the complete payment of
the price of the lot purchased by him, his widow and children shall succeed in all his
rights and obligations with respect to his lot.

Sec. 6. The Chairman of the Land Tenure Administration shall implement and issue
such rules and regulations as may be necessary to carry out the provisions of this
Act.

Sec. 7. The sum of one hundred fifty thousand pesos is appropriated out of any funds
in the National Treasury not otherwise appropriated, to carry out the purposes of this
Act.

Sec. 8. All laws or parts of laws inconsistent with this Act are repealed or modified
accordingly.

Sec. 9. This Act shall take effect upon its approval.

Approved, June 20, 1964.

To implement the provisions of Republic Act No. 4118, and pursuant to the request of the occupants
of the property involved, then Deputy Governor Jose V. Yap of the Land Authority (which succeeded
the Land Tenure Administration) addressed a letter, dated February 18, 1965, to Mayor Antonio
Villegas, furnishing him with a copy of the proposed subdivision plan of said lot as prepared for the
Republic of the Philippines for resale of the subdivision lots by the Land Authority to bona fide
applicants.6

On March 2, 1965, the City Mayor of Manila, through his Executive and Technical Adviser,
acknowledged receipt of the proposed subdivision plan of the property in question and informed the
Land Authority that his office would interpose no objection to the implementation of said law, provided
that its provisions be strictly complied with.7

With the above-mentioned written conformity of the City of Manila for the implementation of Republic
Act No. 4118, the Land Authority, thru then Deputy Governor Jose V. Yap, requested the City
Treasurer of Manila, thru the City Mayor, for the surrender and delivery to the former of the owner's
duplicate of Transfer Certificate of Title No. 22547 in order to obtain title thereto in the name of the
Land Authority. The request was duly granted with the knowledge and consent of the Office of the
City Mayor.8

With the presentation of Transfer Certificate of Title No. 22547, which had been yielded as above
stated by the, City authorities to the Land Authority, Transfer Certificate of Title (T.C.T. No. 22547)
was cancelled by the Register of Deeds of Manila and in lieu thereof Transfer Certificate of Title No.
80876 was issued in the name of the Land Tenure Administration (now Land Authority) pursuant to
the provisions of Republic Act No.
4118.9

But due to reasons which do not appear in the record, the City of Manila made a complete turn-about,
for on December 20, 1966, Antonio J. Villegas, in his capacity as the City Mayor of Manila and the
City of Manila as a duly organized public corporation, brought an action for injunction and/or
prohibition with preliminary injunction to restrain, prohibit and enjoin the herein appellants, particularly
the Governor of the Land Authority and the Register of Deeds of Manila, from further implementing
Republic Act No. 4118, and praying for the declaration of Republic Act No. 4118 as unconstitutional.
324

With the foregoing antecedent facts, which are all contained in the partial stipulation of facts submitted
to the trial court and approved by respondent Judge, the parties waived the presentation of further
evidence and submitted the case for decision. On September 23, 1968, judgment was rendered by
the trial court declaring Republic Act No. 4118 unconstitutional and invalid on the ground that it
deprived the City of Manila of its property without due process of law and payment of just
compensation. The respondents were ordered to undo all that had been done to carry out the
provisions of said Act and were restrained from further implementing the same.

Two issues are presented for determination, on the resolution of which the decision in this case
hinges, to wit:

I. Is the property involved private or patrimonial property of the City of Manila?

II. Is Republic Act No. 4118 valid and not repugnant to the Constitution?

I.

As regards the first issue, appellants maintain that the land involved is a communal land or "legua
comunal" which is a portion of the public domain owned by the State; that it came into existence as
such when the City of Manila, or any pueblo or town in the Philippines for that matter, was founded
under the laws of Spain, the former sovereign; that upon the establishment of a pueblo, the
administrative authority was required to allot and set aside portions of the public domain for a public
plaza, a church site, a site for public buildings, lands to serve as common pastures and for streets and
roads; that in assigning these lands some lots were earmarked for strictly public purposes, and
ownership of these lots (for public purposes) immediately passed to the new municipality; that in the
case of common lands or "legua comunal", there was no such immediate acquisition of ownership by
the pueblo, and the land though administered thereby, did not automatically become its property in
the absence of an express grant from the Central Government, and that the reason for this
arrangement is that this class of land was not absolutely needed for the discharge of the
municipality's governmental functions.

It is argued that the parcel of land involved herein has not been used by the City of Manila for any
public purpose and had not been officially earmarked as a site for the erection of some public
buildings; that this circumstance confirms the fact that it was originally "communal" land alloted to the
City of Manila by the Central Government not because it was needed in connection with its
organization as a municipality but simply for the common use of its inhabitants; that the present City
of Manila as successor of the Ayuntamiento de Manila under the former Spanish sovereign merely
enjoys the usufruct over said land, and its exercise of acts of ownership by selling parts thereof did
not necessarily convert the land into a patrimonial property of the City of Manila nor divest the State of
its paramount title.

Appellants further argue that a municipal corporation, like a city is a governmental agent of the State
with authority to govern a limited portion of its territory or to administer purely local affairs in a given
political subdivision, and the extent of its authority is strictly delimited by the grant of power conferred
by the State; that Congress has the exclusive power to create, change or destroy municipal
corporations; that even if We admit that legislative control over municipal corporations is not absolute
and even if it is true that the City of Manila has a registered title over the property in question, the
mere transfer of such land by an act of the legislature from one class of public land to another, without
compensation, does not invade the vested rights of the City.

Appellants finally argue that Republic Act No. 4118 has treated the land involved as one reserved for
communal use, and this classification is conclusive upon the courts; that if the City of Manila feels that
this is wrong and its interests have been thereby prejudiced, the matter should be brought to the
attention of Congress for correction; and that since Congress, in the exercise of its wide discretionary
powers has seen fit to classify the land in question as communal, the Courts certainly owe it to a
coordinate branch of the Government to respect such determination and should not interfere with the
enforcement of the law.
325

Upon the other hand, appellees argue by simply quoting portions of the appealed decision of the trial
court, which read thus:

The respondents (petitioners-appellants herein) contend, among other defenses, that


the property in question is communal property. This contention is, however, disproved
by Original Certificate of Title No. 4329 issued on August 21, 1920 in favor of the City
of Manila after the land in question was registered in the City's favor. The Torrens
Title expressly states that the City of Manila was the owner in 'fee simple' of the said
land. Under Sec. 38 of the Land Registration Act, as amended, the decree of
confirmation and registration in favor of the City of Manila ... shall be conclusive upon
and against all persons including the Insular Government and all the branches there
... There is nothing in the said certificate of title indicating that the land was
'communal' land as contended by the respondents. The erroneous assumption by the
Municipal Board of Manila that the land in question was communal land did not make
it so. The Municipal Board had no authority to do that.

The respondents, however, contend that Congress had the power and authority to
declare that the land in question was 'communal' land and the courts have no power
or authority to make a contrary finding. This contention is not entirely correct or
accurate. Congress has the power to classify 'land of the public domain', transfer
them from one classification to another and declare them disposable or not. Such
power does not, however, extend to properties which are owned by cities, provinces
and municipalities in their 'patrimonial' capacity.

Art. 324 of the Civil Code provides that properties of provinces, cities and
municipalities are divided into properties for public use and patrimonial property. Art.
424 of the same code provides that properties for public use consist of provincial
roads, city streets, municipal streets, the squares, fountains, public waters,
promenades and public works for public service paid for by said province, cities or
municipalities. All other property possessed by any of them is patrimonial. Tested by
this criterion the Court finds and holds that the land in question is patrimonial property
of the City of Manila.

Respondents contend that Congress has declared the land in question to be


'communal' and, therefore, such designation is conclusive upon the courts. The
Courts holds otherwise. When a statute is assailed as unconstitutional the Courts
have the power and authority to inquire into the question and pass upon it. This has
long ago been settled in Marbury vs. Madison, 2 L. ed. 60, when the United States
Supreme Court speaking thru Chief Justice Marshall held:

... If an act of the legislature, repugnant to the constitution, is void,


does it, notwithstanding its validity, bind the courts, and oblige them
to give effect? It is emphatically the province and duty of the judicial
department to say what the law is ... So if a law be in opposition to
the constitution; if both the law and the constitution apply to a
particular case, so that the court must either decide that case
conformable to the constitution, disregarding the law, the court must
determine which of these conflicting rules governs the case. This is
of the very essence of unconstitutional judicial duty.

Appellees finally concluded that when the courts declare a law unconstitutional it does not mean that
the judicial power is superior to the legislative power. It simply means that the power of the people is
superior to both and that when the will of the legislature, declared in statutes, stands in opposition to
that of the people, declared in the Constitution, the judges ought to be governed by the Constitution
rather than by the statutes.

There is one outstanding factor that should be borne in mind in resolving the character of the land
involved, and it is that the City of Manila, although declared by the Cadastral Court as owner in fee
simple, has not shown by any shred of evidence in what manner it acquired said land as its private or
326

patrimonial property. It is true that the City of Manila as well as its predecessor, the Ayuntamiento de
Manila, could validly acquire property in its corporate or private capacity, following the accepted
doctrine on the dual character — public and private — of a municipal corporation. And when it
acquires property in its private capacity, it acts like an ordinary person capable of entering into
contracts or making transactions for the transmission of title or other real rights. When it comes to
acquisition of land, it must have done so under any of the modes established by law for the
acquisition of ownership and other real rights. In the absence of a title deed to any land claimed by
the City of Manila as its own, showing that it was acquired with its private or corporate funds, the
presumption is that such land came from the State upon the creation of the municipality (Unson vs.
Lacson, et al., 100 Phil. 695). Originally the municipality owned no patrimonial property except those
that were granted by the State not for its public but for private use. Other properties it owns are
acquired in the course of the exercise of its corporate powers as a juridical entity to which category a
municipal corporation pertains.

Communal lands or "legua comunal" came into existence when a town or pueblo was established in
this country under the laws of Spain (Law VII, Title III, Book VI, Recopilacion de las Leyes de Indios).
The municipalities of the Philippines were not entitled, as a matter of right, to any part of the public
domain for use as communal lands. The Spanish law provided that the usufruct of a portion of the
public domain adjoining municipal territory might be granted by the Government for communal
purposes, upon proper petition, but, until granted, no rights therein passed to the municipalities, and,
in any event, the ultimate title remained in the sovereign (City of Manila vs. Insular Government, 10
Phil. 327).

For the establishment, then, of new pueblos the administrative authority of the
province, in representation of the Governor General, designated the territory for their
location and extension and the metes and bounds of the same; and before alloting
the lands among the new settlers, a special demarcation was made of the places
which were to serve as the public square of the pueblo, for the erection of the church,
and as sites for the public buildings, among others, the municipal building or the casa
real, as well as of the lands whick were to constitute the common pastures, and
propios of the municipality and the streets and roads which were to intersect the new
town were laid out, ... . (Municipality of Catbalogan vs. Director of Lands, 17 Phil.
216, 220) (Emphasis supplied)

It may, therefore, be laid down as a general rule that regardless of the source or classification of land
in the possession of a municipality, excepting those acquired with its own funds in its private or
corporate capacity, such property is held in trust for the State for the benefit of its inhabitants, whether
it be for governmental or proprietary purposes. It holds such lands subject to the paramount power of
the legislature to dispose of the same, for after all it owes its creation to it as an agent for the
performance of a part of its public work, the municipality being but a subdivision or instrumentality
thereof for purposes of local administration. Accordingly, the legal situation is the same as if the State
itself holds the property and puts it to a different use (2 McQuilin,Municipal Corporations, 3rd Ed., p.
197, citing Monagham vs. Armatage, 218 Minn. 27, 15 N. W. 2nd 241).

True it is that the legislative control over a municipal corporation is not absolute even when it comes
to its property devoted to public use, for such control must not be exercised to the extent of depriving
persons of their property or rights without due process of law, or in a manner impairing the obligations
of contracts. Nevertheless, when it comes to property of the municipality which it did not acquire in its
private or corporate capacity with its own funds, the legislature can transfer its administration and
disposition to an agency of the National Government to be disposed of according to its discretion.
Here it did so in obedience to the constitutional mandate of promoting social justice to insure the well-
being and economic security of the people.

It has been held that a statute authorizing the transfer of a Municipal airport to an Airport Commission
created by the legislature, even without compensation to the city, was not violative of the due process
clause of the American Federal Constitution. The Supreme Court of Minnessota in Monagham vs.
Armatage, supra, said:
327

... The case is controlled by the further rule that the legislature, having plenary control
of the local municipality, of its creation and of all its affairs, has the right to authorize
or direct the expenditures of money in its treasury, though raised, for a particular
purpose, for any legitimate municipal purpose, or to order and direct a distribution
thereof upon a division of the territory into separate municipalities ... . The local
municipality has no such vested right in or to its public funds, like that which the
Constitution protects in the individual as precludes legislative interferences. People
vs. Power, 25 Ill. 187; State Board (of Education) vs. City, 56 Miss. 518. As remarked
by the supreme court of Maryland in Mayor vs. Sehner, 37 Md. 180: "It is of the
essence of such a corporation, that the government has the sole right as trustee of
the public interest, at its own good will and pleasure, to inspect, regulate, control, and
direct the corporation, its funds, and franchises."

We therefore hold that c.500, in authorizing the transfer of the use and possession of
the municipal airport to the commission without compensation to the city or to the
park board, does not violate the Fourteenth Amendment to the Constitution of the
United States.

The Congress has dealt with the land involved as one reserved for communal use (terreno comunal).
The act of classifying State property calls for the exercise of wide discretionary legislative power and it
should not be interfered with by the courts.

This brings Us to the second question as regards the validity of Republic Act No. 4118, viewed in the
light of Article III, Sections 1, subsection (1) and (2) of the Constitution which ordain that no person
shall be deprived of his property without due process of law and that no private property shall be
taken for public use without just compensation.

II .

The trial court declared Republic Act No. 4118 unconstitutional for allegedly depriving the City of
Manila of its property without due process of law and without payment of just compensation. It is now
well established that the presumption is always in favor of the constitutionality of a law (U S. vs. Ten
Yu, 24 Phil. 1; Go Ching, et al. vs. Dinglasan, et al., 45 O.G. No. 2, pp. 703, 705). To declare a law
unconstitutional, the repugnancy of that law to the Constitution must be clear and unequivocal, for
even if a law is aimed at the attainment of some public good, no infringement of constitutional rights is
allowed. To strike down a law there must be a clear showing that what the fundamental law
condemns or prohibits, the statute allows it to be done (Morfe vs. Mutuc, et al., G.R. No. L-20387,
Jan. 31, 1968; 22 SCRA 424). That situation does not obtain in this case as the law assailed does not
in any manner trench upon the constitution as will hereafter be shown. Republic Act No. 4118 was
intended to implement the social justice policy of the Constitution and the Government program of
"Land for the Landless". The explanatory note of House Bill No. 1453 which became Republic Act No.
4118, reads in part as follows:

Approval of this bill will implement the policy of the administration of "land for the
landless" and the Fifth Declaration of Principles of the Constitution which states that
"the promotion of social justice to insure the well-being and economic security of all
people should be the concern of the State." We are ready and willing to enact
legislation promoting the social and economic well-being of the people whenever an
opportunity for enacting such kind of legislation arises.

The respondent Court held that Republic Act No. 4118, "by converting the land in question — which is
the patrimonial property of the City of Manila into disposable alienable land of the State and placing it
under the disposal of the Land Tenure Administration — violates the provisions of Article III (Secs. 1
and 2) of the Constitution which ordain that "private property shall not be taken for public use without
just compensation, and that no person shall be deprived of life, liberty or property without due process
of law". In support thereof reliance is placed on the ruling in Province of Zamboanga del Norte vs. City
of Zamboanga, G.R. No. 2440, March 28, 1968; 22 SCRA 1334, which holds that Congress cannot
deprive a municipality of its private or patrimonial property without due process of law and without
payment of just compensation since it has no absolute control thereof. There is no quarrel over this
328

rule if it is undisputed that the property sought to be taken is in reality a private or patrimonial property
of the municipality or city. But it would be simply begging the question to classify the land in question
as such. The property, as has been previously shown, was not acquired by the City of Manila with its
own funds in its private or proprietary capacity. That it has in its name a registered title is not
questioned, but this title should be deemed to be held in trust for the State as the land covered
thereby was part of the territory of the City of Manila granted by the sovereign upon its creation. That
the National Government, through the Director of Lands, represented by the Solicitor General, in the
cadastral proceedings did not contest the claim of the City of Manila that the land is its property, does
not detract from its character as State property and in no way divests the legislature of its power to
deal with it as such, the state not being bound by the mistakes and/or negligence of its officers.

One decisive fact that should be noted is that the City of Manila expressly recognized the paramount
title of the State over said land when by its resolution of September 20, 1960, the Municipal Board,
presided by then Vice-Mayor Antonio Villegas, requested "His Excellency the President of the
Philippines to consider the feasibility of declaring the city property bounded by Florida, San Andres
and Nebraska Streets, under Transfer Certificate of Title Nos. 25545 and 25547, containing an area
of 7,450 square meters, as patrimonial property of the City of Manila for the purpose of reselling these
lots to the actual occupants thereof." (See Annex E, Partial Stipulation of Facts, Civil Case No. 67945,
CFI, Manila, p. 121, Record of the Case) [Emphasis Supplied]

The alleged patrimonial character of the land under the ownership of the City of Manila is totally belied
by the City's own official act, which is fatal to its claim since the Congress did not do as bidden. If it
were its patrimonial property why should the City of Manila be requesting the President to make
representation to the legislature to declare it as such so it can be disposed of in favor of the actual
occupants? There could be no more blatant recognition of the fact that said land belongs to the State
and was simply granted in usufruct to the City of Manila for municipal purposes. But since the City did
not actually use said land for any recognized public purpose and allowed it to remain idle and
unoccupied for a long time until it was overrun by squatters, no presumption of State grant of
ownership in favor of the City of Manila may be acquiesced in to justify the claim that it is its own
private or patrimonial property (Municipality of Tigbauan vs. Director of Lands, 35 Phil. 798; City of
Manila vs. Insular Government, 10 Phil. 327; Municipality of Luzuriaga vs. Director of Lands, 24 Phil.
193). The conclusion of the respondent court that Republic Act No. 4118 converted a patrimonial
property of the City of Manila into a parcel of disposable land of the State and took it away from the
City without compensation is, therefore, unfounded. In the last analysis the land in question pertains
to the State and the City of Manila merely acted as trustee for the benefit of the people therein for
whom the State can legislate in the exercise of its legitimate powers.

Republic Act No. 4118 was never intended to expropriate the property involved but merely to confirm
its character as communal land of the State and to make it available for disposition by the National
Government: And this was done at the instance or upon the request of the City of Manila itself. The
subdivision of the land and conveyance of the resulting subdivision lots to the occupants by
Congressional authorization does not operate as an exercise of the power of eminent domain without
just compensation in violation of Section 1, subsection (2), Article III of the Constitution, but simply as
a manifestation of its right and power to deal with state property.

It should be emphasized that the law assailed was enacted upon formal written petition of the
Municipal Board of Manila in the form of a legally approved resolution. The certificate of title over the
property in the name of the City of Manila was accordingly cancelled and another issued to the Land
Tenure Administration after the voluntary surrender of the City's duplicate certificate of title by the City
Treasurer with the knowledge and consent of the City Mayor. To implement the provisions of Republic
Act No. 4118, the then Deputy Governor of the Land Authority sent a letter, dated February 18, 1965,
to the City Mayor furnishing him with a copy of the "proposed subdivision plan of the said lot as
prepared for the Republic of the Philippines for subdivision and resale by the Land Authority to bona
fide applicants." On March 2, 1965, the Mayor of Manila, through his Executive and Technical
Adviser, acknowledged receipt of the subdivision plan and informed the Land Authority that his Office
"will interpose no objection to the implementation of said law provided that its provisions are strictly
complied with." The foregoing sequence of events, clearly indicate a pattern of regularity and
observance of due process in the reversion of the property to the National Government. All such acts
329

were done in recognition by the City of Manila of the right and power of the Congress to dispose of
the land involved.

Consequently, the City of Manila was not deprived of anything it owns, either under the due process
clause or under the eminent domain provisions of the Constitution. If it failed to get from the Congress
the concession it sought of having the land involved given to it as its patrimonial property, the Courts
possess no power to grant that relief. Republic Act No. 4118 does not, therefore, suffer from any
constitutional infirmity.

WHEREFORE, the appealed decision is hereby reversed, and petitioners shall proceed with the free
and untrammeled implementation of Republic Act No. 4118 without any obstacle from the
respondents. Without costs.

Concepcion, C.J., Makalintal, Zaldivar, Castro, Fernando, Teehankee and Antonio, JJ., concur.

Barredo and Makasiar, JJ., took no part.

Footnotes

1 Pages 79-80, Rollo.

2 Annex "E" to the Partial Stipulation of Facts, page 121, Records.

3 Annex "E-1" to the Partial Stipulation of Facts, page 122, Records.

4 Annex "F" to the Partial Stipulation of Facts, page l23, Records.

5 Annex "F-1", page 128, Records.

6 Annex "J", page 142, Records.

7 Annex "K", page 145, Records.

8 Annexes "L" and "L-1", pages 145-147, Records.

9 Annexes "A" and "N", pages 148-150, Records.


330

MUNICIPALITY OF SAN MIGUEL v FERNANDEZ (G.R. No. L-61744, June 25, 1984)

Republic of the Philippines


SUPREME COURT
Manila

FIRST DIVISION

G.R. No. L-61744 June 25, 1984

MUNICIPALITY OF SAN MIGUEL, BULACAN, petitioner,


vs.
HONORABLE OSCAR C. FERNANDEZ, in his capacity as the Presiding Judge, Branch IV,
Baliuag, Bulacan, The PROVINCIAL SHERIFF of Bulacan, MARGARITA D. VDA. DE IMPERIO,
ADORACION IMPERIO, RODOLFO IMPERIO, CONRADO IMPERIO, ERNESTO IMPERIO,
ALFREDO IMPERIO, CARLOS IMPERIO, JR., JUAN IMPERIO and SPOUSES MARCELO PINEDA
and LUCILA PONGCO, respondents.

Pascual C. Liatchko for petitioner.

The Solicitor General and Marcelo Pineda for respondents.

RELOVA, J.:

In Civil Case No. 604-B, entitled "Margarita D. Vda. de Imperio, et al. vs. Municipal Government of
San Miguel, Bulacan, et al.", the then Court of First Instance of Bulacan, on April 28, 1978, rendered
judgment holding herein petitioner municipality liable to private respondents, as follows:

WHEREFORE, premises considered, judgment is hereby rendered in favor of the


plaintiffs and against the defendant Municipal Government of San Miguel Bulacan,
represented by Mayor Mar Marcelo G. Aure and its Municipal Treasurer:

1. ordering the partial revocation of the Deed of Donation signed by the deceased
Carlos Imperio in favor of the Municipality of San Miguel Bulacan, dated October 27,
1947 insofar as Lots Nos. 1, 2, 3, 4 and 5, Block 11 of Subdivision Plan Psd-20831
are concerned, with an aggregate total area of 4,646 square meters, which lots are
among those covered and described under TCT No. T-1831 of the Register of Deeds
of Bulacan in the name of the Municipal Government of San Miguel Bulacan,

2. ordering the defendant to execute the corresponding Deed of Reconveyance over


the aforementioned five lots in favor of the plaintiffs in the proportion of the undivided
one-half (½) share in the name of plaintiffs Margarita D. Vda. de Imperio, Adoracion,
Rodolfo, Conrado, Ernesto, Alfredo, Carlos, Jr. and Juan, all surnamed Imperio, and
the remaining undivided one-half (½) share in favor of plaintiffs uses Marcelo E.
Pineda and Lucila Pongco;

3. ordering the defendant municipality to pay to the plaintiffs in the proportion


mentioned in the immediately preceding paragraph the sum of P64,440.00
corresponding to the rentals it has collected from the occupants for their use and
occupation of the premises from 1970 up to and including 1975, plus interest thereon
at the legal rate from January 1970 until fully paid;

4. ordering the restoration of ownership and possession over the five lots in question
in favor of the plaintiffs in the same proportion aforementioned;
331

5. ordering the defendant to pay the plaintiffs the sum of P3,000.00 for attomey's
fees; and to pay the cost of suit.

The counterclaim of the defendant is hereby ordered dismissed for lack of evidence
presented to substantiate the same.

SO ORDERED. (pp. 11-12, Rollo)

The foregoing judgment became final when herein petitioner's appeal was dismissed due to its failure
to file the record on appeal on time. The dismissal was affirmed by the then Court of Appeals in CA-
G.R. No. SP-12118 and by this Court in G.R. No. 59938. Thereafter, herein private respondents
moved for issuance of a writ of execution for the satisfaction of the judgment. Respondent judge, on
July 27, 1982, issued an order, to wit:

Considering that an entry of judgment had already been made on June 14, 1982 in G.
R. No. L-59938 and;

Considering further that there is no opposition to plaintiffs' motion for execution dated
July 23, 1983;

Let a writ of execution be so issued, as prayed for in the aforestated motion. (p. 10,
Rollo)

Petitioner, on July 30, 1982, filed a Motion to Quash the writ of execution on the ground that the
municipality's property or funds are all public funds exempt from execution. The said motion to quash
was, however, denied by the respondent judge in an order dated August 23, 1982 and the alias writ of
execution stands in full force and effect.

On September 13, 1982, respondent judge issued an order which in part, states:

It is clear and evident from the foregoing that defendant has more than enough funds
to meet its judgment obligation. Municipal Treasurer Miguel C, Roura of San Miguel,
Bulacan and Provincial Treasurer of Bulacan Agustin O. Talavera are therefor hereby
ordered to comply with the money judgment rendered by Judge Agustin C. Bagasao
against said municipality. In like manner, the municipal authorities of San Miguel,
Bulacan are likewise ordered to desist from plaintiffs' legal possession of the property
already returned to plaintiffs by virtue of the alias writ of execution.

Finally, defendants are hereby given an inextendible period of ten (10) days from
receipt of a copy of this order by the Office of the Provincial Fiscal of Bulacan within
which to submit their written compliance, (p. 24, Rollo)

When the treasurers (provincial and municipal) failed to comply with the order of September 13, 1982,
respondent judge issued an order for their arrest and that they will be release only upon compliance
thereof.

Hence, the present petition on the issue whether the funds of the Municipality of San Miguel, Bulacan,
in the hands of the provincial and municipal treasurers of Bulacan and San Miguel, respectively, are
public funds which are exempt from execution for the satisfaction of the money judgment in Civil Case
No. 604-B.

Well settled is the rule that public funds are not subject to levy and execution. The reason for this was
explained in the case of Municipality of Paoay vs. Manaois, 86 Phil. 629 "that they are held in trust for
the people, intended and used for the accomplishment of the purposes for which municipal
corporations are created, and that to subject said properties and public funds to execution would
materially impede, even defeat and in some instances destroy said purpose." And, in Tantoco vs.
Municipal Council of Iloilo, 49 Phil. 52, it was held that "it is the settled doctrine of the law that not only
332

the public property but also the taxes and public revenues of such corporations Cannot be seized
under execution against them, either in the treasury or when in transit to it. Judgments rendered for
taxes, and the proceeds of such judgments in the hands of officers of the law, are not subject to
execution unless so declared by statute." Thus, it is clear that all the funds of petitioner municipality in
the possession of the Municipal Treasurer of San Miguel, as well as those in the possession of the
Provincial Treasurer of Bulacan, are also public funds and as such they are exempt from execution.

Besides, Presidential Decree No. 477, known as "The Decree on Local Fiscal Administration", Section
2 (a), provides:

SEC. 2. Fundamental Principles. — Local government financial affairs, transactions,


and operations shall be governed by the fundamental principles set forth hereunder:

(a) No money shall be paid out of the treasury except in pursuance of a lawful
appropriation or other specific statutory authority.

xxx xxx xxx

Otherwise stated, there must be a corresponding appropriation in the form of an ordinance duly
passed by the Sangguniang Bayan before any money of the municipality may be paid out. In the case
at bar, it has not been shown that the Sangguniang Bayan has passed an ordinance to this effect.

Furthermore, Section 15, Rule 39 of the New Rules of Court, outlines the procedure for the
enforcement of money judgment:

(a) By levying on all the property of the debtor, whether real or personal, not
otherwise exempt from execution, or only on such part of the property as is sufficient
to satisfy the judgment and accruing cost, if he has more than sufficient property for
the purpose;

(b) By selling the property levied upon;

(c) By paying the judgment-creditor so much of the proceeds as will satisfy the
judgment and accruing costs; and

(d) By delivering to the judgment-debtor the excess, if any, unless otherwise, directed
by judgment or order of the court.

The foregoing has not been followed in the case at bar.

ACCORDINGLY, the petition is granted and the order of respondent judge, dated July 27, 1982,
granting issuance of a writ of execution; the alias writ of execution, dated July 27, 1982; and the order
of respondent judge, dated September 13, 1982, directing the Provincial Treasurer of Bulacan and the
Municipal Treasurer of San Miguel, Bulacan to comply with the money judgments, are SET ASIDE;
and respondents are hereby enjoined from implementing the writ of execution.

SO ORDERED.

Teehankee (Chairman), Melencio-Herrera, Plana, Gutierrez, Jr., and De la Fuente, JJ,. concur.
333

ZAMBOANGA DEL NORTE v CITY OF ZAMBOANGA (G.R. No. L-24440, March 28, 1968)

Republic of the Philippines


SUPREME COURT
Manila

EN BANC

G.R. No. L-24440 March 28, 1968

THE PROVINCE OF ZAMBOANGA DEL NORTE, plaintiff-appellee,


vs.
CITY OF ZAMBOANGA, SECRETARY OF FINANCE and COMMISSIONER OF INTERNAL
REVENUE,defendants-appellants.

Fortugaleza, Lood, Sarmiento, M. T. Yap & Associates for plaintiff-appellee.


Office of the Solicitor General for defendants-appellants.

BENGZON, J.P., J.:

Prior to its incorporation as a chartered city, the Municipality of Zamboanga used to be the
provincial capital of the then Zamboanga Province. On October 12, 1936, Commonwealth Act 39 was
approved converting the Municipality of Zamboanga into Zamboanga City. Sec. 50 of the Act also
provided that —

Buildings and properties which the province shall abandon upon the transfer of the
capital to another place will be acquired and paid for by the City of Zamboanga at a price to
be fixed by the Auditor General.

The properties and buildings referred to consisted of 50 lots and some buildings constructed
thereon, located in the City of Zamboanga and covered individually by Torrens certificates of title in
the name of Zamboanga Province. As far as can be gleaned from the records, 1 said properties were
being utilized as follows —

No. of Lots Use


1 ................................................ Capitol Site
3 ................................................ School Site
3 ................................................ Hospital Site
3 ................................................ Leprosarium
1 ................................................ Curuan School
1 ................................................ Trade School
2 ................................................ Burleigh School
2 ................................................ High School Playground
9 ................................................ Burleighs
1 ................................................ Hydro-Electric Site (Magay)
1 ................................................ San Roque
23 ................................................ vacant

It appears that in 1945, the capital of Zamboanga Province was transferred to


Dipolog. 2 Subsequently, or on June 16, 1948, Republic Act 286 was approved creating the
municipality of Molave and making it the capital of Zamboanga Province.
334

On May 26, 1949, the Appraisal Committee formed by the Auditor General, pursuant to
Commonwealth Act 39, fixed the value of the properties and buildings in question left by Zamboanga
Province in Zamboanga City at P1,294,244.00. 3

On June 6, 1952, Republic Act 711 was approved dividing the province of Zamboanga into two
(2): Zamboanga del Norte and Zamboanga del Sur. As to how the assets and obligations of the old
province were to be divided between the two new ones, Sec. 6 of that law provided:

Upon the approval of this Act, the funds, assets and other properties and the
obligations of the province of Zamboanga shall be divided equitably between the Province of
Zamboanga del Norte and the Province of Zamboanga del Sur by the President of the
Philippines, upon the recommendation of the Auditor General.

Pursuant thereto, the Auditor General, on January 11, 1955, apportioned the assets and
obligations of the defunct Province of Zamboanga as follows: 54.39% for Zamboanga del Norte and
45.61% for Zamboanga del Sur. Zamboanga del Norte therefore became entitled to 54.39% of
P1,294,244.00, the total value of the lots and buildings in question, or P704,220.05 payable by
Zamboanga City.

On March 17, 1959, the Executive Secretary, by order of the President, issued a ruling 4 holding
that Zamboanga del Norte had a vested right as owner (should be co-owner pro-indiviso) of the
properties mentioned in Sec. 50 of Commonwealth Act 39, and is entitled to the price thereof, payable
by Zamboanga City. This ruling revoked the previous Cabinet Resolution of July 13, 1951 conveying
all the said 50 lots and buildings thereon to Zamboanga City for P1.00, effective as of 1945, when the
provincial capital of the then Zamboanga Province was transferred to Dipolog.

The Secretary of Finance then authorized the Commissioner of Internal Revenue to deduct an
amount equal to 25% of the regular internal revenue allotment for the City of Zamboanga for the
quarter ending March 31, 1960, then for the quarter ending June 30, 1960, and again for the first
quarter of the fiscal year 1960-1961. The deductions, all aggregating P57,373.46, was credited to the
province of Zamboanga del Norte, in partial payment of the P764,220.05 due it.

However, on June 17, 1961, Republic Act 3039 was approved amending Sec. 50 of
Commonwealth Act 39 by providing that —

All buildings, properties and assets belonging to the former province of Zamboanga
and located within the City of Zamboanga are hereby transferred, free of charge, in favor of
the said City of Zamboanga. (Stressed for emphasis).

Consequently, the Secretary of Finance, on July 12, 1961, ordered the Commissioner of
Internal Revenue to stop from effecting further payments to Zamboanga del Norte and to return to
Zamboanga City the sum of P57,373.46 taken from it out of the internal revenue allotment of
Zamboanga del Norte. Zamboanga City admits that since the enactment of Republic Act 3039,
P43,030.11 of the P57,373.46 has already been returned to it.

This constrained plaintiff-appellee Zamboanga del Norte to file on March 5, 1962, a complaint
entitled "Declaratory Relief with Preliminary Mandatory Injunction" in the Court of First Instance of
Zamboanga del Norte against defendants-appellants Zamboanga City, the Secretary of Finance and
the Commissioner of Internal Revenue. It was prayed that: (a) Republic Act 3039 be declared
unconstitutional for depriving plaintiff province of property without due process and just compensation;
(b) Plaintiff's rights and obligations under said law be declared; (c) The Secretary of Finance and the
Internal Revenue Commissioner be enjoined from reimbursing the sum of P57,373.46 to defendant
City; and (d) The latter be ordered to continue paying the balance of P704,220.05 in quarterly
installments of 25% of its internal revenue allotments.

On June 4, 1962, the lower court ordered the issuance of preliminary injunction as prayed for.
After defendants filed their respective answers, trial was held. On August 12, 1963, judgment was
rendered, the dispositive portion of which reads:
335

WHEREFORE, judgment is hereby rendered declaring Republic Act No. 3039


unconstitutional insofar as it deprives plaintiff Zamboanga del Norte of its private properties,
consisting of 50 parcels of land and the improvements thereon under certificates of title
(Exhibits "A" to "A-49") in the name of the defunct province of Zamboanga; ordering
defendant City of Zamboanga to pay to the plaintiff the sum of P704,220.05 payment thereof
to be deducted from its regular quarterly internal revenue allotment equivalent to 25% thereof
every quarter until said amount shall have been fully paid; ordering defendant Secretary of
Finance to direct defendant Commissioner of Internal Revenue to deduct 25% from the
regular quarterly internal revenue allotment for defendant City of Zamboanga and to remit the
same to plaintiff Zamboanga del Norte until said sum of P704,220.05 shall have been fully
paid; ordering plaintiff Zamboanga del Norte to execute through its proper officials the
corresponding public instrument deeding to defendant City of Zamboanga the 50 parcels of
land and the improvements thereon under the certificates of title (Exhibits "A" to "A-49") upon
payment by the latter of the aforesaid sum of P704,220.05 in full; dismissing the counterclaim
of defendant City of Zamboanga; and declaring permanent the preliminary mandatory
injunction issued on June 8, 1962, pursuant to the order of the Court dated June 4, 1962. No
costs are assessed against the defendants.

It is SO ORDERED.

Subsequently, but prior to the perfection of defendants' appeal, plaintiff province filed a motion
to reconsider praying that Zamboanga City be ordered instead to pay the P704,220.05 in lump
sum with 6% interest per annum. Over defendants' opposition, the lower court granted plaintiff
province's motion.

The defendants then brought the case before Us on appeal.

Brushing aside the procedural point concerning the property of declaratory relief filed in the
lower court on the assertion that the law had already been violated and that plaintiff sought to give it
coercive effect, since assuming the same to be true, the Rules anyway authorize the conversion of
the proceedings to an ordinary action, 5 We proceed to the more important and principal question of
the validity of Republic Act 3039.

The validity of the law ultimately depends on the nature of the 50 lots and buildings thereon in
question. For, the matter involved here is the extent of legislative control over the properties of a
municipal corporation, of which a province is one. The principle itself is simple: If the property is
owned by the municipality (meaning municipal corporation) in its public and governmental capacity,
the property is public and Congress has absolute control over it. But if the property is owned in its
private or proprietary capacity, then it is patrimonial and Congress has no absolute control. The
municipality cannot be deprived of it without due process and payment of just compensation. 6

The capacity in which the property is held is, however, dependent on the use to which it is
intended and devoted. Now, which of two norms, i.e., that of the Civil Code or that obtaining under the
law of Municipal Corporations, must be used in classifying the properties in question?

The Civil Code classification is embodied in its Arts. 423 and 424 which provide:1äwphï1.ñët

ART. 423. The property of provinces, cities, and municipalities is divided into property
for public use and patrimonial property.

ART. 424. Property for public use, in the provinces, cities, and municipalities, consists
of the provincial roads, city streets, municipal streets, the squares, fountains, public waters,
promenades, and public works for public service paid for by said provinces, cities, or
municipalities.

All other property possessed by any of them is patrimonial and shall be governed by this
Code, without prejudice to the provisions of special laws. (Stressed for emphasis).
336

Applying the above cited norm, all the properties in question, except the two (2) lots used as
High School playgrounds, could be considered as patrimonial properties of the former Zamboanga
province. Even the capital site, the hospital and leprosarium sites, and the school sites will be
considered patrimonial for they are not for public use. They would fall under the phrase "public works
for public service" for it has been held that under the ejusdem generis rule, such public works must be
for free and indiscriminate use by anyone, just like the preceding enumerated properties in the first
paragraph of Art 424. 7 The playgrounds, however, would fit into this category.

This was the norm applied by the lower court. And it cannot be said that its actuation was
without jurisprudential precedent for in Municipality of Catbalogan v. Director of Lands, 8 and
in Municipality of Tacloban v. Director of Lands, 9 it was held that the capitol site and the school sites
in municipalities constitute their patrimonial properties. This result is understandable because, unlike
in the classification regarding State properties, properties for public service in the municipalities are
not classified as public. Assuming then the Civil Code classification to be the chosen norm, the lower
court must be affirmed except with regard to the two (2) lots used as playgrounds.

On the other hand, applying the norm obtaining under the principles constituting the law of
Municipal Corporations, all those of the 50 properties in question which are devoted to public service
are deemed public; the rest remain patrimonial. Under this norm, to be considered public, it is enough
that the property be held and, devoted for governmental purposes like local administration, public
education, public health, etc. 10

Supporting jurisprudence are found in the following cases: (1) HINUNANGAN V. DIRECTOR
OF LANDS, 11where it was stated that "... where the municipality has occupied lands distinctly for
public purposes, such as for the municipal court house, the public school, the public market, or other
necessary municipal building, we will, in the absence of proof to the contrary, presume a grant from
the States in favor of the municipality; but, as indicated by the wording, that rule may be invoked only
as to property which is used distinctly for public purposes...." (2) VIUDA DE TANTOCO V.
MUNICIPAL COUNCIL OF ILOILO 12 held that municipal properties necessary for governmental
purposes are public in nature. Thus, the auto trucks used by the municipality for street sprinkling, the
police patrol automobile, police stations and concrete structures with the corresponding lots used as
markets were declared exempt from execution and attachment since they were not patrimonial
properties. (3) MUNICIPALITY OF BATANGAS VS. CANTOS 13 held squarely that a municipal lot
which had always been devoted to school purposes is one dedicated to public use and is not
patrimonial property of a municipality.

Following this classification, Republic Act 3039 is valid insofar as it affects the lots used as
capitol site, school sites and its grounds, hospital and leprosarium sites and the high school
playground sites — a total of 24 lots — since these were held by the former Zamboanga province in
its governmental capacity and therefore are subject to the absolute control of Congress. Said lots
considered as public property are the following:

TCT
Lot Number Use
Number
2200 ...................................... 4-B ...................................... Capitol Site
2816 ...................................... 149 ...................................... School Site
3281 ...................................... 1224 ...................................... Hospital Site
3282 ...................................... 1226 ...................................... Hospital Site
3283 ...................................... 1225 ...................................... Hospital Site
3748 ...................................... 434-A-1 ...................................... School Site
5406 ...................................... 171 ...................................... School Site
High School Play-
5564 ...................................... 168 ......................................
ground
157 &
5567 ...................................... ...................................... Trade School
158
337

High School Play-


5583 ...................................... 167 ......................................
ground
6181 ...................................... (O.C.T.) ...................................... Curuan School
11942 ...................................... 926 ...................................... Leprosarium
11943 ...................................... 927 ...................................... Leprosarium
11944 ...................................... 925 ...................................... Leprosarium
5557 ...................................... 170 ...................................... Burleigh School
5562 ...................................... 180 ...................................... Burleigh School
5565 ...................................... 172-B ...................................... Burleigh
5570 ...................................... 171-A ...................................... Burleigh
5571 ...................................... 172-C ...................................... Burleigh
5572 ...................................... 174 ...................................... Burleigh
5573 ...................................... 178 ...................................... Burleigh
5585 ...................................... 171-B ...................................... Burleigh
5586 ...................................... 173 ...................................... Burleigh
5587 ...................................... 172-A ...................................... Burleigh

We noticed that the eight Burleigh lots above described are adjoining each other and in turn are
between the two lots wherein the Burleigh schools are built, as per records appearing herein and in
the Bureau of Lands. Hence, there is sufficient basis for holding that said eight lots constitute the
appurtenant grounds of the Burleigh schools, and partake of the nature of the same.

Regarding the several buildings existing on the lots above-mentioned, the records do not
disclose whether they were constructed at the expense of the former Province of Zamboanga.
Considering however the fact that said buildings must have been erected even before 1936 when
Commonwealth Act 39 was enacted and the further fact that provinces then had no power to
authorize construction of buildings such as those in the case at bar at their own expense, 14 it can be
assumed that said buildings were erected by the National Government, using national funds. Hence,
Congress could very well dispose of said buildings in the same manner that it did with the lots in
question.

But even assuming that provincial funds were used, still the buildings constitute mere
accessories to the lands, which are public in nature, and so, they follow the nature of said lands,
i.e., public. Moreover, said buildings, though located in the city, will not be for the exclusive use and
benefit of city residents for they could be availed of also by the provincial residents. The province then
— and its successors-in-interest — are not really deprived of the benefits thereof.

But Republic Act 3039 cannot be applied to deprive Zamboanga del Norte of its share in the
value of the rest of the 26 remaining lots which are patrimonial properties since they are not being
utilized for distinctly, governmental purposes. Said lots are:

TCT Number Lot Number Use


5577 ...................................... 177 ...................................... Mydro, Magay
13198 ...................................... 127-0 ...................................... San Roque
5569 ...................................... 169 ...................................... Burleigh 15
5558 ...................................... 175 ...................................... Vacant
5559 ...................................... 188 ...................................... "
5560 ...................................... 183 ...................................... "
5561 ...................................... 186 ...................................... "
5563 ...................................... 191 ...................................... "
5566 ...................................... 176 ...................................... "
5568 ...................................... 179 ...................................... "
338

5574 ...................................... 196 ...................................... "


5575 ...................................... 181-A ...................................... "
5576 ...................................... 181-B ...................................... "
5578 ...................................... 182 ...................................... "
5579 ...................................... 197 ...................................... "
5580 ...................................... 195 ...................................... "
5581 ...................................... 159-B ...................................... "
5582 ...................................... 194 ...................................... "
5584 ...................................... 190 ...................................... "
5588 ...................................... 184 ...................................... "
5589 ...................................... 187 ...................................... "
5590 ...................................... 189 ...................................... "
5591 ...................................... 192 ...................................... "
5592 ...................................... 193 ...................................... "
5593 ...................................... 185 ...................................... "
7379 ...................................... 4147 ...................................... "

Moreover, the fact that these 26 lots are registered strengthens the proposition that they are
truly private in nature. On the other hand, that the 24 lots used for governmental purposes are also
registered is of no significance since registration cannot convert public property to private. 16

We are more inclined to uphold this latter view. The controversy here is more along the
domains of the Law of Municipal Corporations — State vs. Province — than along that of Civil Law.
Moreover, this Court is not inclined to hold that municipal property held and devoted to public service
is in the same category as ordinary private property. The consequences are dire. As ordinary private
properties, they can be levied upon and attached. They can even be acquired thru adverse
possession — all these to the detriment of the local community. Lastly, the classification of properties
other than those for public use in the municipalities as patrimonial under Art. 424 of the Civil Code —
is "... without prejudice to the provisions of special laws." For purpose of this article, the principles,
obtaining under the Law of Municipal Corporations can be considered as "special laws". Hence, the
classification of municipal property devoted for distinctly governmental purposes as public should
prevail over the Civil Code classification in this particular case.

Defendants' claim that plaintiff and its predecessor-in-interest are "guilty of laches is without
merit. Under Commonwealth Act 39, Sec. 50, the cause of action in favor of the defunct Zamboanga
Province arose only in 1949 after the Auditor General fixed the value of the properties in question.
While in 1951, the Cabinet resolved transfer said properties practically for free to Zamboanga City, a
reconsideration thereof was seasonably sought. In 1952, the old province was dissolved. As
successor-in-interest to more than half of the properties involved, Zamboanga del Norte was able to
get a reconsideration of the Cabinet Resolution in 1959. In fact, partial payments were effected
subsequently and it was only after the passage of Republic Act 3039 in 1961 that the present
controversy arose. Plaintiff brought suit in 1962. All the foregoing, negative laches.

It results then that Zamboanga del Norte is still entitled to collect from the City of Zamboanga
the former's 54.39% share in the 26 properties which are patrimonial in nature, said share to
computed on the basis of the valuation of said 26 properties as contained in Resolution No. 7, dated
March 26, 1949, of the Appraisal Committee formed by the Auditor General.

Plaintiff's share, however, cannot be paid in lump sum, except as to the P43,030.11 already
returned to defendant City. The return of said amount to defendant was without legal basis. Republic
Act 3039 took effect only on June 17, 1961 after a partial payment of P57,373.46 had already been
made. Since the law did not provide for retroactivity, it could not have validly affected a completed act.
Hence, the amount of P43,030.11 should be immediately returned by defendant City to plaintiff
province. The remaining balance, if any, in the amount of plaintiff's 54.39% share in the 26 lots should
then be paid by defendant City in the same manner originally adopted by the Secretary of Finance
339

and the Commissioner of Internal Revenue, and not in lump sum. Plaintiff's prayer, particularly pars. 5
and 6, read together with pars. 10 and 11 of the first cause of action recited in the complaint 17clearly
shows that the relief sought was merely the continuance of the quarterly payments from the internal
revenue allotments of defendant City. Art. 1169 of the Civil Code on reciprocal obligations invoked by
plaintiff to justify lump sum payment is inapplicable since there has been so far in legal contemplation
no complete delivery of the lots in question. The titles to the registered lots are not yet in the name of
defendant Zamboanga City.

WHEREFORE, the decision appealed from is hereby set aside and another judgment is hereby
entered as follows:.

(1) Defendant Zamboanga City is hereby ordered to return to plaintiff Zamboanga del Norte in
lump sum the amount of P43,030.11 which the former took back from the latter out of the sum of
P57,373.46 previously paid to the latter; and

(2) Defendants are hereby ordered to effect payments in favor of plaintiff of whatever balance
remains of plaintiff's 54.39% share in the 26 patrimonial properties, after deducting therefrom the sum
of P57,373.46, on the basis of Resolution No. 7 dated March 26, 1949 of the Appraisal Committee
formed by the Auditor General, by way of quarterly payments from the allotments of defendant City, in
the manner originally adopted by the Secretary of Finance and the Commissioner of Internal
Revenue. No costs. So ordered.

Reyes, J.B.L., Actg. C.J., Dizon, Makalintal, Zaldivar, Sanchez, Castro, Angeles and Fernando, JJ.,
concur.
Concepcion, C.J., is on leave.

Footnotes

1See Record on Appeal, pp. 4-6.

2See Exhibit C.

3The Committee report itself was not submitted as evidence

4Exhibit C.

5Rule 64, Sec. 6, Rules of Court.

62McQuillin, Municipal Corporations, 3rd ed., 191-196; Martin Public Corporation, 5th ed., 31-
32; Gonzales, Law on Public Corporations, 1962 ed., 29-30; Municipality of Naguilian v.
NWSA, L-18452, Nov. 29, 1963.

7Cebu City v. NWSA, L-12892, Apr. 30, 1962.

817 Phil. 216.

917 Phil. 426.

10Martin, op. cit., supra.; Gonzales, op cit., supra.; 62 C.J. 8. 437-439.

1124 Phil. 124.

1249 Phil. 52.

1391 Phil. 514.


340

14Itwas only in Republic Act 2264, Sec. 3, last paragraph, that provinces, cities and
municipalities were "... authorized to undertake and carry out any public works
projects, financed by the provincial city and municipalfunds or any other fund borrowed from
or advanced by private third parties .. without the intervention of the Department of Public
Works and Communications." (Stressed for emphasis) This law was approved and took effect
on June 19, 1959.

15This could not be considered as forming part of the appurtenant grounds of the Burleigh
school sites since the records here and in the Bureau of Lands show that this lot is set apart
from the other Burleigh lots.

16Republic v. Sioson, L-13687, Nov. 29, 1963; Hodges V. City of Iloilo, L-17573, June 30,
1962.

17Record on Appeal, pp. 8-9, 13.

CHAVEZ v PEA AMARI (G.R. No. 133250, July 9, 2002)

EN BANC

G.R. No. 133250 July 9, 2002

FRANCISCO I. CHAVEZ, petitioner,


vs.
PUBLIC ESTATES AUTHORITY and AMARI COASTAL BAY DEVELOPMENT
CORPORATION, respondents.

CARPIO, J.:

This is an original Petition for Mandamus with prayer for a writ of preliminary injunction and a
temporary restraining order. The petition seeks to compel the Public Estates Authority ("PEA" for
brevity) to disclose all facts on PEA's then on-going renegotiations with Amari Coastal Bay and
Development Corporation ("AMARI" for brevity) to reclaim portions of Manila Bay. The petition further
seeks to enjoin PEA from signing a new agreement with AMARI involving such reclamation.

The Facts

On November 20, 1973, the government, through the Commissioner of Public Highways, signed a
contract with the Construction and Development Corporation of the Philippines ("CDCP" for brevity) to
reclaim certain foreshore and offshore areas of Manila Bay. The contract also included the
construction of Phases I and II of the Manila-Cavite Coastal Road. CDCP obligated itself to carry out
all the works in consideration of fifty percent of the total reclaimed land.

On February 4, 1977, then President Ferdinand E. Marcos issued Presidential Decree No. 1084
creating PEA. PD No. 1084 tasked PEA "to reclaim land, including foreshore and submerged areas,"
and "to develop, improve, acquire, x x x lease and sell any and all kinds of lands." 1 On the same date,
then President Marcos issued Presidential Decree No. 1085 transferring to PEA the "lands reclaimed
in the foreshore and offshore of the Manila Bay"2 under the Manila-Cavite Coastal Road and
Reclamation Project (MCCRRP).
341

On December 29, 1981, then President Marcos issued a memorandum directing PEA to amend its
contract with CDCP, so that "[A]ll future works in MCCRRP x x x shall be funded and owned by PEA."
Accordingly, PEA and CDCP executed a Memorandum of Agreement dated December 29, 1981,
which stated:

"(i) CDCP shall undertake all reclamation, construction, and such other works in the
MCCRRP as may be agreed upon by the parties, to be paid according to progress of works
on a unit price/lump sum basis for items of work to be agreed upon, subject to price
escalation, retention and other terms and conditions provided for in Presidential Decree No.
1594. All the financing required for such works shall be provided by PEA.

xxx

(iii) x x x CDCP shall give up all its development rights and hereby agrees to cede and
transfer in favor of PEA, all of the rights, title, interest and participation of CDCP in and to all
the areas of land reclaimed by CDCP in the MCCRRP as of December 30, 1981 which have
not yet been sold, transferred or otherwise disposed of by CDCP as of said date, which areas
consist of approximately Ninety-Nine Thousand Four Hundred Seventy Three (99,473)
square meters in the Financial Center Area covered by land pledge No. 5 and approximately
Three Million Three Hundred Eighty Two Thousand Eight Hundred Eighty Eight (3,382,888)
square meters of reclaimed areas at varying elevations above Mean Low Water Level located
outside the Financial Center Area and the First Neighborhood Unit." 3

On January 19, 1988, then President Corazon C. Aquino issued Special Patent No. 3517, granting
and transferring to PEA "the parcels of land so reclaimed under the Manila-Cavite Coastal Road and
Reclamation Project (MCCRRP) containing a total area of one million nine hundred fifteen thousand
eight hundred ninety four (1,915,894) square meters." Subsequently, on April 9, 1988, the Register of
Deeds of the Municipality of Parañaque issued Transfer Certificates of Title Nos. 7309, 7311, and
7312, in the name of PEA, covering the three reclaimed islands known as the "Freedom Islands"
located at the southern portion of the Manila-Cavite Coastal Road, Parañaque City. The Freedom
Islands have a total land area of One Million Five Hundred Seventy Eight Thousand Four Hundred
and Forty One (1,578,441) square meters or 157.841 hectares.

On April 25, 1995, PEA entered into a Joint Venture Agreement ("JVA" for brevity) with AMARI, a
private corporation, to develop the Freedom Islands. The JVA also required the reclamation of an
additional 250 hectares of submerged areas surrounding these islands to complete the configuration
in the Master Development Plan of the Southern Reclamation Project-MCCRRP. PEA and AMARI
entered into the JVA through negotiation without public bidding.4 On April 28, 1995, the Board of
Directors of PEA, in its Resolution No. 1245, confirmed the JVA.5 On June 8, 1995, then President
Fidel V. Ramos, through then Executive Secretary Ruben Torres, approved the JVA. 6

On November 29, 1996, then Senate President Ernesto Maceda delivered a privilege speech in the
Senate and denounced the JVA as the "grandmother of all scams." As a result, the Senate
Committee on Government Corporations and Public Enterprises, and the Committee on
Accountability of Public Officers and Investigations, conducted a joint investigation. The Senate
Committees reported the results of their investigation in Senate Committee Report No. 560 dated
September 16, 1997.7 Among the conclusions of their report are: (1) the reclaimed lands PEA seeks
to transfer to AMARI under the JVA are lands of the public domain which the government has not
classified as alienable lands and therefore PEA cannot alienate these lands; (2) the certificates of title
covering the Freedom Islands are thus void, and (3) the JVA itself is illegal.

On December 5, 1997, then President Fidel V. Ramos issued Presidential Administrative Order No.
365 creating a Legal Task Force to conduct a study on the legality of the JVA in view of Senate
Committee Report No. 560. The members of the Legal Task Force were the Secretary of Justice,8 the
Chief Presidential Legal Counsel,9 and the Government Corporate Counsel.10 The Legal Task Force
upheld the legality of the JVA, contrary to the conclusions reached by the Senate Committees. 11
342

On April 4 and 5, 1998, the Philippine Daily Inquirer and Today published reports that there were on-
going renegotiations between PEA and AMARI under an order issued by then President Fidel V.
Ramos. According to these reports, PEA Director Nestor Kalaw, PEA Chairman Arsenio Yulo and
retired Navy Officer Sergio Cruz composed the negotiating panel of PEA.

On April 13, 1998, Antonio M. Zulueta filed before the Court a Petition for Prohibition with Application
for the Issuance of a Temporary Restraining Order and Preliminary Injunction docketed as G.R. No.
132994 seeking to nullify the JVA. The Court dismissed the petition "for unwarranted disregard of
judicial hierarchy, without prejudice to the refiling of the case before the proper court." 12

On April 27, 1998, petitioner Frank I. Chavez ("Petitioner" for brevity) as a taxpayer, filed the
instant Petition for Mandamus with Prayer for the Issuance of a Writ of Preliminary Injunction and
Temporary Restraining Order. Petitioner contends the government stands to lose billions of pesos in
the sale by PEA of the reclaimed lands to AMARI. Petitioner prays that PEA publicly disclose the
terms of any renegotiation of the JVA, invoking Section 28, Article II, and Section 7, Article III, of the
1987 Constitution on the right of the people to information on matters of public concern. Petitioner
assails the sale to AMARI of lands of the public domain as a blatant violation of Section 3, Article XII
of the 1987 Constitution prohibiting the sale of alienable lands of the public domain to private
corporations. Finally, petitioner asserts that he seeks to enjoin the loss of billions of pesos in
properties of the State that are of public dominion.

After several motions for extension of time,13 PEA and AMARI filed their Comments on October 19,
1998 and June 25, 1998, respectively. Meanwhile, on December 28, 1998, petitioner filed an
Omnibus Motion: (a) to require PEA to submit the terms of the renegotiated PEA-AMARI contract; (b)
for issuance of a temporary restraining order; and (c) to set the case for hearing on oral argument.
Petitioner filed a Reiterative Motion for Issuance of a TRO dated May 26, 1999, which the Court
denied in a Resolution dated June 22, 1999.

In a Resolution dated March 23, 1999, the Court gave due course to the petition and required the
parties to file their respective memoranda.

On March 30, 1999, PEA and AMARI signed the Amended Joint Venture Agreement ("Amended
JVA," for brevity). On May 28, 1999, the Office of the President under the administration of then
President Joseph E. Estrada approved the Amended JVA.

Due to the approval of the Amended JVA by the Office of the President, petitioner now prays that on
"constitutional and statutory grounds the renegotiated contract be declared null and void." 14

The Issues

The issues raised by petitioner, PEA15 and AMARI16 are as follows:

I. WHETHER THE PRINCIPAL RELIEFS PRAYED FOR IN THE PETITION ARE MOOT AND
ACADEMIC BECAUSE OF SUBSEQUENT EVENTS;

II. WHETHER THE PETITION MERITS DISMISSAL FOR FAILING TO OBSERVE THE
PRINCIPLE GOVERNING THE HIERARCHY OF COURTS;

III. WHETHER THE PETITION MERITS DISMISSAL FOR NON-EXHAUSTION OF


ADMINISTRATIVE REMEDIES;

IV. WHETHER PETITIONER HAS LOCUS STANDI TO BRING THIS SUIT;

V. WHETHER THE CONSTITUTIONAL RIGHT TO INFORMATION INCLUDES OFFICIAL


INFORMATION ON ON-GOING NEGOTIATIONS BEFORE A FINAL AGREEMENT;
343

VI. WHETHER THE STIPULATIONS IN THE AMENDED JOINT VENTURE AGREEMENT


FOR THE TRANSFER TO AMARI OF CERTAIN LANDS, RECLAIMED AND STILL TO BE
RECLAIMED, VIOLATE THE 1987 CONSTITUTION; AND

VII. WHETHER THE COURT IS THE PROPER FORUM FOR RAISING THE ISSUE OF
WHETHER THE AMENDED JOINT VENTURE AGREEMENT IS GROSSLY
DISADVANTAGEOUS TO THE GOVERNMENT.

The Court's Ruling

First issue: whether the principal reliefs prayed for in the petition are moot and academic
because of subsequent events.

The petition prays that PEA publicly disclose the "terms and conditions of the on-going negotiations
for a new agreement." The petition also prays that the Court enjoin PEA from "privately entering into,
perfecting and/or executing any new agreement with AMARI."

PEA and AMARI claim the petition is now moot and academic because AMARI furnished petitioner on
June 21, 1999 a copy of the signed Amended JVA containing the terms and conditions agreed upon
in the renegotiations. Thus, PEA has satisfied petitioner's prayer for a public disclosure of the
renegotiations. Likewise, petitioner's prayer to enjoin the signing of the Amended JVA is now moot
because PEA and AMARI have already signed the Amended JVA on March 30, 1999. Moreover, the
Office of the President has approved the Amended JVA on May 28, 1999.

Petitioner counters that PEA and AMARI cannot avoid the constitutional issue by simply fast-tracking
the signing and approval of the Amended JVA before the Court could act on the issue. Presidential
approval does not resolve the constitutional issue or remove it from the ambit of judicial review.

We rule that the signing of the Amended JVA by PEA and AMARI and its approval by the President
cannot operate to moot the petition and divest the Court of its jurisdiction. PEA and AMARI have still
to implement the Amended JVA. The prayer to enjoin the signing of the Amended JVA on
constitutional grounds necessarily includes preventing its implementation if in the meantime PEA and
AMARI have signed one in violation of the Constitution. Petitioner's principal basis in assailing the
renegotiation of the JVA is its violation of Section 3, Article XII of the Constitution, which prohibits the
government from alienating lands of the public domain to private corporations. If the Amended JVA
indeed violates the Constitution, it is the duty of the Court to enjoin its implementation, and if already
implemented, to annul the effects of such unconstitutional contract.

The Amended JVA is not an ordinary commercial contract but one which seeks to transfer title and
ownership to 367.5 hectares of reclaimed lands and submerged areas of Manila Bay to a single
private corporation. It now becomes more compelling for the Court to resolve the issue to insure the
government itself does not violate a provision of the Constitution intended to safeguard the national
patrimony. Supervening events, whether intended or accidental, cannot prevent the Court from
rendering a decision if there is a grave violation of the Constitution. In the instant case, if the
Amended JVA runs counter to the Constitution, the Court can still prevent the transfer of title and
ownership of alienable lands of the public domain in the name of AMARI. Even in cases where
supervening events had made the cases moot, the Court did not hesitate to resolve the legal or
constitutional issues raised to formulate controlling principles to guide the bench, bar, and the
public.17

Also, the instant petition is a case of first impression. All previous decisions of the Court involving
Section 3, Article XII of the 1987 Constitution, or its counterpart provision in the 1973
Constitution,18 covered agricultural lands sold to private corporations which acquired the lands from
private parties. The transferors of the private corporations claimed or could claim the right to judicial
confirmation of their imperfect titles19 under Title II of Commonwealth Act. 141 ("CA No. 141" for
brevity). In the instant case, AMARI seeks to acquire from PEA, a public corporation, reclaimed lands
and submerged areas for non-agricultural purposes by purchase under PD No. 1084 (charter of
PEA) and Title III of CA No. 141. Certain undertakings by AMARI under the Amended JVA constitute
344

the consideration for the purchase. Neither AMARI nor PEA can claim judicial confirmation of their
titles because the lands covered by the Amended JVA are newly reclaimed or still to be reclaimed.
Judicial confirmation of imperfect title requires open, continuous, exclusive and notorious occupation
of agricultural lands of the public domain for at least thirty years since June 12, 1945 or earlier.
Besides, the deadline for filing applications for judicial confirmation of imperfect title expired on
December 31, 1987.20

Lastly, there is a need to resolve immediately the constitutional issue raised in this petition because of
the possible transfer at any time by PEA to AMARI of title and ownership to portions of the reclaimed
lands. Under the Amended JVA, PEA is obligated to transfer to AMARI the latter's seventy percent
proportionate share in the reclaimed areas as the reclamation progresses. The Amended JVA even
allows AMARI to mortgage at any time the entire reclaimed area to raise financing for the reclamation
project.21

Second issue: whether the petition merits dismissal for failing to observe the principle
governing the hierarchy of courts.

PEA and AMARI claim petitioner ignored the judicial hierarchy by seeking relief directly from the
Court. The principle of hierarchy of courts applies generally to cases involving factual questions. As it
is not a trier of facts, the Court cannot entertain cases involving factual issues. The instant case,
however, raises constitutional issues of transcendental importance to the public. 22 The Court can
resolve this case without determining any factual issue related to the case. Also, the instant case is a
petition for mandamus which falls under the original jurisdiction of the Court under Section 5, Article
VIII of the Constitution. We resolve to exercise primary jurisdiction over the instant case.

Third issue: whether the petition merits dismissal for non-exhaustion of administrative
remedies.

PEA faults petitioner for seeking judicial intervention in compelling PEA to disclose publicly certain
information without first asking PEA the needed information. PEA claims petitioner's direct resort to
the Court violates the principle of exhaustion of administrative remedies. It also violates the rule that
mandamus may issue only if there is no other plain, speedy and adequate remedy in the ordinary
course of law.

PEA distinguishes the instant case from Tañada v. Tuvera23 where the Court granted the petition for
mandamus even if the petitioners there did not initially demand from the Office of the President the
publication of the presidential decrees. PEA points out that in Tañada, the Executive Department had
an affirmative statutory duty under Article 2 of the Civil Code24 and Section 1 of Commonwealth Act
No. 63825 to publish the presidential decrees. There was, therefore, no need for the petitioners in
Tañada to make an initial demand from the Office of the President. In the instant case, PEA claims it
has no affirmative statutory duty to disclose publicly information about its renegotiation of the JVA.
Thus, PEA asserts that the Court must apply the principle of exhaustion of administrative remedies to
the instant case in view of the failure of petitioner here to demand initially from PEA the needed
information.

The original JVA sought to dispose to AMARI public lands held by PEA, a government corporation.
Under Section 79 of the Government Auditing Code,26 the disposition of government lands to private
parties requires public bidding. PEA was under a positive legal duty to disclose to the public the
terms and conditions for the sale of its lands. The law obligated PEA to make this public
disclosure even without demand from petitioner or from anyone. PEA failed to make this public
disclosure because the original JVA, like the Amended JVA, was the result of a negotiated contract,
not of a public bidding. Considering that PEA had an affirmative statutory duty to make the public
disclosure, and was even in breach of this legal duty, petitioner had the right to seek direct judicial
intervention.

Moreover, and this alone is determinative of this issue, the principle of exhaustion of administrative
remedies does not apply when the issue involved is a purely legal or constitutional question. 27 The
principal issue in the instant case is the capacity of AMARI to acquire lands held by PEA in view of the
345

constitutional ban prohibiting the alienation of lands of the public domain to private corporations. We
rule that the principle of exhaustion of administrative remedies does not apply in the instant case.

Fourth issue: whether petitioner has locus standi to bring this suit

PEA argues that petitioner has no standing to institute mandamus proceedings to enforce his
constitutional right to information without a showing that PEA refused to perform an affirmative duty
imposed on PEA by the Constitution. PEA also claims that petitioner has not shown that he will suffer
any concrete injury because of the signing or implementation of the Amended JVA. Thus, there is no
actual controversy requiring the exercise of the power of judicial review.

The petitioner has standing to bring this taxpayer's suit because the petition seeks to compel PEA to
comply with its constitutional duties. There are two constitutional issues involved here. First is the
right of citizens to information on matters of public concern. Second is the application of a
constitutional provision intended to insure the equitable distribution of alienable lands of the public
domain among Filipino citizens. The thrust of the first issue is to compel PEA to disclose publicly
information on the sale of government lands worth billions of pesos, information which the
Constitution and statutory law mandate PEA to disclose. The thrust of the second issue is to prevent
PEA from alienating hundreds of hectares of alienable lands of the public domain in violation of the
Constitution, compelling PEA to comply with a constitutional duty to the nation.

Moreover, the petition raises matters of transcendental importance to the public. In Chavez v.
PCGG,28 the Court upheld the right of a citizen to bring a taxpayer's suit on matters of transcendental
importance to the public, thus -

"Besides, petitioner emphasizes, the matter of recovering the ill-gotten wealth of the
Marcoses is an issue of 'transcendental importance to the public.' He asserts that ordinary
taxpayers have a right to initiate and prosecute actions questioning the validity of acts or
orders of government agencies or instrumentalities, if the issues raised are of 'paramount
public interest,' and if they 'immediately affect the social, economic and moral well being of
the people.'

Moreover, the mere fact that he is a citizen satisfies the requirement of personal interest,
when the proceeding involves the assertion of a public right, such as in this case. He invokes
several decisions of this Court which have set aside the procedural matter of locus standi,
when the subject of the case involved public interest.

xxx

In Tañada v. Tuvera, the Court asserted that when the issue concerns a public right and the
object of mandamus is to obtain the enforcement of a public duty, the people are regarded as
the real parties in interest; and because it is sufficient that petitioner is a citizen and as such is
interested in the execution of the laws, he need not show that he has any legal or special
interest in the result of the action. In the aforesaid case, the petitioners sought to enforce their
right to be informed on matters of public concern, a right then recognized in Section 6, Article
IV of the 1973 Constitution, in connection with the rule that laws in order to be valid and
enforceable must be published in the Official Gazette or otherwise effectively promulgated. In
ruling for the petitioners' legal standing, the Court declared that the right they sought to be
enforced 'is a public right recognized by no less than the fundamental law of the land.'

Legaspi v. Civil Service Commission, while reiterating Tañada, further declared that 'when a
mandamus proceeding involves the assertion of a public right, the requirement of personal
interest is satisfied by the mere fact that petitioner is a citizen and, therefore, part of the
general 'public' which possesses the right.'

Further, in Albano v. Reyes, we said that while expenditure of public funds may not have
been involved under the questioned contract for the development, management and
operation of the Manila International Container Terminal, 'public interest [was] definitely
346

involved considering the important role [of the subject contract] . . . in the economic
development of the country and the magnitude of the financial consideration involved.' We
concluded that, as a consequence, the disclosure provision in the Constitution would
constitute sufficient authority for upholding the petitioner's standing.

Similarly, the instant petition is anchored on the right of the people to information and access
to official records, documents and papers — a right guaranteed under Section 7, Article III of
the 1987 Constitution. Petitioner, a former solicitor general, is a Filipino citizen. Because of
the satisfaction of the two basic requisites laid down by decisional law to sustain petitioner's
legal standing, i.e. (1) the enforcement of a public right (2) espoused by a Filipino citizen, we
rule that the petition at bar should be allowed."

We rule that since the instant petition, brought by a citizen, involves the enforcement of constitutional
rights - to information and to the equitable diffusion of natural resources - matters of transcendental
public importance, the petitioner has the requisite locus standi.

Fifth issue: whether the constitutional right to information includes official information on on-
going negotiations before a final agreement.

Section 7, Article III of the Constitution explains the people's right to information on matters of public
concern in this manner:

"Sec. 7. The right of the people to information on matters of public concern shall be
recognized. Access to official records, and to documents, and papers pertaining to
official acts, transactions, or decisions, as well as to government research data used as
basis for policy development, shall be afforded the citizen, subject to such limitations as may
be provided by law." (Emphasis supplied)

The State policy of full transparency in all transactions involving public interest reinforces the people's
right to information on matters of public concern. This State policy is expressed in Section 28, Article
II of the Constitution, thus:

"Sec. 28. Subject to reasonable conditions prescribed by law, the State adopts and
implements a policy of full public disclosure of all its transactions involving public
interest." (Emphasis supplied)

These twin provisions of the Constitution seek to promote transparency in policy-making and in the
operations of the government, as well as provide the people sufficient information to exercise
effectively other constitutional rights. These twin provisions are essential to the exercise of freedom of
expression. If the government does not disclose its official acts, transactions and decisions to citizens,
whatever citizens say, even if expressed without any restraint, will be speculative and amount to
nothing. These twin provisions are also essential to hold public officials "at all times x x x accountable
to the people,"29 for unless citizens have the proper information, they cannot hold public officials
accountable for anything. Armed with the right information, citizens can participate in public
discussions leading to the formulation of government policies and their effective implementation. An
informed citizenry is essential to the existence and proper functioning of any democracy. As explained
by the Court in Valmonte v. Belmonte, Jr.30 –

"An essential element of these freedoms is to keep open a continuing dialogue or process of
communication between the government and the people. It is in the interest of the State that
the channels for free political discussion be maintained to the end that the government may
perceive and be responsive to the people's will. Yet, this open dialogue can be effective only
to the extent that the citizenry is informed and thus able to formulate its will intelligently. Only
when the participants in the discussion are aware of the issues and have access to
information relating thereto can such bear fruit."

PEA asserts, citing Chavez v. PCGG,31 that in cases of on-going negotiations the right to information
is limited to "definite propositions of the government." PEA maintains the right does not include
347

access to "intra-agency or inter-agency recommendations or communications during the stage when


common assertions are still in the process of being formulated or are in the 'exploratory stage'."

Also, AMARI contends that petitioner cannot invoke the right at the pre-decisional stage or before the
closing of the transaction. To support its contention, AMARI cites the following discussion in the 1986
Constitutional Commission:

"Mr. Suarez. And when we say 'transactions' which should be distinguished from contracts,
agreements, or treaties or whatever, does the Gentleman refer to the steps leading to the
consummation of the contract, or does he refer to the contract itself?

Mr. Ople: The 'transactions' used here, I suppose is generic and therefore, it can cover
both steps leading to a contract and already a consummated contract, Mr. Presiding
Officer.

Mr. Suarez: This contemplates inclusion of negotiations leading to the consummation


of the transaction.

Mr. Ople: Yes, subject only to reasonable safeguards on the national interest.

Mr. Suarez: Thank you."32 (Emphasis supplied)

AMARI argues there must first be a consummated contract before petitioner can invoke the right.
Requiring government officials to reveal their deliberations at the pre-decisional stage will degrade the
quality of decision-making in government agencies. Government officials will hesitate to express their
real sentiments during deliberations if there is immediate public dissemination of their discussions,
putting them under all kinds of pressure before they decide.

We must first distinguish between information the law on public bidding requires PEA to disclose
publicly, and information the constitutional right to information requires PEA to release to the public.
Before the consummation of the contract, PEA must, on its own and without demand from anyone,
disclose to the public matters relating to the disposition of its property. These include the size,
location, technical description and nature of the property being disposed of, the terms and conditions
of the disposition, the parties qualified to bid, the minimum price and similar information. PEA must
prepare all these data and disclose them to the public at the start of the disposition process, long
before the consummation of the contract, because the Government Auditing Code requires public
bidding. If PEA fails to make this disclosure, any citizen can demand from PEA this information at
any time during the bidding process.

Information, however, on on-going evaluation or review of bids or proposals being undertaken by


the bidding or review committee is not immediately accessible under the right to information. While
the evaluation or review is still on-going, there are no "official acts, transactions, or decisions" on the
bids or proposals. However, once the committee makes its official recommendation, there arises
a "definite proposition" on the part of the government. From this moment, the public's right to
information attaches, and any citizen can access all the non-proprietary information leading to such
definite proposition. In Chavez v. PCGG,33 the Court ruled as follows:

"Considering the intent of the framers of the Constitution, we believe that it is incumbent upon
the PCGG and its officers, as well as other government representatives, to disclose sufficient
public information on any proposed settlement they have decided to take up with the
ostensible owners and holders of ill-gotten wealth. Such information, though, must pertain
to definite propositions of the government, not necessarily to intra-agency or inter-agency
recommendations or communications during the stage when common assertions are still in
the process of being formulated or are in the "exploratory" stage. There is need, of course, to
observe the same restrictions on disclosure of information in general, as discussed earlier –
such as on matters involving national security, diplomatic or foreign relations, intelligence and
other classified information." (Emphasis supplied)
348

Contrary to AMARI's contention, the commissioners of the 1986 Constitutional Commission


understood that the right to information "contemplates inclusion of negotiations leading to the
consummation of the transaction."Certainly, a consummated contract is not a requirement for the
exercise of the right to information. Otherwise, the people can never exercise the right if no contract is
consummated, and if one is consummated, it may be too late for the public to expose its
defects.1âwphi1.nêt

Requiring a consummated contract will keep the public in the dark until the contract, which may be
grossly disadvantageous to the government or even illegal, becomes a fait accompli. This negates the
State policy of full transparency on matters of public concern, a situation which the framers of the
Constitution could not have intended. Such a requirement will prevent the citizenry from participating
in the public discussion of any proposed contract, effectively truncating a basic right enshrined in the
Bill of Rights. We can allow neither an emasculation of a constitutional right, nor a retreat by the State
of its avowed "policy of full disclosure of all its transactions involving public interest."

The right covers three categories of information which are "matters of public concern," namely: (1)
official records; (2) documents and papers pertaining to official acts, transactions and decisions; and
(3) government research data used in formulating policies. The first category refers to any document
that is part of the public records in the custody of government agencies or officials. The second
category refers to documents and papers recording, evidencing, establishing, confirming, supporting,
justifying or explaining official acts, transactions or decisions of government agencies or officials. The
third category refers to research data, whether raw, collated or processed, owned by the government
and used in formulating government policies.

The information that petitioner may access on the renegotiation of the JVA includes evaluation
reports, recommendations, legal and expert opinions, minutes of meetings, terms of reference and
other documents attached to such reports or minutes, all relating to the JVA. However, the right to
information does not compel PEA to prepare lists, abstracts, summaries and the like relating to the
renegotiation of the JVA.34 The right only affords access to records, documents and papers, which
means the opportunity to inspect and copy them. One who exercises the right must copy the records,
documents and papers at his expense. The exercise of the right is also subject to reasonable
regulations to protect the integrity of the public records and to minimize disruption to government
operations, like rules specifying when and how to conduct the inspection and copying. 35

The right to information, however, does not extend to matters recognized as privileged information
under the separation of powers.36 The right does not also apply to information on military and
diplomatic secrets, information affecting national security, and information on investigations of crimes
by law enforcement agencies before the prosecution of the accused, which courts have long
recognized as confidential.37 The right may also be subject to other limitations that Congress may
impose by law.

There is no claim by PEA that the information demanded by petitioner is privileged information rooted
in the separation of powers. The information does not cover Presidential conversations,
correspondences, or discussions during closed-door Cabinet meetings which, like internal
deliberations of the Supreme Court and other collegiate courts, or executive sessions of either house
of Congress,38 are recognized as confidential. This kind of information cannot be pried open by a co-
equal branch of government. A frank exchange of exploratory ideas and assessments, free from the
glare of publicity and pressure by interested parties, is essential to protect the independence of
decision-making of those tasked to exercise Presidential, Legislative and Judicial power. 39 This is not
the situation in the instant case.

We rule, therefore, that the constitutional right to information includes official information on on-going
negotiationsbefore a final contract. The information, however, must constitute definite propositions
by the government and should not cover recognized exceptions like privileged information, military
and diplomatic secrets and similar matters affecting national security and public order. 40 Congress
has also prescribed other limitations on the right to information in several legislations. 41

Sixth issue: whether stipulations in the Amended JVA for the transfer to AMARI of lands,
reclaimed or to be reclaimed, violate the Constitution.
349

The Regalian Doctrine

The ownership of lands reclaimed from foreshore and submerged areas is rooted in the Regalian
doctrine which holds that the State owns all lands and waters of the public domain. Upon the Spanish
conquest of the Philippines, ownership of all "lands, territories and possessions" in the Philippines
passed to the Spanish Crown.42 The King, as the sovereign ruler and representative of the people,
acquired and owned all lands and territories in the Philippines except those he disposed of by grant or
sale to private individuals.

The 1935, 1973 and 1987 Constitutions adopted the Regalian doctrine substituting, however, the
State, in lieu of the King, as the owner of all lands and waters of the public domain. The Regalian
doctrine is the foundation of the time-honored principle of land ownership that "all lands that were not
acquired from the Government, either by purchase or by grant, belong to the public domain."43 Article
339 of the Civil Code of 1889, which is now Article 420 of the Civil Code of 1950, incorporated the
Regalian doctrine.

Ownership and Disposition of Reclaimed Lands

The Spanish Law of Waters of 1866 was the first statutory law governing the ownership and
disposition of reclaimed lands in the Philippines. On May 18, 1907, the Philippine Commission
enacted Act No. 1654 which provided for the lease, but not the sale, of reclaimed lands of the
government to corporations and individuals. Later, on November 29, 1919, the Philippine
Legislature approved Act No. 2874, the Public Land Act, which authorized the lease, but not the
sale, of reclaimed lands of the government to corporations and individuals. On November 7,
1936, the National Assembly passed Commonwealth Act No. 141, also known as the Public Land Act,
which authorized the lease, but not the sale, of reclaimed lands of the government to
corporations and individuals. CA No. 141 continues to this day as the general law governing the
classification and disposition of lands of the public domain.

The Spanish Law of Waters of 1866 and the Civil Code of 1889

Under the Spanish Law of Waters of 1866, the shores, bays, coves, inlets and all waters within the
maritime zone of the Spanish territory belonged to the public domain for public use.44 The Spanish
Law of Waters of 1866 allowed the reclamation of the sea under Article 5, which provided as follows:

"Article 5. Lands reclaimed from the sea in consequence of works constructed by the State, or
by the provinces, pueblos or private persons, with proper permission, shall become the
property of the party constructing such works, unless otherwise provided by the terms of the
grant of authority."

Under the Spanish Law of Waters, land reclaimed from the sea belonged to the party undertaking the
reclamation, provided the government issued the necessary permit and did not reserve ownership of
the reclaimed land to the State.

Article 339 of the Civil Code of 1889 defined property of public dominion as follows:

"Art. 339. Property of public dominion is –

1. That devoted to public use, such as roads, canals, rivers, torrents, ports and bridges
constructed by the State, riverbanks, shores, roadsteads, and that of a similar character;

2. That belonging exclusively to the State which, without being of general public use, is
employed in some public service, or in the development of the national wealth, such as walls,
fortresses, and other works for the defense of the territory, and mines, until granted to private
individuals."
350

Property devoted to public use referred to property open for use by the public. In contrast, property
devoted to public service referred to property used for some specific public service and open only to
those authorized to use the property.

Property of public dominion referred not only to property devoted to public use, but also to property
not so used but employed to develop the national wealth. This class of property constituted
property of public dominion although employed for some economic or commercial activity to increase
the national wealth.

Article 341 of the Civil Code of 1889 governed the re-classification of property of public dominion into
private property, to wit:

"Art. 341. Property of public dominion, when no longer devoted to public use or to the defense
of the territory, shall become a part of the private property of the State."

This provision, however, was not self-executing. The legislature, or the executive department
pursuant to law, must declare the property no longer needed for public use or territorial defense
before the government could lease or alienate the property to private parties. 45

Act No. 1654 of the Philippine Commission

On May 8, 1907, the Philippine Commission enacted Act No. 1654 which regulated the lease of
reclaimed and foreshore lands. The salient provisions of this law were as follows:

"Section 1. The control and disposition of the foreshore as defined in existing law, and
the title to all Government or public lands made or reclaimed by the Government by
dredging or filling or otherwise throughout the Philippine Islands, shall be retained by the
Government without prejudice to vested rights and without prejudice to rights conceded to
the City of Manila in the Luneta Extension.

Section 2. (a) The Secretary of the Interior shall cause all Government or public lands made
or reclaimed by the Government by dredging or filling or otherwise to be divided into lots or
blocks, with the necessary streets and alleyways located thereon, and shall cause plats and
plans of such surveys to be prepared and filed with the Bureau of Lands.

(b) Upon completion of such plats and plans the Governor-General shall give notice to the
public that such parts of the lands so made or reclaimed as are not needed for public
purposes will be leased for commercial and business purposes, x x x.

xxx

(e) The leases above provided for shall be disposed of to the highest and best
bidder therefore, subject to such regulations and safeguards as the Governor-General may
by executive order prescribe." (Emphasis supplied)

Act No. 1654 mandated that the government should retain title to all lands reclaimed by the
government. The Act also vested in the government control and disposition of foreshore lands.
Private parties could lease lands reclaimed by the government only if these lands were no longer
needed for public purpose. Act No. 1654 mandated public bidding in the lease of government
reclaimed lands. Act No. 1654 made government reclaimed lands sui generis in that unlike other
public lands which the government could sell to private parties, these reclaimed lands were available
only for lease to private parties.

Act No. 1654, however, did not repeal Section 5 of the Spanish Law of Waters of 1866. Act No. 1654
did not prohibit private parties from reclaiming parts of the sea under Section 5 of the Spanish Law of
Waters. Lands reclaimed from the sea by private parties with government permission remained
private lands.
351

Act No. 2874 of the Philippine Legislature

On November 29, 1919, the Philippine Legislature enacted Act No. 2874, the Public Land Act.46 The
salient provisions of Act No. 2874, on reclaimed lands, were as follows:

"Sec. 6. The Governor-General, upon the recommendation of the Secretary of


Agriculture and Natural Resources, shall from time to time classify the lands of the
public domain into –

(a) Alienable or disposable,

(b) Timber, and

(c) Mineral lands, x x x.

Sec. 7. For the purposes of the government and disposition of alienable or disposable public
lands, the Governor-General, upon recommendation by the Secretary of Agriculture
and Natural Resources, shall from time to time declare what lands are open to
disposition or concession under this Act."

Sec. 8. Only those lands shall be declared open to disposition or concession which
have been officially delimited or classified x x x.

xxx

Sec. 55. Any tract of land of the public domain which, being neither timber nor mineral land,
shall be classified as suitable for residential purposes or for commercial, industrial, or
other productive purposes other than agricultural purposes, and shall be open to
disposition or concession, shall be disposed of under the provisions of this chapter, and not
otherwise.

Sec. 56. The lands disposable under this title shall be classified as follows:

(a) Lands reclaimed by the Government by dredging, filling, or other means;

(b) Foreshore;

(c) Marshy lands or lands covered with water bordering upon the shores or banks of
navigable lakes or rivers;

(d) Lands not included in any of the foregoing classes.

x x x.

Sec. 58. The lands comprised in classes (a), (b), and (c) of section fifty-six shall be
disposed of to private parties by lease only and not otherwise, as soon as the
Governor-General, upon recommendation by the Secretary of Agriculture and Natural
Resources, shall declare that the same are not necessary for the public service and are
open to disposition under this chapter. The lands included in class (d) may be disposed
of by sale or lease under the provisions of this Act." (Emphasis supplied)

Section 6 of Act No. 2874 authorized the Governor-General to "classify lands of the public domain
into x x x alienable or disposable"47 lands. Section 7 of the Act empowered the Governor-General to
"declare what lands are open to disposition or concession." Section 8 of the Act limited alienable or
disposable lands only to those lands which have been "officially delimited and classified."
352

Section 56 of Act No. 2874 stated that lands "disposable under this title 48 shall be classified" as
government reclaimed, foreshore and marshy lands, as well as other lands. All these lands, however,
must be suitable for residential, commercial, industrial or other productive non-agricultural purposes.
These provisions vested upon the Governor-General the power to classify inalienable lands of the
public domain into disposable lands of the public domain. These provisions also empowered the
Governor-General to classify further such disposable lands of the public domain into government
reclaimed, foreshore or marshy lands of the public domain, as well as other non-agricultural lands.

Section 58 of Act No. 2874 categorically mandated that disposable lands of the public domain
classified as government reclaimed, foreshore and marshy lands "shall be disposed of to private
parties by lease only and not otherwise." The Governor-General, before allowing the lease of
these lands to private parties, must formally declare that the lands were "not necessary for the public
service." Act No. 2874 reiterated the State policy to lease and not to sell government reclaimed,
foreshore and marshy lands of the public domain, a policy first enunciated in 1907 in Act No. 1654.
Government reclaimed, foreshore and marshy lands remained sui generis, as the only alienable or
disposable lands of the public domain that the government could not sell to private parties.

The rationale behind this State policy is obvious. Government reclaimed, foreshore and marshy public
lands for non-agricultural purposes retain their inherent potential as areas for public service. This is
the reason the government prohibited the sale, and only allowed the lease, of these lands to private
parties. The State always reserved these lands for some future public service.

Act No. 2874 did not authorize the reclassification of government reclaimed, foreshore and marshy
lands into other non-agricultural lands under Section 56 (d). Lands falling under Section 56 (d) were
the only lands for non-agricultural purposes the government could sell to private parties. Thus, under
Act No. 2874, the government could not sell government reclaimed, foreshore and marshy lands to
private parties, unless the legislature passed a law allowing their sale.49

Act No. 2874 did not prohibit private parties from reclaiming parts of the sea pursuant to Section 5 of
the Spanish Law of Waters of 1866. Lands reclaimed from the sea by private parties with government
permission remained private lands.

Dispositions under the 1935 Constitution

On May 14, 1935, the 1935 Constitution took effect upon its ratification by the Filipino people. The
1935 Constitution, in adopting the Regalian doctrine, declared in Section 1, Article XIII, that –

"Section 1. All agricultural, timber, and mineral lands of the public domain, waters, minerals,
coal, petroleum, and other mineral oils, all forces of potential energy and other natural
resources of the Philippines belong to the State, and their disposition, exploitation,
development, or utilization shall be limited to citizens of the Philippines or to corporations or
associations at least sixty per centum of the capital of which is owned by such citizens,
subject to any existing right, grant, lease, or concession at the time of the inauguration of the
Government established under this Constitution. Natural resources, with the exception of
public agricultural land, shall not be alienated, and no license, concession, or lease for
the exploitation, development, or utilization of any of the natural resources shall be granted
for a period exceeding twenty-five years, renewable for another twenty-five years, except as
to water rights for irrigation, water supply, fisheries, or industrial uses other than the
development of water power, in which cases beneficial use may be the measure and limit of
the grant." (Emphasis supplied)

The 1935 Constitution barred the alienation of all natural resources except public agricultural lands,
which were the only natural resources the State could alienate. Thus, foreshore lands, considered
part of the State's natural resources, became inalienable by constitutional fiat, available only for lease
for 25 years, renewable for another 25 years. The government could alienate foreshore lands only
after these lands were reclaimed and classified as alienable agricultural lands of the public domain.
Government reclaimed and marshy lands of the public domain, being neither timber nor mineral
lands, fell under the classification of public agricultural lands. 50 However, government reclaimed and
353

marshy lands, although subject to classification as disposable public agricultural lands, could only be
leased and not sold to private parties because of Act No. 2874.

The prohibition on private parties from acquiring ownership of government reclaimed and marshy
lands of the public domain was only a statutory prohibition and the legislature could therefore remove
such prohibition. The 1935 Constitution did not prohibit individuals and corporations from acquiring
government reclaimed and marshy lands of the public domain that were classified as agricultural
lands under existing public land laws. Section 2, Article XIII of the 1935 Constitution provided as
follows:

"Section 2. No private corporation or association may acquire, lease, or hold public


agricultural lands in excess of one thousand and twenty four hectares, nor may any
individual acquire such lands by purchase in excess of one hundred and forty
hectares, or by lease in excess of one thousand and twenty-four hectares, or by
homestead in excess of twenty-four hectares. Lands adapted to grazing, not exceeding two
thousand hectares, may be leased to an individual, private corporation, or association."
(Emphasis supplied)

Still, after the effectivity of the 1935 Constitution, the legislature did not repeal Section 58 of Act No.
2874 to open for sale to private parties government reclaimed and marshy lands of the public domain.
On the contrary, the legislature continued the long established State policy of retaining for the
government title and ownership of government reclaimed and marshy lands of the public domain.

Commonwealth Act No. 141 of the Philippine National Assembly

On November 7, 1936, the National Assembly approved Commonwealth Act No. 141, also known as
the Public Land Act, which compiled the then existing laws on lands of the public domain. CA No.
141, as amended, remains to this day the existing general law governing the classification and
disposition of lands of the public domain other than timber and mineral lands. 51

Section 6 of CA No. 141 empowers the President to classify lands of the public domain into "alienable
or disposable"52 lands of the public domain, which prior to such classification are inalienable and
outside the commerce of man. Section 7 of CA No. 141 authorizes the President to "declare what
lands are open to disposition or concession." Section 8 of CA No. 141 states that the government can
declare open for disposition or concession only lands that are "officially delimited and classified."
Sections 6, 7 and 8 of CA No. 141 read as follows:

"Sec. 6. The President, upon the recommendation of the Secretary of Agriculture and
Commerce, shall from time to time classify the lands of the public domain into –

(a) Alienable or disposable,

(b) Timber, and

(c) Mineral lands,

and may at any time and in like manner transfer such lands from one class to another,53 for
the purpose of their administration and disposition.

Sec. 7. For the purposes of the administration and disposition of alienable or disposable
public lands, the President, upon recommendation by the Secretary of Agriculture and
Commerce, shall from time to time declare what lands are open to disposition or
concession under this Act.

Sec. 8. Only those lands shall be declared open to disposition or concession which
have been officially delimited and classified and, when practicable, surveyed, and which
have not been reserved for public or quasi-public uses, nor appropriated by the
354

Government, nor in any manner become private property, nor those on which a private right
authorized and recognized by this Act or any other valid law may be claimed, or which, having
been reserved or appropriated, have ceased to be so. x x x."

Thus, before the government could alienate or dispose of lands of the public domain, the President
must first officially classify these lands as alienable or disposable, and then declare them open to
disposition or concession. There must be no law reserving these lands for public or quasi-public uses.

The salient provisions of CA No. 141, on government reclaimed, foreshore and marshy lands of the
public domain, are as follows:

"Sec. 58. Any tract of land of the public domain which, being neither timber nor mineral
land, is intended to be used for residential purposes or for commercial, industrial, or
other productive purposes other than agricultural, and is open to disposition or
concession, shall be disposed of under the provisions of this chapter and not
otherwise.

Sec. 59. The lands disposable under this title shall be classified as follows:

(a) Lands reclaimed by the Government by dredging, filling, or other means;

(b) Foreshore;

(c) Marshy lands or lands covered with water bordering upon the shores or banks of
navigable lakes or rivers;

(d) Lands not included in any of the foregoing classes.

Sec. 60. Any tract of land comprised under this title may be leased or sold, as the case may
be, to any person, corporation, or association authorized to purchase or lease public lands for
agricultural purposes. x x x.

Sec. 61. The lands comprised in classes (a), (b), and (c) of section fifty-nine shall be
disposed of to private parties by lease only and not otherwise, as soon as the
President, upon recommendation by the Secretary of Agriculture, shall declare that the
same are not necessary for the public service and are open to disposition under this
chapter. The lands included in class (d) may be disposed of by sale or lease under the
provisions of this Act." (Emphasis supplied)

Section 61 of CA No. 141 readopted, after the effectivity of the 1935 Constitution, Section 58 of Act
No. 2874 prohibiting the sale of government reclaimed, foreshore and marshy disposable lands of the
public domain. All these lands are intended for residential, commercial, industrial or other non-
agricultural purposes. As before, Section 61 allowed only the lease of such lands to private parties.
The government could sell to private parties only lands falling under Section 59 (d) of CA No. 141, or
those lands for non-agricultural purposes not classified as government reclaimed, foreshore and
marshy disposable lands of the public domain. Foreshore lands, however, became inalienable under
the 1935 Constitution which only allowed the lease of these lands to qualified private parties.

Section 58 of CA No. 141 expressly states that disposable lands of the public domain intended for
residential, commercial, industrial or other productive purposes other than agricultural "shall be
disposed of under the provisions of this chapter and not otherwise." Under Section 10 of CA No.
141, the term "disposition" includes lease of the land. Any disposition of government reclaimed,
foreshore and marshy disposable lands for non-agricultural purposes must comply with Chapter IX,
Title III of CA No. 141,54 unless a subsequent law amended or repealed these provisions.

In his concurring opinion in the landmark case of Republic Real Estate Corporation v. Court of
Appeals,55Justice Reynato S. Puno summarized succinctly the law on this matter, as follows:
355

"Foreshore lands are lands of public dominion intended for public use. So too are lands
reclaimed by the government by dredging, filling, or other means. Act 1654 mandated that the
control and disposition of the foreshore and lands under water remained in the national
government. Said law allowed only the 'leasing' of reclaimed land. The Public Land Acts of
1919 and 1936 also declared that the foreshore and lands reclaimed by the government were
to be "disposed of to private parties by lease only and not otherwise." Before leasing,
however, the Governor-General, upon recommendation of the Secretary of Agriculture and
Natural Resources, had first to determine that the land reclaimed was not necessary for the
public service. This requisite must have been met before the land could be disposed of. But
even then, the foreshore and lands under water were not to be alienated and sold to
private parties. The disposition of the reclaimed land was only by lease. The land
remained property of the State." (Emphasis supplied)

As observed by Justice Puno in his concurring opinion, "Commonwealth Act No. 141 has remained in
effect at present."

The State policy prohibiting the sale to private parties of government reclaimed, foreshore and marshy
alienable lands of the public domain, first implemented in 1907 was thus reaffirmed in CA No. 141
after the 1935 Constitution took effect. The prohibition on the sale of foreshore lands, however,
became a constitutional edict under the 1935 Constitution. Foreshore lands became inalienable as
natural resources of the State, unless reclaimed by the government and classified as agricultural
lands of the public domain, in which case they would fall under the classification of government
reclaimed lands.

After the effectivity of the 1935 Constitution, government reclaimed and marshy disposable lands of
the public domain continued to be only leased and not sold to private parties. 56 These lands
remained sui generis, as the only alienable or disposable lands of the public domain the government
could not sell to private parties.

Since then and until now, the only way the government can sell to private parties government
reclaimed and marshy disposable lands of the public domain is for the legislature to pass a law
authorizing such sale. CA No. 141 does not authorize the President to reclassify government
reclaimed and marshy lands into other non-agricultural lands under Section 59 (d). Lands classified
under Section 59 (d) are the only alienable or disposable lands for non-agricultural purposes that the
government could sell to private parties.

Moreover, Section 60 of CA No. 141 expressly requires congressional authority before lands under
Section 59 that the government previously transferred to government units or entities could be sold to
private parties. Section 60 of CA No. 141 declares that –

"Sec. 60. x x x The area so leased or sold shall be such as shall, in the judgment of the
Secretary of Agriculture and Natural Resources, be reasonably necessary for the purposes
for which such sale or lease is requested, and shall not exceed one hundred and forty-four
hectares: Provided, however, That this limitation shall not apply to grants, donations, or
transfers made to a province, municipality or branch or subdivision of the Government for the
purposes deemed by said entities conducive to the public interest; but the land so granted,
donated, or transferred to a province, municipality or branch or subdivision of the
Government shall not be alienated, encumbered, or otherwise disposed of in a manner
affecting its title, except when authorized by Congress: x x x." (Emphasis supplied)

The congressional authority required in Section 60 of CA No. 141 mirrors the legislative authority
required in Section 56 of Act No. 2874.

One reason for the congressional authority is that Section 60 of CA No. 141 exempted government
units and entities from the maximum area of public lands that could be acquired from the State. These
government units and entities should not just turn around and sell these lands to private parties in
violation of constitutional or statutory limitations. Otherwise, the transfer of lands for non-agricultural
purposes to government units and entities could be used to circumvent constitutional limitations on
356

ownership of alienable or disposable lands of the public domain. In the same manner, such transfers
could also be used to evade the statutory prohibition in CA No. 141 on the sale of government
reclaimed and marshy lands of the public domain to private parties. Section 60 of CA No. 141
constitutes by operation of law a lien on these lands.57

In case of sale or lease of disposable lands of the public domain falling under Section 59 of CA No.
141, Sections 63 and 67 require a public bidding. Sections 63 and 67 of CA No. 141 provide as
follows:

"Sec. 63. Whenever it is decided that lands covered by this chapter are not needed for public
purposes, the Director of Lands shall ask the Secretary of Agriculture and Commerce (now
the Secretary of Natural Resources) for authority to dispose of the same. Upon receipt of
such authority, the Director of Lands shall give notice by public advertisement in the same
manner as in the case of leases or sales of agricultural public land, x x x.

Sec. 67. The lease or sale shall be made by oral bidding; and adjudication shall be
made to the highest bidder. x x x." (Emphasis supplied)

Thus, CA No. 141 mandates the Government to put to public auction all leases or sales of alienable
or disposable lands of the public domain.58

Like Act No. 1654 and Act No. 2874 before it, CA No. 141 did not repeal Section 5 of the Spanish Law
of Waters of 1866. Private parties could still reclaim portions of the sea with government permission.
However, the reclaimed land could become private land only if classified as alienable
agricultural land of the public domain open to disposition under CA No. 141. The 1935 Constitution
prohibited the alienation of all natural resources except public agricultural lands.

The Civil Code of 1950

The Civil Code of 1950 readopted substantially the definition of property of public dominion found in
the Civil Code of 1889. Articles 420 and 422 of the Civil Code of 1950 state that –

"Art. 420. The following things are property of public dominion:

(1) Those intended for public use, such as roads, canals, rivers, torrents, ports and bridges
constructed by the State, banks, shores, roadsteads, and others of similar character;

(2) Those which belong to the State, without being for public use, and are intended for some
public service or for the development of the national wealth.

x x x.

Art. 422. Property of public dominion, when no longer intended for public use or for public
service, shall form part of the patrimonial property of the State."

Again, the government must formally declare that the property of public dominion is no longer needed
for public use or public service, before the same could be classified as patrimonial property of the
State.59 In the case of government reclaimed and marshy lands of the public domain, the declaration
of their being disposable, as well as the manner of their disposition, is governed by the applicable
provisions of CA No. 141.

Like the Civil Code of 1889, the Civil Code of 1950 included as property of public dominion those
properties of the State which, without being for public use, are intended for public service or the
"development of the national wealth." Thus, government reclaimed and marshy lands of the State,
even if not employed for public use or public service, if developed to enhance the national wealth, are
classified as property of public dominion.
357

Dispositions under the 1973 Constitution

The 1973 Constitution, which took effect on January 17, 1973, likewise adopted the Regalian
doctrine. Section 8, Article XIV of the 1973 Constitution stated that –

"Sec. 8. All lands of the public domain, waters, minerals, coal, petroleum and other mineral
oils, all forces of potential energy, fisheries, wildlife, and other natural resources of the
Philippines belong to the State. With the exception of agricultural, industrial or
commercial, residential, and resettlement lands of the public domain, natural
resources shall not be alienated, and no license, concession, or lease for the exploration,
development, exploitation, or utilization of any of the natural resources shall be granted for a
period exceeding twenty-five years, renewable for not more than twenty-five years, except as
to water rights for irrigation, water supply, fisheries, or industrial uses other than the
development of water power, in which cases, beneficial use may be the measure and the limit
of the grant." (Emphasis supplied)

The 1973 Constitution prohibited the alienation of all natural resources with the exception of
"agricultural, industrial or commercial, residential, and resettlement lands of the public domain." In
contrast, the 1935 Constitution barred the alienation of all natural resources except "public agricultural
lands." However, the term "public agricultural lands" in the 1935 Constitution encompassed industrial,
commercial, residential and resettlement lands of the public domain.60 If the land of public domain
were neither timber nor mineral land, it would fall under the classification of agricultural land of the
public domain. Both the 1935 and 1973 Constitutions, therefore, prohibited the alienation of all
natural resources except agricultural lands of the public domain.

The 1973 Constitution, however, limited the alienation of lands of the public domain to individuals who
were citizens of the Philippines. Private corporations, even if wholly owned by Philippine citizens,
were no longer allowed to acquire alienable lands of the public domain unlike in the 1935 Constitution.
Section 11, Article XIV of the 1973 Constitution declared that –

"Sec. 11. The Batasang Pambansa, taking into account conservation, ecological, and
development requirements of the natural resources, shall determine by law the size of land of
the public domain which may be developed, held or acquired by, or leased to, any qualified
individual, corporation, or association, and the conditions therefor. No private corporation or
association may hold alienable lands of the public domain except by lease not to
exceed one thousand hectares in area nor may any citizen hold such lands by lease in
excess of five hundred hectares or acquire by purchase, homestead or grant, in excess of
twenty-four hectares. No private corporation or association may hold by lease, concession,
license or permit, timber or forest lands and other timber or forest resources in excess of one
hundred thousand hectares. However, such area may be increased by the Batasang
Pambansa upon recommendation of the National Economic and Development Authority."
(Emphasis supplied)

Thus, under the 1973 Constitution, private corporations could hold alienable lands of the public
domain only through lease. Only individuals could now acquire alienable lands of the public domain,
and private corporations became absolutely barred from acquiring any kind of alienable land
of the public domain. The constitutional ban extended to all kinds of alienable lands of the public
domain, while the statutory ban under CA No. 141 applied only to government reclaimed, foreshore
and marshy alienable lands of the public domain.

PD No. 1084 Creating the Public Estates Authority

On February 4, 1977, then President Ferdinand Marcos issued Presidential Decree No. 1084 creating
PEA, a wholly government owned and controlled corporation with a special charter. Sections 4 and 8
of PD No. 1084, vests PEA with the following purposes and powers:

"Sec. 4. Purpose. The Authority is hereby created for the following purposes:
358

(a) To reclaim land, including foreshore and submerged areas, by dredging, filling or
other means, or to acquire reclaimed land;

(b) To develop, improve, acquire, administer, deal in, subdivide, dispose, lease and sell any
and all kinds of lands, buildings, estates and other forms of real property, owned, managed,
controlled and/or operated by the government;

(c) To provide for, operate or administer such service as may be necessary for the efficient,
economical and beneficial utilization of the above properties.

Sec. 5. Powers and functions of the Authority. The Authority shall, in carrying out the
purposes for which it is created, have the following powers and functions:

(a)To prescribe its by-laws.

xxx

(i) To hold lands of the public domain in excess of the area permitted to private
corporations by statute.

(j) To reclaim lands and to construct work across, or otherwise, any stream, watercourse,
canal, ditch, flume x x x.

xxx

(o) To perform such acts and exercise such functions as may be necessary for the attainment
of the purposes and objectives herein specified." (Emphasis supplied)

PD No. 1084 authorizes PEA to reclaim both foreshore and submerged areas of the public domain.
Foreshore areas are those covered and uncovered by the ebb and flow of the tide. 61 Submerged
areas are those permanently under water regardless of the ebb and flow of the tide. 62 Foreshore and
submerged areas indisputably belong to the public domain63 and are inalienable unless reclaimed,
classified as alienable lands open to disposition, and further declared no longer needed for public
service.

The ban in the 1973 Constitution on private corporations from acquiring alienable lands of the public
domain did not apply to PEA since it was then, and until today, a fully owned government corporation.
The constitutional ban applied then, as it still applies now, only to "private corporations and
associations." PD No. 1084 expressly empowers PEA "to hold lands of the public domain" even
"in excess of the area permitted to private corporations by statute." Thus, PEA can hold title to
private lands, as well as title to lands of the public domain.

In order for PEA to sell its reclaimed foreshore and submerged alienable lands of the public domain,
there must be legislative authority empowering PEA to sell these lands. This legislative authority is
necessary in view of Section 60 of CA No.141, which states –

"Sec. 60. x x x; but the land so granted, donated or transferred to a province, municipality, or
branch or subdivision of the Government shall not be alienated, encumbered or otherwise
disposed of in a manner affecting its title, except when authorized by Congress; x x x."
(Emphasis supplied)

Without such legislative authority, PEA could not sell but only lease its reclaimed foreshore and
submerged alienable lands of the public domain. Nevertheless, any legislative authority granted to
PEA to sell its reclaimed alienable lands of the public domain would be subject to the constitutional
ban on private corporations from acquiring alienable lands of the public domain. Hence, such
legislative authority could only benefit private individuals.
359

Dispositions under the 1987 Constitution

The 1987 Constitution, like the 1935 and 1973 Constitutions before it, has adopted the Regalian
doctrine. The 1987 Constitution declares that all natural resources are "owned by the State," and
except for alienable agricultural lands of the public domain, natural resources cannot be alienated.
Sections 2 and 3, Article XII of the 1987 Constitution state that –

"Section 2. All lands of the public domain, waters, minerals, coal, petroleum and other mineral
oils, all forces of potential energy, fisheries, forests or timber, wildlife, flora and fauna, and
other natural resources are owned by the State. With the exception of agricultural lands,
all other natural resources shall not be alienated. The exploration, development, and
utilization of natural resources shall be under the full control and supervision of the State. x x
x.

Section 3. Lands of the public domain are classified into agricultural, forest or timber, mineral
lands, and national parks. Agricultural lands of the public domain may be further classified by
law according to the uses which they may be devoted. Alienable lands of the public
domain shall be limited to agricultural lands. Private corporations or associations may
not hold such alienable lands of the public domain except by lease, for a period not
exceeding twenty-five years, renewable for not more than twenty-five years, and not to
exceed one thousand hectares in area. Citizens of the Philippines may lease not more than
five hundred hectares, or acquire not more than twelve hectares thereof by purchase,
homestead, or grant.

Taking into account the requirements of conservation, ecology, and development, and subject
to the requirements of agrarian reform, the Congress shall determine, by law, the size of
lands of the public domain which may be acquired, developed, held, or leased and the
conditions therefor." (Emphasis supplied)

The 1987 Constitution continues the State policy in the 1973 Constitution banning private
corporations from acquiring any kind of alienable land of the public domain. Like the 1973
Constitution, the 1987 Constitution allows private corporations to hold alienable lands of the public
domain only through lease. As in the 1935 and 1973 Constitutions, the general law governing the
lease to private corporations of reclaimed, foreshore and marshy alienable lands of the public domain
is still CA No. 141.

The Rationale behind the Constitutional Ban

The rationale behind the constitutional ban on corporations from acquiring, except through lease,
alienable lands of the public domain is not well understood. During the deliberations of the 1986
Constitutional Commission, the commissioners probed the rationale behind this ban, thus:

"FR. BERNAS: Mr. Vice-President, my questions have reference to page 3, line 5 which says:

`No private corporation or association may hold alienable lands of the public domain except
by lease, not to exceed one thousand hectares in area.'

If we recall, this provision did not exist under the 1935 Constitution, but this was introduced in
the 1973 Constitution. In effect, it prohibits private corporations from acquiring alienable
public lands. But it has not been very clear in jurisprudence what the reason for this is.
In some of the cases decided in 1982 and 1983, it was indicated that the purpose of this
is to prevent large landholdings. Is that the intent of this provision?

MR. VILLEGAS: I think that is the spirit of the provision.

FR. BERNAS: In existing decisions involving the Iglesia ni Cristo, there were instances where
the Iglesia ni Cristo was not allowed to acquire a mere 313-square meter land where a chapel
stood because the Supreme Court said it would be in violation of this." (Emphasis supplied)
360

In Ayog v. Cusi,64 the Court explained the rationale behind this constitutional ban in this way:

"Indeed, one purpose of the constitutional prohibition against purchases of public agricultural
lands by private corporations is to equitably diffuse land ownership or to encourage 'owner-
cultivatorship and the economic family-size farm' and to prevent a recurrence of cases like the
instant case. Huge landholdings by corporations or private persons had spawned social
unrest."

However, if the constitutional intent is to prevent huge landholdings, the Constitution could have
simply limited the size of alienable lands of the public domain that corporations could acquire. The
Constitution could have followed the limitations on individuals, who could acquire not more than 24
hectares of alienable lands of the public domain under the 1973 Constitution, and not more than 12
hectares under the 1987 Constitution.

If the constitutional intent is to encourage economic family-size farms, placing the land in the name of
a corporation would be more effective in preventing the break-up of farmlands. If the farmland is
registered in the name of a corporation, upon the death of the owner, his heirs would inherit shares in
the corporation instead of subdivided parcels of the farmland. This would prevent the continuing
break-up of farmlands into smaller and smaller plots from one generation to the next.

In actual practice, the constitutional ban strengthens the constitutional limitation on individuals from
acquiring more than the allowed area of alienable lands of the public domain. Without the
constitutional ban, individuals who already acquired the maximum area of alienable lands of the public
domain could easily set up corporations to acquire more alienable public lands. An individual could
own as many corporations as his means would allow him. An individual could even hide his ownership
of a corporation by putting his nominees as stockholders of the corporation. The corporation is a
convenient vehicle to circumvent the constitutional limitation on acquisition by individuals of alienable
lands of the public domain.

The constitutional intent, under the 1973 and 1987 Constitutions, is to transfer ownership of only a
limited area of alienable land of the public domain to a qualified individual. This constitutional intent is
safeguarded by the provision prohibiting corporations from acquiring alienable lands of the public
domain, since the vehicle to circumvent the constitutional intent is removed. The available alienable
public lands are gradually decreasing in the face of an ever-growing population. The most effective
way to insure faithful adherence to this constitutional intent is to grant or sell alienable lands of the
public domain only to individuals. This, it would seem, is the practical benefit arising from the
constitutional ban.

The Amended Joint Venture Agreement

The subject matter of the Amended JVA, as stated in its second Whereas clause, consists of three
properties, namely:

1. "[T]hree partially reclaimed and substantially eroded islands along Emilio Aguinaldo
Boulevard in Paranaque and Las Pinas, Metro Manila, with a combined titled area of
1,578,441 square meters;"

2. "[A]nother area of 2,421,559 square meters contiguous to the three islands;" and

3. "[A]t AMARI's option as approved by PEA, an additional 350 hectares more or less to
regularize the configuration of the reclaimed area."65

PEA confirms that the Amended JVA involves "the development of the Freedom Islands and further
reclamation of about 250 hectares x x x," plus an option "granted to AMARI to subsequently reclaim
another 350 hectares x x x."66
361

In short, the Amended JVA covers a reclamation area of 750 hectares. Only 157.84 hectares of the
750-hectare reclamation project have been reclaimed, and the rest of the 592.15 hectares are
still submerged areas forming part of Manila Bay.

Under the Amended JVA, AMARI will reimburse PEA the sum of P1,894,129,200.00 for PEA's "actual
cost" in partially reclaiming the Freedom Islands. AMARI will also complete, at its own expense, the
reclamation of the Freedom Islands. AMARI will further shoulder all the reclamation costs of all the
other areas, totaling 592.15 hectares, still to be reclaimed. AMARI and PEA will share, in the
proportion of 70 percent and 30 percent, respectively, the total net usable area which is defined in the
Amended JVA as the total reclaimed area less 30 percent earmarked for common areas. Title to
AMARI's share in the net usable area, totaling 367.5 hectares, will be issued in the name of AMARI.
Section 5.2 (c) of the Amended JVA provides that –

"x x x, PEA shall have the duty to execute without delay the necessary deed of transfer or
conveyance of the title pertaining to AMARI's Land share based on the Land Allocation
Plan. PEA, when requested in writing by AMARI, shall then cause the issuance and
delivery of the proper certificates of title covering AMARI's Land Share in the name of
AMARI, x x x; provided, that if more than seventy percent (70%) of the titled area at any given
time pertains to AMARI, PEA shall deliver to AMARI only seventy percent (70%) of the titles
pertaining to AMARI, until such time when a corresponding proportionate area of additional
land pertaining to PEA has been titled." (Emphasis supplied)

Indisputably, under the Amended JVA AMARI will acquire and own a maximum of 367.5
hectares of reclaimed land which will be titled in its name.

To implement the Amended JVA, PEA delegated to the unincorporated PEA-AMARI joint venture
PEA's statutory authority, rights and privileges to reclaim foreshore and submerged areas in Manila
Bay. Section 3.2.a of the Amended JVA states that –

"PEA hereby contributes to the joint venture its rights and privileges to perform Rawland
Reclamation and Horizontal Development as well as own the Reclamation Area, thereby
granting the Joint Venture the full and exclusive right, authority and privilege to undertake the
Project in accordance with the Master Development Plan."

The Amended JVA is the product of a renegotiation of the original JVA dated April 25, 1995 and its
supplemental agreement dated August 9, 1995.

The Threshold Issue

The threshold issue is whether AMARI, a private corporation, can acquire and own under the
Amended JVA 367.5 hectares of reclaimed foreshore and submerged areas in Manila Bay in view of
Sections 2 and 3, Article XII of the 1987 Constitution which state that:

"Section 2. All lands of the public domain, waters, minerals, coal, petroleum, and other
mineral oils, all forces of potential energy, fisheries, forests or timber, wildlife, flora and fauna,
and other natural resources are owned by the State. With the exception of agricultural
lands, all other natural resources shall not be alienated. x x x.

xxx

Section 3. x x x Alienable lands of the public domain shall be limited to agricultural


lands. Private corporations or associations may not hold such alienable lands of the
public domain except by lease, x x x."(Emphasis supplied)

Classification of Reclaimed Foreshore and Submerged Areas


362

PEA readily concedes that lands reclaimed from foreshore or submerged areas of Manila Bay are
alienable or disposable lands of the public domain. In its Memorandum,67 PEA admits that –

"Under the Public Land Act (CA 141, as amended), reclaimed lands are classified as
alienable and disposable lands of the public domain:

'Sec. 59. The lands disposable under this title shall be classified as follows:

(a) Lands reclaimed by the government by dredging, filling, or other means;

x x x.'" (Emphasis supplied)

Likewise, the Legal Task Force68 constituted under Presidential Administrative Order No. 365
admitted in its Report and Recommendation to then President Fidel V. Ramos, "[R]eclaimed lands
are classified as alienable and disposable lands of the public domain."69 The Legal Task Force
concluded that –

"D. Conclusion

Reclaimed lands are lands of the public domain. However, by statutory authority, the rights of
ownership and disposition over reclaimed lands have been transferred to PEA, by virtue of
which PEA, as owner, may validly convey the same to any qualified person without violating
the Constitution or any statute.

The constitutional provision prohibiting private corporations from holding public land, except
by lease (Sec. 3, Art. XVII,70 1987 Constitution), does not apply to reclaimed lands whose
ownership has passed on to PEA by statutory grant."

Under Section 2, Article XII of the 1987 Constitution, the foreshore and submerged areas of Manila
Bay are part of the "lands of the public domain, waters x x x and other natural resources" and
consequently "owned by the State." As such, foreshore and submerged areas "shall not be alienated,"
unless they are classified as "agricultural lands" of the public domain. The mere reclamation of these
areas by PEA does not convert these inalienable natural resources of the State into alienable or
disposable lands of the public domain. There must be a law or presidential proclamation officially
classifying these reclaimed lands as alienable or disposable and open to disposition or concession.
Moreover, these reclaimed lands cannot be classified as alienable or disposable if the law has
reserved them for some public or quasi-public use.71

Section 8 of CA No. 141 provides that "only those lands shall be declared open to disposition or
concession which have been officially delimited and classified."72 The President has the authority
to classify inalienable lands of the public domain into alienable or disposable lands of the public
domain, pursuant to Section 6 of CA No. 141. In Laurel vs. Garcia,73 the Executive Department
attempted to sell the Roppongi property in Tokyo, Japan, which was acquired by the Philippine
Government for use as the Chancery of the Philippine Embassy. Although the Chancery had
transferred to another location thirteen years earlier, the Court still ruled that, under Article 422 74 of
the Civil Code, a property of public dominion retains such character until formally declared otherwise.
The Court ruled that –

"The fact that the Roppongi site has not been used for a long time for actual Embassy service
does not automatically convert it to patrimonial property. Any such conversion happens only if
the property is withdrawn from public use (Cebu Oxygen and Acetylene Co. v. Bercilles, 66
SCRA 481 [1975]. A property continues to be part of the public domain, not available
for private appropriation or ownership 'until there is a formal declaration on the part of
the government to withdraw it from being such' (Ignacio v. Director of Lands, 108 Phil.
335 [1960]." (Emphasis supplied)

PD No. 1085, issued on February 4, 1977, authorized the issuance of special land patents for lands
reclaimed by PEA from the foreshore or submerged areas of Manila Bay. On January 19, 1988 then
363

President Corazon C. Aquino issued Special Patent No. 3517 in the name of PEA for the 157.84
hectares comprising the partially reclaimed Freedom Islands. Subsequently, on April 9, 1999 the
Register of Deeds of the Municipality of Paranaque issued TCT Nos. 7309, 7311 and 7312 in the
name of PEA pursuant to Section 103 of PD No. 1529 authorizing the issuance of certificates of title
corresponding to land patents. To this day, these certificates of title are still in the name of PEA.

PD No. 1085, coupled with President Aquino's actual issuance of a special patent covering the
Freedom Islands, is equivalent to an official proclamation classifying the Freedom Islands as alienable
or disposable lands of the public domain. PD No. 1085 and President Aquino's issuance of a land
patent also constitute a declaration that the Freedom Islands are no longer needed for public
service. The Freedom Islands are thus alienable or disposable lands of the public domain,
open to disposition or concession to qualified parties.

At the time then President Aquino issued Special Patent No. 3517, PEA had already reclaimed the
Freedom Islands although subsequently there were partial erosions on some areas. The government
had also completed the necessary surveys on these islands. Thus, the Freedom Islands were no
longer part of Manila Bay but part of the land mass. Section 3, Article XII of the 1987 Constitution
classifies lands of the public domain into "agricultural, forest or timber, mineral lands, and national
parks." Being neither timber, mineral, nor national park lands, the reclaimed Freedom Islands
necessarily fall under the classification of agricultural lands of the public domain. Under the 1987
Constitution, agricultural lands of the public domain are the only natural resources that the State may
alienate to qualified private parties. All other natural resources, such as the seas or bays, are "waters
x x x owned by the State" forming part of the public domain, and are inalienable pursuant to Section
2, Article XII of the 1987 Constitution.

AMARI claims that the Freedom Islands are private lands because CDCP, then a private corporation,
reclaimed the islands under a contract dated November 20, 1973 with the Commissioner of Public
Highways. AMARI, citing Article 5 of the Spanish Law of Waters of 1866, argues that "if the ownership
of reclaimed lands may be given to the party constructing the works, then it cannot be said that
reclaimed lands are lands of the public domain which the State may not alienate."75 Article 5 of the
Spanish Law of Waters reads as follows:

"Article 5. Lands reclaimed from the sea in consequence of works constructed by the State, or
by the provinces, pueblos or private persons, with proper permission, shall become the
property of the party constructing such works, unless otherwise provided by the terms of
the grant of authority." (Emphasis supplied)

Under Article 5 of the Spanish Law of Waters of 1866, private parties could reclaim from the sea only
with "proper permission" from the State. Private parties could own the reclaimed land only if not
"otherwise provided by the terms of the grant of authority." This clearly meant that no one could
reclaim from the sea without permission from the State because the sea is property of public
dominion. It also meant that the State could grant or withhold ownership of the reclaimed land
because any reclaimed land, like the sea from which it emerged, belonged to the State. Thus, a
private person reclaiming from the sea without permission from the State could not acquire ownership
of the reclaimed land which would remain property of public dominion like the sea it replaced. 76 Article
5 of the Spanish Law of Waters of 1866 adopted the time-honored principle of land ownership that "all
lands that were not acquired from the government, either by purchase or by grant, belong to the
public domain."77

Article 5 of the Spanish Law of Waters must be read together with laws subsequently enacted on the
disposition of public lands. In particular, CA No. 141 requires that lands of the public domain must first
be classified as alienable or disposable before the government can alienate them. These lands must
not be reserved for public or quasi-public purposes.78 Moreover, the contract between CDCP and the
government was executed after the effectivity of the 1973 Constitution which barred private
corporations from acquiring any kind of alienable land of the public domain. This contract could not
have converted the Freedom Islands into private lands of a private corporation.
364

Presidential Decree No. 3-A, issued on January 11, 1973, revoked all laws authorizing the
reclamation of areas under water and revested solely in the National Government the power to
reclaim lands. Section 1 of PD No. 3-A declared that –

"The provisions of any law to the contrary notwithstanding, the reclamation of areas
under water, whether foreshore or inland, shall be limited to the National Government or
any person authorized by it under a proper contract. (Emphasis supplied)

x x x."

PD No. 3-A repealed Section 5 of the Spanish Law of Waters of 1866 because reclamation of areas
under water could now be undertaken only by the National Government or by a person contracted by
the National Government. Private parties may reclaim from the sea only under a contract with the
National Government, and no longer by grant or permission as provided in Section 5 of the Spanish
Law of Waters of 1866.

Executive Order No. 525, issued on February 14, 1979, designated PEA as the National
Government's implementing arm to undertake "all reclamation projects of the government," which
"shall be undertaken by the PEA or through a proper contract executed by it with any person
or entity." Under such contract, a private party receives compensation for reclamation services
rendered to PEA. Payment to the contractor may be in cash, or in kind consisting of portions of the
reclaimed land, subject to the constitutional ban on private corporations from acquiring alienable lands
of the public domain. The reclaimed land can be used as payment in kind only if the reclaimed land is
first classified as alienable or disposable land open to disposition, and then declared no longer
needed for public service.

The Amended JVA covers not only the Freedom Islands, but also an additional 592.15 hectares which
are still submerged and forming part of Manila Bay. There is no legislative or Presidential act
classifying these submerged areas as alienable or disposable lands of the public domain open
to disposition. These submerged areas are not covered by any patent or certificate of title. There
can be no dispute that these submerged areas form part of the public domain, and in their present
state are inalienable and outside the commerce of man. Until reclaimed from the sea, these
submerged areas are, under the Constitution, "waters x x x owned by the State," forming part of the
public domain and consequently inalienable. Only when actually reclaimed from the sea can these
submerged areas be classified as public agricultural lands, which under the Constitution are the only
natural resources that the State may alienate. Once reclaimed and transformed into public agricultural
lands, the government may then officially classify these lands as alienable or disposable lands open
to disposition. Thereafter, the government may declare these lands no longer needed for public
service. Only then can these reclaimed lands be considered alienable or disposable lands of the
public domain and within the commerce of man.

The classification of PEA's reclaimed foreshore and submerged lands into alienable or disposable
lands open to disposition is necessary because PEA is tasked under its charter to undertake public
services that require the use of lands of the public domain. Under Section 5 of PD No. 1084, the
functions of PEA include the following: "[T]o own or operate railroads, tramways and other kinds of
land transportation, x x x; [T]o construct, maintain and operate such systems of sanitary sewers as
may be necessary; [T]o construct, maintain and operate such storm drains as may be necessary."
PEA is empowered to issue "rules and regulations as may be necessary for the proper use by private
parties of any or all of the highways, roads, utilities, buildings and/or any of its properties and
to impose or collect fees or tolls for their use." Thus, part of the reclaimed foreshore and submerged
lands held by the PEA would actually be needed for public use or service since many of the functions
imposed on PEA by its charter constitute essential public services.

Moreover, Section 1 of Executive Order No. 525 provides that PEA "shall be primarily responsible for
integrating, directing, and coordinating all reclamation projects for and on behalf of the National
Government." The same section also states that "[A]ll reclamation projects shall be approved by the
President upon recommendation of the PEA, and shall be undertaken by the PEA or through a proper
contract executed by it with any person or entity; x x x." Thus, under EO No. 525, in relation to PD No.
3-A and PD No.1084, PEA became the primary implementing agency of the National Government to
365

reclaim foreshore and submerged lands of the public domain. EO No. 525 recognized PEA as the
government entity "to undertake the reclamation of lands and ensure their maximum utilization
in promoting public welfare and interests."79 Since large portions of these reclaimed lands would
obviously be needed for public service, there must be a formal declaration segregating reclaimed
lands no longer needed for public service from those still needed for public service.1âwphi1.nêt

Section 3 of EO No. 525, by declaring that all lands reclaimed by PEA "shall belong to or be owned by
the PEA," could not automatically operate to classify inalienable lands into alienable or disposable
lands of the public domain. Otherwise, reclaimed foreshore and submerged lands of the public
domain would automatically become alienable once reclaimed by PEA, whether or not classified as
alienable or disposable.

The Revised Administrative Code of 1987, a later law than either PD No. 1084 or EO No. 525, vests
in the Department of Environment and Natural Resources ("DENR" for brevity) the following powers
and functions:

"Sec. 4. Powers and Functions. The Department shall:

(1) x x x

xxx

(4) Exercise supervision and control over forest lands, alienable and disposable public
lands, mineral resources and, in the process of exercising such control, impose appropriate
taxes, fees, charges, rentals and any such form of levy and collect such revenues for the
exploration, development, utilization or gathering of such resources;

xxx

(14) Promulgate rules, regulations and guidelines on the issuance of licenses, permits,
concessions, lease agreements and such other privileges concerning the
development, exploration and utilization of the country's marine, freshwater, and
brackish water and over all aquatic resources of the country and shall continue to
oversee, supervise and police our natural resources; cancel or cause to cancel such
privileges upon failure, non-compliance or violations of any regulation, order, and for all other
causes which are in furtherance of the conservation of natural resources and supportive of
the national interest;

(15) Exercise exclusive jurisdiction on the management and disposition of all lands of
the public domain and serve as the sole agency responsible for classification, sub-
classification, surveying and titling of lands in consultation with appropriate
agencies."80 (Emphasis supplied)

As manager, conservator and overseer of the natural resources of the State, DENR exercises
"supervision and control over alienable and disposable public lands." DENR also exercises "exclusive
jurisdiction on the management and disposition of all lands of the public domain." Thus, DENR
decides whether areas under water, like foreshore or submerged areas of Manila Bay, should be
reclaimed or not. This means that PEA needs authorization from DENR before PEA can undertake
reclamation projects in Manila Bay, or in any part of the country.

DENR also exercises exclusive jurisdiction over the disposition of all lands of the public domain.
Hence, DENR decides whether reclaimed lands of PEA should be classified as alienable under
Sections 681 and 782 of CA No. 141. Once DENR decides that the reclaimed lands should be so
classified, it then recommends to the President the issuance of a proclamation classifying the lands
as alienable or disposable lands of the public domain open to disposition. We note that then DENR
Secretary Fulgencio S. Factoran, Jr. countersigned Special Patent No. 3517 in compliance with the
Revised Administrative Code and Sections 6 and 7 of CA No. 141.
366

In short, DENR is vested with the power to authorize the reclamation of areas under water, while PEA
is vested with the power to undertake the physical reclamation of areas under water, whether directly
or through private contractors. DENR is also empowered to classify lands of the public domain into
alienable or disposable lands subject to the approval of the President. On the other hand, PEA is
tasked to develop, sell or lease the reclaimed alienable lands of the public domain.

Clearly, the mere physical act of reclamation by PEA of foreshore or submerged areas does not make
the reclaimed lands alienable or disposable lands of the public domain, much less patrimonial lands of
PEA. Likewise, the mere transfer by the National Government of lands of the public domain to PEA
does not make the lands alienable or disposable lands of the public domain, much less patrimonial
lands of PEA.

Absent two official acts – a classification that these lands are alienable or disposable and open to
disposition and a declaration that these lands are not needed for public service, lands reclaimed by
PEA remain inalienable lands of the public domain. Only such an official classification and formal
declaration can convert reclaimed lands into alienable or disposable lands of the public domain, open
to disposition under the Constitution, Title I and Title III 83 of CA No. 141 and other applicable laws.84

PEA's Authority to Sell Reclaimed Lands

PEA, like the Legal Task Force, argues that as alienable or disposable lands of the public domain, the
reclaimed lands shall be disposed of in accordance with CA No. 141, the Public Land Act. PEA, citing
Section 60 of CA No. 141, admits that reclaimed lands transferred to a branch or subdivision of the
government "shall not be alienated, encumbered, or otherwise disposed of in a manner affecting its
title, except when authorized by Congress: x x x."85 (Emphasis by PEA)

In Laurel vs. Garcia,86 the Court cited Section 48 of the Revised Administrative Code of 1987, which
states that –

"Sec. 48. Official Authorized to Convey Real Property. Whenever real property of the
Government is authorized by law to be conveyed, the deed of conveyance shall be
executed in behalf of the government by the following: x x x."

Thus, the Court concluded that a law is needed to convey any real property belonging to the
Government. The Court declared that -

"It is not for the President to convey real property of the government on his or her own sole
will. Any such conveyance must be authorized and approved by a law enacted by the
Congress. It requires executive and legislative concurrence." (Emphasis supplied)

PEA contends that PD No. 1085 and EO No. 525 constitute the legislative authority allowing PEA to
sell its reclaimed lands. PD No. 1085, issued on February 4, 1977, provides that –

"The land reclaimed in the foreshore and offshore area of Manila Bay pursuant to the
contract for the reclamation and construction of the Manila-Cavite Coastal Road Project
between the Republic of the Philippines and the Construction and Development Corporation
of the Philippines dated November 20, 1973 and/or any other contract or reclamation
covering the same area is hereby transferred, conveyed and assigned to the ownership
and administration of the Public Estates Authority established pursuant to PD No. 1084;
Provided, however, That the rights and interests of the Construction and Development
Corporation of the Philippines pursuant to the aforesaid contract shall be recognized and
respected.

Henceforth, the Public Estates Authority shall exercise the rights and assume the obligations
of the Republic of the Philippines (Department of Public Highways) arising from, or incident
to, the aforesaid contract between the Republic of the Philippines and the Construction and
Development Corporation of the Philippines.
367

In consideration of the foregoing transfer and assignment, the Public Estates Authority shall
issue in favor of the Republic of the Philippines the corresponding shares of stock in said
entity with an issued value of said shares of stock (which) shall be deemed fully paid and non-
assessable.

The Secretary of Public Highways and the General Manager of the Public Estates Authority
shall execute such contracts or agreements, including appropriate agreements with the
Construction and Development Corporation of the Philippines, as may be necessary to
implement the above.

Special land patent/patents shall be issued by the Secretary of Natural Resources in


favor of the Public Estates Authority without prejudice to the subsequent transfer to
the contractor or his assignees of such portion or portions of the land reclaimed or to
be reclaimed as provided for in the above-mentioned contract. On the basis of such
patents, the Land Registration Commission shall issue the corresponding certificate of
title." (Emphasis supplied)

On the other hand, Section 3 of EO No. 525, issued on February 14, 1979, provides that -

"Sec. 3. All lands reclaimed by PEA shall belong to or be owned by the PEA which shall
be responsible for its administration, development, utilization or disposition in accordance with
the provisions of Presidential Decree No. 1084. Any and all income that the PEA may derive
from the sale, lease or use of reclaimed lands shall be used in accordance with the provisions
of Presidential Decree No. 1084."

There is no express authority under either PD No. 1085 or EO No. 525 for PEA to sell its reclaimed
lands. PD No. 1085 merely transferred "ownership and administration" of lands reclaimed from Manila
Bay to PEA, while EO No. 525 declared that lands reclaimed by PEA "shall belong to or be owned by
PEA." EO No. 525 expressly states that PEA should dispose of its reclaimed lands "in accordance
with the provisions of Presidential Decree No. 1084," the charter of PEA.

PEA's charter, however, expressly tasks PEA "to develop, improve, acquire, administer, deal in,
subdivide, dispose, lease and sell any and all kinds of lands x x x owned, managed, controlled
and/or operated by the government."87(Emphasis supplied) There is, therefore, legislative authority
granted to PEA to sell its lands, whether patrimonial or alienable lands of the public domain.
PEA may sell to private parties its patrimonial propertiesin accordance with the PEA charter free
from constitutional limitations. The constitutional ban on private corporations from acquiring alienable
lands of the public domain does not apply to the sale of PEA's patrimonial lands.

PEA may also sell its alienable or disposable lands of the public domain to private individuals
since, with the legislative authority, there is no longer any statutory prohibition against such sales and
the constitutional ban does not apply to individuals. PEA, however, cannot sell any of its alienable or
disposable lands of the public domain to private corporations since Section 3, Article XII of the 1987
Constitution expressly prohibits such sales. The legislative authority benefits only individuals. Private
corporations remain barred from acquiring any kind of alienable land of the public domain, including
government reclaimed lands.

The provision in PD No. 1085 stating that portions of the reclaimed lands could be transferred by PEA
to the "contractor or his assignees" (Emphasis supplied) would not apply to private corporations but
only to individuals because of the constitutional ban. Otherwise, the provisions of PD No. 1085 would
violate both the 1973 and 1987 Constitutions.

The requirement of public auction in the sale of reclaimed lands

Assuming the reclaimed lands of PEA are classified as alienable or disposable lands open to
disposition, and further declared no longer needed for public service, PEA would have to conduct a
public bidding in selling or leasing these lands. PEA must observe the provisions of Sections 63 and
67 of CA No. 141 requiring public auction, in the absence of a law exempting PEA from holding a
368

public auction.88 Special Patent No. 3517 expressly states that the patent is issued by authority of the
Constitution and PD No. 1084, "supplemented by Commonwealth Act No. 141, as amended." This is
an acknowledgment that the provisions of CA No. 141 apply to the disposition of reclaimed alienable
lands of the public domain unless otherwise provided by law. Executive Order No. 654,89 which
authorizes PEA "to determine the kind and manner of payment for the transfer" of its assets and
properties, does not exempt PEA from the requirement of public auction. EO No. 654 merely
authorizes PEA to decide the mode of payment, whether in kind and in installment, but does not
authorize PEA to dispense with public auction.

Moreover, under Section 79 of PD No. 1445, otherwise known as the Government Auditing Code, the
government is required to sell valuable government property through public bidding. Section 79 of PD
No. 1445 mandates that –

"Section 79. When government property has become unserviceable for any cause, or is no
longer needed, it shall, upon application of the officer accountable therefor, be inspected by
the head of the agency or his duly authorized representative in the presence of the auditor
concerned and, if found to be valueless or unsaleable, it may be destroyed in their
presence. If found to be valuable, it may be sold at public auction to the highest
bidder under the supervision of the proper committee on award or similar body in the
presence of the auditor concerned or other authorized representative of the
Commission, after advertising by printed notice in the Official Gazette, or for not less
than three consecutive days in any newspaper of general circulation, or where the value
of the property does not warrant the expense of publication, by notices posted for a like
period in at least three public places in the locality where the property is to be sold. In the
event that the public auction fails, the property may be sold at a private sale at such
price as may be fixed by the same committee or body concerned and approved by the
Commission."

It is only when the public auction fails that a negotiated sale is allowed, in which case the Commission
on Audit must approve the selling price.90 The Commission on Audit implements Section 79 of the
Government Auditing Code through Circular No. 89-29691 dated January 27, 1989. This circular
emphasizes that government assets must be disposed of only through public auction, and a
negotiated sale can be resorted to only in case of "failure of public auction."

At the public auction sale, only Philippine citizens are qualified to bid for PEA's reclaimed foreshore
and submerged alienable lands of the public domain. Private corporations are barred from bidding at
the auction sale of any kind of alienable land of the public domain.

PEA originally scheduled a public bidding for the Freedom Islands on December 10, 1991. PEA
imposed a condition that the winning bidder should reclaim another 250 hectares of submerged areas
to regularize the shape of the Freedom Islands, under a 60-40 sharing of the additional reclaimed
areas in favor of the winning bidder.92 No one, however, submitted a bid. On December 23, 1994, the
Government Corporate Counsel advised PEA it could sell the Freedom Islands through negotiation,
without need of another public bidding, because of the failure of the public bidding on December 10,
1991.93

However, the original JVA dated April 25, 1995 covered not only the Freedom Islands and the
additional 250 hectares still to be reclaimed, it also granted an option to AMARI to reclaim another
350 hectares. The original JVA, a negotiated contract, enlarged the reclamation area to 750
hectares.94 The failure of public bidding on December 10, 1991, involving only 407.84 hectares, 95 is
not a valid justification for a negotiated sale of 750 hectares, almost double the area publicly
auctioned. Besides, the failure of public bidding happened on December 10, 1991, more than three
years before the signing of the original JVA on April 25, 1995. The economic situation in the country
had greatly improved during the intervening period.

Reclamation under the BOT Law and the Local Government Code
369

The constitutional prohibition in Section 3, Article XII of the 1987 Constitution is absolute and clear:
"Private corporations or associations may not hold such alienable lands of the public domain except
by lease, x x x." Even Republic Act No. 6957 ("BOT Law," for brevity), cited by PEA and AMARI as
legislative authority to sell reclaimed lands to private parties, recognizes the constitutional ban.
Section 6 of RA No. 6957 states –

"Sec. 6. Repayment Scheme. - For the financing, construction, operation and maintenance of
any infrastructure projects undertaken through the build-operate-and-transfer arrangement or
any of its variations pursuant to the provisions of this Act, the project proponent x x x may
likewise be repaid in the form of a share in the revenue of the project or other non-monetary
payments, such as, but not limited to, the grant of a portion or percentage of the reclaimed
land, subject to the constitutional requirements with respect to the ownership of the
land: x x x." (Emphasis supplied)

A private corporation, even one that undertakes the physical reclamation of a government BOT
project, cannot acquire reclaimed alienable lands of the public domain in view of the constitutional
ban.

Section 302 of the Local Government Code, also mentioned by PEA and AMARI, authorizes local
governments in land reclamation projects to pay the contractor or developer in kind consisting of a
percentage of the reclaimed land, to wit:

"Section 302. Financing, Construction, Maintenance, Operation, and Management of


Infrastructure Projects by the Private Sector. x x x

xxx

In case of land reclamation or construction of industrial estates, the repayment plan may
consist of the grant of a portion or percentage of the reclaimed land or the industrial estate
constructed."

Although Section 302 of the Local Government Code does not contain a proviso similar to that of the
BOT Law, the constitutional restrictions on land ownership automatically apply even though not
expressly mentioned in the Local Government Code.

Thus, under either the BOT Law or the Local Government Code, the contractor or developer, if a
corporate entity, can only be paid with leaseholds on portions of the reclaimed land. If the contractor
or developer is an individual, portions of the reclaimed land, not exceeding 12 hectares 96 of non-
agricultural lands, may be conveyed to him in ownership in view of the legislative authority allowing
such conveyance. This is the only way these provisions of the BOT Law and the Local Government
Code can avoid a direct collision with Section 3, Article XII of the 1987 Constitution.

Registration of lands of the public domain

Finally, PEA theorizes that the "act of conveying the ownership of the reclaimed lands to public
respondent PEA transformed such lands of the public domain to private lands." This theory is echoed
by AMARI which maintains that the "issuance of the special patent leading to the eventual issuance of
title takes the subject land away from the land of public domain and converts the property into
patrimonial or private property." In short, PEA and AMARI contend that with the issuance of Special
Patent No. 3517 and the corresponding certificates of titles, the 157.84 hectares comprising the
Freedom Islands have become private lands of PEA. In support of their theory, PEA and AMARI cite
the following rulings of the Court:

1. Sumail v. Judge of CFI of Cotabato,97 where the Court held –

"Once the patent was granted and the corresponding certificate of title was issued, the land
ceased to be part of the public domain and became private property over which the Director
of Lands has neither control nor jurisdiction."
370

2. Lee Hong Hok v. David,98 where the Court declared -

"After the registration and issuance of the certificate and duplicate certificate of title based on
a public land patent, the land covered thereby automatically comes under the operation of
Republic Act 496 subject to all the safeguards provided therein."3. Heirs of Gregorio Tengco
v. Heirs of Jose Aliwalas,99 where the Court ruled -

"While the Director of Lands has the power to review homestead patents, he may do so only
so long as the land remains part of the public domain and continues to be under his exclusive
control; but once the patent is registered and a certificate of title is issued, the land ceases to
be part of the public domain and becomes private property over which the Director of Lands
has neither control nor jurisdiction."

4. Manalo v. Intermediate Appellate Court,100 where the Court held –

"When the lots in dispute were certified as disposable on May 19, 1971, and free patents
were issued covering the same in favor of the private respondents, the said lots ceased to be
part of the public domain and, therefore, the Director of Lands lost jurisdiction over the same."

5.Republic v. Court of Appeals,101 where the Court stated –

"Proclamation No. 350, dated October 9, 1956, of President Magsaysay legally effected a
land grant to the Mindanao Medical Center, Bureau of Medical Services, Department of
Health, of the whole lot, validly sufficient for initial registration under the Land Registration
Act. Such land grant is constitutive of a 'fee simple' title or absolute title in favor of petitioner
Mindanao Medical Center. Thus, Section 122 of the Act, which governs the registration of
grants or patents involving public lands, provides that 'Whenever public lands in the Philippine
Islands belonging to the Government of the United States or to the Government of the
Philippines are alienated, granted or conveyed to persons or to public or private corporations,
the same shall be brought forthwith under the operation of this Act (Land Registration Act, Act
496) and shall become registered lands.'"

The first four cases cited involve petitions to cancel the land patents and the corresponding
certificates of titles issued to private parties. These four cases uniformly hold that the Director of
Lands has no jurisdiction over private lands or that upon issuance of the certificate of title the land
automatically comes under the Torrens System. The fifth case cited involves the registration under
the Torrens System of a 12.8-hectare public land granted by the National Government to Mindanao
Medical Center, a government unit under the Department of Health. The National Government
transferred the 12.8-hectare public land to serve as the site for the hospital buildings and other
facilities of Mindanao Medical Center, which performed a public service. The Court affirmed the
registration of the 12.8-hectare public land in the name of Mindanao Medical Center under Section
122 of Act No. 496. This fifth case is an example of a public land being registered under Act No. 496
without the land losing its character as a property of public dominion.

In the instant case, the only patent and certificates of title issued are those in the name of PEA, a
wholly government owned corporation performing public as well as proprietary functions. No patent or
certificate of title has been issued to any private party. No one is asking the Director of Lands to
cancel PEA's patent or certificates of title. In fact, the thrust of the instant petition is that PEA's
certificates of title should remain with PEA, and the land covered by these certificates, being alienable
lands of the public domain, should not be sold to a private corporation.

Registration of land under Act No. 496 or PD No. 1529 does not vest in the registrant private or public
ownership of the land. Registration is not a mode of acquiring ownership but is merely evidence of
ownership previously conferred by any of the recognized modes of acquiring ownership. Registration
does not give the registrant a better right than what the registrant had prior to the registration. 102 The
registration of lands of the public domain under the Torrens system, by itself, cannot convert public
lands into private lands.103
371

Jurisprudence holding that upon the grant of the patent or issuance of the certificate of title the
alienable land of the public domain automatically becomes private land cannot apply to government
units and entities like PEA. The transfer of the Freedom Islands to PEA was made subject to the
provisions of CA No. 141 as expressly stated in Special Patent No. 3517 issued by then President
Aquino, to wit:

"NOW, THEREFORE, KNOW YE, that by authority of the Constitution of the Philippines and
in conformity with the provisions of Presidential Decree No. 1084, supplemented by
Commonwealth Act No. 141, as amended, there are hereby granted and conveyed unto the
Public Estates Authority the aforesaid tracts of land containing a total area of one million nine
hundred fifteen thousand eight hundred ninety four (1,915,894) square meters; the technical
description of which are hereto attached and made an integral part hereof." (Emphasis
supplied)

Thus, the provisions of CA No. 141 apply to the Freedom Islands on matters not covered by PD No.
1084. Section 60 of CA No. 141 prohibits, "except when authorized by Congress," the sale of
alienable lands of the public domain that are transferred to government units or entities. Section 60 of
CA No. 141 constitutes, under Section 44 of PD No. 1529, a "statutory lien affecting title" of the
registered land even if not annotated on the certificate of title.104Alienable lands of the public domain
held by government entities under Section 60 of CA No. 141 remain public lands because they cannot
be alienated or encumbered unless Congress passes a law authorizing their disposition. Congress,
however, cannot authorize the sale to private corporations of reclaimed alienable lands of the public
domain because of the constitutional ban. Only individuals can benefit from such law.

The grant of legislative authority to sell public lands in accordance with Section 60 of CA No. 141
does not automatically convert alienable lands of the public domain into private or patrimonial lands.
The alienable lands of the public domain must be transferred to qualified private parties, or to
government entities not tasked to dispose of public lands, before these lands can become private or
patrimonial lands. Otherwise, the constitutional ban will become illusory if Congress can declare lands
of the public domain as private or patrimonial lands in the hands of a government agency tasked to
dispose of public lands. This will allow private corporations to acquire directly from government
agencies limitless areas of lands which, prior to such law, are concededly public lands.

Under EO No. 525, PEA became the central implementing agency of the National Government to
reclaim foreshore and submerged areas of the public domain. Thus, EO No. 525 declares that –

"EXECUTIVE ORDER NO. 525

Designating the Public Estates Authority as the Agency Primarily Responsible for all
Reclamation Projects

Whereas, there are several reclamation projects which are ongoing or being proposed to be
undertaken in various parts of the country which need to be evaluated for consistency with
national programs;

Whereas, there is a need to give further institutional support to the Government's declared
policy to provide for a coordinated, economical and efficient reclamation of lands;

Whereas, Presidential Decree No. 3-A requires that all reclamation of areas shall be limited to
the National Government or any person authorized by it under proper contract;

Whereas, a central authority is needed to act on behalf of the National Government


which shall ensure a coordinated and integrated approach in the reclamation of lands;

Whereas, Presidential Decree No. 1084 creates the Public Estates Authority as a
government corporation to undertake reclamation of lands and ensure their maximum
utilization in promoting public welfare and interests; and
372

Whereas, Presidential Decree No. 1416 provides the President with continuing authority to
reorganize the national government including the transfer, abolition, or merger of functions
and offices.

NOW, THEREFORE, I, FERDINAND E. MARCOS, President of the Philippines, by virtue of


the powers vested in me by the Constitution and pursuant to Presidential Decree No. 1416,
do hereby order and direct the following:

Section 1. The Public Estates Authority (PEA) shall be primarily responsible for
integrating, directing, and coordinating all reclamation projects for and on behalf of the
National Government. All reclamation projects shall be approved by the President upon
recommendation of the PEA, and shall be undertaken by the PEA or through a proper
contract executed by it with any person or entity; Provided, that, reclamation projects of any
national government agency or entity authorized under its charter shall be undertaken in
consultation with the PEA upon approval of the President.

x x x ."

As the central implementing agency tasked to undertake reclamation projects nationwide, with
authority to sell reclaimed lands, PEA took the place of DENR as the government agency charged
with leasing or selling reclaimed lands of the public domain. The reclaimed lands being leased or sold
by PEA are not private lands, in the same manner that DENR, when it disposes of other alienable
lands, does not dispose of private lands but alienable lands of the public domain. Only when qualified
private parties acquire these lands will the lands become private lands. In the hands of the
government agency tasked and authorized to dispose of alienable of disposable lands of the
public domain, these lands are still public, not private lands.

Furthermore, PEA's charter expressly states that PEA "shall hold lands of the public domain" as
well as "any and all kinds of lands." PEA can hold both lands of the public domain and private lands.
Thus, the mere fact that alienable lands of the public domain like the Freedom Islands are transferred
to PEA and issued land patents or certificates of title in PEA's name does not automatically make
such lands private.

To allow vast areas of reclaimed lands of the public domain to be transferred to PEA as private lands
will sanction a gross violation of the constitutional ban on private corporations from acquiring any kind
of alienable land of the public domain. PEA will simply turn around, as PEA has now done under the
Amended JVA, and transfer several hundreds of hectares of these reclaimed and still to be reclaimed
lands to a single private corporation in only one transaction. This scheme will effectively nullify the
constitutional ban in Section 3, Article XII of the 1987 Constitution which was intended to diffuse
equitably the ownership of alienable lands of the public domain among Filipinos, now numbering over
80 million strong.

This scheme, if allowed, can even be applied to alienable agricultural lands of the public domain since
PEA can "acquire x x x any and all kinds of lands." This will open the floodgates to corporations and
even individuals acquiring hundreds of hectares of alienable lands of the public domain under the
guise that in the hands of PEA these lands are private lands. This will result in corporations amassing
huge landholdings never before seen in this country - creating the very evil that the constitutional ban
was designed to prevent. This will completely reverse the clear direction of constitutional development
in this country. The 1935 Constitution allowed private corporations to acquire not more than 1,024
hectares of public lands.105 The 1973 Constitution prohibited private corporations from acquiring any
kind of public land, and the 1987 Constitution has unequivocally reiterated this prohibition.

The contention of PEA and AMARI that public lands, once registered under Act No. 496 or PD No.
1529, automatically become private lands is contrary to existing laws. Several laws authorize lands of
the public domain to be registered under the Torrens System or Act No. 496, now PD No. 1529,
without losing their character as public lands. Section 122 of Act No. 496, and Section 103 of PD No.
1529, respectively, provide as follows:
373

Act No. 496

"Sec. 122. Whenever public lands in the Philippine Islands belonging to the x x x Government
of the Philippine Islands are alienated, granted, or conveyed to persons or the public or
private corporations, the same shall be brought forthwith under the operation of this Act and
shall become registered lands."

PD No. 1529

"Sec. 103. Certificate of Title to Patents. Whenever public land is by the Government
alienated, granted or conveyed to any person, the same shall be brought forthwith under the
operation of this Decree." (Emphasis supplied)

Based on its legislative history, the phrase "conveyed to any person" in Section 103 of PD No. 1529
includes conveyances of public lands to public corporations.

Alienable lands of the public domain "granted, donated, or transferred to a province, municipality, or
branch or subdivision of the Government," as provided in Section 60 of CA No. 141, may be
registered under the Torrens System pursuant to Section 103 of PD No. 1529. Such registration,
however, is expressly subject to the condition in Section 60 of CA No. 141 that the land "shall not be
alienated, encumbered or otherwise disposed of in a manner affecting its title, except when
authorized by Congress." This provision refers to government reclaimed, foreshore and marshy
lands of the public domain that have been titled but still cannot be alienated or encumbered unless
expressly authorized by Congress. The need for legislative authority prevents the registered land of
the public domain from becoming private land that can be disposed of to qualified private parties.

The Revised Administrative Code of 1987 also recognizes that lands of the public domain may be
registered under the Torrens System. Section 48, Chapter 12, Book I of the Code states –

"Sec. 48. Official Authorized to Convey Real Property. Whenever real property of the
Government is authorized by law to be conveyed, the deed of conveyance shall be executed
in behalf of the government by the following:

(1) x x x

(2) For property belonging to the Republic of the Philippines, but titled in the name of
any political subdivision or of any corporate agency or instrumentality, by the executive
head of the agency or instrumentality." (Emphasis supplied)

Thus, private property purchased by the National Government for expansion of a public wharf may be
titled in the name of a government corporation regulating port operations in the country. Private
property purchased by the National Government for expansion of an airport may also be titled in the
name of the government agency tasked to administer the airport. Private property donated to a
municipality for use as a town plaza or public school site may likewise be titled in the name of the
municipality.106 All these properties become properties of the public domain, and if already registered
under Act No. 496 or PD No. 1529, remain registered land. There is no requirement or provision in
any existing law for the de-registration of land from the Torrens System.

Private lands taken by the Government for public use under its power of eminent domain become
unquestionably part of the public domain. Nevertheless, Section 85 of PD No. 1529 authorizes the
Register of Deeds to issue in the name of the National Government new certificates of title covering
such expropriated lands. Section 85 of PD No. 1529 states –

"Sec. 85. Land taken by eminent domain. Whenever any registered land, or interest therein, is
expropriated or taken by eminent domain, the National Government, province, city or
municipality, or any other agency or instrumentality exercising such right shall file for
registration in the proper Registry a certified copy of the judgment which shall state definitely
by an adequate description, the particular property or interest expropriated, the number of the
374

certificate of title, and the nature of the public use. A memorandum of the right or interest
taken shall be made on each certificate of title by the Register of Deeds, and where the fee
simple is taken, a new certificate shall be issued in favor of the National Government,
province, city, municipality, or any other agency or instrumentality exercising such right for
the land so taken. The legal expenses incident to the memorandum of registration or
issuance of a new certificate of title shall be for the account of the authority taking the land or
interest therein." (Emphasis supplied)

Consequently, lands registered under Act No. 496 or PD No. 1529 are not exclusively private or
patrimonial lands. Lands of the public domain may also be registered pursuant to existing laws.

AMARI makes a parting shot that the Amended JVA is not a sale to AMARI of the Freedom Islands or
of the lands to be reclaimed from submerged areas of Manila Bay. In the words of AMARI, the
Amended JVA "is not a sale but a joint venture with a stipulation for reimbursement of the original cost
incurred by PEA for the earlier reclamation and construction works performed by the CDCP under its
1973 contract with the Republic." Whether the Amended JVA is a sale or a joint venture, the fact
remains that the Amended JVA requires PEA to "cause the issuance and delivery of the certificates of
title conveying AMARI's Land Share in the name of AMARI."107

This stipulation still contravenes Section 3, Article XII of the 1987 Constitution which provides that
private corporations "shall not hold such alienable lands of the public domain except by lease." The
transfer of title and ownership to AMARI clearly means that AMARI will "hold" the reclaimed lands
other than by lease. The transfer of title and ownership is a "disposition" of the reclaimed lands, a
transaction considered a sale or alienation under CA No. 141,108 the Government Auditing
Code,109 and Section 3, Article XII of the 1987 Constitution.

The Regalian doctrine is deeply implanted in our legal system. Foreshore and submerged areas form
part of the public domain and are inalienable. Lands reclaimed from foreshore and submerged areas
also form part of the public domain and are also inalienable, unless converted pursuant to law into
alienable or disposable lands of the public domain. Historically, lands reclaimed by the government
are sui generis, not available for sale to private parties unlike other alienable public lands. Reclaimed
lands retain their inherent potential as areas for public use or public service. Alienable lands of the
public domain, increasingly becoming scarce natural resources, are to be distributed equitably among
our ever-growing population. To insure such equitable distribution, the 1973 and 1987 Constitutions
have barred private corporations from acquiring any kind of alienable land of the public domain.
Those who attempt to dispose of inalienable natural resources of the State, or seek to circumvent the
constitutional ban on alienation of lands of the public domain to private corporations, do so at their
own risk.

We can now summarize our conclusions as follows:

1. The 157.84 hectares of reclaimed lands comprising the Freedom Islands, now covered by
certificates of title in the name of PEA, are alienable lands of the public domain. PEA may
lease these lands to private corporations but may not sell or transfer ownership of these lands
to private corporations. PEA may only sell these lands to Philippine citizens, subject to the
ownership limitations in the 1987 Constitution and existing laws.

2. The 592.15 hectares of submerged areas of Manila Bay remain inalienable natural
resources of the public domain until classified as alienable or disposable lands open to
disposition and declared no longer needed for public service. The government can make such
classification and declaration only after PEA has reclaimed these submerged areas. Only
then can these lands qualify as agricultural lands of the public domain, which are the only
natural resources the government can alienate. In their present state, the 592.15 hectares of
submerged areas are inalienable and outside the commerce of man.

3. Since the Amended JVA seeks to transfer to AMARI, a private corporation, ownership of
77.34 hectares110of the Freedom Islands, such transfer is void for being contrary to Section 3,
375

Article XII of the 1987 Constitution which prohibits private corporations from acquiring any
kind of alienable land of the public domain.

4. Since the Amended JVA also seeks to transfer to AMARI ownership of 290.156
hectares111 of still submerged areas of Manila Bay, such transfer is void for being contrary to
Section 2, Article XII of the 1987 Constitution which prohibits the alienation of natural
resources other than agricultural lands of the public domain. PEA may reclaim these
submerged areas. Thereafter, the government can classify the reclaimed lands as alienable
or disposable, and further declare them no longer needed for public service. Still, the transfer
of such reclaimed alienable lands of the public domain to AMARI will be void in view of
Section 3, Article XII of the 1987 Constitution which prohibits private corporations from
acquiring any kind of alienable land of the public domain.

Clearly, the Amended JVA violates glaringly Sections 2 and 3, Article XII of the 1987 Constitution.
Under Article 1409112 of the Civil Code, contracts whose "object or purpose is contrary to law," or
whose "object is outside the commerce of men," are "inexistent and void from the beginning." The
Court must perform its duty to defend and uphold the Constitution, and therefore declares the
Amended JVA null and void ab initio.

Seventh issue: whether the Court is the proper forum to raise the issue of whether the
Amended JVA is grossly disadvantageous to the government.

Considering that the Amended JVA is null and void ab initio, there is no necessity to rule on this last
issue. Besides, the Court is not a trier of facts, and this last issue involves a determination of factual
matters.

WHEREFORE, the petition is GRANTED. The Public Estates Authority and Amari Coastal Bay
Development Corporation are PERMANENTLY ENJOINED from implementing the Amended Joint
Venture Agreement which is hereby declared NULL and VOID ab initio.

SO ORDERED.

Davide, Jr., C.J., Bellosillo, Puno, Vitug, Kapunan, Mendoza, Panganiban, Quisumbing, Ynares-
Santiago, Sandoval-Gutierrez, Austria-Martinez, and Corona, JJ., concur.

Footnote

1 Section 4 of PD No. 1084.

2 PEA's Memorandum dated August 4, 1999, p. 3.

3PEA's Memorandum, supra note 2 at 7. PEA's Memorandum quoted extensively, in its


Statement of Facts and the Case, the Statement of Facts in Senate Committee Report No.
560 dated September 16, 1997.

4In Opinion No. 330 dated December 23, 1994, the Government Corporate Counsel, citing
COA Audit Circular No. 89-296, advised PEA that PEA could negotiate the sale of the 157.84-
hectare Freedom Islands in view of the failure of the public bidding held on December 10,
1991 where there was not a single bidder. See also Senate Committee Report No. 560, p. 12.

5 PEA's Memorandum, supra note 2 at 9.

6 Ibid.
376

7 The existence of this report is a matter of judicial notice pursuant to Section 1, Rule 129 of
the Rules of Court which provides, "A court shall take judicial notice, without the introduction
of evidence, of x x x the official acts of the legislature x x x."

8 Teofisto Guingona, Jr.

9 Renato Cayetano.

10 Virgilio C. Abejo.

11Report and Recommendation of the Legal Task Force, Annex "C", AMARI's Memorandum
dated June 19, 1999.

12 AMARI's Comment dated June 24, 1998, p. 3; Rollo, p. 68.

13AMARI filed three motions for extension of time to file comment (Rollo, pp. 32, 38, 48),
while PEA filed nine motions for extension of time (Rollo, pp. 127, 139).

14 Petitioner's Memorandum dated July 6, 1999, p. 42.

15
Represented by the Office of the Solicitor General, with Solicitor General Ricardo P.
Galvez, Assistant Solicitor General Azucena R. Balanon-Corpuz, and Associate Solicitor
Raymund I. Rigodon signing PEA's Memorandum.

16Represented by Azcuna Yorac Arroyo & Chua Law Offices, and Romulo Mabanta Sayoc &
De los Angeles Law Offices.

17Salonga v. Paño, 134 SCRA 438 (1985); Gonzales v. Marcos, 65 SCRA 624 (1975 );
Aquino v. Enrile, 59 SCRA 183 (1974 ); Dela Camara v. Enage, 41 SCRA 1 (1971 ).

18 Section 11, Article XIV.

19 Manila Electric Co. v. Judge F. Castro-Bartolome, 114 SCRA 799 (1982); Republic v. CA
and Iglesia, and Republic v. Cendana and Iglesia ni Cristo, 119 SCRA 449 (1982); Republic
v. Villanueva and Iglesia ni Cristo, 114 SCRA 875 (1982); Director of Lands v. Lood, 124
SCRA 460 (1983); Republic v. Iglesia ni Cristo, 128 SCRA 44 (1984); Director of Lands v.
Hermanos y Hermanas de Sta. Cruz de Mayo, Inc., 141 SCRA 21 (1986); Director of Lands v.
IAC and Acme Plywood & Veneer Co., 146 SCRA 509 (1986); Republic v. IAC and Roman
Catholic Bishop of Lucena, 168 SCRA 165 (1988); Natividad v. CA, 202 SCRA 493 (1991);
Villaflor v. CA and Nasipit Lumber Co., 280 SCRA 297 (1997). In Ayog v. Cusi, 118 SCRA
492 (1982), the Court did not apply the constitutional ban in the 1973 Constitution because
the applicant corporation, Biñan Development Co., Inc., had fully complied with all its
obligations and even paid the full purchase price before the effectivity of the 1973
Constitution, although the sales patent was issued after the 1973 Constitution took effect.

20 PD No. 1073.

21
Annex "B", AMARI's Memorandum dated June 19, 1999, Section 5.2 (c) and (e) of the
Amended JVA, pp. 16-17.

22 Chavez v. PCGG, 299 SCRA 744 (1998).

23 136 SCRA 27 (1985).

24Article 2 of the Civil Code (prior to its amendment by EO No. 200) provided as follows:
"Laws shall take effect after fifteen days following the completion of their publication in the
Official Gazette, unless it is provided otherwise, x x x."
377

25Section 1 of CA No. 638 provides as follows: "There shall be published in the Official
Gazette all important legislative acts and resolutions of the Congress of the Philippines; all
executive and administrative orders and proclamations, except such as have no general
applicability; x x x."

26Section 79 of the Government Auditing Codes provides as follows: "When government


property has become unserviceable for any cause, or is no longer needed, it shall, upon
application of the officer accountable therefor, be inspected by the head of the agency or his
duly authorized representative in the presence of the auditor concerned and, if found to be
valueless or unsaleable, it may be destroyed in their presence. If found to be valuable, it
may be sold at public auction to the highest bidder under the supervision of the proper
committee on award or similar body in the presence of the auditor concerned or other
authorized representative of the Commission, after advertising by printed notice in the
Official Gazette, or for not less than three consecutive days in any newspaper of
general circulation, or where the value of the property does not warrant the expense of
publication, by notices posted for a like period in at least three public places in the locality
where the property is to be sold. In the event that the public auction fails, the property
may be sold at a private sale at such price as may be fixed by the same committee or
body concerned and approved by the Commission."

27
Paat v. Court of Appeals, 266 SCRA 167 (1997); Quisumbing v. Judge Gumban, 193
SCRA 520 (1991); Valmonte v. Belmonte, Jr., 170 SCRA 256 (1989).

28 See note 22.

29 Section 1, Article XI of the 1987 Constitution states as follows: "Public office is a public
trust. Public officers and employees must at all times be accountable to the people, serve
them with utmost responsibility, integrity, loyalty, and efficiency, act with patriotism and
justice, and lead modest lives."

30 170 SCRA 256 (1989).

31 See note 22.

32 Record of the Constitutional Commission, Vol. V, pp. 24-25, (1986).

33 Supra, Note 22.

34 Ibid.

35 Legaspi v. Civil Service Commission, 150 SCRA 530 (1987).

36 Almonte v. Vasquez, 244 SCRA 286 (1995).

37 See Note 22.

38 Chavez v. PCGG, see note 22; Aquino-Sarmiento v. Morato, 203 SCRA 515 (1991).

39 Almonte v. Vasquez, see note 36.

40People's Movement for Press Freedom, et al. v. Hon. Raul Manglapus, G.R. No. 84642, En
Banc Resolution dated April 13, 1988; Chavez v. PCGG, see note 22.

41 Section 270 of the National Internal Revenue Code punishes any officer or employee of the
Bureau of Internal Revenue who divulges to any person, except as allowed by law,
information regarding the business, income, or estate of any taxpayer, the secrets, operation,
style of work, or apparatus of any manufacturer or producer, or confidential information
378

regarding the business of any taxpayer, knowledge of which was acquired by him in the
discharge of his official duties. Section 14 of R.A. No. 8800 (Safeguard Measures Act)
prohibits the release to the public of confidential information submitted in evidence to the
Tariff Commission. Section 3 (n) of R.A. No. 8504 (Philippine AIDS Prevention and Control
Act) classifies as confidential the medical records of HIV patients. Section 6 (j) of R.A. No.
8043 (Inter-Country Adoption Act) classifies as confidential the records of the adopted child,
adopting parents, and natural parents. Section 94 (f) of R.A. No. 7942 (Philippine Mining Act)
requires the Department of Environment and Natural Resources to maintain the confidentiality
of confidential information supplied by contractors who are parties to mineral agreements or
financial and technical assistance agreements.

42The Recopilacion de Leyes de las Indias declared that: "We, having acquired full
sovereignty over the Indies, and all lands, territories, and possessions not heretofore ceded
away by our royal predecessors, or by us, or in our name, still pertaining to the royal crown
and patrimony, it is our will that all lands which are held without proper and true deeds of
grant be restored to us according as they belong to us, in order that after reserving before all
what to us or to our viceroys, audiencias, and governors may seem necessary for public
squares, ways, pastures, and commons in those places which are peopled, taking into
consideration not only their present condition, but also their future and their probable
increase, and after distributing to the natives what may be necessary for tillage and
pasturage, confirming them in what they now have and giving them more if necessary, all the
rest of said lands may remain free and unencumbered for us to dispose of as we may
wish." See concurring opinion of Justice Reynato S. Puno in Republic Real Estate
Corporation v. Court of Appeals, 299 SCRA 199 (1998).

43 Cariño v. Insular Government, 41 Phil. 935 (1909). The exception mentioned in Cariño,
referring to lands in the possession of an occupant and of his predecessors-in-interest, since
time immemorial, is actually a species of a grant by the State. The United States Supreme
Court, speaking through Justice Oliver Wendell Holmes, Jr., declared in Cariño: "Prescription
is mentioned again in the royal cedula of October 15, 1754, cited in 3 Philippine, 546; 'Where
such possessors shall not be able to produce title deeds, it shall be sufficient if they shall
show that ancient possession, as a valid title by prescription.' It may be that this means
possession from before 1700; but, at all events, the principle is admitted. As prescription,
even against the Crown lands, was recognized by the laws of Spain, we see no sufficient
reason for hesitating to admit that it was recognized in the Philippines in regard to lands over
which Spain had only a paper sovereignty." See also Republic v. Lee, 197 SCRA 13 (1991).

44 Article 1 of the Spanish Law of Waters of 1866.

45Ignacio v. Director of Lands, 108 Phil. 335 (1960); Joven v. Director of Lands, 93 Phil. 134
(1953); Laurel v. Garcia, 187 SCRA 797 (1990). See concurring opinion of Justice Reynato S.
Puno in Republic Real Estate Corporation v. Court of Appeals, 299 SCRA 199 (1998).

46 Act No. 926, enacted on October 7, 1903, was also titled the Public Land Act. This Act,
however, did not cover reclaimed lands. Nevertheless, Section 23 of this Act provided as
follows: "x x x In no case may lands leased under the provisions of this chapter be taken so
as to gain control of adjacent land, water, stream, shore line, way, roadstead, or other
valuable right which in the opinion of the Chief of the Bureau of Public Lands would be
prejudicial to the interests of the public."

47Section 10 of Act No. 2874 provided as follows: "The words "alienation," "disposition," or
"concession" as used in this Act, shall mean any of the methods authorized by this Act for the
acquisition, lease, use, or benefit of the lands of the public domain other than timber or
mineral lands."

48Title II of Act No. 2874 governed alienable lands of the public domain for agricultural
purposes, while Title III of the same Act governed alienable lands of the public domain for
non-agricultural purposes.
379

49 Section 57 of Act No. 2874 provided as follows: "x x x; but the land so granted, donated, or
transferred to a province, municipality, or branch or subdivision of the Government shall not
be alienated, encumbered, or otherwise disposed of in a manner affecting its title, except
when authorized by the legislature; x x x."

50 Krivenko v. Register of Deeds, 79 Phil. 461 (1947).

51Section 2 of CA No. 141 states as follows: "The provisions of this Act shall apply to the
lands of the public domain; but timber and mineral lands shall be governed by special laws
and nothing in this Act provided shall be understood or construed to change or modify the
administration and disposition of the lands commonly called "friar lands" and those which,
being privately owned, have reverted to or become the property of the Commonwealth of the
Philippines, which administration and disposition shall be governed by the laws at present in
force or which may hereafter be enacted."

52Like Act No. 2874, Section 10 of CA No. 141 defined the terms "alienation" and
"disposition" as follows: "The words "alienation," "disposition," or "concession" as used in this
Act, shall mean any of the methods authorized by this Act for the acquisition, lease, use, or
benefit of the lands of the public domain other than timber or mineral lands."

53R.A. No. 6657 has suspended the authority of the President to reclassify forest or mineral
lands into agricultural lands. Section 4 (a) of RA No. 6657 (Comprehensive Agrarian Reform
Law of 1988) states, "No reclassification of forest or mineral lands to agricultural lands shall
be undertaken after the approval of this Act until Congress, taking into account ecological,
developmental and equity considerations, shall have delimited by law, the specific limits of the
public domain."

54 Covering Sections 58 to 68 of CA No. 141.

55 299 SCRA 199 (1998).

56 Section 1, Article XIII of the 1935 Constitution limited the disposition and utilization of public
agricultural lands to Philippine citizens or to corporations at least sixty percent owned by
Philippine citizens. This was, however, subject to the original Ordinance appended to the
1935 Constitution stating, among others, that until the withdrawal of United States sovereignty
in the Philippines, "Citizens and corporations of the United States shall enjoy in the
Commonwealth of the Philippines all the civil rights of the citizens and corporations,
respectively, thereof."

57 Section 44 of PD No. 1529 (previously Section 39 of Act No. 496) provides that "liens,
claims or rights arising or existing under the laws and the Constitution of the Philippines which
are not by law required to appear of record in the Registry of Deeds in order to be valid
against subsequent purchasers or encumbrancers of record" constitute statutory liens
affecting the title.1âwphi1.nêt

58 RA No. 730, which took effect on June 18, 1952, authorized the private sale of home lots
to actual occupants of public lands not needed for public service. Section 1 of RA No. 730
provided as follows: "Notwithstanding the provisions of Sections 61 and 67 of Commonwealth
Act No. 141, as amended by RA No. 293, any Filipino citizen of legal age who is not the
owner of a home lot in the municipality or city in which he resides and who had in good faith
established his residence on a parcel of land of the Republic of the Philippines which is not
needed for public service, shall be given preference to purchase at a private sale of which
reasonable notice shall be given to him, not more than one thousand square meters at a price
to be fixed by the Director of Lands with the approval of the Secretary of Agriculture and
Natural Resources. x x x." In addition, on June 16, 1948, Congress enacted R.A. No. 293
allowing the private sale of marshy alienable or disposable lands of the public domain to
lessees who have improved and utilized the same as farms, fishponds or other similar
purposes for at least five years from the date of the lease contract with the government. R.A.
380

No. 293, however, did not apply to marshy lands under Section 56 (c), Title III of CA No. 141
which refers to marshy lands leased for residential, commercial, industrial or other non-
agricultural purposes.

59 See note 49.

60 See note 60.

61 Republic Real Estate Corporation v. Court of Appeals, see note 56.

62 Ibid.

63Insular Government v. Aldecoa, 19 Phil. 505 (1911); Government v. Cabangis, 53 Phil. 112
(1929).

64 118 SCRA 492 (1982).

65 Annex "B", AMARI's Memorandum, see note 2 at 1 & 2.

66 PEA's Memorandum, see note 6.

67 Ibid., p. 44.

68 See notes 9, 10 & 11.

69 Annex "C", p. 3, AMARI's Memorandum, see note 12 at 3.

70 This should read Article XII.

71 Section 8 of CA No. 141.

72 Emphasis supplied.

73 187 SCRA 797 (1990).

74Article 422 of the Civil Code states as follows: "Property of public dominion, when no longer
needed for public use or public service, shall form part of the patrimonial property of the
State."

75 AMARI's Comment dated June 24, 1998, p. 20; Rollo, p. 85.

76Dizon v. Rodriguez, 13 SCRA 705 (1965); Republic v. Lat Vda. de Castillo, 163 SCRA 286
(1988).

77 Cariño v. Insular Government, 41 Phil. 935 (1909).

78 Proclamation No. 41, issued by President Ramon Magsaysay on July 5, 1954, reserved for
"National Park purposes" 464.66 hectares of the public domain in Manila Bay "situated in the
cities of Manila and Pasay and the municipality of Paranaque, Province of Rizal, Island of
Luzon," which area, as described in detail in the Proclamation, is "B]ounded on the North, by
Manila Bay; on the East, by Dewey Boulevard; and on the south and west, by Manila Bay."
See concurring opinion of Justice Reynato S. Puno in Republic Real Estate Corporation v.
Court of Appeals, 299 SCRA 1999 (1998). Under Sections 2 and 3, Article XII of the 1987
Constitution, "national parks" are inalienable natural resources of the State.

79 Fifth Whereas clause of EO No. 525.


381

80 Section 4, Chapter I, Title XIV, Book IV.

81Section 6 of CA No 141 provides as follows: "The President, upon the recommendation


of the Secretary of Agriculture and Commerce, shall from time to time classify the lands of
the public domain into – (a) Alienable or disposable, x x x."

82Section 7 of CA No. 141 provides as follows: "For purposes of the administration and
disposition of alienable or disposable public lands, the President, upon recommendation by
the Secretary of Agriculture and Commerce, shall from time to time declare what lands are
open to disposition or concession under this Act."

83 On "Lands for Residential, Commercial, or Industrial and other Similar Purposes."

84 RA No. 293, enacted on June 16, 1948, authorized the sale of marshy lands under certain
conditions. Section 1 of RA No. 293 provided as follows: "The provisions of section sixty-one
of Commonwealth Act Numbered One hundred and forty-one to the contrary notwithstanding,
marshy lands and lands under water bordering on shores or banks or navigable lakes or
rivers which are covered by subsisting leases or leases which may hereafter be duly granted
under the provisions of the said Act and are already improved and have been utilized for
farming, fishpond, or similar purposes for at least five years from the date of the contract of
lease, may be sold to the lessees thereof under the provisions of Chapter Five of the said Act
as soon as the President, upon recommendation of the Secretary of Agriculture and Natural
Resources, shall declare that the same are not necessary for the public service."

85 PEA's Memorandum, see note 2 at 45.

86 See note 73.

87 Section 4 (b) of PD No. 1084

88R.A. No. 730 allows the private sale of home lots to actual occupants of public
lands. See note 63.

89 Issued on February 26, 1981.

90While PEA claims there was a failure of public bidding on December 10, 1991, there is no
showing that the Commission on Audit approved the price or consideration stipulated in the
negotiated Amended JVA as required by Section 79 of the Government Auditing Code.
Senate Committee Report No. 560 did not discuss this issue.

91Paragraph 2 (a) of COA Circular No. 89-296, on "Sale Thru Negotiation," states that
disposal through negotiated sale may be resorted to if "[T]here was a failure of public
auction."

92Senate Committee Report No. 560, Statement of Facts, p. 7, citing PEA Board Resolution
No. 835, as appearing in the Minutes of the PEA Board of Directors Meeting held on May 30,
1991, per Certification of Jaime T. De Veyra, Corporate Secretary, dated June 11, 1991.

93 Opinion No. 330, citing COA Audit Circular No. 89-296. See note 5.

94 PEA's Memorandum, see note 2.

95 Senate Committee Report No. 560, pp. 7-8, citing the Minutes of Meeting of the PEA Board
of Directors held on December 19, 1991.

96Section 3, Article XII of the 1987 Constitution provides as follows: "x x x Citizens of the
Philippines may x x x acquire not more than twelve hectares thereof by purchase, homestead
382

or grant." However, Section 6 of R.A. No. 6657 (Comprehensive Agrarian Reform Law) limits
the ownership of "public or private agricultural land" to a maximum of five hectares per
person.

97 96 Phil. 946 (1955).

98 48 SCRA 372 (1977).

99 168 SCRA 198 (1988).

100 172 SCRA 795 (1989).

101 73 SCRA 146 (1976).

102 Avila v. Tapucar, 201 SCRA 148 (1991).

103Republic v. Ayala Cia, et al., 14 SCRA 259 (1965); Dizon v. Rodriguez, 13 SCRA 705
(1965).

104Section 44 of PD No. 1529 states as follows: "Every registered owner receiving a


certificate of title in pursuance of a decree of registration, and every subsequent purchaser of
registered land taking a certificate of title for value and in good faith, shall hold the same free
from all encumbrances except those noted on said certificate and any of the following
encumbrances which may be subsisting, namely: First. Liens, claims or rights arising or
existing under the laws and Constitution of the Philippines which are not by law
required to appear of record in the Registry of Deeds in order to be valid against
subsequent purchasers or encumbrancers of record. x x x." Under Section 103 of PD No.
1529, Section 44 applies to certificates of title issued pursuant to a land patent granted by the
government.

105 Section 2, Article XIII of the 1935 Constitution.

106 Harty v. Municipality of Victoria, 13 Phil. 152 (1909).

107
Annex "B", AMARI's Memorandum, see note 21 at 16, Section 5.2 (c) of the Amended
JVA.

108Section 10 of CA No. 141 provides as follows: "Sec. 10. The words "alienation,"
"disposition," or "concession" as used in this Act, shall mean any of the methods authorized
by this Act for the acquisition, lease, use, or benefit of the lands of the public domain other
than timber or mineral lands."

109 Section 79 of the Government Auditing Code, which requires public auction in the sale of
government assets, includes all kinds of disposal or divestment of government assets. Thus,
COA Audit Circular No. 86-264 dated October 16, 1986 speaks of "guidelines (which) shall
govern the general procedures on the divestment or disposal of assets of government-
owned and/or controlled corporations and their subsidiaries." Likewise, COA Audit Circular
No. 89-296 dated January 27, speaks of "guidelines (which) shall be observed and adhered
to in the divestment or disposal of property and other assets of all government
entities/instrumentalities" and that "divestment shall refer to the manner or scheme of
taking away, depriving, withdrawing of an authority, power or title." These COA Circulars
implement Section 79 of the Government Auditing Code.

110 The share of AMARI in the Freedom Islands is 77.34 hectares, which is 70 percent of the
net usable area of 110.49 hectares. The net usable area is the total land area of the Freedom
Islands less 30 percent allocated for common areas.
383

111The share of AMARI in the submerged areas for reclamation is 290.129 hectares, which is
70 percent of the net usable area of 414.47 hectares.

112 Article 1409 of the Civil Code provides as follows: "The following contracts are inexistent
and void from the beginning: (1) Those whose cause, object or purpose is contrary to law; x x
x; (4) Those whose object is outside the commerce of men; x x x."

ON MOTION FOR RECONSIDERATION (G.R. No. 133250, May 6, 2003)

EN BANC

G.R. No. 133250 July 9, 2002

FRANCISCO I. CHAVEZ, petitioner,


vs.
PUBLIC ESTATES AUTHORITY and AMARI COASTAL BAY DEVELOPMENT
CORPORATION, respondents.

CARPIO, J.:

This is an original Petition for Mandamus with prayer for a writ of preliminary injunction and a
temporary restraining order. The petition seeks to compel the Public Estates Authority ("PEA" for
brevity) to disclose all facts on PEA's then on-going renegotiations with Amari Coastal Bay and
Development Corporation ("AMARI" for brevity) to reclaim portions of Manila Bay. The petition further
seeks to enjoin PEA from signing a new agreement with AMARI involving such reclamation.

The Facts

On November 20, 1973, the government, through the Commissioner of Public Highways, signed a
contract with the Construction and Development Corporation of the Philippines ("CDCP" for brevity) to
reclaim certain foreshore and offshore areas of Manila Bay. The contract also included the
construction of Phases I and II of the Manila-Cavite Coastal Road. CDCP obligated itself to carry out
all the works in consideration of fifty percent of the total reclaimed land.

On February 4, 1977, then President Ferdinand E. Marcos issued Presidential Decree No. 1084
creating PEA. PD No. 1084 tasked PEA "to reclaim land, including foreshore and submerged areas,"
and "to develop, improve, acquire, x x x lease and sell any and all kinds of lands." 1 On the same date,
then President Marcos issued Presidential Decree No. 1085 transferring to PEA the "lands reclaimed
in the foreshore and offshore of the Manila Bay"2 under the Manila-Cavite Coastal Road and
Reclamation Project (MCCRRP).

On December 29, 1981, then President Marcos issued a memorandum directing PEA to amend its
contract with CDCP, so that "[A]ll future works in MCCRRP x x x shall be funded and owned by PEA."
Accordingly, PEA and CDCP executed a Memorandum of Agreement dated December 29, 1981,
which stated:

"(i) CDCP shall undertake all reclamation, construction, and such other works in the
MCCRRP as may be agreed upon by the parties, to be paid according to progress of works
on a unit price/lump sum basis for items of work to be agreed upon, subject to price
escalation, retention and other terms and conditions provided for in Presidential Decree No.
1594. All the financing required for such works shall be provided by PEA.

xxx
384

(iii) x x x CDCP shall give up all its development rights and hereby agrees to cede and
transfer in favor of PEA, all of the rights, title, interest and participation of CDCP in and to all
the areas of land reclaimed by CDCP in the MCCRRP as of December 30, 1981 which have
not yet been sold, transferred or otherwise disposed of by CDCP as of said date, which areas
consist of approximately Ninety-Nine Thousand Four Hundred Seventy Three (99,473)
square meters in the Financial Center Area covered by land pledge No. 5 and approximately
Three Million Three Hundred Eighty Two Thousand Eight Hundred Eighty Eight (3,382,888)
square meters of reclaimed areas at varying elevations above Mean Low Water Level located
outside the Financial Center Area and the First Neighborhood Unit." 3

On January 19, 1988, then President Corazon C. Aquino issued Special Patent No. 3517, granting
and transferring to PEA "the parcels of land so reclaimed under the Manila-Cavite Coastal Road and
Reclamation Project (MCCRRP) containing a total area of one million nine hundred fifteen thousand
eight hundred ninety four (1,915,894) square meters." Subsequently, on April 9, 1988, the Register of
Deeds of the Municipality of Parañaque issued Transfer Certificates of Title Nos. 7309, 7311, and
7312, in the name of PEA, covering the three reclaimed islands known as the "Freedom Islands"
located at the southern portion of the Manila-Cavite Coastal Road, Parañaque City. The Freedom
Islands have a total land area of One Million Five Hundred Seventy Eight Thousand Four Hundred
and Forty One (1,578,441) square meters or 157.841 hectares.

On April 25, 1995, PEA entered into a Joint Venture Agreement ("JVA" for brevity) with AMARI, a
private corporation, to develop the Freedom Islands. The JVA also required the reclamation of an
additional 250 hectares of submerged areas surrounding these islands to complete the configuration
in the Master Development Plan of the Southern Reclamation Project-MCCRRP. PEA and AMARI
entered into the JVA through negotiation without public bidding.4 On April 28, 1995, the Board of
Directors of PEA, in its Resolution No. 1245, confirmed the JVA.5 On June 8, 1995, then President
Fidel V. Ramos, through then Executive Secretary Ruben Torres, approved the JVA.6

On November 29, 1996, then Senate President Ernesto Maceda delivered a privilege speech in the
Senate and denounced the JVA as the "grandmother of all scams." As a result, the Senate
Committee on Government Corporations and Public Enterprises, and the Committee on
Accountability of Public Officers and Investigations, conducted a joint investigation. The Senate
Committees reported the results of their investigation in Senate Committee Report No. 560 dated
September 16, 1997.7 Among the conclusions of their report are: (1) the reclaimed lands PEA seeks
to transfer to AMARI under the JVA are lands of the public domain which the government has not
classified as alienable lands and therefore PEA cannot alienate these lands; (2) the certificates of title
covering the Freedom Islands are thus void, and (3) the JVA itself is illegal.

On December 5, 1997, then President Fidel V. Ramos issued Presidential Administrative Order No.
365 creating a Legal Task Force to conduct a study on the legality of the JVA in view of Senate
Committee Report No. 560. The members of the Legal Task Force were the Secretary of Justice, 8 the
Chief Presidential Legal Counsel,9 and the Government Corporate Counsel.10 The Legal Task Force
upheld the legality of the JVA, contrary to the conclusions reached by the Senate Committees. 11

On April 4 and 5, 1998, the Philippine Daily Inquirer and Today published reports that there were on-
going renegotiations between PEA and AMARI under an order issued by then President Fidel V.
Ramos. According to these reports, PEA Director Nestor Kalaw, PEA Chairman Arsenio Yulo and
retired Navy Officer Sergio Cruz composed the negotiating panel of PEA.

On April 13, 1998, Antonio M. Zulueta filed before the Court a Petition for Prohibition with Application
for the Issuance of a Temporary Restraining Order and Preliminary Injunction docketed as G.R. No.
132994 seeking to nullify the JVA. The Court dismissed the petition "for unwarranted disregard of
judicial hierarchy, without prejudice to the refiling of the case before the proper court."12

On April 27, 1998, petitioner Frank I. Chavez ("Petitioner" for brevity) as a taxpayer, filed the
instant Petition for Mandamus with Prayer for the Issuance of a Writ of Preliminary Injunction and
Temporary Restraining Order. Petitioner contends the government stands to lose billions of pesos in
the sale by PEA of the reclaimed lands to AMARI. Petitioner prays that PEA publicly disclose the
terms of any renegotiation of the JVA, invoking Section 28, Article II, and Section 7, Article III, of the
385

1987 Constitution on the right of the people to information on matters of public concern. Petitioner
assails the sale to AMARI of lands of the public domain as a blatant violation of Section 3, Article XII
of the 1987 Constitution prohibiting the sale of alienable lands of the public domain to private
corporations. Finally, petitioner asserts that he seeks to enjoin the loss of billions of pesos in
properties of the State that are of public dominion.

After several motions for extension of time,13 PEA and AMARI filed their Comments on October 19,
1998 and June 25, 1998, respectively. Meanwhile, on December 28, 1998, petitioner filed an
Omnibus Motion: (a) to require PEA to submit the terms of the renegotiated PEA-AMARI contract; (b)
for issuance of a temporary restraining order; and (c) to set the case for hearing on oral argument.
Petitioner filed a Reiterative Motion for Issuance of a TRO dated May 26, 1999, which the Court
denied in a Resolution dated June 22, 1999.

In a Resolution dated March 23, 1999, the Court gave due course to the petition and required the
parties to file their respective memoranda.

On March 30, 1999, PEA and AMARI signed the Amended Joint Venture Agreement ("Amended
JVA," for brevity). On May 28, 1999, the Office of the President under the administration of then
President Joseph E. Estrada approved the Amended JVA.

Due to the approval of the Amended JVA by the Office of the President, petitioner now prays that on
"constitutional and statutory grounds the renegotiated contract be declared null and void."14

The Issues

The issues raised by petitioner, PEA15 and AMARI16 are as follows:

I. WHETHER THE PRINCIPAL RELIEFS PRAYED FOR IN THE PETITION ARE MOOT AND
ACADEMIC BECAUSE OF SUBSEQUENT EVENTS;

II. WHETHER THE PETITION MERITS DISMISSAL FOR FAILING TO OBSERVE THE
PRINCIPLE GOVERNING THE HIERARCHY OF COURTS;

III. WHETHER THE PETITION MERITS DISMISSAL FOR NON-EXHAUSTION OF


ADMINISTRATIVE REMEDIES;

IV. WHETHER PETITIONER HAS LOCUS STANDI TO BRING THIS SUIT;

V. WHETHER THE CONSTITUTIONAL RIGHT TO INFORMATION INCLUDES OFFICIAL


INFORMATION ON ON-GOING NEGOTIATIONS BEFORE A FINAL AGREEMENT;

VI. WHETHER THE STIPULATIONS IN THE AMENDED JOINT VENTURE AGREEMENT


FOR THE TRANSFER TO AMARI OF CERTAIN LANDS, RECLAIMED AND STILL TO BE
RECLAIMED, VIOLATE THE 1987 CONSTITUTION; AND

VII. WHETHER THE COURT IS THE PROPER FORUM FOR RAISING THE ISSUE OF
WHETHER THE AMENDED JOINT VENTURE AGREEMENT IS GROSSLY
DISADVANTAGEOUS TO THE GOVERNMENT.

The Court's Ruling

First issue: whether the principal reliefs prayed for in the petition are moot and academic
because of subsequent events.

The petition prays that PEA publicly disclose the "terms and conditions of the on-going negotiations
for a new agreement." The petition also prays that the Court enjoin PEA from "privately entering into,
perfecting and/or executing any new agreement with AMARI."
386

PEA and AMARI claim the petition is now moot and academic because AMARI furnished petitioner on
June 21, 1999 a copy of the signed Amended JVA containing the terms and conditions agreed upon
in the renegotiations. Thus, PEA has satisfied petitioner's prayer for a public disclosure of the
renegotiations. Likewise, petitioner's prayer to enjoin the signing of the Amended JVA is now moot
because PEA and AMARI have already signed the Amended JVA on March 30, 1999. Moreover, the
Office of the President has approved the Amended JVA on May 28, 1999.

Petitioner counters that PEA and AMARI cannot avoid the constitutional issue by simply fast-tracking
the signing and approval of the Amended JVA before the Court could act on the issue. Presidential
approval does not resolve the constitutional issue or remove it from the ambit of judicial review.

We rule that the signing of the Amended JVA by PEA and AMARI and its approval by the President
cannot operate to moot the petition and divest the Court of its jurisdiction. PEA and AMARI have still
to implement the Amended JVA. The prayer to enjoin the signing of the Amended JVA on
constitutional grounds necessarily includes preventing its implementation if in the meantime PEA and
AMARI have signed one in violation of the Constitution. Petitioner's principal basis in assailing the
renegotiation of the JVA is its violation of Section 3, Article XII of the Constitution, which prohibits the
government from alienating lands of the public domain to private corporations. If the Amended JVA
indeed violates the Constitution, it is the duty of the Court to enjoin its implementation, and if already
implemented, to annul the effects of such unconstitutional contract.

The Amended JVA is not an ordinary commercial contract but one which seeks to transfer title and
ownership to 367.5 hectares of reclaimed lands and submerged areas of Manila Bay to a single
private corporation. It now becomes more compelling for the Court to resolve the issue to insure the
government itself does not violate a provision of the Constitution intended to safeguard the national
patrimony. Supervening events, whether intended or accidental, cannot prevent the Court from
rendering a decision if there is a grave violation of the Constitution. In the instant case, if the
Amended JVA runs counter to the Constitution, the Court can still prevent the transfer of title and
ownership of alienable lands of the public domain in the name of AMARI. Even in cases where
supervening events had made the cases moot, the Court did not hesitate to resolve the legal or
constitutional issues raised to formulate controlling principles to guide the bench, bar, and the
public.17

Also, the instant petition is a case of first impression. All previous decisions of the Court involving
Section 3, Article XII of the 1987 Constitution, or its counterpart provision in the 1973
Constitution,18 covered agricultural lands sold to private corporations which acquired the lands from
private parties. The transferors of the private corporations claimed or could claim the right to judicial
confirmation of their imperfect titles19 under Title II of Commonwealth Act. 141 ("CA No. 141" for
brevity). In the instant case, AMARI seeks to acquire from PEA, a public corporation, reclaimed lands
and submerged areas for non-agricultural purposes by purchase under PD No. 1084 (charter of
PEA) and Title III of CA No. 141. Certain undertakings by AMARI under the Amended JVA constitute
the consideration for the purchase. Neither AMARI nor PEA can claim judicial confirmation of their
titles because the lands covered by the Amended JVA are newly reclaimed or still to be reclaimed.
Judicial confirmation of imperfect title requires open, continuous, exclusive and notorious occupation
of agricultural lands of the public domain for at least thirty years since June 12, 1945 or earlier.
Besides, the deadline for filing applications for judicial confirmation of imperfect title expired on
December 31, 1987.20

Lastly, there is a need to resolve immediately the constitutional issue raised in this petition because of
the possible transfer at any time by PEA to AMARI of title and ownership to portions of the reclaimed
lands. Under the Amended JVA, PEA is obligated to transfer to AMARI the latter's seventy percent
proportionate share in the reclaimed areas as the reclamation progresses. The Amended JVA even
allows AMARI to mortgage at any time the entire reclaimed area to raise financing for the reclamation
project.21

Second issue: whether the petition merits dismissal for failing to observe the principle
governing the hierarchy of courts.
387

PEA and AMARI claim petitioner ignored the judicial hierarchy by seeking relief directly from the
Court. The principle of hierarchy of courts applies generally to cases involving factual questions. As it
is not a trier of facts, the Court cannot entertain cases involving factual issues. The instant case,
however, raises constitutional issues of transcendental importance to the public.22 The Court can
resolve this case without determining any factual issue related to the case. Also, the instant case is a
petition for mandamus which falls under the original jurisdiction of the Court under Section 5, Article
VIII of the Constitution. We resolve to exercise primary jurisdiction over the instant case.

Third issue: whether the petition merits dismissal for non-exhaustion of administrative
remedies.

PEA faults petitioner for seeking judicial intervention in compelling PEA to disclose publicly certain
information without first asking PEA the needed information. PEA claims petitioner's direct resort to
the Court violates the principle of exhaustion of administrative remedies. It also violates the rule that
mandamus may issue only if there is no other plain, speedy and adequate remedy in the ordinary
course of law.

PEA distinguishes the instant case from Tañada v. Tuvera23 where the Court granted the petition for
mandamus even if the petitioners there did not initially demand from the Office of the President the
publication of the presidential decrees. PEA points out that in Tañada, the Executive Department had
an affirmative statutory duty under Article 2 of the Civil Code24 and Section 1 of Commonwealth Act
No. 63825 to publish the presidential decrees. There was, therefore, no need for the petitioners in
Tañada to make an initial demand from the Office of the President. In the instant case, PEA claims it
has no affirmative statutory duty to disclose publicly information about its renegotiation of the JVA.
Thus, PEA asserts that the Court must apply the principle of exhaustion of administrative remedies to
the instant case in view of the failure of petitioner here to demand initially from PEA the needed
information.

The original JVA sought to dispose to AMARI public lands held by PEA, a government corporation.
Under Section 79 of the Government Auditing Code,26 the disposition of government lands to private
parties requires public bidding. PEA was under a positive legal duty to disclose to the public the
terms and conditions for the sale of its lands. The law obligated PEA to make this public
disclosure even without demand from petitioner or from anyone. PEA failed to make this public
disclosure because the original JVA, like the Amended JVA, was the result of a negotiated contract,
not of a public bidding. Considering that PEA had an affirmative statutory duty to make the public
disclosure, and was even in breach of this legal duty, petitioner had the right to seek direct judicial
intervention.

Moreover, and this alone is determinative of this issue, the principle of exhaustion of administrative
remedies does not apply when the issue involved is a purely legal or constitutional question. 27 The
principal issue in the instant case is the capacity of AMARI to acquire lands held by PEA in view of the
constitutional ban prohibiting the alienation of lands of the public domain to private corporations. We
rule that the principle of exhaustion of administrative remedies does not apply in the instant case.

Fourth issue: whether petitioner has locus standi to bring this suit

PEA argues that petitioner has no standing to institute mandamus proceedings to enforce his
constitutional right to information without a showing that PEA refused to perform an affirmative duty
imposed on PEA by the Constitution. PEA also claims that petitioner has not shown that he will suffer
any concrete injury because of the signing or implementation of the Amended JVA. Thus, there is no
actual controversy requiring the exercise of the power of judicial review.

The petitioner has standing to bring this taxpayer's suit because the petition seeks to compel PEA to
comply with its constitutional duties. There are two constitutional issues involved here. First is the
right of citizens to information on matters of public concern. Second is the application of a
constitutional provision intended to insure the equitable distribution of alienable lands of the public
domain among Filipino citizens. The thrust of the first issue is to compel PEA to disclose publicly
information on the sale of government lands worth billions of pesos, information which the
388

Constitution and statutory law mandate PEA to disclose. The thrust of the second issue is to prevent
PEA from alienating hundreds of hectares of alienable lands of the public domain in violation of the
Constitution, compelling PEA to comply with a constitutional duty to the nation.

Moreover, the petition raises matters of transcendental importance to the public. In Chavez v.
PCGG,28 the Court upheld the right of a citizen to bring a taxpayer's suit on matters of transcendental
importance to the public, thus -

"Besides, petitioner emphasizes, the matter of recovering the ill-gotten wealth of the
Marcoses is an issue of 'transcendental importance to the public.' He asserts that ordinary
taxpayers have a right to initiate and prosecute actions questioning the validity of acts or
orders of government agencies or instrumentalities, if the issues raised are of 'paramount
public interest,' and if they 'immediately affect the social, economic and moral well being of
the people.'

Moreover, the mere fact that he is a citizen satisfies the requirement of personal interest,
when the proceeding involves the assertion of a public right, such as in this case. He invokes
several decisions of this Court which have set aside the procedural matter of locus standi,
when the subject of the case involved public interest.

xxx

In Tañada v. Tuvera, the Court asserted that when the issue concerns a public right and the
object of mandamus is to obtain the enforcement of a public duty, the people are regarded as
the real parties in interest; and because it is sufficient that petitioner is a citizen and as such is
interested in the execution of the laws, he need not show that he has any legal or special
interest in the result of the action. In the aforesaid case, the petitioners sought to enforce their
right to be informed on matters of public concern, a right then recognized in Section 6, Article
IV of the 1973 Constitution, in connection with the rule that laws in order to be valid and
enforceable must be published in the Official Gazette or otherwise effectively promulgated. In
ruling for the petitioners' legal standing, the Court declared that the right they sought to be
enforced 'is a public right recognized by no less than the fundamental law of the land.'

Legaspi v. Civil Service Commission, while reiterating Tañada, further declared that 'when a
mandamus proceeding involves the assertion of a public right, the requirement of personal
interest is satisfied by the mere fact that petitioner is a citizen and, therefore, part of the
general 'public' which possesses the right.'

Further, in Albano v. Reyes, we said that while expenditure of public funds may not have
been involved under the questioned contract for the development, management and
operation of the Manila International Container Terminal, 'public interest [was] definitely
involved considering the important role [of the subject contract] . . . in the economic
development of the country and the magnitude of the financial consideration involved.' We
concluded that, as a consequence, the disclosure provision in the Constitution would
constitute sufficient authority for upholding the petitioner's standing.

Similarly, the instant petition is anchored on the right of the people to information and access
to official records, documents and papers — a right guaranteed under Section 7, Article III of
the 1987 Constitution. Petitioner, a former solicitor general, is a Filipino citizen. Because of
the satisfaction of the two basic requisites laid down by decisional law to sustain petitioner's
legal standing, i.e. (1) the enforcement of a public right (2) espoused by a Filipino citizen, we
rule that the petition at bar should be allowed."

We rule that since the instant petition, brought by a citizen, involves the enforcement of constitutional
rights - to information and to the equitable diffusion of natural resources - matters of transcendental
public importance, the petitioner has the requisite locus standi.
389

Fifth issue: whether the constitutional right to information includes official information on on-
going negotiations before a final agreement.

Section 7, Article III of the Constitution explains the people's right to information on matters of public
concern in this manner:

"Sec. 7. The right of the people to information on matters of public concern shall be
recognized. Access to official records, and to documents, and papers pertaining to
official acts, transactions, or decisions, as well as to government research data used as
basis for policy development, shall be afforded the citizen, subject to such limitations as may
be provided by law." (Emphasis supplied)

The State policy of full transparency in all transactions involving public interest reinforces the people's
right to information on matters of public concern. This State policy is expressed in Section 28, Article
II of the Constitution, thus:

"Sec. 28. Subject to reasonable conditions prescribed by law, the State adopts and
implements a policy of full public disclosure of all its transactions involving public
interest." (Emphasis supplied)

These twin provisions of the Constitution seek to promote transparency in policy-making and in the
operations of the government, as well as provide the people sufficient information to exercise
effectively other constitutional rights. These twin provisions are essential to the exercise of freedom of
expression. If the government does not disclose its official acts, transactions and decisions to citizens,
whatever citizens say, even if expressed without any restraint, will be speculative and amount to
nothing. These twin provisions are also essential to hold public officials "at all times x x x accountable
to the people,"29 for unless citizens have the proper information, they cannot hold public officials
accountable for anything. Armed with the right information, citizens can participate in public
discussions leading to the formulation of government policies and their effective implementation. An
informed citizenry is essential to the existence and proper functioning of any democracy. As explained
by the Court in Valmonte v. Belmonte, Jr.30 –

"An essential element of these freedoms is to keep open a continuing dialogue or process of
communication between the government and the people. It is in the interest of the State that
the channels for free political discussion be maintained to the end that the government may
perceive and be responsive to the people's will. Yet, this open dialogue can be effective only
to the extent that the citizenry is informed and thus able to formulate its will intelligently. Only
when the participants in the discussion are aware of the issues and have access to
information relating thereto can such bear fruit."

PEA asserts, citing Chavez v. PCGG,31 that in cases of on-going negotiations the right to information
is limited to "definite propositions of the government." PEA maintains the right does not include
access to "intra-agency or inter-agency recommendations or communications during the stage when
common assertions are still in the process of being formulated or are in the 'exploratory stage'."

Also, AMARI contends that petitioner cannot invoke the right at the pre-decisional stage or before the
closing of the transaction. To support its contention, AMARI cites the following discussion in the 1986
Constitutional Commission:

"Mr. Suarez. And when we say 'transactions' which should be distinguished from contracts,
agreements, or treaties or whatever, does the Gentleman refer to the steps leading to the
consummation of the contract, or does he refer to the contract itself?

Mr. Ople: The 'transactions' used here, I suppose is generic and therefore, it can cover
both steps leading to a contract and already a consummated contract, Mr. Presiding
Officer.
390

Mr. Suarez: This contemplates inclusion of negotiations leading to the consummation


of the transaction.

Mr. Ople: Yes, subject only to reasonable safeguards on the national interest.

Mr. Suarez: Thank you."32 (Emphasis supplied)

AMARI argues there must first be a consummated contract before petitioner can invoke the right.
Requiring government officials to reveal their deliberations at the pre-decisional stage will degrade the
quality of decision-making in government agencies. Government officials will hesitate to express their
real sentiments during deliberations if there is immediate public dissemination of their discussions,
putting them under all kinds of pressure before they decide.

We must first distinguish between information the law on public bidding requires PEA to disclose
publicly, and information the constitutional right to information requires PEA to release to the public.
Before the consummation of the contract, PEA must, on its own and without demand from anyone,
disclose to the public matters relating to the disposition of its property. These include the size,
location, technical description and nature of the property being disposed of, the terms and conditions
of the disposition, the parties qualified to bid, the minimum price and similar information. PEA must
prepare all these data and disclose them to the public at the start of the disposition process, long
before the consummation of the contract, because the Government Auditing Code requires public
bidding. If PEA fails to make this disclosure, any citizen can demand from PEA this information at
any time during the bidding process.

Information, however, on on-going evaluation or review of bids or proposals being undertaken by


the bidding or review committee is not immediately accessible under the right to information. While
the evaluation or review is still on-going, there are no "official acts, transactions, or decisions" on the
bids or proposals. However, once the committee makes its official recommendation, there arises
a "definite proposition" on the part of the government. From this moment, the public's right to
information attaches, and any citizen can access all the non-proprietary information leading to such
definite proposition. In Chavez v. PCGG,33 the Court ruled as follows:

"Considering the intent of the framers of the Constitution, we believe that it is incumbent upon
the PCGG and its officers, as well as other government representatives, to disclose sufficient
public information on any proposed settlement they have decided to take up with the
ostensible owners and holders of ill-gotten wealth. Such information, though, must pertain
to definite propositions of the government, not necessarily to intra-agency or inter-agency
recommendations or communications during the stage when common assertions are still in
the process of being formulated or are in the "exploratory" stage. There is need, of course, to
observe the same restrictions on disclosure of information in general, as discussed earlier –
such as on matters involving national security, diplomatic or foreign relations, intelligence and
other classified information." (Emphasis supplied)

Contrary to AMARI's contention, the commissioners of the 1986 Constitutional Commission


understood that the right to information "contemplates inclusion of negotiations leading to the
consummation of the transaction."Certainly, a consummated contract is not a requirement for the
exercise of the right to information. Otherwise, the people can never exercise the right if no contract is
consummated, and if one is consummated, it may be too late for the public to expose its
defects.1âwphi1.nêt

Requiring a consummated contract will keep the public in the dark until the contract, which may be
grossly disadvantageous to the government or even illegal, becomes a fait accompli. This negates the
State policy of full transparency on matters of public concern, a situation which the framers of the
Constitution could not have intended. Such a requirement will prevent the citizenry from participating
in the public discussion of any proposed contract, effectively truncating a basic right enshrined in the
Bill of Rights. We can allow neither an emasculation of a constitutional right, nor a retreat by the State
of its avowed "policy of full disclosure of all its transactions involving public interest."
391

The right covers three categories of information which are "matters of public concern," namely: (1)
official records; (2) documents and papers pertaining to official acts, transactions and decisions; and
(3) government research data used in formulating policies. The first category refers to any document
that is part of the public records in the custody of government agencies or officials. The second
category refers to documents and papers recording, evidencing, establishing, confirming, supporting,
justifying or explaining official acts, transactions or decisions of government agencies or officials. The
third category refers to research data, whether raw, collated or processed, owned by the government
and used in formulating government policies.

The information that petitioner may access on the renegotiation of the JVA includes evaluation
reports, recommendations, legal and expert opinions, minutes of meetings, terms of reference and
other documents attached to such reports or minutes, all relating to the JVA. However, the right to
information does not compel PEA to prepare lists, abstracts, summaries and the like relating to the
renegotiation of the JVA.34 The right only affords access to records, documents and papers, which
means the opportunity to inspect and copy them. One who exercises the right must copy the records,
documents and papers at his expense. The exercise of the right is also subject to reasonable
regulations to protect the integrity of the public records and to minimize disruption to government
operations, like rules specifying when and how to conduct the inspection and copying. 35

The right to information, however, does not extend to matters recognized as privileged information
under the separation of powers.36 The right does not also apply to information on military and
diplomatic secrets, information affecting national security, and information on investigations of crimes
by law enforcement agencies before the prosecution of the accused, which courts have long
recognized as confidential.37 The right may also be subject to other limitations that Congress may
impose by law.

There is no claim by PEA that the information demanded by petitioner is privileged information rooted
in the separation of powers. The information does not cover Presidential conversations,
correspondences, or discussions during closed-door Cabinet meetings which, like internal
deliberations of the Supreme Court and other collegiate courts, or executive sessions of either house
of Congress,38 are recognized as confidential. This kind of information cannot be pried open by a co-
equal branch of government. A frank exchange of exploratory ideas and assessments, free from the
glare of publicity and pressure by interested parties, is essential to protect the independence of
decision-making of those tasked to exercise Presidential, Legislative and Judicial power. 39 This is not
the situation in the instant case.

We rule, therefore, that the constitutional right to information includes official information on on-going
negotiationsbefore a final contract. The information, however, must constitute definite propositions
by the government and should not cover recognized exceptions like privileged information, military
and diplomatic secrets and similar matters affecting national security and public order. 40 Congress
has also prescribed other limitations on the right to information in several legislations. 41

Sixth issue: whether stipulations in the Amended JVA for the transfer to AMARI of lands,
reclaimed or to be reclaimed, violate the Constitution.

The Regalian Doctrine

The ownership of lands reclaimed from foreshore and submerged areas is rooted in the Regalian
doctrine which holds that the State owns all lands and waters of the public domain. Upon the Spanish
conquest of the Philippines, ownership of all "lands, territories and possessions" in the Philippines
passed to the Spanish Crown.42 The King, as the sovereign ruler and representative of the people,
acquired and owned all lands and territories in the Philippines except those he disposed of by grant or
sale to private individuals.

The 1935, 1973 and 1987 Constitutions adopted the Regalian doctrine substituting, however, the
State, in lieu of the King, as the owner of all lands and waters of the public domain. The Regalian
doctrine is the foundation of the time-honored principle of land ownership that "all lands that were not
acquired from the Government, either by purchase or by grant, belong to the public domain."43 Article
392

339 of the Civil Code of 1889, which is now Article 420 of the Civil Code of 1950, incorporated the
Regalian doctrine.

Ownership and Disposition of Reclaimed Lands

The Spanish Law of Waters of 1866 was the first statutory law governing the ownership and
disposition of reclaimed lands in the Philippines. On May 18, 1907, the Philippine Commission
enacted Act No. 1654 which provided for the lease, but not the sale, of reclaimed lands of the
government to corporations and individuals. Later, on November 29, 1919, the Philippine
Legislature approved Act No. 2874, the Public Land Act, which authorized the lease, but not the
sale, of reclaimed lands of the government to corporations and individuals. On November 7,
1936, the National Assembly passed Commonwealth Act No. 141, also known as the Public Land Act,
which authorized the lease, but not the sale, of reclaimed lands of the government to
corporations and individuals. CA No. 141 continues to this day as the general law governing the
classification and disposition of lands of the public domain.

The Spanish Law of Waters of 1866 and the Civil Code of 1889

Under the Spanish Law of Waters of 1866, the shores, bays, coves, inlets and all waters within the
maritime zone of the Spanish territory belonged to the public domain for public use.44 The Spanish
Law of Waters of 1866 allowed the reclamation of the sea under Article 5, which provided as follows:

"Article 5. Lands reclaimed from the sea in consequence of works constructed by the State, or
by the provinces, pueblos or private persons, with proper permission, shall become the
property of the party constructing such works, unless otherwise provided by the terms of the
grant of authority."

Under the Spanish Law of Waters, land reclaimed from the sea belonged to the party undertaking the
reclamation, provided the government issued the necessary permit and did not reserve ownership of
the reclaimed land to the State.

Article 339 of the Civil Code of 1889 defined property of public dominion as follows:

"Art. 339. Property of public dominion is –

1. That devoted to public use, such as roads, canals, rivers, torrents, ports and bridges
constructed by the State, riverbanks, shores, roadsteads, and that of a similar character;

2. That belonging exclusively to the State which, without being of general public use, is
employed in some public service, or in the development of the national wealth, such as walls,
fortresses, and other works for the defense of the territory, and mines, until granted to private
individuals."

Property devoted to public use referred to property open for use by the public. In contrast, property
devoted to public service referred to property used for some specific public service and open only to
those authorized to use the property.

Property of public dominion referred not only to property devoted to public use, but also to property
not so used but employed to develop the national wealth. This class of property constituted
property of public dominion although employed for some economic or commercial activity to increase
the national wealth.

Article 341 of the Civil Code of 1889 governed the re-classification of property of public dominion into
private property, to wit:

"Art. 341. Property of public dominion, when no longer devoted to public use or to the defense
of the territory, shall become a part of the private property of the State."
393

This provision, however, was not self-executing. The legislature, or the executive department
pursuant to law, must declare the property no longer needed for public use or territorial defense
before the government could lease or alienate the property to private parties. 45

Act No. 1654 of the Philippine Commission

On May 8, 1907, the Philippine Commission enacted Act No. 1654 which regulated the lease of
reclaimed and foreshore lands. The salient provisions of this law were as follows:

"Section 1. The control and disposition of the foreshore as defined in existing law, and
the title to all Government or public lands made or reclaimed by the Government by
dredging or filling or otherwise throughout the Philippine Islands, shall be retained by the
Government without prejudice to vested rights and without prejudice to rights conceded to
the City of Manila in the Luneta Extension.

Section 2. (a) The Secretary of the Interior shall cause all Government or public lands made
or reclaimed by the Government by dredging or filling or otherwise to be divided into lots or
blocks, with the necessary streets and alleyways located thereon, and shall cause plats and
plans of such surveys to be prepared and filed with the Bureau of Lands.

(b) Upon completion of such plats and plans the Governor-General shall give notice to the
public that such parts of the lands so made or reclaimed as are not needed for public
purposes will be leased for commercial and business purposes, x x x.

xxx

(e) The leases above provided for shall be disposed of to the highest and best
bidder therefore, subject to such regulations and safeguards as the Governor-General may
by executive order prescribe." (Emphasis supplied)

Act No. 1654 mandated that the government should retain title to all lands reclaimed by the
government. The Act also vested in the government control and disposition of foreshore lands.
Private parties could lease lands reclaimed by the government only if these lands were no longer
needed for public purpose. Act No. 1654 mandated public bidding in the lease of government
reclaimed lands. Act No. 1654 made government reclaimed lands sui generis in that unlike other
public lands which the government could sell to private parties, these reclaimed lands were available
only for lease to private parties.

Act No. 1654, however, did not repeal Section 5 of the Spanish Law of Waters of 1866. Act No. 1654
did not prohibit private parties from reclaiming parts of the sea under Section 5 of the Spanish Law of
Waters. Lands reclaimed from the sea by private parties with government permission remained
private lands.

Act No. 2874 of the Philippine Legislature

On November 29, 1919, the Philippine Legislature enacted Act No. 2874, the Public Land Act. 46 The
salient provisions of Act No. 2874, on reclaimed lands, were as follows:

"Sec. 6. The Governor-General, upon the recommendation of the Secretary of


Agriculture and Natural Resources, shall from time to time classify the lands of the
public domain into –

(a) Alienable or disposable,

(b) Timber, and

(c) Mineral lands, x x x.


394

Sec. 7. For the purposes of the government and disposition of alienable or disposable public
lands, the Governor-General, upon recommendation by the Secretary of Agriculture
and Natural Resources, shall from time to time declare what lands are open to
disposition or concession under this Act."

Sec. 8. Only those lands shall be declared open to disposition or concession which
have been officially delimited or classified x x x.

xxx

Sec. 55. Any tract of land of the public domain which, being neither timber nor mineral land,
shall be classified as suitable for residential purposes or for commercial, industrial, or
other productive purposes other than agricultural purposes, and shall be open to
disposition or concession, shall be disposed of under the provisions of this chapter, and not
otherwise.

Sec. 56. The lands disposable under this title shall be classified as follows:

(a) Lands reclaimed by the Government by dredging, filling, or other means;

(b) Foreshore;

(c) Marshy lands or lands covered with water bordering upon the shores or banks of
navigable lakes or rivers;

(d) Lands not included in any of the foregoing classes.

x x x.

Sec. 58. The lands comprised in classes (a), (b), and (c) of section fifty-six shall be
disposed of to private parties by lease only and not otherwise, as soon as the
Governor-General, upon recommendation by the Secretary of Agriculture and Natural
Resources, shall declare that the same are not necessary for the public service and are
open to disposition under this chapter. The lands included in class (d) may be disposed
of by sale or lease under the provisions of this Act." (Emphasis supplied)

Section 6 of Act No. 2874 authorized the Governor-General to "classify lands of the public domain
into x x x alienable or disposable"47 lands. Section 7 of the Act empowered the Governor-General to
"declare what lands are open to disposition or concession." Section 8 of the Act limited alienable or
disposable lands only to those lands which have been "officially delimited and classified."

Section 56 of Act No. 2874 stated that lands "disposable under this title 48 shall be classified" as
government reclaimed, foreshore and marshy lands, as well as other lands. All these lands, however,
must be suitable for residential, commercial, industrial or other productive non-agricultural purposes.
These provisions vested upon the Governor-General the power to classify inalienable lands of the
public domain into disposable lands of the public domain. These provisions also empowered the
Governor-General to classify further such disposable lands of the public domain into government
reclaimed, foreshore or marshy lands of the public domain, as well as other non-agricultural lands.

Section 58 of Act No. 2874 categorically mandated that disposable lands of the public domain
classified as government reclaimed, foreshore and marshy lands "shall be disposed of to private
parties by lease only and not otherwise." The Governor-General, before allowing the lease of
these lands to private parties, must formally declare that the lands were "not necessary for the public
service." Act No. 2874 reiterated the State policy to lease and not to sell government reclaimed,
foreshore and marshy lands of the public domain, a policy first enunciated in 1907 in Act No. 1654.
Government reclaimed, foreshore and marshy lands remained sui generis, as the only alienable or
disposable lands of the public domain that the government could not sell to private parties.
395

The rationale behind this State policy is obvious. Government reclaimed, foreshore and marshy public
lands for non-agricultural purposes retain their inherent potential as areas for public service. This is
the reason the government prohibited the sale, and only allowed the lease, of these lands to private
parties. The State always reserved these lands for some future public service.

Act No. 2874 did not authorize the reclassification of government reclaimed, foreshore and marshy
lands into other non-agricultural lands under Section 56 (d). Lands falling under Section 56 (d) were
the only lands for non-agricultural purposes the government could sell to private parties. Thus, under
Act No. 2874, the government could not sell government reclaimed, foreshore and marshy lands to
private parties, unless the legislature passed a law allowing their sale.49

Act No. 2874 did not prohibit private parties from reclaiming parts of the sea pursuant to Section 5 of
the Spanish Law of Waters of 1866. Lands reclaimed from the sea by private parties with government
permission remained private lands.

Dispositions under the 1935 Constitution

On May 14, 1935, the 1935 Constitution took effect upon its ratification by the Filipino people. The
1935 Constitution, in adopting the Regalian doctrine, declared in Section 1, Article XIII, that –

"Section 1. All agricultural, timber, and mineral lands of the public domain, waters, minerals,
coal, petroleum, and other mineral oils, all forces of potential energy and other natural
resources of the Philippines belong to the State, and their disposition, exploitation,
development, or utilization shall be limited to citizens of the Philippines or to corporations or
associations at least sixty per centum of the capital of which is owned by such citizens,
subject to any existing right, grant, lease, or concession at the time of the inauguration of the
Government established under this Constitution. Natural resources, with the exception of
public agricultural land, shall not be alienated, and no license, concession, or lease for
the exploitation, development, or utilization of any of the natural resources shall be granted
for a period exceeding twenty-five years, renewable for another twenty-five years, except as
to water rights for irrigation, water supply, fisheries, or industrial uses other than the
development of water power, in which cases beneficial use may be the measure and limit of
the grant." (Emphasis supplied)

The 1935 Constitution barred the alienation of all natural resources except public agricultural lands,
which were the only natural resources the State could alienate. Thus, foreshore lands, considered
part of the State's natural resources, became inalienable by constitutional fiat, available only for lease
for 25 years, renewable for another 25 years. The government could alienate foreshore lands only
after these lands were reclaimed and classified as alienable agricultural lands of the public domain.
Government reclaimed and marshy lands of the public domain, being neither timber nor mineral
lands, fell under the classification of public agricultural lands. 50 However, government reclaimed and
marshy lands, although subject to classification as disposable public agricultural lands, could only be
leased and not sold to private parties because of Act No. 2874.

The prohibition on private parties from acquiring ownership of government reclaimed and marshy
lands of the public domain was only a statutory prohibition and the legislature could therefore remove
such prohibition. The 1935 Constitution did not prohibit individuals and corporations from acquiring
government reclaimed and marshy lands of the public domain that were classified as agricultural
lands under existing public land laws. Section 2, Article XIII of the 1935 Constitution provided as
follows:

"Section 2. No private corporation or association may acquire, lease, or hold public


agricultural lands in excess of one thousand and twenty four hectares, nor may any
individual acquire such lands by purchase in excess of one hundred and forty
hectares, or by lease in excess of one thousand and twenty-four hectares, or by
homestead in excess of twenty-four hectares. Lands adapted to grazing, not exceeding two
thousand hectares, may be leased to an individual, private corporation, or association."
(Emphasis supplied)
396

Still, after the effectivity of the 1935 Constitution, the legislature did not repeal Section 58 of Act No.
2874 to open for sale to private parties government reclaimed and marshy lands of the public domain.
On the contrary, the legislature continued the long established State policy of retaining for the
government title and ownership of government reclaimed and marshy lands of the public domain.

Commonwealth Act No. 141 of the Philippine National Assembly

On November 7, 1936, the National Assembly approved Commonwealth Act No. 141, also known as
the Public Land Act, which compiled the then existing laws on lands of the public domain. CA No.
141, as amended, remains to this day the existing general law governing the classification and
disposition of lands of the public domain other than timber and mineral lands. 51

Section 6 of CA No. 141 empowers the President to classify lands of the public domain into "alienable
or disposable"52 lands of the public domain, which prior to such classification are inalienable and
outside the commerce of man. Section 7 of CA No. 141 authorizes the President to "declare what
lands are open to disposition or concession." Section 8 of CA No. 141 states that the government can
declare open for disposition or concession only lands that are "officially delimited and classified."
Sections 6, 7 and 8 of CA No. 141 read as follows:

"Sec. 6. The President, upon the recommendation of the Secretary of Agriculture and
Commerce, shall from time to time classify the lands of the public domain into –

(a) Alienable or disposable,

(b) Timber, and

(c) Mineral lands,

and may at any time and in like manner transfer such lands from one class to another, 53 for
the purpose of their administration and disposition.

Sec. 7. For the purposes of the administration and disposition of alienable or disposable
public lands, the President, upon recommendation by the Secretary of Agriculture and
Commerce, shall from time to time declare what lands are open to disposition or
concession under this Act.

Sec. 8. Only those lands shall be declared open to disposition or concession which
have been officially delimited and classified and, when practicable, surveyed, and which
have not been reserved for public or quasi-public uses, nor appropriated by the
Government, nor in any manner become private property, nor those on which a private right
authorized and recognized by this Act or any other valid law may be claimed, or which, having
been reserved or appropriated, have ceased to be so. x x x."

Thus, before the government could alienate or dispose of lands of the public domain, the President
must first officially classify these lands as alienable or disposable, and then declare them open to
disposition or concession. There must be no law reserving these lands for public or quasi-public uses.

The salient provisions of CA No. 141, on government reclaimed, foreshore and marshy lands of the
public domain, are as follows:

"Sec. 58. Any tract of land of the public domain which, being neither timber nor mineral
land, is intended to be used for residential purposes or for commercial, industrial, or
other productive purposes other than agricultural, and is open to disposition or
concession, shall be disposed of under the provisions of this chapter and not
otherwise.

Sec. 59. The lands disposable under this title shall be classified as follows:
397

(a) Lands reclaimed by the Government by dredging, filling, or other means;

(b) Foreshore;

(c) Marshy lands or lands covered with water bordering upon the shores or banks of
navigable lakes or rivers;

(d) Lands not included in any of the foregoing classes.

Sec. 60. Any tract of land comprised under this title may be leased or sold, as the case may
be, to any person, corporation, or association authorized to purchase or lease public lands for
agricultural purposes. x x x.

Sec. 61. The lands comprised in classes (a), (b), and (c) of section fifty-nine shall be
disposed of to private parties by lease only and not otherwise, as soon as the
President, upon recommendation by the Secretary of Agriculture, shall declare that the
same are not necessary for the public service and are open to disposition under this
chapter. The lands included in class (d) may be disposed of by sale or lease under the
provisions of this Act." (Emphasis supplied)

Section 61 of CA No. 141 readopted, after the effectivity of the 1935 Constitution, Section 58 of Act
No. 2874 prohibiting the sale of government reclaimed, foreshore and marshy disposable lands of the
public domain. All these lands are intended for residential, commercial, industrial or other non-
agricultural purposes. As before, Section 61 allowed only the lease of such lands to private parties.
The government could sell to private parties only lands falling under Section 59 (d) of CA No. 141, or
those lands for non-agricultural purposes not classified as government reclaimed, foreshore and
marshy disposable lands of the public domain. Foreshore lands, however, became inalienable under
the 1935 Constitution which only allowed the lease of these lands to qualified private parties.

Section 58 of CA No. 141 expressly states that disposable lands of the public domain intended for
residential, commercial, industrial or other productive purposes other than agricultural "shall be
disposed of under the provisions of this chapter and not otherwise." Under Section 10 of CA No.
141, the term "disposition" includes lease of the land. Any disposition of government reclaimed,
foreshore and marshy disposable lands for non-agricultural purposes must comply with Chapter IX,
Title III of CA No. 141,54 unless a subsequent law amended or repealed these provisions.

In his concurring opinion in the landmark case of Republic Real Estate Corporation v. Court of
Appeals,55Justice Reynato S. Puno summarized succinctly the law on this matter, as follows:

"Foreshore lands are lands of public dominion intended for public use. So too are lands
reclaimed by the government by dredging, filling, or other means. Act 1654 mandated that the
control and disposition of the foreshore and lands under water remained in the national
government. Said law allowed only the 'leasing' of reclaimed land. The Public Land Acts of
1919 and 1936 also declared that the foreshore and lands reclaimed by the government were
to be "disposed of to private parties by lease only and not otherwise." Before leasing,
however, the Governor-General, upon recommendation of the Secretary of Agriculture and
Natural Resources, had first to determine that the land reclaimed was not necessary for the
public service. This requisite must have been met before the land could be disposed of. But
even then, the foreshore and lands under water were not to be alienated and sold to
private parties. The disposition of the reclaimed land was only by lease. The land
remained property of the State." (Emphasis supplied)

As observed by Justice Puno in his concurring opinion, "Commonwealth Act No. 141 has remained in
effect at present."

The State policy prohibiting the sale to private parties of government reclaimed, foreshore and marshy
alienable lands of the public domain, first implemented in 1907 was thus reaffirmed in CA No. 141
after the 1935 Constitution took effect. The prohibition on the sale of foreshore lands, however,
398

became a constitutional edict under the 1935 Constitution. Foreshore lands became inalienable as
natural resources of the State, unless reclaimed by the government and classified as agricultural
lands of the public domain, in which case they would fall under the classification of government
reclaimed lands.

After the effectivity of the 1935 Constitution, government reclaimed and marshy disposable lands of
the public domain continued to be only leased and not sold to private parties. 56 These lands
remained sui generis, as the only alienable or disposable lands of the public domain the government
could not sell to private parties.

Since then and until now, the only way the government can sell to private parties government
reclaimed and marshy disposable lands of the public domain is for the legislature to pass a law
authorizing such sale. CA No. 141 does not authorize the President to reclassify government
reclaimed and marshy lands into other non-agricultural lands under Section 59 (d). Lands classified
under Section 59 (d) are the only alienable or disposable lands for non-agricultural purposes that the
government could sell to private parties.

Moreover, Section 60 of CA No. 141 expressly requires congressional authority before lands under
Section 59 that the government previously transferred to government units or entities could be sold to
private parties. Section 60 of CA No. 141 declares that –

"Sec. 60. x x x The area so leased or sold shall be such as shall, in the judgment of the
Secretary of Agriculture and Natural Resources, be reasonably necessary for the purposes
for which such sale or lease is requested, and shall not exceed one hundred and forty-four
hectares: Provided, however, That this limitation shall not apply to grants, donations, or
transfers made to a province, municipality or branch or subdivision of the Government for the
purposes deemed by said entities conducive to the public interest; but the land so granted,
donated, or transferred to a province, municipality or branch or subdivision of the
Government shall not be alienated, encumbered, or otherwise disposed of in a manner
affecting its title, except when authorized by Congress: x x x." (Emphasis supplied)

The congressional authority required in Section 60 of CA No. 141 mirrors the legislative authority
required in Section 56 of Act No. 2874.

One reason for the congressional authority is that Section 60 of CA No. 141 exempted government
units and entities from the maximum area of public lands that could be acquired from the State. These
government units and entities should not just turn around and sell these lands to private parties in
violation of constitutional or statutory limitations. Otherwise, the transfer of lands for non-agricultural
purposes to government units and entities could be used to circumvent constitutional limitations on
ownership of alienable or disposable lands of the public domain. In the same manner, such transfers
could also be used to evade the statutory prohibition in CA No. 141 on the sale of government
reclaimed and marshy lands of the public domain to private parties. Section 60 of CA No. 141
constitutes by operation of law a lien on these lands.57

In case of sale or lease of disposable lands of the public domain falling under Section 59 of CA No.
141, Sections 63 and 67 require a public bidding. Sections 63 and 67 of CA No. 141 provide as
follows:

"Sec. 63. Whenever it is decided that lands covered by this chapter are not needed for public
purposes, the Director of Lands shall ask the Secretary of Agriculture and Commerce (now
the Secretary of Natural Resources) for authority to dispose of the same. Upon receipt of
such authority, the Director of Lands shall give notice by public advertisement in the same
manner as in the case of leases or sales of agricultural public land, x x x.

Sec. 67. The lease or sale shall be made by oral bidding; and adjudication shall be
made to the highest bidder. x x x." (Emphasis supplied)
399

Thus, CA No. 141 mandates the Government to put to public auction all leases or sales of alienable
or disposable lands of the public domain.58

Like Act No. 1654 and Act No. 2874 before it, CA No. 141 did not repeal Section 5 of the Spanish Law
of Waters of 1866. Private parties could still reclaim portions of the sea with government permission.
However, the reclaimed land could become private land only if classified as alienable
agricultural land of the public domain open to disposition under CA No. 141. The 1935 Constitution
prohibited the alienation of all natural resources except public agricultural lands.

The Civil Code of 1950

The Civil Code of 1950 readopted substantially the definition of property of public dominion found in
the Civil Code of 1889. Articles 420 and 422 of the Civil Code of 1950 state that –

"Art. 420. The following things are property of public dominion:

(1) Those intended for public use, such as roads, canals, rivers, torrents, ports and bridges
constructed by the State, banks, shores, roadsteads, and others of similar character;

(2) Those which belong to the State, without being for public use, and are intended for some
public service or for the development of the national wealth.

x x x.

Art. 422. Property of public dominion, when no longer intended for public use or for public
service, shall form part of the patrimonial property of the State."

Again, the government must formally declare that the property of public dominion is no longer needed
for public use or public service, before the same could be classified as patrimonial property of the
State.59 In the case of government reclaimed and marshy lands of the public domain, the declaration
of their being disposable, as well as the manner of their disposition, is governed by the applicable
provisions of CA No. 141.

Like the Civil Code of 1889, the Civil Code of 1950 included as property of public dominion those
properties of the State which, without being for public use, are intended for public service or the
"development of the national wealth." Thus, government reclaimed and marshy lands of the State,
even if not employed for public use or public service, if developed to enhance the national wealth, are
classified as property of public dominion.

Dispositions under the 1973 Constitution

The 1973 Constitution, which took effect on January 17, 1973, likewise adopted the Regalian
doctrine. Section 8, Article XIV of the 1973 Constitution stated that –

"Sec. 8. All lands of the public domain, waters, minerals, coal, petroleum and other mineral
oils, all forces of potential energy, fisheries, wildlife, and other natural resources of the
Philippines belong to the State. With the exception of agricultural, industrial or
commercial, residential, and resettlement lands of the public domain, natural
resources shall not be alienated, and no license, concession, or lease for the exploration,
development, exploitation, or utilization of any of the natural resources shall be granted for a
period exceeding twenty-five years, renewable for not more than twenty-five years, except as
to water rights for irrigation, water supply, fisheries, or industrial uses other than the
development of water power, in which cases, beneficial use may be the measure and the limit
of the grant." (Emphasis supplied)

The 1973 Constitution prohibited the alienation of all natural resources with the exception of
"agricultural, industrial or commercial, residential, and resettlement lands of the public domain." In
400

contrast, the 1935 Constitution barred the alienation of all natural resources except "public agricultural
lands." However, the term "public agricultural lands" in the 1935 Constitution encompassed industrial,
commercial, residential and resettlement lands of the public domain.60 If the land of public domain
were neither timber nor mineral land, it would fall under the classification of agricultural land of the
public domain. Both the 1935 and 1973 Constitutions, therefore, prohibited the alienation of all
natural resources except agricultural lands of the public domain.

The 1973 Constitution, however, limited the alienation of lands of the public domain to individuals who
were citizens of the Philippines. Private corporations, even if wholly owned by Philippine citizens,
were no longer allowed to acquire alienable lands of the public domain unlike in the 1935 Constitution.
Section 11, Article XIV of the 1973 Constitution declared that –

"Sec. 11. The Batasang Pambansa, taking into account conservation, ecological, and
development requirements of the natural resources, shall determine by law the size of land of
the public domain which may be developed, held or acquired by, or leased to, any qualified
individual, corporation, or association, and the conditions therefor. No private corporation or
association may hold alienable lands of the public domain except by lease not to
exceed one thousand hectares in area nor may any citizen hold such lands by lease in
excess of five hundred hectares or acquire by purchase, homestead or grant, in excess of
twenty-four hectares. No private corporation or association may hold by lease, concession,
license or permit, timber or forest lands and other timber or forest resources in excess of one
hundred thousand hectares. However, such area may be increased by the Batasang
Pambansa upon recommendation of the National Economic and Development Authority."
(Emphasis supplied)

Thus, under the 1973 Constitution, private corporations could hold alienable lands of the public
domain only through lease. Only individuals could now acquire alienable lands of the public domain,
and private corporations became absolutely barred from acquiring any kind of alienable land
of the public domain. The constitutional ban extended to all kinds of alienable lands of the public
domain, while the statutory ban under CA No. 141 applied only to government reclaimed, foreshore
and marshy alienable lands of the public domain.

PD No. 1084 Creating the Public Estates Authority

On February 4, 1977, then President Ferdinand Marcos issued Presidential Decree No. 1084 creating
PEA, a wholly government owned and controlled corporation with a special charter. Sections 4 and 8
of PD No. 1084, vests PEA with the following purposes and powers:

"Sec. 4. Purpose. The Authority is hereby created for the following purposes:

(a) To reclaim land, including foreshore and submerged areas, by dredging, filling or
other means, or to acquire reclaimed land;

(b) To develop, improve, acquire, administer, deal in, subdivide, dispose, lease and sell any
and all kinds of lands, buildings, estates and other forms of real property, owned, managed,
controlled and/or operated by the government;

(c) To provide for, operate or administer such service as may be necessary for the efficient,
economical and beneficial utilization of the above properties.

Sec. 5. Powers and functions of the Authority. The Authority shall, in carrying out the
purposes for which it is created, have the following powers and functions:

(a)To prescribe its by-laws.

xxx
401

(i) To hold lands of the public domain in excess of the area permitted to private
corporations by statute.

(j) To reclaim lands and to construct work across, or otherwise, any stream, watercourse,
canal, ditch, flume x x x.

xxx

(o) To perform such acts and exercise such functions as may be necessary for the attainment
of the purposes and objectives herein specified." (Emphasis supplied)

PD No. 1084 authorizes PEA to reclaim both foreshore and submerged areas of the public domain.
Foreshore areas are those covered and uncovered by the ebb and flow of the tide. 61 Submerged
areas are those permanently under water regardless of the ebb and flow of the tide.62 Foreshore and
submerged areas indisputably belong to the public domain63 and are inalienable unless reclaimed,
classified as alienable lands open to disposition, and further declared no longer needed for public
service.

The ban in the 1973 Constitution on private corporations from acquiring alienable lands of the public
domain did not apply to PEA since it was then, and until today, a fully owned government corporation.
The constitutional ban applied then, as it still applies now, only to "private corporations and
associations." PD No. 1084 expressly empowers PEA "to hold lands of the public domain" even
"in excess of the area permitted to private corporations by statute." Thus, PEA can hold title to
private lands, as well as title to lands of the public domain.

In order for PEA to sell its reclaimed foreshore and submerged alienable lands of the public domain,
there must be legislative authority empowering PEA to sell these lands. This legislative authority is
necessary in view of Section 60 of CA No.141, which states –

"Sec. 60. x x x; but the land so granted, donated or transferred to a province, municipality, or
branch or subdivision of the Government shall not be alienated, encumbered or otherwise
disposed of in a manner affecting its title, except when authorized by Congress; x x x."
(Emphasis supplied)

Without such legislative authority, PEA could not sell but only lease its reclaimed foreshore and
submerged alienable lands of the public domain. Nevertheless, any legislative authority granted to
PEA to sell its reclaimed alienable lands of the public domain would be subject to the constitutional
ban on private corporations from acquiring alienable lands of the public domain. Hence, such
legislative authority could only benefit private individuals.

Dispositions under the 1987 Constitution

The 1987 Constitution, like the 1935 and 1973 Constitutions before it, has adopted the Regalian
doctrine. The 1987 Constitution declares that all natural resources are "owned by the State," and
except for alienable agricultural lands of the public domain, natural resources cannot be alienated.
Sections 2 and 3, Article XII of the 1987 Constitution state that –

"Section 2. All lands of the public domain, waters, minerals, coal, petroleum and other mineral
oils, all forces of potential energy, fisheries, forests or timber, wildlife, flora and fauna, and
other natural resources are owned by the State. With the exception of agricultural lands,
all other natural resources shall not be alienated. The exploration, development, and
utilization of natural resources shall be under the full control and supervision of the State. x x
x.

Section 3. Lands of the public domain are classified into agricultural, forest or timber, mineral
lands, and national parks. Agricultural lands of the public domain may be further classified by
law according to the uses which they may be devoted. Alienable lands of the public
domain shall be limited to agricultural lands. Private corporations or associations may
402

not hold such alienable lands of the public domain except by lease, for a period not
exceeding twenty-five years, renewable for not more than twenty-five years, and not to
exceed one thousand hectares in area. Citizens of the Philippines may lease not more than
five hundred hectares, or acquire not more than twelve hectares thereof by purchase,
homestead, or grant.

Taking into account the requirements of conservation, ecology, and development, and subject
to the requirements of agrarian reform, the Congress shall determine, by law, the size of
lands of the public domain which may be acquired, developed, held, or leased and the
conditions therefor." (Emphasis supplied)

The 1987 Constitution continues the State policy in the 1973 Constitution banning private
corporations from acquiring any kind of alienable land of the public domain. Like the 1973
Constitution, the 1987 Constitution allows private corporations to hold alienable lands of the public
domain only through lease. As in the 1935 and 1973 Constitutions, the general law governing the
lease to private corporations of reclaimed, foreshore and marshy alienable lands of the public domain
is still CA No. 141.

The Rationale behind the Constitutional Ban

The rationale behind the constitutional ban on corporations from acquiring, except through lease,
alienable lands of the public domain is not well understood. During the deliberations of the 1986
Constitutional Commission, the commissioners probed the rationale behind this ban, thus:

"FR. BERNAS: Mr. Vice-President, my questions have reference to page 3, line 5 which says:

`No private corporation or association may hold alienable lands of the public domain except
by lease, not to exceed one thousand hectares in area.'

If we recall, this provision did not exist under the 1935 Constitution, but this was introduced in
the 1973 Constitution. In effect, it prohibits private corporations from acquiring alienable
public lands. But it has not been very clear in jurisprudence what the reason for this is.
In some of the cases decided in 1982 and 1983, it was indicated that the purpose of this
is to prevent large landholdings. Is that the intent of this provision?

MR. VILLEGAS: I think that is the spirit of the provision.

FR. BERNAS: In existing decisions involving the Iglesia ni Cristo, there were instances where
the Iglesia ni Cristo was not allowed to acquire a mere 313-square meter land where a chapel
stood because the Supreme Court said it would be in violation of this." (Emphasis supplied)

In Ayog v. Cusi,64 the Court explained the rationale behind this constitutional ban in this way:

"Indeed, one purpose of the constitutional prohibition against purchases of public agricultural
lands by private corporations is to equitably diffuse land ownership or to encourage 'owner-
cultivatorship and the economic family-size farm' and to prevent a recurrence of cases like the
instant case. Huge landholdings by corporations or private persons had spawned social
unrest."

However, if the constitutional intent is to prevent huge landholdings, the Constitution could have
simply limited the size of alienable lands of the public domain that corporations could acquire. The
Constitution could have followed the limitations on individuals, who could acquire not more than 24
hectares of alienable lands of the public domain under the 1973 Constitution, and not more than 12
hectares under the 1987 Constitution.

If the constitutional intent is to encourage economic family-size farms, placing the land in the name of
a corporation would be more effective in preventing the break-up of farmlands. If the farmland is
403

registered in the name of a corporation, upon the death of the owner, his heirs would inherit shares in
the corporation instead of subdivided parcels of the farmland. This would prevent the continuing
break-up of farmlands into smaller and smaller plots from one generation to the next.

In actual practice, the constitutional ban strengthens the constitutional limitation on individuals from
acquiring more than the allowed area of alienable lands of the public domain. Without the
constitutional ban, individuals who already acquired the maximum area of alienable lands of the public
domain could easily set up corporations to acquire more alienable public lands. An individual could
own as many corporations as his means would allow him. An individual could even hide his ownership
of a corporation by putting his nominees as stockholders of the corporation. The corporation is a
convenient vehicle to circumvent the constitutional limitation on acquisition by individuals of alienable
lands of the public domain.

The constitutional intent, under the 1973 and 1987 Constitutions, is to transfer ownership of only a
limited area of alienable land of the public domain to a qualified individual. This constitutional intent is
safeguarded by the provision prohibiting corporations from acquiring alienable lands of the public
domain, since the vehicle to circumvent the constitutional intent is removed. The available alienable
public lands are gradually decreasing in the face of an ever-growing population. The most effective
way to insure faithful adherence to this constitutional intent is to grant or sell alienable lands of the
public domain only to individuals. This, it would seem, is the practical benefit arising from the
constitutional ban.

The Amended Joint Venture Agreement

The subject matter of the Amended JVA, as stated in its second Whereas clause, consists of three
properties, namely:

1. "[T]hree partially reclaimed and substantially eroded islands along Emilio Aguinaldo
Boulevard in Paranaque and Las Pinas, Metro Manila, with a combined titled area of
1,578,441 square meters;"

2. "[A]nother area of 2,421,559 square meters contiguous to the three islands;" and

3. "[A]t AMARI's option as approved by PEA, an additional 350 hectares more or less to
regularize the configuration of the reclaimed area."65

PEA confirms that the Amended JVA involves "the development of the Freedom Islands and further
reclamation of about 250 hectares x x x," plus an option "granted to AMARI to subsequently reclaim
another 350 hectares x x x."66

In short, the Amended JVA covers a reclamation area of 750 hectares. Only 157.84 hectares of the
750-hectare reclamation project have been reclaimed, and the rest of the 592.15 hectares are
still submerged areas forming part of Manila Bay.

Under the Amended JVA, AMARI will reimburse PEA the sum of P1,894,129,200.00 for PEA's "actual
cost" in partially reclaiming the Freedom Islands. AMARI will also complete, at its own expense, the
reclamation of the Freedom Islands. AMARI will further shoulder all the reclamation costs of all the
other areas, totaling 592.15 hectares, still to be reclaimed. AMARI and PEA will share, in the
proportion of 70 percent and 30 percent, respectively, the total net usable area which is defined in the
Amended JVA as the total reclaimed area less 30 percent earmarked for common areas. Title to
AMARI's share in the net usable area, totaling 367.5 hectares, will be issued in the name of AMARI.
Section 5.2 (c) of the Amended JVA provides that –

"x x x, PEA shall have the duty to execute without delay the necessary deed of transfer or
conveyance of the title pertaining to AMARI's Land share based on the Land Allocation
Plan. PEA, when requested in writing by AMARI, shall then cause the issuance and
delivery of the proper certificates of title covering AMARI's Land Share in the name of
AMARI, x x x; provided, that if more than seventy percent (70%) of the titled area at any given
404

time pertains to AMARI, PEA shall deliver to AMARI only seventy percent (70%) of the titles
pertaining to AMARI, until such time when a corresponding proportionate area of additional
land pertaining to PEA has been titled." (Emphasis supplied)

Indisputably, under the Amended JVA AMARI will acquire and own a maximum of 367.5
hectares of reclaimed land which will be titled in its name.

To implement the Amended JVA, PEA delegated to the unincorporated PEA-AMARI joint venture
PEA's statutory authority, rights and privileges to reclaim foreshore and submerged areas in Manila
Bay. Section 3.2.a of the Amended JVA states that –

"PEA hereby contributes to the joint venture its rights and privileges to perform Rawland
Reclamation and Horizontal Development as well as own the Reclamation Area, thereby
granting the Joint Venture the full and exclusive right, authority and privilege to undertake the
Project in accordance with the Master Development Plan."

The Amended JVA is the product of a renegotiation of the original JVA dated April 25, 1995 and its
supplemental agreement dated August 9, 1995.

The Threshold Issue

The threshold issue is whether AMARI, a private corporation, can acquire and own under the
Amended JVA 367.5 hectares of reclaimed foreshore and submerged areas in Manila Bay in view of
Sections 2 and 3, Article XII of the 1987 Constitution which state that:

"Section 2. All lands of the public domain, waters, minerals, coal, petroleum, and other
mineral oils, all forces of potential energy, fisheries, forests or timber, wildlife, flora and fauna,
and other natural resources are owned by the State. With the exception of agricultural
lands, all other natural resources shall not be alienated. x x x.

xxx

Section 3. x x x Alienable lands of the public domain shall be limited to agricultural


lands. Private corporations or associations may not hold such alienable lands of the
public domain except by lease, x x x."(Emphasis supplied)

Classification of Reclaimed Foreshore and Submerged Areas

PEA readily concedes that lands reclaimed from foreshore or submerged areas of Manila Bay are
alienable or disposable lands of the public domain. In its Memorandum, 67 PEA admits that –

"Under the Public Land Act (CA 141, as amended), reclaimed lands are classified as
alienable and disposable lands of the public domain:

'Sec. 59. The lands disposable under this title shall be classified as follows:

(a) Lands reclaimed by the government by dredging, filling, or other means;

x x x.'" (Emphasis supplied)

Likewise, the Legal Task Force68 constituted under Presidential Administrative Order No. 365
admitted in its Report and Recommendation to then President Fidel V. Ramos, "[R]eclaimed lands
are classified as alienable and disposable lands of the public domain."69 The Legal Task Force
concluded that –

"D. Conclusion
405

Reclaimed lands are lands of the public domain. However, by statutory authority, the rights of
ownership and disposition over reclaimed lands have been transferred to PEA, by virtue of
which PEA, as owner, may validly convey the same to any qualified person without violating
the Constitution or any statute.

The constitutional provision prohibiting private corporations from holding public land, except
by lease (Sec. 3, Art. XVII,70 1987 Constitution), does not apply to reclaimed lands whose
ownership has passed on to PEA by statutory grant."

Under Section 2, Article XII of the 1987 Constitution, the foreshore and submerged areas of Manila
Bay are part of the "lands of the public domain, waters x x x and other natural resources" and
consequently "owned by the State." As such, foreshore and submerged areas "shall not be alienated,"
unless they are classified as "agricultural lands" of the public domain. The mere reclamation of these
areas by PEA does not convert these inalienable natural resources of the State into alienable or
disposable lands of the public domain. There must be a law or presidential proclamation officially
classifying these reclaimed lands as alienable or disposable and open to disposition or concession.
Moreover, these reclaimed lands cannot be classified as alienable or disposable if the law has
reserved them for some public or quasi-public use.71

Section 8 of CA No. 141 provides that "only those lands shall be declared open to disposition or
concession which have been officially delimited and classified."72 The President has the authority
to classify inalienable lands of the public domain into alienable or disposable lands of the public
domain, pursuant to Section 6 of CA No. 141. In Laurel vs. Garcia,73 the Executive Department
attempted to sell the Roppongi property in Tokyo, Japan, which was acquired by the Philippine
Government for use as the Chancery of the Philippine Embassy. Although the Chancery had
transferred to another location thirteen years earlier, the Court still ruled that, under Article 422 74 of
the Civil Code, a property of public dominion retains such character until formally declared otherwise.
The Court ruled that –

"The fact that the Roppongi site has not been used for a long time for actual Embassy service
does not automatically convert it to patrimonial property. Any such conversion happens only if
the property is withdrawn from public use (Cebu Oxygen and Acetylene Co. v. Bercilles, 66
SCRA 481 [1975]. A property continues to be part of the public domain, not available
for private appropriation or ownership 'until there is a formal declaration on the part of
the government to withdraw it from being such' (Ignacio v. Director of Lands, 108 Phil.
335 [1960]." (Emphasis supplied)

PD No. 1085, issued on February 4, 1977, authorized the issuance of special land patents for lands
reclaimed by PEA from the foreshore or submerged areas of Manila Bay. On January 19, 1988 then
President Corazon C. Aquino issued Special Patent No. 3517 in the name of PEA for the 157.84
hectares comprising the partially reclaimed Freedom Islands. Subsequently, on April 9, 1999 the
Register of Deeds of the Municipality of Paranaque issued TCT Nos. 7309, 7311 and 7312 in the
name of PEA pursuant to Section 103 of PD No. 1529 authorizing the issuance of certificates of title
corresponding to land patents. To this day, these certificates of title are still in the name of PEA.

PD No. 1085, coupled with President Aquino's actual issuance of a special patent covering the
Freedom Islands, is equivalent to an official proclamation classifying the Freedom Islands as alienable
or disposable lands of the public domain. PD No. 1085 and President Aquino's issuance of a land
patent also constitute a declaration that the Freedom Islands are no longer needed for public
service. The Freedom Islands are thus alienable or disposable lands of the public domain,
open to disposition or concession to qualified parties.

At the time then President Aquino issued Special Patent No. 3517, PEA had already reclaimed the
Freedom Islands although subsequently there were partial erosions on some areas. The government
had also completed the necessary surveys on these islands. Thus, the Freedom Islands were no
longer part of Manila Bay but part of the land mass. Section 3, Article XII of the 1987 Constitution
classifies lands of the public domain into "agricultural, forest or timber, mineral lands, and national
parks." Being neither timber, mineral, nor national park lands, the reclaimed Freedom Islands
necessarily fall under the classification of agricultural lands of the public domain. Under the 1987
406

Constitution, agricultural lands of the public domain are the only natural resources that the State may
alienate to qualified private parties. All other natural resources, such as the seas or bays, are "waters
x x x owned by the State" forming part of the public domain, and are inalienable pursuant to Section
2, Article XII of the 1987 Constitution.

AMARI claims that the Freedom Islands are private lands because CDCP, then a private corporation,
reclaimed the islands under a contract dated November 20, 1973 with the Commissioner of Public
Highways. AMARI, citing Article 5 of the Spanish Law of Waters of 1866, argues that "if the ownership
of reclaimed lands may be given to the party constructing the works, then it cannot be said that
reclaimed lands are lands of the public domain which the State may not alienate." 75 Article 5 of the
Spanish Law of Waters reads as follows:

"Article 5. Lands reclaimed from the sea in consequence of works constructed by the State, or
by the provinces, pueblos or private persons, with proper permission, shall become the
property of the party constructing such works, unless otherwise provided by the terms of
the grant of authority." (Emphasis supplied)

Under Article 5 of the Spanish Law of Waters of 1866, private parties could reclaim from the sea only
with "proper permission" from the State. Private parties could own the reclaimed land only if not
"otherwise provided by the terms of the grant of authority." This clearly meant that no one could
reclaim from the sea without permission from the State because the sea is property of public
dominion. It also meant that the State could grant or withhold ownership of the reclaimed land
because any reclaimed land, like the sea from which it emerged, belonged to the State. Thus, a
private person reclaiming from the sea without permission from the State could not acquire ownership
of the reclaimed land which would remain property of public dominion like the sea it replaced. 76 Article
5 of the Spanish Law of Waters of 1866 adopted the time-honored principle of land ownership that "all
lands that were not acquired from the government, either by purchase or by grant, belong to the
public domain."77

Article 5 of the Spanish Law of Waters must be read together with laws subsequently enacted on the
disposition of public lands. In particular, CA No. 141 requires that lands of the public domain must first
be classified as alienable or disposable before the government can alienate them. These lands must
not be reserved for public or quasi-public purposes.78 Moreover, the contract between CDCP and the
government was executed after the effectivity of the 1973 Constitution which barred private
corporations from acquiring any kind of alienable land of the public domain. This contract could not
have converted the Freedom Islands into private lands of a private corporation.

Presidential Decree No. 3-A, issued on January 11, 1973, revoked all laws authorizing the
reclamation of areas under water and revested solely in the National Government the power to
reclaim lands. Section 1 of PD No. 3-A declared that –

"The provisions of any law to the contrary notwithstanding, the reclamation of areas
under water, whether foreshore or inland, shall be limited to the National Government or
any person authorized by it under a proper contract. (Emphasis supplied)

x x x."

PD No. 3-A repealed Section 5 of the Spanish Law of Waters of 1866 because reclamation of areas
under water could now be undertaken only by the National Government or by a person contracted by
the National Government. Private parties may reclaim from the sea only under a contract with the
National Government, and no longer by grant or permission as provided in Section 5 of the Spanish
Law of Waters of 1866.

Executive Order No. 525, issued on February 14, 1979, designated PEA as the National
Government's implementing arm to undertake "all reclamation projects of the government," which
"shall be undertaken by the PEA or through a proper contract executed by it with any person
or entity." Under such contract, a private party receives compensation for reclamation services
rendered to PEA. Payment to the contractor may be in cash, or in kind consisting of portions of the
407

reclaimed land, subject to the constitutional ban on private corporations from acquiring alienable lands
of the public domain. The reclaimed land can be used as payment in kind only if the reclaimed land is
first classified as alienable or disposable land open to disposition, and then declared no longer
needed for public service.

The Amended JVA covers not only the Freedom Islands, but also an additional 592.15 hectares which
are still submerged and forming part of Manila Bay. There is no legislative or Presidential act
classifying these submerged areas as alienable or disposable lands of the public domain open
to disposition. These submerged areas are not covered by any patent or certificate of title. There
can be no dispute that these submerged areas form part of the public domain, and in their present
state are inalienable and outside the commerce of man. Until reclaimed from the sea, these
submerged areas are, under the Constitution, "waters x x x owned by the State," forming part of the
public domain and consequently inalienable. Only when actually reclaimed from the sea can these
submerged areas be classified as public agricultural lands, which under the Constitution are the only
natural resources that the State may alienate. Once reclaimed and transformed into public agricultural
lands, the government may then officially classify these lands as alienable or disposable lands open
to disposition. Thereafter, the government may declare these lands no longer needed for public
service. Only then can these reclaimed lands be considered alienable or disposable lands of the
public domain and within the commerce of man.

The classification of PEA's reclaimed foreshore and submerged lands into alienable or disposable
lands open to disposition is necessary because PEA is tasked under its charter to undertake public
services that require the use of lands of the public domain. Under Section 5 of PD No. 1084, the
functions of PEA include the following: "[T]o own or operate railroads, tramways and other kinds of
land transportation, x x x; [T]o construct, maintain and operate such systems of sanitary sewers as
may be necessary; [T]o construct, maintain and operate such storm drains as may be necessary."
PEA is empowered to issue "rules and regulations as may be necessary for the proper use by private
parties of any or all of the highways, roads, utilities, buildings and/or any of its properties and
to impose or collect fees or tolls for their use." Thus, part of the reclaimed foreshore and submerged
lands held by the PEA would actually be needed for public use or service since many of the functions
imposed on PEA by its charter constitute essential public services.

Moreover, Section 1 of Executive Order No. 525 provides that PEA "shall be primarily responsible for
integrating, directing, and coordinating all reclamation projects for and on behalf of the National
Government." The same section also states that "[A]ll reclamation projects shall be approved by the
President upon recommendation of the PEA, and shall be undertaken by the PEA or through a proper
contract executed by it with any person or entity; x x x." Thus, under EO No. 525, in relation to PD No.
3-A and PD No.1084, PEA became the primary implementing agency of the National Government to
reclaim foreshore and submerged lands of the public domain. EO No. 525 recognized PEA as the
government entity "to undertake the reclamation of lands and ensure their maximum utilization
in promoting public welfare and interests."79 Since large portions of these reclaimed lands would
obviously be needed for public service, there must be a formal declaration segregating reclaimed
lands no longer needed for public service from those still needed for public service.1âwphi1.nêt

Section 3 of EO No. 525, by declaring that all lands reclaimed by PEA "shall belong to or be owned by
the PEA," could not automatically operate to classify inalienable lands into alienable or disposable
lands of the public domain. Otherwise, reclaimed foreshore and submerged lands of the public
domain would automatically become alienable once reclaimed by PEA, whether or not classified as
alienable or disposable.

The Revised Administrative Code of 1987, a later law than either PD No. 1084 or EO No. 525, vests
in the Department of Environment and Natural Resources ("DENR" for brevity) the following powers
and functions:

"Sec. 4. Powers and Functions. The Department shall:

(1) x x x
408

xxx

(4) Exercise supervision and control over forest lands, alienable and disposable public
lands, mineral resources and, in the process of exercising such control, impose appropriate
taxes, fees, charges, rentals and any such form of levy and collect such revenues for the
exploration, development, utilization or gathering of such resources;

xxx

(14) Promulgate rules, regulations and guidelines on the issuance of licenses, permits,
concessions, lease agreements and such other privileges concerning the
development, exploration and utilization of the country's marine, freshwater, and
brackish water and over all aquatic resources of the country and shall continue to
oversee, supervise and police our natural resources; cancel or cause to cancel such
privileges upon failure, non-compliance or violations of any regulation, order, and for all other
causes which are in furtherance of the conservation of natural resources and supportive of
the national interest;

(15) Exercise exclusive jurisdiction on the management and disposition of all lands of
the public domain and serve as the sole agency responsible for classification, sub-
classification, surveying and titling of lands in consultation with appropriate
agencies."80 (Emphasis supplied)

As manager, conservator and overseer of the natural resources of the State, DENR exercises
"supervision and control over alienable and disposable public lands." DENR also exercises "exclusive
jurisdiction on the management and disposition of all lands of the public domain." Thus, DENR
decides whether areas under water, like foreshore or submerged areas of Manila Bay, should be
reclaimed or not. This means that PEA needs authorization from DENR before PEA can undertake
reclamation projects in Manila Bay, or in any part of the country.

DENR also exercises exclusive jurisdiction over the disposition of all lands of the public domain.
Hence, DENR decides whether reclaimed lands of PEA should be classified as alienable under
Sections 681 and 782 of CA No. 141. Once DENR decides that the reclaimed lands should be so
classified, it then recommends to the President the issuance of a proclamation classifying the lands
as alienable or disposable lands of the public domain open to disposition. We note that then DENR
Secretary Fulgencio S. Factoran, Jr. countersigned Special Patent No. 3517 in compliance with the
Revised Administrative Code and Sections 6 and 7 of CA No. 141.

In short, DENR is vested with the power to authorize the reclamation of areas under water, while PEA
is vested with the power to undertake the physical reclamation of areas under water, whether directly
or through private contractors. DENR is also empowered to classify lands of the public domain into
alienable or disposable lands subject to the approval of the President. On the other hand, PEA is
tasked to develop, sell or lease the reclaimed alienable lands of the public domain.

Clearly, the mere physical act of reclamation by PEA of foreshore or submerged areas does not make
the reclaimed lands alienable or disposable lands of the public domain, much less patrimonial lands of
PEA. Likewise, the mere transfer by the National Government of lands of the public domain to PEA
does not make the lands alienable or disposable lands of the public domain, much less patrimonial
lands of PEA.

Absent two official acts – a classification that these lands are alienable or disposable and open to
disposition and a declaration that these lands are not needed for public service, lands reclaimed by
PEA remain inalienable lands of the public domain. Only such an official classification and formal
declaration can convert reclaimed lands into alienable or disposable lands of the public domain, open
to disposition under the Constitution, Title I and Title III 83 of CA No. 141 and other applicable laws.84

PEA's Authority to Sell Reclaimed Lands


409

PEA, like the Legal Task Force, argues that as alienable or disposable lands of the public domain, the
reclaimed lands shall be disposed of in accordance with CA No. 141, the Public Land Act. PEA, citing
Section 60 of CA No. 141, admits that reclaimed lands transferred to a branch or subdivision of the
government "shall not be alienated, encumbered, or otherwise disposed of in a manner affecting its
title, except when authorized by Congress: x x x."85 (Emphasis by PEA)

In Laurel vs. Garcia,86 the Court cited Section 48 of the Revised Administrative Code of 1987, which
states that –

"Sec. 48. Official Authorized to Convey Real Property. Whenever real property of the
Government is authorized by law to be conveyed, the deed of conveyance shall be
executed in behalf of the government by the following: x x x."

Thus, the Court concluded that a law is needed to convey any real property belonging to the
Government. The Court declared that -

"It is not for the President to convey real property of the government on his or her own sole
will. Any such conveyance must be authorized and approved by a law enacted by the
Congress. It requires executive and legislative concurrence." (Emphasis supplied)

PEA contends that PD No. 1085 and EO No. 525 constitute the legislative authority allowing PEA to
sell its reclaimed lands. PD No. 1085, issued on February 4, 1977, provides that –

"The land reclaimed in the foreshore and offshore area of Manila Bay pursuant to the
contract for the reclamation and construction of the Manila-Cavite Coastal Road Project
between the Republic of the Philippines and the Construction and Development Corporation
of the Philippines dated November 20, 1973 and/or any other contract or reclamation
covering the same area is hereby transferred, conveyed and assigned to the ownership
and administration of the Public Estates Authority established pursuant to PD No. 1084;
Provided, however, That the rights and interests of the Construction and Development
Corporation of the Philippines pursuant to the aforesaid contract shall be recognized and
respected.

Henceforth, the Public Estates Authority shall exercise the rights and assume the obligations
of the Republic of the Philippines (Department of Public Highways) arising from, or incident
to, the aforesaid contract between the Republic of the Philippines and the Construction and
Development Corporation of the Philippines.

In consideration of the foregoing transfer and assignment, the Public Estates Authority shall
issue in favor of the Republic of the Philippines the corresponding shares of stock in said
entity with an issued value of said shares of stock (which) shall be deemed fully paid and non-
assessable.

The Secretary of Public Highways and the General Manager of the Public Estates Authority
shall execute such contracts or agreements, including appropriate agreements with the
Construction and Development Corporation of the Philippines, as may be necessary to
implement the above.

Special land patent/patents shall be issued by the Secretary of Natural Resources in


favor of the Public Estates Authority without prejudice to the subsequent transfer to
the contractor or his assignees of such portion or portions of the land reclaimed or to
be reclaimed as provided for in the above-mentioned contract. On the basis of such
patents, the Land Registration Commission shall issue the corresponding certificate of
title." (Emphasis supplied)

On the other hand, Section 3 of EO No. 525, issued on February 14, 1979, provides that -
410

"Sec. 3. All lands reclaimed by PEA shall belong to or be owned by the PEA which shall
be responsible for its administration, development, utilization or disposition in accordance with
the provisions of Presidential Decree No. 1084. Any and all income that the PEA may derive
from the sale, lease or use of reclaimed lands shall be used in accordance with the provisions
of Presidential Decree No. 1084."

There is no express authority under either PD No. 1085 or EO No. 525 for PEA to sell its reclaimed
lands. PD No. 1085 merely transferred "ownership and administration" of lands reclaimed from Manila
Bay to PEA, while EO No. 525 declared that lands reclaimed by PEA "shall belong to or be owned by
PEA." EO No. 525 expressly states that PEA should dispose of its reclaimed lands "in accordance
with the provisions of Presidential Decree No. 1084," the charter of PEA.

PEA's charter, however, expressly tasks PEA "to develop, improve, acquire, administer, deal in,
subdivide, dispose, lease and sell any and all kinds of lands x x x owned, managed, controlled
and/or operated by the government."87(Emphasis supplied) There is, therefore, legislative authority
granted to PEA to sell its lands, whether patrimonial or alienable lands of the public domain.
PEA may sell to private parties its patrimonial propertiesin accordance with the PEA charter free
from constitutional limitations. The constitutional ban on private corporations from acquiring alienable
lands of the public domain does not apply to the sale of PEA's patrimonial lands.

PEA may also sell its alienable or disposable lands of the public domain to private individuals
since, with the legislative authority, there is no longer any statutory prohibition against such sales and
the constitutional ban does not apply to individuals. PEA, however, cannot sell any of its alienable or
disposable lands of the public domain to private corporations since Section 3, Article XII of the 1987
Constitution expressly prohibits such sales. The legislative authority benefits only individuals. Private
corporations remain barred from acquiring any kind of alienable land of the public domain, including
government reclaimed lands.

The provision in PD No. 1085 stating that portions of the reclaimed lands could be transferred by PEA
to the "contractor or his assignees" (Emphasis supplied) would not apply to private corporations but
only to individuals because of the constitutional ban. Otherwise, the provisions of PD No. 1085 would
violate both the 1973 and 1987 Constitutions.

The requirement of public auction in the sale of reclaimed lands

Assuming the reclaimed lands of PEA are classified as alienable or disposable lands open to
disposition, and further declared no longer needed for public service, PEA would have to conduct a
public bidding in selling or leasing these lands. PEA must observe the provisions of Sections 63 and
67 of CA No. 141 requiring public auction, in the absence of a law exempting PEA from holding a
public auction.88 Special Patent No. 3517 expressly states that the patent is issued by authority of the
Constitution and PD No. 1084, "supplemented by Commonwealth Act No. 141, as amended." This is
an acknowledgment that the provisions of CA No. 141 apply to the disposition of reclaimed alienable
lands of the public domain unless otherwise provided by law. Executive Order No. 654,89 which
authorizes PEA "to determine the kind and manner of payment for the transfer" of its assets and
properties, does not exempt PEA from the requirement of public auction. EO No. 654 merely
authorizes PEA to decide the mode of payment, whether in kind and in installment, but does not
authorize PEA to dispense with public auction.

Moreover, under Section 79 of PD No. 1445, otherwise known as the Government Auditing Code, the
government is required to sell valuable government property through public bidding. Section 79 of PD
No. 1445 mandates that –

"Section 79. When government property has become unserviceable for any cause, or is no
longer needed, it shall, upon application of the officer accountable therefor, be inspected by
the head of the agency or his duly authorized representative in the presence of the auditor
concerned and, if found to be valueless or unsaleable, it may be destroyed in their
presence. If found to be valuable, it may be sold at public auction to the highest
bidder under the supervision of the proper committee on award or similar body in the
411

presence of the auditor concerned or other authorized representative of the


Commission, after advertising by printed notice in the Official Gazette, or for not less
than three consecutive days in any newspaper of general circulation, or where the value
of the property does not warrant the expense of publication, by notices posted for a like
period in at least three public places in the locality where the property is to be sold. In the
event that the public auction fails, the property may be sold at a private sale at such
price as may be fixed by the same committee or body concerned and approved by the
Commission."

It is only when the public auction fails that a negotiated sale is allowed, in which case the Commission
on Audit must approve the selling price.90 The Commission on Audit implements Section 79 of the
Government Auditing Code through Circular No. 89-29691 dated January 27, 1989. This circular
emphasizes that government assets must be disposed of only through public auction, and a
negotiated sale can be resorted to only in case of "failure of public auction."

At the public auction sale, only Philippine citizens are qualified to bid for PEA's reclaimed foreshore
and submerged alienable lands of the public domain. Private corporations are barred from bidding at
the auction sale of any kind of alienable land of the public domain.

PEA originally scheduled a public bidding for the Freedom Islands on December 10, 1991. PEA
imposed a condition that the winning bidder should reclaim another 250 hectares of submerged areas
to regularize the shape of the Freedom Islands, under a 60-40 sharing of the additional reclaimed
areas in favor of the winning bidder.92 No one, however, submitted a bid. On December 23, 1994, the
Government Corporate Counsel advised PEA it could sell the Freedom Islands through negotiation,
without need of another public bidding, because of the failure of the public bidding on December 10,
1991.93

However, the original JVA dated April 25, 1995 covered not only the Freedom Islands and the
additional 250 hectares still to be reclaimed, it also granted an option to AMARI to reclaim another
350 hectares. The original JVA, a negotiated contract, enlarged the reclamation area to 750
hectares.94 The failure of public bidding on December 10, 1991, involving only 407.84 hectares, 95 is
not a valid justification for a negotiated sale of 750 hectares, almost double the area publicly
auctioned. Besides, the failure of public bidding happened on December 10, 1991, more than three
years before the signing of the original JVA on April 25, 1995. The economic situation in the country
had greatly improved during the intervening period.

Reclamation under the BOT Law and the Local Government Code

The constitutional prohibition in Section 3, Article XII of the 1987 Constitution is absolute and clear:
"Private corporations or associations may not hold such alienable lands of the public domain except
by lease, x x x." Even Republic Act No. 6957 ("BOT Law," for brevity), cited by PEA and AMARI as
legislative authority to sell reclaimed lands to private parties, recognizes the constitutional ban.
Section 6 of RA No. 6957 states –

"Sec. 6. Repayment Scheme. - For the financing, construction, operation and maintenance of
any infrastructure projects undertaken through the build-operate-and-transfer arrangement or
any of its variations pursuant to the provisions of this Act, the project proponent x x x may
likewise be repaid in the form of a share in the revenue of the project or other non-monetary
payments, such as, but not limited to, the grant of a portion or percentage of the reclaimed
land, subject to the constitutional requirements with respect to the ownership of the
land: x x x." (Emphasis supplied)

A private corporation, even one that undertakes the physical reclamation of a government BOT
project, cannot acquire reclaimed alienable lands of the public domain in view of the constitutional
ban.
412

Section 302 of the Local Government Code, also mentioned by PEA and AMARI, authorizes local
governments in land reclamation projects to pay the contractor or developer in kind consisting of a
percentage of the reclaimed land, to wit:

"Section 302. Financing, Construction, Maintenance, Operation, and Management of


Infrastructure Projects by the Private Sector. x x x

xxx

In case of land reclamation or construction of industrial estates, the repayment plan may
consist of the grant of a portion or percentage of the reclaimed land or the industrial estate
constructed."

Although Section 302 of the Local Government Code does not contain a proviso similar to that of the
BOT Law, the constitutional restrictions on land ownership automatically apply even though not
expressly mentioned in the Local Government Code.

Thus, under either the BOT Law or the Local Government Code, the contractor or developer, if a
corporate entity, can only be paid with leaseholds on portions of the reclaimed land. If the contractor
or developer is an individual, portions of the reclaimed land, not exceeding 12 hectares 96 of non-
agricultural lands, may be conveyed to him in ownership in view of the legislative authority allowing
such conveyance. This is the only way these provisions of the BOT Law and the Local Government
Code can avoid a direct collision with Section 3, Article XII of the 1987 Constitution.

Registration of lands of the public domain

Finally, PEA theorizes that the "act of conveying the ownership of the reclaimed lands to public
respondent PEA transformed such lands of the public domain to private lands." This theory is echoed
by AMARI which maintains that the "issuance of the special patent leading to the eventual issuance of
title takes the subject land away from the land of public domain and converts the property into
patrimonial or private property." In short, PEA and AMARI contend that with the issuance of Special
Patent No. 3517 and the corresponding certificates of titles, the 157.84 hectares comprising the
Freedom Islands have become private lands of PEA. In support of their theory, PEA and AMARI cite
the following rulings of the Court:

1. Sumail v. Judge of CFI of Cotabato,97 where the Court held –

"Once the patent was granted and the corresponding certificate of title was issued, the land
ceased to be part of the public domain and became private property over which the Director
of Lands has neither control nor jurisdiction."

2. Lee Hong Hok v. David,98 where the Court declared -

"After the registration and issuance of the certificate and duplicate certificate of title based on
a public land patent, the land covered thereby automatically comes under the operation of
Republic Act 496 subject to all the safeguards provided therein."3. Heirs of Gregorio Tengco
v. Heirs of Jose Aliwalas,99 where the Court ruled -

"While the Director of Lands has the power to review homestead patents, he may do so only
so long as the land remains part of the public domain and continues to be under his exclusive
control; but once the patent is registered and a certificate of title is issued, the land ceases to
be part of the public domain and becomes private property over which the Director of Lands
has neither control nor jurisdiction."

4. Manalo v. Intermediate Appellate Court,100 where the Court held –


413

"When the lots in dispute were certified as disposable on May 19, 1971, and free patents
were issued covering the same in favor of the private respondents, the said lots ceased to be
part of the public domain and, therefore, the Director of Lands lost jurisdiction over the same."

5.Republic v. Court of Appeals,101 where the Court stated –

"Proclamation No. 350, dated October 9, 1956, of President Magsaysay legally effected a
land grant to the Mindanao Medical Center, Bureau of Medical Services, Department of
Health, of the whole lot, validly sufficient for initial registration under the Land Registration
Act. Such land grant is constitutive of a 'fee simple' title or absolute title in favor of petitioner
Mindanao Medical Center. Thus, Section 122 of the Act, which governs the registration of
grants or patents involving public lands, provides that 'Whenever public lands in the Philippine
Islands belonging to the Government of the United States or to the Government of the
Philippines are alienated, granted or conveyed to persons or to public or private corporations,
the same shall be brought forthwith under the operation of this Act (Land Registration Act, Act
496) and shall become registered lands.'"

The first four cases cited involve petitions to cancel the land patents and the corresponding
certificates of titles issued to private parties. These four cases uniformly hold that the Director of
Lands has no jurisdiction over private lands or that upon issuance of the certificate of title the land
automatically comes under the Torrens System. The fifth case cited involves the registration under
the Torrens System of a 12.8-hectare public land granted by the National Government to Mindanao
Medical Center, a government unit under the Department of Health. The National Government
transferred the 12.8-hectare public land to serve as the site for the hospital buildings and other
facilities of Mindanao Medical Center, which performed a public service. The Court affirmed the
registration of the 12.8-hectare public land in the name of Mindanao Medical Center under Section
122 of Act No. 496. This fifth case is an example of a public land being registered under Act No. 496
without the land losing its character as a property of public dominion.

In the instant case, the only patent and certificates of title issued are those in the name of PEA, a
wholly government owned corporation performing public as well as proprietary functions. No patent or
certificate of title has been issued to any private party. No one is asking the Director of Lands to
cancel PEA's patent or certificates of title. In fact, the thrust of the instant petition is that PEA's
certificates of title should remain with PEA, and the land covered by these certificates, being alienable
lands of the public domain, should not be sold to a private corporation.

Registration of land under Act No. 496 or PD No. 1529 does not vest in the registrant private or public
ownership of the land. Registration is not a mode of acquiring ownership but is merely evidence of
ownership previously conferred by any of the recognized modes of acquiring ownership. Registration
does not give the registrant a better right than what the registrant had prior to the registration. 102 The
registration of lands of the public domain under the Torrens system, by itself, cannot convert public
lands into private lands.103

Jurisprudence holding that upon the grant of the patent or issuance of the certificate of title the
alienable land of the public domain automatically becomes private land cannot apply to government
units and entities like PEA. The transfer of the Freedom Islands to PEA was made subject to the
provisions of CA No. 141 as expressly stated in Special Patent No. 3517 issued by then President
Aquino, to wit:

"NOW, THEREFORE, KNOW YE, that by authority of the Constitution of the Philippines and
in conformity with the provisions of Presidential Decree No. 1084, supplemented by
Commonwealth Act No. 141, as amended, there are hereby granted and conveyed unto the
Public Estates Authority the aforesaid tracts of land containing a total area of one million nine
hundred fifteen thousand eight hundred ninety four (1,915,894) square meters; the technical
description of which are hereto attached and made an integral part hereof." (Emphasis
supplied)
414

Thus, the provisions of CA No. 141 apply to the Freedom Islands on matters not covered by PD No.
1084. Section 60 of CA No. 141 prohibits, "except when authorized by Congress," the sale of
alienable lands of the public domain that are transferred to government units or entities. Section 60 of
CA No. 141 constitutes, under Section 44 of PD No. 1529, a "statutory lien affecting title" of the
registered land even if not annotated on the certificate of title.104Alienable lands of the public domain
held by government entities under Section 60 of CA No. 141 remain public lands because they cannot
be alienated or encumbered unless Congress passes a law authorizing their disposition. Congress,
however, cannot authorize the sale to private corporations of reclaimed alienable lands of the public
domain because of the constitutional ban. Only individuals can benefit from such law.

The grant of legislative authority to sell public lands in accordance with Section 60 of CA No. 141
does not automatically convert alienable lands of the public domain into private or patrimonial lands.
The alienable lands of the public domain must be transferred to qualified private parties, or to
government entities not tasked to dispose of public lands, before these lands can become private or
patrimonial lands. Otherwise, the constitutional ban will become illusory if Congress can declare lands
of the public domain as private or patrimonial lands in the hands of a government agency tasked to
dispose of public lands. This will allow private corporations to acquire directly from government
agencies limitless areas of lands which, prior to such law, are concededly public lands.

Under EO No. 525, PEA became the central implementing agency of the National Government to
reclaim foreshore and submerged areas of the public domain. Thus, EO No. 525 declares that –

"EXECUTIVE ORDER NO. 525

Designating the Public Estates Authority as the Agency Primarily Responsible for all
Reclamation Projects

Whereas, there are several reclamation projects which are ongoing or being proposed to be
undertaken in various parts of the country which need to be evaluated for consistency with
national programs;

Whereas, there is a need to give further institutional support to the Government's declared
policy to provide for a coordinated, economical and efficient reclamation of lands;

Whereas, Presidential Decree No. 3-A requires that all reclamation of areas shall be limited to
the National Government or any person authorized by it under proper contract;

Whereas, a central authority is needed to act on behalf of the National Government


which shall ensure a coordinated and integrated approach in the reclamation of lands;

Whereas, Presidential Decree No. 1084 creates the Public Estates Authority as a
government corporation to undertake reclamation of lands and ensure their maximum
utilization in promoting public welfare and interests; and

Whereas, Presidential Decree No. 1416 provides the President with continuing authority to
reorganize the national government including the transfer, abolition, or merger of functions
and offices.

NOW, THEREFORE, I, FERDINAND E. MARCOS, President of the Philippines, by virtue of


the powers vested in me by the Constitution and pursuant to Presidential Decree No. 1416,
do hereby order and direct the following:

Section 1. The Public Estates Authority (PEA) shall be primarily responsible for
integrating, directing, and coordinating all reclamation projects for and on behalf of the
National Government. All reclamation projects shall be approved by the President upon
recommendation of the PEA, and shall be undertaken by the PEA or through a proper
contract executed by it with any person or entity; Provided, that, reclamation projects of any
415

national government agency or entity authorized under its charter shall be undertaken in
consultation with the PEA upon approval of the President.

x x x ."

As the central implementing agency tasked to undertake reclamation projects nationwide, with
authority to sell reclaimed lands, PEA took the place of DENR as the government agency charged
with leasing or selling reclaimed lands of the public domain. The reclaimed lands being leased or sold
by PEA are not private lands, in the same manner that DENR, when it disposes of other alienable
lands, does not dispose of private lands but alienable lands of the public domain. Only when qualified
private parties acquire these lands will the lands become private lands. In the hands of the
government agency tasked and authorized to dispose of alienable of disposable lands of the
public domain, these lands are still public, not private lands.

Furthermore, PEA's charter expressly states that PEA "shall hold lands of the public domain" as
well as "any and all kinds of lands." PEA can hold both lands of the public domain and private lands.
Thus, the mere fact that alienable lands of the public domain like the Freedom Islands are transferred
to PEA and issued land patents or certificates of title in PEA's name does not automatically make
such lands private.

To allow vast areas of reclaimed lands of the public domain to be transferred to PEA as private lands
will sanction a gross violation of the constitutional ban on private corporations from acquiring any kind
of alienable land of the public domain. PEA will simply turn around, as PEA has now done under the
Amended JVA, and transfer several hundreds of hectares of these reclaimed and still to be reclaimed
lands to a single private corporation in only one transaction. This scheme will effectively nullify the
constitutional ban in Section 3, Article XII of the 1987 Constitution which was intended to diffuse
equitably the ownership of alienable lands of the public domain among Filipinos, now numbering over
80 million strong.

This scheme, if allowed, can even be applied to alienable agricultural lands of the public domain since
PEA can "acquire x x x any and all kinds of lands." This will open the floodgates to corporations and
even individuals acquiring hundreds of hectares of alienable lands of the public domain under the
guise that in the hands of PEA these lands are private lands. This will result in corporations amassing
huge landholdings never before seen in this country - creating the very evil that the constitutional ban
was designed to prevent. This will completely reverse the clear direction of constitutional development
in this country. The 1935 Constitution allowed private corporations to acquire not more than 1,024
hectares of public lands.105 The 1973 Constitution prohibited private corporations from acquiring any
kind of public land, and the 1987 Constitution has unequivocally reiterated this prohibition.

The contention of PEA and AMARI that public lands, once registered under Act No. 496 or PD No.
1529, automatically become private lands is contrary to existing laws. Several laws authorize lands of
the public domain to be registered under the Torrens System or Act No. 496, now PD No. 1529,
without losing their character as public lands. Section 122 of Act No. 496, and Section 103 of PD No.
1529, respectively, provide as follows:

Act No. 496

"Sec. 122. Whenever public lands in the Philippine Islands belonging to the x x x Government
of the Philippine Islands are alienated, granted, or conveyed to persons or the public or
private corporations, the same shall be brought forthwith under the operation of this Act and
shall become registered lands."

PD No. 1529

"Sec. 103. Certificate of Title to Patents. Whenever public land is by the Government
alienated, granted or conveyed to any person, the same shall be brought forthwith under the
operation of this Decree." (Emphasis supplied)
416

Based on its legislative history, the phrase "conveyed to any person" in Section 103 of PD No. 1529
includes conveyances of public lands to public corporations.

Alienable lands of the public domain "granted, donated, or transferred to a province, municipality, or
branch or subdivision of the Government," as provided in Section 60 of CA No. 141, may be
registered under the Torrens System pursuant to Section 103 of PD No. 1529. Such registration,
however, is expressly subject to the condition in Section 60 of CA No. 141 that the land "shall not be
alienated, encumbered or otherwise disposed of in a manner affecting its title, except when
authorized by Congress." This provision refers to government reclaimed, foreshore and marshy
lands of the public domain that have been titled but still cannot be alienated or encumbered unless
expressly authorized by Congress. The need for legislative authority prevents the registered land of
the public domain from becoming private land that can be disposed of to qualified private parties.

The Revised Administrative Code of 1987 also recognizes that lands of the public domain may be
registered under the Torrens System. Section 48, Chapter 12, Book I of the Code states –

"Sec. 48. Official Authorized to Convey Real Property. Whenever real property of the
Government is authorized by law to be conveyed, the deed of conveyance shall be executed
in behalf of the government by the following:

(1) x x x

(2) For property belonging to the Republic of the Philippines, but titled in the name of
any political subdivision or of any corporate agency or instrumentality, by the executive
head of the agency or instrumentality." (Emphasis supplied)

Thus, private property purchased by the National Government for expansion of a public wharf may be
titled in the name of a government corporation regulating port operations in the country. Private
property purchased by the National Government for expansion of an airport may also be titled in the
name of the government agency tasked to administer the airport. Private property donated to a
municipality for use as a town plaza or public school site may likewise be titled in the name of the
municipality.106 All these properties become properties of the public domain, and if already registered
under Act No. 496 or PD No. 1529, remain registered land. There is no requirement or provision in
any existing law for the de-registration of land from the Torrens System.

Private lands taken by the Government for public use under its power of eminent domain become
unquestionably part of the public domain. Nevertheless, Section 85 of PD No. 1529 authorizes the
Register of Deeds to issue in the name of the National Government new certificates of title covering
such expropriated lands. Section 85 of PD No. 1529 states –

"Sec. 85. Land taken by eminent domain. Whenever any registered land, or interest therein, is
expropriated or taken by eminent domain, the National Government, province, city or
municipality, or any other agency or instrumentality exercising such right shall file for
registration in the proper Registry a certified copy of the judgment which shall state definitely
by an adequate description, the particular property or interest expropriated, the number of the
certificate of title, and the nature of the public use. A memorandum of the right or interest
taken shall be made on each certificate of title by the Register of Deeds, and where the fee
simple is taken, a new certificate shall be issued in favor of the National Government,
province, city, municipality, or any other agency or instrumentality exercising such right for
the land so taken. The legal expenses incident to the memorandum of registration or
issuance of a new certificate of title shall be for the account of the authority taking the land or
interest therein." (Emphasis supplied)

Consequently, lands registered under Act No. 496 or PD No. 1529 are not exclusively private or
patrimonial lands. Lands of the public domain may also be registered pursuant to existing laws.

AMARI makes a parting shot that the Amended JVA is not a sale to AMARI of the Freedom Islands or
of the lands to be reclaimed from submerged areas of Manila Bay. In the words of AMARI, the
417

Amended JVA "is not a sale but a joint venture with a stipulation for reimbursement of the original cost
incurred by PEA for the earlier reclamation and construction works performed by the CDCP under its
1973 contract with the Republic." Whether the Amended JVA is a sale or a joint venture, the fact
remains that the Amended JVA requires PEA to "cause the issuance and delivery of the certificates of
title conveying AMARI's Land Share in the name of AMARI."107

This stipulation still contravenes Section 3, Article XII of the 1987 Constitution which provides that
private corporations "shall not hold such alienable lands of the public domain except by lease." The
transfer of title and ownership to AMARI clearly means that AMARI will "hold" the reclaimed lands
other than by lease. The transfer of title and ownership is a "disposition" of the reclaimed lands, a
transaction considered a sale or alienation under CA No. 141,108 the Government Auditing
Code,109 and Section 3, Article XII of the 1987 Constitution.

The Regalian doctrine is deeply implanted in our legal system. Foreshore and submerged areas form
part of the public domain and are inalienable. Lands reclaimed from foreshore and submerged areas
also form part of the public domain and are also inalienable, unless converted pursuant to law into
alienable or disposable lands of the public domain. Historically, lands reclaimed by the government
are sui generis, not available for sale to private parties unlike other alienable public lands. Reclaimed
lands retain their inherent potential as areas for public use or public service. Alienable lands of the
public domain, increasingly becoming scarce natural resources, are to be distributed equitably among
our ever-growing population. To insure such equitable distribution, the 1973 and 1987 Constitutions
have barred private corporations from acquiring any kind of alienable land of the public domain.
Those who attempt to dispose of inalienable natural resources of the State, or seek to circumvent the
constitutional ban on alienation of lands of the public domain to private corporations, do so at their
own risk.

We can now summarize our conclusions as follows:

1. The 157.84 hectares of reclaimed lands comprising the Freedom Islands, now covered by
certificates of title in the name of PEA, are alienable lands of the public domain. PEA may
lease these lands to private corporations but may not sell or transfer ownership of these lands
to private corporations. PEA may only sell these lands to Philippine citizens, subject to the
ownership limitations in the 1987 Constitution and existing laws.

2. The 592.15 hectares of submerged areas of Manila Bay remain inalienable natural
resources of the public domain until classified as alienable or disposable lands open to
disposition and declared no longer needed for public service. The government can make such
classification and declaration only after PEA has reclaimed these submerged areas. Only
then can these lands qualify as agricultural lands of the public domain, which are the only
natural resources the government can alienate. In their present state, the 592.15 hectares of
submerged areas are inalienable and outside the commerce of man.

3. Since the Amended JVA seeks to transfer to AMARI, a private corporation, ownership of
77.34 hectares110of the Freedom Islands, such transfer is void for being contrary to Section 3,
Article XII of the 1987 Constitution which prohibits private corporations from acquiring any
kind of alienable land of the public domain.

4. Since the Amended JVA also seeks to transfer to AMARI ownership of 290.156
hectares111 of still submerged areas of Manila Bay, such transfer is void for being contrary to
Section 2, Article XII of the 1987 Constitution which prohibits the alienation of natural
resources other than agricultural lands of the public domain. PEA may reclaim these
submerged areas. Thereafter, the government can classify the reclaimed lands as alienable
or disposable, and further declare them no longer needed for public service. Still, the transfer
of such reclaimed alienable lands of the public domain to AMARI will be void in view of
Section 3, Article XII of the 1987 Constitution which prohibits private corporations from
acquiring any kind of alienable land of the public domain.
418

Clearly, the Amended JVA violates glaringly Sections 2 and 3, Article XII of the 1987 Constitution.
Under Article 1409112 of the Civil Code, contracts whose "object or purpose is contrary to law," or
whose "object is outside the commerce of men," are "inexistent and void from the beginning." The
Court must perform its duty to defend and uphold the Constitution, and therefore declares the
Amended JVA null and void ab initio.

Seventh issue: whether the Court is the proper forum to raise the issue of whether the
Amended JVA is grossly disadvantageous to the government.

Considering that the Amended JVA is null and void ab initio, there is no necessity to rule on this last
issue. Besides, the Court is not a trier of facts, and this last issue involves a determination of factual
matters.

WHEREFORE, the petition is GRANTED. The Public Estates Authority and Amari Coastal Bay
Development Corporation are PERMANENTLY ENJOINED from implementing the Amended Joint
Venture Agreement which is hereby declared NULL and VOID ab initio.

SO ORDERED.

Davide, Jr., C.J., Bellosillo, Puno, Vitug, Kapunan, Mendoza, Panganiban, Quisumbing, Ynares-
Santiago, Sandoval-Gutierrez, Austria-Martinez, and Corona, JJ., concur.

Footnote

1 Section 4 of PD No. 1084.

2 PEA's Memorandum dated August 4, 1999, p. 3.

3PEA's Memorandum, supra note 2 at 7. PEA's Memorandum quoted extensively, in its


Statement of Facts and the Case, the Statement of Facts in Senate Committee Report No.
560 dated September 16, 1997.

4In Opinion No. 330 dated December 23, 1994, the Government Corporate Counsel, citing
COA Audit Circular No. 89-296, advised PEA that PEA could negotiate the sale of the 157.84-
hectare Freedom Islands in view of the failure of the public bidding held on December 10,
1991 where there was not a single bidder. See also Senate Committee Report No. 560, p. 12.

5 PEA's Memorandum, supra note 2 at 9.

6 Ibid.

7 The existence of this report is a matter of judicial notice pursuant to Section 1, Rule 129 of
the Rules of Court which provides, "A court shall take judicial notice, without the introduction
of evidence, of x x x the official acts of the legislature x x x."

8 Teofisto Guingona, Jr.

9 Renato Cayetano.

10 Virgilio C. Abejo.

11Report and Recommendation of the Legal Task Force, Annex "C", AMARI's Memorandum
dated June 19, 1999.
419

12 AMARI's Comment dated June 24, 1998, p. 3; Rollo, p. 68.

13AMARI filed three motions for extension of time to file comment (Rollo, pp. 32, 38, 48),
while PEA filed nine motions for extension of time (Rollo, pp. 127, 139).

14 Petitioner's Memorandum dated July 6, 1999, p. 42.

15
Represented by the Office of the Solicitor General, with Solicitor General Ricardo P.
Galvez, Assistant Solicitor General Azucena R. Balanon-Corpuz, and Associate Solicitor
Raymund I. Rigodon signing PEA's Memorandum.

16Represented by Azcuna Yorac Arroyo & Chua Law Offices, and Romulo Mabanta Sayoc &
De los Angeles Law Offices.

17Salonga v. Paño, 134 SCRA 438 (1985); Gonzales v. Marcos, 65 SCRA 624 (1975 );
Aquino v. Enrile, 59 SCRA 183 (1974 ); Dela Camara v. Enage, 41 SCRA 1 (1971 ).

18 Section 11, Article XIV.

19 Manila Electric Co. v. Judge F. Castro-Bartolome, 114 SCRA 799 (1982); Republic v. CA
and Iglesia, and Republic v. Cendana and Iglesia ni Cristo, 119 SCRA 449 (1982); Republic
v. Villanueva and Iglesia ni Cristo, 114 SCRA 875 (1982); Director of Lands v. Lood, 124
SCRA 460 (1983); Republic v. Iglesia ni Cristo, 128 SCRA 44 (1984); Director of Lands v.
Hermanos y Hermanas de Sta. Cruz de Mayo, Inc., 141 SCRA 21 (1986); Director of Lands v.
IAC and Acme Plywood & Veneer Co., 146 SCRA 509 (1986); Republic v. IAC and Roman
Catholic Bishop of Lucena, 168 SCRA 165 (1988); Natividad v. CA, 202 SCRA 493 (1991);
Villaflor v. CA and Nasipit Lumber Co., 280 SCRA 297 (1997). In Ayog v. Cusi, 118 SCRA
492 (1982), the Court did not apply the constitutional ban in the 1973 Constitution because
the applicant corporation, Biñan Development Co., Inc., had fully complied with all its
obligations and even paid the full purchase price before the effectivity of the 1973
Constitution, although the sales patent was issued after the 1973 Constitution took effect.

20 PD No. 1073.

21
Annex "B", AMARI's Memorandum dated June 19, 1999, Section 5.2 (c) and (e) of the
Amended JVA, pp. 16-17.

22 Chavez v. PCGG, 299 SCRA 744 (1998).

23 136 SCRA 27 (1985).

24Article 2 of the Civil Code (prior to its amendment by EO No. 200) provided as follows:
"Laws shall take effect after fifteen days following the completion of their publication in the
Official Gazette, unless it is provided otherwise, x x x."

25Section 1 of CA No. 638 provides as follows: "There shall be published in the Official
Gazette all important legislative acts and resolutions of the Congress of the Philippines; all
executive and administrative orders and proclamations, except such as have no general
applicability; x x x."

26Section 79 of the Government Auditing Codes provides as follows: "When government


property has become unserviceable for any cause, or is no longer needed, it shall, upon
application of the officer accountable therefor, be inspected by the head of the agency or his
duly authorized representative in the presence of the auditor concerned and, if found to be
valueless or unsaleable, it may be destroyed in their presence. If found to be valuable, it
may be sold at public auction to the highest bidder under the supervision of the proper
committee on award or similar body in the presence of the auditor concerned or other
420

authorized representative of the Commission, after advertising by printed notice in the


Official Gazette, or for not less than three consecutive days in any newspaper of
general circulation, or where the value of the property does not warrant the expense of
publication, by notices posted for a like period in at least three public places in the locality
where the property is to be sold. In the event that the public auction fails, the property
may be sold at a private sale at such price as may be fixed by the same committee or
body concerned and approved by the Commission."

27
Paat v. Court of Appeals, 266 SCRA 167 (1997); Quisumbing v. Judge Gumban, 193
SCRA 520 (1991); Valmonte v. Belmonte, Jr., 170 SCRA 256 (1989).

28 See note 22.

29 Section 1, Article XI of the 1987 Constitution states as follows: "Public office is a public
trust. Public officers and employees must at all times be accountable to the people, serve
them with utmost responsibility, integrity, loyalty, and efficiency, act with patriotism and
justice, and lead modest lives."

30 170 SCRA 256 (1989).

31 See note 22.

32 Record of the Constitutional Commission, Vol. V, pp. 24-25, (1986).

33 Supra, Note 22.

34 Ibid.

35 Legaspi v. Civil Service Commission, 150 SCRA 530 (1987).

36 Almonte v. Vasquez, 244 SCRA 286 (1995).

37 See Note 22.

38 Chavez v. PCGG, see note 22; Aquino-Sarmiento v. Morato, 203 SCRA 515 (1991).

39 Almonte v. Vasquez, see note 36.

40People's Movement for Press Freedom, et al. v. Hon. Raul Manglapus, G.R. No. 84642, En
Banc Resolution dated April 13, 1988; Chavez v. PCGG, see note 22.

41 Section 270 of the National Internal Revenue Code punishes any officer or employee of the
Bureau of Internal Revenue who divulges to any person, except as allowed by law,
information regarding the business, income, or estate of any taxpayer, the secrets, operation,
style of work, or apparatus of any manufacturer or producer, or confidential information
regarding the business of any taxpayer, knowledge of which was acquired by him in the
discharge of his official duties. Section 14 of R.A. No. 8800 (Safeguard Measures Act)
prohibits the release to the public of confidential information submitted in evidence to the
Tariff Commission. Section 3 (n) of R.A. No. 8504 (Philippine AIDS Prevention and Control
Act) classifies as confidential the medical records of HIV patients. Section 6 (j) of R.A. No.
8043 (Inter-Country Adoption Act) classifies as confidential the records of the adopted child,
adopting parents, and natural parents. Section 94 (f) of R.A. No. 7942 (Philippine Mining Act)
requires the Department of Environment and Natural Resources to maintain the confidentiality
of confidential information supplied by contractors who are parties to mineral agreements or
financial and technical assistance agreements.
421

42The Recopilacion de Leyes de las Indias declared that: "We, having acquired full
sovereignty over the Indies, and all lands, territories, and possessions not heretofore ceded
away by our royal predecessors, or by us, or in our name, still pertaining to the royal crown
and patrimony, it is our will that all lands which are held without proper and true deeds of
grant be restored to us according as they belong to us, in order that after reserving before all
what to us or to our viceroys, audiencias, and governors may seem necessary for public
squares, ways, pastures, and commons in those places which are peopled, taking into
consideration not only their present condition, but also their future and their probable
increase, and after distributing to the natives what may be necessary for tillage and
pasturage, confirming them in what they now have and giving them more if necessary, all the
rest of said lands may remain free and unencumbered for us to dispose of as we may
wish." See concurring opinion of Justice Reynato S. Puno in Republic Real Estate
Corporation v. Court of Appeals, 299 SCRA 199 (1998).

43 Cariño v. Insular Government, 41 Phil. 935 (1909). The exception mentioned in Cariño,
referring to lands in the possession of an occupant and of his predecessors-in-interest, since
time immemorial, is actually a species of a grant by the State. The United States Supreme
Court, speaking through Justice Oliver Wendell Holmes, Jr., declared in Cariño: "Prescription
is mentioned again in the royal cedula of October 15, 1754, cited in 3 Philippine, 546; 'Where
such possessors shall not be able to produce title deeds, it shall be sufficient if they shall
show that ancient possession, as a valid title by prescription.' It may be that this means
possession from before 1700; but, at all events, the principle is admitted. As prescription,
even against the Crown lands, was recognized by the laws of Spain, we see no sufficient
reason for hesitating to admit that it was recognized in the Philippines in regard to lands over
which Spain had only a paper sovereignty." See also Republic v. Lee, 197 SCRA 13 (1991).

44 Article 1 of the Spanish Law of Waters of 1866.

45Ignacio v. Director of Lands, 108 Phil. 335 (1960); Joven v. Director of Lands, 93 Phil. 134
(1953); Laurel v. Garcia, 187 SCRA 797 (1990). See concurring opinion of Justice Reynato S.
Puno in Republic Real Estate Corporation v. Court of Appeals, 299 SCRA 199 (1998).

46 Act No. 926, enacted on October 7, 1903, was also titled the Public Land Act. This Act,
however, did not cover reclaimed lands. Nevertheless, Section 23 of this Act provided as
follows: "x x x In no case may lands leased under the provisions of this chapter be taken so
as to gain control of adjacent land, water, stream, shore line, way, roadstead, or other
valuable right which in the opinion of the Chief of the Bureau of Public Lands would be
prejudicial to the interests of the public."

47Section 10 of Act No. 2874 provided as follows: "The words "alienation," "disposition," or
"concession" as used in this Act, shall mean any of the methods authorized by this Act for the
acquisition, lease, use, or benefit of the lands of the public domain other than timber or
mineral lands."

48Title II of Act No. 2874 governed alienable lands of the public domain for agricultural
purposes, while Title III of the same Act governed alienable lands of the public domain for
non-agricultural purposes.

49 Section 57 of Act No. 2874 provided as follows: "x x x; but the land so granted, donated, or
transferred to a province, municipality, or branch or subdivision of the Government shall not
be alienated, encumbered, or otherwise disposed of in a manner affecting its title, except
when authorized by the legislature; x x x."

50 Krivenko v. Register of Deeds, 79 Phil. 461 (1947).

51Section 2 of CA No. 141 states as follows: "The provisions of this Act shall apply to the
lands of the public domain; but timber and mineral lands shall be governed by special laws
and nothing in this Act provided shall be understood or construed to change or modify the
422

administration and disposition of the lands commonly called "friar lands" and those which,
being privately owned, have reverted to or become the property of the Commonwealth of the
Philippines, which administration and disposition shall be governed by the laws at present in
force or which may hereafter be enacted."

52Like Act No. 2874, Section 10 of CA No. 141 defined the terms "alienation" and
"disposition" as follows: "The words "alienation," "disposition," or "concession" as used in this
Act, shall mean any of the methods authorized by this Act for the acquisition, lease, use, or
benefit of the lands of the public domain other than timber or mineral lands."

53R.A. No. 6657 has suspended the authority of the President to reclassify forest or mineral
lands into agricultural lands. Section 4 (a) of RA No. 6657 (Comprehensive Agrarian Reform
Law of 1988) states, "No reclassification of forest or mineral lands to agricultural lands shall
be undertaken after the approval of this Act until Congress, taking into account ecological,
developmental and equity considerations, shall have delimited by law, the specific limits of the
public domain."

54 Covering Sections 58 to 68 of CA No. 141.

55 299 SCRA 199 (1998).

56 Section 1, Article XIII of the 1935 Constitution limited the disposition and utilization of public
agricultural lands to Philippine citizens or to corporations at least sixty percent owned by
Philippine citizens. This was, however, subject to the original Ordinance appended to the
1935 Constitution stating, among others, that until the withdrawal of United States sovereignty
in the Philippines, "Citizens and corporations of the United States shall enjoy in the
Commonwealth of the Philippines all the civil rights of the citizens and corporations,
respectively, thereof."

57 Section 44 of PD No. 1529 (previously Section 39 of Act No. 496) provides that "liens,
claims or rights arising or existing under the laws and the Constitution of the Philippines which
are not by law required to appear of record in the Registry of Deeds in order to be valid
against subsequent purchasers or encumbrancers of record" constitute statutory liens
affecting the title.1âwphi1.nêt

58 RA No. 730, which took effect on June 18, 1952, authorized the private sale of home lots
to actual occupants of public lands not needed for public service. Section 1 of RA No. 730
provided as follows: "Notwithstanding the provisions of Sections 61 and 67 of Commonwealth
Act No. 141, as amended by RA No. 293, any Filipino citizen of legal age who is not the
owner of a home lot in the municipality or city in which he resides and who had in good faith
established his residence on a parcel of land of the Republic of the Philippines which is not
needed for public service, shall be given preference to purchase at a private sale of which
reasonable notice shall be given to him, not more than one thousand square meters at a price
to be fixed by the Director of Lands with the approval of the Secretary of Agriculture and
Natural Resources. x x x." In addition, on June 16, 1948, Congress enacted R.A. No. 293
allowing the private sale of marshy alienable or disposable lands of the public domain to
lessees who have improved and utilized the same as farms, fishponds or other similar
purposes for at least five years from the date of the lease contract with the government. R.A.
No. 293, however, did not apply to marshy lands under Section 56 (c), Title III of CA No. 141
which refers to marshy lands leased for residential, commercial, industrial or other non-
agricultural purposes.

59 See note 49.

60 See note 60.

61 Republic Real Estate Corporation v. Court of Appeals, see note 56.


423

62 Ibid.

63Insular Government v. Aldecoa, 19 Phil. 505 (1911); Government v. Cabangis, 53 Phil. 112
(1929).

64 118 SCRA 492 (1982).

65 Annex "B", AMARI's Memorandum, see note 2 at 1 & 2.

66 PEA's Memorandum, see note 6.

67 Ibid., p. 44.

68 See notes 9, 10 & 11.

69 Annex "C", p. 3, AMARI's Memorandum, see note 12 at 3.

70 This should read Article XII.

71 Section 8 of CA No. 141.

72 Emphasis supplied.

73 187 SCRA 797 (1990).

74Article 422 of the Civil Code states as follows: "Property of public dominion, when no longer
needed for public use or public service, shall form part of the patrimonial property of the
State."

75 AMARI's Comment dated June 24, 1998, p. 20; Rollo, p. 85.

76Dizon v. Rodriguez, 13 SCRA 705 (1965); Republic v. Lat Vda. de Castillo, 163 SCRA 286
(1988).

77 Cariño v. Insular Government, 41 Phil. 935 (1909).

78 Proclamation No. 41, issued by President Ramon Magsaysay on July 5, 1954, reserved for
"National Park purposes" 464.66 hectares of the public domain in Manila Bay "situated in the
cities of Manila and Pasay and the municipality of Paranaque, Province of Rizal, Island of
Luzon," which area, as described in detail in the Proclamation, is "B]ounded on the North, by
Manila Bay; on the East, by Dewey Boulevard; and on the south and west, by Manila Bay."
See concurring opinion of Justice Reynato S. Puno in Republic Real Estate Corporation v.
Court of Appeals, 299 SCRA 1999 (1998). Under Sections 2 and 3, Article XII of the 1987
Constitution, "national parks" are inalienable natural resources of the State.

79 Fifth Whereas clause of EO No. 525.

80 Section 4, Chapter I, Title XIV, Book IV.

81Section 6 of CA No 141 provides as follows: "The President, upon the recommendation


of the Secretary of Agriculture and Commerce, shall from time to time classify the lands of
the public domain into – (a) Alienable or disposable, x x x."

82Section 7 of CA No. 141 provides as follows: "For purposes of the administration and
disposition of alienable or disposable public lands, the President, upon recommendation by
424

the Secretary of Agriculture and Commerce, shall from time to time declare what lands are
open to disposition or concession under this Act."

83 On "Lands for Residential, Commercial, or Industrial and other Similar Purposes."

84 RA No. 293, enacted on June 16, 1948, authorized the sale of marshy lands under certain
conditions. Section 1 of RA No. 293 provided as follows: "The provisions of section sixty-one
of Commonwealth Act Numbered One hundred and forty-one to the contrary notwithstanding,
marshy lands and lands under water bordering on shores or banks or navigable lakes or
rivers which are covered by subsisting leases or leases which may hereafter be duly granted
under the provisions of the said Act and are already improved and have been utilized for
farming, fishpond, or similar purposes for at least five years from the date of the contract of
lease, may be sold to the lessees thereof under the provisions of Chapter Five of the said Act
as soon as the President, upon recommendation of the Secretary of Agriculture and Natural
Resources, shall declare that the same are not necessary for the public service."

85 PEA's Memorandum, see note 2 at 45.

86 See note 73.

87 Section 4 (b) of PD No. 1084

88R.A. No. 730 allows the private sale of home lots to actual occupants of public
lands. See note 63.

89 Issued on February 26, 1981.

90While PEA claims there was a failure of public bidding on December 10, 1991, there is no
showing that the Commission on Audit approved the price or consideration stipulated in the
negotiated Amended JVA as required by Section 79 of the Government Auditing Code.
Senate Committee Report No. 560 did not discuss this issue.

91Paragraph 2 (a) of COA Circular No. 89-296, on "Sale Thru Negotiation," states that
disposal through negotiated sale may be resorted to if "[T]here was a failure of public
auction."

92Senate Committee Report No. 560, Statement of Facts, p. 7, citing PEA Board Resolution
No. 835, as appearing in the Minutes of the PEA Board of Directors Meeting held on May 30,
1991, per Certification of Jaime T. De Veyra, Corporate Secretary, dated June 11, 1991.

93 Opinion No. 330, citing COA Audit Circular No. 89-296. See note 5.

94 PEA's Memorandum, see note 2.

95 Senate Committee Report No. 560, pp. 7-8, citing the Minutes of Meeting of the PEA Board
of Directors held on December 19, 1991.

96Section 3, Article XII of the 1987 Constitution provides as follows: "x x x Citizens of the
Philippines may x x x acquire not more than twelve hectares thereof by purchase, homestead
or grant." However, Section 6 of R.A. No. 6657 (Comprehensive Agrarian Reform Law) limits
the ownership of "public or private agricultural land" to a maximum of five hectares per
person.

97 96 Phil. 946 (1955).

98 48 SCRA 372 (1977).


425

99 168 SCRA 198 (1988).

100 172 SCRA 795 (1989).

101 73 SCRA 146 (1976).

102 Avila v. Tapucar, 201 SCRA 148 (1991).

103Republic v. Ayala Cia, et al., 14 SCRA 259 (1965); Dizon v. Rodriguez, 13 SCRA 705
(1965).

104Section 44 of PD No. 1529 states as follows: "Every registered owner receiving a


certificate of title in pursuance of a decree of registration, and every subsequent purchaser of
registered land taking a certificate of title for value and in good faith, shall hold the same free
from all encumbrances except those noted on said certificate and any of the following
encumbrances which may be subsisting, namely: First. Liens, claims or rights arising or
existing under the laws and Constitution of the Philippines which are not by law
required to appear of record in the Registry of Deeds in order to be valid against
subsequent purchasers or encumbrancers of record. x x x." Under Section 103 of PD No.
1529, Section 44 applies to certificates of title issued pursuant to a land patent granted by the
government.

105 Section 2, Article XIII of the 1935 Constitution.

106 Harty v. Municipality of Victoria, 13 Phil. 152 (1909).

107
Annex "B", AMARI's Memorandum, see note 21 at 16, Section 5.2 (c) of the Amended
JVA.

108Section 10 of CA No. 141 provides as follows: "Sec. 10. The words "alienation,"
"disposition," or "concession" as used in this Act, shall mean any of the methods authorized
by this Act for the acquisition, lease, use, or benefit of the lands of the public domain other
than timber or mineral lands."

109 Section 79 of the Government Auditing Code, which requires public auction in the sale of
government assets, includes all kinds of disposal or divestment of government assets. Thus,
COA Audit Circular No. 86-264 dated October 16, 1986 speaks of "guidelines (which) shall
govern the general procedures on the divestment or disposal of assets of government-
owned and/or controlled corporations and their subsidiaries." Likewise, COA Audit Circular
No. 89-296 dated January 27, speaks of "guidelines (which) shall be observed and adhered
to in the divestment or disposal of property and other assets of all government
entities/instrumentalities" and that "divestment shall refer to the manner or scheme of
taking away, depriving, withdrawing of an authority, power or title." These COA Circulars
implement Section 79 of the Government Auditing Code.

110 The share of AMARI in the Freedom Islands is 77.34 hectares, which is 70 percent of the
net usable area of 110.49 hectares. The net usable area is the total land area of the Freedom
Islands less 30 percent allocated for common areas.

111The share of AMARI in the submerged areas for reclamation is 290.129 hectares, which is
70 percent of the net usable area of 414.47 hectares.

112 Article 1409 of the Civil Code provides as follows: "The following contracts are inexistent
and void from the beginning: (1) Those whose cause, object or purpose is contrary to law; x x
x; (4) Those whose object is outside the commerce of men; x x x."
426

(OTHER SPECIFIC RIGHTS FOUND IN THE CIVIL CODE)

HILARIO v SALVADOR (G.R. No. 160384, April 29, 2005)

Republic of the Philippines


SUPREME COURT

SECOND DIVISION

G.R. No. 160384. April 29, 2005

CESAR T. HILARIO, for himself and as Attorney-in-Fact of IBARRA, NESTOR, LINA and
PRESCILLA, all surnamed HILARIO, Petitioners,
vs.
ALLAN T. SALVADOR, Respondents.

HEIRS OF SALUSTIANO SALVADOR, namely, REGIDOR M. SALVADOR and VIRGINIA


SALVADOR-LIM,respondents-intervenors.

DECISION

CALLEJO, SR., J.:

This is a petition for review on certiorari under Rule 45 of the Revised Rules of Court of the
Decision1 of the Court of Appeals (CA) in CA-G.R. CV No. 63737 as well as its Resolution2 denying
the motion for the reconsideration of the said decision.

The Antecedents

On September 3, 1996, petitioners Cesar, Ibarra, Nestor, Lina and Prescilla, all surnamed Hilario,
filed a complaint with the Regional Trial Court (RTC) of Romblon, Romblon, Branch 71, against
private respondent Allan T. Salvador. They alleged therein, inter alia, as follows:

2. That, the plaintiffs are co-owners by inheritance from Concepcion Mazo Salvador of a parcel of
land designated as Cad. Lot No. 3113-part, located at Sawang, Romblon, Romblon, which property
was [adjudged] as the hereditary share of their father, Brigido M. Hilario, Jr. when their father was still
single, and which adjudication was known by the plaintiffs[’] father’s co-heirs;

3. That, sometime in 1989, defendant constructed his dwelling unit of mixed materials on the property
of the plaintiffs’ father without the knowledge of the herein plaintiffs or their predecessors-in-interest;

4. That, demands have been made of the defendant to vacate the premises but the latter manifested
that he have (sic) asked the prior consent of their grandmother, Concepcion Mazo Salvador;

5. That, to reach a possible amicable settlement, the plaintiffs brought the matter to the Lupon of
Barangay Sawang, to no avail, evidenced by the CERTIFICATE TO FILE ACTION hereto attached as
ANNEX B;

6. That, the unjustified refusal of the defendant to vacate the property has caused the plaintiffs to
suffer shame, humiliation, wounded feelings, anxiety and sleepless nights;

7. That, to protect their rights and interest, plaintiffs were constrained to engage the services of a
lawyer.3
427

The petitioners prayed that, after due proceedings, judgment be rendered in their favor, thus:

WHEREFORE, it is prayed of this Honorable Court that after due process (sic), an order be issued for
the defendant to vacate and peacefully turn over to the plaintiffs the occupied property and that
defendant be made to pay plaintiffs:

a. actual damages, as follows:

a.1. transportation expenses in connection with the projected settlement of the case amounting to
₱1,500.00 and for the subsequent attendance to the hearing of this case at ₱1,500.00 each schedule;

a.2. attorney’s fees in the amount of ₱20,000.00 and ₱500.00 for every court appearance;

b. moral and exemplary damages in such amount incumbent upon the Honorable Court to determine;
and

c. such other relief and remedies just and equitable under the premises. 4

The private respondent filed a motion to dismiss the complaint on the ground of lack of jurisdiction
over the nature of the action, citing Section 33 of Batas Pambansa (B.P.) Blg. 129, as amended by
Section 3(3) of Republic Act (R.A.) No. 7691.5 He averred that –

(1) the complaint failed to state the assessed value of the land in dispute;

(2) the complaint does not sufficiently identify and/or describe the parcel of land referred to as the
subject-matter of this action;

both of which are essential requisites for determining the jurisdiction of the Court where the case is
filed. In this case, however, the assessed value of the land in question is totally absent in the
allegations of the complaint and there is nothing in the relief prayed for which can be picked-up for
determining the Court’s jurisdiction as provided by law.

In the face of this predicament, it can nevertheless be surmised by reading between the lines, that the
assessed value of the land in question cannot exceed ₱20,000.00 and, as such, it falls within the
jurisdiction of the Municipal Trial Court of Romblon and should have been filed before said Court
rather than before the RTC. …6

The petitioners opposed the motion.7 They contended that the RTC had jurisdiction over the action
since the court can take judicial notice of the market value of the property in question, which was
₱200.00 per square meter and considering that the property was 14,797 square meters, more or less,
the total value thereof is ₱3,500,000.00. Besides, according to the petitioners, the motion to dismiss
was premature and "the proper time to interpose it is when the [petitioners] introduced evidence that
the land is of such value."

On November 7, 1996, the RTC issued an Order8 denying the motion to dismiss, holding that the
action was incapable of pecuniary estimation, and therefore, cognizable by the RTC as provided in
Section 19(1) of B.P. Blg. 129, as amended.

After the denial of the motion to dismiss, the private respondent filed his answer with
counterclaim.9 Traversing the material allegations of the complaint, he contended that the petitioners
had no cause of action against him since the property in dispute was the conjugal property of his
grandparents, the spouses Salustiano Salvador and Concepcion Mazo-Salvador.

On April 8, 1997, Regidor and Virginia Salvador filed their Answer-in-Intervention10 making common
cause with the private respondent. On her own motion, however, Virginia Salvador was dropped as
intervenor.11
428

During trial, the petitioners adduced in evidence Tax Declaration No. 8590-A showing that in 1991 the
property had an assessed value of ₱5,950.00.12

On June 3, 1999, the trial court rendered judgment finding in favor of the petitioners. The dispositive
portion of the decision reads:

WHEREFORE, as prayed for, judgment is rendered:

Ordering the defendant to vacate and peacefully turn over to the plaintiffs the occupied property; and

Dismissing defendant’s counterclaim.

SO ORDERED.13

Aggrieved, the private respondent and respondent-intervenor Regidor Salvador appealed the decision
to the CA, which rendered judgment on May 23, 2003 reversing the ruling of the RTC and dismissing
the complaint for want of jurisdiction. The fallo of the decision is as follows:

IN VIEW OF THE FOREGOING, the appealed decision is REVERSED, and the case DISMISSED,
without prejudice to its refilling in the proper court.

SO ORDERED.14

The CA declared that the action of the petitioners was one for the recovery of ownership and
possession of real property. Absent any allegation in the complaint of the assessed value of the
property, the Municipal Trial Court (MTC) had exclusive jurisdiction over the action, conformably to
Section 3315 of R.A. No. 7691.

The petitioners filed a motion for reconsideration of the said decision, which the appellate court
denied.16 Hence, they filed the instant petition, with the following assignment of errors:

THE HONORABLE COURT OF APPEALS COMMITTED GRAVE REVERSIBLE ERROR IN


HOLDING THAT THE INSTANT CASE, ACCION REINVINDICATORIA, FALLS WITHIN THE
EXCLUSIVE ORIGINAL JURISDICTION OF THE MUNICIPAL TRIAL COURT OF ROMBLON, AND
NOT WITH THE REGIONAL TRIAL COURT OF ROMBLON.

II

THE HONORABLE COURT OF APPEALS COMMITTED SERIOUS REVERSIBLE ERROR IN


ORDERING THE REFILING OF THE CASE IN THE [PROPER] COURT, INSTEAD OF DECIDING
THE CASE ON THE MERITS BASED ON THE COMPLETE RECORDS ELEVATED BEFORE SAID
APPELLATE COURT AND IN NOT AFFIRMING IN TOTO THE DECISION OF THE TRIAL COURT.17

The Ruling of the Court

The lone issue for our resolution is whether the RTC had jurisdiction over the action of the petitioners,
the plaintiffs in the RTC, against the private respondent, who was the defendant therein.

The petitioners maintain that the RTC has jurisdiction since their action is an accion reinvindicatoria,
an action incapable of pecuniary estimation; thus, regardless of the assessed value of the subject
property, exclusive jurisdiction falls within the said court. Besides, according to the petitioners, in their
opposition to respondent’s motion to dismiss, they made mention of the increase in the assessed
value of the land in question in the amount of ₱3.5 million. Moreover, the petitioners maintain that
their action is also one for damages exceeding ₱20,000.00, over which the RTC has exclusive
jurisdiction under R.A. No. 7691.
429

The petition has no merit.

It bears stressing that the nature of the action and which court has original and exclusive jurisdiction
over the same is determined by the material allegations of the complaint, the type of relief prayed for
by the plaintiff and the law in effect when the action is filed, irrespective of whether the plaintiffs are
entitled to some or all of the claims asserted therein.18 The caption of the complaint is not
determinative of the nature of the action. Nor does the jurisdiction of the court depend upon the
answer of the defendant or agreement of the parties or to the waiver or acquiescence of the parties.

We do not agree with the contention of the petitioners and the ruling of the CA that the action of the
petitioners in the RTC was an accion reinvindicatoria. We find and so rule that the action of the
petitioners was an accion publiciana, or one for the recovery of possession of the real property
subject matter thereof. An accion reinvindicatoria is a suit which has for its object the recovery of
possession over the real property as owner. It involves recovery of ownership and possession based
on the said ownership. On the other hand, an accion publiciana is one for the recovery of possession
of the right to possess. It is also referred to as an ejectment suit filed after the expiration of one year
after the occurrence of the cause of action or from the unlawful withholding of possession of the
realty.19

The action of the petitioners filed on September 3, 1996 does not involve a claim of ownership over
the property. They allege that they are co-owners thereof, and as such, entitled to its possession, and
that the private respondent, who was the defendant, constructed his house thereon in 1989 without
their knowledge and refused to vacate the property despite demands for him to do so. They prayed
that the private respondent vacate the property and restore possession thereof to them.

When the petitioners filed their complaint on September 3, 1996, R.A. No. 7691 was already in effect.
Section 33(3) of the law provides:

Sec. 33. Jurisdiction of Metropolitan Trial Courts, Municipal Trial Courts and Municipal Circuit Trial
Courts in Civil Cases. – Metropolitan Trial Courts, Municipal Trial Courts and Municipal Circuit Trial
Courts shall exercise:

(3) Exclusive original jurisdiction in all civil actions which involve title to, or possession of, real
property, or any interest therein where the assessed value of the property or interest therein does not
exceed Twenty Thousand Pesos (₱20,000.00) or, in civil actions in Metro Manila, where such
assessed value does not exceed Fifty Thousand Pesos (₱50,000.00) exclusive of interest, damages
of whatever kind, attorney’s fees, litigation expenses and costs: Provided, That in cases of land not
declared for taxation purposes, the value of such property shall be determined by the assessed value
of the adjacent lots.

Section 19(2) of the law, likewise, provides that:

Sec. 19. Jurisdiction in civil cases. – The Regional Trial Court shall exercise exclusive original
jurisdiction:

(2) In all civil actions, which involve the title to, or possession of, real property, or any interest therein,
where the assessed value of the property involved exceeds Twenty Thousand Pesos (₱20,000.00) or,
for civil actions in Metro Manila, where such value exceeds Fifty Thousand Pesos (₱50,000.00)
except actions for forcible entry into and unlawful detainer of lands or buildings, original jurisdiction
over which is conferred upon the Metropolitan Trial Courts, Municipal Trial Courts, and Municipal
Circuit Trial Courts.

The jurisdiction of the court over an action involving title to or possession of land is now
determined by the assessed value of the said property and not the market value thereof. The
430

assessed value of real property is the fair market value of the real property multiplied by the
assessment level. It is synonymous to taxable value.20 The fair market value is the price at which a
property may be sold by a seller, who is not compelled to sell, and bought by a buyer, who is not
compelled to buy.

Even a cursory reading of the complaint will show that it does not contain an allegation stating the
assessed value of the property subject of the complaint.21 The court cannot take judicial notice of the
assessed or market value of lands.22 Absent any allegation in the complaint of the assessed value of
the property, it cannot thus be determined whether the RTC or the MTC had original and exclusive
jurisdiction over the petitioners’ action.

We note that during the trial, the petitioners adduced in evidence Tax Declaration No. 8590-A,
showing that the assessed value of the property in 1991 was ₱5,950.00. The petitioners, however, did
not bother to adduce in evidence the tax declaration containing the assessed value of the property
when they filed their complaint in 1996. Even assuming that the assessed value of the property in
1991 was the same in 1995 or 1996, the MTC, and not the RTC had jurisdiction over the action of the
petitioners since the case involved title to or possession of real property with an assessed value of
less than ₱20,000.00.23

We quote with approval, in this connection, the CA’s disquisition:

The determining jurisdictional element for the accion reinvindicatoria is, as RA 7691 discloses, the
assessed value of the property in question. For properties in the provinces, the RTC has jurisdiction if
the assessed value exceeds ₱20,000, and the MTC, if the value is ₱20,000 or below. An assessed
value can have reference only to the tax rolls in the municipality where the property is located, and is
contained in the tax declaration. In the case at bench, the most recent tax declaration secured and
presented by the plaintiffs-appellees is Exhibit B. The loose remark made by them that the property
was worth 3.5 million pesos, not to mention that there is absolutely no evidence for this, is irrelevant in
the light of the fact that there is an assessed value. It is the amount in the tax declaration that should
be consulted and no other kind of value, and as appearing in Exhibit B, this is ₱5,950. The case,
therefore, falls within the exclusive original jurisdiction of the Municipal Trial Court of Romblon which
has jurisdiction over the territory where the property is located, and not the court a quo.24

It is elementary that the tax declaration indicating the assessed value of the property enjoys the
presumption of regularity as it has been issued by the proper government agency. 25

Unavailing also is the petitioners’ argumentation that since the complaint, likewise, seeks the recovery
of damages exceeding ₱20,000.00, then the RTC had original jurisdiction over their actions. Section
33(3) of B.P. Blg. 129, as amended, quoted earlier, explicitly excludes from the determination of the
jurisdictional amount the demand for "interest, damages of whatever kind, attorney’s fees, litigation
expenses, and costs." This Court issued Administrative Circular No. 09-94 setting the guidelines in
the implementation of R.A. No. 7691, and paragraph 2 thereof states that –

2. The exclusion of the term "damages of whatever kind" in determining the jurisdictional amount
under Section 19(8) and Section 33(1) of B.P. Blg. 129, as amended by R.A. 7691, applies to cases
where the damages are merely incidental to or a consequence of the main cause of action. However,
in cases where the claim for damages is the main cause of action, or one of the causes of action, the
amount of such claim shall be considered in determining the jurisdiction of the court.

Neither may the petitioners find comfort and solace in Section 19(8) of B.P. Blg. 129, as amended,
which states:

SEC. 19. Jurisdiction in civil cases. – Regional Trial Courts shall exercise exclusive original
jurisdiction:


431

(8) In all other cases in which the demand, exclusive of interest, damages of whatever kind, attorney's
fees, litigation expenses, and costs or the value of the property in controversy exceeds One Hundred
Thousand Pesos (₱100,000.00) or, in such other cases in Metro Manila, where the demand, exclusive
of the above-mentioned items exceeds Two Hundred Thousand Pesos (₱200,000.00).

The said provision is applicable only to "all other cases" other than an action involving title to, or
possession of real property in which the assessed value is the controlling factor in determining the
court’s jurisdiction. The said damages are merely incidental to, or a consequence of, the main cause
of action for recovery of possession of real property.26

Since the RTC had no jurisdiction over the action of the petitioners, all the proceedings therein,
including the decision of the RTC, are null and void. The complaint should perforce be dismissed. 27

WHEREFORE, the petition is DENIED. The assailed Decision and Resolution of the Court of Appeals
in CA-G.R. CV No. 63737 are AFFIRMED. Costs against the petitioners.

SO ORDERED.

Puno, (Chairman), Austria-Martinez, Tinga, and Chico-Nazario, JJ., concur.

Footnotes

1 Pennedby Associate Justice Mario L. Guariña III, with Associate Justices Eubulo G. Verzola
(deceased) and Martin S. Villarama, Jr., concurring.

2 Rollo, p. 57.

3 Rollo, p. 58.

4 Rollo, pp. 58-59.

5 SECTION 33. Jurisdiction of Metropolitan Trial Courts, Municipal Trial Courts and Municipal
Circuit Trial Courts in Civil Cases. – Metropolitan Trial Courts, Municipal Trial Courts and
Municipal Circuit Trial Courts shall exercise:

(3) Exclusive original jurisdiction in all civil actions which involve title to, or possession of, real
property, or any interest therein where the assessed value of the property or interest therein
does not exceed Twenty Thousand Pesos (₱20,000.00) or, in civil actions in Metro Manila,
where such assessed value does not exceed Fifty Thousand Pesos (₱50,000.00) exclusive of
interest, damages of whatever kind, attorney's fees, litigation expenses and costs: Provided,
That in cases of land not declared for taxation purposes, the value of such property shall be
determined by the assessed value of the adjacent lots.

6 Rollo, pp. 61-62.

7 Rollo, p. 65.

8 Id. at 73.

9 Id. at 75.
432

10 Id. at 79.

11 Id. at 88.

12 Rollo, p. 49.

13 Id. at 94.

14 Id. at 54.

15 (3)Exclusive original jurisdiction in all civil actions which involve title to, or possession of,
real property, or any interest therein where the assessed value of the property or interest
therein does not exceed Twenty Thousand Pesos (₱20,000.00) … exclusive of interest,
damages of whatever kind, attorney’s fees, litigation expenses and costs; Provided, that in
cases of land not declared for taxation purposes, the value of such property shall be
determined by the assessed value of the adjacent lots.

16 Rollo, p. 57.

17 Id. at 21.

18 RadioCommunications of the Philippines, Inc. v. Court of Appeals, G.R. No. 136109, 1


August 2002, 386 SCRA 67; Korea Exchange Bank v. Filkor Business Integrated, Inc., G.R.
No. 138292, 10 April 2002, 380 SCRA 381.

19 Cruz v. Torres, G.R. No. 121939, 4 October 1999, 316 SCRA 193.

20 Section 199 of Republic Act No. 7160.

21 Ouano
v. PGTT International Investment Corporation, G.R. No. 134230, 17 July 2002, 384
SCRA 589.

22 Ibid.

23 See Aliabo v. Carampatan, G.R. No. 128922, 16 March 2001, 354 SCRA 548.

24 Rollo, p. 54.

25 Ouano v. PGTT International Investment Corporation, supra.

26 Ouano v. PGTT International Investment Corporation, supra.

27 Ibid.
433

SAMPAYANG v CA (G.R. No. 156360, January 14, 2005)

THIRD DIVISION

G.R. No. 156360 January 14, 2005

CESAR SAMPAYAN, petitioner,


vs.
The HONORABLE COURT OF APPEALS, CRISPULO VASQUEZ and FLORENCIA VASQUEZ
GILSANO,respondents.

DECISION

GARCIA, J.:

In this verified petition for review on certiorari under Rule 45 of the Rules of Court, petitioner Cesar
Sampayan seeks the annulment and setting aside of the following issuances of the Court of Appeals
in CA-G.R. SP No. 43557, to wit:

1. Decision dated May 16, 2002, denying his petition for review and affirming an earlier decision of
the Regional Trial Court at Agusan del Sur, Branch VII, which in turn reversed on appeal a favorable
judgment of the Municipal Circuit Trial Court (MCTC) of Bayugan and Sibagat, Agusan del Sur in a
forcible entry case thereat commenced against him by herein private respondents, the brother-and-
sister Crispulo Vasquez and Florencia Vasquez-Gilsano; and

2. Resolution dated November 7, 2002, which denied his motion for reconsideration.

From the pleadings and memoranda respectively filed by the parties, the Court gathers the following
factual antecedents:

On July 8, 1992, in the MCTC of Bayugan and Sibagat, Agusan del Sur, the siblings Crispulo
Vasquez and Florencia Vasquez-Gilsano filed complaint for forcible entry against Cesar Sampayan
for allegedly having entered and occupied a parcel of land, identified as Lot No. 1959, PLS-225, and
built a house thereon without their knowledge, consent or authority, the entry having been supposedly
effected through strategy and stealth.

In their complaint, the plaintiffs (now private respondents), substantially alleged that their mother
Cristita Quita was the owner and actual possessor of Lot No. 1959; that after their mother’s death on
January 11, 1984, they became co-owners pro-indiviso and lawful possessors of the same lot; that on
June 1, 1992, while they were temporarily absent from the lot in question, defendant Cesar
Sampayan, through strategy and stealth, entered the lot and built a house thereon, to their exclusion;
and that, despite their repeated demands for Sampayan to vacate the lot and surrender the
possession thereof to them, the latter failed and refused to do so.

In his answer, defendant Sampayan denied the material allegations of the complaint and averred that
neither the plaintiffs nor their mother have ever been in possession of Lot No. 1959 and that he does
not even know plaintiffs’ identities or their places of residence. He claimed that he did not enter the
subject lot by stealth or strategy because he asked and was given permission therefor by Maria
Ybañez, the overseer of the lot’s true owners, Mr. and Mrs. Anastacio Terrado who were then
temporarily residing in Cebu City for business purposes. In the same answer, Sampayan alleged that
the plaintiffs’ claim has long prescribed for the reason that the lot in dispute had been possessed and
declared for taxation purposes by the spouses Felicisimo Oriol and Concordia Balida-Oriol in 1960,
and that in 1978, the Oriol spouses sold one-half (1/2) of the lot to the spouses Mr. and Mrs.
Anastacio Terrado, while the other half, to the couple Manolito Occida and Juliana Sambale-Occida in
1979. Both vendees, so Sampayan averred, have actually possessed the respective portions
purchased by them up to the present. He thus prayed for the dismissal of the complaint.
434

In the ensuing proceedings following the joinder of issues, the plaintiffs, to prove that they have been
in actual possession of Lot No. 1959 when defendant Sampayan effected his entry thereto, submitted
in evidence the following documents:

1. Tax Declaration No. 3180 in the name of Cristita Quita;

2. Certificate of Death showing the date of death of Cristita Quita on January 11, 1984;

3. Certificate issued by Fermina R. Labonete, Land Management Officer-III of CENRO X-3-E,


DENR-X-3-9, Bayugan, Agusan del Sur showing that Lot 1959, PLS-225 is covered by a
Miscellaneous Sales Application of Cristita Quita;

4. Affidavit of one Emiliano G. Gatillo to the effect that he was the one who gave the lot in
question to Cristita Quita sometime in 1957 and that since then the latter had been occupying
the lot;

Plaintiffs also filed a Supplemental Position Paper dated July 13, 1994 for the purpose of
showing that Cristita Quita is one of the oppositors in Cadastral Case No. 149. Together with
said position paper, they submitted a copy of the Answer/Opposition earlier filed in Cadastral
Case No. 149. In said cadastral case, Cristita Quita was claiming Lot 1959, thus her name
appeared in the list of oppositors therein.

5. The decision in the said Cadastral Case No. 149 showing that the then Court of First
Instance of Agusan del Sur declared Lot No. 1959 as one of the lots subject of the same
cadastral case.

For his part, defendant Sampayan, to prove the allegations in his answer, offered in evidence the
following:

1. Tax Declaration No. A-11698 in the name of Felicisimo Oriol, which cancels Tax
Declaration 8103;

2. Tax Declaration No. GRB-01-930 in the name of Felicisimo Oriol which cancels Tax
Declaration No. A-11698;

3. Deed of Absolute Sale of Portion of Land, dated April 30, 1979, executed by Jesus Oriol for
and in behalf of the spouses Felicisimo Oriol and Concordia Balida-Oriol, conveying the one-
half (1/2) portion of Lot No. 1959 to the couple Manolito Occida and Juliana Sambale-Occida
who possessed the one-half (1/2) portion and introduced improvements thereon, such as
coconut and caimito trees;

4. Deed of Relinquishment of Rights of Portion of Land, executed by the spouses Oriol in


favor of the same couple Manolito Occida and Juliana Sambale-Occida, to further strengthen
the transfer of possession and whatever possessory rights the Oriols had in the lot in
question;

5. Deed of Absolute Sale of Land executed by Concordia Balida-Oriol with the conformity of
Teodosio Mosquito (another claimant), to prove that the other half of Lot No. 1959 was sold in
1978 to Mr. and Mrs. Anastacio Terrado whose overseer allowed Sampayan to enter and
occupy the premises;

6. Protest filed with the CENRO, Agusan del Sur by the vendee Juliana Sambale-Occida
against the Miscellaneous Sales Application of Cristita Quita;

7. Affidavit of Dionesia Noynay attesting to the fact that she is residing in Lot No. 1957, a lot
adjacent to the lot in question, since 1960 up to the present. In the same affidavit, Dionisia
claimed that neither Cristita Quita, much less the plaintiffs, had ever possessed Lot No. 1959.
435

She claimed that it was the Occida couple who possessed said lot and introduced
improvements thereon; and

8. Affidavit of Juliana Occida and Maria Ybañez to show the impossibility of plaintiffs’
possession of the same lot.

Meanwhile, on March 21, 1996, while the case was pending with the MCTC, the presiding judge
thereof personally conducted an ocular inspection of the contested lot in the presence of the parties
and/or their counsels. Among those found in the area during the inspection are: the house of
defendant Sampayan; the dilapidated house of a certain Peter Siscon; and a portion of the house of
Macario Noynay, husband of Dionisia Noynay, one of Sampayan’s witnesses.

Based on his ocular findings, the judge concluded that the improvements he saw in the premises
could never have been introduced by the plaintiffs nor by their mother Cristita Quita but by the
vendees of the same lot. Reproduced by petitioner Jose Sampayan in the instant petition as well as in
the Memorandum he subsequently filed with this Court, the MCTC judge’s findings and observations
during the ocular inspection, about which the herein private respondents took no exception
whatsoever, are hereunder quoted, as follows:

"Noted inside the land are the house of the defendant, Cesar Sampayan, of Peter Siscon, which
appears to be dilapidated, and part of the house of Macario Noynay which encroached to the land in
question. Planted on the land are five (5) coconut trees, fruit bearing, three (3) not fruit bearing
coconut trees, and three (3) star apple or caimito trees. Defendant Sampayan admitted that he started
occupying the land since 1992. It is admitted by the parties during the ocular inspection that one-half
(1/2) portion of the land was bought by a certain Occida from certain Mr. and Mrs. Felicisimo Oriol.

The findings in the ocular inspection have confirmed the allegation of the defendant that his
predecessors-in-interest have introduced improvements by planting caimito trees, coconut
trees, and others on the land in question.

Nothing can be seen on the land that plaintiffs had once upon a time been in possession of the
land. The allegation that Cristita Quita, the predecessor-in-interest of the plaintiffs had been in
possession of the said property since 1957, openly, exclusively, continuously, adversely and
in the concept of an owner is a naked claim, unsupported by any evidence.

Clearly, from the appearance of the improvements introduced by the predecessors-in-interest of the
defendant, it is showed that they have been in possession of the land for more than one (1) year.
Hence, the action of the plaintiffs, if any, is accion publiciana or plenaria de possession"1 (Emphasis
supplied).

In time, the MCTC rendered judgment dismissing the compliant "for lack of merit".

Therefrom, the plaintiffs appealed to the Regional Trial Court (RTC) at Agusan del Sur, which appeal
was raffled to Branch VII thereof. In a decision dated December 5, 1996, said court reversed that of
the MCTC, taking note of the fact that Cristita Quita was among the oppositors in Cadastral Case No.
149 and that she filed a Miscellaneous Sales Application over the lot. On the basis of such finding, the
RTC concluded that it was Cristita Quita, predecessor-in-interest of the herein private respondents,
who was in actual prior physical possession of Lot No. 1959.

Unable to accept the RTC judgment, Sampayan went to the Court of Appeals on a petition for review,
thereat docketed as CA-G.R. SP No. 43557.

As stated at the threshold hereof, the Court of Appeals, in the herein assailed Decision dated May
16, 2002,2denied Sampayan’s petition. His motion for reconsideration having been similarly denied by
that court in its Resolution of November 7, 2002,3 Sampayan is now with us via the present
recourse, it being his submissions -

"I.
436

THAT THE COURT OF APPEALS ERRED IN RULING THAT THE MUNICIPAL CIRCUIT TRIAL
COURT OF BAYUGAN, AGUSAN DEL SUR, HAS JURISDICTION OVER THE CASE,
CONSIDERING THAT DURING THE HEARING THEREOF IT WAS FOUND OUT BY THE SAID
MUNICIPAL COURT THAT ACCION PUBLICIANA OR PLENARIA DE POSESION, AND NOT
FORCIBLE ENTRY, IS THE PROPER ACTION;

II.

THAT THE CONCLUSION OF THE HONORABLE COURT OF APPEALS THAT PRIVATE


RESPONDENTS HAVE BEEN IN PRIOR ACTUAL POSSESSION IS CONTRADICTED BY
EVIDENCE ON RECORD, AND CONSIDERING THAT THE POSSESSION TO BE LEGALLY
SUFFICIENT, CONSIST (SIC) IN THE EXERCISE OF DOMINIUM OVER IT, SUCH AS FENCING,
CULTIVATING OR OTHER UNMISTAKABLE ACTS OF EXCLUSIVE CUSTODY AND CONTROL –
FACTS WHICH THE PRIVATE RESPONDENTS HAVE NEVER DONE - IS CONTRARY TO LAW".4

In the main, petitioner maintains that based on the pieces of evidence on record, he had sufficiently
proven his prior physical possession of the subject lot. Upon this premise, he argues that private
respondents’ complaint for forcible entry has no leg to stand on, adding that the proper remedy
available to the latter is accion publiciana or plenaria de posesion which falls under the original
jurisdiction of Regional Trial Courts and not of Municipal Circuit Trial Courts.

As we see it, the arguments put forward by the petitioner crystallize to one pivotal question: will the
complaint for forcible entry in this case prosper? To resolve this, however, we must first determine as
to who between the herein parties was in prior actual physical possession of the subject lot at the time
the complaint was filed in the MCTC. For, as we have said in Gaza vs. Lim 5 ,

"xxx In an action for forcible entry, the plaintiff must prove that he was in prior possession of the land
or building and that he was deprived thereof by means of force, intimidation, threat, strategy or
stealth. xxx"

We emphasize, absence of prior physical possession by the plaintiff in a forcible entry case warrants
the dismissal of his complaint.1a\^/phi1.net

Undoubtedly, the issue of prior physical possession is one of fact, and settled is the rule that this
Court is not a trier of facts and does not normally embark on a re-examination of the evidence
adduced by the parties during trial. Of course, the rule admits of exceptions. So it is that in Insular Life
Assurance Company, Ltd. vs. CA ,6 we wrote:

"[i]t is a settled rule that in the exercise of the Supreme Court's power of review, the Court is not a trier
of facts and does not normally undertake the re-examination of the evidence presented by the
contending parties' during the trial of the case considering that the findings of facts of the CA are
conclusive and binding on the Court. However, the Court had recognized several exceptions to this
rule, to wit: (1) when the findings are grounded entirely on speculation, surmises or conjectures; (2)
when the inference made is manifestly mistaken, absurd or impossible; (3) when there is grave abuse
of discretion; (4) when the judgment is based on a misapprehension of facts; (5) when the findings of
facts are conflicting; (6) when in making its findings the Court of Appeals went beyond the issues of
the case, or its findings are contrary to the admissions of both the appellant and the appellee; (7)
when the findings are contrary to the trial court; (8) when the findings are conclusions without citation
of specific evidence on which they are based; (9) when the facts set forth in the petition as well as in
the petitioner's main and reply briefs are not disputed by the respondent; (10) when the findings of
fact are premised on the supposed absence of evidence and contradicted by the evidence on record;
and (11) when the Court of Appeals manifestly overlooked certain relevant facts not disputed by the
parties, which, if properly considered, would justify a different conclusion."l^vvphi1.net

To our mind, exceptions (5) and (11) are present in this case.

However, before delving into the question of who as between the petitioner and private respondents
had prior physical possession of the subject lot, we deem it best to first resolve the issue of whether
437

or not the MCTC had jurisdiction over the complaint filed in this case, an issue also raised by the
petitioner.

Relying on the conclusion of the MCTC that private respondents’ proper remedy is accion publiciana
or plenaria de posesion, and not forcible entry, petitioner would deny the MCTC’s jurisdiction over the
case.

Petitioner is in error.

In Sarmiento vs. CA7 , we held:

"[t]o give the court jurisdiction to effect the ejectment of an occupant or deforciant on the land, it is
necessary that the complaint should embody such a statement of facts as brings the party clearly
within the class of cases for which the statutes provide a remedy, as these proceedings are summary
in nature. The complaint must show enough on its face to give the court jurisdiction without resort to
parol testimony. The jurisdictional facts must appear on the face of the complaint. x x x"

Clear it is from the above that for the MCTC to acquire jurisdiction over a forcible entry case, it is
enough that the complaint avers the jurisdictional facts, i.e. that the plaintiff had prior physical
possession and that he was deprived thereof by the defendant through force, intimidation, threats,
strategy and stealth.8 The complaint in this case makes such an averment. Hence, the irrelevant
circumstance that the evidence adduced during the hearing rendered improper an action for forcible
entry is of no moment and cannot deprive the MCTC of its jurisdiction over the case. The MCTC
continues to have that jurisdiction.

We shall now address the more decisive question of prior physical possession.

After a careful evaluation of the evidence at hand, we find for the petitioner.

To begin with, we are at once confronted by the uncontested findings of the MCTC judge himself
during his ocular inspection of the premises in dispute that what he saw thereat "confirmed the
allegations of the defendant [now petitioner Sampayan] that his predecessors-in-interest have
introduced improvements by planting caimito trees, coconut trees, and others on the land in
question", adding that "[N]othing can be seen on the land that plaintiff had once upon a time been in
possession of the land", and categorically stating that "[T]he allegation that Cristita Quita, the
predecessor-in-interest of the plaintiffs had been in possession of the said property since 1957,
openly, exclusively, continuously, adversely and in the concept of an owner is a naked claim,
unsupported by any evidence".1awphi1.nét

Then, too, there is the sworn affidavit of Dionesia Noynay to the effect that she had been residing
since 1960 onward on Lot No. 1957, the lot adjacent to Lot No. 1959, and that neither the private
respondents nor their mother had ever possessed Lot No. 1959. Coming as it does from an
immediate neighbor, Dionesia’s statement commands great weight and respect. Incidentally, the
MCTC judge himself found during the ocular inspection that a portion of the house of Macario
Noynay, husband of Dionesia, protruded on Lot No. 1959.

We note that in the herein assailed decision, the Court of Appeals attached much significance to the
fact that private respondents’ mother Cristita Quita was an oppositor in Cadastral Case No. 149. We
rule and so hold that the mother’s being an oppositor in said cadastral case does not, by itself,
establish prior physical possession because not all oppositors in cadastral cases are actual
possessors of the lots or lands subject thereof.

WHEREFORE, the instant petition is hereby GRANTED and the Decision and Resolution,
respectively dated May 16, 2002 and November 7, 2002, of the Court of Appeals REVERSED and
SET ASIDE.

SO ORDERED.
438

Panganiban, (Chairman), Sandoval-Gutierrez, and Corona, Carpio-Morales, JJ., concur.

Footnotes

1 Petition, p. 3; Rollo, p. 14; Petitioner’s Memorandum, pp. 7-8; Rollo, pp. 65-66.

2 Annex "A", Petition; Rollo, pp. 26-34.

3 Annex "C", Petition, Rollo, p. 41.

4 Petitioner’s Memorandum, Rollo, pp. 60, et seq., at pp. 74-75.

5 395 SCRA 261, 269 [2003], citing Sps. Benitez vs. CA, 334 Phil. 216, 222 [1997].

6G.R. No. 126850, April 28, 2004, citing Langkaan Realty Development, Inc. vs. United
Coconut Planters Bank ,347 SCRA 542, 549 [2000], Nokom vs. National Labor Relations
Commission, 390 Phil. 1228,1242 [2000], Commissioner of Internal Revenue vs. Embroidery
and Garments Industries (Phil.), Inc., 363 Phil. 541, 546 [1999], Sta. Maria vs. Court of
Appeals, 349 Phil. 275, 282-283 [1998].

7 320 Phil. 146, 156 [1995].

8 Spouses Tirona vs. Hon. Alejo, 419 Phil. 285 [2001].

SANTOS v AYON (G.R. No. 137013, May 6, 2005)

THIRD DIVISION

G.R. No. 137013 May 6, 2005

RUBEN SANTOS, petitioner,


vs.
SPOUSES TONY AYON and MERCY AYON, respondents.

DECISION

SANDOVAL-GUTIERREZ, J.:

For our resolution is the petition for review on certiorari assailing the Decision1 of the Court of Appeals
dated October 5, 1998 in CA-G.R. SP No. 4735 and its Resolution2 dated December 11, 1998
denying the motion for reconsideration.

The petition alleges that on November 6, 1996, Ruben Santos, petitioner, filed with the Municipal Trial
Court in Cities (MTCC), Branch 2, Davao City a complaint for illegal detainer against spouses Tony
and Mercy Ayon, respondents, docketed as Civil Case No. 3506-B-96.
439

In his complaint, petitioner averred that he is the registered owner of three lots situated at Lanzona
Subdivision, Matina, Davao City, covered by Transfer Certificates of Title (TCT) Nos. 108174,
108175, and 108176. Respondent spouses are the registered owners of an adjacent parcel of land
covered by TCT No. T-247792. The previous occupant of this property built a building which straddled
both the lots of the herein parties. Respondents have been using the building as a warehouse.

Petitioner further alleged in his complaint that in 1985, when he bought the three lots, he informed
respondents that the building occupies a portion of his land. However, he allowed them to continue
using the building. But in 1996, he needed the entire portion of his lot, hence, he demanded that
respondents demolish and remove the part of the building encroaching his property and turn over to
him their possession. But they refused. Instead, they continued occupying the contested portion and
even made improvements on the building. The dispute was then referred to the barangay lupon, but
the parties failed to reach an amicable settlement. Accordingly, on March 27, 1996, a certification to
file action was issued.

In their answer, respondents sought a dismissal of this case on the ground that the court has no
jurisdiction over it since there is no lessor-lessee relationship between the parties. Respondents
denied they were occupying petitioner's property by mere tolerance, claiming they own the contested
portion and have been occupying the same long before petitioner acquired his lots in 1985.

On July 31, 1997, the MTCC rendered its Decision in favor of petitioner, thus:

"WHEREFORE, judgment is rendered in favor of the plaintiff and against the defendants
ordering the latter, their successors-in-interest and other persons acting in their behalf to
vacate the portion of the subject properties and peacefully surrender possession thereof to
plaintiff as well as dismantle/remove the structures found thereon.

Defendants are further ordered to pay reasonable value for the use and occupation of the
encroached area in the amount of One Thousand Pesos (P1,000.00) a month beginning
September 1996 and the subsequent months thereafter until premises are vacated; to pay
attorney's fees of Ten Thousand Pesos (P10,000.00); and to pay the costs of suit.

SO ORDERED."3

On appeal, the Regional Trial Court (RTC), Branch 11, Davao City, in its Decision dated February 12,
1998 in Civil Case No. 25, 654-97, affirmed in toto the MTCC judgment.4 The RTC upheld the finding
of the MTCC that respondents' occupation of the contested portion was by mere tolerance. Hence,
when petitioner needed the same, he has the right to eject them through court action.

Respondents then elevated the case to the Court of Appeals through a petition for review. In its
Decision dated October 5, 1988 now being challenged by petitioner, the Court of Appeals held that
petitioner's proper remedy should have been an accion publiciana before the RTC, not an action for
unlawful detainer, thus:

"In this case, petitioners were already in possession of the premises in question at the time
private respondent bought three (3) lots at the Lanzona Subdivision in 1985, a portion of
which is occupied by a building being used by the former as a bodega. Apart from private
respondent's bare claim, no evidence was alluded to show that petitioners' possession was
tolerated by (his) predecessor-in-interest. The fact that respondent might have tolerated
petitioners' possession is not decisive. What matters for purposes of determining the proper
cause of action is the nature of petitioners' possession from its inception. And in this regard,
the Court notes that the complaint itself merely alleges that defendants-petitioners have been
'occupying a portion of the above properties of the plaintiff for the past several years by virtue
of the tolerance of the plaintiff.' Nowhere is it alleged that his predecessor likewise tolerated
petitioners' possession of the premises. x x x.
440

Consequently, x x x, respondent should present his claim before the Regional Trial Court in
an accion publiciana and not before the Municipal Trial Court in a summary proceeding of
unlawful detainer.

WHEREFORE, the decision under review is hereby REVERSED and SET ASIDE.
Accordingly, the complaint for unlawful detainer is ordered DISMISSED."5

Petitioner filed a motion for reconsideration, but was denied by the Appellate Court in its Resolution
dated December 11, 1998.

Hence, the instant petition for review on certiorari ascribing to the Court of Appeals the following
errors:

"I

THE HONORABLE COURT OF APPEALS MISAPPLIED THE LAW IN DISMISSING THE


INSTANT CASE ON THE GROUND THAT PETITIONER SHOULD PRESENT HIS CLAIM
BEFORE THE REGIONAL TRIAL COURT IN AN ACCION PUBLICIANA.

II

THE FINDINGS OF THE HONORABLE COURT OF APPEALS IS NOT IN CONSONANCE


WITH EXISTING LAWS AND JURISPRUDENCE."

The sole issue here is whether the Court of Appeals committed a reversible error of law in holding that
petitioner's complaint is within the competence of the RTC, not the MTCC.

Petitioner contends that it is not necessary that he has prior physical possession of the questioned
property before he could file an action for unlawful detainer. He stresses that he tolerated
respondents' occupancy of the portion in controversy until he needed it. After his demand that they
vacate, their continued possession became illegal. Hence, his action for unlawful detainer before the
MTCC is proper.

Respondents, in their comment, insisted that they have been in possession of the disputed property
even before petitioner purchased the same on April 10, 1985. Hence, he cannot claim that they were
occupying the property by mere tolerance because they were ahead in time in physical possession.

We sustain the petition.

It is an elementary rule that the jurisdiction of a court over the subject matter is determined by the
allegations of the complaint and cannot be made to depend upon the defenses set up in the answer
or pleadings filed by the defendant.6 This rule is no different in an action for forcible entry or unlawful
detainer.7 All actions for forcible entry or unlawful detainer shall be filed with the proper Metropolitan
Trial Courts, the Municipal Trial Courts and the Municipal Circuit Trial Courts, which actions shall
include not only the plea for restoration of possession but also all claims for damages and costs
arising therefrom.8 The said courts are not divested of jurisdiction over such cases even if the
defendants therein raises the question of ownership over the litigated property in his pleadings and
the question of possession cannot be resolved without deciding the issue of ownership. 9

Section 1, Rule 70 on forcible entry and unlawful detainer of the 1997 Rules of Civil Procedure, as
amended, reads:

"Section 1. Who may institute proceedings, and when. – Subject to the provisions of the next
succeeding section, a person deprived of the possession of any land or building by force,
intimidation, threat, strategy, or stealth, or a lessor, vendor, vendee, or other person against
whom the possession of any land or building is unlawfully withheld after the expiration or
termination of the right to hold possession, by virtue of any contract, express or implied, or the
441

legal representatives or assigns of any such lessor, vendor, vendee or other person may, at
any time within one (1) year after such unlawful deprivation or withholding of possession,
bring an action in the proper Municipal Trial Court against the person or persons unlawfully
withholding or depriving of possession, or any person or persons claiming under them, for the
restitution of such possession, together with damages and costs."

Under the above provision, there are two entirely distinct and different causes of action, to wit: (1) a
case for forcible entry, which is an action to recover possession of a property from the defendant
whose occupation thereof is illegal from the beginning as he acquired possession by force,
intimidation, threat, strategy or stealth; and (2) a case for unlawful detainer, which is an action for
recovery of possession from defendant whose possession of the property was inceptively lawful by
virtue of a contract (express or implied) with the plaintiff, but became illegal when he continued his
possession despite the termination of his right thereunder.10

Petitioner's complaint for unlawful detainer in Civil Case No. 3506-B-96 is properly within the
competence of the MTCC. His pertinent allegations in the complaint read:

"4. That defendants (spouses) have constructed an extension of their residential house as
well as other structures and have been occupying a portion of the above PROPERTIES of the
plaintiff for the past several years by virtue of the tolerance of the plaintiff since at the
time he has no need of the property;

5. That plaintiff needed the property in the early part of 1996 and made demands to the
defendants to vacate and turn over the premises as well as the removal (of) their
structures found inside the PROPERTIES of plaintiff; that without any justifiable
reasons, defendants refused to vacate the portion of the PROPERTIES occupied by
them to the damage and prejudice of the plaintiff.

6. Hence, plaintiff referred the matter to the Office of the Barangay Captain of Matina
Crossing 74-A, Davao City for a possible settlement sometime in the latter part of February
1996. The barangay case reached the Pangkat but no settlement was had. Thereafter, a
'Certification To File Action' dated March 27, 1996 was issued x x x;

x x x."11 (underscoring ours)

Verily, petitioner's allegations in his complaint clearly make a case for an unlawful detainer. We find
no error in the MTCC assuming jurisdiction over petitioner's complaint. A complaint for unlawful
detainer is sufficient if it alleges that the withholding of the possession or the refusal to vacate is
unlawful without necessarily employing the terminology of the law. 12 Here, there is an allegation in
petitioner's complaint that respondents occupancy on the portion of his property is by virtue of
his tolerance. Petitioner's cause of action for unlawful detainer springs from respondents' failure to
vacate the questioned premises upon his demand sometime in 1996. Within one (1) year therefrom,
or on November 6, 1996, petitioner filed the instant complaint.

It bears stressing that possession by tolerance is lawful, but such possession becomes unlawful when
the possessor by tolerance refuses to vacate upon demand made by the owner. Our ruling in Roxas
vs. Court of Appeals13 is applicable in this case: "A person who occupies the land of another at the
latter's tolerance or permission, without any contract between them, is necessarily bound by an
implied promise that he will vacate upon demand, failing which, a summary action for ejectment is the
proper remedy against him."

WHEREFORE, the petition is GRANTED. The assailed Decision and Resolution of the Court of
Appeals in CA-G.R. SP No. 47435 are hereby REVERSED and SET ASIDE. The Decision dated
February 12, 1998 of the Regional Trial Court, Branch 11, Davao City in Civil Case No. 25, 654-97,
affirming the Decision dated July 31, 1997 of the Municipal Trial Court in Cities, Branch 2, Davao City
in Civil Case No. 3506-B-96, is hereby REINSTATED.

SO ORDERED.
442

Panganiban, (Chairman), Corona, Carpio-Morales, and Garcia, JJ., concur.

Footnotes

1Rollo, pp. 55-59. Penned by Associate Justice Artemio G. Tuquero, retired, and concurred in
by Associate Justice Arturo B. Buena, now retired Justice of this Court, and Associate Justice
Eubolo G. Verzola (deceased).

2 Rollo at 60.

3 Id. at 50.

4 Id. at 53.

5 Rollo at 59.

6 Rozas vs. Court of Appeals, G.R. No. 138955, October 29, 2002, 391 SCRA 351,
citing Vda. de Cruz vs. Court of Appeals, 304 SCRA 197 (1999).

7 Lavido vs. Court of Appeals, G.R. No. 123462, April 10, 1997, 271 SCRA 143.

8Progressive Development Corp., Inc. vs. Court of Appeals, G.R. No. 123555, January 22,
1999, 301 SCRA 637.

9Hilario vs. Court of Appeakls, 260 SCRA 420 (1996); Caniza vs Court of Appeals, 268
SCRA 640 (1997); Heirs of Placido Miranda vs. Court of Appeals, 255 SCRA 368 (1996).

10 Dikit vs. Ycasiano, 89 Phil. 44, 48 (1951); Medel vs. Militante, 41 Phil. 526, 530 (1921).

11 Rollo at 80.

12Jimenez vs. Patricia, Inc., G.R. No. 134651, September 18, 2000, 340 SCRA
525; Sumulong vs. Court of Appeals, G.R. No. 108817, May 10, 1994, 232 SCRA
372; Pangilinan vs. Aguilar, 43 SCRA 136.

13 Supra; Jimenez vs. Patricia, Inc., id.; Banco de Oro Savings and Mortgage Bank vs. Court
of Appeals, 182 SCRA 464, 469 (1990); Dacudao vs. Consolacion, 122 SCRA 877, 883
(1983); Vda. de Catchuela vs. Francisco, 98 SCRA 172, 177 (1980); Calubayan vs. Pascual,
21 SCRA 146, 148 (1967); Munoz vs. Court of Appeals, G.R. No. 102693, September 23,
1992, 214 SCRA 216.
443

GANILA v CA (G.R. No. 150755, June 28, 2005)

FIRST DIVISION

G.R. No. 150755 June 28, 2005

RENE GANILA,* EDUARDO DUMADA-OG, SR., RAFAEL GANILA, JOSE PASTRANA, LOURDES
GANILA, FLORENTINO GANILA, SERAFIN GANILA, LORETO ARELLANO, CONRADO GANILA,
VIVENCIO ALVIOR, EDUARDO GANTALA, AMPARO VILLANUEVA, ELEUTERIO SILVA,
ADELINA GANILA, FELIZARDO GANILA, SR., ENRIQUE GANILA, ABRAHAM TANONG, EMILIO
ALFARAS, JR., BAPTIST CHRISTIAN LEARNING CENTER, petitioners,
vs.
HON. COURT OF APPEALS AND VIOLETA C. HERRERA, respondents.

DECISION

QUISUMBING, J.:

For review on certiorari are the D E C I S I O N1 dated March 30, 2001 of the Court of Appeals in CA-
G.R. SP No. 58191, and its Resolution2 dated October 18, 2001 denying the motion for
reconsideration. The assailed decision denied the petition to set aside the Resolution3 of the
Regional Trial Court (RTC) of San Miguel, Jordan, Guimaras, Branch 65, affirming the Order of the
Municipal Circuit Trial Court (MCTC) for the 19 petitioners to vacate the contested parcel of land.

The facts are as follows:

On March 19, 1997, private respondent Violeta Herrera filed 21 ejectment Complaints4 before the
16th MCTC, Jordan-Buenavista-Nueva Valencia, Jordan, Guimaras. Private respondent alleged that
she owns Lot 1227 of the Cadastral Survey of Jordan, Guimaras, with an area of 43,210 square
meters; that she inherited the lot from her parents; and that she only tolerated petitioners to construct
residential houses or other improvements on certain portions of the lot without rental. Sometime in
September or October 1996, private respondent demanded that the petitioners vacate the lot and
remove their houses and other improvements thereon. Petitioners refused, despite offer of money by
way of assistance to them. After the barangay conciliation failed, private respondent filed
the complaints.

In their Answers,5 eight6 of the petitioners claimed that Lot 1227 was formerly a shoreline which they
developed when they constructed their respective houses. Another eight 7 maintained that their
houses stood on Lot 1229 of the Cadastral Survey of Jordan, Guimaras. The other three8 asserted
that Lot 1227 is a social forest area.

At the preliminary conference, the parties agreed to designate two geodetic engineers as
commissioners of the MCTC to conduct a relocation survey of Lot 1227 and to identify who among the
petitioners have houses within the lot.9
444

The commissioners reported that: (1) the house of Henry Gabasa, defendant in Civil Case No. 288-J,
is almost outside Lot 1227; (2) the house of Ludovico Amatorio, defendant in Civil Case No. 289-J,
diagonally traversed the boundary; and (3) the houses of the 19 petitioners are inside Lot 1227.10

Eight months after herein petitioners’ failure to comment on the manifestation of private respondent to
terminate the preliminary conference, the MCTC terminated the preliminary conference. 11 Thereafter,
petitioners’ counsel Atty. Nelia Jesusa L. Gonzales failed to file her clients’ position papers and
affidavits, even after they sought a 30-day extension to file the same.12

Consequently, the MCTC decided the cases as follows:

WHEREFORE, premises considered, judgment is hereby rendered in favor of the plaintiff whereby
each of the twenty-one (21) defendants are hereby ordered:

1. To vacate Lot 1227 of the Cadastral Survey of Jordan, Guimaras;

2. To pay Two Hundred Pesos (₱200.00) per month from October, 1996 as compensation for
the use of the property until the same is vacated; and

3. To pay Two Thousand Pesos (₱2,000.00) as attorney’s fees and litigation expenses.

SO ORDERED.13

Petitioners appealed to the RTC, Branch 65, at Jordan, Guimaras, which decided as follows:

WHEREFORE, premises considered, the decision in Civil Cases Nos. 0270-J, 0272-J, 0273-J, 0274-
J, 0275-J, 0276-J, 0277-J, 0278-J, 0279-J, 0280-J, 0281-J, 0282-J, 0283-J, 0284-J, 0285-J, 0286-J,
0287-J, 0291-J and 0292-J are hereby affirmed.

The decision of the court below in Civil Cases Nos. 0288-J and 0289-J are set aside. Civil Cases Nos.
0288-J and 0289-J are hereby DISMISSED.

SO ORDERED.14

The RTC ruled that the evidence showed the better right of private respondent to possess Lot 1227.
Private respondent’s position paper, affidavit and tax declaration supported her allegations. In
addition, the commissioners’ report and sketch plan showed that indeed petitioners occupy Lot 1227.
On the other hand, according to the RTC, the petitioners failed to present evidence which would show
that they are entitled to possess the lot.

Based on the sketch plan, the RTC dismissed the cases against Gabasa and Amatorio since their
houses occupy only a small area of Lot 1227. It declared that Gabasa and Amatorio believed in good
faith that the whole area they occupied was part of the seashore.

The 19 petitioners, who were ordered to vacate the lot, filed a joint petition for review with the Court of
Appeals. The appellate court denied the petition. Petitioners moved for reconsideration and filed an
amended petition. The Court of Appeals, however, affirmed the factual findings and conclusions
arrived at by the trial courts and denied the amended petition for lack of merit. 15 It also denied the
motion for reconsideration.

Petitioners are now before us, on a petition for review, alleging that:

The Honorable Court of Appeals, with due respect and deference, committed a reversible error in the
interpretation/application of the law in the instant case and in the appreciation of the facts and
evidence presented. The Court of Appeals gravely abused its discretion when it denied and dismissed
the petition filed by the petitioners.16
445

After considering the parties’ submissions, we find three basic issues: (1) Did the MCTC err in taking
jurisdiction over and deciding the cases? (2) Did the RTC err in sustaining the MCTC’s judgment? (3)
Did the CA err in denying the petition for review filed by the 19 petitioners ordered to be ejected?

Petitioners insist that private respondent should have filed an action to recover possession de
jure, not a mere complaint for ejectment, for two reasons. One, they possessed Lot 1227 in good faith
for more than 30 years in the concept of owners. And two, there was no withholding of possession
since private respondent was not in prior possession of the lot.

Private respondent states in her Comment before us that the allegations in her Complaints make out
a clear case of unlawful detainer which is cognizable by the MCTC. We are in agreement with her
stance. There was no error in the choice of the complainant’s remedy, a matter left to her
determination as the suitor. And the complaint itself is defined by the allegations therein, not the
allegations of the defendants.

At the outset, we note that petitioners question the MCTC’s jurisdiction yet they admit in their
preliminary statement that the Complaints filed are indeed for unlawful detainer, and that the only
issue to be determined is mere physical possession (possession de facto) and not juridical
possession (possession de jure), much less ownership.17

While petitioners assert that this case involves only deprivation of possession, they confuse the
remedy of an action for forcible entry with that of unlawful detainer. In unlawful detainer, prior physical
possession by the plaintiff is not necessary. It is enough that plaintiff has a better right of possession.
Actual, prior physical possession of a property by a party is indispensable only in forcible entry cases.
In unlawful detainer cases, the defendant is necessarily in prior lawful possession of the property but
his possession eventually becomes unlawful upon termination or expiration of his right to
possess.18 Thus, the fact that petitioners are in possession of the lot does not automatically entitle
them to remain in possession. And the issue of prior lawful possession by the defendants does not
arise at all in a suit for unlawful detainer, simply because prior lawful possession by virtue of contract
or other reasons is given or admitted. Unlike in forcible entry where defendants, by force, intimidation,
threat, strategy or stealth, deprive the plaintiff or the prior physical possessor of possession. Here
there is no evidence to show that petitioners entered the lot by any of these acts.

If only to stress the fundamental principles related to present controversy, jurisdiction over unlawful
detainer suits is vested in municipal trial courts.19 And in ejectment cases, the jurisdiction of the court
is determined by the allegations of the complaint.20

In this case for ejectment, private respondent’s allegations sufficiently present a case of unlawful
detainer. She alleged that (1) she owns Lot 1227; (2) she tolerated petitioners to construct their
houses thereon; (3) she withdrew her tolerance; and (4) petitioners refused to heed her demand to
vacate the lot. The Complaints were also filed within one year from the date of her demand. The
cause of action for unlawful detainer between the parties springs from the failure of petitioners to
vacate the lot upon lawful demand of the private respondent. When they refused to vacate the lot after
her demand, petitioners’ continued possession became unlawful. Her complaint for ejectment against
respondent, to put it simply, is not without sufficient basis.

Petitioners’ contention that private respondent should have filed an action to recover possession de
jure with the RTC is not supported by law or jurisprudence. The distinction between a summary action
of ejectment and a plenary action for recovery of possession and/or ownership of the land is settled in
our jurisprudence.

What really distinguishes an action for unlawful detainer from a possessory action (accion publiciana)
and from a reinvindicatory action (accion reinvindicatoria) is that the first is limited to the question
of possession de facto. An unlawful detainer suit (accion interdictal) together with forcible entry are
the two forms of an ejectment suit that may be filed to recover possession of real property. Aside from
the summary action of ejectment, accion publiciana or the plenary action to recover the right of
possession and accion reinvindicatoria or the action to recover ownership which includes recovery of
possession, make up the three kinds of actions to judicially recover possession. 21
446

It is not up to defendants, now petitioners herein, to dictate upon plaintiff, now the private respondent,
what her initial recourse should be. Her choice of an action for ejectment against so-called squatters
is well within her rights.

Petitioners cite the case of Bayubay v. Court of Appeals,22 and argue that the MCTC’s decision was
without jurisdictional or legal basis because the MCTC did not issue a preliminary conference order.
They assert that the 10-day period to file position papers and affidavits only starts after the parties
had received a preliminary conference order. They insist they were denied due process when the
MCTC decided the cases based merely on private respondent’s Complaints and affidavit, without
considering their Answers.

For her part, private respondent maintains that there was substantial compliance with the rules in the
MCTC’s conduct of the preliminary conference, hence there was no violation of due process nor
disregard of its proper jurisdiction.

Petitioners’ present contention was first raised only in their appeal to the RTC. Raising it before the
appellate tribunal is barred by estoppel. 23 They should have raised it in the proceedings before the
MCTC. In our view, this issue is a mere afterthought, when the MCTC decided against them. Basic
rules of fair play, justice and due process require that as a rule an issue cannot be raised by the
petitioners for the first time on appeal.24

Besides, petitioners did not question initially the MCTC’s Order dated February 19, 1999, when they
moved for an extension of time to file their position papers and affidavits. They wanted another 30
days on top of the 30 days set by the MCTC, which strictly should have been 10 days only. In this
regard, petitioners could not claim that they were denied sufficient time to file their position papers
and affidavits before the trial court. Further, they cannot validly invoke our ruling 25 in Bayubay, for in
that case there was no order at all terminating the preliminary conference and requiring the parties to
submit position papers and affidavits.

We note with dismay petitioners’ insistence that we order the MCTC "to conduct the requisite
preliminary conference." The summary character of ejectment suits will be disregarded if we allow
petitioners to further delay this case by allowing a second preliminary conference. Ejectment by way
of forcible entry and unlawful detainer cases are summary proceedings, designed to provide an
expeditious means of protecting actual possession or the right to possession over the property
involved. It is a timely procedure designed to remedy the delay in the resolution of such cases. 26

Lastly, petitioners aver that private respondent failed to prove her allegation of ownership of Lot 1227
as it is only based on a tax declaration which is not an evidence of ownership. They also claim that
their possession of the lot was not and could not be by mere tolerance. However, this is a factual
matter best left to the trial courts.

What we have now is sufficient evidence showing that private respondent has a better right to
possess Lot 1227. The commissioners’ report and sketch plan show that the 19 petitioners occupy the
lot, which corroborate private respondent’s allegation and disprove petitioners’ defense that Lot 1227
is a shoreline; or that Lot 1227 is a social forest area. While not a conclusive evidence of ownership,
private respondent’s tax declaration constitutes proof that she has a claim of title over the lot. It has
been held that:

Although tax declarations or realty tax payment of property are not conclusive evidence of ownership,
nevertheless, they are good indicia of possession in the concept of owner for no one in his right mind
would be paying taxes for a property that is not in his actual or at least constructive possession. They
constitute at least proof that the holder has a claim of title over the property. The voluntary declaration
of a piece of property for taxation purposes manifests not only one’s sincere and honest desire to
obtain title to the property and announces his adverse claim against the State and all other interested
parties, but also the intention to contribute needed revenues to the Government. Such an act
strengthens one’s bona fide claim of acquisition of ownership.27
447

The lower courts did not err in adjudicating the issue of possession. Mere absence of title over the lot
is not a ground for the courts to withhold relief from the parties in an ejectment case. Plainly stated,
the trial court has validly exercised its jurisdiction over the ejectment cases below. The policy behind
ejectment suits is to prevent breaches of the peace and criminal disorder, and to compel the party out
of possession to respect and resort to the law alone to obtain what she claims is hers. The party
deprived of possession must not take the law into his or her own hands.28For their part, herein
petitioners could not be barred from defending themselves before the court adequately, as a matter of
law and right.

However, petitioners in their defense should show that they are entitled to possess Lot 1227. If they
had any evidence to prove their defenses, they should have presented it to the MCTC with their
position papers and affidavits. But they ignored the court’s order and missed the given opportunity to
have their defenses heard, the very essence of due process.29 Their allegations were not only
unsubstantiated but were also disproved by the plaintiff’s evidence.

In sum, we find no reversible error much less any grave abuse of discretion committed by the Court of
Appeals. A person who occupies the land of another at the latter’s tolerance or permission, without
any contract between them, is necessarily bound by an implied promise that he will vacate upon
demand, failing which a summary action for ejectment is the proper remedy against him. 30 His status
is analogous to that of a lessee or tenant whose term of lease has expired but whose occupancy
continued by tolerance of the owner. In such a case, the date of unlawful deprivation or withholding of
possession is to be counted from the date of the demand to vacate.31

WHEREFORE, the instant petition is DENIED for lack of merit. The Decision of the Court of Appeals
dated March 30, 2001 and its Resolution dated October 18, 2001 are AFFIRMED.

Costs against petitioners.

SO ORDERED.

Davide, Jr., C.J., (Chairman), Ynares-Santiago, Carpio, and Azcuna, JJ., concur.

Footnotes

* "Galila" in some parts of the records.

1Rollo, pp. 24-34. Penned by Associate Justice Buenaventura J. Guerrero, with Associate
Justices Eriberto U. Rosario, Jr., and Alicia L. Santos concurring.

2 Id. at 39.

3 Id. at 51-63.

4 Civil Case Nos. 270-J; 272-J; 273-J; 274-J; 275-J; 276-J; 277-J; 278-J; 279-J; 280-J; 281-J;
282-J; 283-J; 284-J; 285-J; 286-J; 287-J; 288-J; 289-J; 291-J; 292-J entitled VIOLETA C.
HERRERA v. RENE GANILA; EDUARDO DUMADA-OG, SR.; RAFAEL GANILA; JOSE
PASTRANA; LOURDES GANILA; FLORENTINO GANILA; SERAFIN GANILA; LORETO
ARELLANO; CONRADO GANILA; VIVENCIO ALVIOR; EDUARDO GANTALA; AMPARO
VILLANUEVA; ELEUTERIO SILVA; ADELINA GANILA; FELIZARDO GANILA, SR.;
ENRIQUE GANILA; ABRAHAM TANONG; HENRY GABASA; LUDOVICO AMATORIO;
EMILIO ALFARAS, JR.; and BAPTIST CHRISTIAN LEARNING CENTER; respectively
(CA Rollo, pp. 115-261). Henry Gabasa and Ludovico Amatorio are not among the petitioners
since the Complaints against them were dismissed by the RTC.
448

5 CA Rollo, pp. 262-392.

6Rene Ganila, Eduardo Dumada-og, Sr., Rafael Ganila, Jose Pastrana, Lourdes Ganila,
Florentino Ganila, Serafin Ganila and Loreto Arellano.

7Conrado Ganila, Vivencio Alvior, Eduardo Gantala, Eleuterio Silva, Amparo Villanueva,
Adelina Ganila, Felizardo Ganila, Sr. and Enrique Ganila.

8Emilio Alfaras, Jr., Abraham Tanong and the Baptist Christian Learning Center. Henry
Gabasa and Ludovico Amatorio also interposed the same defense.

9 Rollo, p. 26; See also Petitioners’ Amended Petition for Review with the CA, CA Rollo, p. 89.

10 CA Rollo, p. 421.

11 Id. at 455.

12 Rollo, pp. 32, 58, 59.

13 Id. at 49-50.

14 Id. at 63.

15 Id. at 33-34.

16 Id. at 9.

17 Id. at 10.

18 Rivera v. Rivera, G.R. No. 154203, 8 July 2003, 405 SCRA 466, 470.

19 Rules of Court, Rule 70, Section 1. – Who may institute proceedings, and when. –
Subject to the provisions of the next succeeding section, a person deprived of the possession
of any land or building by force, intimidation, threat, strategy, or stealth, or a lessor, vendor,
vendee, or other person against whom the possession of any land or building is unlawfully
withheld after the expiration or termination of the right to hold possession, by virtue of any
contract, express or implied, or the legal representatives or assigns of any such lessor,
vendor, vendee, or other person, may, at any time within one (1) year after such unlawful
deprivation or withholding of possession, bring an action in the proper Municipal Trial Court
against the person or persons unlawfully withholding or depriving of possession, or any
person or persons claiming under them, for the restitution of such possession, together with
damages and costs.

20Heirs of Demetrio Melchor v. Melchor, G.R. No. 150633, 12 November 2003, 415 SCRA
726, 732.

21 Custodio v. Corrado, G.R. No. 146082, 30 July 2004, 435 SCRA 500, 510.

22 G.R. No. 105866, 6 July 1993, 224 SCRA 557.

23 Villaranda v. Villaranda, G.R. No. 153447, 23 February 2004, 423 SCRA 571, 580.

24 Id. at 579-580.

25 Supra, note 22 at 560.


449

[I]mmediately after the preliminary conference, the Municipal Trial Court should issue
an order clearly and distinctly setting forth the issues of the case and the other
matters taken up during the preliminary conference.

The order is an important part of the summary procedure because it is its receipt by
the parties that begins the ten-day period to submit the affidavits and other evidence
mentioned in Sec. 7.

The minutes of the Municipal Trial Court dated August 22, 1989, contained a notation
that the pre-trial had been "terminated" and that the parties were to submit position
papers. However, there was no order to this effect nor was there an indication of
when the position papers were to be submitted for the purpose of discussing the
factual questions raised. (Underscoring supplied.)

26Don Tino Realty and Dev’t. Corp. v. Florentino, G.R. No. 134222, 10 September 1999, 372
Phil. 882, 889.

27 Alcaraz v. Tangga-an, G.R. No. 128568, 9 April 2003, 401 SCRA 84, 90-91.

28 Pajuyo v. Court of Appeals, G.R. No. 146364, 3 June 2004, 430 SCRA 492, 512.

29 Tubiano v. Razo, G.R. No. 132598, 13 July 2000, 390 Phil. 863, 869.

30 Rollo, p. 33; Boy v. Court of Appeals, G.R. No. 125088, 14 April 2004, 427 SCRA 196, 206.

31 Rollo, p. 49.

ROSS RICA SALES CENTER v SPS. ONG (G.R. No. 132197, August 16, 2005)

Republic of the Philippines


SUPREME COURT

SECOND DIVISION

G.R. No. 132197 August 16, 2005

ROSS RICA SALES CENTER, INC. and JUANITO KING & SONS, INC., Petitioners,
vs.
SPOUSES GERRY ONG and ELIZABETH ONG, Respondent.

DECISION

Tinga, J.:

In a Decision1 dated 6 January 1998, the Former First Division of the Court of Appeals overturned the
decisions of the Municipal Trial Court (MTC) and the Regional Trial Court (RTC) of Mandaue City,
ruling instead that the MTC had no jurisdiction over the subject complaint for unlawful detainer. This
petition for review prays for the reversal of the aforesaid Court of Appeals’ Decision.

The case originated from a complaint for ejectment filed by petitioners against respondents, docketed
as Civil Case No. 2376, before the MTC of Mandaue City, Branch I. In the complaint, petitioners
alleged the fact of their ownership of three (3) parcels of land covered by Transfer Certificates of Title
(TCT) Nos. 36466, 36467 and 36468. Petitioners likewise acknowledged respondent Elizabeth Ong’s
ownership of the lots previous to theirs. On 26 January 1995, Atty. Joseph M. Baduel, representing
Mandaue Prime Estate Realty, wrote respondents informing them of its intent to use the lots and
450

asking them to vacate within thirty (30) days from receipt of the letter. But respondents refused to
vacate, thereby unlawfully withholding possession of said lots, so petitioners alleged.

Ross Rica Sales Center, Inc. and Juanito King and Sons, Inc. (petitioners) had acquired the lands
from Mandaue Prime Estate Realty through a sale made on 23 March 1995. In turn, it appears that
Mandaue Prime Estate Realty had acquired the properties from the respondents through a Deed of
Absolute Sale dated 14 July 1994. However, this latter deed of sale and the transfers of title
consequential thereto were subsequently sought to be annulled by respondents in a complaint filed on
13 February 1995 before the Mandaue RTC against Mandaue Prime Estate Realty.2 Per record, this
case is still pending resolution.

Meanwhile, the MYC resolved the ejectment case on 24 April 1996, with the decision ordering
respondents to vacate the premises in question and to peacefully turn over possession thereof to
petitioners.

On appeal, the RTC rendered on 1 March 1997 a judgment affirming the MTC’s decision in its
entirety.

On 8 May 1997, respondents filed a notice of appeal. However, on the following day, they filed a
motion for reconsideration.

On 23 June 1997, the RTC issued an Order which concurrently gave due course to respondents’
notice of appeal filed on 8 May 1997; denied their motion for reconsideration dated 9 May 1997,3 and
granted petitioners’ motion for immediate execution pending appeal.

In a Petition for Certiorari with Injunction filed with the Court of Appeals and treated as a Petition for
Review, the appellate court ruled that the MTC had no jurisdiction over said case as there was no
contract between the parties, express or implied, as would qualify the same as one for unlawful
detainer. Thus, the assailed Orders of the MTC and RTC were set aside.

Petitioners then took this recourse via Petition for Review under Rule 45 of the Rules of Court. The
principal issues raised before this Court are: (i) whether the RTC decision has already become final
and executory at the time the petition for review was filed; (ii) whether the allegations in the complaint
constitute a case for unlawful detainer properly cognizable by the MTC; and, (iii) whether petitioners,
as registered owners, are entitled to the possession of the subject premises.

We resolve the first argument to be without merit.

The following sequence of events is undisputed:

(1) On 1 March 1997, the RTC rendered the questioned decision affirming the judgment of the MTC.

(2) On 28 April 1997, respondents received a copy of the aforementioned decision.

(3) On 8 May 1997, respondents filed a Notice of Appeal with the RTC.

(4) On 9 May 1997, respondents filed likewise with the RTC a Motion for Reconsideration of the
aforementioned 1 March 1997 decision.

(5) On 23 June 1997, the RTC of Mandaue issued an Order denying respondents’ Motion for
Reconsideration.

(6) On 9 July 1997, respondents received a copy of the aforementioned 23 June 1997 Order.

(7) On 24 July 1997, respondents filed with the Court of Appeals their motion for an additional period
of ten (10) days within which to file their Petition for Review.
451

(8) On 30 July 1997, respondents filed with the Court of Appeals their Petition for Review.

Petitioners assert that the Petition for Review was filed beyond the fifteen (15)-day period for appeal.
They theorize that the period started running on 28 April 1995, the date of receipt of the RTC
decision, and ended on 13 May 1997. According to them, this reglementary period could not have
been interrupted by the filing on 9 May 1997 of the Motion for Reconsideration because of the filing
one day earlier of the Notice of Appeal. This Notice of Appealdated 8 May 1997, albeit the wrong
mode of appeal, expressly manifested their intention to file a petition for review to either the Court of
Appeals or the Supreme Court.4

Petitioners further argue that respondents, after having filed the Notice of Appeal which was given
due course by the RTC, cannot take an inconsistent stand such as filing a Motion for Reconsideration.
Such filing, therefore, did not toll the fifteen (15)-day period which started running from the date of
receipt of the RTC decision on 28 April 1997 and ended on 13 May 1997.

Respondents, in their Comment,5 submit that the filing of the Notice of Appeal dated 8 May 1997 was
improper, and as such did not produce any legal effect. Therefore, the filing of the Motion for
Reconsideration immediately on the following day cured this defect. The RTC refused to subscribe
respondents’ position. It justified the denial of the Motion for Reconsideration on the ground that the
respondents had already filed a Notice of Appeal. The Orderdated 23 June 1997 stated:

On record is a Notice of Appeal by Certiorari filed by Defendants on May 8, 1997.

Likewise filed by Defendants on May 9, 1997 is a Motion for Reconsideration.

Considering the Notice of Appeal filed earlier which the court hereby approves, the Motion for
Reconsideration is DENIED.

The Motion for Immediate Execution Pending Appeal being meritorious, is GRANTED. 6 (Emphasis in
the original.)

Strangely enough, the Court of Appeals passed no comment on this point when it took cognizance of
respondents’ position and reversed the RTC. But does this necessarily mean that the RTC was
correct when it declared that the Motion for Reconsideration was barred by the filing of the Notice of
Appeal, no matter how erroneous the latter mode was?

Rule 42 governs the mode of appeal applicable in this case. Sec. 1 provides:

Section 1. How appeal taken; time for filing. -- A party desiring to appeal from a decision of the RTC
rendered in the exercise of its appellate jurisdiction may file a verified petition for review with the Court
of Appeals, paying at the same time to the clerk of said court the corresponding docket and other
lawful fees, depositing the amount of ₱500.00 for costs, and furnishing the Regional Trial Court and
the adverse party with a copy of the petition. The petition shall be filed and served within fifteen (15)
days from notice of the decision sought to be reviewed or of the denial of petitioner’s motion for new
trial or reconsideration filed in due time after judgment. Upon proper motion and the payment of the
full amount of the docket and other lawful fees and the deposit for costs before the expiration of the
reglementary period, the Court of Appeals may grant an additional period of fifteen (15) days only
within which to file the petition for review. No further extension shall be granted except for the most
compelling reason and in no case to exceed fifteen (15) days.

Since the unlawful detainer case was filed with the MTC and affirmed by the RTC, petitioners should
have filed a Petition for Review with the Court of Appeals and not a Notice of Appeal with the RTC.
However, we consider this to have been remedied by the timely filing of the Motion for
Reconsideration on the following day. Section 3, Rule 50 of the Rules of Court allows the withdrawal
of appeal at any time, as a matter of right, before the filing of the appellee’s brief. Applying this rule
contextually, the filing of the Motion for Reconsideration may be deemed as an effective withdrawal of
the defective Notice of Appeal.
452

Perforce, the period of appeal was tolled by the Motion for Reconsideration and started to run again
from the receipt of the order denying the Motion for Reconsideration. A Motion for Additional Time to
File the Petition was likewise filed with the Court of Appeals. Counting fifteen (15) days from receipt of
the denial of the Motion for Reconsideration and the ten (10)-day request for additional period, it is
clear that respondents filed their Petition for Review on time.

Petitioners invoke to the ruling in People v. De la Cruz7 that once a notice of appeal is filed, it cannot
be validly withdrawn to give way to a motion for reconsideration. The factual circumstances in the two
cases are different.

De la Cruz is a criminal case, governed by criminal procedure. Section 3, Rule 122 of the Rules of
Court provides that the proper mode of appeal from a decision of the RTC is a notice of appeal and
an appeal is deemed perfected upon filing of the notice of appeal.

In the case at bar, a petition for review before the Court of Appeals is the proper mode of appeal from
a decision of the RTC. Since the filing of the notice of appeal is erroneous, it is considered as if no
appeal was interposed.

Now on the second and more important issue raised by petitioners: whether the Complaint satisfies
the jurisdictional requirements for a case of unlawful detainer properly cognizable by the MTC.

The MTC considered itself as having jurisdiction over the ejectment complaint and disposed of the
same in favor of petitioners. Said ruling was affirmed by the RTC. The Court of Appeals reversed the
lower courts and found the complaint to be one not for unlawful detainer based on two (2) grounds,
namely: that the allegations fail to show that petitioners were deprived of possession by force,
intimidation, threat, strategy or stealth; and that there is no contract, express or implied, between the
parties as would qualify the case as one of unlawful detainer.

We disagree with the Court of Appeals.

The complaint for unlawful detainer contained the following material allegations:

....

3. That plaintiffs are the owners of Lot No. 2, which is covered by T.C.T. No. 36466 of the Register of
Deeds of Mandaue City, Lot No. 1-A which is covered by T.C.T. No. 36467 of the Register of Deeds
of Mandaue City and Lot No. 86-A which is covered by T.C.T. No. 36468 of the Register of Deeds of
Mandaue City, all situated in the City of Mandaue. Copies of said Transfer Certificate of Titles are
hereto attached as Annexes "A", "B", and "C" respectively and made an integral part hereof;

4. That defendant Elizabeth Ong is the previous registered owner of said lots;

5. That as the previous registered owner of said lots, defendant Elizabeth Ong and her husband and
co-defendant Jerry Ong have been living in the house constructed on said lots;

6. That on May 6, 1995, plaintiffs, through the undersigned counsel, wrote defendants a letter
informing them or their intent to use said lots and demanded of them to vacate said lots within 30
days from receipt of said letter. Copy of said letter is hereto attached as Annex "D" and made an
integral part thereof;

7. That despite demand to vacate, the defendants have refused and still refuse to vacate said lots,
thus, unlawfully withholding possession of said lots from plaintiffs and depriving plaintiffs of the use of
their lots;

8. That in unlawfully withholding the possession of said lots from the plaintiffs, plaintiffs have suffered
damages in the form of unearned rentals in the amount of ₱10,000.00 a month
453

. . . .8

Well-settled is the rule that what determines the nature of an action as well as which court has
jurisdiction over it are the allegations of the complaint and the character of the relief sought. 9

Respondents contend that the complaint did not allege that petitioners’ possession was originally
lawful but had ceased to be so due to the expiration of the right to possess by virtue of any express or
implied contract.

The emphasis placed by the Court of Appeals on the presence of a contract as a requisite to qualify
the case as one of unlawful detainer contradicts the various jurisprudence dealing on the matter.

In Javelosa v. Court of the Appeals,10 it was held that the allegation in the complaint that there was
unlawful withholding of possession is sufficient to make out a case for unlawful detainer. It is equally
settled that in an action for unlawful detainer, an allegation that the defendant is unlawfully withholding
possession from the plaintiff is deemed sufficient, without necessarily employing the terminology of
the law.11

Hence, the phrase "unlawful withholding" has been held to imply possession on the part of defendant,
which was legal in the beginning, having no other source than a contract, express or implied, and
which later expired as a right and is being withheld by defendant.12 In Rosanna B. Barba v. Court of
Appeals,13 we held that a simple allegation

that the defendant is unlawfully withholding possession from plaintiff is sufficient.

Based on this premise, the allegation in the Complaint that:

. . . . despite demand to vacate, the defendants have refused and still refuse to vacate said lots, thus,
unlawfully withholding possession of said lots from plaintiffs and depriving plaintiffs of the use of their
lots;14

is already sufficient to constitute an unlawful detainer case.

In the subject complaint, petitioners alleged that they are the registered owners of the lots covered by
TCT Nos. 36466, 36467 and 36468. By their implied tolerance, they have allowed respondents, the
former owners of the properties, to remain therein. Nonetheless, they eventually sent a letter to
respondents asking that the latter vacate the said lots. Respondents refused, thereby depriving
petitioners of possession of the lots. Clearly, the complaint establishes the basic elements of an
unlawful detainer case, certainly sufficient for the purpose of vesting jurisdiction over it in the MTC.

Respondents would like to capitalize on the requisites as cited in the case of Raymundo dela Paz v.
Panis.15 But the citation is a mere reiteration of Sec. 1, Rule 7016 of the Rules of Court. The case
doesid not provide for rigid standards in the drafting of the ejectment complaint. The case of Co
Tiamco v. Diaz17 justifies a more liberal approach, thus:

. . . The principle underlying the brevity and simplicity of pleadings in forcible entry and unlawful
detainer cases rests upon considerations of public policy. Cases of forcible entry and detainer are
summary in nature, for they involve perturbation of social order which must be restored as promptly
as possible and, accordingly, technicalities or details of procedure should be carefully avoided. 18

Moreover, petitioners fail to mention any of the incidents of the pending case involving the annulment
of deed of sale and title over said property. Petitioners know better than to question this in an
ejectment proceeding, which brings us to the nature of the action in this case.

Respondents insist that the RTC, and not the MTC, had jurisdiction over the action, it being an accion
reivindicatoriaaccording to them, on the ground that petitioners were constantly claiming ownership
over the lands in the guise of filing an action for ejectment. In their Comment,19 respondents maintain
454

that they occupy the subject lots as the legal owners. Petitioners, on the other hand, are seeking
recovery of possession under a claim of ownership which is tantamount to recovery of possession
based on alleged title to the lands, and therefore is within the original jurisdiction of the RTC, so
respondents conclude.

This contention is not tenable.

The issue involved in accion reivindicatoria is the recovery of ownership of real property. This differs
from accion publiciana where the issue is the better right of possession or possession de
jure, and accion interdictal where the issue is material possession or possession de facto. In an action
for unlawful detainer, the question of possession is primordial while the issue of ownership is
generally unessential.20

Neither the allegation in petitioners’ complaint for ejectment nor the defenses thereto raised by
respondents sufficiently convert this case into an accion reivindicatoria which is beyond the province
of the MTC to decide. Petitioners did not institute the complaint for ejectment as a means of claiming
or obtaining ownership of the properties. The acknowledgment in their pleadings of the fact of prior
ownership by respondents does not constitute a recognition of respondents’ present ownership. This
is meant only to establish one of the necessary elements for a case of unlawful detainer, specifically
the unlawful withholding of possession. Petitioners, in all their pleadings, only sought to recover
physical possession of the subject property. The mere fact that they claim ownership over the parcels
of land as well did not deprive the MTC of jurisdiction to try the ejectment case.

Even if respondents claim ownership as a defense to the complaint for ejectment, the conclusion
would be the same for mere assertion of ownership by the defendant in an ejectment case will not
therefore oust the municipal court of its summary jurisdiction.21 This Court in Ganadin

v. Ramos22 stated that if what is prayed for is ejectment or recovery of possession, it does not matter if
ownership is claimed by either party. Therefore, the pending actions for declaration of nullity of deed
of sale and Transfer Certificates of Title and quieting of title in Civil Case No. MAN-2356 will not abate
the ejectment case.

In Drilon v. Gaurana,23 this Court ruled that the filing of an action for reconveyance of title over the
same property or for annulment of the deed of sale over the land does not divest the MTC of its
jurisdiction to try the forcible entry or unlawful detainer case before it, the rationale being that, while
there may be identity of parties and subject matter in the forcible entry case and the suit for
annulment of title and/or reconveyance, the rights asserted and the relief prayed for are not the
same.24

In Oronce v. Court of Appeals,25 this Court held that the fact that respondents had previously filed a
separate action for the reformation of a deed of absolute sale into one of pacto de retro sale or
equitable mortgage in the same

Court of First Instance is not a valid reason to frustrate the summary remedy of ejectment afforded by
law to the plaintiff. Consequently, an adjudication made in an ejectment proceeding regarding the
issue of ownership should be regarded as merely provisional and, therefore, would not bar or
prejudice an action between the same parties involving title to the land. The foregoing doctrine is a
necessary consequence of the nature of forcible entry and unlawful detainer cases where the only
issue to be settled is the physical or material possession over the real property, that is, possession de
facto and not possession de jure.

The Court reiterated this in the case of Tecson v. Gutierrez26 when it ruled:

We must stress, however, that before us is only the initial determination of ownership over the lot in
dispute, for the purpose of settling the issue of possession, although the issue of ownership is
inseparably linked thereto. As such, the lower court's adjudication of ownership in the ejectment case
is merely provisional, and our affirmance of the trial courts' decisions as well, would not bar or
455

prejudice an action between the same parties involving title to the property, if and when such action is
brought seasonably before the proper forum.

The long settled rule is that the issue of ownership cannot be subject of a collateral attack.

In Apostol v. Court of Appeals,27 this Court had the occasion to clarify this:

. . . Under Section 48 of Presidential Decree No. 1529, a certificate of title shall not be subject to
collateral attack. It cannot be altered, modified or cancelled, except in a direct proceeding for that
purpose in accordance with law. The issue of the validity of the title of the respondents can only be
assailed in an action expressly instituted for that purpose. Whether or not the petitioners have the
right to claim ownership over the property is beyond the power of the court a quo to determine in an
action for unlawful detainer.28

With the conclusion of the second issue in favor of petitioners, there is no need to discuss the third
assignment of error which is related to the second issue.

WHEREFORE, the Petition is GRANTED. The Decision of the Court of Appeals dated 6 January 1998
is REVERSED and SET ASIDE and the Decision dated 24

April 1996 of the Municipal Trial Court of Mandaue City REINSTATED and AFFIRMED. Costs against
respondents.

SO ORDERED.

DANTE O. TINGA Associate Justice

WE CONCUR:

REYNATO S. PUNO

Associate Justice

Chairman

MA. ALICIA AUSTRIA-MARTINEZ ROMEO J. CALLEJO, SR.

Associate Justice Associate Justice

MINITA V. CHICO-NAZARIO

Associate Justice

ATTESTATION

I attest that the conclusions in the above Decision were reached in consultation before the case was
assigned to the writer of the opinion of the Court’s Division.

REYNATO S. PUNO

Associate Justice
Chairman, Second Division

CERTIFICATION
456

Pursuant to Section 13, Article VIII of the Constitution, and the Division Chairman’s Attestation, it is
hereby certified that the conclusions in the above Decision were reached in consultation before the
case was assigned to the writer of the opinion of the Court’s Division.

HILARIO G. DAVIDE, JR.

Chief Justice

Footnotes

1 Rollo,
pp. 24-32; Penned by Acting Presiding Justice Fidel P. Purisima, (later named
Associate Justice of the Supreme Court), concurred in by Associate Justices Ricardo P.
Galvez and B.A. Adefuin-De la Cruz.

2 Docketed as Civil Case No. MAN-2356 for Declaration of Deed of Sale and Transfer
Certificates of Titles as Null and Void ab initio and Quieting of Title, Damages and Attorney’s
Fees.

3A motion for execution pending appeal was also granted in the same Order. Rollo, p. 13.

4 Id. at 12.

5 Id. at 61-84.

6 Id. at 12-13.

7 312 Phil. 158 (1995).

8 Id. at 86-88.

9 Caniza v. Court of Appeals, 335 Phil. 1107 (1997); Ten Forty Realty and Development Corp.
v. Cruz, G.R. No. 151212, 410 SCRA 484, 10 September 2003.

10 333 Phil. 331 (1996).

11 Sumulong v. Court of Appeals, G.R. No. 108817, 10 May 1994, 232 SCRA 372; Pangilinan
v. Aguilar, 150 Phil. 166 (1972); Virgilio v. Jimenez, 394 Phil. 877 (2000).

12 Javelosa v. Court of Appeals, supra note 9 at 339.

13 426 Phil. 598 (2002).

14 Rollo, p. 87.

15 315 Phil. 238 (1995).

16 SECTION 1. Who may institute proceedings, and when. – Subject to the provisions of the
next succeeding section, a person deprived of the possession of any land or building by force,
intimidation, threat, strategy, or stealth, or a lessor, vendor, vendee, or other person against
whom the possession of any land or building is unlawfully withheld after the expiration or
termination of the right to hold possession, by virtue of any contract, express or implied, or the
legal representatives or assigns of any such lessor, vendor, vendee or other person, may, at
any time within one (1) year after such unlawful deprivation or withholding of possession,
457

bring an action in the proper Municipal Trial Court against the person or persons unlawfully
withholding or depriving of possession, or any person or persons claiming under them, for the
restitution of such possession, together with damages and costs.

17 75 Phil. 672 (1946).

18 Id. at 686.

19 Rollo, p. 73.

20 Fige v. Court of Appeals, G.R. No. 107951, 30 June 1994, 233 SCRA 586.

21 Ching v. Malaya, G.R. No. L-56449, 31 August 1987, 153 SCRA 412.

22 L-23547, 11 September 1980, 99 SCRA 613.

23 G.R. No. L-35482 30 April 1987, 149 SCRA 342.

24 Feliciano v. Court of Appeals, 336 Phil. 499 (1998).

25 331 Phil. 616 (1998).

26 G.R.No. 152978, 4 March 2005, citing Balanon-Anicete v. Balanon, G.R. Nos. 150820-21,
30 April 2003, 402 SCRA 514; Co v. Militar, G.R. No. 149912, 29 January 2004, 421 SCRA
455.

27 G.R. No. 125375, 17 June 2004, 432 SCRA 351.

28 Id. at 359.

PERALTA-LABRADOR v BUARIN (G.R. No. 165177, August 25, 2005)

Republic of the Philippines


SUPREME COURT

FIRST DIVISION

G.R. No. 165177 August 25, 2005

LILIA V. PERALTA-LABRADOR, Petitioners,


vs.
SILVERIO BUGARIN, substituted by his widow, CONSOLACION BUGARIN,1 Respondent.
458

DECISION

YNARES-SANTIAGO, J.:

Challenged in this petition for review on certiorari is the March 12, 2004 decision 2 of the Court of
Appeals in CA-G.R. SP No. 57475, which affirmed with modification the January 26, 2000
judgment3 of the Regional Trial Court (RTC) of Iba, Zambales, Branch 71, in Civil Case No. RTC-
1590-I, which in turn affirmed the decision4 dated May 16, 1999 of the Municipal Trial Court (MTC) of
San Felipe, Zambales, in Civil Case No. 328, and its September 6, 2004 resolution 5 denying
reconsideration thereof.

On January 18, 1996, petitioner Lilia V. Peralta-Labrador filed a case for "Recovery of Possession
and Ownership," docketed as Civil Case No. 328, with the MTC of San Felipe, Zambales. She alleged
that she is the owner of Cadastral Lot No. 2650, with an area of 400 sq. m. located at Sitio
Caarosipan, Barangay Manglicmot, San Felipe, Zambales, having purchased the same in 1976 from
spouses Artemio and Angela Pronto. In 1977, she was issued Tax Declaration No. 10462 and paid
the taxes due thereon.6

In 1990, the Department of Public Works and Highways constructed a road which traversed Cadastral
Lot No. 2650 thereby separating 108 sq. m. from the rest of petitioner’s lot, for which she was issued
Tax Declaration No. 02-2460R in 1991.7

Sometime in 1994, respondent Silverio Bugarin forcibly took possession of the 108 sq. m. lot and
refused to vacate the same despite the pleas of petitioner. Hence, on January 18, 1996, she instituted
a complaint for recovery of possession and ownership against respondent.

In his Answer with Counterclaims,8 respondent contended that the area claimed by petitioner is
included in the 4,473 square meter lot, covered by the Original Certificate of Title (OCT) No. P-13011;
and that he has been in continuous possession and occupation thereof since 1955. In his Amended
Answer with Counterclaim,9 however, respondent failed to allege that the questioned lot is covered by
the OCT No. P-13011, and instead asserted that he planted fruit bearing trees in the property.
Respondent further pleaded the defenses of lack of cause of action and prescription.

On May 16, 1999, the court a quo ruled in favor of respondent declaring him as the owner of the
controverted lot on the basis of the OCT No. P-13011. The complaint was dismissed for failure of
petitioner to prove prior physical possession and ownership thereof. The dispositive portion thereof,
reads:

WHEREFORE, all the foregoing premises considered and for failure on the part of the plaintiff to
establish the preponderance of evidence of prior actual physical possession and present title over the
lot in her favor, let the instant case be ordered DISMISSED, and the defendant be awarded the
rightful possession and ownership of the same and the plaintiff is hereby ordered to pay FIFTEEN
THOUSAND (P15,000.00) PESOS as reasonable Attorney’s fee and FIVE THOUSAND (P5,000.00)
PESOS as appearance fee plus costs.

SO ORDERED.10

The RTC affirmed the assailed decision,11 hence petitioner filed a petition for review before the Court
of Appeals which was however denied for insufficiency of evidence to prove ownership or prior actual
physical possession. The appellate court deleted the monetary awards in favor of respondent as well
as the declaration of the MTC that respondent is the owner of the questioned lot on the ground that
the OCT No. P-13011, relied upon by said court was not formally offered in evidence, hence, cannot
be considered by the court. The decretal portion thereof, states:

WHEREFORE, in view of the foregoing discussion, the instant petition is hereby PARTIALLY
GRANTED. The assailed Decision dated January 26, 2000, in Civil Case No. RTC 1590 I of the
Regional Trial Court (RTC), Branch 71, Iba, Zambales, and Decision dated May 16, 1999, in Civil
Case No. 328 of the Municipal Trial Court of San Felipe, Zambales are MODIFIED by deleting the
459

declaration of ownership as to the disputed 108 square meters and the monetary award in favor of
respondent Silverio Bugarin. However, the dismissal of the complaint is AFFIRMED.

SO ORDERED.12

The motion for reconsideration filed by petitioner was denied. Hence the instant petition.

Pertinent portion of Section 1, Rule 70 of the Revised Rules of Civil Procedure, provides:

SECTION 1. Who may institute proceedings, and when. – … a person deprived of the possession of
any land or building by force, intimidation, threat, strategy, or stealth, … may at any time within one
(1) year after such unlawful deprivation or withholding of possession, bring an action in the proper
Municipal Trial Court against the person or persons unlawfully withholding or depriving of possession,
or any person or persons claiming under them, for the restitution of such possession, together with
the damages and costs. (Emphasis supplied)

In Lopez v. David Jr.,13 it was held that an action for forcible entry is a quieting process and the one
year time bar for filing a suit is in pursuance of the summary nature of the action. Thus, we have
nullified proceedings in the MTCs when it improperly assumed jurisdiction of a case in which the
unlawful deprivation or withholding of possession had exceeded one year. After the lapse of the one
year period, the suit must be commenced in the RTC via an accion publiciana, a suit for recovery of
the right to possess. It is an ordinary civil proceeding to determine the better right of possession of
realty independently of title. It also refers to an ejectment suit filed after the expiration of one year
from the accrual of the cause of action or from the unlawful withholding of possession of the realty
independently of title. Likewise, the case may be instituted before the same court as an accion
reivindicatoria, which is an action to recover ownership as well as possession. 14

Corrollarily, jurisdiction of a court is determined by the allegations of the complaint. Thus, in


ascertaining whether or not the action falls within the exclusive jurisdiction of the inferior courts, the
averments of the complaint and the character of the relief sought are to be examined.15

In the instant case, petitioner’s complaint alleges that:

2. That plaintiff is the owner of a parcel of land denominated as Cadastral lot No. 2650, San Felipe
Cadastre, situated at sitio Caarosipan, Barangay Manglicmot, San Felipe, Zambales which she
bought in 1976 from Spouses Artemio Pronto and Angela Merano when she was still a widow, with
the following boundaries: North, Alipio Abad, East, Antonio Cueva, South, Juan Borja, and West, Old
Provincial Road, containing an area of 108 square meters, declared under Tax Declaration No. 002-
1860R and assessed at P1,120.00;

3. That plaintiff has been in open, continuous, exclusive and adverse as well as notorious
possession of the said lot and in the concept of an owner since she [acquired] it in 1976 until
the time when defendant took possession forcibly, two years ago;

4. That in or before 1990 the land was traversed by a new National Highway and the land was
segregated from a bigger portion of the land, the western portion is now the land in question and
since the new provincial road which traversed the whole land of the plaintiff, the old highway which is
west of Lot 2650 shall belong to the plaintiff in compensation of the portion of her lot traversed by the
new highway, said old highway is also taken by defendant unlawfully; 16

It is clear that petitioner’s averment make out a case for forcible entry because she alleged prior
physical possession of the subject lot way back in 1976, and the forcible entry thereon by respondent.
Considering her allegation that the unlawful possession of respondent occurred two years17 prior to
the filing of the complaint on January 18, 1996, the cause of action for forcible entry has prescribed
and the MTC had no jurisdiction to entertain the case. Petitioner’s complaint therefore should have
been filed with the proper RTC.
460

It is settled that jurisdiction over the subject matter cannot be waived by the parties or cured by their
silence, acquiescence or even express consent.18 Hence, the failure of respondent to insist on the
defenses of lack of cause of action and prescription stated in his Amended Answer with Counterclaim
will not vest the MTC with jurisdiction over the case.

On this point, the Court held in Bongato v. Malvar19 that:

It is wise to be reminded that forcible entry is a quieting process, and that the restrictive time bar is
prescribed to complement the summary nature of such process. Indeed, the one-year period within
which to bring an action for forcible entry is generally counted from the date of actual entry to the land.
However, when entry is made through stealth, then the one-year period is counted from the time the
plaintiff learned about it. After the lapse of the one-year period, the party dispossessed of a parcel of
land may file either an accion publiciana, which is a plenary action to recover the right of possession;
or an accion reivindicatoria, which is an action to recover ownership as well as possession.

On the basis of the foregoing facts, it is clear that the cause of action for forcible entry filed by
respondents had already prescribed when they filed the Complaint for ejectment on July 10, 1992.
Hence, even if Severo Malvar may be the owner of the land, possession thereof cannot be wrested
through a summary action for ejectment of petitioner, who had been occupying it for more than one
(1) year. Respondents should have presented their suit before the RTC in an accion publiciana or
an accion reivindicatoria, not before the MTCC in summary proceedings for forcible entry. Their cause
of action for forcible entry had prescribed already, and the MTCC had no more jurisdiction to hear and
decide it.

...

Further, a court’s lack of jurisdiction over the subject matter cannot be waived by the parties or cured
by their silence, acquiescence or even express consent. A party may assail the jurisdiction of the
court over the action at any stage of the proceedings and even on appeal. That the MTCC can take
cognizance of a motion to dismiss on the ground of lack of jurisdiction, even if an answer has been
belatedly filed we likewise held in Bayog v. Natino[.]

Moreover, even if the MTC has jurisdiction over the subject matter, the complaint should still be
dismissed because petitioner failed to prove that the controverted 108 sq. m. lot is part of Cadastral
Lot No. 2650. Petitioner admitted that she has never seen the Cadastral Map of San Felipe,
Zambales, and relied only on the Survey Notification Card20 from the Bureau of Lands,21 with a sketch
of Cadastral Lot No. 2650. Said card, however, does not reflect the 108 sq. m. lot subject of this case.
Neither did petitioner cause the survey of Cadastral Lot No. 2650 after the construction of a new road
to prove that the segregated portion on the western side is part thereof. Ei incumbit probotio qui dicit,
non qui negat. He who asserts, not he who denies, must prove.22 Failing to discharge this burden, the
dismissal of the complaint is proper.

In the same vein, ownership of the lot in question cannot be awarded to respondent considering that
OCT No. P-13011,23 and the Survey Plan24 were not formally offered in evidence. While the issue of
ownership may be passed upon in ejectment cases for the sole purpose of determining the nature of
possession,25 no evidence conclusively show that the lot in question is covered by said OCT No. P-
13011 or any other title of respondent.

WHEREFORE, the May 16, 1999 decision of the Municipal Trial Court of San Felipe, Zambales, the
January 26, 2000 decision of the Regional Trial Court, Branch 71, Iba, Zambales, and the March 12,
2004 decision of the Court of Appeals, are ANNULLED and SET ASIDE for lack of jurisdiction. The
complaint in Civil Case No. 328 is DISMISSED.

SO ORDERED.

CONSUELO YNARES-SANTIAGO

Associate Justice
461

WE CONCUR:

HILARIO G. DAVIDE, JR.

Chief Justice

LEONARDO A. QUISUMBING ANTONIO T. CARPIO

Associate Justice Associate Justice

ADOLFO S. AZCUNA

Associate Justice

CERTIFICATION

Pursuant to Section 13, Article VIII of the Constitution, it is hereby certified that the conclusions in the
above Decision were reached in consultation before the case was assigned to the writer of the
opinion of the Court’s Division.

HILARIO G. DAVIDE, JR.

Chief Justice

Footnotes

1 Seethe July 18, 2000 Resolution of the Court of Appeals noting the death of Silverio
Bugarin and granting the substitution of Consolacion Bugarin as respondent. CA Rollo, p.
119.

2 Rollo,pp. 17-23. Penned by Associate Justice Hakim S. Abdulwahid with Associate Justices
Delilah Vidallon-Magtolis and Jose L. Sabio, Jr., concurring.

3 CA Rollo, pp. 142-144. Penned by Judge Romulo M. Estrada.

4 Id. at 53-62. Penned by Judge Lavezares C. Leomo.

5 Rollo, pp. 24-25.

6 Exhibit "B", Records, p. 349.

7 Exhibit "B-3", Id. at 352.

8 Records, pp. 11-13.

9 Id. at 147-150.

10 CA Rollo, pp. 61-62.

11 Id. at 142-144.

12 Rollo, p. 23.
462

13 G.R. No. 152145, 30 March 2004, 426 SCRA 535, 542-543.

14 Bongato v. Malvar, G.R. No. 141614, 14 August 2002, 387 SCRA 327, 338.

15 Sps. Tirona v. Hon. Alejo, 419 Phil. 285, 297. (2001).

16 Records, p. 1.

17 Petitionereven stated on direct examination that the unlawful possession of respondent


started in 1990. (TSN, 17 April 1996; Records, p. 88)

18 Bongato v. Malvar, supra at 340-341.

19 Id. at 338-341.

20 Exhibit "D," Records, p. 368.

21 TSN, 23 April 1996, p. 127.

22 Sps. Boyboy v. Atty. Yabut, Jr., 449 Phil. 664, 668 (2003).

23 Records, p. 17.

24 Id. at 396.

25 Sps. Refugia v. CA, 327 Phil. 982, 1001-1002 (1996).


463

SERINA v CABALLERO (G.R. No. 127382, August 17, 2004)

SECOND DIVISION

G.R. No. 127382 August 17, 2004

DR. JESUS SERIÑA and ENRIQUETA SERIÑA (deceased), represented by DR. JESUS SERIÑA,
JR., ANTONIO SERIÑA, VIOLETA SERIÑA TAN, REYNALDO SERIÑA and EMMANUEL
SERIÑA, petitioners,
vs.
VICTOR CABALLERO, TEODORO DONELA, OLIVER DONELA, COURT OF APPEALS, and THE
HONORABLE REGIONAL TRIAL COURT, BRANCH 20, MISAMIS ORIENTAL, respondents.

DECISION

CALLEJO, SR., J.:

Before us is a petition for review on certiorari of the Decision1 of the Court of Appeals (CA) dated
August 23, 1996, affirming the dismissal of the complaint for quieting of title, recovery of possession,
and damages by the Regional Trial Court (RTC) of Misamis Oriental, Cagayan de Oro City, in Civil
Case No. 8716.

The Antecedents

On August 11, 1982, Dr. Jesus Seriña and his wife, Enriqueta Seriña filed a Complaint for quieting of
title, recovery of possession, and damages with a prayer for a writ of preliminary mandatory injunction
against respondents Victor Caballero and his tenants, Teodoro Donela and Oliver Donela. When Dr.
Seriña died on August 6, 1983, he was substituted by his children, petitioners Jesus, Jr., Antonio,
Violeta, Reynaldo and Emmanuel.2

The petitioners alleged in their complaint that they are the absolute owners and have been in actual
and constructive possession for thirty-five (35) years of a parcel of land described as follows:

Lot No. 3533-A, Cad-237, Cagayan Cadastre

Tax Declaration No. 02161

Location - Mantadiao, Opol,

Misamis Oriental

Area - 2.5000 has.

Boundaries:

North - Alejo Seriña


464

South - T. Sabornido

East - A. Seriña & T. Sabornido

West - F. Caballero3

The petitioners averred that sometime in March 1982, they discovered that respondent Caballero was
claiming ownership over the said land and offering it for sale or mortgage to third parties. They also
discovered that the respondents Donelas were occupying the land as tenants and caretakers of the
land. 4

The petitioners claimed that their father, Dr. Seriña, bought the land from Lucia Vda. de Marbella who
inherited it from her father, Ramon Neri.5 They presented a Deed of Sale6 dated August 23, 1947
showing that Dr. Seriña bought 5 hectares of ricefield, bounded on the North by Raymundo Seriña, on
the East by Teofilo Saburnido, on the South by Obdelio Caballero, on the West by Obdullo Caballero,
from Lucia Vda. de Marbella. Dr. Seriña was issued Tax Declaration No. 4029 allegedly for the said
property. As indicated in the tax declaration and subsequent tax declarations issued in the name of
Dr. Seriña, they were issued for Cadastral Lot No. 3533 and covered a 2.5-hectare ricefield with the
same boundary owners as those in the complaint.7 The petitioners also averred that they regularly
paid taxes thereon since 1947 up to the present.8

In his answer, respondent Caballero alleged that he was the lawful owner, and had been in actual
physical possession of the disputed land since time immemorial. He averred that the disputed land is
part of Cadastral Lot No. 3533, C-7 of the Cagayan Cadastre and originally owned by his grandfather,
Eustaquio Caballero.9

The respondents averred that Eustaquio Caballero declared the entire parcel of land for tax purposes
even before the war. Tax Declaration No. 2442 was issued in lieu of the records that were destroyed
during the war.

This tax declaration indicated that the 119,490 square-meter parcel of land was located at Pontacon,
Iponan, Cagayan de Oro City, bounded on North by Rustico Dablio, on the East by J. Seriña and T.
Saburnido, on the South by Victor Obsioma, and on the West by Victorino Caballero. 10

Emiliana Ibarat, respondent Caballero’s sister, testified that when Eustaquio Caballero died in 1944,
the land was divided among his three children, Vicenta, Benita and Victorino, the father of respondent
Caballero. Lot A, with an area of 39,625 square meters, was given to Victorino, which was later
inherited by the respondent. Lot B, with an area of 71, 450 square meters, was given to Benita; and
Lot C, with only 7,938 square meters was given to Vicenta. Lots B and C were, thereafter, sold to one
Gaga Yasay. Because of the trouble between the petitioners and the respondents, Yasay agreed to
buy only a portion of Lot A.11

The land was surveyed during the trial and it was determined that it now consisted of only 23,373
square meters,12and not 25,000 square meters as claimed by the petitioners. Gliceria Legaspi,
respondent Caballero’s other sister, also testified that the disputed land was now bounded on the
North by Seriña and Nangcas, on the East by Teofilo Saburnido, on the South by Gaga Yasay, and
on the West by Nangcas.13

The RTC rendered judgment14 on January 21, 1992, dismissing the complaint, and upholding the right
of the respondents over the land. The dispositive portion reads:

WHEREFORE, judgment is hereby rendered in favor of the defendant Victor Caballero and
against the plaintiffs herein, to wit:

1. Ordering the dismissal of the complaint with costs.


465

2. Ordering the defendant Victor Caballero as the absolute and lawful owner and possessor
of the land in question.

3. Ordering the plaintiffs, their heirs, lawyers, servants or privies not to disturb or molest the
possession and ownership of Victor Caballero over the land in question.

4. Ordering the plaintiffs to pay to defendant Victor Caballero, jointly and severally the sum of
FIVE THOUSAND (P5,000.00) pesos for expenses of litigation, and THREE THOUSAND
(P3,000.00) pesos for and as attorney's fees having been compelled to retain the services of
counsel to protect his interest herein.

SO ORDERED.15

The trial court ruled that it was not clearly shown that the land bought by Dr. Seriña from Lucia Vda.
de Marbella was the same land owned by Victor Caballero, and that the petitioners failed to show that
Lucia Vda. de Marbella bought the land from Eustaquio Caballero, the original owner and cadastral
claimant of the land. It also noted that the deed of sale between Lucia Vda. de Marbella and Dr.
Seriña showed that the land had an area of 5 hectares, whereas, the petitioners only claimed 2.5
hectares. Furthermore, the boundaries of the land stated in the complaint did not coincide with what
was stated in the Deed of Sale, or in Tax Declaration No. 2442 in the name of Eustaquio Caballero.
The trial court ruled that the petitioners failed to explain these discrepancies, and that there was no
showing that Tax Declaration No. 2442 was cancelled by Tax Declaration No. 4029 in the name of Dr.
Seriña. The trial court interpreted this to mean that Eustaquio Caballero's right as owner of the land
remained.

Dissatisfied, the petitioners appealed the case to the CA, which rendered a Decision16 affirming in toto
the decision of the RTC. The petitioners filed a Motion for Reconsideration on September 30,
1996.17 The CA denied the motion.18

Hence, the instant petition.

The petitioners assign the following errors:

1. THAT IT IS ERROR FOR THE HONORABLE COURT OF APPEALS TO UPHOLD THE


HONORABLE RTC ON THE ISSUE THAT THE ALLEGED IDENTITY OF THE LAND IN
LITIGATION IS UNESTABLISHED BETWEEN THE PARTIES-LITIGANTS.

2. THAT IT IS ERROR FOR THE HONORABLE COURT OF APPEALS TO FAIL TO


APPRECIATE THE 35-YEAR ACQUISITIVE PRESCRIPTION IN FAVOR OF THE
PLAINTIFFS-APPELLANTS.19

The issues in this petition are, therefore, the following: (1) whether the petitioners were able to
establish the identity of the land being claimed by them; and (2) whether acquisitive prescription
should be appreciated in favor of the petitioners.

The Ruling of the Court

The first issue deals clearly with a question of fact which is beyond the province of this Court in a
petition for review on certiorari. Well-entrenched is the rule that the Court's jurisdiction in a petition for
review is limited to reviewing or revising errors of law allegedly committed by the appellate court.
Factual findings of the Court of Appeals are conclusive on the parties and not reviewable by this
Court—and they carry even more weight when the Court of Appeals affirms the factual findings of the
trial court.20 The exceptions to this rule are the following:

(1) when the conclusion is a finding grounded entirely on speculations, surmises or


conjectures; (2) when the inference made is manifestly mistaken, absurd or impossible; (3)
when there is grave abuse of discretion; (4) when the judgment is based on misapprehension
466

of facts; (5) when the findings of facts are conflicting; (6) when the Court of Appeals, in
making its findings, went beyond the issues of the case and the same is contrary to the
admissions of both appellant and appellee; (7) when the findings of the Court of Appeals are
contrary to those of the trial court; (8) when the findings of fact are conclusions without
citation of specific evidence on which they are based; (9) when the Court of Appeals
manifestly overlooked certain relevant facts not disputed by the parties, which, if properly
considered, would justify a different conclusion; and (10) when the findings of fact of the Court
of Appeals are premised on the absence of evidence and are contradicted by the evidence on
record.21

We find no cogent reason to reverse the findings of the CA. None of the aforementioned exceptions is
present in this case. The CA was correct in concluding that the petitioners failed to establish that the
parcel of land in the possession of the respondents is the same as that subject of their complaint.

The CA noted that the land subject of the complaint has boundaries different from the land in
possession of the respondents. In fact, the land described in the complaint appears to be different
from the land described in the Deed of Sale which the petitioners invoke as the basis of their
ownership.

First. The petitioners alleged in their complaint that the boundaries of their property are as follows:

North - Alejo Seriña

South - T. Sabornido

East - A. Seriña & T. Sabornido

West - F. Caballero22

On the other hand, the Deed of Sale provides that the property sold to them has the following
boundaries:

North - Raymundo Seriña

South - Obdullo Caballero

East - Teofilo Saburnido

West - Obdullo Caballero23

Second. The complaint24 of the petitioners states that the property they are claiming has an area of
2.5 hectares. On the other hand, the Deed of Sale25 provides that the subject property has an area of
5 hectares.

Third. The complaint alleged that the property is located in "Mantadiao, Opol, Misamis
Oriental,"26 while the Deed of Sale shows that the property purchased is located in "Puntakon, Igpit,
Cagayan Or. Misamis."27

We agree with the CA that there was no showing that Tax Declaration No. 2442 in the name of
Eustaquio Caballero was cancelled. Absent any specific statement therein to that effect, it cannot be
presumed that Tax Declaration No. 4029 in the name of Dr. Seriña cancelled Tax Declaration No.
2442.

Moreover, the land covered by Tax Declaration No. 2442 is different from that covered by Tax
Declaration No. 4029 for the following reasons:
467

The boundary owners of the land as indicated in Tax Declaration No. 2442 differ from those stated in
Tax Declaration No. 4029. The boundary owners as indicated in Tax Declaration No. 2442 are as
follows:

North - Rustico Dablio

South -Victor Obsioma

East - J. Seriña & T. Saburnido

West - Victorino Caballero28

Under Tax Declaration No. 4029, on the other hand, the boundary owners are as follows:

North - Alejo Seriña

South - Teofilo Saburnido

East - A. Seriña [and] T. Saburnido

West - Eustaquio Caballero29

Moreover, Tax Declaration No. 2442 covers an area of 119,490 square meters 30 while Tax
Declaration No. 4029 covers only 25,000 square meters or 2.5 hectares.31

The petitioners argue that the Deed of Sale and Tax Declaration No. 4029 should not be compared to
Tax Declaration No. 2442 and the Technical Description of Cadastral Lot No. 3533 because the
former refers only to a portion of the area referred to by the latter.32 While the petitioners are correct
on this point, such mistake would still not justify a different conclusion. The fact remains that the
documentary and testimonial evidence presented by the petitioners did not prove the identity of the
land being claimed. The petitioners did not present evidence to prove that the land registered in the
name of Eustaquio Caballero was sold to Lucia Vda. de Marbella or her predecessor-in-interest from
whom they purchased the land subject of their complaint.

The failure to establish the identity of the land is obviously fatal to the petitioners’ case. In Beo vs.
Court of Appeals,33 a case which also involves an action for possession and quieting of title, the Court
had the occasion to state:

…[B]ecause petitioners failed to explain the discrepancy or present other evidence to prove
with certainty the location and area of the land they seek to recover, respondent court
correctly applied the invariable rule that a person who claims ownership of real property
is duty-bound to clearly identify the land being claimed, in accordance with the title on
which he anchors his right of ownership. When the record does not show that the land
subject matter of the action for recovery of possession has been exactly determined, such
action cannot prosper, as in the case of petitioners. In sum, proof of ownership coupled
with identity of the land is the basic rule.

Corollarily, the rule is likewise well-settled that in order that an action for recovery of
possession may prosper, it is indispensable that he who brings the action fully proves
not only his ownership but also the identity of the property claimed, by describing the
location, area and boundaries thereof. As the appellate court succinctly stated, he who
claims to have a better right to the property must clearly show that the land possessed by the
other party is the very land that belongs to him.34

On the second issue, the CA ruled that inasmuch as the petitioners failed to establish that the parcel
of land in possession of the respondents is the same as the subject of their complaint, their claim of
acquisitive prescription is clearly untenable.
468

The petitioners argue that they would not have regularly paid taxes on the land since 1947 had they
not believed that they owned the same.35 The respondents, for their part, aver that the petitioners
were only able to prove seven (7) years of actual possession of the land through cultivation by their
tenants. They argue that such seven-year period of cultivation cannot be considered in the petitioners’
favor, since the witness who testified on this fact did not personally know the boundaries of the land
cultivated, or whether it was the same land bought by Dr. Seriña. The respondents contend that
acquisitive prescription applies only when there is no dispute as to the identity of the property.36

We agree with the respondents. Since the property has not been clearly identified by the petitioners,
their claim of acquisitive prescription cannot be considered. Insufficient identification of the portion of
land claimed in absolute ownership cannot ripen into ownership. Possession as a means of acquiring
ownership, while it may be constructive, is not a mere fiction.37

Assuming, however, that the disputed land has been clearly identified, acquisitive prescription will still
not lie in favor of the petitioners because they were not able to prove that they have been in
possession of the property for the requisite number of years. Prescription requires public, peaceful,
uninterrupted and adverse possession of the property in the concept of an owner for ten years, in
case the possession is in good faith and with just title.38

Aside from the testimony of Leonardo Vacalares that certain tenants of the petitioners cultivated the
land for a total of seven years, the petitioners did not present any other evidence to show that they
have been in actual possession of the property for at least ten years.

The petitioners’ argument that the payment of taxes on the property since May 31, 1948 constitutes
proof of their possession of the subject land for thirty-five years is untenable. Tax declarations and
receipts are not conclusive evidence of ownership. At most, they constitute mere prima facie proof of
ownership of the property for which taxes have been paid. In the absence of actual, public and
adverse possession, the declaration of the land for tax purposes does not prove ownership. 39

IN LIGHT OF ALL THE FOREGOING, the petition is DENIED. The Decision of the Court of Appeals
is AFFIRMED. No costs.

SO ORDERED.

Puno, J., Chairman, Austria-Martinez, Tinga, and Chico-Nazario, JJ., concur.

Footnotes

1 Penned by Associate Justice Artemio G. Tuquero (retired), with Associate Justices Cancio
C. Garcia and Eugenio S. Labitoria, concurring.

2 Rollo, p. 11.

3 Id. at 82-83.

4 Id.

5 Id. at 70.

6 Id. at 90.

7 Id. at 92-95.

8 Id. at 83.
469

9 Id. at 87.

10 Id. at 101.

11 Id. at 75.

12 Id.

13 Id. at 35.

14 Penned by Judge Alejandro M. Velez.

15 Rollo, pp. 80-81.

16 Id. at 66.

17 CA Rollo, p. 81.

18 Rollo, p. 60.

19 Id. at 9-10.

20 Reyes vs. Court of Appeals, 363 SCRA 51 (2001).

21 Nokom vs. National Labor Relations Commission, 336 SCRA 97 (2000).

22 Rollo, p. 83.

23 Id. at 90.

24 Id. at 83.

25 Id. at 90.

26 Id. at 83.

27 Id. at 90.

28 Id. at 101.

29 Id. at 92.

30 Id. at 101.

31 Id. at 92.

32 Id. at 157.

33 200 SCRA 575 (1991).

34 Id. at 581-582. (Emphasis supplied)

35 Rollo, p. 50.
470

36 Id. at 146-147.

37 Republic vs. Court of Appeals, 345 SCRA 104 (2000).

38 Development Bank of the Philippines vs. Court of Appeals, 331 SCRA 267 (2000).

39 Cequeña vs. Bolante, 330 SCRA 216 (2000).

PEREZ v MENDOZA (G.R. No. L-22006, July 28, 1975)

Republic of the Philippines


SUPREME COURT
Manila

FIRST DIVISION

G.R. No. L-22006 July 28, 1975

BASILIO PEREZ and PETRA MONTALBO, petitioners,


vs.
NICOLAS MENDOZA, MARGARITA MACALALAD and the HONORABLE COURT OF
APPEALS, respondents.

Pedro T. Panganiban for petitioners.

Julio D. Enriquez, Sr. for respondents.

MUNOZ PALMA, J.:

Civil Case 689 of the Court of First Instance of Batangas was an action to quiet title over a piece of
land filed on March 20, 1959, by spouses Basilio Perez and Petra Montalbo with spouses Nicolas
Mendoza and Margarita Macalalad as defendants. According to the complaint, the land in controversy
is located in barrio Dagatan, municipality of Taysan, Batangas, with an area of approximately 4,765
sq. meters, declared for taxation purposes in the name of the "Heirs of Estanislao Montalbo", and is
"bounded on the north by a school site, on the east by Calixto Flores, on the south by a creek, and on
the west by a creek and the land of Gregorio Mendoza." On the basis of evidence adduced by the
parties, the trial court then presided by Hon. Lorenzo Relova rendered judgment on February 19,
1962, dismissing the complaint and declaring the spouses Mendoza "to have a better right to the
property in question."1

Spouses Perez elevated the Relova decision to the Court of Appeals which, however, affirmed in
toto the findings of the court a quo, and declared that "upon the evidence it has been shown by a
great preponderance that the land in question belongs to the defendants."2

The case is now before Us on a petition for certiorari filed by spouses Perez.
471

The findings of fact both of the trial court and the Court of Appeals may be briefly summarized as
follows:

The litigated parcel of land was originally part of a bigger tract owned by Estanislao Montalbo. When
Estanislao died in 1918, his properties passed on to his children Petra, Felisa, and Pedro all
surnamed Montalbo, and because Pedro died single the two women remained as the only heirs. By
mutual agreement Petra and Felisa divided between themselves the lands of their father and the
parcel of which the litigated land was a part was assigned to Felisa. Sometime in 1922 Felisa
exchanged the above-mentioned parcel with a land belonging to her aunt. Andrea Montalbo, a sister
of her father. The reason for the exchange was that Andrea wanted to donate a piece of land to the
municipality for use as a school site and the land of Felisa was what the municipality preferred as it
was adjacent to other properties of the municipality. (Exh. 5 for defendants Mendoza) Upon her
acquisition of Felisa's aforementioned land, Andrea donated to the municipality the northern portion
thereof which constituted almost one-half of the entire parcel, and since then that portion was
declared for taxation purposes by the municipality together with its adjoining properties (Exhs. 6, 6-A,
6-B).1äwphï1.ñët In 1927 the remainder of the lot was given by Andrea Montalbo to her daughter
Margarita Macalalad on the occasion of her marriage to Nicolas Mendoza, and from the time of their
marriage the couple possessed the said property. That donation was confirmed subsequently in a
public instrument dated August 15, 1951 (Exh. 2 for the Mendozas). Nicolas Mendoza sought to
transfer the tax declaration of the property to his name and of his wife and for that purpose he
submitted a deed of exchange of property dated January 14, 1922, allegedly executed by Felisa
Montalbo and Andrea Montalbo in the presence of the municipal secretary Rafael Manahan (Exh. 5).
When Basilio Perez came to know about the supposed deed of exchange, he had it investigated and
upon discovering that the signature of Rafael Manahan appearing on the document was forged, he
filed a criminal complaint before the Fiscal's office which led to an accusation for falsification of private
document against Andrea Montalbo and Nicolas Mendoza. Only Nicolas Mendoza was arraigned and
tried and was convicted by the Court of First Instance of Batangas, but on appeal he was acquitted by
the Court of Appeals for insufficiency of evidence to show that he participated in affixing the signature
of Rafael Manahan or that he was aware of the falsity of the document in question when he presented
it to the tax assessor's office.3 Notwithstanding the forged signature of Rafael Manahan on the
document Exhibit 5, there is sufficient evidence to prove that an exchange of property did in fact occur
in 1922 between Andrea and Felisa Montalbo, and that Felisa's land passed on to Andrea who in turn
gave part of it to the municipality and part to her daughter, Margarita; hence, the decision in favor of
the spouses Mendoza.

On the other hand, petitioners contend that the disputed property was inherited by Petra and Felisa
Montalbo from their father Estanislao who died in 1918 and since that date the two sisters were in
possession of said land. In 1934 a deed of partition of the various properties of Estanislao was
executed between Petra and the heirs of Felisa, and the land in question was divided equally,
between them; among those who signed as witnesses to that agreement was Andrea Montalbo(Exh.
D for petitioners). In 1952 Felisa's husband, Jose Ortega, and children sold their one-half share to
spouses Petra Montalbo and Basilio Perez, now petitioners, but the deed of sale was lost a year after.
Sometime in 1946 petitioners leased the property to the Mendozas and when the lease expired in
1951 they demanded for the return of the land but the Mendozas refused and so petitioners had to file
an ejectment suit before the justice of the peace court of Taysan which was still pending at the time of
the trial of the civil case in 1960. (tsn. witness Basilio Perez, December 15, 1960, pp. 16-34)

For not giving credit to the foregoing evidence, petitioners now assail the adverse decision of
respondent court on four assigned errors.

1. Petitioners contend that respondent court erred in considering the criminal case for falsification res
adjudicata on the matter of ownership of the land in litigation when the "question of ownership was not
actually and directly in issue in the criminal case and the latter was not the proper vehicle for the
determination of the ownership of the land." (p. 9, petitioners brief) Petitioners refer to portions in the
decision of respondent court, viz:

The land in question, together with that portion that was acquired by the municipality
of Taysan, the identity of which is admitted by the parties, belonged to Felisa
Montalbo, as held in the decision of the Court of Appeals, thus — "The said parcel of
472

land previously belonged to Felisa Montalbo (married to Jose Ortega), who inherited
it from her deceased father, the aforecited Estanislao Montalbo;", and the land in
question was donated propter nuptias by Andrea Montalbo to Margarita Macalalad
and Nicolas Mendoza, the defendants, (Margarita Macalalad is the daughter of
Andrea Montalbo) on the occasion of their marriage on February 27, 1927, as found
and held in the decision of the Court of Appeals, thus — "and this land was acquired
by the donor (Andrea Montalbo) by means of a barter with her own parcel of land
planted with bamboos and mango trees"

Upon the basis of the findings of fact and conclusion arrived at in the decision of the
Court of Appeals, it clearly appears that although the document of exchange of the
lands was found to be falsified, nevertheless the Court found upon the facts as
demonstrated by the evidence that the land in question "previously belonged to Felisa
Montalbo (married to Jose Ortega), who inherited it from her deceased father, the
aforesaid Estanislao Montalbo ..."; that said land was donated propter nuptias by
Andrea Montalbo to the defendants on the occasion of their marriage on February 27,
1927; and that "this land was acquired by the donor by means of a barter with her
own parcel of land planted with bamboos and mango trees". From the context of the
decision the natural and logical inference is that factually the exchange of the lands
had been consummated.... (pp. 6-7, CA decision at pp. 20-21, rollo; emphasis
supplied to indicate disputed statements)

Undoubtedly, there is merit to the contention of petitioners that the pronouncements or findings of fact
made by the Court of Appeals in the criminal case concerning the possession and ownership of the
land now in litigation in the civil case, do not constitute the law on the matter and cannot be taken or
adopted as a basis for deciding the question of ownership of said land in this civil case. Since there is
no identity of parties in the two cases — the petitioners here not being parties in the criminal case —
and the object or subject matter in the criminal prosecution is different, the latter being concerned with
the guilt or innocence of accused Nicolas Mendoza for falsification of private document, it follows that
the judgment in the criminal action cannot be used as evidence in the civil case where the issue is
ownership of a piece of land. It is the rule that the plea of res judicata generally cannot be interposed
except where the parties, facts, and questions are the
same,4 hence, the judgment in a criminal case cannot be pleaded as res judicata in a civil action. 5

But whatever error was committed by respondent court in this regard, the same is not sufficient to
nullify the appealed decision.

Analyzing the decision of respondent court. We see that the latter made its own appraisal and
evaluation of the evidence existing in the record relative to the possession and ownership of the land
in question. Thus it said that the conclusions arrived at by the Court of Appeals in the criminal case to
wit(1) that there was an exchange of lands consummated between Andrea and Felisa and (2) that the
exchanged land was later donated by Andrea to her daughter Margarita in 1927, "can hardly be
doubted if we take account of the undisputed fact that the defendants have been in possession of the
land since 1927, and the plaintiffs (meaning spouses Perez) have not attempted to
disturb defendants' possession of the land until 1952 when said plaintiffs filed an action of unlawful
detainer against the defendants." (p. 7 of appealed decision at p. 21, SC rollo; emphasis supplied)
Continuing, respondent court expounded:

Contrary to the allegation in the complaint — "That plaintiffs were in possession of the
land prior and up to January, 1946, when the same was leased to the defendants ...",
and the testimony of Basilio Perez to the same tenor, the evidence has conclusively
shown that the defendants have been in continuous possession of the land since
1927 to the present time, and they have built a house on the land in 1928 where they
have resided and lived to the present, as testified to by the defendant Mendoza, ....

The plaintiffs have contended, however, with the support of the testimony of Basilio
Perez, that the possession of the defendants since 1946 was that of a mere lessee of
the land. On this matter, the trial court said, "the records do not show any
documentary evidence to support such contention. Nor is any document, say receipts
473

of payment of rentals presented to bolster their theory. On the contrary their averment
has been strongly denied by the defendants and the records show that it was only in
1952 that a civil action was instituted by the plaintiffs against the defendants in the
Justice of the Peace Court of Taysan, Batangas, for detainer and damages", and said
allegation of possession of the defendants as lessees of the land "is not supported by
positive and convincing evidence". We find no reason to disagree with the foregoing
findings of fact and conclusion of the trial court because the same is supported by the
preponderance of evidence, and the plaintiffs have not pointed to us any fact of
significance or influence which have been disregarded by the court, other than the
testimony of Basilio Perez who testified about the supposed contract of lease. (pp.
21-22, 23, ibid.; emphasis supplied)

Digging further into the evidence of herein petitioners, respondent court found for itself that the
agreement of partition dated May 27, 1934, Exhibit D, is not incontrovertible proof that in 1934 the
litigated property belonged in common to Petra and the heirs of Felisa Montalbo both of whom may
have been guided by the fact that the property was still declared for taxation purposes in the name of
Estanislao Montalbo, and that the document of partition "did not overcome the evidence on record
that Andrea Montalbo became the owner of the land, and that since 1927 the defendants have been
in continuous possession of the land, openly, adversely and in the concept of owners thereby
acquiring ownership of the land through acquisitive prescription." (p. 10 of CA decision at p. 24,
SC rollo)

Independently therefore of the pronouncements of the Court of Appeals in the criminal case,
respondent court examined the evidence in this civil case and made its own findings of fact on the
basis of which it affirmed the decision of the trial court.

We could have stopped here and resolved this petition under well-entrenched precepts in Philippine
jurisprudence that findings of fact of the Court of Appeals are as a rule conclusive and binding upon
this Court;6 nonetheless, to set our mind at rest that the conclusions of respondent court were not
grounded on speculation, surmises or conjectures,7 We went over the evidence before Us.

Certain salient facts strongly support the claim of respondents Mendoza over the property in dispute:

First, the northern boundary of the land in controversy is undisputably a school site which originally
was part of a bigger tract belonging to Estanislao Montalbo. This is admitted by petitioner Basilio
Perez who to a question propounded by his counsel, Atty. Panganiban, declared:

Mr. Panganiban: (Counsel of petitioners)

Q. According to these tax declarations which you said covers the


land in question, the boundaries on the north, school site; on the
east, land of Calixto Flores; on the south, estero; and on the west,
estero and Gregoria Mendoza, why is it that there is a discrepancy?

A. Because from the whole parcel of land a portion was taken for the
school site, and that which remains now is the land in question, sir.
(tsn December 15, 1960, pp. 22-23)

No explanation however was offered by Perez as to how that portion became a school site. On the
other hand, there is evidence of respondent Mendoza that because Andrea Montalbo wanted to
donate a piece of land to be used as a school site and the municipality preferred the location of the
land inherited by Felisa from her father, the two women exchanged lands after which Andrea gave
one-half of the property to the municipality while the remaining portion which is the land now in
litigation was donated propter nuptias to her daughter Margarita way back in 1927. (tsn October 24,
1961, pp. 14-18) This donation of Andrea was not disproved by any evidence of petitioners. On the
part of respondents Mendoza, their documentary evidence, Exhibits 6, 6-A and 6-B, show that the
municipality of Taysan declared the donated property in its name as early as July, 1925, which
474

supports respondents' claim that the exchange of properties between Andrea and Felisa Montalbo
took place sometime in 1922.

Second, the provincial authorities authorities dealt with the Mendozas for the widening of the
provincial road which traverses the land in question. Nicolas Mendoza testified that the land covered
by the complaint actually consists of two lots which he described in his sketch, Exhibit 1, with letters
"A" and "B" respectively, separated by a provincial road leading to the municipality of Lobo; that lot "A"
which is the bigger parcel is the one donated to his wife, Margarita, by Andrea Montalbo on the
occasion of their marriage in 1927 (Exh. 2); while lot "B" was bought from Donata Mendoza in 1951
as shown by the deed of sale, Exhibit 7; that sometime in 1937-38, the province widened the
provincial road traversing the two lots, and he and his wife were approached by the provincial
authorities more particularly, Engineer Ramirez, for them to give without compensation from lot "A" a
stretch of land of one meter in width to widen said road, and they agreed. At that time Donata
Mendoza still owned lot "B" and she was also asked to give part of her land for the road but she was
paid for the value of the plants destroyed in the process.(tsn October 24, 1961, pp. 32-34) For his
part, petitioner Perez admitted during the cross-examination conducted by the opposite counsel, Atty.
Julio Enriquez, that the provincial authorities did not deal with him at all during the widening of that
particular road. (tsn September 25, 1961, p. 34) This is of marked significance, because if it were true
as claimed by petitioners that they were in possession of the property since the death of Estanislao
Montalbo in 1918 or even after the deed of partition in 1934, they would have been the persons
approached by the authorities for the widening of the road. The fact that the Mendozas were the ones
who gave away part of the land for the widening of the Lobo road shows that they were in possession
of the property and were living there at the time.

Third, respondents Mendoza have been in possession of the property since 1927 in concept of
owners thereof. We have the testimony of respondent Nicolas Mendoza that after the land was
donated to his wife in 1927 they built a house on it and lived there continuously, witness referring
particularly to what he described as lot "A" in his sketch Exhibit 1. (tsn October 24, 1961, pp. 7, .30-
31) Respondent's testimony was found both by the trial and appellate courts credible because (1)
petitioner Basilio Perez himself admitted during cross-examination that even before the last world war
the Mendozas had constructed a house on the land in litigation (tsn September 25, 1971, pp. 37-39;
see Exh. E-3) which admission disproves the allegation in the complaint and Perez' testimony that it
was only in 1946 when the Mendozas occupied the property as lessees; (2) the testimony of Nicolas
Mendoza was corroborated by witness Adriano Gonzales, a retired justice of the peace of Taysan,
Batangas, who declared that he knew the Mendozas since 1937 and he saw them living on the land in
question and they have not changed residence at all since he had known them (tsn December 6,
1961, pp. 5-6); and (3) the respondents Mendoza were the ones who were living on the property and
not the petitioners at the time the provincial government in 1937 widened the Lobo road which
crosses said land.

The court a quo and the respondent appellate court did not err when they upheld the claim of
ownership of the Mendozas principally on the ground that the latter were in actual possession of the
property since 1927 and were sought to be dispossessed by petitioners herein only in 1952 when an
ejectment suit was filed against them.

Possession is an indicium of ownership of the thing possessed and to the possessor goes the
presumption that he holds the thing under a claim of ownership.8 Article 433 of the Civil Code
provides that "(A)ctual possession under claim of ownership raises a disputable presumption of
ownership. The true owner must resort to judicial process for the recovery of the property." In Chan
vs. Court of Appeals, et al., L-27488, June 30, 1970, 33 SCRA 737, this Court upheld the finding of
the Court of Appeals that the litigated property belonged to the private respondents therein based on
their possession of the property, not only because such findings of fact of the appellate court are
conclusive and binding on this Court but because the conclusion is in accordance with Articles 433
and 531 of the Civil Code. 9

As we have here conflicting claims of possession by the parties over the land in controversy and
because the fact of possession cannot be recognized at the same time in two different personalities
except in cases of co-possession, the present possessor is to be preferred pursuant to Article 538 of
the Civil Code which We quote:
475

Possession as a fact cannot be recognized at the same time in two different


personalities except in the cases of co-possession. Should a question arise regarding
the fact of possession, the present possessor shall be preferred; if there are two
possessors, the one longer in possession; if the dates of the possession are the
same, the one who presents a title; and if all these conditions are equal, the thing
shall be placed in judicial deposit pending determination of its possession or
ownership through proper proceedings." 10

The pretension of petitioners that the possession of the Mendozas is that of a mere lessee was not
believed by the trial judge and the appellate court not only because of the absence of any written or
oral evidence on the matter other than the bare testimony of petitioner Basilio Perez, but also due to
the circumstances present in the case which We indicated and enumerated at pages 7 to 9 of this
decision. In fine, it is a fact that the Mendozas are presently in possession of the property and the
presumption of ownership in their favor has not been successfully rebutted by evidence that they are
mere lessees of the land in their possession as claimed by petitioners.

2. In their second assigned error, petitioners contend that respondent court should not have given
weight to the evidence of respondent Mendoza because the latter's Exhibit 5 was proven to be a
falsified document.

To recall, Exhibit 5 is the alleged deed of exchange or barter of lands between Andrea and Felisa
Montalbo dated January 14, 1922. On this point, petitioners overlook the fact that Exhibit 5 was made
the basis of a criminal accusation of falsification of private document solely on the allegation that the
signature of Rafael Manahan, the person before whom the parties to the document allegedly
appeared, was not his. There was no finding in that criminal case as per decision rendered therein
that the barter or exchange of lands between Andrea and Felisa Montalbo did not in effect take place.
On the contrary, what appears in said decision offered by petitioners as their Exhibit J are the
following findings of the Court of Appeals, viz: that the land donated by Andrea Montalbo to her
daughter Margarita Macalalad "was acquired by the donor by means of a barter with her own parcel
of land planted with bamboos and mango trees"; that while it is true that because of this presentation
of the falsified document appellant (now respondent Nicolas Mendoza) was able to secure the
declaration of the property donated in his name, no criminal liability should be imposed upon him in
the absence of any evidence that he presented said exhibit with the knowledge that it was forged
"especially if we take into consideration the fact that he and his wife were and are still in possession
of the land donated since 1927"; that in fact, the color and appearance of the document in question
show that it is not a new document but an old one thus confirming Mendoza's theory that it was
executed in or about the year 1922 as appearing in the document or five years before his marriage.
(pp. 1, 5, 6 of Exh. J, folder of exhibits) Thus, if the document Exhibit 5 was held to be forged, it was
simply because the municipal secretary, Rafael Manahan, did not sign it and not for any other reason.
What is material and relevant to the civil case is that both the trial court and respondent appellate
court found for a fact that there was an exchange of lands between Andrea and Felisa Montalbo on
the basis of evidence other than the disputed Exhibit 5. As to what the evidence is, has been
discussed above.

Petitioners cite Gonzales vs. Mauricio, 53 Phil. 728 where this Court stated inter alia that the
introduction of a forged instrument by a witness renders the testimony of the latter practically
worthless. That statement however is not applicable to the situation before Us because
in Gonzalez the particular document or receipt referred to was found to be entirely false as to its
contents, handwriting, and signature, whereas here all that was found to be false is the signature of a
witnessing official.

3. The last argument of petitioners is the object of the third assigned error. It is contended that the
appellate court erred in not giving effect to the deed of partition, Exhibit D, notwithstanding the fact
that the name of Andrea Montalbo appears in the document as one of the witnesses thereto.

Exhibit D appears to be a document dated May 27, 1934, wherein certain properties allegedly
belonging to Estanislao Montalbo were divided between Petra Montalbo and Jose Ortega, husband of
deceased Felisa Montalbo. Petitioner Basilio Perez declared that one of the parcels of land mentioned
in the document is the land now in litigation which is particularly marked as Exhibit D-1. He also
476

testified that Exhibit D was signed by him and his wife, Petra Montalbo, by Jose Ortega, husband of
deceased Felisa Montalbo, and thumbmarked by the latter's children all in his presence. (tsn
December 15,1960, pp. 19-24) Surprisingly, however, Basilio Perez did not at all mention during the
course of his testimony that the old woman, Andrea Montalbo, signed the deed of partition as a
witness. We have gone over the transcript of Basilio Perez' declaration on direct and cross-
examination (tsn December 15, 1960, pp. 15-34; September 25, 1961, pp. 3-40) and at no instance
did he ever state that Andrea Montalbo was present during the preparation of the document, that she
read or knew the contents thereof which by the way consists of six handwritten pages, and that she
signed her name on the document. It was incumbent upon petitioners to identify the signature of
Andrea Montalbo on the document if her signature was truly there. As a matter of fact, examining the
document Exhibit D We entertain doubts whether the name referred to by petitioners is "Andrea
Montalbo", for, as written, it also can read "Maria Montalbo". At any rate, whatever is the import of
said deed of partition, the same binds only the parties thereto but does not affect third persons such
as Andrea Montalbo or the herein Mendozas in the absence of proof that they participated in one way
or another in the preparation and execution of thedocument. As it is, Andrea Montalbo was a stranger
to that deed of partition and any recital therein concerning the property under litigation cannot be used
as evidence to prejudice her and her successors-in-interest or place her in estoppel as to her claims
over the property. Res inter alios acta alteri nocere non debet. A transaction between two parties
ought not to operate to the prejudice of a third person or stranger. 11

4. In the fourth assignment of error, petitioners claim that the appellate court should have rendered a
decision in their favor. That both the trial court and respondent appellate court have correctly
evaluated the evidence, has been clearly demonstrated by Us.

IN VIEW OF ALL THE ABOVE CONSIDERATIONS, We find no reversible error in the decision under
review and We AFFIRM the same with costs against petitioners.

So Ordered.

Castro (Chairman), Makasiar, Esguerra and Martin, JJ., concur.

Teehankee, J., is on leave.

Footnotes

1 pp. 8-15, Record on Appeal, civil case 689, at p. 27, SC rollo

2 CA-G.R. No. 30871-R, September 7, 1963, Eugenio Angeles, J., ponente, with
Juan L. Lanting and Magno S. Gatmaitan, JJ. concurring, pp. 15-26, ibid.

3 CA-G.R. No. 13872-R, August 14, 1958, Exh. J for Perez

4 Santos vs. Gabriel, et al., L-22996, May 31, 1972, 45 SCRA 288; Benin vs. Tuason,
L-26127, June 28, 1974 & other cases, 57 SCRA 531, 534 .

5 Ocampo et al. vs. Jenkins, et al., 14 Phil. 681, 684, 688, citing among others,
Chamberlain vs. Pierson, 87 Fed. Rep. 420

6 Abellana vs. Dosdos, et al. L-19498, Feb. 26, 1965, 13 SCRA 244; Roque vs.
Buan, L-22459, Oct. 31, 1967, 21 SCRA 642; Cui vs. Court of Appeals L-24072, July
29, 1968, 24 SCRA 189; Philippine American Life Insurance Company vs. Honorato
R. Santamaria, L-26719, February 27, 1970, 31 SCRA 798; People vs. Boduso, L-
30450-51, Sept. 30, 1974, 60 SCRA 60
477

7 In Pioneer Ins. & Surety Co. vs. Yap, L-36232, Dec. 19, 1974, this Court thru its
Second Division, per Fernandez, J., held that findings by appellate court based on
speculation, surmises or conjectures are not binding on the Court.

8 Rule 131, Sec. 5, par. (j), Rules of Court: Disputable presumptions — The following
presumptions are satisfactory if uncontradicted, but may be contradicted and
overcome by other evidence:

xxx xxx xxx

(j) That a person found in possession of a thing taken in the doing of a recent
wrongful act is the taker and the doer of the whole act; otherwise, that things which a
person possesses, or exercises acts of ownership over, are owned by him;
(emphasis supplied).

9 Art. 531 Civil Code. Possession is acquired by the material occupation of a thing or
the exercise of a right, or by the fact that it is subject to the action of our will, or by the
proper acts and legal formalities established for acquiring such right.

10 See also Molina vs. De Bacud, et al., 19 SCRA 956 .

11 Rule 130, Sec. 25, Rules of Court: Admission by third party. — The rights of a
party cannot be prejudiced by an act, declaration, or omission of another, and
proceedings against one cannot affect another, except as hereinafter provided.

Moran, Rules of Court, Vol. 5, 1970 Ed. 249

Tansioco et al. vs. Ramoso et at. 59 Phil. 672

DIZON v CA (G.R. No. 101929, January 6, 1993)

Republic of the Philippines


SUPREME COURT
Manila

FIRST DIVISION

G.R. No. 101929 January 6, 1993

BENJAMIN DIZON, ZACARIAS DIZON, AFRICA DIZON, PERFECTO DIZON, CARMEN DIZON
(Heirs of Paula Galang), JULIA GALANG, CONSOLACION TABORA, ABELARDO TABORA,
CECILIA TABORA, AVELINA TABORA, TRINIDAD TABORA, REMEDIOS TABORA, VIRGINIA
TABORA, DELFIN TABORA, PENINA TABORA, FRANCISCO TABORA, CIPRIANA GALANG,
RUFINO DELOS SANTOS, PEPITO DELOS SANTOS (Heirs of Donata Vergara), ARNEO
VERGARA, BENIGNO VERGARA, JOSE VERGARA, SCION VERGARA, DEMETRIA VERGARA
(all heirs of Dionisio Galang), petitioners,
vs.
COURT OF APPEALS, AUREO REYES, AURELIO SAMIA, ALFONSO SAMIA, POTENCIANO
GALANG, LEONCIA GARCIA, BIENVENIDO TAPNIO, LYDIA BALINGIT VDA. DE GARCIA,
BENEDICTO GARCIA, ROMULADO GARCIA, AMY GARCIA, ALEXANDER GARCIA, LUDIVINA
GARCIA, MONTANO GUEVARRA, CORAZON LAMPA, RUDY LAMPA, EDUARDO LAMPA,
478

ILLUMINADA GUEVARRA, CARMELITA MASANQUE VDA. DE GARCIA, MA. CONCEPCION


AQUINO VDA. DE GUEVARRA, HONZAI GUEVARRA, RODA REBECCA GUEVARRA, RUTH
GUEVARRA, minors represented by their mother Ma. Concepcion Vda. de Guevarra,
PRIMITIVA GUEVARRA, JOSIAS N. GARCIA, LUCITA M. GARCIA, VICTOR M. GARCIA,
LUTERO M. GARCIA, SAMSON M. GARCIA, FELIXBERTO M. GARCIA, JR., HERMENIGILDA
GARCIA, CONSTANCIO GARCIA, REYNALDO GARCIA, AGAPITA GARCIA, ERNESTO GARCIA,
NORICO GARCIA, PACIFICO GARCIA, NORMANDO GARCIA, ARTURO GARCIA, ESTELLA
GARCIA, DIOSDADO GARCIA (representing LEONCIA GARCIA), GREGORIA MENDOZA,
ELEUTERIA BAUTISTA, PEDRO ATIENZA, BENITA SAMANIEGO, NENE SAMANIEGO
(representing FLAVIANA GALANG), LETICIA REYES, MANUEL REYES (representing
MARCIANA GALANG), CARMEN ROQUE VDA. DE DIMABUYU, PORFIRIO R. DIMABUYU,
CARMEN R. DIMABUYU, CARIDAD R. DIMABUYU, PEDRO R. DIMABUYU, MARCOS
DIMABUYU (representing GERTRUDES GALANG), respondents.

Heminio Z. Canlas for petitioners.

Lagunzad, Juan, Rubin & Cabaron Law Office for respondents.

PADILLA, J.:

In this petition for review on certiorari, petitioners seek to nullify the decision ** of the Court of
Appeals, dated 29 April 1991, in CA-G.R. CV. No. 14312, the dispositive portion of which reads as
follows:

WHEREFORE, the appealed judgment is hereby REVERSED; and the Deed of


Extra-Judicial Settlement of the estate of the deceased Dionisio Galang (Exh. "D"), in
so far as it relates to Lots 3548 and 3562 the Bacolor Cadastre, and Transfer
Certificates of Title Nos. 182670-R and 182671-R issued by virtue thereof are hereby
declared null and void.

Conformably, the Register of Deeds concerned is hereby ordered to cancel the said
titles; and subject Lots 3548 and 3562 are hereby adjudicated to the heirs of the
deceased co-owners to be partitioned among them as follows:

a. one-sixth to the Heirs of Marciana Galang;

b. one-sixth to the Heirs of Dionisio Galang;

c. one-sixth to the Heirs of Flaviana Galang;

d. one-sixth to the Heirs of Gertrudes Galang;

e. one-sixth to the Heirs of Potenciana Galang;

f. one-sixth to the Heirs of Leoncia Galang.

Costs against defendants-appellees.

SO ORDERED. 1

It appears that on 21 August 1984, Aureo Reyes, et al. (hereafter "respondents") filed an amended
complaint before the Regional Trial Court of San Fernando, Pampanga, docketed as Civil Case No.
6752, for the annulment of a deed of extra-judicial settlement and partition of the estate of Dionisio
Galang, claiming to have been deprived thereby of their shares, as co-owners, in Lot Nos. 3548 and
479

3562 Bacolor cadastre, and that OCT Nos. 9010 and 9102, issued in the name of Dionisio Galang,
covering said lots, are fraudulent and should therefore be annulled and cancelled.

The facts of the case, as culled from the Court of Appeals decision, are as follows:

The spouses Hilario Galang and Martina Laxamana owned two (2) lots located in San Agustin,
Potrero, Municipality of Bacolor, Province of Pampanga. They had six (6) children, namely, Dionisio,
Marciana, Potenciana, Flaviana, Leonora and Gertrudes.

The spouses (Hilario and Martina) mortgaged the aforesaid lots to Camilo Angeles. It is alleged by the
respondents that Dionisio Galang redeemed these lots in his own name, despite the fact that part of
the funds used for the redemption came from his sisters.2 A cadastral survey involving the two (2) lots
was conducted, and on 19 May 1919, the Court of First Instance ordered the issuance in Cadastral
Case No. 14, of OCT Nos. 9010 (for lot 3548) and 9102 (for lot 3562) in the name solely of Dionisio
Galang ( hereafter Galang).

Respondents, who are heirs of Galang's sisters, claim that Galang and his five (5) sisters had
partitioned the subject lots on 27 June 1920, as embodied in an unnotarized affidavit executed by
Galang (Exh. "C"). As a consequence thereof, Galang's sisters constructed their houses on Lot 3548.
The structures passed on from generation to generation, with each of Galang's sisters and their
descendants enjoying the benefits therefrom. No one questioned or disturbed them until the
petitioners (heirs of Galang), informed them that the lots in question were titled in Galang's name and
had been partitioned, on the basis of a Deed of Extrajudicial Partition (Exh. "D"), into three (3) equal
parts corresponding to his (Galang's) three (3) children; that petitioners had succeeded in subdividing
the lots and in obtaining titles thereto in their name (TCT Nos. 182670-R and 182671-R) despite their
(respondents') earlier demands for an extrajudicial settlement of their dispute.

Petitioners, on the other hand, contend that the cadastral case which culminated in the issuance of
the original certificates of title over the subject lots in the sole name of Galang, was a proceeding in
rem, thus binding on the whole world; that when original certificates of title (OCT Nos. 9010 and 9102)
were issued on 9 January 1922 to Galang, respondents did not raise any objection until March 1983
when they filed the complaint in Civil Case No. 6752, or after a lapse of sixty-one (61) years.

The trial court3 upheld Galang's titles over the lots which, as aforestated, had been issued as early as
1922 in his name. The trial court further held that respondents' action had long prescribed, having
been filed only on 24 March 1983, or after a lapse of sixty-one (61) long years from the issuance of
said titles. The court also noted respondents' failure to establish their relationship to Galang's five (5)
sisters, premising their claim solely on an unsubstantiated assertion that they are descendants of the
deceased Galang sisters.4 The presence or construction of the houses on Lot No. 3548 was also not
considered as evidence in respondents' favor, since no proof was submitted establishing respondents'
right to occupy the place. The documentary evidence (Exh. "C" and "C-1") allegedly showing co-
ownership among Dionisio and his co-heirs, was likewise ignored by the trial court as this did not
specifically refer to the disputed Lots 3548 and 3562.

On appeal by the respondents, respondent Court of Appeals reversed the trial court by upholding
respondents' rights. It focused on two (2) issues.
Thus —

Are the properties in question owned in common by the predecessors-in-interest of


appellants and appellees? And has appellants' present action for partition
prescribed?5

The appellate court declared that co-ownership existed between respondents' predecessors-in-
interest and those of petitioners, on the basis of Galang's affidavit which, although unnotarized, was
nonetheless an ancient document, pursuant to Sec. 22, Rule 132 of the Rules of Court, since it was
executed on 27 June 1920. As such, proof of its due execution and authentication could be dispensed
with, according to the appellate court.
480

Hence, this recourse in turn by the petitioners.

We find the petition impressed with merit.

It is a fact that Dionisio Galang's ownership over the disputed lots (3548 and 3562) had been judicially
confirmed on 19 May 1919 in Cadastral Case No. 14, G.L.R.O. No. 51, which is a proceeding in
rem and hence binding "on the whole world." OCT No. 1056 (9010) and OCT No. 1057 (9102) were,
as a consequence, issued on 9 January 1922. None of Galang's co-heirs objected to or protested
their issuance. These titles became indefeasible and incontrovertible. Then it was only after sixty-one
(61) years or on 24 March 1983 that the descendants of Galang's co-heirs asserted co-ownership
claims over the subject lots.

It is true that Galang executed an affidavit, unnotarized at that, on 27 June 1920 which states in part
as follows (per English translation [Exh.
"C-1"]):6

That on this date, I have received from all my sisters and nephews who are my co-
heirs, namely Potenciana Galang, Flaviana Galang, Gertrudes Galang, who are my
sisters, and Silverio Garcia and Hilarion Samia, in their own names and for their
brothers and sisters who are also co-heirs, the sum of ONE HUNDRED AND SIX
PESOS (P106.00), Philippine Currency, as complete payment for the discharge of
the land we co-inherited, which is the one we partitioned this date also, which was
mortgaged to the Angeles family.7

However, as can be gleaned from the foregoing, there is no reference to Lot Nos. 3548 and 3562.
Said affidavit is not therefore a sufficient basis or support for what is alleged by respondents as a
partition among Dionisio and his now deceased sisters. It does not, as correctly stated by the trial
court, amount to anything insofar as the two (2) lots involved in this case are concerned:

Even their presentation of the document purportedly executed by Dionisio Galang on


June 27, 1910 (Exh. "C" and "C-1") where the latter acknowledges that he and his co-
heirs named therein as co-owners of a certain property which they had mortgaged to
a certain family surnamed Angeles does not amount to anything for nothing in this
document shows that it pertains to the two lots involved herein. It merely referred to a
certain "land" which Dionisio Galang and his co-heirs "co-inherited" and partitioned
without any indication as to which property is being referred
to.8

We likewise agree with the trial court that in the absence of definite proof establishing respondents'
link/relationship to their alleged predecessors-in-interest, i.e., the Galang sisters, they do not have any
cause of action, and the suit for partition must necessarily fall. The trial court aptly observed:

. . . the plaintiffs thru their witnesses Bienvenido Tapnio, Marcos Dimabuyu, Pedro
Atienza, and Carmelita Galang, tried to prove that all the plaintiffs herein are heirs
and direct descendants, respectively, of Marciana Galang, Potenciana Galang,
Flaviana Galang, Leoncia Galang and Gertrudes Galang who, in their lifetime,
together with their late brother Dionisio Galang, are the co-owners of these two lots,
namely, Lots Nos. 3548 and 3562. Lamentably, all that was proved in the process by
the plaintiffs thru these witnesses despite several proddings and suggestions made
by the court toward this end was that each of these plaintiffs are just related to one
another in varying degrees of relationship. They failed to establish their connection or
relationship with any of these five sisters save for their unfounded averment that they
are indeed descendants and heirs of these deceased individuals. 9

WHEREFORE, the petition is GRANTED. The appealed decision of the Court of Appeals is hereby
SET ASIDE and the decision of the trial court dated 3 October 1986 in Civil Case No. 6752 is hereby
REINSTATED. No costs.
481

SO ORDERED.

Cruz, Griño-Aquino and Bellosillo, JJ., concur.

# Footnotes

** Penned by Justice Fidel P. Purisima with the concurrence of Justices Eduardo R.


Bengzon and Salome A. Montoya.

1 Rollo, pp. 40-41.

2 Rollo, p. 40.

3 RTC, San Fernando, Pampanga, Br. XLIV, Judge Nathanael M. Gorospe,


presiding.

4 Rollo, p. 36.

5 Rollo, p. 38.

6 Records, p. 241.

7 Rollo, p. 243.

8 Records, p. 294.

9 Records, p. 293.
482

(LIMITATIOIN OF REAL RIGHT OF OWNERSHIP)

US v CAUSBY (328 US 256, 1946) (to be emailed)

LUNOD v MENESES (G.R. No. 4223, August 19, 1908)

Republic of the Philippines


SUPREME COURT
Manila

EN BANC

G.R. No. 4223 August 19, 1908

NICOLAS LUNOD, ET AL., plaintiffs-appellees,


vs.
HIGINO MENESES, defendant-appellant.

T. Icasiano, for appellant.


R. Salinas, for appellee.

TORRES, J.:

On the 14th of March, 1904, Nicolas Lunod, Juan de la Vega, Evaristo Rodriguez, Fernando Marcelo,
Esteban Villena, Benito Litao, Ventura Hernandez, and Casimiro Pantanilla, residents of the town of
Bulacan, province of the same name, filed a written complaint against Higino Meneses, alleging that
they each owned and possessed farm lands, situated in the places known as Maytunas and Balot,
near a small lake named Calalaran; that the defendant is the owner of a fish-pond and a strip of land
situated in Paraanan, adjoining the said lake on one side, and the River Taliptip on the other; that
from time immemorial, and consequently for more than twenty years before 1901, there existed and
still exists in favor of the rice fields of the plaintiffs a statutory easement permitting the flow of water
over the said land in Paraanan, which easement the said plaintiffs enjoyed until the year 1901 and
consisted in that the water collected upon their lands and in the Calalaran Lake flow through
Paraanan into the Taliptip River. From that year however, the defendant, without any right or reason,
converted the land in Paraanan into a fishpond and by means of a dam and a bamboo net, prevented
the free passage of the water through said place into the Taliptip River, that in consequence the lands
of the plaintiff became flooded and damaged by the stagnant waters, there being no outlet except
through the land in Paraanan; that their plantation were destroyed, causing the loss and damages to
the extent of about P1,000, which loss and damage will continue if the obstructions to the flow of the
water are allowed to remain, preventing its passage through said land and injuring the rice plantations
of the plaintiffs. They therefore asked that judgment be entered against the defendant, declaring that
the said tract of land in Paraanan is subject to a statutory easement permitting the flow of water from
the property of the plaintiffs, and that, without prejudice to the issuing of a preliminary injunction, the
defendant be ordered to remove and destroy the obstructions that impede the passage of the waters
through Paraanan, and that in future, and forever, he abstain from closing in any manner the
aforesaid tract of land; that, upon judgment being entered, the said injunction be declared to be final
and that the defendant be sentenced to pay to the plaintiffs an indemnity of P1,000, and the costs in
the proceedings; that they be granted any other and further equitable or proper remedy in accordance
with the facts alleged and proven.

In view of the demurrer interposed by the plaintiffs to the answer of the defendant, the latter, on the
29th of August, 1904, filed an amended answer, denying each and everyone of the allegations of the
complaint, and alleged that no statutory easement existed nor could exist in favor of the lands
described in the complaint, permitting the waters to flow over the fish pond that he, together with his
483

brothers, owned in the sitio of Bambang, the area and boundaries of which were stated by him, and
which he and his brothers had inherited from their deceased mother.

Apolinara de Leon; that the same had been surveyed by a land surveyor in September, 1881, he also
denied that he had occupied or converted any land in the barrio of Bambang into a fishpond;
therefore, and to sentence the plaintiffs to pay the costs and corresponding damages.

Upon the evidence adduced by both parties to the suit, the court, on the 13th of March, 1907, entered
judgment declaring that the plaintiffs were entitled to a decision in their favor, and sentenced the
defendant to remove the dam placed on the east of the Paraanan passage on the side of the Taliptip
River opposite the old dam in the barrio of Bambang, as well as to remove and destroy the obstacles
to the free passage of the waters through the strip of land in Paraanan; to abstain in future, and
forever, from obstructing or closing in any manner the course of the waters through the said strip of
land. The request that the defendant be sentenced to pay an indemnity was denied, and no ruling was
made as to costs.

The defendant excepted to the above judgment and furthermore asked for a new trial which was
denied and also excepted to, and, upon approval of the bill of exceptions, the question was submitted
to this court.

Notwithstanding the defendant's denial in his amended answer, it appears to have been clearly
proven in this case that the lands owned by the plaintiffs in the aforesaid barrio, as well as the small
adjoining lake, named Calalaran, are located in places relatively higher than the sitio called Paraanan
where the land and fish pond of the defendant are situated, and which border on the Taliptip River;
that during the rainy season the rain water which falls on he land of the plaintiffs, and which flows
toward the small Calalaran Lake at flood time, has no outlet to the Taliptip River other than through
the low land of Paraanan: that the border line between Calalaran and Paraanan there has existed
from time immemorial a dam, constructed by the community for the purpose of preventing the salt
waters from the Taliptip River, at high tide, from flooding the land in Calalaran, passing through the
lowlands of Paraanan; but when rainfall was abundant, one of the residents was designated in his
turn by the lieutenant or justice of the barrio to open the sluice gate in order to let out the water that
flooded the rice fields, through the land of Paraanan to the above-mentioned river, that since 1901,
the defendant constructed another dam along the boundary of this fishpond in Paraanan, thereby
impeding the outlet of the waters that flood the fields of Calalaran, to the serious detriment of the
growing crops.

According to article 530 of the Civil Code, an easement is charge imposed upon one estate for the
benefit of another estate belonging to a different owner, and the realty in favor of which the easement
is established is called the dominant estate, and the one charged with it the servient estate.

The lands of Paraanan being the lower are subject to the easement of receiving and giving passage
to the waters proceeding from the higher lands and the lake of Calalaran; this easement was not
constituted by agreement between the interested parties; it is of a statutory nature, and the law had
imposed it for the common public utility in view of the difference in the altitude of the lands in the
barrio Bambang.

Article 552 of the Civil code provides:

Lower estates must receive the waters which naturally and without the intervention of man
descend from the higher estates, as well as the stone or earth which they carry with them.

Neither may the owner of the lower estates construct works preventing this easement, nor the
one of the higher estate works increasing the burden.

Article 563 of the said code reads also:


484

The establishment, extent, form, and conditions of the easements of waters to which this
section refers shall be governed by the special law relating thereto in everything not provided
for in this code.

The special law cited in the Law of Waters of August 3, 1866, article 111 of which, treating of natural
easements relating to waters, provides:

Lands situated at a lower level are subject to receive the waters that flow naturally, without
the work of man, from the higher lands together with the stone or earth which they carry with
them.

Hence, the owner of the lower lands can not erect works that will impede or prevent such an
easement or charge, constituted and imposed by the law upon his estate for the benefit of the higher
lands belonging to different owners; neither can the latter do anything to increase or extend the
easement.

According to the provisions of law above referred to, the defendant, Meneses, had no right to
construct the works, nor the dam which blocks the passage, through his lands and the outlet to the
Taliptip River, of the waters which flood the higher lands of the plaintiffs; and having done so, to the
detriment of the easement charged on his estate, he has violated the law which protects and
guarantees the respective rights and regulates the duties of the owners of the fields in Calalaran and
Paraanan.

It is true that article 388 of said code authorizes every owner to enclose his estate by means of walls,
ditches fences or any other device, but his right is limited by the easement imposed upon his estate.

The defendant Meneses might have constructed the works necessary to make and maintain a fish
pond within his own land, but he was always under the strict and necessary obligation to respect the
statutory easement of waters charged upon his property, and had no right to close the passage and
outlet of the waters flowing from the lands of the plaintiffs and the lake of Calalaran into the Taliptip
River. He could not lawfully injure the owners of the dominant estates by obstructing the outlet to the
Taliptip River of the waters flooding the upper lands belonging to the plaintiffs.

It is perhaps useful and advantageous to the plaintiffs and other owners of high lands in Calalaran, in
addition to the old dike between the lake of said place and the low lands in Paraanan, to have another
made by the defendant at the border of Paraanan adjoining the said river, for the purpose of
preventing the salt waters of the Taliptip River flooding, at high tide, not only the lowlands in Paraanan
but also the higher ones of Calalaran and its lake, since the plaintiffs can not prevent the defendant
from protecting his lands against the influx of salt water; but the defendant could never be permitted to
obstruct the flow of the waters through his lands to the Taliptip River during the heavy rains, when the
high lands in Calalaran and the lake in said place are flooded, thereby impairing the right of the
owners of the dominant estates.

For the above reasons, and accepting the findings of the court below in the judgment appealed from
in so far as they agree with the terms of this decision, we must and do hereby declare that the
defendant, Higino Meneses, as the owner of the servient estate, is obliged to give passage to and
allow the flow of the waters descending from the Calalaran Lake and from the land of the plaintiffs
through his lands in Paraanan for their discharge into the Taliptip River; and he is hereby ordered to
remove any obstacle that may obstruct the free passage of the waters whenever there may be either
a small or large volume of running water through his lands in the sitio of Paraanan for their discharge
into the Taliptip River; and in future to abstain from impeding, in any manner, the flow of the waters
coming from the higher lands. The judgment appealed from is affirmed, in so far as it agrees with
decision, and reversed in other respects, with the costs of this instance against the appellants. So
ordered.

Carson, Willard and Tracey, JJ., concur.


485

S-ar putea să vă placă și